You are on page 1of 454

SELF ASSESSMENT AND REVIEW OF

BIOCHEMISTRY
SELF ASSESSMENT AND REVIEW OF

BIOCHEMISTRY

Second Edition

Rebecca James Perumcheril MD (Biochemistry)


Assistant Professor
Government Medical College
Calicut, Kerala, India

Edited by
Jomy P Thomas

Forewords
Ravindran Chirukandathu
Tajan Jose
NC Cherian
MG Jose Raj

The Health Sciences Publisher


New Delhi | London | Philadelphia | Panama
Jaypee Brothers Medical Publishers (P) Ltd

Disclaimer: National Board Pattern Questions


Headquarters The questions which are discussed here are a model of AIPGME/DNB conducted
Jaypee Brothers Medical Publishers (P) Ltd. in Nov-Dec 2012 to 2015 and not obtained from NBE/DNB question bank, neither
4838/24, Ansari Road, Daryaganj by force or compulsion by students, with due respect to the NDA (non disclosure
New Delhi 110 002, India agreement). If any similarities occur it may be due to mere coincidence and the
Phone: +91-11-43574357 author or the publisher is not to be blamed about. All questions discussed here
Fax: +91-11-43574314 can be found in one or the other question banks, guides and not any persons
Email: jaypee@jaypeebrothers.com private property. The author is not related in anyway to the NBE/PGI/AIIMS
exam conducting authority or agencies.
Overseas Off ces
J.P. Medical Ltd. Jaypee-Highlights Medical Publishers Inc.
83, Victoria Street, London City of Knowledge, Bld. 237, Clayton
SW1H 0HW (UK) Panama City, Panama
Phone: +44-20 3170 8910 Phone: +1 507-301-0496
Fax: +44-(0)20 3008 6180 Fax: +1 507-301-0499
Email: info@jpmedpub.com Email: cservice@jphmedical.com
JP Medical Inc. Jaypee Brothers Medical Publishers (P) Ltd.
325 Chestnut Street, Suite 412 Philadelphia, 17/1-B, Babar Road, Block-B, Shaymali
PA 19106, USA Mohammadpur, Dhaka-1207, Bangladesh
Phone: +1-267-519-9789 Mobile: +08801912003485
Email: support@jpmedus.com Email: jaypeedhaka@gmail.com
Jaypee Brothers Medical Publishers (P) Ltd.
Bhotahity, Kathmandu, Nepal
Phone: +977-9741283608
Email: kathmandu@jaypeebrothers.com
Website: www.jaypeebrothers.com
Website: www.jaypeedigital.com
© Digital Version 2017, Jaypee Brothers Medical Publishers
The views and opinions expressed in this book are solely those of the original contributor(s)/author(s) and do not necessarily represent
those of editor(s) of the book.
All rights reserved. No part of this publication may be reproduced, stored or transmitted in any form or by any means, electronic,
mechanical, photocopying, recording or otherwise, without the prior permission in writing of the publishers.
All brand names and product names used in this book are trade names, service marks, trademarks or registered trademarks of their
respective owners. The publisher is not associated with any product or vendor mentioned in this book.
Medical knowledge and practice change constantly. This book is designed to provide accurate, authoritative information about the
subject matter in question. However, readers are advised to check the most current information available on procedures included and
check information from the manufacturer of each product to be administered, to verify the recommended dose, formula, method and
duration of administration, adverse effects and contraindications. It is the responsibility of the practitioner to take all appropriate safety
precautions. Neither the publisher nor the author(s)/editor(s) assume any liability for any injury and/or damage to persons or property
arising from or related to use of material in this book.
This book is sold on the understanding that the publisher is not engaged in providing professional medical services. If such advice or
services are required, the services of a competent medical professional should be sought.
Every effort has been made where necessary to contact holders of copyright to obtain permission to reproduce copyright material. If
any have been inadvertently overlooked, the publisher will be pleased to make the necessary arrangements at the first opportunity.
Inquiries for bulk sales may be solicited at: jaypee@jaypeebrothers.com

Self Assessment and Review of Biochemistry


First Edition: 2015
Second Edition: Digital Version 2017
ISBN: 978-93-85891-74-8
Dedication
This book is dedicated to my son, my husband, my parents and in laws, my brothers and to all the students of my classes and
facebook group, and the students who could not attend my classes and inspired me to write a book.
Foreword
Thrissur Medical College Alumni Association (TMCAA) is a CME Programme started in the year 2000 for Junior
Doctors guiding and coaching for various PG entrance exams. Our principle is professionalism and structured
orientation. Beyond guiding thousands of students to their dream PG and being one of the top institutes in the
field we have also generated and promoted many deserving faculties and authors since the inception. Dr Rebecca
had started her career as a PG guidance faculty from this institute four years back. Her teaching methodology and
updated notes and techniques are well appreciated by students and her notes are of much demand as a “must for
exam points”. We welcome and appreciate her effort in compiling her notes into students friendly entrance guide.
Wishing all success to her and students reading second edition, Self Assessment and Review of Biochemistry.

Ravindran Chirukandathu
MBBS MS (Surgery) DNB FRCS
Chief Coordinator and Secretary
CME for Junior Doctors
Thrissur Medical College Alumni Association
Thrissur, Kerala, India

Tajan Jose MBBS (Orthopedic)


President and Coordinator
CME for Junior Doctors, Thrissur Medical College
Alumni Association (TMCAA)
Thrissur, Kerala, India
Foreword
It gives me immense pleasure to welcome the second edition of the book “Self Assessment and Review of
Biochemistry” written by Dr Rebecca James. She is one of the best faculties in her subject in Calicut Medical College
Alumni CME entrance guidance programme. The book comprises a vast collection of questions from the recent
exams and thoroughly described. I am sure that now this book will become a necessity among the PG entrance
Aspirant’s books. Wishing all the success for the book.

NC Cherian MBBS MD (Paediatrics)


Additional Professor of Paediatrics and
Coordinator, Calicut Medical College
Alumni, Medical updates programme
Government Medical College
Calicut, Kerala, India
Foreword
The second edition of the book “Self Assessment and Review of Biochemistry” has been written as a course material
for the various national level (NBE pattern), central institutes, state level, and private medical entrance exams. Dr
Rebecca James has been a very keen and devoted teacher in her career. Her new and innovative visual and easy
learning techniques have been much useful for her students. The references given are up-to-date and authentic
and the presentation and charts are much students friendly. I do recommend this book for PG entrance aspirants. I
heartily congratulate her and wish her all the success for this excellent piece of work.

MG Jose Raj MD (Biochemistry)


Professor and HOD, Biochemistry
Government Medical College
Kozhikode, Kerala, India
Preface to Second Edition
“To improve is to change, to be perfect is to change often …..” Winston Churchill
After the overwhelming reception of the first edition of the book across India I am more than excited to release of
the second edition. The responses from toppers of various PGMEE speak for the success of the first edition. In this
edition we have incorporated the reviews and suggestions about the first edition from readers, yet the same format
have been kept in order to remain it as a perfect book for PG aspirants.

Rebecca James MD (Biochemistry)


drrebeccajomy@gmail.com
Preface to First Edition
Writing a book for Postgraduate Medical Entrance in Biochemistry was a dream for me when I was doing my PG
career. But when I started Postgraduate medical entrance coaching, gradually I recognised how hard it was for
students to learn the subject. There is no subject expert written book in biochemistry for the major exams like PGI,
AIIMS, AIPGME, JIPMER, or even state entrance exams. None of the students had learned biochemistry properly
or were not taught properly in their MBBS course and for entrance. Students were forced for learning from books
not by a subject expert which are just to pick some points from standard books without explanation or internet
reference. Biochemistry comprises up to 8% in NBE exams and up to 30% in PGI exams and lots of researches and
updates are coming up in this field. My students often would come asking with doubts quoting some of these
guides in which many answers I found wrong. As four years passed in postgraduate medical entrance coaching I
made my own notes based on standard textbooks with clinical scenarios and emphasised on teaching to think and
correlate rather than mugging up biochemistry. Gradually students from other institutes started asking for my
notes and requested me to write a book so that they could also read it. This book is meant for it. I approached Jaypee
Brothers Medical Publishers (P) Ltd., and they accepted my proposal, the remaining is in front of you.
This is a PGMEE book and do not compare it to any standard books, but you will get up to 96% if you learn this
properly. Biochemistry is a very vast subject with future of research, genetics, which will be overtaking the future of
the medical field in time. This book is just a small flower in a big garden. I know that learning in my class would be
better than reading this book but due to practical difficulties it is not possible, I have done much justice in including
my own made pictures and mnemonics to make it as understandable as possible. I wish that you thoroughly go
through this book and gain maximum knowledge from it as it has all the points for scoring top rank in major exams.
Wishing you best of luck for your dream PG in the coming exams. Genuine doubts are always welcome. Students
can feel free to contact me at gmail/whatsapp. drrebeccajomy@gmail.com

Rebecca James
Acknowledgements
Writing this book has never been easy to me. “No tears in the writer, no tears in the reader. No surprise in the
writer, no surprise in the reader.” ― Robert Frost.
First and foremost I would like to thank God for enabling me in completing this work. In the process of putting
this book together I realized how true the gift of writing is in me. You have given me the power to believe in my
passion and pursue my dreams. This would not have been possible without the faith I have in you, the Almighty.
To my husband Dr Jomy, you have been the greatest inspiration and pillar of strength throughout the work.
Without your relentless support, this book would have never been materialized.
To my son John, you are the best thing that ever happened to me in my life! This book would have never been
complete without you letting me do. Thank you so much.
To my beloved Father Mr. James P George, I am speechless! I can barely find the words to express all the wisdom,
love and support you have given me. I am forever grateful for being born to and raised by such an amazing person.
If I am blessed to live long enough, I hope I will be a good a parent as you are and always been to me. I love you too,
Amma. I also extend my gratitude to my brothers, Mr George James and Mr Philip James.
To my in laws Mrs and Mr Thomas P John, thank you for all the support as my parents and for encouraging me.
I acknowledge Shri Jitendar P Vij, (Group Chairman), Jaypee Brothers Medical Publishers (P) Ltd., Mrs Chetna
Vohra, (Associate Director), Jaypee Brothers Medical Publishers (P) Ltd., Mr Venugopal, (Associate Director—Sales
South), Mr Jagadeesh S, (Branch Manager), Jaypee Brothers, Kochi and Ms Payal Bharti, (Project Manager) in
making this dream a reality.
Deep gratitude to all the Jaypee team all over India for their tremendous efforts in marketing.
Special thanks to my colleague, Dr Shibu TS, Assistant Professor, Department of Biochemistry, Medical College,
Thrissur for his guidance and mental support.
I thank the colleagues of my department (Dr Geetha PA, Dr Jayaraj K, Dr MohamedAshraf, Dr Asha E, Dr Shaji
Sreedhar) for their expert opinions and valuable suggestions.
I thank all Senior residents (Dr Anjana, Dr Rejitha Reghunath) and Junior Residents (Dr Anju V, Dr Shajna, Dr
Ashuthosh, Dr Sreevalsan, Dr Nasid, Dr Fesina, Dr Rashid) in Department of Biochemistry, Government Medical
College, Calicut for your helping hands in arranging the previous questions of various entrance exams and also in
the tedious job of proofreading.
I thankfully remember Ms Pradheera M who helped me a lot in typing works.
Also Thanking
• Dr MG Jose Raj, Professor and HOD, Government Medical College, Kozhikode
• Dr Tajan Jose, Coordinator and President TMCAA, Thrissur
• Dr Ravindran Chirukandath, Coordinator and Secretary, TMCAA, Thrissur
• Dr NC Cherian, Calicut Medical College, Weekly Medical Updates, Coordinator
• Dr Arun Kumar, ADR Plexus-Digital strides Coordinator
• Dr Arun Kumar, AIIMS Academic Director
• Dr Manorajan Pozitive, PG Coordinator
Staff and Residents of Department of Biochemistry, Government Medical College, Kozhikode
Dr Akhil K, Dr Goutham Nallaiyan for their mental support
Mr Praveen TMCAA staff
All my students who supported and contributed questions of various PGMEE. Their immense support for
making the second edition of the book is impressive. I thank all those who had been on my side on this journey.
HOW TO LEARN BIOCHEMISTRY
Dear students,

I consider you as one of my students by using this book and I believe it is my responsibility to guide you through
in your postgraduate medical exams and I wish you all the best to come out with flying colors. I know that many
of you feel that Biochemistry is one of the most boring subjects you have learned in MBBS. This is primarily due to
the methodology you have been taught and you tend to think of it as a subject of cycles and test tubes. But the fact
is that if it was taught with all clinical correlations and the practical applications the subject becomes interesting and
beautiful. I hope you will have a renewed perception about biochemistry after reading this book.
This is a concept-oriented book where you learn biochemistry and its applications This is made in a review book
style to enable you to answer questions with ease. Another aspect of it is the clinical approach which makes it easier
to learn, understand and recollect. Previous questions from all the major exams (PGI, AIIMS, AIPGMEE, STATE
ENTRANCE, FMGE, PRIVATE EXAMS and NATIONAL BOARD) are included for discussion.
In learning biochemistry it is recommended to finish the topic you started before moving to another subject so
that you can correlate all portions together. You can finish reading the whole book in less than ten days for the first
study, five days for revision after three months and two to three days for revision just before your exam. I assure
you 96% score. After a careful analysis I find that 40% of questions are direct and another 40% are twisted and a
20% of them are from recent updates. 30 of 200 PGI exam questions are from biochemistry and molecular genetics.
AIIMS exam always introduces 2–3 questions about recent updates and NBE which conducts DNB/PGET comprises
of 9% questions from Biochemistry which might include pictorial questions and missing parts of certain reactions
and clinical questions. Please make sure that you revise the subject as it is a little volatile. This book is different from
the usual guide for postgraduate medical entrance exams since every part of it are referred from standard books.
You may feel free to ask your questions, you can mail me or post in my facebook discussion forum. As always,
your feedback is important to us. If you believe you have identified an error in the book, please send an email to
drrebeccajomy@gmail.com. If you have general comments or suggestions please drop me a line directly to my
email. We are continually striving to meet the needs of all individuals preparing for the entrance exams.
Wishing you all the best

Rebecca James
Mob 9447440193
drrebeccajomy@gmail.com
For author online support, follow
‘Dr Rebecca biochemistry discussion group’ in facebook
Contents
Section 1: Amino Acids and Proteins
1. Chemistry and Metabolism of Amino Acids 3
2. Proteins 56
3. Enzymes 100

Section 2: Carbohydrates
4. Chemistry of Carbohydrates 119
5. Metabolism of Carbohydrates 137

Section 3: Lipids
6. Chemistry of Lipids and Biomembranes 179
7. Metabolism of Lipids 199

Section 4: Molecular Genetics


8. Chemistry and Metabolism of Nucleotides 241
9. Structure and Organization of DNA 252
10. DNA Replication 262
11. Transcription 271
12. Translation 286
13. Regulation of Gene Expression 296
14. Molecular Biology Techniques and Recent advances in Molecular Biology 315

Section 5: Miscellaneous
15. Vitamins and Minerals 351
16. Heme Metabolism and Hemoglobins 382
17. TCA Cycle and Biological Oxidation 399
18. Free Radicals, Xenobiotics and Metabolism of Alcohol 411

Section 6: Image-Based Questions


Image-Based Questions
Students’ Testimonials
Dr Fen Saj
PGI Nov 2015 3rd rank, JIPMER Nov 2015 6th Rank
“I consider myself fortunate to have attended Dr Rebecca’s high yield lectures at TMCAA. She is a very dedicated
teacher and makes the core concepts in Biochemistry simple and interesting. Her book, Self assessment and review of
Biochemistry is a must read for all entrances especially for PGI as they ask almost 20 + questions from biochemistry
and related topics. The book will compensate for those who cannot attend her classes as it is well written, updated,
error free and revision friendly.”
Dr Ram Manohar Talupula
AIIMS Nov 2015 2nd Rank
“I heard about Dr Rebecca’s Self assessment and review of Biochemistry during my PG preparation time, when I
was confused about what book to read for Biochemistry. I feel this book as a perfect example of how an entrance
preparation book should be. This helped me a lot especially during the last few days before the exam, as it is concise
and at the same time high yielding”
Dr Vimal Chacko
JIPMER Nov 2015 Rank 6, AIIMS Nov 2015 Rank 53
“Madam, I am your student in TMCAA. I want to thank you so much. All the questions that came from Biochemistry
was covered by you. Your classes made the difference.”
Dr Nabeel Faizal
AIIMS Rank 193
“Thank you for your valuable teaching and guidance.”
Dr Mohammed Ameen
AIIMS Nov 2015 Rank 13
“Thanks a lot for your classes and wonderful book. I could answer many questions from Biochemistry. Madam’s
class and book matter a lot to me.”
Dr Aishwarya
DNB Dec 2015 Rank 25
“Thank you mam…Your classes and book have always given us the extra edge. Whatever we know about
Biochemistry is only because of you.”
Dr Akhil K Aravind
AIIMS Rank 92
“Thank you so much mam. All biochemistry questions were from your class and book only. I attended all
biochemistry questions with confidence. That is the success of a good teacher like you mam. God bless u”
Dr Uma Shankar
“Mam got 250 rank in AIIMS. Your page and information provided by you helped me a lot”
Dr Arjun
“I got 27th rank in JIPMER Nov 2015. Your classes were excellent and inspiring and everything you taught only was
asked. Your last minute revision in the facebook group was really useful. Thank you so much mam”
Students’ Testimonials  | XV

PG Aspirant from Thirunelveli


“It is written in my fate that I should understand the true core of biochemistry after MBBS through your book. In
Islam it is told that to sustain your life even after death is to give charity and to teach the ignorant. Mam you are one
who deserves that place. Hats off to you mam to give us knowledge of Biochemistry through your book”
Dr Nikhil
“ Hello mam this is Nikhil from Banglore. I am preparing for PG entrance, I bought your book. I usually hate
biochemistry, but after reading the first chapter itself I fell in love with biochemistry. It is an awesome way of
explanation you have given mam”
Dr Sindhu Jonnala
“ Thank you mam for such a wonderful book. You have just simplified everything. Mentioned all must know topics.
The high-yielding points represented very well. I was always having fear for Biochem, it was a nightmare for me.
Your book boost up my confidence for preparation. Thank you mam for such big help”
Dr Payal Kumari
“Thank you mam for your book and forum in facebook. I attempted all biochem questions correctly in AIPGMEE
and DNB”
Dr Swati Dash
“Biochem was the dreadful subject in my career. I am very poor in basics. Then I got to know about your book. I
can not express my gratitude and words enough to express the amount of confidence your notes in the book have
instilled in me now. Thank you for the wonderful book”
Dr Nilotpal Bhattacherjee
“Mam exam was good. All from your book and updates in the facebook group”
Dr Deepika Mishra
“Hello mam. Someone suggest your book last year and may I just say that its brilliant. I have struggled with
biochemistry, the book made so many concepts clear to me, even everything asked in PGI and AIIMs this year was
mentioned in your book. So thanks a lot.”
Dr Surbhi Jain
“Thank you mam for this time in AIIMS biochemistry questions I was more confident in answering because I could
recall the questions as direct lines from your book”
Dr Sharath M Adiga
“Hi mam I would like to thank you for your guidance in TMCAA and your awesome book. I got rank 158 in AIIMS”
Dr Saravanan Ganesan
“Thank you for your wonderful lecture and book I got rank 78 in AIIMS in UR category”
Dr Vigneshwar Sivaraman
All questions in DNB were direct from your book. Your lecture is the best of biochem I ever attended”
Dr Navin Srinivasan
“Mam your book is too good. I like biochemistry and Harper a lot. Your book is no less than Harper “
Dr Naveen Murali
“I read your book though not fully, but I attended all biochem question in Nov AIIMS 2015. I always felt I would
not read biochem, but after buying your book things changed. Thanks a lot”
XVI |  
Self Assessment and Review of Biochemistry

Dr Aqhib Nadeem
“Mam I am your student in TMCAA. I am extremely to your ultimate teaching and your book. I could read whole
biochemistry in just 2 and half days, and answer almost all the biochemistry questions correctly”
Dr Ragesh Ravindran
“All questions from your book mam. Me and Calicut Medical College is proud of you”
Dr Neha Thakur
“Your book is an epic mam. I marked almost all questions right. I am happy because for the first time I could solve
biochemistry”
Dr Vimmi Gautam
“You are the best mam. Your words were questions. Mam biochem I felt so happy to answer because of you”
Dr Prachi Gupta
“Biochemistry was a dreaded subject for me. And in my previous attempt I did not even touch this subject as it
looked scary. A friend of mine referred me your book. I bought it and read it. I must say “What an effort mam!!!
I am so happy and satisfied with your book and it gave me confidence that ‘Yes, even I can attempt biochemistry’
questions.”
SECTION
1 Amino Acids and Proteins
C H A P T E R S

1. Chemistry and Metabolism of


Amino Acids
2. Proteins
3. Enzymes
1 Chemistry and Metabolism
of Amino Acids

Topics Included
 Chemistry of Amino Acids:  Metabolism of Amino Acids:
–  


 –   
  

– 
 

 –   
  


CHEMISTRY OF AMINO ACIDS Imino Acid


 $   
 

General Structure of Alpha Amino Acid  % 
   
     
&



 ' 
   
 &
  

Fig. 1.2: Structure of imino acid, proline


Fig. 1.1: General structure of alpha amino acid CLASSIFICATION OF AMINO ACIDS
Alpha Amino Acid (VERY IMPORTANT TOPIC)
 
   

     
 
     !
"
  
   
     
 #"
   
       Based on nutritional requirement
 
 
  $
Non Alpha Amino Acid
     


 
  BASED ON VARIABLE SIDE CHAINQ

      
 
     
 


 Aliphatic Amino Acid
 !   Simple Amino Acid
 ! 
 ( )Q
 "# 
 (  
4 Self Assessment and Review of Biochemistry

 Branched Chain Amino AcidQ  Polar Amino AcidsQ (Hydrophilic)


( L  ( *

( I  -  1
 +
-
( V   ,2)  +)   ,
 

    
 %&' 

- -   1/2 
2
.
( 

 Sulfur Containing Amino Acid
-     3

  
( *Q
 '
2%

( Q
-      3 
 
 Amino Acid with Hydroxyl Group 
2)  
( '
Q - '  ) 
( %
Q
- ' 
   * 
 Amino Acid with Amide Group
 Nonpolar Amino AcidQ (Hydrophobic)
( 
 ( '     
( )   ( ' 
     
 Acidic Amino Acid ( 
     / (Think
( 
+
 , as it is basic amino acid it is already included among
( )  +)   , polar amino acid)
 Basic Amino Acid (  
     . 2
( 
+-  ,Q $  4 
( . ( $ &


Aromatic Amino Acid Concept


 & 
  

 
 #  # !  

 &  Q      !   

 %
Q  ! 
  
$ ! 
  
   

With heterocyclic aromatic ring (Ring structure contain   
 - Acidic and B 
  !
more than one type of atom.) amino acids
     
 


 %
   Q
  
  
 ) +   

$
 /+- ,Q

 
  
 (  #
)# !
  *#
  BASED ON METABOLIC FATEQQQ
 
  
 (   
   +  $

 
 Ketogenic:  
5

Imino Acid
* 
6 & 3 
 &

 Glucogenic:  
)  
3 
BASED ON SIDE CHAIN CHARACTERISTIC
 Both Glucogenic and Ketogenic:%  

(POLARITY)Q       3 
 


Amino acid
# .
!  & Q

 .
!    
!  P #  
Isoleucine
T#
 
T#

Fig. 1.3:  


 

 # Contd...
Chemistry and Metabolism of Amino Acids 5

Contd... Conservative (Homologous) Substitution


 


Amino acid 8  
   
  

Lysine* (Predominantly 


Ketogenic)
9           3  

!   # 
  


! 
 53

 
 !

Conservative Mutation
Concept Hydrophilic, Acid  Glu
/   
 (  .
! 0     

Hydrophilic, Basic + ! &#
 (  
 
!    .
! 
 

 
 .
!    
! %&** &# 0 Polar, Uncharged Ser * Gln 
 #  0 
 0 *#
 0 *#
 " Hydrophobic  Phe & Ile ' Pro

BASED ON NUTRITIONAL REQUIREMENTQQQ Nonconservative (Nonhomologous) Substitution


8  
    
   
 Essential: %   3    :



Q/  

  ABBREVIATIONS OF AMINO ACIDS
 Semiessential:)
3 

7 

2     
 
 
 


 Nonessential:  3  Three letter One letter
Q2
7 
 Amino acid Abbreviation Abbreviation
#   # C
Essential Semiessential Nonessential
+   + +
Methionine !    
 

Isoleucine Ile I
Threonine
 
   
T#

Serine Ser S
Valine
'  ' '
Isoleucine
Leucine  3 5 7 ;


Ph #   Abbreviated based on the commonly occurring amino acids


Lysine Three letter One letter
Histidine Amino acid Abbreviation Abbreviation
Glycine Gly G
Alanine Ala A
  

   
%** '& +&1  
 ( 2#"$  
 0*
 0 *#
 0' 0 
 0 Leucine Leu L
& 0  #  0 &# 
Proline Pro P
Threonine Thr T
Special Groups Present in Amino Acids
Abbreviated based on phonetically sounding letters
Amino acid Special group
Three letter One letter
!   GuanidiniumQ
Amino acid Abbreviation Abbreviation
 #   Benzene
Arginine Arg R (aRginine)
*#
  Phenol
Asparagine Asn N (asparagiNe)
+   ImidazoleQ Aspartatic Acid Asp D (asparDic acid)
Proline #
  Glutamic Acid Glu E (glutEmic acid)
 
  *
   3! Glutamine Gln Q (Qtamine)
*#
 Indole Phenylalanine Phe F (Fenylalanine)
#   *


 4+" Contd...
6 Self Assessment and Review of Biochemistry

Contd...
RECENT UPDATES
Abbreviated based on phonetically sounding letters Extraterrestrial Amino Acids
Three letter One letter I # 5=6> foll
( ! )losion of appro)ima # 5=0=== metric
Amino acid Abbreviation Abbreviation ton meteo   3s in c#  s30 (est si )  estrial
amino acids 3   0  ic acid0 ! mic acid, isoleucine,
Tyrosine Tyr Y (tYrosine) leuc 0  #  0 serine, threo  0 #osi 0  0  #
Tryptophan Trp W (tWiptophan) !# 0 ?    (
      
meteor$ *ese
 di !s demonstrated potential insi! s to )istence o )  estrial
Abbreviated based on letter close to initial letter
 $
Lysine Lys K (letter close to L)

21st and 22nd Amino AcidsQ


Selenocysteine
PROPERTIES OF AMINO ACID
56 

 ! 
Q

 

 
#    4 Q
    
   

Serine  

#  $ 
 <== 


   
4    
#   
  
#  
 3>=   

   
) # 5 
7 

   0  

  
)Q    Q$
Seen in the active site of following Enzymes and ProteinsQ Remember
 *

)    


 # !   
     

   
 
) $Q
 

#
  
   
 
 # 
 

 4
#  0 #
# 
 Selenoprotein P

 #  4
 

, UGAQ #  
  Recoding
484 8   9    
 Amino Acid Exists in Three Charged State,
Pyrrolysine Positive, Negative or Neutral
 55  

 ! 

 # 
 ! UAG 
 

0  # 1&4   
? 3 

9$ @ $
 /  
> /

   
DERIVED AMINO ACIDS Isoelectric pH of Amino Acids
Derived Amino Acid seen in ProteinQ  At pH = Isoelectric pH
4-Hydroxyproline  /
   
!  At pH < Isoelectric pH
5-Hydroxylysine  '    
 #
)# 
$  At pH > Isoelectric pH
Methyllysine /
   #
 At pH = Isoelectric pH (pI)
Gammacarboxy  /
   
 ! 
0 3  %  

7  


 5 
glutamate 

    2+  5
2NO NET CHARGE.
 '  .  


)# 
$    ZWITTER ION (AMPHOLYTE)
Cystine  /
   
  (  
Zwitter Ions or Ampholytes

$Q 2014 DNB

 ( # @ 

7 !



 
 *(
#   
 :



!   
 
 !   Zwitter ions or
#  0 $!$   0 
!

ampholytesQ. A(    ! (
 (   
 $
Desmosine  Found in ElastinQ AIIMS Nov 2014

Properties of Amino acid at Isoelectric pH(pI)


Derived Amino Acid not seen in ProteinQ  9

 #    Q
Ornithine    
 #
  
; #  )   #Q
Arginosuccinate
     !  #
Citrulline
Homocysteine < 
  
 Q
At pH less than isoelectric pH (pI)
Homoserine 

#   
# 
    
  
 5
    4   
  

Chemistry and Metabolism of Amino Acids 7

At pH greater than isoelectric pH (pI) Examples of Toxic L-Amino Acids


     
  
  5 L- Amino acids Clinical implication

 &%+

!    #  9
 # 
! 
& &#    ; humans
The charge of carboxyl group and amino group at physiological ? 9 H)# <


 9

)
pH (pH = 7.4)  ? H<"  9
 #   
 
)# !
  ! # !
 
!
  
 # !$ ? 9  # 



 H 
 #
!
9  9"
50 J%<
 #    
   
Amino Acids Exhibit Isomerism  #0   ! 


) #$
  5 
+
,   
 
% 


  
  3

     ?% # 
   
 3 


  # " 
!    3

 2
 ? .

 

 3 
K <
 #

 &  4
 &4"

Amino Acid Absorb UV LightQ


3 
250–290 nm +   280
nm,4 
tryptophan, phenylalanine, tyrosine.
   
4 tryptophan.Q
Remember
 
  
 
 ;' ! $
 
  

  # 


  ! $
Fig. 1.4: &   < 


    
 

   

L-Isomers
 ' 


  
?


Naturally occurring D Amino Acid


 A
?
  A
?'

 
 ?1   ? )      3  
1
5 

 Bacillus subtilis 
 ?12 ?1
2
?1 2  ?1
     
 
 
 
 Vibrio cholerae 

 ?1  
?1        

    


Remember

 (  9
D9
# D9
H #   

GlycineQQQ
Fig. 1.5: ; 
 
 
 
4

<%
     )
!
$
of aromatic amino acids

BETA–ALANINE (Very important topic for national


Potentially Toxic L-Amino Acids  


 *
 .B 
     A

* 
QDNB/AIPGMEE
5
 :     8

-  
- 
 &

  . 
    
8 Self Assessment and Review of Biochemistry

 -  Q DNB/AIPGMEE Remember


( &  <      
 

 (  
( *C       
(
  
$
( *


&

( -  ?   Ninhydrin Test
  
 
  
 )

    
( *
D/E-   F  E>
1111111G3
( 
D*
F-
 @
 E*8>E&
* +H  I
Skeletal muscle. &
,
Uses of Carnosine Remember
 Amino acid which do not give purple color are:
 5 %& 
 
    +#
)# 
  1
( 

"
 * * 
      !  
( 

"
 9 * 
  
 - :
 /  
 
   Colour Reactions Test answered by
M 

 * (Conc Aromatic Amino Acid2014 DNB
Remember Q
HNO3 is a reagent )  #  0 *#
 0 *#
 "
 +


    L +  
 * 
      


  
K  *#
  
"
#    
  *#
  
 !
"
(
 
-
DECARBOXYLATION OF AMINO ACID   
  *
/
#  
 %   
    
    4    "
 Hopkin’s Cole TestQ



  
 #
)#   "
 PLPQ is the coenzyme for this reaction.
43 GK  G   G   !
"
Examples of Amino Acid Decarboxylation
Amino acid Biologic amines Mnemonic–G is common to all
+   +   Sulfur test #  
*#
  *#  #  9 
 * +

#  
*#
 *#   #K * +   7
" *#
 
&#  Cadaverine 
"
  Q  
 #  "
Methionine 

 ( 4        
 
Serine Ethanolamine    
   3! (   
3$
#   Beta mercaptoethanol amine

Points to Ponder Buffering Action of Amino Acids


 
  !

)# 


 

 !  - :

 3 
  3
 $ 

  

!# 
    


!  $ 
   
 &   
 7#
 
 <
)# 
Henderson Hasselbalch Equation
 
  !
  


 

 !
+ N . L 
! ORDOR
.
$
 
 


   !
H)    
 T OR N OR + N .
     "
     !

)# 


     :
   pH = pKa' 
     !
  


  .
!   3  6 
 
   /  
 :5 :
.
COLOR REACTIONS OF AMINO ACIDS pKa Range of Amino Acid
Biuret Test Dissociating group pKa range

 )


  
)# !
 >$5UJ$6

 *
       
 5 
 9
 HH+
    J$=UJ$V
3  
  Contd...
Chemistry and Metabolism of Amino Acids 9

Contd... - *
   C


Dissociating group pKa range
   
7
 !

   W$XUY$J - 9    C

 
4+ !

#   V$XUZ$=  
     
H+ !

*#
  Z$XU6=$X > Exopeptidases catalyze the hydrolysis of peptide
 
!
 V$=UZ$= bonds, one at a time, from the ends of peptides
   !

!   > 12 ( *
   2 
   
 
J 2
   



At physiologic pH Imidazole group of Histidine has the maximum 
 
buffering capacityQ
(    2 
    
  2
   
 
High-yielding Facts-Amino Acids 
 
 4 
%# 
( ?     
      

 
 #

 

" 
%
 
 4
 


 
  ' 


          C 
 
 
 
  !   
1 
  23

 
   
   
     
#   

 1 carboxypeptidases.
   %
 
 
   5 zymogens; 
 
   
   %   5    C       
 
Amino Acids and Amino Acid Derivatives as Neurotransmitters     
5
 
 # %:
  
# 
            ;  
cord
   %:
 ) 
# 
  $  &   5    
  
Amino Acid Derivative as neurotransmitter by activated pepsin.
 Dopamine  $    2 
  2 



 Epinephrine  
 2  5   enteropeptidaseQ2 3 
 9
   
    K
  
 Serotonin 5 
  
 2
  
  
 #  "$    2
1
     
   2

1        
DIGESTION OF PROTEINS
GENERAL AMINO ACID METABOLISM
 5


       3
 
 
C Biosynthesis of Urea
3 

 


+  
 

 
    
 ,
1. Transamination
Enzymes Catalyze the Digestion of Proteins ?;
%

 3     
  5  %

    
 
  
C+
 ,   
 
 
   
@ Endopeptidases 
C  3  
 ;  
   %  
 
  



;
C 2  

 
 
)   
 

  -   Glutamate can undergo oxidative
( Pepsin  
J   C
 deamination  ;   

  
  J   3    

 
1 +
  
1  
 ,
( Trypsin, chymotrypsin, and elastase


  
 
- %
    C 
   


 Fig. 1.6: *   

10Self Assessment and Review of Biochemistry

Important Points—Transamination Oxidative Deamination—Important Points


 *   
  "    7#     <#
!  <+"    7#
 #
) 
  &"   
'  W    & 

 
    <#
!  <+"

 7#  9<+
 9<+   
 7#
   7#
 H       9 
!  
 (   
; #
 
 *  

  <
 <  ! 
!"
 & <+  
 #    # *0 *0   9<+Q
mechanism
 & <+     # <$
 9
 
   
L-Amino acid oxidase
 /# 

  (#
  


-
 #  
  
  
# 
  
#
 
 *3       3 #

$
 /9   
 7#
  

 +2H2 is formedQDNB
Examples of transamination
 Alanine Amino Transferase (ALT) or Serum
Glutamate Pyruvate Transaminase (SGPT)
  EB16  
o&
5 E)   
 Aspartate Amino Transferase (AST) or Serum
Glutamate Oxaloacetate Transaminase (SGOT)
 
  E B16  
 o 8    E
)   
Fig. 1.8: & 

)

Amino Acid that do not undergo TransaminationQDNB/AIPGMEE Some examples of Nonoxidative DeaminationQ NBE
 Proline 
 +#
)#
   Amino acid Dehydrases 
   3
 *
  

 +'
2%
,
 &#   Histidase 

 

 

   3
Delta Ornithine Aminotransferase  L

 2* /
  
 _ 
!


0 ` 
!

H   
  !
   
$ Transdeamination
Clinical Correlation *5
  B  
      
 H    ` 
    #  
#   
# 
    

  )   

     

 ?
  
 %
  
 *   
   

 # !  
 #
)   !     $ *   
L H)  <  
N *   

2. Oxidative Deamination 3. Transport of Ammonia (Very Important Topic


 %
5   
  
   
PGMEE Exams)
    A
    
9
 8       
  ;   5   
 5


   %
 
5


 
 %
 
    
     
  

 %
 
 
  
Transport of Ammonia from most of the tissues
including the brain.
 As Glutamine Q 3      C
Fig. 1.7: H)    
)  ' 
Chemistry and Metabolism of Amino Acids 11

Glutamine Synthetase
  
   


)   
)  
 % ;

  
 %&
7 


 

Glutaminase
 $5
2    
5  

)  
  

 5


Fig. 1.10B: * 




 
  
! 

4. Disposal of Ammonia
 %   
  5
  
  
5
$;
 5
2


 
 
 Q  J

 
  


Sources of UreaQ

Fig. 1.9:    #     !  

Transport of Ammonia from skeletal muscle.


 A
'   AlanineQ Fig. 1.11: 4

 
!   



 $  2 
 
 




 
5 
  
UREA CYCLE (VERY IMPORTANT TOPIC)
%
     
   3   
    '
 /  6
      
 2 6
 /   Krebs Henseleit
Cycle
 8
    
  > 
 
-
  
  N /   Ornithine
Cycle.

Site of Urea Cycle


 Organ:%   5

 Organelle: &
 mitochondrial and partly cyto-
Fig. 1.10A: * 


 
  
!  plasmic
12Self Assessment and Review of Biochemistry

Fig. 1.12:  



 #

Reactions of Urea Cycle  **&'1$$55&



%;
3
    
   *&'1$  5   
  1
%

       )   2  
 5 
Q
 % 
7 
>%&
Carbamoyl Phosphate Synthetase–I (CPS-I)
 *
  &    
 
  1 Ornithine Transcarbamoylase (OTC)
 *8> 2  %&  %


 
 *
 &  
 %   
 8

 *

 *&'1$  
   +  
, C   %   

 3   ' 7       
Chemistry and Metabolism of Amino Acids 13

Transporters of Urea Cycle Clinical Symptoms common to all


 !   
"   
A

8
 Urea Cycle Disorders
   
"   
A
 *
 In the neonatal period
Arginino Succinate Synthetase Symptoms and signs

 
 -
 . 
  
 
  



  
-
 
 
5
 
  
 %C .  %     
   
   
 %
 
7 
ATP    3   
   

 %3
   
 C

Arginino Succinate Lyase  H   
* 5 
  
 A 
  4
%C .   %  
ArginaseQ  .

Hydrolytic  5  
2
 
    *5 




3


   * 7 

   
Arginase is a HydrolaseQ
In infants and older children
Remember
 4
   7#   #
    (     bK$
 
  
 +   2  2
N-Acetyl Glutamate Synthase  2

 2  5,


 8 7# (  # 
 

9  #   
9"$
  # 
   )   7#
; #$ 
 
 
 
     

 Because CPS-I is active only in the presence of NAGQ. two enzymes.

    # 7 
 
  # 
9  #

 #  3

!   
 $
!  
# 
&%   L  # 
 9% #    L 
 4+

BIOCHEMICAL DEFECT IN UREA


Energetics of Urea Cycle CYCLE DISORDERS
 
 
7 
4 high energy phosphates
 
 
7 
<%&'Q 
   
 
   
Urea Bicycle Disorder Enzyme defective

 %*
 A 
  Hyperammonemia Type-I 
# 
 

 / urea bicycle 4#    4%"

Remember by Urea cycle Hyperammonemia type-II H    * %


#
 H20 *0 9+4+      
$ H*"
 H   0   0 ! 
  0 !  


Citrullinemia Type I (Classic !
   #  
 * 

     
 #   !   
Citrullinemia)
H   
Arginosuccinic aciduria !
   #
Hyperargininemia ! 
CLINICAL CORRELATIONS: UREA
CYCLE DISORDERS Urea cycle disorders due to transporter defect
Citrullinemia Type II     
     
Key Points of Urea Cycle Disorders !  "  
Characterized by Hyperammonemia- H     
   
 / 
 hyperornithinemia-
 9    homocitrullinuria (HHH)
 H 

   syndrome
14Self Assessment and Review of Biochemistry

HIGH YIELDING FACTS—UREA CYCLE Contd...


DISORDERS  !  Q  
   

9 4#  
 4%
    9% #       4%

 ! "" #$! % &   !   


 9% #     #0 
 
 *

Q   7#      $

 ?

 3 X-linked partially dominant
Hyperammonemia-Hyperornithinemia-
inheritance + 
 
  * ?



  H5, Homocitrullinemia (HHH) Syndrome
 Autosomal recessively inherited disorder
 
 

38


 
 5      
   Biochemical Defect is   ORNT 1 gene
     335
    
  
 Ornithine

 Permease
 8
  
  
   
  %
      
 
  

5
 



 %     

Orotic aciduria in Hyperammonemia Type II  hyperornithinemia
 8
 %

   5  *
-  ?;

 
 
 &      
 

  hyperammonemia
 *
 &  
   
  Homocitrulline 
   
 
 
&
'
 
 
    3
 8
 2  
    &
 
    3 8

 
Citrullinemia Type II
 %  
+ $$, ;
Arginosuccinic Aciduria
 
 
 

 

 %


   +
  
  
,  
;  *
 + 
 1     



,  

  
 

    
Hyperargininemia (Argininemia), A Distinct Urea Cycle Disorder +'.*>N@<, 
O7
 +#!      # 
 (  
+#
$  #   !   
   8
I 
 
 
 
(
 
!  #  

  

   
 *  5 !  #    !      
  
7 
 
 5
  
 
 H   #

 6"    )      argininosuccinic acid
# 
# 0   
 5" 
       
mitochondria  '*
    
 * ! 
 60   7#        ( 
!   #
Biochemical Investigation in a Case with
 *       

 
 
 @    

Hyperammonemia



  #  7#   
 A progressive spastic diplegia (  
 !
 
( 9
 

 
  20–40 μg/dl
)  0 

 
 
 0   


%
Methods of estimation of blood ammonia:
  
   
#
   $
   
-  
 

* 

  )      #!  -
 8 7#   
-    <#
!   

 #  0 H   0 &# 0 !   OH&R
 ; ! 
   8 
  

  

;
  3
! 
 $
Methods of estimation of Urea:
'       %     
- < # 

) *
7  

 * )   7#  # ( 
  # 
  8 7#   
- ; ! ;
 * 
 
 
 
+#
 *#  *   4 
 #   
   


 
Contd...  
 <
$
Chemistry and Metabolism of Amino Acids 15

Fig. 1.13: 
   ! 

#


Biochemical Basis of Treatment  &E*o&*E


of Urea Cycle Disorder )  o&)  
 #  
( 9 
INDIVIDUAL AMINO ACIDS
( &
58
 Phenylalanine and Tyrosine
( 
  5 
)   
Phenylalanine
'  
 
 ?
 
   
  
% 
  

 9   


      C  
 /
   
  
 &
   
 
$ 
9   

 

  3     
%  
  3 
 


 % 
 &  
-C  
 
 
3   5
9 
C 
5@    
Benzoic Acid + CoA o Benzoyl CoA + Glycine o
Fig. 1.14:  #  
Benzoyl Glycine [Hippuric Acid]
 % 
$   
&   J  Tyrosine
3     
     2  
   
3 
  
  
 8   'C
   
   
5 >        
  
  Partly glucogenic and partly ketogenic.
16Self Assessment and Review of Biochemistry

CATABOLISM OF TYROSINE

Fig. 1.15: *#


 

Synthesis of Tyrosine from Phenylalanine

Fig. 1.17:  

#
 

Enzymes
$ *#
  *  
$ +#
)# ##  +#
)#
$ +

!   H)
$ #
   %   

$ /# 
   +#


Important Points in the Catabolism of Tyrosine


   5

2
 5
Fig. 1.16: 


 #  
#
  3  
&   %

Tyrosine Transaminase
Phenylalanine Hydroxylase
&.&C

 
 9C     A  8 
Concept
+  ,
 
  (#

  
    (  
 H7 
C%

 
2?&/    
) 
 
 ( 

 !

 8 

    
"$ *   
!
  
) 
3
(

$
 $

5

 
Para Hydroxyphenylpyruvate Hydroxylase (4 Hydroxy-
Tetrahydrobiopterin Phenylpyruvate Dioxygenase)
 H   5   %C ?  2


 &


  %

 
  )    
%
   +)%&,  * 

C* 

 H    C   3   )%&  


 

 
*
 
Homogentisate Oxidase
 - ?  
 * $
  5
Maleyl acetoacetate cis-trans isomerase
 - $

Enzymes with Tetrahydrobiopterin as Coenzyme  )  +)'/,  

  #   +#
)# Specialized Products from Tyrosine (Very important Topic)
 *#
  +#
)#   
 *#
 +#
)#  Catecholamines
 9  H) 4#   *#
) 
Chemistry and Metabolism of Amino Acids 17

SYNTHESIS OF MELANIN
 %      melanosome of melanocyte
present in the deeper layers of epidermis.
 
P '/
   5     


Fig. 1.19: Catecholamine


Important Points Catecholamine Synthesis
Tyrosine Hydroxylase
 H     
Fig. 1.18:   # 
 '
&  
 
Tyrosinase    
 Rate limiting step  H7 


 

    Tyrosinase vs Tyrosine Hydroxylase
 * 
 

C  -C5
%
?8&
 ' C  3
   %
   
     3

?8& C 
SYNTHESIS OF CATECHOLAMINES  %
/
 

3
  
C23
?8&
Catecholamines are: C
*  
 ?   %
          * 2+ in
 9  
  5
 
  
  %
 /
       3
*  
  3 *  %

 
  

   DOPA Decarboxylase
% 
'
% 
*
 Q
     &
  
' )     &.&C
CQ
 $ 
      J

   9  

+R=S, DEGRADATION OF CATECHOLAMINES
 $

5 ' 
5 J
  %   
5

2

 
  
+R=S, >#N 
Remember
8    
   0 
     
medulla

Conversion of Tyrosine to Epinephrine involves 4


 
 
@ H /
 
> ?
 
< ' 
 
T 1  Fig. 1.20: <! 

 
 
18Self Assessment and Review of Biochemistry

 9  
  
  
    C  PHENYLKETONURIA
Catechol O Methyl Transferase (COMT)Q then by
Monoamino Oxidase (MAO)Q Classic Phenylketonuria (Type I PKU)
 % J

   
 
 -      

 
 3
 
Vanillyl Mandelic Acid (VMA)Q amino acid
 
 54

>#U V>T Biochemical Defect

 &  /
 ?;
 The major end product of Dopamine is Homo  &   5
%

4 +/4,  &  

Synthesis of Thyroid Hormones  
   3 
 
%
 

 C  
  2
 
    
 $   115
tyrosine residues.
%

 
    
 1$#
%
+$%, ?1%
+?$%,*  
MIT and DIT 
  
 %

 $%E?$%o%
1
+%<,
 ?$%E?$%o%
1
+%T,
%


CLINICAL CORRELATIONS (PHENYLALANINE


Fig. 1.22:      
  (#  .;
AND TYROSINE METABOLISM)
The reason for the name Phenylketonuria
Metabolic Disorders associated with catabolic pathway of Phenyl 8)  #     
7
 #3
  ## 
alanine and Tyrosine    #  "   )     0 ! ! 

  #3
   #3
 (PKU).
 3

 *#
 
 +(3   Clinical Presentation of Phenylketonuria
 4!( # 
  % : 
  

Disorders associated with melanin synthesis
    &
   5 
 
Disorders associated with excess Catecholamines  
  
 



#
  4 2  5
    
   

Metabolic Disorders Associated with Catabolic
 8
 
 
 
53
Pathway of Phenylalanine and Tyrosine
  5
2   
  
52

 2  
 %  
  
  
    
 : +&  5

%
2
  ,
 %
 5      
 

phenylacetic acid.Q X$&9H>=@N
The brain is the main organ affected by hyperphenylalaninemia.
* 94 !          #   

  

 #     $ * ! 

 

 #    .;      
 #  
 


%   !   



   3


!   
   #
    #
 $

Lab Diagnosis of PKUQ


Guthries Test (Bacterial Inhibition Assay of Guthrie)
Fig. 1.21: 
       

 H 
 %  
aromatic amino acids  A
 


Chemistry and Metabolism of Amino Acids 19

 *
 
-  ' &   Nonclassical Phenylketonuria
  
3 
Biochemical Defect
 - 
 
3 
 
    1 Hyperphenylalaninemia due to Tetrahydrobiopterin
  . defect
Ferric Chloride Test  ? ?
 
H  Q ?+% $$
 Screening 
   % $$$&6,
 *  
$ 3 

 
  ?      C   C
%

 
(Type IV & Type V PKU)
  
  35 1 ( U1 
5

 
  2 +
  
  
  *,
 3      
 

 ( ) %
   +)%&,*
 
  5  
 Lab Diagnosis of Nonclassical PKU
 % 5
  

    
 
 - 
D8 -
7  5  +fluorometric and tandem 5&
 ?
 
 %

-
mass spectrometry).  
F5

Tandem Mass Spectrometry  %

 
+-/T,  
 C
&  &  5
 %   

2
 9C 
 ;
 

3; 
 
   
 )        V
Other methods    
  5  
 
  
 -  %7   Phenylalanine C  ;
 
6  

 
Segawa Syndrome (Hereditary Progressive
 Y  5 
-&  
+-5G>= V&6,
Dystonia)
 %

 
; 
 9C  
  
C)%&*
 
Treatment of Classical PKU  - 
 / 
  
   ? $
 
 A low–phenylalanine diet
 ? 3
 5
 
 

 
  +.,  A 
 :
  
 

 

 Sapropterin dihydrochloride (Kuvan)2   Alkaptonuria

  -/T2 3     
      
5


3
 &/ 52
5A?  @





    5&6  - )

I%
+ 
 22
 &


 3
     &
 2*
 ,
   5
  1
Biochemical Defect


 5   
 
 /   ;

 
    / +/  ,
Rationale for using Large Neutral Amino Acids (LNAAs ) as 3 
C
 
treatment for PKU
 &9  #
 0 #
 0 !  0  0 
 0  0
 
 0   0 # 0 
     #  " 
   
  
 &9 # 60 &*%6
  

!           

% $
   !
&9
  
  
   
   
$
 *  

 
&9    
 
 
(   #  
  
 
  

% j

0 ! 
  


 &9      
     

    3
 #   



      $ Fig. 1.23: 


   3

20Self Assessment and Review of Biochemistry

Clinical Presentation of Alkaptonuria Clinical manifestations


 
 .<

T   & 
   
 


 
?
      /


  


  $  
 $  ochronosis,   
 
5

 2 
   2  2  Type III (Neonatal Tyrosinemia)
      Biochemical defect
 

 &

  
5 /
 +T&
/
1
 89%.H9%H?%$8Q   
5 ?  T1/&&?;
Laboratory DiagnosisQ Remember
   C 
 
 
 F #
      7#K  
  (    F,
#
   #
  #$
 - 5
   T# t(
#
      7# 
  (  T and T, $$

    #
    $
 A


 5
 '5

 5
Hawkinsinuria
Treatment  / 3
 
     
 3?
D%-*F3
 

/
  
5 
 3   *
         
 &

       
  
5  
 , +T &

 '  
  /
  
5 ?  ,
 %
   
 C 5
Tyrosinemia  %  C2 
 C 
%

 
    %
  %  $2 %  $$ T1
  
5   
% $$$  $   
  
   
  3

   
  3
Type I (Hepatorenal Tyrosinemia, Hereditary
 / 3   
  
   
Tyrosinemia)

   ; 

 
'       
 +
      
 A 
   
 ; 3  ,
 8
 :
 5
2 2  
 


5 Disorders Associated With Excess Catecholamines
 


#

 8
  5
 
      ! !

    
 
 2     


#
%
  4# 

 
        9


 # 6 9/ 6"
    
  
 # 5   # 5 89 50
 *  
 '  
89 5"
Diagnosis
 &  
5   5  - 
  Pheochromocytoma
 95 '  
 
 '     1
    
2 
  adrenal and extraadrenal

 
 2 52 
Treatment 

 ?3&   %
 Clinical Presentation
 3?
D%-*F The classic triad of Pheochromocytoma:
 9    
Type II Tyrosinemia (Oculocutaneous Tyrosinemia,  /  
Richner-Hanhart Syndrome)  &
 3  
      
Biochemical defect 
 
 %

  ;  
 
  3 

Chemistry and Metabolism of Amino Acids 21

Biochemical Testing of Pheochromocytoma and Tryptophan Pyrrolase (Tryptophan Oxygenase)


paraganglioma  ?  
Elevated plasma and urinary levels of:  $
 &

  
 +    
 *     &
,
   
 Kynureninase
 4     *C&.&
Biochemical methods used for pheochromocytoma
and paraganglioma diagnosis
Clinical Correlation-Kynurenine Anthranilate
Pathway
Diagnostic method Sensitivity  
24 hour Urinary Testing
$#
3
 

$:$
 
 ?
 6
  
 ?E
6$ ' #  acid LL LLLL 3 
5$  
  LLL LLL  / ;23 &

>$ / 
  metanephrines Q
LLLL LL  
J$ *
     LLL LLLL ?   

 

 

$:$
 
Plasma Tests  %    &.& ;    
 
6
  5
6$  
  LLL LL
 / 6
     3  5

5$ /     LLLL LLL
Z 
 

Albinism
?

  3;%
 C
3C 2   
 

* 
Generalized Albinism or Oculocutaneous Albinism (OCA)
 OCA-1 *#
  
 OCA-2 *#
  
  
 

 "
 OCA-3 
0  H"
Syndromes associated with Oculocutaneous Syndrome
  (    ! # 

 + 3#%3 # 

 k3%+! # 

Ocular Albinism
 H   9   #"
Localized Albinism:
4  -
 
 T ! 4# 


Fig. 1.24:  
  (#
#

TRYPTOPHAN
Specialized Products from Tryptophan
 
     9 9
  "
 9     Serotonin
 

 '  
 


 )    
Nicotinic Acid Pathway of Tryptophan
Catabolic Pathway Tryptophan  <S%
  
 3 
(Kynurenine -Anthranilate Pathway)  60 mg Tryptophan is converted to 1 mg of NiacinQ
  J
  %
   C  Quinolinate Phosphoribosyl Transferase 


    

     3 


22Self Assessment and Review of Biochemistry

Serotonin (5 Hydroxytryptamine) Hydroxylases dependent on Tetrahydrobiopterin


'C
 Q2    #   #
)#
2   
  *#
  #
)#
 *#
 #
)#
Functions of Serotonin
Remember all are aromatic amino acids
 




 5 
 )$ Amino Acid Decarboxylase
 % 
    N 8/ %
    
   N 8/ %
 1
 2
'

 &5 



Melatonin CATABOLISM OF SEROTONIN


'C&     Mono amino oxidase C
Functions of Melatonin  5 Hydroxy indole acetic acid (5HIAA)  

 

 '

 ?
 5
 
 
 


N/$[N V 
 -  

 ' 3 
SYNTHESIS OF MELATONIN
  
31 
  
 
 
'1+',

Excretory Product of Tryptophan



 


 %
   
5
Hydroxy Indole Acetate and Indole 3 Acetate
METABOLIC DISORDERS ASSOCIATED WITH TRYPTOPHAN
METABOLISM
 
 4# 

 + K <
  < 4# 



 !
 ;<  
>
 -  
 

 

 % 

 Q* 
'

 $
 ''


Clinical Symptoms
   
 $
 ?
 
+<>#RTS, A  +U<#ONS,
Fig. 1.25: 47 
  
 #
  '3 
 A    

Important Enzymes in the synthesis of Serotonin  &
 
Tryptophan Hydroxylase Diagnosis
 H        
     '


 
  

N/$
 
 -  C 
     5
  N  \   
3

' 
# 



    ( *


 %

 
C
C ( 
 ; 
   ( '  
Chemistry and Metabolism of Amino Acids 23

Typical and Atypical Carcinoid Clinical Features


Typical Carcinoid    
 $  by a midgut 
 
 Cutaneous Photosensitivity    
 $
 N'

   
 $
         3
 9 
 C2
  
  
 

 Pellagra like symptoms.
 $
 

 N1
   +N1
Pellagra Like Symptoms in Hartnup’s Disorder
/$,
< 
 

*#
 
    $
<  #
*
 9<+  (#  !
9
Atypical Carcinoid  #$
 Foregut 

       
  

 Laboratory diagnosis of hartnup disease
Biochemical Defect  Obermeyer Test +%
  
 ?  ;C
  
, 5

  Treatment
 N1/
%
   +N/%&, 5
  . 1 
   
   
N1/

  +N/%,+
, 

 N1/

     
   -  %
 3 
 +N=#<==

   V>T
,   1

 &  
5

  Blue Diaper Syndrome (Drummond syndrome)
 *

 2

N1/

      %
    ;   

N1/

  +N1/%&5
N1/%  %
only in the intestine + 
,

  /
 ? , 
 - 

N1/$5
 5  Blue Staining of the Diaper in Drummond Syndrome
*  
  3
(
 
 *#


     !
$

SIMPLE AMINO ACIDS


Hartnup Disorder
Glycine
   H5*
 '   
   
 
    3  

   
;  )  
 8   5  Q
Biochemical Defect
 ?5 
tryptophan and other neutral Biosynthesis of GlycineQ
amino acid from intestine and renal tubules  )

  
 %
 



  +-=%@, )
Glyoxylate, Glutamate and AlanineQ
SLC6A19   From Serine  '
 



 %3    

   
2 % 
5

 
 15
  C +*9>,  
 
    2  
  

   2

7 


-=%@

 


SLC6A19 
 %    3/
 

2
 
3 5

  
2

 ;-=%@




   
 Fig. 1.26: 


 
!# 
24Self Assessment and Review of Biochemistry

 -Glycine Synthase System $5



 Step II Guanidinoacetate Methyltransferase
 From Threonine %
   ' .5

Serine Hydroxymethyltransferase  *
 

 
!
   7#$  '

 /#  Step III Creatine Kinase


 '    
  


4 
#   &  
 %
  
/
 $QDNB
 T 4   
 
# 0  ? 
Q AIPGMEE atom  *
    


4   
 
*+/$ Step IV
 8
   
Catabolism of Glycine  *
 

By Glycine Cleavage systemQ
&
5

 
Glycine cleavage system consists of three enzymes and

    
 
  
moiety. The three enzymes are:
 )?
  
 


 ?
 ?
  
 %5

 
 
     o CO2 + NH3 + N5, N10
Methenyl THFA + NADH + H+
Specialized Products from Glycine
 Creatine, Creatine Phosphate and Creatinine
 + Fig. 1.27: 4# 
   
   9

 Glutathione Heme
 ' *E)o/
Glycine as Conjugating Agent  $5
 





 *J   -  +)2 )1
*, Formation of Purine Ring
 *J -C  *T2*N2O&

 
 )
Glycine + Benzoyl CoA o Benzoyl Glycine (Hippuric Remember glycine do not contribute to Pyrimidine Ring.Q
Acid)
GLUTATHIONEQ
Glycine as Neurotransmitter
 $ 
3   )   )   *
 - 
 




)
Glycine is the recurring Amino acid present in  %
  Q 

)  2
the Collagen * )
 95

  * )  &   Q
 
5  )'/
CREATININE  - 
)  ' L

 
 'C
<(Glycine, Arginine *
and Methionine Q)
Functions of GlutathioneQ
Steps of Synthesis of Creatinine  Meister’s Cycle or Gamma Glutamyl Cycle
Step I Glycine Arginine Amido Transferase ( 
 
  2
 A
     

 Guanidino
   
  


  ( <%& C



 
 
)
)   
Chemistry and Metabolism of Amino Acids 25

 Free Radical Scavenging  $  ; 


 C
( 9  H-*2
 
H-*  1   

2+



Q   5
 
 
7 
 

+5 -U,  
,
 &

 

Primary Hyperoxaluria Type II (Glyceric Aciduria)


 ?    ;  ?1 
 
  
+  
  C ,
Secondary Hyperoxaluria
 &
;+ 

  1  


,
 
  
 / 5 *
 
 
     -
P
+3C
  ,
Fig. 1.28: /   ! !  $  3   
 3   

 Reduction of MethemoglobinQ 5
3+enteric hyperoxaluria,
( 6 


  
Nonketotic Hyper Glycinemia

    
 ?  )* 5 '
 Conjugation reactions in Phase II Xenobiotic
reactions
ALANINE
( )  '

C
  C

   '   
Remember
    
 4
   9% # #   &
      
     * # #   %
 
 
 
  Q
 &
   1  +* ,
METABOLIC DISORDERS ASSOCIATED WITH GLYCINE
 # +#
) *#  Biosynthesis of Alanine
 # +#
) *#   A
&
5 %
  .
 9
3
 +#!# 

SERINE
Primary Hyperoxaluria Type I
 /

     
 %


 
 
 
 )    
    
 Polar amino acid

Biosynthesis of Serine
 A
 




&.& C
 
 A
 
 <&  

Remember
'   @
 


 
!#  
 Q
  #
) $Q

Metabolic Functions of Serine


Fig. 1.29: 
    #
)  &





26Self Assessment and Review of Biochemistry

Cysteine
  
 Glucogenic Amino Acid.

Specialized Products Derived from CysteineQDNB


 #  

)# 
!  
 
 $
 
 7# 
Fig. 1.30: 


 
!#   * 
 Glutathione
 '
 

   #  - 
  


(
#  $
( '
EE

 A
    Remember
( &  
  4 

)# 
! 8 
 
 '
    
  #  

)# 
!  
 
 
( *
 
  

( C 
 

The Amino Acids that Decreases Aging
 '
 
  *2     
3   *
( 9  ' ;'

( *+%
  ,  %

( - +%
 ,
The Amino Acid that Accelerate Ageing
 /

METABOLISM OF SULFUR CONTAINING


AMINO ACIDS
Steps of Methionine metabolism
 *5
'1
+', %
 
 
 '/
Fig. 1.31: 4# 
  
 '



'QDNB/AIPGMEE
 '
 
)
'18) -
    '
 %

 
 
& 
 
'

%

 '
 & *

  
 

'  2
*

Remember
 4 !
 
 
 

SULFUR CONTAINING AMINO ACIDS


%
 *

Methionine
 ' 
*  
 9 
 )   Fig. 1.32:  

  
  ! 

Chemistry and Metabolism of Amino Acids 27

 %3 / Contd...


 '* Acceptor of methyl group Methylated compound
 H Ethanolamine Choline
 ?
 * Carnosine   
 # 4

 

Step I Methionine Adenosyl Transferase (MAT)
 5
'1 2 POLYAMINE SYNTHESIS

 

 %& 
  & 
 
     5    
 3 Isoenzyme QDNB 

%
%1$2%1$$ 
 %

and MAT-III  Cadaverine 
5

 .
 %1$  %1$$$   5
 %1$$    Putrescine 
5 
 
   8
-

   
Methionine has to be activated to S-Adenosyl Methionine (SAM)
 Spermidine 
5
8
 
  
 0  
   3! %4%"  #  $ 
#  Spermine 
5
8
 
!
    
4  
 3   # !
 $
+   # !
   #   

$

Step II Fates of Homocysteine


@ Resynthesis of Methionine
 - 


   
   /2

C
 N5 Methyl THFA and Vitamin B12 55A 

 3
> Synthesis of Cysteine (Trans sulfuration reactions)
Cystathionine Beta synthase
Fig. 1.33: 4# 

# 
 /  3 '
  

*  
5   />8   C Steps of Polyamine Synthesis
* - '   8
  
   
 & 
2
 &.&C   C 
 
  8

Cystathionase 
 
  & 
 *   *  /
  
*    '    
   

 &.&C ?
 '
 - 


  /
  5
   ?
 ' <
  @B
&
 *' * 
 & 

' 

 ?
 ' <
  @B
Functions of S-Adenosyl Methionine 
 ' 

' 

 %
 
  <  
?
 
 ?   %
   5
2  
 & '
 3? H
Transmethylation Reactions  A 

 
3
  
3 

 
   
Acceptor of methyl group Methylated compound
 ' C  
 

Guanidinoacetate Creatine
 H
 
9
   Epinephrine
Methionine can be synthesized from Homocysteine but it is an
Epinephrine     essential Amino Acid.
Contd... *   +

#     
  
 
28Self Assessment and Review of Biochemistry

Vitamins in the metabolism of Sulfur containing Clinical Features


Amino Acids  
  

 %
 5  
 4  -@>2 A  ' 
  
 ;  
4 -U   

5 5   
 4 -@> A
'   %     
<
 2

  3    
(ectopia lentis)
 Vitamin B 6 for Cystathionine Beta Synthase 
% 5
   

+%
 

 , *   +7 5
 
,
 &

5intellectual disability 
Folate Trap (THFA Starvation)  Skeletal abnormalities
 Marfan
syndrome    2 3    

  2    5   


  2   5  2   5 2  1

 2 
3 

 %
   5 
 2 
2    
 
P 
Fig. 1.34: /
   *+/ 4  
"  Thromboembolic episodes 55   

 52  
2

 '  12  #  


9X  # *+/
*+/   
   
 
3$ *  
#  
 (  *+/  $


 
# /
     9X  # *+/$ *   Diagnosis
/
  *$ *      #
 *+/

 

 
$ + 0   
 THFA Starvation.  95  +

 In Folate trap, the folic acid is trapped as its  # ,P 
 
derivative QDNB  *53  
 '  65  # +

#   
 
 
 '
   
 /
 1*  
-
 
 $ +  +

#    $ *   3 

%Q 
5
 
for acute coronary Syndrome.
   
METABOLIC DISORDERS ASSOCIATED WITH SULFUR
 9C  5
  
 

CONTAINING AMINO ACIDS ;
 
 +

#  -  ?     
–  +

#    &
      9C     

– 9
 +

#  $


5
?  
 #  
  #  
 "
 +# 
  Treatment
Defective reabsorption:  / vitamin B6+>==#@2=== V>T
,
 #  
 
 
5 
 H 
 4# 

Lysosomal Storage Disorder:
 '   
   4  -U2  
 # 
  A  A
 +@#N
 V>T 
,       
 

 
Classic Homocystinuria
 H
      J 




  3  


Biochemical defect 3
 
 55 -U
 ? ;* - '   -  +
 , 3
 
 /5
*2
 5P 
  
; 
 
   5   
   

5 *+@ V ,
2
 
   
5  
Chemistry and Metabolism of Amino Acids 29

Nonclassic Homocystinuria Cystathioninuria


*      
 
 ?  
   

 2  2
  
 ?;

 H   Cyanide nitroprusside test negative.
+%/AH,
Cystinuria
Homocystinuria due to defect in Methylcobalamin
'     $ )

I%
+*
 22
&
 2 
 ,
       
 
  C
 23  C
   ?? %
 


methionine  ?5
 
*28
2.

+H
1*8.,
 / 
 
 Cystine, Ornithine, Lysine and Arginine +*8.,Q
 /    


 5
 
 *'

:  
  #  * 

%&5
 ^#  
 
%
    %
 3  
   C 
  :
     
 

 
 
     -


 
  ; Oasthouse Syndrome
 /
    
    
  

 /   

Remember Primary Hypermethioninemia


*  
# 
     
  ?   ;    


#   
 #  
  ?%#  < #$ %

+%$ $$$,
 %$$

 
5
Treatment  & 
-*  
4 -@>

  +@#> V>T

,    

       Cystinosis


;   . '
?


Homocystinuria Caused by Deficiency of  '    


Methylenetetrahydrofolate Reductase 


 
'
^      '  
_ `  *        *%' 2 3
uctase (MTHFR)  5
2
 %C
 N2@=1

-  * /+
5 
 


 
N1

   H     
 
 N%/A
5
   J




  ( 6
 //  ( .5

 /  /
 ( 9
        + 1 ( -

*  
 , Diagnosis
Treatment  ?
 

 *  25 -U25 -@>  *;
 

  
     +-   

C, Treatment
 - +

 3  5  ' ;
 5  3cysteamine, 3
; :,  5

30Self Assessment and Review of Biochemistry

 %      


 
  
  MAPLE SYRUP URINE DISEASE
 
 6
    5    Biochemical defect
3
  
  ?;C-
* 6 
?
  
 ?5
 ?5?
 
BRANCHED CHAIN AMINO ACIDS
Components of Branched Chain Ketoacid dehydroge-
nase Complex and defective components in MSUDQ
Branched chain amino acid Metabolic fate
Gene Component MSUD types
'  
! 
86_    _ .
 *#   4;<
&  .
!  
)# 
  *"
Isoleucine 
 .
!    
!  86?    _ .
 *#   4;<

)#
E2 <#

# * # 
  *#  4;<
Remember
&
"
 w 3  
    
 !

 
 
   8> <#

 <#
!  *#  4;<
    
   $Q 
  /<"

Clinical features
Three Common Steps in the Metabolism of
 : 3

  
5  

Branched Chain Amino Acids
  5 @ 3
Reaction Enzyme Coenzyme  .
  25   3
6$ *   
Branched Chain & 3 


  
*  
 &    
5   
 
5$ H)     .
* 
<
)# 
 <#
!  #

 0   

 35
 
/<0 9<L0  &
 
  
 3 
&
   
 P   
 55
>$ <#
!  
# 
 /< 
+   ,
<#
! 
 %  

 
+

, 

23 2 

After the First Three Common Steps   H
 
Lab diagnosis
 &  3
5  2 2
5 2   + 

 

  ,
 
  5 2 2 
5  

 5 
 6 
by Dinitrophenylhydrazine
(DNPH,%
 H
I%
 9C     
;
 
 %  ' 


Main Metabolic Disorders Associated with Branched Chain
Amino Acid Treatment
  4# ;  < H
-
* 
 
 
)5 % 
Chemistry and Metabolism of Amino Acids 31

Isovaleric Aciduria Nitric Oxide


Biochemical defect  
    5    
 
2
 ?5.    
 52

 
 ?59C$5 
*?
    4

 1+=@,
 *

odor of Sweaty FeetQ 
  A

   9  ?
5 H  
A 

Intermittent Branched Chain Ketonuria  )   


 H  5-
* B6   ' 
)&

  Functions of Nitric Oxide
BASIC AMINO ACID  &4  

 $55&

Lysine  



 &
 

5 
 H

6 '
 9   .3 5  8 55  &
  
 ' 
    
    . *  / 

 &
'
*   3   $ 2 5  
  
 ;*
  & 
 &
 6  Therapeutic uses of Nitric Oxide
Functions of Lysine  $    
 8   
  
& 
/ 

 /
. 
 *5 *

 %
   $  +' ;  )&
*  ?
9 
& 
,
 e
 .
':I   )

35

8
 .   3  +'       &


,



*

 - 
  & 
  +?
 ,  . Synthesis of Nitric Oxide

*  5

 /&

.


Fig. 1.36: 4# 


 
)
Nitric Oxide Synthase
 *9C
   
Fig. 1.35:  

!  
A5H* 


Arginine @ ?&/
 )   > A?
 '  < A
 .)   ' B6) 
 T /
)   3  N %

 

Metabolic Functions of Arginine Three Major Isoforms of Nitric Oxide Synthase
 
8' Subtype Name Characteristics %  

    6$ 9H4 /     #


 4 

 
 8
 
 +%
   neurons !! 4)
   #    2+ Behavior

 *,
 *
  Contd...
32Self Assessment and Review of Biochemistry

Contd... Important points of the histidine metabolism pathway


Subtype Name Characteristics %  
 
   
5 5/
5$ 9H4 Prominent in macrophages 
    A$).A
)  
Independent of elevated Ca2+
  #
 A$).
5
/
infection  $A;A$).


>$ 9H4 /    Elevated mean
 
 $ 

 $ Histidine Load Test


   # 2+  %A;
 A$). 
 
   
 3  
Mechanism of Action of Nitric Oxide / 
Biologically important compounds derived from
Histidine
 /  
   
  &.&
 C
 *
+-  /,
 
+*
,
 /
+)  - 
/,
 9

Function of Histamine and receptor responsible for its
action
Type of receptor Effect
+6 4

  
 
$  
Agmatine
  #$
 ?
5


 
+5   +  
$
 &
 




+> 4#    


    
   5 
5&
 

 $
HISTIDINE
   


    ;  /  
 '     
 * $ C

    :
       / ACIDIC AMINO ACIDS
Metabolism of Histidine Glutamic Acid (Glutamate)
  
 )  
 *

 
 
  

 
GlutamateQ 
  
Biosynthesis of Glutamate
-
 5  B6  
  C
)   
  

Fig. 1.37:  

   Fig. 1.38: 
# 
!  
Chemistry and Metabolism of Amino Acids 33

Metabolic Functions of Glutamic Acid Canavan Disease


 Synthesis of N-Acetyl Glutamate    
5


 &5
  
  *
  &   -  

5 5  CX3
 1@
   

 
 )  E* 1)  - Biochemical defect
 E*'/  ?  
  2    *  5 
 Synthesis of Glutathoine (Gamma Glutamyl  
Cysteinyl Glycine,  N1 
 2 
5 5 
 2
 Synthesis of Gamma Amino Butyric Acid (GABA) C

 %      N1  
  
( )  
  5)-
32  
5  

5

  2
( &.&C 3

 
  2  5  N1 
  

Glutamine
N1  
 
Biosynthesis of Glutamine
*

C
)  C
)  )    Leukodystrophy
'   95
N1  
 
Diagnosis
 
  ; 

;
 
 $
  
  N1  
   


Asparagine
Synthesis of Asparagine

   5
  
  

Fig. 1.39: 
# 
!   ' 
Metabolic functions of glutamine
 *5

  B 

   %
  ;


 
 *

 
 
-
 
 
 < f&

 
5
)  
 <&

5
   
 '
/>
 )  *
 )   J    Fig. 1.40: 
# 
! 
 '
 

623 
Asparagine Synthetase


 
      
 
 '        )  
Aspartic Acid (Aspartate) ' 
    $
 ' 2 )  

  
 )      2
5
 
 / ;  )  ' 
Synthesis of Aspartate  - 
 
'  35
2 
  
%
  8   

 
Catabolism of Glutamate, Glutamine,
Functions of Aspartate Aspartate and Asparagine
 *
     
 

 '  )    )    
    6
 *
 &
' ) 
Q
 *
 &
'  
 
 
8   Q
34Self Assessment and Review of Biochemistry

To Succinyl CoAQ
 4 
 $ 
Fig. 1.41:  

!   
 %

 Remember VIM to Succinyl CoA.

To FumarateQ
Fig. 1.42:  

!    %

 &  
AMINO ACIDS ENTER INTO TCA CYCLE AT
 
 
DIFFERENT LEVELS

To Alpha Ketoglutarate To OxaloacetateQ



2 /2 )  2 &
  )    Asparagine to Aspartate, which is transaminated to
%
    6  
 Oxaloacetate.

Fig. 1.43: 8 #


* #

QUICK REVIEW POINTS FOR NATIONAL    


 / 

BOARD PATTERN OF EXAMS        
  5
  )   
     
 4  1%
   2  *C

  
 &
 
&  2%
 &  +&.&,
  3 

 1)      
 
   
 
 -  
5

 * 
  
)  
Chemistry and Metabolism of Amino Acids 35

     
 
   
   Contd...
   Metabolic disorder Biochemical defect
 % 
    
 
 
   


#
 8) 
 

 
 
  
  
 4# 
 8) 
 

4


 %
      
  *  *
  + K < <   
 

*#
  
&  ' $
   
 
   
and intestines
 
 

/ 
 
% $$+8
%

  ?, #   U#
)#  
  
+#
) *# 
 & 

5
8
2
# <U#  <#
! D#
)# 
. +#
) *#    8 7# 
)
  55* *  9
3
 #  ! 4# 
         
 
 
  +#!# 
'
5        Classic #  
    4# 
+

#  
 %
   
 
  
)    9
 $  #

 
 
+

#   $  #  *+/  
 &



.  #  
  #  

 '
5
'
 #   <   
 

#  0 H   0
 )   
 )- &#    !  
 )       6  
 H 
 4# 
 
 

 
   

neutral amino acids
 A 

%/A ?
5 5
*#  4;< E6_ !   

    .

     
 %* *  '   <
)# 


 
*4 2$    .
 <#
!  
)
*#  4;< E6? !   

    .

Metabolic Disorder and Biochemical Defect
 <
)# 


 
Metabolic disorder Biochemical defect  .
 <#
!  
)
+#
 
# 
  4#    *#  4;< 85 !   

 <#

#
*#  * # 


  
+#
 H    * 
# .
 <#
!  
)
*#  *# ' 4;< 86_ !   
 <#


<#
!  


 
   *#  ! 
4   4#  
 .
 <#
!  
)
   *#      %  " * 
 

  
# 
 <#
! 
! 
4  ! 
4   &# Canavan Disease 9 
#

!   !   Specialized Products from Amino Acids
+++ # 
 H9*%   H    "
Amino acid Metabolic products
  #  #   +#
)# *#
    
.

  
  8  0
3
 +

!   H) 9
  0 <
 "
*#
  *#  /#
   +#
  *#
) 

*#
  *#  *#
  *   *#
  Serotonin
 

*#
  *#   +#
)# ##  #
)#D  9
 #
)#  ## %
<
)#!  #    #  
 * 
+(3    +#
)# ##  #
)#D
 Glutathione
 #
)# ##  <
)#! 
   0 
   #7
#    Betamercaptoethanolamine
 
$ #   Purine
 +
4!( 4# 
 * #
#

 Glutathione
  *#
   Creatinine
Contd... Contd...
36Self Assessment and Review of Biochemistry

Contd... Peculiar Odors in Different Amino Acidurias


Amino acid Metabolic products Inborn error of Metabolism Urine odor
!    9 
) Glutaric acidemia (type II) Sweaty feet, acrid
 !  
Hawkinsinuria Swimming pool
 !    
H      ;
 Creatine Isovaleric Acidemia Sweaty feet, acrid
3-Hydroxy-3- methylglutaric aciduria Cat urine
+    /&;
 +   Maple syrup urine disease Maple syrup
Glutamate  9  #      
 Hypermethioninemia Boiled cabbage
  
 #  @  *
   Tomcat urine
Glutamine  9>   9Z
  Oasthouse urine disease Hops-like
 9>
#  Phenylketonuria Mousey or musty
    Purine Trimethylaminuria J
 < 
 #  Tyrosinemia Boiled cabbage,
 ; 4#  rancid butter

REVIEW QUESTIONS

CLASSIFICATION OF AMINO ACIDS Essential (MettVilPhly All the other


Read As Met Will Fly) amino acid Nonessential
q{
%   

 

|**^%
\ }
~q€  
  !    
 

 ) Threonine
 ) T#

Valine
 
Isoleucine
 )
Leucine
Ans. b. UGA
Ph #  
 ' )' Lysine
 ' )&



‚{
$ 

 
ƒ
|$„*
^
~q~
2. All of the following are essential amino acids  '

 
|**^%
^
~  %
   
   %

 .  4 
    .
 .  Ans. a., e. '
2.
Ans. c.  
(Ref: Harper 30/e p18, Table 3-2)
(Ref: Harper 30/e p282, Table 27-1) Classification of amino acids based on side chain


   


 

characteristics (polarity)
   $ 

 
|6  
 Essential: %   3    ( 
  
 '
2 %
2
C    /    )  2 
2 *2 )

  *
  
 
 2
 Semiessential: )
3 

7 
 )  2/2
2.
2      Nonpolar Amino Acid (Hydrophobic)
 Nonessential:  3 C (  2. 2$ 24 2& -
2
7 
 2%
2%
   2&
2
Chemistry and Metabolism of Amino Acids 37

†{
\  

 

|$„*
\ }
~q  %

    '

 %
    Ans. c%
 (Ref: Harper 30/e page 19 table 3-2)
 $   '  
    C

 .  '  
 %
&

 %

 % C   
  &   
Ans. a, b, c, e (Ref: Harper 30/e p18 table 3-2) %
&  /
 

€{
6   

 

|$„*
^
~q

‰{
%'
   #



|Š
~‰
 
 *
 $ 
 . 
 %

 

  
 %

 

Ans. a. *
Ans. a, b, c, d (Ref: Harper 30/e p18, table 3-2)
 ' 
     
 * 

{


 
ƒ 
 .   %'  
*
5
 
  '  
   *  ' 

 
 . +% +1'/,
 /  '  
 % +*1'1*,
Ans. b, c, d (Ref: Harper 30/e p17, Table 3-1)
 -   
/2
 . 10. Which of the following is a nonaromatic amino
   

 +
 ,2 acid with a hydroxyl R-group? (Kerala 2012)
)  +)   ,  &  
 .

‡{
„   
# 

  
   %

 %
 (PGI June 2009)  
 
 Ans. c. %
 (Ref: Harper 30/e p17, Table 3.1)
 /  
   3

 1%

 .
 
    3 

 

 %
   
'
 %

Ans. b. 

(Ref: Harper 30/e page 19) 11. Which is not an essential amino acid?
 %
    (Kerala 2006)
Special Groups Present in Amino Acids  %

Amino acid Special group  /
!   GuanidiniumQ  *
 #   Benzene Ans. d. * (Ref: Harper 30/e p282)
*#
  Phenol 9   
 2 %
2
+   ImidazoleQ %
   24 2$ 2. 2&  2
Proline #
 
.+%%4$.&., /
 
  *
  & 3!
'  

*#
 Indole 12. Which of the following is not an aromatic amino
#   *


 4+" acid?
 &  
8. Amino acid produced by adding hydroxyl group  %


 & 
 #

'
     %
   
 %
 (Kerala 2011)  4 
 / Ans. d.4  (Ref: Harper 30/e p17, Table 3-1)
38Self Assessment and Review of Biochemistry


   
  ?
5  
7 
 H
 /+3$ C
 ,     5   /

  
 & +-C
 , 
5  
 %
+&
 ,
Derived Amino Acid Seen in ProteinQ
 %
   +$
 ,
4-Hydroxy Proline  /
   
!
13. Which of the following side chains is least polar? 5-Hydroxy Lysine  '    
 #
)# 
$
  (AI 2009) Methyl lysine  /
   #

 *
 Gamma carboxy  /
   
 ! 
0 3 


  glutamate    2+
 &    '  .  
  
)%
# 
$
Ans. a.
Cystine  /
   
  (  
14. Which of the following group contains only 
$Q 2014 DNB
 *(
#   
 :


 

 Œ
|\

Ž
 #   $!$0   0 
!

  Desmosine  Found in ElastinQ AIIMS Nov 2014
 - 
 
 
 -
   Derived Amino Acid Not seen in ProteinQ
Ans. a. Acidic Amino acid (Ref: Harper 30/e p282) Ornithine
 )
            Arginosuccinate   
; #
amino acid is Branched chain amino acids (Leucine, Citrulline
Isoleucine, Valine) Homocysteine < 
  
 Q
Homoserine 

#   
# 
 )
      
Glutamate-J semialdehyde 4   
 
amino acid is Acidic Amino acids, Amide group
containing amino acids, Imino acid, Simple amino
acids Properties of Amino Acids

q€{
 
# 
   #


 18. Replacing alanine by which amino acid will
 )    |\
$
Ž increase UV absorbance of protein at 280 nm
 )    } # 
|**^%
\ }
~
 )    . 
 
   &

Ans. c.)    

 )    

 
   %
   
     Ans. d. %
    (Ref: Harper 30/e p21, 22)
 
  )   
   Amino Acid Absorb UV Light
Amino Acids which absorb 250–290 nm (Maximum at 280

q{
% 

 

|$„*
‰† nm) UV light are tryptophan, phenylalanine, tyrosine.
 

   
4 
   
 /
 ) Remember
 &   
  
 
 ;' !

Ans. a.
 (Ref: Harper 30/e p282, Table 27-1)
19. Which of the following proteins cannot be phos-
Semiessential Amino acid-Arginine phorylated using Protein kinase in prokaryotic

q‡{
 
`_\


' 

  organisms? (AI 2012)
 /

 (AI 2000)  %

   %

 *  '

 .  

Ans. a./

 Ans. d. 
 (Ref: Harper 30/e p93, Chapter 9)
Chemistry and Metabolism of Amino Acids 39

&
    
 
    C  Amino Acid Absorb UV Light



   

 %& Amino Acids which absorb 250–290 nm (Maximum at 280


 
2
2


 2 nm)4 

   2   2


  81  
2 81  
2 
    
4 
   
81  

 2
 5
Remember
 *    
   '
  
  
 
 ;' !
%
3%


~{
  
'
  #
' 

} 
Š

23. The property of proteins to absorb ultraviolet
required to be biologically active. Which of the 
'
#



|**^%
’ 
‰‰
following amino acid is carboxylated?  & 
 / (AIIMS Nov 2008)  $

 /   ? ;
 )     
   
 
  Ans. d. 
   
Ans. c. )    (Ref: Harper 30/e p717) (Ref: Harper 30/e p21, 22)
 %4    C

 
-
~†{

  #
 }

 

 % 5      C 
    .1


 4 6  ?1

 %&
 
  
 4 6  ?1


  ?1 .1

( A 
$$+&

,2 Ans. a. .
 (Refe: Harper 30/e p18)
( A 
4$$+&
5

'
&

  Amino acids mostly exists in L forms
5


2'&*,2  Carbohydrates exists in D forms
( A 
 $Z +   
 
 *
 
~€{
! 
  }



|*
‰‰
 
,2  &

( A 
Z+'
&
3
 
,2  )
( &
*2&
'2  .
( 8 2 
   . 
( &
   U Ans. b.)
 8   5  )
21. Which of the following is/are not optically
inactive amino acids? (PGI May 2014)
~{
“ 
'
 
 

|*
q‰‰†
 %
  )
 %
  %
   
 4   &  
 )  /
 '
 Ans. a. ) (Ref: Harper 30/e p39)
Ans. a, b, c, e. (Ref: Harper 30/e p19)  ) 5  H
 ; 
 )   5         
 )  
 

22. Property of photochromisity is seen amongst the
'   #

 
|*
q‰‰‡ 27. Which amino acid can protonate and deprotonate
  
   at neutral pH? (AIIMS May 95)
 
     /
 
   . 
 ?
   )
Ans. b. 
     

(Ref: Harper 30/e p21, 22) Ans. a./
40Self Assessment and Review of Biochemistry

   3  


   
  31. Transfer of an amino group from an amino acid to
 3    :
 

3




|*
~qq
   3 6  k /        %
  
   :
     
 6  C
 UN#OT  %
 
  /kO2$ C
      ?  
 :
 Ans. a.%
   (Ref: Harper 30/e p290)

~{
$   
'



|$„*
’ 
‰ Keypoints transamination
 '
  $
5
 
B   B6 
 %
 ( 6 

  
 
 .  &
5 
 %
   
( 6 

  

-
Ans. a. '
2 b.%

 8   
(Ref: Harper 30/e p93, Chapter 9)
( 6 

  
)  -
29. Which of the following amino acid is purely  B6) 

„ # Œ
|\
$
Ž  A

5

 4   %
  
 B 
  
 . 
  .1)   
    .1)       C   

 )
       5    
Ans. c. G4 V)
  
 .    + , 
 %   5      
 )2  4 

)  
 %   
 
 
 
 -  
 )    
   
Remember  ' ;
 
 
  ;


    #    

 


  

!

! $

 
 

 &
 &  C
GENERAL AMINO ACID METABOLISM 32. The amino acid which serves as a carrier of
Digestion and Absorption of Proteins,  
' 
3 


}

Transamination and Transport of Amino Acids    (AI 2006)
 
30. Increased alanine during prolonged fasting rep-  

 
|**^%
\ }
~qq
 )  
 $
 
 3 

 $ 

   Ans. a.  ( 
 
 ?
  C     
  Transport form of Ammonia from most tissues
)   including brain is Glutamine
 .      
      Transport form of Ammonia from skeletal muscle
  
   is Alanine.
Ans. a.$
 
 3 


‚‚{
„  


 

|$„*

‰
?
 
   2

   -
 /<
 


 
5  -


     %
 % )  *
* *  '


'   5 

 
5   
  
Ans. a. 
 


Remember
  

!   !  
   $ %
 

  

 

  #
  
   $  
Chemistry and Metabolism of Amino Acids 41

‚†{


  


|\
$
  Contd...
 A  
 
 &#  
 4&YY"    
 
 & 5
 
 Intolerance  
        
     0 (  

 5
 
 #

   
 
 

 &     ) !
 9+
Ans. c.5%
 
        
$

A
  
 
 
    H 
 ;    
 %  !  
  
Disease       ( !! 

mucosa by sodium-dependent active transport%

   $ !%3

0 
  !
(Smith Strang
5
:
  
 

23 ; Disease) 5%#
)# # 0  0    $

 
  1  
!#  
 


 0 #
)#
 0
Transporters of Amino Acids   !#  
 



 
  4&>W5  
 A
 

<
)#  8) 
# 
  4&66 
 A
-   *
  $
 A
$ ) 
  (  

! #
 
 For Acidic Amino Acids  H& 
# 
 #0
 
%
!0 3
 
 #0     
%
 A
- +-  ,
  

Meisters Cycle
 A
 
   
   
 35. Nontoxic form of storage and transportation of
 
|\
$
Ž
$26   

 
 
 % 
  )  +)'/,  )   
 A
 
 
  @   
 
   )  
)'/<%&

7 
  )   
Disorders associated with Meister’s Cycle Oxoprolinuria Ans. c.)  
 5 Oxoprolinase ; 8

  %
 
 
   
    
)  
Disorders Associated with  % C
     )  
Absorption of Amino acids ' 
+ K < 
 

  
0  - .  
  !    
  H7 
%&
#

SLC6A19, (   :
   Urea Cycle

%    

transporter of small intestine and renal
‚{
•

 &

|$„*
^
~q
0           )   
protein  
 
 < 4# 
 *#
  # 
    
   
or Drummond     )
#        8


  
4# 
     
  3 #$        )   
  

   
 #


  0 (  

   Ans. c. 
    2 d. 8



 

       #
# 

   (Ref: Harper 30/e p293)
 
) 
Reactions of Urea Cycle
#   < 
0   ! cystine, 
  
  

 
   !
ornithine, lysine, and arginine are


"

   

 
  !
3  #  9%   4&>6D
4&YZ0      (  Carbamoyl Phosphate Synthetase–I (CPS-I)
    #    *
  &    
 
 
Most common disorder associated with  *8~–2  %&
Amino acid malabsorption$
 *&'1$  
   +  
, C 
Contd...  3 
42Self Assessment and Review of Biochemistry

 *&'1$  5   


  1  A
  3 
 
)   2  
 5 

 
2
2
  2
 % 
7 
>%&


Ornithine Transcarbamoylase (OTC) 39. In which of the following condition there is


%

 
 
   *
  &    increased level of ammonia in blood? (Ker 2008)
8
 
  *
 ' 7       8


  ;
     )  
 /
Argininosuccinate Synthetase
 &

 .
 
 *
 

5
 
 Ans. a. 8


  
 %
 
7 
%& Increased ammonia in blood is suggestive of a urea
cycle disorder. So answer is an enzyme of urea cycle.
 %3
   
 C
Argininosuccinate Lyase 
 ! "" #$! % &
* 5 
  
 A 
  
 *

Q
ArginaseQ  ?

 3 X-linked partially dominant
Hydrolytic cleavage of Arginine2
  
   inheritance + 
 
  * ?





8
3


   H5,
Remember  
 

38


   7#   #
    (     bK  
 5      
 
     335

37. Which enzymes are part of urea cycle? 

 8
%

   (PGI 2012)  8
  
  
   

 
 
5

 )   ' 
 
  
†{
•


 
|Š
~
Ans. a. 8
%

  2 d.
    *  
(Ref: Harper 30/e p293)  

Enzymes of urea cycle and its classes  -
 9  
  
Class of enzyme it
Enzymes name belongs Ans. c.-
Carbamoyl-phosphate synthase I Class 6 (Ligase) "
 

3



   

Ornithine carbamoyl transferase Class 2 (Transferase)
 /'
Argininosuccinate synthase Class 6 (Ligase)
 
 *
Argininosuccinate lyase (Arginino- Class 4 (Lyase)
 )  
Succinase)
41. Which of the following enzymes(s) is/are not
Arginase Class 3 (Hydrolase)
involved in Urea Cycle? (PGI May 2012)
 )   ?
  

‚{
•


 
|*
~qq
 
  ' 
 .5

 B6  
?
  
 )$%
 $
?
  
 ' 
 A 

 6
Ans. a, c, d, e (Ref: Harper 30/e p276, 277)
Ans. a..5

 ' 
    5
 
    )   ?
  18 5  
  
  ' 1
 *
 ?
5   3     5    6) 
?
   $



 
8
2*
2
1 ?
  1%**
    A 
1%**
Chemistry and Metabolism of Amino Acids 43

42. Glutamate dehydrogenase in mitochondria is Ans. a.  


 
 } 

|\

Ž (Ref: Harper 30/e p293)
 %&  A

  t
 
 )%& CPS-I
 ?/  ' 
   
 t 
 
 ?& 
  ' 
Ans. d. ADP (Ref: Harper 30/e p291)
 )   ?
  +)?/,
††{

` ` 
 
 
 
'

 .5
)   ?
  +)?/, 
- thrive with high glutamine and Uracil in urine
 %&2)%&2?/ Hypoglycemia, high blood ammonia. Treatment
 .5
)   ?
  +)?/, 
- #}
' 
~
 {


 
#
 

  5 ?& for failure to gain weight. Gastric tube feeding
was not tolerated Child became comatose.
 H5

    
   5
 )   
Parenteral Dextrose given. Child recovered from

 
coma within 24 hours. What is the enzyme defect?
 *  
?E or NADPE
 *&'@ (AIIMS May 2015)

†‚{
\  #
 
'
•
  
  8


  
   
  |\

Ž  
 
   )     
  ' 
   ) Ans. b.8


  
   
 (Ref: Nelson: Defects in metabolism of amino acids page 672)

Urea Cycle Disorders


44Self Assessment and Review of Biochemistry

*

#}



# 
  ?

  3  
 
 High glutamine:   
  


-      
 
 
 
  
 
   
 

 
 '
 
 )  55  Disorder Enzyme defective

 Increased uracil 


      8
 Hyperammonemia Type I 
# 
 
4#    4%"
%

       8%* -
52
    
   Hyperammonemia type -II H    * 
#
H*"
  %
&

Citrullinemia Type I (Classic ! 
   #  
&
 
   
  
Citrullinemia)
   /2


Arginosuccinic aciduria ! 
   #
45. Which of the following is true in relation of urea Hyperargininemia ! 
cycle? (PGI Dec 05) Urea Cycle Disorders due to Transporter Defect
 A
>    Citrullinemia Type II   * 
     
 A
> 
   " < 
 ?  C         ;- Hyperammonemia H    * 
  < 
  Hyperornithinemia
 
 
/<2     *8> Homocitrullinuria (HHH)
 *
  
     
- Syndrome
     .

Ans. b. A
> 
 2c. ?C 46. A baby presents with refusal to feed, skin lesions,
    ; e.*
 seizures, ketosis, organic acids in urine with

     
     
 —
3
# 
|*
~q
.
  &

 

Urea Cycle    
 ;
 A
3 
 2

    

 
    
 C;
 8
  3 *
  &   Ans. b.  *
 ;


 C8%* (
 
!
 "!# ! $
Chemistry and Metabolism of Amino Acids 45

†‡{
"
 


|$„*
^
~q€  Histidase 

 
   
   
     

 

   3
ammonia  L

 2* /
 %&
 5

 -
 

€{
˜
'



 } }
   
 8         
2  )   1    (AIIMS Dec 98)

 
  /1 
 8
      1 
 ' 
      
1 
 

 Ans. c.  1 
Ans. c. 8     
2 *
5 5      
  

 
2d. 8
   2 e.' 
   



   
   - 9! #   - ; ! 

 
 
 
  
  ! 

 %& 
7 
 
 *&'1$  
   +  - 
 # ! # - ; ! 9


'  Polar
 8 N
 2<
 
   Alanine-Uncharged Nonpolar Leucine: Uncharged Nonpolar
'
    !  - 
 # !   - 9! #
 8      
2
    Polar ! 



 

†{



#  #


  INDIVIDUAL AMINO ACID METABOLISM
(PGI Nov 2014) Aromatic Amino Acids
 
 

 
 H    C  8
 

-
€q{
˜

} 

$:$
 Œ
    A$). |\
$
Ž
 H7 

 
  Z 
 
    
 
     
 8
 
 

   /
Ans. a. 
 

  2 c. H7 

 Ans. b.Z 
 
 
2 e.8
 

 e  

    4  - U  
 
 
 

    Z 
 

   

    A  ?
 <%&


7 


  A
)   2/
 H   *
 &  ' 1$  

    4  - @>  
 A 
 
 
  /2 
49. Enzyme involved in nonoxidative deamination
€~{
  

 &
' 
|\
$
Ž

|\
$
Ž  %
   
 .1  8   %

 )   ?
    %

 )     .
  ?
 Ans. c.%

Ans. d. ?
 ^  
  
' 
' 
"  

Some examples of Nonoxidative   
DeaminationQ NBE pattern  %

 Amino acid Dehydrases 
   3  *   +? 2 9  
2 
 -


 +'
2%
,  
,
46Self Assessment and Review of Biochemistry

€‚{
*
$ 3 



'

 
 
?
    


|**^%
\ }
~q in

 H  5C  $  8
 -$
5
-
 H  ;
  
?2
 2  
 .  

;C
 )5     Laboratory Diagnosis
Ans. c..  

;C    C 
 
 

(Ref: Nelson: Defects in metabolism of Amino  - 5
  
acids 20/e page 638) 
   
 %

  

      A

*
 5
5    
  '5

% 5
Treatment of Classical PKU   H
 
 A low–phenylalanine diet
Treatment
 

 
  +.,
 

 
  3?
D%-*F3

 Sapropterin dihydrochloride (Kuvan),   /
 & &
5  
  3

  -/T2 3     
  
5      
3
 &/ 52
5A?  '  %
 

    5&6

€€{
  
  
 
|Š
~‡
Preliminary trials with recombinant phenylalanine
 ? 
  

ammonia lyase have been encouraging and demonstrated
 ? ? 
reduced blood levels of phenylalanine during treatment
 
  
9  

54. A 40-year-old woman presents with progressive  %
?
palmoplantar pigmentation X-ray spine shows Ans. a.? 
  

 
'

3{
!
  #
   
(Ref: Harper 30/e p320)
reagent to urine, it gives greenish brown
*5
  %
  9  
 55 T
  

`3
  
š {

7   


What is the diagnosis? (AIIMS Nov 2008)
 &
 @ H /
 
  
 > ?
 
 %
  > < ' 
 
 
  
 T 1 
Ans. b.  

€{
"
*
"  



(Ref: Nelson: Defects in metabolism of Amino
|\

Ž  
acids 20/e page 642)  %
%
  
Alkaptonuria  A 
  /
 
   H5?

  ;  T1/
& 
5 /
 
/ 8       $

 A




 Ans. b. A 
  /
 
 - )

I%
x 
 22 (Ref: Nelson: Defects in metabolism of Amino
&
 2*
 E acids 20/e page 640)
Biochemical defect Amino acidurias and enzyme defect
/  8 ;        # .
  #   +#
)#
/ +/  ,3 
 3
 +

!   H)

  *#
  *#  /# 
   +#


Clinical presentation *#


  *#  *#
  *  

 
 .<

T  Contd...
Chemistry and Metabolism of Amino Acids 47

Contd...
Aminoaciduria [   
*#
  *#  #
)# ##    *#
 
#
)#D#
)#  #
#  <
)#!   # .
  #   #
)#
+(3    +#
)#  # #  3
 +

!  
)
#
)#D#
)# # +

#   #  
    4# 
#  
)#!     0 

   #7
#    
 # ;  <    .

<#
! 
4!( 4# 
 * #
#

  *#
 

€‰{
^ 
 




|’*$^_
^
~q€
 &  
57. Terminal product of Phenylalanine metabolism  &  

|$„*
^
~q†  &  C
 A 
  &    
 * Ans. b.&  
 8   
(Ref: Nelson: Defects in metabolism of Amino
Ans. a. A 
2 b. * acids 20/e page 637, 638)
(Ref: Harper 30/e p304) Clinical Manifestations of PKU
 %
  
   &     %     
   
 &
 
%
 A 
2     

 5 
  
CoA
  
  * 5      
5
@3
Amino acid Terminal end products
 4 25
  -
! 0    H)
  
  
2  
 
Glutamine, Glutamate _ .
!  
 %  
  
  
    
Proline _ .
!    : 
!  0 H    _ .
!    '    5  

 
 C 
2
+   _ .
!   3       
   
# 0 4  H20 9+>0 9X96=  #  *+/

3

#   %
 5     
pheny-
   #  lacetic acid23 
  

*
  # 0  #  
 
  #  0 4 # 
  
   C
+z>NS,2 2
#   # 0 > 
  
 

P 2  

K 
   N=S  5

  
 
 
 #  0 *#
  / 0  # 
0   
 
  2
    3 3
*#
  # 

  2       2 
3
&  
  0  # % 


 

;  
 
Isoleucine  # 
0 4 # 
 

'  4 # 
0 ? 

 # 
60. The amino acid that can be converted into a
}  
|Š
‰q

€{
 &
 

 
  )
 %
   %
   
 %

   &   
 &  
   .
 /    Ans. b.%
   
Ans. a.%
   %
    5
 
(Ref: Nelson: Defects in metabolism of Amino  %
   C     
acids 20/e page 642) Y  & 
%

+Y&H% ,
48Self Assessment and Review of Biochemistry

Specialized products of Tryptophan  Renal involvement      A 1


 '
+N/
%
  ,  
 3 
     
   2  
  
 2      2
 Niacin 5 ?1
 
 
  
 
    
  
 
61. Which of the following amino acids is involved   
in the synthesis of thyroxine? (Karnat 97)  / 

 
   
 
 )
 
 
 %

‚{
^   
'
 

#}


 %
  ?

  (PGI June 02)


Ans. d.%
  4 

 A 
'  C
 %

 2%
2
 * 
  
*  +? 29  
2
  -
 &


,
Ans. a, b, c. ?
 24 
2A 

~{
"  

 
 

}    
'
% 
|^   
'
"  

(AIIMS Feb 97)  


-

 
   95 
  
 )$
 H 
 % H  
 / 

Ans. d. / 
  *   P   5 

(Ref: Nelson: Defects in metabolism of Amino 64. Correct combination of Urine odor in various
acids 20/e page 642)   
  
|$„*
\ }
~q‚
%
 % $+%
2/

%
 2  &
  

/ 
  %
 , *       %
 H  
%
 % $  / 3
 & 
 
 2 :  

  
   
 H 
  
3> U    
 H
    hepatic crisis  
   {›{&
  
2
        
    {{{{{b.%
 H  
 


   
      $


œ

 
  A5
2 

 2 5 2 

 2
Inborn error of metabolism Urine odor
  2 J 2 5  5  

Glutaricacidemia (type II) Sweaty feet, acrid

  2    



    
2  Hawkinsinuria Swimming pool


  *

  Isovaleric acidemia Sweaty feet, acrid


eventually hepatocellular carcinoma 
 3 3-Hydroxy-3-methylglutaric Cat urine

    *
      
 3 aciduria

  Maple syrup urine disease Maple syrup
 9   peripheral neuropathy
  Hypermethioninemia Boiled cabbage
  

 
zT=S :
 @  *
   Tomcat urine
% 
2  
 
   
 2 Oasthouse urine disease Hops-like

 

C  5
 2     2 Phenylketonuria Mousey or musty
 3 
 
   
Trimethylaminuria J
 < 

2 5 2
  2 2   2
Tyrosinemia Boiled cabbage, rancid butter
1 J
 
    
Chemistry and Metabolism of Amino Acids 49

65. Which of the following is true regarding Phenyl Serine hydroxyl Methyltransferase
Ketonuria? (PGI nov 2014)
 ?
& 

   

 
 8
    ;  &  
/
  Fig. 1.44: 


4 
# 
 8
 
  5     -Glycine Synthase System $5



 
 From Threonine %
 
 ?         
   68. Glycine cleavage system in liver mitochondria is
 %
   associated with which enzyme?
Ans. a, b, e
|\

Ž  
 &
        &  )?
  
 /
   )%
  
 ?


&  3 -  )?
 
 %
 %
 -  )?
 
   C
&   Ans. a. )?
  
 &  /
  (Ref: Harper 30/e p302)
Glycine Cleavage system consists of three enzymes and
Simple Amino acids 
    
 
  
66. Which of the following is true about glycine? moiety. The three enzymes are:
(Ker 2008) @ )?
  
 )     > %


 '  
   T  < ?
 ?
  
 /   

 8   5 69. Guanidoacetic acid is formed in.....from……
Ans. d.8  $ 5 (Ref: Harper 30/e p19) (JIPMER 2000, DNB 98)
 G   5    6K
E)
 ' 
     
 *   .5
KE)
  .5
K*E

 )  
 

   K*
E
 
 )     Ans. a.6K
E)

67. Which of the following would not act as source of Steps of synthesis of Creatinine
glycine by transamination? Step I Glycine Arginine Amidotransferase
   |\

Ž  
 A
 6
 
 
 Guanidino
     
         
   

    
 )   
)
)  
 ) 
Ans. b.
  (Ref: Harper 30/e p283) Step II Guanidinoacetate Methyltransferase
 ' .5

Biosynthesis of Glycine
 ) %

       *
 

 ) 
 Glyoxylate, Glutamate and  '

AlanineQ Step III Creatine Kinase


 From Serine '
/
%

  %
  
% 
5

   *
 &  

50Self Assessment and Review of Biochemistry

Step IV Primary Hyperoxaluria Type II (Glyceric Aciduria)


 8
     ?    ;  ?1 
 
  
 *
 
 +  
  C ,
Secondary Hyperoxaluria

‡{
 }
'
# 

 

 &
;+ 

  1  
 A  (PGI Dec 02)


,
 % 
 4*  
  
 A>E  / 5 *
 &
      
 
  1
Ans. a. A 2 e. &
 &   P
+3C
  ,
 )  5
  '
  '
 /
  $  3   
 3   

%

 5
3+enteric hyperoxaluria,
 *C
7 

 A 2 &
  Nonketotic Hyperglycemia
&    ?  )* 5 '

‡q{
\
^ 
„ 

3 

|\
$
Ž
‡‚{



 
#   
 
 9
  $ 
   (AIIMS Nov 2008)
 '
  $5
  
 *
  $J 
 -   $1 
5
 C
Ans. b.'
 Ans. b. )   (Ref: Harper 30/e p23)
Sarcosine N Methyl Glycine Functions of glutathione
Betaine * #!#  Glutathione is a tripeptide
Choline * # 
   A

  5 
Ethanolamine 4 

)# 
 %
 
  


8!

  < 
+    6 


  2
5 
Betamercaptoethanolamine #  

)# 

 
Carnosine    # +     C

 C
   
 
 # 
Phase II Xenobiotic reaction in Conjugation
+


   L +  
Serotonin X +#
)# *#   Sulfur Containing Amino Acids
74. Sulfur of cysteine are not used/utilized in the
72. What is the metabolic defect in Primary Oxaluria  
' 

'   #
 ƒ  
"
**Œ
|\
$
Ž (PGI May 2015)
 ) 5   / 5
   
  )  %

  %  
 
 ?)
?
    $
  

 94 *  ? ;  
  3 3
Ans. c. ?)
?
   J   
Primary Hyperoxaluria Type I Ans. c. $
  

 %
&

 
 
        2     
 $  ; 
 C C
*
 1   

2+
  -  ' 
        
 
  5
 
 
7 
 
 
+5 -U,  
,  % 
 

 
 &

   &.&C



Chemistry and Metabolism of Amino Acids 51

 %
     C  *     1'/+' L
,
  5

'  *  C    

‡€{
  

  
' 
 
|Š
~ Acidic and Basic Amino acids
 6

 *
'   79. Nitric Oxide synthesized from?
 *
   
 |\

Ž  
 ' 
  *

Ans. a.6
   
 %
 ;


  *
 *
  ; Ans. a.
 (Ref: Harper 30/e p661)
'
8

‡{
\` `  
 
|’*$^_
~q~
 )  
 )
 )   
 )-
Fig. 1.45: 4# 
 
)
Ans. a. )  
 % 5
   )    1 {
6   
 



' 
*' L

 *'1  A ?; |\

Ž  
*
 )    /?;
 / ;
77. Which of the following is true about Glutathione? Ans. a. A ?; (Ref: Harper 30/e p299)
 *  L

  (PGI 2000) Important Points of the histidine metabolism pathway
 A
/
/  
   
5 5/
 $ 

   A$).A
    
 %
 
   

  A$).
5
/
 &
C  $A;A$).


Ans. a. *  L

 2d. %
 
   Histidine Load Test


2e. &
C  %A?;
Functions of glutathione  A$). 
 
   
 3  
 A

  5  / 
 %
 
  



q{
"
 
\ 
! 


 
 6 


  2
5 
|\

Ž  

   &
 


  C

 C  
8'  


78. In glutathione which amino acid is reducing  8
3 9 
5H -
agent? (AIIMS June 1997)  A 

 
 &
 )   
 ) Ans. d. 
 & (Ref: Harper 30/e p661)
 *  
8 
 )&
    A



Ans. c. * (Ref: Harper 30/e p23)  8'28'28'



8
' 
 )   
  +)  )   E
*E), Nitric Oxide
 )     ) 
  5    

2
       
 3 )    
 52

 
)      4

 +=@,
52Self Assessment and Review of Biochemistry

 A

   9  ?
5 H   Branched Chain Amino Acid
A 

84. Branched chain ketoacid decarboxylation is
 )   
' }
 
|*
~q
 ' 
)&   '

 
Functions of Nitric Oxide  /
  
 &4  
   

 $55&
  )$)  
 



 &
 

5  Ans. a.  '

 
' (Ref: Harper 30/e p276-278)
 .3 5  8 55  &
   Maple Syrup Urine Disease
*  / 

 &
' Biochemical defect
 $ 2 5  
    ?;C-
* 6 
&  ?
  
 ?5
 ?5?
 

~{
   

' 
' 
|$„*
’ 

 
 Clinical features
 .   H
 
 .   *5 
 /   2*
Ans. a.
 (Ref: Harper 30/e p320)  '-
'
D '
F
Three amino acids from which Creatine and creatinine Tests for MSUD
is synthesized are  ?
&/
C%+?&/%,
@ )  H
I%
> 
  9C 
< 
Treatment
83. Histidine is converted to Histamine by which  H
-
* 
  
|\
$
Ž  )5 % 
 *
 
 8 
€{
^%•

*




  
'
 ?
   9$B |\

Ž  
    9$!
Ans. c.?
   9>
 9<
  5
 ?  


   Ans. a. 9@B
   5
       (Ref: Nelson Defects in metabolism of

  Amino acids 20/e page 649; Harper 30/e p311)
 Types of MSUDQ
Amino acid Biologic amines
Gene Component MSUD Types
+   +  
86_    _ .
 *#   4;<
*#
  *#  
)# 
  *"
*#
 *#   86?    _ .
 *#   4;<

)#
&#  Cadaverine
   Q
 
 #  " E2 <#

# * # *#  4;<

  &
"
Serine Ethanolamine
8> <#

 *#  4;<
#   Betamercapto Ethanolamine <#
!  
  /<"
Chemistry and Metabolism of Amino Acids 53

86. Which is not formed from branched chain amino  


  5 2 2 
acid? (Latest Q) 5  

 5 
 Z 
 

‰{

'
 



  
 % *
 &
  (PGI Nov 2013)
  * *
  '

? 
 * ' *
 % 1'  
Ans. a. Xanthurenate (Ref: Harper 30/e p309)
 $5 

 Z 
 

%
   6
-   & 
 C5
Ans. b. '?2d.$5 


‡{
" 


* }
 
|: 
Ž  &
    3
 
 
 acid
 .  % 1' I ?     &  

 ) '   
 
Other Amino acids and Entry of Amino Acid to
Ans. c. )
TCA Cycle
(Ref: Nelson’s textbook of Pediatrics 20/e chapter 79.6)
90. The nitrogen atom of aspartate formed from
Treatment of Isovaleric Acidemia # 
 #
 &
# 

' 
/
    |\

Ž  
H5
         +
5  7   )   
 



5 ,2

   )  


+   
 ,    6  

H5 5 
  Ans. c. )   (Ref: Harper 30/e p267)
By Administering Glycine
Remember
- 5 
    


2 *  
! 

  
 
     


   +>N=  V V>T 
,  
 
$

 
 5 
 
By administering L-Carnitine Asparagine Synthetase
L-carnitine (100 mg/kg/24 hr orally) also increases  
 '        )  
removal of isovaleric acid by forming isovalerylcarnitine, ' 
which is excreted in the urine.  $
 ' 2 )  

  
 2
5
 
88. Which of the following amino acid is excreted in  / ;  )  ' 
 



 

|*
q‰‰‰
 - 
 
'  35
2 
 %
      
 &  
 . 
‰q{
!   

' 
' 
 
  &
 |\

Ž  
 / 

Ans. c.. 
 )    )  
Lab Diagnosis of MSUD
 
  

 &  3
5  2 2
5 2   + 

  Ans. d. 
  


  , (Ref: Harper 30/e p298)
54Self Assessment and Review of Biochemistry

Fig. 1.46: 8 #

 
* #

92. Amino acid responsible for Thioredoxin reduc-


‰†{
%
'
 
'


 
|\

Ž

 }  
|\

Ž    '?
 '
  &

 '  /

 *  ) 

   Ans. d.) 

Ans. b. ' (Ref: Harper 30/e p16) (
%&# ' *"  $#+5<
Selenocysteine is seen in the active site of following $


œ

 
Enzymes and ProteinsQ Inborn error of metabolism Urine odor
 %

       # " 4( #  0 
 )  &
  +(3   4( ! 


 $
?  
  4( #  0 
 '
& >%+#
)#%>% #!  aciduria Cat urine
 #     #
‰‚{
!   

}
' 


  +# 
  
 !

|\

Ž     
)#  # *
  
 )        H 
    +
%3
 
  
 
 #3
 
#
  #
 / 

* #  
 ! 
 )   &

*#
  
 !0    
Ans. b. 
 
 
(Ref: Harper 30/e p299) 95. During the formation of hydroxyl proline and
 
  
 
    hydroxyl lysine, the essential factors required is/
 )   )   
   6  
 
|$„*

~‚
Chemistry and Metabolism of Amino Acids 55

 &
     97. In one carbon metabolism Serine loses which
 
  
 Œ
|\
$
Ž
 %  
      
     -
 -  ) 
Ans. b. Ascorbic acid  ?
 /
 &
 . Ans. b.-  (Ref: Harper 30/e p284)
 9C&
 ./
 
 *C4 *

‰{
% 
 

' 

|$„*
’ 
q‰‰
 /
 . 
 4 
 .
Ans. c. 4  9C'
/



 '  *  
  4 2 . 2 '
 *
 
) 
 %/A
2 Proteins

Topics Included
 Chemistry of Proteins  Protein Folding
 Structural Organization of Proteins  Glycoproteins
 Separatory Techniques of Proteins  Protein Sorting
 Precipitation Reactions of Proteins  Plasma Proteins and
 Methods of Quantitation of Total Proteins. Immunoglobulins

CHEMISTRY OF PROTEINS Bond


 N-terminal end: The free NH2 group of the terminal
Proteins are polymers of amino acid. Proteins contain
amino acid is called as N terminal end
carbon, hydrogen, oxygen and nitrogen as the major
 C-terminal end: The free CO-OH end is called
components. Nitrogen is characteristic of protein. On
C-terminal end.
an average, the nitrogen content of ordinary proteins is
The amino acids are sequenced from N-terminal end to
16% by weight. C-terminal end.
Peptide Bond Characteristics of Peptide Bond
All proteins are linked by peptide bond. Alpha carboxyl  Partial double bond character
group of one amino acid reacts with alpha amino group  Rigid and planar
of another amino acid to form a peptide bond or CO-NH  C-N bond is trans in nature
bridge.  Uncharged but polar.

Fig. 2.1: Formation of peptide bond

Atypical Peptide Bond (Pseudopeptide Bond) Characteristic features of an atypical peptide bond
(Isopeptide Bond)(PGI 2011)  Occurs post-translationally
An amide bond formed between an amino group and  Can be formed spontaneously or enzymatically
a carboxyl group at least one of which is not an alpha  Can produce stably linked protein dimers, multimers
group. Seen in the side chains of proteins. or complexes
Proteins 57

 Makes the protein resistant as proteases cannot Incomplete Protein


hydrolyse isopeptide bond. They lack one essential amino acid
Examples          
 Glutathione in Lysine.
 Thyrotropin-releasing hormone Mnemonic: Member of Parliament takes a Casual Leave.
 

 Bloodclots Poor Protein
 Cyclic peptide antibiotic like thyrocidin and grami- They lack many essential amino acids
cidin  E.g. Zein from Corn lacks Tryptophan and Lysine.
 Heptapeptides like dermorphin, deltorphin.
Based on Composition
Application of Spontaneous Isopeptide Bond Formation
 Develop a new peptide tag called Isopeptag. Used in: Simple Proteins
 In vivo protein targeting Proteins which contain only aminoacids.
 Fluorescence microscopy imaging  E.g. Albumin, Globulin.

Some Biologically Important Peptides Conjugated Proteins


Combination of Protein with nonprotein part called
Peptide Example
Prosthetic group.
Tripeptide Thyrotropin-releasing Hormone (TRH) Glutathione
Conjugated
Pentapeptide Enkephalin Constituents Example
Protein
Octapeptide Angiotensin-II
Glycoproteins Proteins + Blood Group Antigens
Nonapeptide Oxytocin, Vasopressin [ADH], Bradykinin Carbohydrates Plasma Proteins except
Decapeptide Angiotensin-I Albumin TSHQ, FSHQ, LHQ
Lipoproteins Proteins + Lipids Chylomicrons, LDL, HDL,
VLDL
CLASSIFICATION OF PROTEINS Nucleoproteins Proteins + Nucleic Histones
Acid
Based on the Shape of Proteins Chromoprotein Proteins + Colored Hemoprotein-Hemoglobin
Fibrous Protein Prosthetic group Flavoproteins
 Elongated or Needle-shaped or long cylindrical or Phosphoprotein Contains Casein of Milk, Vitellin of
rod-like Phosphorus egg yolk
 Minimum Solubility in water Metalloproteins Protein + Metal Tyrosinase (Contain
ions Copper), Carbonic
 Regular Secondary Structure
Anhydrase (Contains Zinc)
 Axial Ratio > 10
 They are Structural Proteins
ORGANIZATION OF PROTEIN STRUCTURE
 E.g. Collagen, Elastin, Keratin.
  !" !#     $
Globular Proteins primary, secondary, tertiary and quaternary.
 Spherical or Oval or Spheroidal in shape
 Easily Soluble Primary Structure
 Axial Ratio < 3 It is the linear sequence of amino acid held together by
 They perform dynamic functions peptide bonds in its peptide chains. Bond involved in
primary structure is Peptide Bond, a type of covalent
 E.g. Albumin, Globulin, most enzymes.
bond.
Based on Nutritional Value Amino acid sequence determines the 3D structure of
the protein.Q
Nutritionally Rich (Complete Protein
or First Class Proteins) Secondary Structure
They contain all the essential amino acids in the required       & & 
proportion. E.g. Casein of milk are about 3–4 amino acids apart in linear sequence
58Self Assessment and Review of Biochemistry

 The folding of short (3- to 30-residue), contiguous  Parallel Beta Sheet-Flavodoxin


segments of polypeptide into geometrically ordered  Antiparallel Beta sheet-Silk Fibroin
units  Both Parallel and Antiparallel Beta sheet-
 Bonds involved in the secondary structure are Carbonic Anhydrase.
primarily noncovalent bonds like:
 Hydrogen Bond (Most important bond) TURNS AND BENDS
 Hydrophobic Bond Short segments of amino acid that join two units of
 Electrostatic Bond (Ionic Bond, Salt Bridges) secondary structures
 van der Waals forces. Example: Beta turn.
Secondary Structures of Proteins include:
Beta Turn
 Alpha Helix
 Beta Pleated Sheet
 Loops
 Bends
 Turns

ALPHA HELIXQ
 Alpha helix is the most common and stable secondary
Structure
 Right-handed Spiral Structure
 Structure stabilized primarily by intrachain hydrogen
bond between carbonyl oxygen of 1st and amide
nitrogen of 4th amino acid
 Each turn formed by 3.6 Amino Acyl residues
 Distance of 1 turn of alpha helix (called Pitch) is
0.54 nm
 Proline can only be stably accommodated within
 # '*
 Examples of Proteins whose major secondary
structure is Alpha Helix: Fig. 2.2: Structure of beta turn

 Hemoglobin  Involves four aminoacyl residues


 Myoglobin.  First aminoacyl residue is hydrogen bonded to the
fourth resulting in a tight 180° turn
BETA-PLEATED SHEETQ  Proline and Glycine #   9 

 The second most common (hence‚ ‘beta’) recognisable Loops


regular secondary structure in proteins Unlike bends and turns, loops are long segments of amino
 Polypeptide chain is almost fully extended acid that join two secondary structures.
 +/ ! 77    Amino acids have different propensity for forming alpha helix,
Beta sheets and Beta turns
 In contrast to intrachain hydrogen bond in alpha The frequency of occurrence of certain amino acid residues
helix, here it is interchain hydrogen bond between determines the secondary structure formed
Carbonyl Oxygen and Amide nitrogen of two Alpha helix
adjacent chains  Residues like Alanine, Glutamate, Methionine and Leucine tend
to be present in the alpha helix
 Adjacent strands in a sheet can run in the same  Most abundant amino acid is Methionine (relative frequency is
direction (parallel) or opposite direction (antiparallel). 1.47) followed by Glutamate (1.44)
 Example of proteins whose major secondary  Amino acid least present in alpha helix is Proline.
structure is: Contd...
Proteins 59

Contd...
Oxidoreductase with Rossmann FoldQ NBEPattern
Q
Amino acids that do not favour alpha helix are :  Lactate dehydrogenase
 
   
  
 valine, threonine  Alcohol dehydrogenase
and isoleucine disrupt the stability  Glyceraldehyde-3-phosphate dehydrogenase
 Other amino acids that disrupt the stability are Serine, Aspartate  Malate dehydrogenase
and Asparagine
 Quinone oxidoreductase
 Proline also disrupts the stability of alpha helix  6-phosphogluconate dehydrogenase
 Glycine does not favor alpha helix formation.  D-glycerate dehydrogenase
Betasheet  Formate dehydrogenase
 Valine and isoleucine tend to be present in beta strands  %&'*/$ $   $  0
 Most abundant amino acid in beta sheet is Valine
 Amino acid least present in beta sheet is Proline
Turns QUATERNARY STRUCTURE
 Most abundant amino acid in turns is Proline (1.91) followed by
Glycine (1.64). If more than one polypeptide aggregate to form one
functional protein, the spatial relationship between
the polypeptide subunits is referred to as Quaternary
SUPER SECONDARY STRUCTURE (MOTIFS) structure.
Secondary structural elements join to form Super Bonds involved in Tertiary and Quaternary Structures
Secondary Structures. Examples are: are primarily noncovalent bonds.
 Beta-alpha-beta motif  Hydrophobic Interaction
 Greek key motif  Hydrogen Bond
 Beta meander motif  Electrostatic Bond
 Beta barrel.  van der Waals Forces.
DNA-binding Motifs are examples of Super Secondary Insulin has two polypeptide chains but it does not have
Structure. They are: Quaternary Structure.
 Helix-Turn-Helix Motif In Quaternary structure, the bond involved is primarily noncovalent
 022 ' 5$55 
   

 $   
 Leucine zipper motif   
    8#  : 0 
 #  2    *
 ;  =# polypeptide chains, it does not have quaternary structure.
Points to Ponder—DNA-binding Motifs
   
  
STRUCTURE OF INSULIN
  
   

  ! 
" 
 
#$
  +V= *  #=
 DNA-binding Motif with leucine residues at every seventh position  This was done by Banting and Best
is Leucine Zipper.
 Banting along with the director of the institute John
Macleod received Nobel Prize for the work
TERTIARY STRUCTURE  +  &  =  & 
done
The entire three-dimensional conformation of a  Mr Frederick Sanger was the man behind this work
polypeptide is referred to as tertiary structure.
 He used Sanger’s Reagent for this
Domain  He won Nobel Prize for his Work
?=     #  @    +        / Y = 
perform a particular chemical or physical task, such as DNA Technology.
binding of a substrate.
Primary Structure of Insulin
Rossmann Fold  It consists of two polypeptide chains
 It is a domain seen in the family of oxidoreductases  Number of Amino Acids is 51
 They share a common N terminal NAD(P)+ binding  A chain with 21 Amino Acids
region called Rossmann fold.  B chain-30 Amino Acids.
60Self Assessment and Review of Biochemistry

D
    STUDY OF PROTEIN STRUCTURE
Two interchain Disulphide Bonds:
 7th Amino Acid in A chain to 7th Amino Acid in B Study of Primary Structure/Sequencing of
chain Proteins
 20th Amino Acid in A chain to 19th Amino Acid in Methods of protein sequencing
B chain  End group analysis
] ^    _`   Mass spectrometry
 6th Amino Acid in A chain to 11th Amino Acid in A  Molecular biology techniques.
chain itself.
Species variation in insulin
End Group Analysis
^   #}
=  
= ?= ? 
 Restricted to 8, 9, 10 in A chain and C terminal Amino
in a polypeptide chain is called end-group analysis.
Acid of B chain
       
 Porcine and Human Insulin vary only in the terminal
Amino Acid of B chain.  Sanger’s Technique using Sanger’s reagent (1, Fluoro
2, 4 Dinitro Benzene, FDNB)Q
DENATURATION OF PROTEINS  Edman’s Degradation Technique using Edman’s
reagent (Phenyl Isothiocyanate).
Nonspecific alteration in secondary, tertiary and        
quaternary structures of protein molecule when treated  Using Carboxypeptidase A and B.
with a denaturing agent. Sanger’s technique
Denaturing Agents are:  +&   =  
 Mild heating of protein
 Treating with Urea  Sanger’s Reagent is 1, Fluoro 2, 4 Dinitrobenzene 
 Salicylates  Sanger’s Reagent derivatizes the amino terminal
 X-ray residues
 UV rays  +  /=
 High pressure Insulin by Fredrick Sanger. He got Nobel prize in 1958
 Vigorous shaking.  Only dipeptides or tripeptides can be sequenced.
Two types of denaturation Edman’s technique
1. Reversible Denaturation: Denatured proteins are some-  By using Edman’s Reagent (Phenyl Isothiocyanate)
times renatured when physical agent is removed.  Phenyl Isothiocyanate derivatizes the amino terminal
2. Irreversible Denaturation: Denatured proteins are not of Polypeptide
renatured when physical agent is removed.  Edman’s Technique can sequence many residues
(5–30 residues) of a single polypeptide sample unlike
E.g. Albumin heated is irreversibly denatured called
Sanger’s Technique.
Heat Coagulation.
Steps of sequencing the proteins
Characteristic features of denaturation areQ:
 To determine complete sequence of a large
 Loss of Biological Activity
/      !  = 
 Primary Structure (i.e. the peptide bond) is not altered peptides. Hydrolysis of large polypeptide by using:
 Loss of Secondary and Tertiary Structures  Trypsin: Cleave carboxyl side basic amino acids
 Loss of Folding like Lysine and Arginine
 They assume a Random Coil Structure.  Chymotrypsin: Cleave carboxyl group of amino
Concept acids, aromatic amino acids and other bulky
 Everything is lost in denaturation except the primary structure (i.e. nonpolar amino acids like Phe, Trp, Tyr, Leu, Met
the peptide bond). Remember the peptide bond is a covalent bond,  Cyanogen Bromide attacks carboxyl side of
which is the strongest bond. methionine residue.
Proteins 61

  !/ /Y! 


HPLC
 Short peptides are sequenced by Edman’s sequencing
in automated sequenator.

Mass Spectrometry
 Today mass spectrometer has emerged as the method
#  #   
 The principle used to identify protein based on mass
(precisely saying on mass/charge ratio)
 The molecular mass of each amino acid is unique, the
sequence of the peptide can be reconstructed from
the masses of its fragments.
The exceptions are mol mass of:
 Leucine and isoleucine
 Glutamine and lysine. Fig. 2.3: Mass spectrometer
Analyte has to be converted to vapor phase by using  Time of flight (TOF) mass spectrometer: Following
various techniques. vaporization of the sample in the presence of a
Methods for dispersion of analyte into vapor phase.           /  
accelerate the ions toward the detector at the end
 Heating in a vacuum: But proteins and oligonucleotides
#= #    
are destroyed by heat
the velocity to which they are accelerated—and
 Electrospray Ionization hence the time required to reach the detector—is
 Matrix-Assisted Laser Desorption and Ionization  !/ = +
(MALDI)Q # =
#
‚+]ƒ=  =  
 Fast Atom Bombardment (FAB). }=     # =
is required.
Types of Mass Spectrometers
 +=
#
 =   =   
          !
determine the large masses of complete proteins
 Quadrupole mass spectrometers: In a simple, single ( > 4KDa)
quadrupole mass spectrometer a sample is placed  Tandem mass spectrometry: Two mass spectrometers
under vacuum and allowed to vaporize in the are linked in series. For this reason, such tandem
presence of a proton donor to impart a positive instruments are often referred to as MS–MS.
 ?            Advantages of Mass Spectrometers
&   !&/     Method of choice: Protein determination
=   &  =   
 Superior sensitivity, speed and versatility
to their original direction of flight. The current
 _     =   €= †
powering the electromagnet is gradually increased
reaction and DNA-derived Protein sequence
  #   @  /€
 Can be used for other biomolecules like oligonucleo-
 =   #For
tide, carbohydrates as mass and charge are common
ions of identical net charge, the force required to
properties of all this.
bend their path to the same extent is proportionate
to their mass. Molecular Biology Revolutionized the
 Quadrupole mass spectrometers generally are Determination of Primary Structure
used to determine the masses of molecules of  The basic principle of Edman’s chemistry is employed
4000 Da or less. to sequence a small portion of protein
62Self Assessment and Review of Biochemistry

 Using the data from genetic code, remaining sequence Contd...


is determined. Property used
Study of secondary structure Type of Supporting to separate the
electrophoresis media used proteins
 Circular Dichroism
Cellulose acetate Cellulose acetate
 Optical Rotatory Dispersion Chromatography. electrophoresis membrane
Study of tertiary structures Polyacrylamide gel Polymer of Based on charge
 X-ray Crystallography electrophoresis acrylamide and molecular weight
 NMR Spectroscopy (PAGE) (size)
 UV Light Spectroscopy (Most rapid method) Sodium dodecyl Sodium dodecyl Based on molecular
sulfate (SDS)— sulfate and weight (size)
 Fluorescence Spectroscopy PAGE polyacrylamide
 Molecular Modeling. SDS imparts equal
negative charge so
SEPARATORY TECHNIQUES OF PROTEINS that it masks the
inherent charge of
"#   $ % &     ! the protein. Now
 Salt fractionation proteins separate
based on size only
 Ultracentrifugation
 Electrophoresis Capillary Separation done in Based on charge
electrophoresis a capillary tube
 Chromatography.
Isoelectric focusing Supporting media Based on isoelectric
Salt Fractionation (Salting Out) with pH gradient pH
Principle
The solubility of proteins is generally lowered at high
Two-dimensional Electrophoresis
        "   ‡   † +
addition of a salt in the right amount can selectively In one direction SDS-PAGE and in the other direction,
precipitate some proteins, while others remain in Isoelectric focusing. So, separation is based on both
solution. Ammonium sulfate ((NH4)2SO4) is often used molecular weight (size) and isoelectric pH.
for this purpose because of its high solubility in water.
Ultracentrifugation
Principle
Method to separate the protein based on the mass,
density, to an extent to the shape also.
Electrophoresis
  #         
electrophoresis.
Concept of Electrophoresis
 Sample is applied usually at the cathode end and the analyte
moves towards the anode
 So negatively charged particles move faster
 If a mixture of amino acids is separated, then the negatively
charged amino acid moves faster.

Classified based on the supporting media used to


separate the analyte of interest.
Fig. 2.4: Isoelectric focusing
Property used
Type of Supporting to separate the
electrophoresis media used proteins
High Pressure Liquid Chromatography (HPLC)
Agarose gel Agarose gel Based on charge
electrophoresis Versatile technique among the Column methods of
Contd... Chromatography.
Proteins 63

Concept of HPLC Various Chromatographic Techniques


 
  5
  HPLC, or high- 
 %  $  '  (
performance liquid chromatography. HPLC makes use of high- used for separation
pressure pumps that speed the movement of the protein molecules
down the column, as well as higher-quality chromatographic materials Stationary Property used
 
     
  
   5 ;  < 0 By Chromatography phase used for separation
reducing the transit time on the column, HPLC can limit diffusional
 Paper  Water held on a  Based on the
spreading of protein bands and thus greatly improve resolution.
chromatography solid support of polarity
5 5=  Least Polar
Advantages Cellulose) moves faster
 Separation can be based on different property
 Thin layer  Silica gel  Based on
   = ‚ @ / =  /  chromatography (Kieselguhr) spread Polarity
exchange columns, size exclusion, etc. used on a glass plate or  Least Polar
 ` Y  ˆ+ += a plastic sheet or moves faster
aluminium sheet.
 Reproducible Results
 Ion exchange  Column of Ion  Based on
 Hence High Performance (So also called High
chromatography exchange resins Charge-Charge
Performance Liquid Chromatography)  Anion exchange or Interaction
 For separating Hemoglobin fraction HPLCQ is used. Cation exchange
resins
Densitometry  Size exclusion  Column of porous  Based on
Method to quantitate the separated protein. chromatography beads molecular
Other names: weight (size)
Chromatography  Molecular sieve  Particles
#   #   "   chromatography emerge in the
     descending
distribution of analyte between stationary and mobile
chromatography order of Stokes
phase. RadiusQ
 Gel permeation
Concept of Various Chromatographic Techniques chromatography
Thin Layer/Paper Chromatography
   Column of resins  Based on
Stationary phase is water held on support media and mobile phase is
chromatographyQ   5

 5


a mixture of nonpolar solvents. The sample is applied on the support
ligands used ligand-binding
media. As the nonpolar solvent ascend on the stationary phase, the
behavior or
nonpolar component of the analyte move along the mobile phase,
biological
but polar component will remain in the stationary phase.
activity
Size Exclusion Chromatography
 Hydrophobic  Based on
The column matrix is a cross-linked polymer with pores of selected
interaction hydrophobic
size. Larger proteins migrate faster than smaller ones, because they
chromatography interaction.
are too large to enter the pores in the beads and hence, take a more
direct route through the column. The smaller proteins enter the pores  Absorption  Based on
and are slowed by their more labyrinthine path through the column. chromatography absorption
So, separation is based on molecular weight or size of the particle. property

   
The beads in the column have a covalently attached chemical group. Compare the Different PAGEs
5  $ 5 
 

5   PAGE
beads in the column, and its migration will be retarded as a result. So  Protein is separated based on molecular mass or molecular weight/
this is based on the biological activity. size and charge.
Ion Exchange Chromatography SDS-PAGE
This can be Anion exchange or Cation exchange Chromatography.  SDS imparts equally negative charge so that it masks the inherent
In cation-exchange chromatography, the solid matrix has negatively charge of the Protein
charged groups. In the mobile phase, proteins with a net positive
charge migrate through the matrix more slowly than those with a  Now Proteins separate based on molecular weight (size) only.
net negative charge, because the migration of the former is retarded SDS-PAGE in conjunction with 2 mercaptoethanol or dithiothreitol
more by interaction with the stationary phase. In anion exchange
 ? #$
 #   
chromatography it is vice versa. So, separation is based on the
charge of the analyte.  So, separate the components of multimeric proteins.
64Self Assessment and Review of Biochemistry

Fig. 2.5: Size exclusion chromatography Fig. 2.7:$


  5$

Contd...
 The amino acid which moves fastest in a Thin layer Chromatography/
Paper Chromatography is Isoleucine
 @@JUX  
Y
*!
5  
break component polypeptides of multimeric proteins
 In electrophoresis, negatively charged amino acids and proteins
move faster
 In paper and thin layer chromatography, nonpolar amino acids
move faster.

PRECIPITATION REACTIONS OF PROTEINS


Po #    & = 
around them to produce a shell of hydration. This makes
the protein soluble in water. Any factor that neutralizes
the charge of protein or remove the shell of hydration
will cause precipitation of proteins.
Methods to precipitate protein by neutralizing the charge are
 Precipitation by heavy metallic salt
 Precipitation by acids.
Fig. 2.6: Ion exchange chromatography
Methods to precipitate protein by removing the shell of hydration
 Precipitation by neutral salts
Remember  Precipitation by organic solvents
 !  5

8  5

" $8  5$
 Most rapid method of separation of Proteins is Capillary  Precipitation by heavy metallic salt
Electrophoresis
 Method to quantitate the separated protein is Densitometry  Heavy metals like Hg, Zn, Pb, etc. provide
 Hydrophobic amino acid moves fastest along the stationary phase positively charged ions and neutralise the
of Thin Layer Chromatography or Paper Chromatography. negative charge of the protein
Contd...  Thus precipitate the protein.
Proteins 65

 The reagents used are Mercuric nitrate, Zinc  But depends on Tyrosine and Tryptophan residues
sulphate, Lead acetate, Ferric cholride. in protein.
 Precipitation by acids
Spectrophotometric Estimation
 Acids bring the pH of the medium to isoelectric
pH, precipitability is maximum at isoelectric pH.  ! Based on absorbtion of uv light at 280 nm by
This is because at isoelectric pH, proteins carry aromatic amino acid.
no net charge; hence no shell of hydration
Advantages and Disadvantages
 The reagents used are Phsophotungstic acid,
 Most accurate, simple and highly sensitive
Sulphosalicylic acid, Phosphomolybdic acid,
 But instrument is costly.
Trichloroacetic acid.
 Precipitation by neutral salts
Radial Immunodiffusion
 Concentrated salt solution removes the shell of [Mancini’sTechnique]QNBE Pattern
hydration
 ! On agar gel with specific antibody, wells
 Reagents used are ammonium sulfate. This is containing antigen are made. Antigen moves radially
called salting out.
and a white ring of precipitate is obtained. The diameter
 Precipitation by organic solvents of precipitation ring will be proportional to the log of
 Organic solvents reduce the dielectric constant antigen concentration.
of water and decreases the water available for
protein, hence it is precipitated Advantages
 The reagents used are ether, alcohol, acetone, etc.  Simple, sensitive technique
 Š        /   
METHODS OF QUANTITATION OF
TOTAL PROTEINS Bradford Assay
 ! Dye-binding assay of protein using Coomassie
Kjeldahl’s Procedure Brilliant Blue. The change in color on dye binding is
 ! The nitrogen present in the protein is reduced to assayed colorimetrically or spectrophotometrically.
ammonia, which is absorbed in acid medium and estimated.
Light-scattering Techniques
Advantages and Disadvantages  ! `  = = # /
      = €= / antigen antibody complexes.
days to get the result. ) % !
1. Nephelometry:Š  ‹Œ= 
Biuret Method
2. Turbidimetry:Š  Ž’Œ= 
 ! Cupric ions chelate with peptide bonds of
protein in alkaline medium to produce violet color. The Advantages and Disadvantages
intensity of color is used to quantitate the protein.  Rapid method
Advantages and Disadvantages  Suitable for automated methods
 Simple method and most widely used  Instruments and reagents are costly.
 But sensitivity is less.
RIA and ELISA
Lowry’s Method
Advantages
 ! Based on reduction of Folin-Ciocalteau phenol
reagent (Phosphomolybdic acid and Phosphotungstic Nanogram or Picogram quantities of proteins can be
acid) Tyrosine or Tryptophan residues of protein. measured.
Remember
Advantages and Disadvantages Bromo Cresol Green (BCG) Method is a method to estimate total
 Very sensitive method albumin AND NOT total protein. Densitometry quantitates the
separated protein in an electrophoretogram.
 Microgram quantity of protein is estimated
66Self Assessment and Review of Biochemistry

IMMUNOCHEMICAL METHODS Types of CollagenQ


 Major collagen present in bone-Type I (90%)
Immunochemical methods forms the basis of a diverse
 Major collagen present in dermis, ligaments and
  #  !         / € &  
tendons-Type I (80%)
immunoassay. In immunoassay an antibody is used as a
reagent to detect the analyte (antigen) of interest.  Major collagen present in cartilage-Type II (40–50%)
 Major collagen present in hypertrophic cartilage-
Advantages of Immunoassays Type X
Exquisite specificity and high affinity of antibodies  Major collagen present in aorta-Type I and Type III
#   = € ==   / /     (20–40% each)
sensitive.  Major collagen present in basement membrane-Type
*   % % &! IV
  !“_"  Major collagen present in skin hemidesmoses-Type
 Immunoelectrophoresis XVII
 Western Blot.  Major collagen present in rhabdomyosarcoma cells-
Type XIX
 + "#
 Most abundant collagen-Type I.
 Single Immunodiffusion: Concentration gradient
established for single reactant. Quantitative technique Structure of Collagen
  Y  ^== _" ‚Y^_ƒ Characteristic features
 
  Concentration gradient is established Glycine-X-Y repeats
for both antigen and antibody. This technique
Every third amino acid residue in collagen is a glycine
is widely used and is known as Ouchterlony
residue.
Technique. 
Alpha chain
  %  ' (
 Polyproline helix of three residues per turn twisted
Technique used to separate and identify various protein in left-handed direction
       = 
 Each polypeptide chain contains 1000 amino acids.
     %  '- (! Electro-
Triple helical structure
phoresis used in second dimension to drive antigen
Three of these alpha chains are then wound into a right-
         #  
handed superhelix.
of interestQ
      %  ' (! Two parallel
lines of wells are punched into the agar. One row is
 &         &
& / ”  
across the gel to cause the antigen and antibody to
move towards each other at a faster rate. A precipitin Fig. 2.8: Triple helix–collagen
line is formed where they meet.

FIBROUS PROTEINS

CollagenQQQ
 The major structural protein found in extracellular
matrix (Connective tissue)
 Most abundant protein in the body
 Present in all the tissues of the body
 Highest concentration in the Skin (74%), followed Fig. 2.9: Covalent cross links and quarter
by Cornea (64%). staggered arrangement of collagen
Proteins 67

‘Quarter Staggered’ arrangementQ  Oxidative Deamination


 Lateral association of the triple helical units  Extracellular event
 Each is displaced longitudinally from its neighbor by  Enzyme: Lysyl Oxidase
slightly less than one-quarter of its length  Cofactor: Copper
 Y #  #     Reaction: Oxidative deamination of Lysyl and
Hydroxy Lysyl residues to form Aldehydes.
Synthesis of Collagen
 Covalent Crosslinks
Can be divided into:
 Covalent Cross links by Aldol Condensation of
 Intracellular events: =ˆ// V/*/ˆ///
 ^   oxidative deamination.
 Procollagen is formed  Provide tensile strength to Collagen.
 Extracellular events
Diseases Associated with CollagenQ
 Tropocollagen is formed.
Gene or
Intracellular EventsQ Type of collagen enzyme Disease
 Cleavage of signal peptide Type I COL1A1 and Osteogenesis Imperfecta
8?\^* Osteoporosis
 Hydroxylation of prolyl residues and some lysyl
Ehlers-Danlos Syndrome
residues (Type VII EDS)
 Glycosylation of some hydroxylysyl residues Type II 8?\*^ Chondrodysplasias
 Formation of intrachain and interchain S–S bonds in Osteoarthritis
extension peptides Type III COL3A1 Ehlers-Danlos syndrome
 Formation of triplehelix. (Type IV EDS)
Type IV COL4A3– Alport syndrome (including
Extracellular EventsQ COL4A6 both autosomal and
 Cleavage of amino and carboxyl terminal propeptides X-linked forms)
 ?=/ #        
  Type V and Type I COL 5A1, Classical EDS
alignment 8?\j*
COL1A1
 Oxidative deamination of amino groups of lysyl and
hydroxylysyl residues to aldehydes. Type III COL3A1 Hypermobile EDS (Type
Tenascin XB III EDS)
 Formation of intra and interchain cross-links via (TNXB)
Š "         
Type VII COL7A1 Epidermolysisbullosa,
dystrophic
Unique Events in Collagen Formation
Type X 8?\^/^ Schmid metaphysial
 Hydroxylation
chondrodysplasia
      =        -
Lysyl hydroxylase Lysyl Ehlers-Danlos syndrome
cellularly hydroxylase (Type VI EDS)
 Enzyme: Prolyl and Lysyl Hydroxylase (Kyphoscoliotic EDS)
 Coenzyme: Vitamin C (Ascorbic Acid) and alpha Scurvy

Ketoglutarate ADAM Procollagen Ehlers-Danlos


Metallopeptidase N-proteinase syndrome (Type VII
 Essential for the three chains of the monomer to with autosomal recessive)
fold into a triple helix at body temperature. Thrombospondin Dermatosparaxis type
 Glycosylation Type I motif
=!@*{
 Intracellular event also called
 Hydroxylysine residues are glycosylated with Procollagen
galactose or glucose N-proteinase

 By type III O-glycosidic linkage. Lysyloxidase Lysyloxidase Menke’s disease


68Self Assessment and Review of Biochemistry

CLINICAL CORRELATION–FIBROUS  Progeroid form of EDS


PROTEINS       `•“?ˆ+–    # 9
Ž
4-galactosyltransferase 7.
Ehlers Danlos Syndrome (EDS) or Cutis Hyper % * %   % "   2$ -
Elastica Different Forms of EDS (Imp for drome
AIPGMEE/DNB) Subtype Defect in
Hypermobility Type III collagen, tenascin X
Classical EDSQ
Classical Types I and V collagen
 Type I EDS (Severe) and Type II EDS (Mild)
Vascular Type III collagen
 They have Classical Clinical features.
Kyphoscoliosis Lysylhydroxylase
   !
Arthrochalasis Type I collagen
 Hyperelasticity of Skin (hence called Rubber Person
Dermatosparaxis ADAM metallopeptidase with
Syndrome)
 5 $5^=!@*{
 Hypermobile joints
 ?" +/”   +/^   Points to ponder–EDS
 Most Common Collagen affected is Type III Collagen.
1$
 "2 '$ "2(
 Most common inheritance is Autosomal dominant.
 Joint hypermobility is more prominent than skin  Most serious is Type IV EDS (Vascular) affecting Type III Collagen.
changes.
* "2 '$ * "2( Alport’s Syndrome (AS)Q (Hereditary Nephritis)
 Most serious EDS  X-linked disorder
 +/^^^   "   +/^”   " 
 Skin changes are more prominent than joint changes.  Clinical features
 Predisposed to sudden death from rupture of large  Hematuria
blood vessels or other hollow organs.
 Sensorineural deafness
Type V EDS
 Conical deformation of the anterior surface of the
 Inherited as an X-linked trait lens (lenticonus).
EDS with Tenascin X Defect seen in The pathognomonic of classic Alports Syndrome lenti-
 Recently included in hypermobile EDS (type III). conus together with hematuria
Tenascin X Electron microscopy reveals abnormality in the structure
 Protein coded by TNXB gene of basement membrane and lamina densa.
 Minor component of connective tissue that appears
  =/#    Achondroplasia
EDS due to enzyme defect  Best known cause of Chondrodysplasia
 Arthrochalasic EDS (type VII EDS) or Dermatosparaxis   ==  #=& =
type  Caused by mutation in a gene that codes for the
 Mutations in procollagen N-proteinase (ADAM  # “&# 
—‚“Y
—ƒ
TS2)
Strickler Syndrome
 Very fragile and sagging skin
 An example of Chondrodysplasia.
 Ocular-scoliotic type of EDS (type VI EDS) or
 Characterized by degeneration of joint cartilage and
Kyphoscoliotic
vitreous body of eyes.
 Mutations in the PLOD 1 gene, which encodes
procollagen-lysine 5-dioxygenase (lysylhydroxy-
lase)
ELASTIN
- Scoliosis  A connective tissue protein that is responsible for
- Ocular fragility properties of extensibility and elastic recoil in tissues.
- Keratoconus (Cone-shaped deformity of the  Present in lung, large arterial blood vessels, and some
cornea) elastic ligaments.
Proteins 69

Oxidative Deamination and Triad of Marfan’s Syndrome


DesmosineQAIIMS Cross Links in Elastin  Skeletal changes
Enzyme-lysyl oxidase  Ectopia lentis
 Lysyl residues of tropoelastin are oxidatively deami-  Aortic aneurysms.
nated to aldehydes Biochemical basis of characteristic clinical features in Marfan
 Condensation of three of these lysine-derived Syndrome
aldehydes with an unmodified lysine to forma Explained by location of Fibrillin 1
tetrafunctional cross-link unique to elastin.    lens, hence ectopialentis
 Periosteum, hence arachnodactyly
Hydroxylation in Elastin  X  aorta, hence aortic aneurysms.
Ghent Criteria
Proline hydroxylated to hydroxyproline by Prolyl
An international Criteria to classify the Marfan Syndrome.
hydroxylase.
Remember
Differences Between Collagen and Elastin Fibrillin-1 gene mutation is also recently found to be associated with
Collagen Elastin  Acromicric dysplasia
Different types of Collagen Only one type exist.  Geleophysic dysplasia.
present
Triple helix Structure No triple helix instead Congenital Contractural Arachnodactyly
random coil conformations  Mutation in Fibrillin 2 located in Chromosome 5
(Gly-X-Y) n repeating structure No (Gly-X-Y) n repeating  Fibrillin-2 may be important in deposition of
structure =  / != 
Presence of hydroxylysine No hydroxylysine  Presence of contractures.
Glycosylation present No Glycosylation
Loeys-Dietz Syndrome (LDS)
Intramolecular aldol cross-links Intramolecular desmosine
cross-links  Mutation in Transforming Growth factor Beta
Presence of extension peptides No extension peptides present
Receptors (TGFBR1 and TGFBR2)
during biosynthesis during biosynthesis  Characterized by aortic aneurysms, cleft palate, and
hypertelorism
Clinical Correlation-Elastin  Similar to Marfan’s syndrome except in the absence
of ocular changes in LDS.
   %    '3(   !
 Supravalvular aortic stenosis '5  KERATIN
2$  (
 Cutis laxa.  Alpha helix coiled coil structure, i.e. two alpha helix
are wind around one another to form a super helix
 They belong to the family of Intermediate Filament.
FIBRILLIN
(IF)
 Glycoprotein  As it is an alpha helix coiled coil, Keratin is rich in
 Š   =  #=  hydrophobic amino acids like Ala, Leu, Met, Val, Phe.
 Š  *  = */   Cross links are formed by disulphide bond
 Incorporated into the insoluble  
  Cysteine is involved in the disulphide bond
 Provide a #  for deposition of elastin  The more the Disulphide bond, the harder the
 
Ž= ˜   Keratin
 
™=   #=   Protein present in the hair nails and outer layer of
early in the development. skin.
Clinical Correlation-Keratin
Clinical Correlation-Fibrillin
Epidermolysis Bullosa Simplex
Marfan Syndrome Mutations in the genes for the major keratins of basal
`/=    ‚  ==Žšƒ#
Ž epithelial cells (keratins 5 and 14).
70Self Assessment and Review of Biochemistry

LAMININ  HSP 60 family of chaperones, sometimes called


chaperonins
 It is a major protein component of Renal Glomerular  They form complex barrel-like structures in which
and Other Basal Laminas an unfolded protein is retained, giving it time and
 Elongated cruciform shape. suitable conditions in which to fold properly
The primary components of the basal lamina are three  For example, mtGroEL chaperonin.
proteins:
1 Laminin.
ROLE OF ENDOPLASMIC RETICULUM (ER)
2 Entactin. IN PROTEIN FOLDING (IMP TOPIC FOR AIIMS
3 Type IV collagen. AND PGI)
Glycosamino Glycans present in the Basal Lamina are:
1 Heparin. The ER functions as the quality control compartment
2 Heparan sulphate. of the cell.
  ' 6 )  7   8( The newly synthesized proteins after entering ER
 Is a glycoprotein containing an RGD (Arginine, =  # &    #    
Glycine and Aspartic Acid) sequence folding enzymes in the lumen of ER.
 It binds to laminin ER Stress Sensors Initiate Unfolded Protein
 ^ = ˜  = #  Responses (UPR)
 Mechanism to sense the levels of misfolded proteins
PROTEIN FOLDING and initiate intracellular signaling mechanisms to
Proteins are conformationally dynamic molecule that compensate for the stress conditions and restore ER
can fold into functionally competent conformation. homeostasis is Unfolded Protein Response (UPR)
Auxillary Proteins assist Protein Folding, they are called  UPR increases the ER folding capacity and prevents
Chaperones. a build up of unproductive and potentially toxic
protein products.
Properties of Chaperone ProteinsQ
Present in a wide range of species from bacteria to humans ERAD (Endoplasmic Reticulum
 Many are so-called Heat Shock Proteins (Hsp) Associated Degradation)
 Are inducible by conditions that cause unfolding of newly  Misfolded or incompletely folded proteins interact
synthesized proteins (e.g. elevated temperature and various
chemicals)
with chaperones, which retain them in the ER and
 They bind to predominantly hydrophobic regions of unfolded
prevent them from being exported to their final
proteins and prevent their aggregation destinations
 They act in part as a quality control or editing mechanism for  The misfolded proteins are usually disposed of by
detecting misfolded or otherwise defective proteins    - :  "
 Most chaperones show associated ATPase activity. '-:"(. Explained later.

Molecular Chaperones are (Very imp topic) PROTEIN DEGRADATION


 HSP 70  Intracellular proteases hydrolyze internal peptide
 HSP 90 bonds
 HSP 40 [Cochaperone]  The resulting peptides are then degraded to amino
 BiP [Immunoglobulin heavy chain binding protein] acids
 Glucose Regulated Protein [GRP-94]  By endopeptidases that cleave internal peptide
 Calreticulin bonds
 Calnexin.  By amino peptidases and carboxypeptidases that
 9$     ! remove amino acids sequentially from the amino-
 Protein Disulphide IsomeraseQ and carboxyl-termini, respectively.
 Peptidyl Prolyl Isomerase.Q  PEST sequences, regions rich in proline (P), glutamate
Chaperonins (E), serine (S), and threonine (T), target some proteins
 The second major class of chaperones for rapid degradation.
Proteins 71

Two Types of Proteins Degradation Steps of ERAD


1. ATP Independent.  A target protein which is misfolded undergoes retro-
2. ATP Dependent. grade transport through the ER (Retrotranslocation
or dislocation) membrane into the cytosol
ATP Independent Degradation
 It is subjected to polyubiquitination
Proteins that undergo ATP independent degradation are:
 Then enters a proteasome, inside which it is degraded
 Extracellular proteins
to small peptides that exit and may have several fates.
 Membrane-association proteins
 Long lived intracellular proteins. Transmembrane proteins that helps in the retrotranslocation of
misfolded membrane and luminal protein in ER to cytosol are:
2! 3$  
 Sec61 (Translocon)
By ATP independent mechanism.
 Degradation in ER protein1 (derlin 1)
For example, Blood glycoprotein.  ERAD E3 ligases

ATP Dependent Degradation  Hrd1


  ^/
Proteins that undergo ATP dependent degradation:
 Regulatory proteins with short half-lives
 Abnormal or misfolded proteins. Ubiquitin
2! In the cytosol by proteasomal complex:  Key molecule in protein degradation
 ATP dependent mechanism  Small protein with 76 Amino Acids
 Requires Ubiquitin.  Highly conserved protein
%     -:" ;   -  ? = #   
Associated Degradation of Proteins] is called Kiss of Death
        =   # ˆ/  #
the target protein hence it is a Pseudopeptide or
     < 

 Minimum of four ubiquitin molecules must be
 == =    
     
&
  
Ubiquitin bind to proteins with PEST [Proline, Glutamic
Acid, Serine and Threonine] sequence in the amino
terminal.

Proteasome
 Ubiquitinated proteins are degraded in Proteasome
 Located in the Cytosol
 Large cylindrical structure composed of 50 sub-units
 This is an ATP dependent process.

Structure of Proteasome
Proteasomes are protein complexes
It is a large cylindrical structure
It is composed of:
 Four rings with a hollow core containing the protease
active sites
 One or two caps or regulatory particles that recognize
Fig. 2.10: Endoplasmic reticulum associated protein degradation the polyubiquinated substrates.
72Self Assessment and Review of Biochemistry

Contd...
Mechanism of
Disease Host pathogenesis
Familial CJD Humans Germ-line mutations
in PrP gene located in
8 */
Sporadic CJD Humans Somatic mutation or
spontaneous conversion of
cellular isoform of the prion
protein (PrPC) into disease-
causing isoform of the prion
protein (PrPSc)
Gerstmann-Sträussler- Humans Germ-line mutations
Scheinker (GSS) in PrP gene located in
disease 8 */
Fatal Familial Humans Germ-line mutation in PRNP
Insomnia (FFI)
Sporadic Familial Humans Somatic mutation or
Insomnia spontaneous conversion of
Fig. 2.11: Structure of proteasome and PrPC into PrPSc
steps of proteasomal degradation
Points to Remember
 The most common prion disorder in humans—Sporadic CJD
Steps of Proteasomal Degradation (sCJD)
 The regulatory particle recognizes the ubiquitinated  The most common etiology of Prion Diseases—Sporadic (85%)
 The second most common etiology of Prion Diseases—Germline
protein which are unfolded by ATPases present in the ! =^/‚^jƒ{
regulatory particles or caps  The Prion Diseases with Noninfectious etiology are:
 Protease active sites in the core of the proteosome – sCJD
– fCJD
€    – Gerstmann-Sträussler-Scheinker (GSS) disease.
 Peptides are released into the cytosol for further – Fatal Familial Insomnia (FFI)
degradation by cytosolic peptidases.
Some Terms to Remember
Clinical Correlation
Proteasome Inhibitor [Bortezomib] PRNP JJ
  
 */0
Used in Multiple Myeloma PrP Human prion-related protein
For Hepatocellular Carcinoma PrPC Cellular isoform of the prion protein






PrPSc Disease causing isoform of the prion protein.
PROTEIN MISFOLDING DISORDERS   


 Prion Diseases Biochemical Basis of Prion Diseases


 Prion Related Protein Diseases. By a Conformational Chain Reaction
One pathologic prion or prion-related protein can serve as
Human Prion Diseases
template for the conformational transformation of many
Mechanism of times its number of PrPc molecules.
Disease Host pathogenesis
Kuru Fore Infection through ritualistic
people cannibalism
Creutzfeldt-Jakob
disease (CJD)
Iatrogenic CJD Humans Infection from prion-
contaminated hGH,
duramater grafts, etc.
Variant CJD Humans Infection from bovine prions

Contd...
Proteins 73

The Prion Related Protein DiseasesQ Congo red staining shows apple-green birefringence
Prion like changes underlie many neurodegenerative under polarized light.
  
      
   
 
    By electron microscopy amyloid is seen to be made
   
 
. up largely of    =
 %  
 with a
Diseases like diameter of approximately 7.5 to 10 nm.
 Alzheimers Disease Chemical Nature of Amyloid
 Parkinson’s Disease  žšŸ# =/=    # 
 Huntington’s Disease  5% of the amyloid material consists of P component
 Fronto Temporal Dementia and other glycoproteins.
 Dementia with Lewy Bodies
Nomenclature of Amyloid Fibrils
 Amyloidosis
 Beta thalassemia. The accepted nomenclature is AX, where:
Prion related Protein diseases and abnormally aggre-  A indicates amyloidosis
gated Proteins      
Abnormally aggregated Common Amyloid Fibril and its Characteristics
Disease protein
Alzheimer’s Disease †* 
:3 $ 
Huntington’s Disease Huntingtin  Seen in primary systemic amyloidosis
Fronto Temporal Dementia Tau Inclusions Pick Bodies TDP-  += ˜?=/
[FTD] 43 inclusions FUS inclusions
 Composed of immunoglobulin light chains (LCs)
Dementia with Lewy Bodies &-synuclein inclusions (Lewy
[DLB] bodies)  Associated with a clonal B cell disorder and may be
associated with myeloma or lymphoma.
Beta-Thalassemias AA amyloid
 Thalassemias are caused by genetic defects that  Seen in secondary amyloidosis
impair the synthesis of one of the polypeptide sub-  +  = ˜  # =/
units of hemoglobin  Is composed of the acute-phase reactant serum
 During the burst of hemoglobin synthesis that amyloid A protein
    / / !=       AA Amyloid is derived by proteolysis from a larger
   '
V=
Š 7   (12,000 daltons) precursor in the serum called
‚?VŠƒ #= '
  &   2:: ' $ > (   % 
incorporation into the hemoglobin multimer synthesized in the liver
 ^   #   #'
=   ?  &     == / # 
subunits aggregate, and the resulting precipitate has diseases.
/* "  ! / /
:? $ 
AMYLOIDOSIS  Seen in localized amyloidosis
 ?9= == #=#ˆ ?=/
Amyloidosis is the term for diseases caused by the  ?9    ! / / #=  = 
extracellular deposition of insoluble polymeric protein
larger transmembrane glycoprotein, called amyloid

 in tissues and organs.
  ?=/9
The term amyloid was coined by the pathologist Rudolf
 ?9    ?7=† 
Virchow.
:- $ 
Amyloid Fibrils  Seen in familial Amyloidoses and Systemic Senile
Physical Nature of Amyloid Amyloidosis Composed of Transthyretin.
X-ray crystallography and infrared spectroscopy  %$ '-(
demonstrate a characteristic cross-9 pleated sheet  Normal serum protein that binds and transports
conformation. thyroxine and retinol.
74Self Assessment and Review of Biochemistry

@ :$   '@ $   - Contd...


 %( Amyloi- Chemically related Clinical Organ
dosis precursor protein syndrome involvement
 A mutant form of TTR (and its fragments) is
deposited. Systemic Senile Amyloidosis
ATTR Transthyretin Senile Cardiac
In systemic senile amyloidosis systemic (wild
 Normal TTR is deposited in the heart of aged type)
individuals. Hereditary Amyloidosis (Familial Amyloidosis) (Heredofamilial
Amyloidosis)
?H $ 
ATTR Transthyretin Familial (mu- Cardiac,
 Seen in Hemodialysis associated Systemic Amyloi- tant) Familial peripheral
dosis amyloidotic and
 Composed of ?H>2-microglobulin, a component of neuropathies autonomic
nerves
MHC class I molecules occurs in individuals with
AA Serum Amyloid A Familial Renal or Any
End-Stage Renal Disease (ESRD) of long duration Protein Mediterranean
 In patients on long-term hemodialysis. fever
A ApoA1 Apolipoprotein A1 Familial Hepatic, renal
Protein misfolding results in Amyloidosis
A ApoAII Apolipoprotein A1 Familial Renal
The proteins that form amyloid fall into two general categories:
 Increased production of normal proteins that have an inherent A Gel Gelsolin Familial
 
$ 55$' 
    0X 5 ‡ Corneas, cranial nerves, renal
– @   $ ;  $  #
     <
  A Fib ‰& Familial Renal
cytokines such as IL-6 and IL-1 that are increased during long A Lys Lysozyme Familial Renal
  <   AA Amyloid ALECT2 Leukocyte ? Renal
– Immunoglobulin light chain synthesized by the plasma cells



*
increased in Monoclonal B lymphocyte proliferation results in
AL Amyloid Localized Amyloidosis
 Mutant proteins that are prone to misfolding are produced and
subsequent aggregation. (NB:-No increased production). Example: Chemically
– Mutant TTR aggregation in Familial amyloidosis. Amyloi- related precursor Clinical Organ
dosis protein syndrome involvement
 Amyloid protein Alzheimer’s Down
Systemic Amyloidosis disease; syndrome
ACys Cystatin C Cerebral CNS,
Amyloi- Chemically related Clinical Organ
amyloid vascular
dosis precursor protein syndrome involvement
angiopathy
Systemic Generalized Amyloidosis APrP Prion protein Spongiform en- CNS
AL Immunoglobulin Primary or Any organs cephalopathies
\8 
<$ myeloma as- AIAPP Islet amyloid Diabetes- Islets of
ˆ$5 sociated or polypeptide associated Langerhans
Immunocyte (amylin) of Pancreas
dyscrasias ACal Calcitonin Medullary Thyroid
with amyloi- carcinoma of
dosis
the thyroid
AH Immunoglobulin Primary or Any organs AANF Atrial natriuretic Age-related Cardiac atria
heavy chain myeloma as- factor Isolated Atrial
sociated or Amyloidosis
Immunocyte
APro Prolactin Endocrinopathy Pituitary
dyscrasias
with amyloi-
dosis Heredofamilial Amyloidosis
AA Serum amyloid A Secondary Renal or Any  Familial mediterranean fever
protein or Reactive  The most common and best studied
Systemic
 An autosomal recessive condition
 *!
 Hemodialysis- Synovial
associated membrane,  The gene for familial Mediterranean fever
bone encodes a protein called pyrin (for its relation
Contd... to fever), which is one of a complex of proteins
Proteins 75

that regulate inflammatory reactions via the


  #  == / /€ 
 This disorder is encountered largely in individuals
of Armenian, Sephardic Jewish, and Arabic
origins
 + =/  = #??
proteins
 Familial Amyloidotic Polyneuropathies
 An autosomal dominant familial disorder
 Characterized by deposition of amyloid made up
of mutant TTRs
 Predominantly in peripheral and autonomic
nerves. Fig. 2.12: Structure of immunoglobulin

IMMUNOGLOBULINS Variable Regions of Immunoglobulin


Consist of the VL and VH domains
Immunoglobulins, are synthesized mainly in plasma cells
They are quite heterogeneous.
(specialized cells of B cell lineage) in response to exposure
to a variety of antigens. Variable regions are comprised of
 1$
 - 
Structure of Immunoglobulin  Hypervariable regions comprise the antigen-
It consists of 2 heavy chains and 2 light chains binding site (Fab)
Light chain and Heavy chain are divided into Constant  Hypervariable regions are also termed comple-
Region towards the Carboxyl end and Variable Region mentarity-determining regions (CDRs).
towards the Amino terminal end.
 -$ 
 - 
 In the Light chain VL and CL
 The surrounding polypeptide regions between
 In the Heavy chain CL and CH1, CH2, CH3.
the hypervariable regions are termed as frame
Hinge Region work regions.
The region between the CH1 and CH2 domains.
Proteolytic Cleavage of Immunoglobulin
 +    #*/  &
Fab arms to move independently, thus helping them to  Papain Digestion
bind to antigenic sites.  Site of Cleavage-Beyond the Disulphide Bond in
the Hinge Region
Heavy Chain Types in Immunoglobulins  Products of Cleavage-2 separate Fab fragments
Based on the heavy chain types Immunoglobulins are and 1 Fc fragment.
! !  
 Pepsin Digestion
 ^“¡
 
 Site of Cleavage-Before the disulphide bond in
 ^?'
 
the hinge region
 IgM μ-Chain
 Products of Cleavage-Bivalent Fab fragments,
 ^_¢
  F(ab)2 and digested Fc fragments.
 ^
  
Fc and Fab fractions of Immunoglobulin
Light Chain Types in Immunoglobulins  Fab (Fraction Antibody)
) $! – Fragment that bind with Antigen
  

  – Located in the variable region of Heavy and Light Chain.
 Fc (Fraction Crystallizable)
 ^  ! ==  ™£™¤  !
– Remaining part of the Immunoglobulin molecule
=*#£ ¤
– Concerned with activation of Complement Cascade.
        = £
76Self Assessment and Review of Biochemistry

CLASSES OF IMMUNOGLOBULIN   " ! !  #    == 
AND ITS CHARACTERISTICS pathway.

Immunoglobulin G (IgG) Immunoglobulin D (IgD)


 Monomer  Found in low levels in serum
 Most Versatile as IgG can perform all the functions  Role in serum uncertain
of Immunoglobulin  Primarily found on B cell surfaces where it functions
 Major Immunoglobulin in the Serum (75–80%) as a receptor for antigen
 Subclasses are IgG1, IgG2, IgG3 and IgG4  Does not bind complement.
 Most abundant IgG Subclass is IgG1 (50%) Immunoglobulin E (IgE)
 Main antibody in secondary immune response  Cytophilic Antibody
 IgG is the only class of Ig that crosses the placenta.  Least common Immunoglobulin in the serum
Transfer is mediated by a receptor on placental cells  Involved in allergic reactions as a consequence of its
 Only IgG Subclass that do not cross the placenta is binding to basophils and mast cells
IgG2  IgE also plays a role in parasitic helminth diseases
 Complement activation is present in the order of  _ * == 
IgG3 > IgG1 > IgG2 > IgG4
Quick Revision Immunoglobulins
 Good For opsonization as Fc receptor present on
 Most Common Immunoglobulin present in the serum-IgG
phagocytic cells
 Least Common Immunoglobulin in the serum-IgE
 IgG1 and IgG3 good for opsonization
 Largest Immunoglobulin-IgM
 ^“™ ^“• @ /#  
 Immunoglobulin which is a pentamer-IgM
Immunoglobulin A (IgA)  Immunoglobulin which is a dimer-Secretory IgA
           ! 
Two types:
 ? IgA by Alternate Pathway
1. Serum IgA—Monomer.
 Immunoglobulin which is present in Secretions-Secretory IgA
2. Secretory IgA —Dimer joined by J Chain.  Shape of an Immunoglobulin monomer-Y Shape
+=  " #=  == /=  Immunoglobulin which crosses placenta-IgG
The most abundant immunoglobulin of body secre-  2U 
 
  5
 2U*
tions such as saliva, tears, colostrum and gastroin-  Immunoglobulin involved in Primary Immune response-IgM
testinal secretions  Immunoglobulin involved in Secondary Immune response-IgG
Normally IgA does not fix complement, unless  Immunoglobulin with J Chain-IgM and IgA
aggregated [? IgA Alternate pathway].  Immunoglobulin with secretory piece-IgA.
2 $     '(  2 $  -
globulin is made in mucosal epithelial cells is added to PLASMA PROTEINS
the IgA as it passes into the secretions.
Function of secretory piece By using the method of Salting out Plasma Proteins can
 Protects it from degradation in the secretions be divided into three fragments:
1. Fibrinogen.
 Ensures, the appropriate tissue localization of SIgA
by anchoring the antibody to mucus lining the 2. Albumin.
epithelial surface. 3. Globulins.
By using the Electrophoresis technique Serum Proteins
Immunoglobulin M (IgM) ! ! +/ ‚#= 
 Pentamer joined by J chain cathode)
 Largest Immunoglobulin 1. Albumin.
 ^^==  = / ™ 'Ž“ 
fetus — '™“ 
 Immunoglobulin involved in primary Immune • 9“ 
response š ¡“ 
Proteins 77

Prealbumin or Transthyretin
 Slightly faster mobility than Albumin fraction in electrophoresis
 Major role in transport of Thyroxine and retinol
 Associated with Familial and Senile Amyloidosis.

Functions of Plasma Proteins


Function of the
plasma protein Plasma protein
Maintain the colloid osmotic Albumin
Pressure of the Plasma.
Nutritional Function Albumin
Buffering Action in the Plasma Albumin
Anti proteases Antichymotrypsin
Fig. 2.13: Electrophoretic patterns of plasma proteins &^-Antitrypsin
=&^ 5 {&*-
Abnormal Electrophoretic Patterns Macroglobulin
 Chronic infection: The gamma globulins are increased Antithrombin
as smooth and wide based band Blood clotting Various coagulation factors,
 Multiple myeloma: Sharp band in the gamma globulins 
called M band due to monoclonal origin of Immu- Hormones Erythropoietin
noglobulins Immune defence Immunoglobulins, complement
 Nephrotic syndrome: Alpha 2 fraction is most promi- proteins, &*-microglobulin
nent as all other small proteins lost through urine 2##<  $ Acute phase response proteins
responses =008
#5'&^
 
 Cirrhosis: Albumin band is thin.
glycoprotein [orosomucoid])
Instead of Serum Electrophoresis if Plasma Electrophoresis Oncofetal &^‰5=‰J{
done. What is the difference in electrophoresis pattern?
     "    Π  
   @
and Plasma Plasma Proteins in Transport FunctionQ
Blood collected without adding anticoagulants is Serum and with
anticoagulants is Plasma Transport protein Compound it binds
Serum = Plasma-Clotting factors Albumin  Bilirubin,
In Plasma Electrophoresis: Fibrinogen with other clotting factors  Free fatty acids
 5  U  8
@

‡  Ions [Ca2+]
Confused with M Band in Multiple Myeloma.  Metals [e.g. Cu2+, Zn2+],
 Metheme
 Steroids
 Hormones
Ceruloplasmin  Cu2+
Corticosteroid-binding  Cortisol
globulin (transcortin)
Haptoglobin  Extracorpuscular hemoglobin
Lipoproteins  Plasma Lipids
Hemopexin  Heme
Retinol-binding protein  Retinol
Sex-hormone-binding  Testosterone
globulin  Estradiol
Thyroid-binding globulin  T4
 T3
Transferrin  Iron

Fig. 2.14: Plasma proteins present in Transthyretin (formerly  T4 and forms a complex with
prealbumin) retinol-binding protein
each fractions of electrophoretogram
78Self Assessment and Review of Biochemistry

Quick Review of Plasma Proteins Biochemical Investigations in


 Most abundant Plasma Protein-Albumin Plasma Cell Disorders
 Least abundant Plasma Protein–Alpha-1 Globulin Serum protein electrophoresis and densitometry
 Plasma Protein with fastest electrophoretic mobility-Prealbumin
 Sharp spike (Church Spire Spike) in the globulin
(Transthyretin) followed by albumin. Plasma Protein with least
electrophoretic mobility-Gamma Globulin region is called a M component '     (
 Most Plasma Proteins are synthesized in the liver except  The minimum concentration of monoclonal antibody
Immunoglobulins by Plasma cells of B Lymphocyte lineage for M Component to be seen is 5 g/L (0.5 g/dL)
 Most Plasma Proteins are Glycoproteins except Albumin, which
 This is a method for quantitative assessment of the
is a Simple protein. Many Plasma Proteins exhibit Polymorphism.
They are: M component
– &^ $5   The amount of M component in the serum is a reliable
– Haptoglobin measure of the tumor burden. This makes the M
– Transferrin component an excellent tumor marker
– Ceruloplasmin  In approximately 1% of patients with myeloma,
– Immunoglobulins.
biclonal or triclonal gammopathy is observed.
M Component detected in other diseases
ACUTE PHASE REACTANTS Lymphoid Neoplasms
 Chronic lymphocytic leukemia
Group of plasma proteins whose concentration increases  Lymphomas of B or T cell origin.
      == /     Nonlymphoid Neoplasms
 Chronic myeloid leukemia
conditions.  Breast cancer
 Colon cancer.
Positive Acute Phase Reactants Nonneoplastic Conditions
Plasma proteins whose concentration increases in  Cirrhosis
   == /         Sarcoidosis
 Parasitic diseases
 C reactive Protein
 Gaucher disease
 Ceruloplasmin  Pyodermagangrenosum.
 Haptoglobulin Autoimmune Conditions
 Fibrinogen  Rheumatoid arthritis
 Myasthenia gravis
 'Ž? “/   Cold agglutinin disease.
 'Ž   Skin Diseases
 Lichen myxedematosus (Papular mucinosis)
Negative Acute Phase Reactants  Necrobiotic xanthogranuloma.
Plasma proteins whose concentration decreases in
 Immunoelectrophoresis
   == /       
 This is a method for qualitative assessment of
 Albumin
Immunoglobulins
 Transthyretin
 The nature of the M component is variable in
 RBP plasma cell disorders
 Transferrin.  Type of immunoglobulin is determined by
immunoelectrophoresis
CLINICAL CORRELATION–PLASMA PROTEINS  IgG myelomas (53%) are more common than IgA
and IgD myelomas.
Plasma cell Disorders  J    L    'L(
Monoclonal Neoplasm arising from common pro-genitors  They are Monoclonal Immunoglobulin light
in the B lymphocyte lineage. Synonyms are: chain excreted in urine
 Monoclonal Gammopathy  Seen in 20–30% of Myeloma.
 Paraproteinemias Bradshaw’s Test: Test to detect urine BJP
 Plasma cell Dyscrasias  Urine layered over a few ml of concHCl—White ring
 Dysproteinemias. of Precipitate.
Proteins 79

Special heating test for urine BJP Diagnostic Criteria of Plasma Cell Disorders
 This test is based on special property of BJP Multiple Myeloma
 M Protein in serum and/or urine
 BJP precipitate when heated between 45–60°C but  Bone Marrow Plasma cells
redissolve when heated > 80°C and < 45°C  Myeloma related organ or tissue impairment (end organ damage,
 50% False Negative Test including bone lesions)

 2 @ 3% % :$ Myeloma Variants


        #! !  $#%&
 New Technique to quantitate Serum light chain.
 Most common plasma cell disorder
 2 :6  % % ':3(  !J Ž%/•\
 It is normal since there is no osteoblastic activity.  :  
 5  
 Ž^/ƒ
 2 ?H  
  NO myeloma related organ or tissue impairment (end organ
damage, including bone lesions)
 Single most powerful predictor of Survival
Smoldering Myeloma (Asymptomatic Myeloma)
 Can substitute for staging:  !J “%/•\
- Levels < 0.004 g/L have a median survival of  :  
 5  
 “^/ƒ
43 months  NO myeloma related organ or tissue impairment (end organ
- Levels > 0.004 g/L have a median survival of damage, including bone lesions)
12 months. Nonsecretory Myeloma
 No M Protein in serum or Urine
Staging System in Multiple Myeloma  :  
 5  
 “^/ƒ
 Durie-Salmon staging system  Myeloma related organ or tissue impairment (end organ damage,
including bone lesions)
 Previously used is not widely used nowadays.
Solitary Plasmacytoma of Bone
 I    2  2$ ; 22R  No M Protein in serum or urine
 Most widely used method of assessing prognosis  Single area of bone destruction due to clonal plasma cells
 Parameters employed in ISS are:  Bone marrow not consistent with Multiple Myeloma
 NO myeloma related organ or tissue impairment (end organ
- ` ™  ‚9™ƒ damage, including bone lesions)
- Albumin (alb).

International Staging Systems OTHER PLASMA CELL DISORDERS


Parameters Stage Median survival, months 5    
 Q
*!Ž%0j' " 3.5 2=*ƒ{ ‘*  A malignancy of lymphoplasmacytoid cells that
*!Ž%0j' Ž%0j II (39%) 44 secreted IgM
*!’%0j‚j0j  It originates from a postgerminal center B cell has
*!“j0j III (33%) *” the characteristics of an IgM-bearing memory B cell
 Lymphadenopathy and Hepatosplenomegaly
RNA- and DNA-based Prognostic  Hyperviscosity syndrome present
Staging Systems  Raynaud’s phenomenon present
    / #Y}?` Š  Š/=  No bone lesion, renal involvement. Or hypercalcemia
 Comparative genomic hybridization for DNA based  Sia test Positive
staging system.  Coombs’ test Positive.
POEMS SyndromeQ
 

 

 
 J   
  
   #  
$   Polyneuropathy
overall survival than those secreting kappa light chains.  Organomegaly [Hepatomegaly Lymphadenopathy,
 IgM Myeloma has the greatest tendency for Hyperviscosity. Splenomegaly]
 Among IgG myelomas, it is the IgG3 subclass that has the highest
tendency to hyperviscosity and cold agglutination.  Endocrinopathy
 High labeling index and high levels of lactate dehydrogenase are  Multiple myeloma
also associated with poor prognosis.
 Skin changes.
80Self Assessment and Review of Biochemistry

Heavy Chain Disease  The affected individual lacks a countercheck to


Gamma Heavy Chain Disease (Franklin’s Disease) proteolytic damage of the lung by proteases such
 Lymphadenopathy in Waldeyer’s Ring as elastase.
 Palatal Oedema characteristic. 
#X  –&^—2
#X  &^85—
Proteolysis of Lung
Alpha Heavy Chain Disease (Seligmann’s Disease)

#X  –
  &^—
#X  —J$  
 Most common heavy chain disease \—X5$ 
 Closely related to Mediterranean Lymphoma
Treatment
 ^   # =  #=   &
Truncated Alpha Chain  ^  !  =    # 'Ž
/ 
(augmentation therapy)
 No sharp peak on electrophoresis
 Gene therapy.
 Mu Heavy Chain Disease
Smokers have more chance for developing Emphysema
 The secretion of isolated mu heavy chains.
< Methionine   %j5 &^ $5  ## 
   2   " ' 2 "( in its binding to proteases.
 An infectious pathogen–associated human lymphoma  Smoking oxidizes this methionine to methionine sulfoxide and
that has association with Campylobacter jejuni thus in activates it.
     ' 
  
   &^ $5   
 ?" *= Š= ^   neutralize proteases.
 Excessive Plasma cell differentiation-producing  This is particularly devastating in patients (e.g. PiZZ phenotype)
truncated alpha heavy chain lacking light chain and   $ # # &^ $5 0
1st constant domain.  Hence smoking results in increased proteolytic destruction of lung
tissue, accelerating the development of emphysema.
'*   $'*  & ;
 <WY:    &  #=/  'Ž¦ - Cirrhosis of Liver
rypsin <W : $  $    !
 += ˜ =  ‚§žŒŸƒ#'Ž#  #  “?“??“=   'Ž?+ ‚  =
human plasma. It is synthesized by hepatocytes and unknown)
macrophages  Glu to Lys substitution results in PiZZ Phenotype
 The principal serine protease inhibitor (serpin, or Pi)  PiZZ accumulate in the cisternae of Endoplasmic
of human plasma Reticulum and aggregates of Liver
 It inhibits trypsin, elastase, and certain other pro-  Results in hepatitis leading to Cirrhosis.
    / 'Ž?  
 The major genotype is MM, and its phenotypic GLYCOPROTEINS
product is PiM.
Proteins that contain oligosaccharide chains (glycans)
)     )% <W : $ '<W : Y covalently attached to their polypeptide backbones.
  ( " $! Carbohydrate content is less than 5%.
1. Emphysema
2. Cirrhosis of Liver. Biologically Important GlycoproteinsQ
 Plasma proteins (except albumin)
Emphysema  Blood group substances
 Five percent of emphysema is associated with 'Ž  Hormones (HCG, TSH, LH.FSHQ)
? /   /  Enzyme: Alkaline Phosphatase
 This occurs mainly in subjects with the ZZ genotype,  Structural Protein: Collagen
who synthesize PiZ, and also in PiSZ heterozygotes  Transport Proteins: Ceruloplasmin, Transferrin.
 Both of whom secrete considerably less protein than Eight Sugars that predominate in Human Glycoproteins
PiMM individuals are:
 Polymorphonuclear white blood cells increase in the  Galactose
lung (e.g. during pneumonia)  Glucose
Proteins 81

 Mannose
 N-Acetylneuraminic acid
 Fucose
 N-Acetylgalactosamine
 N-Acetylglucosamine
 Xylose.
Glycation vs Glycosylation
 Glycation: Nonenzymic attachment of sugars to amino group of
proteins.
 Glycosylation: Enzymic attachment of sugars to protein. Fig. 2.15: GPI anchored glycoprotein

Major Classes of Glycoproteins GPI-Linked ProteinsQ are:


 O linked Glycoprotein  Acetylcholinesterase (red cell membrane)
 N linked Glycoprotein
 Alkaline phosphatase (intestinal, placental)
 GPI linked Glycoprotein
 Decay-accelerating factor (red cell membrane)
 5’-Nucleotidase (T lymphocytes, other cells).
O-linked Glycoprotein
O-glycosidic linkage between hydroxyl side chain of CLINICAL CORRELATION OF
serine or threonine and N –acetyl galactosamine is GLYCOPROTEINS
present.
Diseases associated with biosynthesis of Glycoprotein.
Main Features of O-Glycosylation
 ]
“/ /       /     >@  !   ; $>;& 
Ser and Thr residues     "    !/ #  “
 
 Sugar added in a stepwise manner GDP-fucose transporter
 Subcompartments of the Golgi apparatus.  The absence of fucosylated ligands for selectins leads
to a marked decrease in neutrophil rolling
 Dolichol-P-P-oligosaccharide is not involved.
 Life-threatening, recurrent bacterial infections and
N-linked Glycoprotein also psychomotor and mental retardation
N-glycosidic linkage between amide nitrogen of asparagine  Oral fucose is the treatment.
and N–acetylglucosamine is present. Hereditary Erythroblastic Multinuclearity with
 @  Y+$ $  \ ^ !! > _ $`{\%&
 Oligosaccharide Glc3 Man9 (GlcNAc)2 is transferred Congenital Dyserythropoietic Anemia Type II
en bloc from dolichol-P-P-oligosaccharide  ? =      7/='=  ^^
 Oligosaccharide: protein transferase, is inhibited by involved in the biosynthesis of N-glycans
tunicamycin     / "   == 
 + #   ? 
Paroxysmal Nocturnal Hemoglobinuria (PNH)
 Occur cotranslationally in the endoplasmic reticulum.  Acquired mild anemia characterized by the presence
of hemoglobin in urine due to hemolysis of red cells,
Glycosylphosphatidylinositol-anchored particularly during sleep
(GPI-anchored, or GPI-linked) GlycoproteinsQ  1 $  $     1
Third major class of Glycoproteins.  This is due to a slight drop in plasma pH during
They link several proteins to Plasma membrane. sleep, which increases susceptibility to lysis by the
Linked to the carboxyl terminal amino acid of a complement system
protein via a phosphoryl ethanolamine moiety joined to  A test done to diagnose PNH is Hams Test
an oligosaccharide (glycan), which in turn is linked via  A treatment option is Monoclonal Antibody to C5
glucosamine to phosphatidylinositol (PI). (a terminal component of Complement System).
82Self Assessment and Review of Biochemistry

The Basic Defect in PNH  At least 15 distinct disorders


 Isoelectric focusing of transferrin is a useful
biochemical test for assisting in the diagnosis
 ] =  !#   = 
of CDGIa.

ADVANCED GLYCATION
END-PRODUCTS (AGES)
 The end-products of glycation reactions are termed
advanced glycation end-products (AGEs)
 ¨          = 
 #= Š " 
 These can further re-arrange by the Amadori
rearrangement to ketoamines
 The overall series of reactions is known as the
Maillard reaction.
Medical Importance of AGEs
 Aging
Congenital Muscular Dystrophies (CMDs)  Atherogenesis
They are:  Microvascular and macrovascular damage in diabetes
 Walker-Warburg syndrome mellitus.
 Muscle-eye-brain disease Aminoguanidine
 Fukuyama CMD  An inhibitor of the formation of AGEs
Defects in the synthesis of glycans in the protein  Reduce the complication in Diabetes Mellitus.
'
// ‚'
_“ƒ Glycoproteins Important in Fertilization
Rheumatoid Arthritis  Zonapellucida ZP contains the glycoproteins ZP-3
 Associated with an alteration in the glycosylation of  ZP3, an O-linked glycoprotein
circulating immunoglobulin G (IgG) molecules  ?

  
    J% 
 5

$  5'
galactosyl transferase, and induces the acrosomal reaction
 They lack galactose in their Fc regions and terminate
 Maybe possible to inhibit fertilization by developing drugs
in GlcNAc or antibodies that interfere with the normal functions of ZP3
 Mannose-Binding Protein bind agalactosyl IgG (Contraceptives)
molecules
 Leads to activation of the complement system ‘Sugar Code of Life’
        ==   / !  Certain oligosaccharide chains encode biologic informa-
membranes of joints.
tion, e.g. mannose 6-phosphate residues target newly
I Cell Disease synthe-sized lysosomal enzymes to that organelle.
Q
 Mutation in the gene for GlcNAc Phosphotransferase
 Lack of normal transfer of GlcNAc 1-P to mannose PROTEIN SORTING (IMPORTANT FOR
residue of enzymes destined to Lysosomes AIIMS AND PGI)
 Lack of Mannose 6 Phosphate in the enzyme result Proteins must travel from polyribosomes, where they
in defective Protein targeting to Lysosomes.  / 7  = / "       
Congenital Disorders of Glycosylation (CDGs) perform their particular functions. This process is called
 Autosomal recessive disorders ^   + @ # =/ Š .
 Multisystem disorders Remember
 Affect the central nervous system, resulting in Golgi Apparatus is involved in the Glycosylation and Sorting of
Proteins.
psychomotor retardation
Proteins 83

The sorting decision Signal Peptide Hypothesis


 The major decision is at the site of synthesis  The signal hypothesis was proposed by Blobel and
 The second by the Signal sequence in the protein. Sabatini
 To explain the distinction between free and
Site of Synthesis
membrane-bound polyribosomes
Two Sorting branches based on the site of synthesis
 It is proposed that proteins synthesized on membrane-
 Cytosolic Branch bound polyribosomes contained an N terminal
 Rough Endoplasmic Reticulum branch. peptide extension (signal peptide) at their amino
Cytosolic Branch (Free Polyribosome) =  &   =      
membranes of ER, and facilitates the protein transfer
Proteins synthesized on free polyribosomes lack this
particular signal peptide and are delivered to organelle. into the ER lumen
Š Š        From ER lumen they are further sorted to various
 Mitochondria membranes and lysosomes
 Nuclei  Certain ER membrane proteins are transferred
 Peroxisomes directly into the membrane of ER, without reaching
the lumen.
 Cytosol.
2  2&  "     2 Z  Properties of Signal PeptidesQ
Target sequence Organelle targeted
 Located at the amino terminal
 Contain approximately 12–35 amino acids
N terminal Signal Peptide Sequence ER
 Methionine is usually the amino terminal amino acid
Carboxy terminal KDEL Sequence (Lys- Lumen of ER
Asp-Glu-Leu)  Contain a central cluster of hydrophobic amino acids
HDEL Sequence (His-Asp-Glu-Leu)  Contain at least one positively charged amino acid
Diacidic Sequence (Asp-X-Glu) Golgi Membranes near their amino terminal.
Amino terminal Sequence or Matrix Mitochondrial Matrix
Targeting Sequences
ENDOPLASMIC RETICULUM
Nuclear Localization Sequence (NLS) Nucleus TRANSLOCATION
Peroxisomal Matrix targeting Sequence Peroxisome
(PTS) The transfer of protein across ER membrane into the
Mannose 6 Phosphate Lysosome lumen can be of two types:

Cotranslational Translocation
Rough Endoplasmic Reticulum (RER) Branch into the Lumen of ER
Proteins synthesized on membrane-bound polyribosomes This transfer occur when the translation is ongoing. These
contain a signal peptide  =  =  proteins are kept in unfolded state prior to the entry in to
to the membrane of the ER called Signal Peptide conducting channel. The pathway involves a number of
Hypothesis (explained later).
specialized proteins and proceeds in 5 steps.
All the proteins synthesized in membrane bound
Polyribosome (RER) destined for various membranes Specialized proteins Involved in Endoplasmic Reticulum
Proteins synthesized in the free ribosomes are targeted Translocation N-terminal signal peptide
to the organelles like:  Polyribosomes
 Membrane of Endoplasmic Reticulum [ER]  SRP, signal recognition particle (Six proteins associated with an
RNA molecule)
 Membrane of Golgi apparatus [GA]
 SRP-R, signal recognition particle receptor
 Plasma membrane [PM]  TransloconQ (Sec 61 complex) consist of three membrane
 Lysosome proteins, the sec-61 complex that forms the protein
conducting channel in the ER.
Remember
 Signal peptidase
An exception to proteins synthesized in RER branch is to membranes
is proteins destined to Lysosomes, is synthesized in RER
Contd...
84Self Assessment and Review of Biochemistry

Contd...

 Associated proteins (e.g. TRAM and TRAP)


 TRAM is Translocating Chain Associated Membrane Protein.
TRAM accelerates the translocation of certain proteins
 TRAP, Translocon associated Protein Complex. The function of
TRAP is not clear.

Fig. 2.17: Post-translational translocation of proteins


Steps involved in the post-translational translocation
of      %   -`
 The protein to be translocated initially bind to the
translocon
 Cytosolic chaperones are released
Fig. 2.16: Endoplasmic reticulum translocation-Cotranslational  The leading end of peptide bind to the BiP
 ATP bind to BiP interacts with Sec 63/Sec 63
Steps of Cotranslational Endoplasmic Reticulum Translocation  ATP hydrolysis provide energy to move the protein
of Proteins =˜‡‰0*0{
forwards
Step 1: The signal sequence emerges from the ribosome and binds
 When entire protein has entered into the lumen, ADP
to the SRP. This temporarily arrests further elongation of the
polypeptide chain (elongation arrest)Q
is exchanged for ATP, allowing BiP to be released.
Step 2: The SRP–ribosome–nascent protein complex binds to the Cotranslational Insertion of Proteins into the ER
SRP receptor (SRP-R). SRP guides the protein to SRP-R
Membrane
Step 3: The SRP is released from SRP-R, the ribosome binds to the
Certain proteins inserted into the ER membrane without
translocon (Sec 61 complex), and the signal peptide inserts into
the channel in the translocon and translation resumes. SRP and
reaching the lumen.
SRP-R binds to GTPQ. When SRP to SRP-R interaction occurs GTP
is hydrolysed, SRP dissociates from SRP-R
Step 4: The signal peptide induces opening of the channel in the
translocon by binding to certain hydrophobic residues in it, thus
causing the plug on the translocon to move. The tranlocon opens
only when signal peptide is present
Step5: Cleavage of the signal peptide by signal peptidase occurs,
and the fully translocated polypeptide/protein is released into the
lumen of the ER.

Post-Translational Translocation of
Proteins into the lumen of ER Fig. 2.18: Cotranslational insertion of
 Less common pathway proteins into the ER membrane

 This process involves Sec 61 tranlocon complex, Sec Steps involved in the cotranslational insertion into the
[H\2 []  ^, Chaperone protein of hsp-70 - 
 `
family like BiP (Binding Immunoglobulin Protein),  The proteins partly transverses the ER membrane
ATP hydrolysis.  Signal peptide is cleaved
Proteins 85

 These proteins contains a highly hydrophobic  Secretory Vesicles


segment called halt or stop transfer signal or internal  COP I Vesicles
insertion sequence  COP II Vesicles
 The translocon recognize the stop transfer signal  Clathrin Coated Vesicles.
 This allows the protein to exits the channel via lateral Vesicle Function and characteristics
gate
Transport Vesicle They carry proteins to Plasma membrane.
 Protein is anchored to the ER membrane. The transport of protein occur continuously
hence called Constitutive Secretion
Secretory Vesicles Transport of proteins to be secreted out of the
EXOCYTIC PATHWAY OR SECRETORY
'Œ2 
 J 
 0
PATHWAY OF ROUGH ENDOPLASMIC The secretion is regulated by external signals,
RETICULUM BRANCH hence called Regulated Secretion
COP I They carry proteins from Golgi apparatus to
 This was first delineated by George Palade and Vesicle the Endoplasmic Reticulum
colleagues This is called Retrograde Transport
 The entire pathway of proteins traveling from - o COP II They carry proteins from Endoplasmic
GA o Plasma Membrane is often called Exocytic or Vesicle Reticulum to Golgi Apparatus or Endoplasmic
Reticulum – Golgi Intermediate Complex,
Secretory pathway
ERGIC. This is called Anterograde transport
 The proteins is carried in vesicles.
Clathrin coated Involved in Endocytosis of proteins from
The important vesicles are: vesicle late endosome to Lysosomes. This is called
 Transport vesicles Receptor mediated Endocytosis

Fig. 2.19: Functions of vesicles in rough


endoplasmic reticulum branch of protein sorting
86Self Assessment and Review of Biochemistry

CLINICAL CORRELATION–PROTEIN SORTING Contd...

Peroxisome Biogenesis Disorder      !     ! !  


peroxisome biogenesis
 Disorders associated with defect in import of 1 or

 $     
  # 5  
more proteins to Peroxisome
 Due to mutations in PEX genes encoding certain Defective oxidation and abnormal accumulation of very long chain
fatty acids
proteins—so-called peroxins, involved in various
steps of peroxisome biogenesis, such as the import 
     5 

 5$ 


of proteins to peroxisomes. Defects in certain steps of bile acid formation and accumulation of
bile acid intermediates
Peroxisomal Ghosts
Absence or reduction in the number of peroxisomes is pathognomonic Defects in oxidation and accumulation of l-pipecolic acid
for disorders of peroxisome biogenesis. In most disorders, there
are membranous sacs that contain peroxisomal integral membrane Increased urinary excretion of dicarboxylic acids
proteins, which lack the normal complement of matrix proteins; these
are peroxisome ‘ghosts’.

DNA mutations in Peroxisome


Due to gene defects, which involve mainly the import
Biogenesis Disorder
    %   % 2W    
 Zellweger syndrome (Most Severe)  PEX gene mutation in Zellweger Syndrome, Neonatal
 Neonatal adrenoleukodystrophy Adrenoleukodystrophy, Infantile Refsums Disease
 Infantile Refsum disease (Least Severe).  PEX 7 gene mutation in Rhizomelic chondrodys-
plasia.
Due to defect in PEX7, which involves the import of
   % 9 2H
 Rhizomelic chondrodysplasia punctata. Lysosomal Targeting Disorders
"          3$  
Clinical Picture of Peroxisomal
'  (  "
Biogenesis Disorders
 Due to lack of targeting signal, Mannose 6 phosphate
j) 2$   'j2(=   : 6 -
 Deficient in the activity of the cis-Golgi-located
$ %$ ':3"(=   - " ' -"(
+: % %  
These three disorders are considered to form a clinical
continuum, with ZS the most severe, IRD the least severe,  ˆ € #  # # “ }?  Ž      
and NALD intermediate. mannose residue of certain enzymes destined for
Lysosomes
Zellweger Syndrome
 These enzymes lack Mannose 6 Phosphate
 Typical facial appearance (high forehead, unslanting
  /      These enzymes are secreted into plasma rather than
and epicanthal folds) to Lysosomes
 Severe weakness and hypotonia, neonatal seizures  ˆ/7=  7/=      /
digested cellular material, as inclusion bodies
 Eye abnormalities (cataracts, glaucoma, corneal-
 `=  /  /  Clinical picture almost similar to Hurler’s Disease.
and nerve dysplasia) Psuedo-Hurler Dystrophy
 Because of the hypotonia and‚ mongoloid appearance,  Closely related to I-cell disease
Down syndrome may be suspected.  It is a milder condition, and patients may survive to
     !     ! !   adulthood
peroxisome biogenesis  The defect in pseudo-Hurler polydystrophy lies
Peroxisomes absent to reduced in number in the domain of GlcNAc phosphotransferase that
Peroxisomal enzyme Catalase in cytosol recognizes lysosomal enzymes
Contd...  Some catalytic activity is retained.
Proteins 87

REVIEW QUESTIONS

Chemistry of Proteins  Vitellin of egg yolk


W`  
  
 ! '+   H|WW(  Ovalbumin of egg white
a. It makes protein resistant  Glutelin of wheat
b. Bond is formed between carboxy terminal  Oryzenin of Rice
of one protein and amino group of a lysine  Gliadin of Wheat
residue on another  Ferritin-stores iron.
 ^ !!   
    =   Other options
of protein Myoglobin
d. Enzyme act as catalyst for bond formation
 It is a transport protein of Oxygen to Skeletal Muscle.
Ans. a, b, d
Ricin:
:$    '   ( '  -
 Inhibitor of mammalian Protein Synthesis
  (
 From castor bean inactivates eukaryotic 28 S ribo-
A bond formed between an amino group and a carboxyl
somal RNA.
group at least one of which is not an alpha group. Seen
in the side chains of proteins. ]` 5%%   %   )  
 %  Y
Characteristic Features logous substitution for isoleucine in a protein in
 Occur post-translationally & } ': H||[(
 Can be formed spontaneously or enzymatically a. Methionine
 Can produce stably linked protein dimers, multimers b. Aspartic acid
or complexes c. Valine
 Makes the protein resistant as proteases cannot d. Arginine
hydrolyse isopeptide bond. Ans. c. Valine
Examples   ;1    R 2

 Glutathione  One amino acid replaced by another amino acid of
 Thyrotropin Releasing Hormone similar characteristics
     Examples of homologous substitution is shown in
 Blood clots. the diagram given below.
Application of spontaneous Isopeptide Bond formation Hydrophilic, Acidic Asp Glu
 Develop a new peptide tag called Isopeptag. Used in: Hydrophilic, Basic His Arg Lys
 In vivo Protein targeting Polar, Uncharged Ser Thr Gln Asn
 Fluorescence Microscopy Imaging. Hydrophobic Ala Phe Leu Ile Val Pro
H` 5%%  %   )  \     !
   ' %    ( 2

'+ $ H|WW(
 One amino acid replaced by another amino acid of
a. Myoglobin
"     
b. Ovalbumin
c. Ricin 4. In a mutation if valine is replaced by which of the
d. Ferritin following would not result in any change in the
e. Glutelin      ! ': 2 $ |H(
Ans. b, d, e (    a. Proline
Storage Protein b. Leucine
Proteins that act as store house of amino acids and metal c. Glycine
ions that can be easily mobilized. d. Aspartic acid
 Casein of milk Ans. b. Leucine
88Self Assessment and Review of Biochemistry

Homologous substitution or Conservative mutation a. Denatured state


won’t result in any change in function of the proteins b. Molecular aggregation
examples for homologous substitution. c. Precipitation
Hydrophilic, Acidic Asp Glu d. Quaternary structure
Hydrophilic, Basic His Arg Lys Ans. d. Quaternary structure ( !"#"$
Polar, Uncharged Ser Thr Gln Asn Primary Structure is the linear sequence of amino acid
Hydrophobic Ala Phe Leu Ile Val Pro held together by peptide bonds in its peptide chains.
Bond involved in primary structure is Peptide Bond, a
Homologs, Orthologs and Paralogs type of covalent bond.
When two genes share detectable sequence similarities Secondary Structure
(nucleotide sequence in DNA and amino acid sequence       & & 
in the proteins they encode), their sequence are said to are about 3–4 amino Acids apart in linear sequence.
be homologous and the proteins encoded by them are  The folding of short (3- to 30- residue), contiguous
called homologs. segments of polypeptide into geometrically ordered
Homologs arises from a common ancestor. units
Homologs can be orthologs orparalogs.  Bonds involved in the secondary structure are
 Homologous genes in same species, are said to be primarily noncovalent bonds like:
paralogous and their protein products are paralogs.  Hydrogen Bond (Most important Bond)
Paralogous genes are result of gene duplication.  Hydrophobic Bond
 Paralogs are similar not only in sequence of  Electrostatic Bond (Ionic Bond, Salt Bridges)
amino acids but also in the three dimensional  van der Waals forces.
structure. $ ! The entire three-dimensional confor-
 V=  "     mation of a polypeptide is referred to as tertiary structure.
orthologous and their protein products are orthologs.  $ ! If more than one polypeptides
 +   "  &  = aggregate to form one functional protein, the spatial
gene may be taken as a rough measure of degree relationship between the polypeptide subunits is referred
to which two species have diverged during to as quaternary structure.
evolution. Bonds involved in Tertiary and Quaternary Structures
are primarily noncovalent bonds.
~` :   1   ! '+ L  |](
 Hydrophobic Interaction
a. Have net charge ‘0’
 Hydrogen Bond
b. Do not migrate
 Electrostatic Bond
c. Are positively charged
 Van der Waal’s Forces
d. Are negatively charged
Ans. a. Have net charge 0, b. Do not migrtate €`       )% ! ' H|WW(
 !"## a. Net charge of protein is zero
Properties of proteins at isoelctric pH b. Mass of protein is zero
 They have no net charge c. Protein
 They do not have electrophoretic mobility d. Denaturation of protein occurs
 Maximum precipitability Ans. a. Net charge of a protein is zero
 Minimum solubility ( !"##
  =="   ‚`        ! ' H|W|(
6. Proteins are linear polymers of amino acids. They a. Protein
fold into compact structures. Sometimes, these b. Cholesterol
folded structures associate to form homo-or- c. Steroid
% ` 5%%   %   )   d. Sugar
 %    } ': H||[( Ans. a. Protein
Proteins 89

Z3Z- -: Z2 Z@ : Z : "2 WW`  $      


$!
Biuret Test '  Q(
General test for Proteins a. Primary structure
Cupric ions in alkaline medium forms violet color with b. Secondary structure
peptide bond nitrogen. c. Tertiary structure
 %$  d. Quaternary structure
General test for all alpha amino acid Ans. a. Primary structure ( !"#"#
Amino acid + 2 mols of Ninhydrin — Aldehyde with 1 $ 2! It is the linear sequence of amino acid
carbon atom less + CO2 + Purple Complex (Ruhemann’s held together by peptide bonds in its peptide chains. Bond
Purple) involved in primary structure is Peptide Bond, a type of
Color Reactions Test answered by
covalent bond.
Xanthoproteic Test (Conc Aromatic Amino AcidQ2014DNB
Secondary Structure
HNO3 is a reagentQ) (Phenyl Alanine, Tyrosine,       & & 
Tryptophan) are about 3–4 amino Acids apart in linear sequence.
Millon’s test Tyrosine (Phenol)  The folding of short (3- to 30-residue), contiguous
Aldehyde test can be done in Tryptophan (Indole group) segments of polypeptide into geometrically ordered
two methods: units.
 Acree Rosenheim Test
Hopkin’s Cole TestQ
$ ! The entire three-dimensional confor-
mation of a polypeptide is referred to as tertiary structure.
Saka Guchi’s test ArGinine (Guanidinium group)
 



  $ ! If more than one polypeptides
Sulfur test Cysteine
aggregate to form one functional protein, the spatial
relationship between the polypeptide subunits is referred
Cyanide Nitroprusside Test Homocysteine
to as Quaternary structure.
Pauly’s Test Histidine (Imidazole) Tyrosine
(Phenol) WH` -      :"1      
 )%%  9$! '  Q(
ƒ` 5%%   %   )   %   a. Pyruvate Dehydrogenase
myoglobin? b. Lactate Dehydrogenase
a. Monomer
c. Malate Dehydrogenase
b. Homodimer
d. Isocitrate Dehydrogenase
c. Heterodimer
Ans. b. Lactate Dehydrogenase
d. Tetramer
Z^  )% -  @ 
Ans. a. Monomer
 Lactate dehydrogenase
 Myoglobin is a monomer
 Alcohol dehydrogenase
 Hemoglobin is a tetramer
 Glyceraldehyde-3-phosphate dehydrogenase
W|` "   
$ )%%  %   )   Malate dehydrogenase

 } '  Q(  Quinone oxidoreductase
a. Peptide bond  6-phosphogluconate dehydrogenase
b. Hydrogen bond  D-glycerate dehydrogenase
 _   Formate dehydrogenase
d. Electrostatic bond
 —'™Œ9
/*// 
Ans. a. Peptide bond
Denaturation of proteins W]`    ]"    )    %!
 Primary structure is not lost '+ $ H|W…(
 Hence peptide bond is not broken a. Peptide bond
 Secondary and tertiary structure is lost b. Amino acid sequence
 There is loss of folding. c. Interaction between polypeptide
90Self Assessment and Review of Biochemistry

d. Chaperone d. Hemoglobin
e. Side chain e. Albumin
Ans. b, c, d, e Ans. b. Collagen ( !"#0/%*"
 !"#"$%&#%&"%&&'*+/0# Plasma Proteins in Transport FunctionQ
 Amino Acid sequence determines the tertiary Transport Protein Compound it binds
structure of proteins Albumin  Bilirubin
 Side chains help in the formation of bonds involved  Free fatty acids
in tertiary structure of proteins  Ions (Ca2+)
 Interaction of polypeptide also helps the three  Metals (e.g. Cu2+, Zn2+)
 Met Heme
dimensional structure of proteins
 Steroids
 Chaperone helps in protein folding; hence it helps in  Hormones
three dimensional structure. Ceruloplasmin  Cu2+
 But Peptide bond helps in the primary structure. Corticosteroid-binding  Cortisol
globulin (transcortin)
W…`   $       ! Haptoglobin  Extra corpuscular hemoglobin
'+ $ H|W…( Lipoproteins  Plasma Lipids
a. Western Blot Hemopexin  Heme
b. ELISA Retinol-binding protein Retinol
c. Chip assay Sex-hormone-binding  Testosterone
globulin  Estradiol
d. Dot blot
Thyroid-binding globulin  T4
Ans. a, b, c, d  T3
Western blot, ELISA, Chip assay and dot blot is based Transferrin  Iron
on Antigen antibody interaction. Hence, they are Transthyretin (formerly  T4 and forms a complex with
 = /  #       = prealbumin) retinol-binding protein
for Microarray. Just like DNA Chip, where DNA–DA
Hybridization is done, there Protein Microarray or W€` 5%%  %     
  
Protein Chip where Antigen antibody interaction is done. 
! '+   H|W…(
 Dot blot or slot blot is a blot technique in which the a. Amino group
    ==    b. Carboxyl group
 This can be used for proteins also. c. Side chain
W~` 5% $      } d. Amide group
' 3%  H|W…( e. Aldehyde group
a. Lipoprotein Ans. a, b
b. Phosphoprotein  Peptide bond is formed between amino group of one
c. Glycoprotein amino acid with carboxyl group of the next amino
d. Flavoprotein acid.
Ans. b. Phosphoprotein W‚` 5%%   %   )  
  
       ! '+ $ H|W~(
Two important Phosphoproteins are a. Protein consisting of one polypeptide can have
 Casein found in milk quaternary structure
 Ovovitellin found in egg yolk  + #=   #        
requires that the two participating cysteine
W[` 5%%  \       ! residues be adjacent to each other in the
'+ $ H|WH( primary sequence of the protein
a. Transferrin c. The stability of quaternary structure in protein
b. Collagen is mainly the result of covalent bonds among
c. Ceruloplasmin the subunits.
Proteins 91

d. The denaturation of proteins always leads  Acids bring the pH of the medium to isoelectric pH,
to irreversible loss secondary and tertiary precipitability is maximum at isoelectric pH. This is
structure. because at isoelectric pH proteins carry no net charge,
e. The information required for the correct hence no shell of hydration.
# #           The reagents used are Phsophotungstic acid,
sequence of amino acid along the polypeptide Sulphosalicylic acid, Phosphomolybdic acids,
chain. Trichloroacetic acid.
Ans. e. The information required for the correct folding Methods to precipitate protein by removing the shell
#          # =  of hydration
acid along the polypeptide chain.  Precipitation by neutral salts
 Proteins with more than one polypeptide chain can  Concentrated Salt solution removes the shell of
only have quaternary structure. hydration.
 The cysteine residues need not be adjacent for the  Reagents used are Ammonium sulfate. This is
#=  #  called Salting out.
 The stability of quaternary structure is by noncovalent  Precipitaion by organic solvents
bonds.  The reagent used are ether, alcohol, acetone etc.
 Denaturation can be reversible also.
HW` 1
2= +  
$  ` 5%
Wƒ` : % %^        6$ 
 will be its electrophoretic mobility?
disrupted if a missense mutation introduces the a. Increased ': 2  H|W~(
following amino acid within the alpha helical b. Decreased
! ': 2   H||H( c. No change
a. Alanine d. Depends on level of concentration of HbS
b. Aspartic acid Ans. b. Decreased
c. Tyrosine In Hb S, Glutamic acid is replaced by Valine. Hence the
d. Glycine negative charge is decreased. So Hb S moves slower than
Ans. d. Glycine ( !"#" HbA1.
 Glycine induces bends in the alpha helix. The relative mobility of various Hb fractions in Hb
 Proline also disrupts the conformation of alpha helix. electrophoresis.
 `    #  *
Separatory Techniques of Proteins, Study of Protein
Structure, Precipitation of Proteins

H|`         ^!


': 2   H|W~(
a. Adding alcohol and acetone
b. pH changes is moved away from isoelectric pH
c. With Trichloroacetic acid
d. With heavy metals
Ans. b. pH changes away from isoelectric pH
 Precipitability maximum at isoelectric pH, so pH of
medium should be brought to isoelectric pH.
Precipitation reactions of proteins
Methods to precipitate the protein by neutralizing the
charge are
Precipitation by heavy metallic salt.
 The reagents used are Mercuric nitrate, Zinc Sulfate,
Lead acetate, Ferric Chloride.
Precipitation by Acids.
92Self Assessment and Review of Biochemistry

- 
$    1
     Methods for Separating and Purifying Biomolecules
or point of application is  Salt fractionation (e.g. precipitation of proteins with ammonium
sulfate)
1 HbA2, 2. HbS, 3. HbF, 4. HbA1
 8  5$‡ J 5'  
 ' $'  $'  
" '5 " ' 
HH` @  )  2"2 :+  % =    Electrophoresis: Paper, high-voltage, agarose, cellulose acetate,
    
 W|| 6"= :   )% starch gel, polyacrylamide gel, SDS-polyacrylamide gel
 % =  % ) H
   H| "    Ultracentrifugation.
]| " )$ `    ! Methods for Determining Biomolecular Structures
 Elemental analysis
a. Protein has undergone hydrolysis of S-S
 UV, visible, infrared, and NMR spectroscopy
linkage  Use of acid or alkaline hydrolysis to degrade the biomolecule
b. It is a dimer of 2 subunits of 20 and 30 KDa understudy into its basic constituents
 š   $;$ Œ  5

$  
c. It is a tetramer of 2,20 KDa and 2,30 KDa biomolecule under study (e.g. proteases, nucleases, glycosidases)
d. Protein break down due to noncovalent linkage  Mass spectrometry
 @5

 "
 =005   


 {
Ans. c. It is a tetramer of 2,20 KDa and 2,30 KDa
 X-ray crystallography.
( !"#
SDS-PAGE in conjunction with 2 Mercaptoethanol or H~` %    $ %     
dithioreitol    ^!
a. UV Spectroscopy
 ]* !/  ! 
b. NMR Spectroscopy
 So separate the components of multimeric proteins
c. X-ray crystallography
Option A—Protein is undergoing oxidative cleavage of
d. Edman’s technique
S-S bond not hydrolysis
Ans. d. Edman’s Technique
Option B—The protein is 100 KDa. Two subunits of 20 ( !$%1232
KDa and 30 KDa will not make a 100 KDa protein.
 Edman’s reaction is used to detect sequence of amino
Option D—Disulphide bond is a covalent bond not a acids in a polypeptide.
noncovalent bond.
H[` 2 8    %$! '  Q(
a. 2, 4 Dinitro benzene
H]`         
$!
b. 2, 4 Dinitro Cresol
': 2   H|W|(
c. 1, Flouro 2, 4 Dinitro Benzene
a. Salting out
d. 2, 4 Fluoro Dinitro Cresol
b. Ion exchange chromatography Ans. c. 1, Fluoro 2, 4 Dinitro Benzene
c. Mass chromatography  Sanger’s reagent is 1 Fluoro2, 4 Dinitro benzene
d. Molecular size exclusion  Edman’s reagent is Phenyl Isothiocyanate
Ans. a. Salting out ( !$%1232
H€` :  %   )    
 26 
2 @  '  ƒ$   = ^! ': H||‚(
ammonium sulfate. a. Single nucleotide change results in change of
Glutamine to Valine
H…` :  %   )     %   b. RFLP result from a single base change
= ^! ': 2   H||‚(
c. ‘Sticky patch’ is generated as a result of
a. High performance liquid chromatography replacement of a nonpolar residue with a polar
b. Mass spectrometry residue
c. X-ray crystallography d. HbS confers resistance against malaria in
d. NMR spectrometry heterozygotes.
Ans. c. ‘Sticky patch’ is generated as a result of
Ans. a. High performance liquid chromatography
replacement of a nonpolar residue with a polar residue
( !$%1232 Sickle Cell Anemia (HbS)
Proteins 93

Molecular basis of Sickle Cell Disease 5   


Point mutat  *  #9
 ‚“?“““ƒ
Hƒ`   
 \  ) '9( \
that leads to the replacement of a Glutamate (Polar)
     ^! '+ $ H|WH(
residue with a Valine (Nonpolar) residue
   
Investigations
b. Native gel electrophoresis
 Hb electrophoresis—Hb S migrate slower in an
c. 2D Gel Electrophoresis
     ! 
by valine d. Gel Filtration Chromatography
 Sickling Test e. Ultracentrifugation
 HPLC to fractionate Hb Ans. b. Native Gel Electrophoresis
 Isoelectric Focussing   1 4 5 6 *  76 *+2"
 Native gel electrophoresis is based on charge
 Allele-specific oligonucleotide probe detects
hemoglobin (Hb) S allele.     ©  # 
   /
molecular mass
 Direct diagnosis of sickle cell disease using RFLP.
 2D Electrophoresis based on isolelectric pH and Mol
H‚`  
  
 # % & ! wt (size)
'  Q(  Gel filtration chromatography/Molecular sieve
 Y  ^==    chromatography is based on Mol wt/size.
b. Invented by Oakley Fulthrop
]|` %  '(       \!
c. Ougundi technique '+ H|W…(
 ]  /_== " a. X-ray Crystallography
Ans. d.]  /_^== " b. NMR Spectroscopy
( 1 45 6 * 7 776 *+22 c. Electrophoresis
Immunochemical method used for identification of d. Ultrasonography
antigens and antibodies
e. Infra red spectroscopy
For Qualitative Purposes
Ans. a, b, e  !$1232
 !“_"
 Š ^== _" $ ]W` 2"2Y:+=     
 !
 Concentration gradient for only a single reactant. '+ L  H||ƒ(
Diffusion of antigen into agar impregnated a. Mass
& / Y   ^==  _"    b. Charge
quantitative method based on Single Immuno c. Density
"  d. Molecular weight
 __" ‚]  /+  ƒ$ e. Solubility
 Concentration gradient is established for both Ans. a, d  !"#
antigen and antibody. It permits comparison of PAGE
two or more test materials and provides a simple
 Protein is separated based on molecular mass or mol
and direct method used to determine whether wt/size and charge
the antigens in the test specimens are identical,
SDS PAGE
cross reactive, or nonidentical. This technique
was named after the Swedish Physicist Orjan  SDS impart equal negative charge so that it masks
Ouchterlony. the inherent charge of the Protein
Immuno electrophoresis  Now Proteins separate based on Mol wt (Size) only
Technique used to separate and identify the various 2"2Y:+   †  )% H  %  
dithioreitol
protein species contained in a sample. This technique is
used to study of antigen mixtures and human gammo-  ]* !/  ! 
pathies. So separate the components of multimeric proteins
94Self Assessment and Review of Biochemistry

]H` %   %  %$  )%%   FRAP is a technique used to study Fluid mosaic model
that are negatively charged are selectively of cell membrane, movement of proteins, etc. FRAP is
released from stationary phase into the positively Fluorescence Recovery After Photobleaching
%    
 %   ! The technique
': H|W|(
Fluorescent dyes emit colored light when it is illuminated,
 ?@ / =  / but if a very high intensity light is used then these dyes are
b. Ion-Exchange chromatography    &  
c. Adsorption chromatography ˆ  !  
d. Size-Exclusion chromatography This recovery after photobleaching is used to study
Ans. b. Ion-Exchange chromatography movement of proteins lipids carbohydrates, etc.
Stationary Property used for
Chromatography phase used separation ]…` :  &      
$!
Paper Water held on a Based on the polarity '  Q(
Chromatography solid support of Least Polar moves a. Sanger’s reagent
5 5= faster
Cellulose) b. Benedicts reagent
Thin Layer Silica gel Based on Polarity c. Trypsin
Chromatography (Kieselguhr) Least Polar moves d. Cyanogen bromide
spread on a faster
glass plate or a Ans. b. Benedict’s reagent ( !"#"2%"
plastic sheet or Methods of Protein Sequencing
aluminium sheet.
Sanger’s Technique
Ion Exchange Column of Ion Based on Charge-
Chromatography exchange resins Charge Interaction  Sanger’s Reagent is 1, Fluoro 2, 4 DinitrobenzeneQ
Anion exchange  Sanger’s Reagent derivatizes the amino terminal
or Cation residues.
exchange resins
 +  /=
Size Exclusion Column of porous Based on Molecular
Chromatography beads weight (Size) Insulin by Fredrick Sanger. He got Nobel prize in
Other names Particles emerge in 1958.
Molecular Sieve the Descending order  Only dipeptides or tripeptides can be sequenced.
Chromatography of Stokes RadiusQ
Gel Filtration Edman’s Technique
Chromatography  By using Edman’s Reagent (Phenyl Iso-Thio-cyanate).
Gel Permeation
Chromatography
 Phenyl Isothiocyanate derivatizes the amino terminal
of Polypeptide.
$ Column of resins :   5


ChromatographyQ   5

 ligand binding  Edman’s Technique can sequence many residues
ligands used. behavior or Biological (5–30 residues) of a single polypeptide sample unlike
activity
Sanger’s
Hydrophobic Based on hydrophobic
Interaction interaction. ]~`     9 … "   
$!
Chromatography
a. Gene array chip
Absorption Based on absorption
Chromatography property. b. Electron spray ionization
c. Quadrupole mass spectrometry
]]`           $  d. Matrix assisted Laser Desorption ionization

 
$} ': 2 H|W|( Ans. c. Quadrupole mass spectrometry
a. FISH ( !"#"2%"
b. FRAP Quadrupole mass spectrometers generally are used to
c. Confocal microscopy determine the masses of molecules of 4000 Da or less,
d. DNA microscopy 
 
        
Ans. b. FRAP determine the large masses of complete proteins.
Proteins 95

][`      \  


$! Structure of Collagen
a. Western blot Characteristic Features
b. Chromatography Glycine-X-Y repeats
c. Centrifugation Every third amino acid residue in collagen is a glycine
    residue.
e. Electrophoresis Alpha Chain
Ans. b, c, d, e Polyproline helix of three residues per turn twisted in
 Chromatography, Centrifugation, Ultrafiltration, left-handed direction. Each polypeptide chain contains
electrophoresis are separatory techniques 1000 Amino Acids.
 Western blot is method to detect protein by Antigen Triple helical structure
antibody interaction. Three of these alpha chains are then wound into a right-
handed super helix.
Fibrous Proteins
Quarter Staggered arrangement
]€`    )%% $      %$  Lateral association of the triple helical units
! ': 2   H||€( Each is displaced longitudinally from its neighbor by
a. Type I slightly less than one-quarter of its length. Responsible
b. Type II for tensile strength of Collagen Fibers
c. Type III
]ƒ`     
% 6   `  
d. Type IV
 % % 6 ` %  ! ': H|WH(
Ans. b. Type II ( !"#0  ^   # 
Types of Collagen b. Decreased no of water molecules
 Major Collagen present in Bone-Type I (90%) c. Increased Na content
 Major Collagen present I Dermis, ligaments and d. Increased hydrogen bonds
tendons-Type I (80%)
Ans. a.^    
 Major Collagen present in Cartilage-Type II (40–50%)
 +=   
 Major Collagen present in Hypertrophic Cartilage-
 Keratin is rich in cysteine.
Type X
 Major Collagen present in Aorta-Type I and Type III 40. The structural proteins are involved in
(20–40% each) maintaining the shape of a cell or in the formation
 Major Collagen present in Basement Membrane of matrices in the body. The shape of these protein
Type IV ! ': H||[(
 Most abundant Collagen-Type I a. Globular
 Major Collagen in Keloid is Type 3 >> Type 1 b. Fibrous
   5   1  Q c. Stretch of beads
?   !  = *         =  d. Planar
     / ^^^     #=   Ans. b. Fibrous ( 9 ;  <6 /&"
is replaced by a matrix composed primarily of type  % %        !
Icollagen W` @
   
]‚`  
   '+ $ H|WW(  Elongated or Needle shaped or long cylindrical or
a. Triple helix rod like
 9    Minimum Solubility in water
c. Vitamin C is necessary for post-translation  Regular Secondary Structure
=   Axial Ratio>10
d. Glycine residue at every third position  They are Structural Proteins
Ans. a, c, d ( !8 "#0$  Eg: Collagen, Elastin, Keratin
96Self Assessment and Review of Biochemistry

H` +obular Proteins  Type of collagen present in the skin hemidesmoses:


 Spherical or oval or Spheroidal in shape Type XVII
 Easily Soluble  Type of collagen present in the rhabdomyosarcoma
 Axial Ratio < 3 cell: Type XIX
 They perform dynamic functions, e.g. Albumin,  Type of collagen present in the brain: Type XXV
Globulin, most enzymes  Type of collagen present in the testis and ovary:
…W`         ! Type XXVI
a. Immunoglobulin Type Distribution Type Distribution
b. Hemoglobin I Noncartilaginous XV Associated with
c. Collagen connective tissues, collagens close to
including bone, tendon, basement membranes
d. Keratin skin in many tissues
Ans. c. Collagen (Ref: Harper 30/e p629) including in eye,
 Triple Helix in Collagen muscle, microvessels

 Quarter Staggered arrangement in Collagen II Cartilage, vitreous XVI Many tissues


humor
 Covalent cross links in collagen
III Extensible connective XVII Epithelia, skin
 Desmosine cross links in elastin tissues, including skin, hemidesmosomes
lung, vascular system
…H` :  %   )   &   %$ ^$
       $ % ^! ': Wƒƒ€( IV Basement membranes XVIII Associated with
collagens close
a. O2 to basement
b. Vitamin membranes, close
c. Monoxygenases structural homologue
of XV
d. Pyridoxal phosphate
V Minor component in XIX Rare, basement
Ans. d. PLP ( !"#0 tissues containing membranes,
 Prolyl and Lysyl Hydroxylases are Monooxygenases, collagen I rhabdomyosarcoma
require molecular O 2  ” =    ' ª cells
Glutarate. VI Muscle and most XX Many tissues,
connective tissues particularly corneal
…]` †  $    
  
 ! epithelium
': 2 $ H|W~( VII Dermal-epidermal XXI Many tissues
a. Type I junction
b. Type II VIII Endothelium and other XXII Tissue junctions,
c. Type III tissues including cartilage-
$# < ' 
d. Type IV follicle-dermis
Ans. d. Type IV ( !"#0 IX Tissues containing XXIII Limited in
Key points distribution of collagen collagen II tissues, mainly
transmembrane and
 Type of collagen present in the noncartilaginous shed forms
connective tissue, like bone, tendon: Type I
X Hypertrophic cartilage XXIV Developing cornea
 Type of collagen present in the Cartilage: Type II and bone
 Type of collagen present in the vitreous humor: XI Tissues containing XXV Brain
Type II collagen II
 Type of collagen present in the basement membrane- XII Tissues containing XXVI Testis, ovary
mainly: Type IV (rarely Type XIX) collagen I
 Type of collagen present in the dermal epidermal XIII Many tissues, including XXVII Embryonic cartilage
junction: Type VII neuromuscular and other developing
 Type of collagen present in the extensible connective junctions and skin tissues, cartilage in
adults
tissue like skin, lung, vascular system: Type III
 Type of collagen present in the hypertrophic carti- XIV Tissues containing XXVIII Basement membrane
collagen I around Schwann cells
lage: Type X
Proteins 97

Protein Sorting, Glycoprotein 46. Endoplasmic >    


44. Prote    
$! ': H||ƒ( % %! '  Q(
a. Golgi Bodies a. Translocon
b. Mitochondria b. Chaperone
c. Ribosomes c. Ubiquitin
d. Nuclear Membrane d. Mannose 6 Phosphate
Ans. a. Golgi bodies ( !"#0# Ans. a. Translocon ( !$%0
Functions of Golgi bodies The singal hypothesis proposed that the protein is
 O-Glycosylation of proteins inserted into the ER membrane at the same time as its
mRNA is being translated on polyribosomes, so-called
 Protein sorting
cotranslational insertion.
 Processing of oligosaccharide chains of Glycoproteins
Principal Components involved in Endoplasmic
…~`      %   )  ! '+ |~( Reticulum Translocation
a. Sec 61 translocon complex form passage way N-terminal signal peptide
b. SRP-R is a docking protein  Polyribosomes
c. SRP blocks elongation  SRP, signal recognition particle
d. SRP-R releases elongation block  SR, signal recognition particle receptor
e. SRP-R is ATP bound  Sec 61, the transloconQ
Ans. e. SRP-R is ATP bound  Signal peptidase
 2--R, signal recognition particle receptor is a  Associated proteins (e.g, TRAM and TRAP)
docking protein  TRAM is Translocating Chain Associated Membrane
 Translocon Q (Sec 61 complex) consist of three Protein. TRAM accelerates the translocation of certain
membrane proteins, the sec-61 complex that forms proteins.
the protein conducting channel in the ER.  TRAP, Translocon –associated Protein Complex. The
 The signal sequence emerges from the ribosome and function of TRAP is not clear.
binds to the SRP. This temporarily arrests further …€`          !
   %  $ % '  '  Q(
(`Q a. Protein synthesis
 2-   2-Y-
   +Q not ATP b. Muscle contraction
 The SRP is released from SRP-R translation resumes c. Protein Sorting
Steps of Co translational Endoplasmic d. Glycoproteins
Reticulum Translocation of Proteins Ans. b. Muscle Contraction ( !"#0#
Step 1: The signal sequence emerges from the ribosome and binds  Rough Endoplasmic reticulum is involved in protein
to the SRP. This temporarily arrests further elongation of the
synthesis
polypeptide chain (elongation arrest).Q
 N linked glycoproteins are glycosylated in the EPR
Step 2: The SRP–ribosome–nascent protein complex binds to the
SRP receptor (SRP-R). SRP guides the protein to SRP-R  RER along with GA is a part of exocytic pathway of
Step 3: The SRP is released from SRP-R, the ribosome binds to the
protein sorting
translocon (Sec 61 complex), and the signal peptide inserts into
the channel in the translocon and translation resumes. SRP and
…‚`  9$     "!
SRP-R binds to GTPQ. When SRP to SRP-R interaction occurs GTP '  Q(
is hydrolyzed, SRP dissociates from SRP-R a. GlcNAcphosphotransferase
Step 4: The signal peptide induces opening of the channel in the b. Mannose Phosphotransferase
translocon by binding to certain hydrophobic residues in it, thus
c. Phospho Diesterase
causing the plug on the translocon to move. The translocon opens
only when signal peptide is present. d. Mannose 6 Phosphate Transferase
Step 5:Cleavage of the signal peptide by signal peptidase occurs, Ans. a. GlcNAcphosphotransferase
and the fully translocated polypeptide/protein is released into the  ' (  
lumen of the ER.  Lysosomal enzymes targeting disorder
98Self Assessment and Review of Biochemistry

 Recognition marker of Lysosomal enzymes is Man-  Alzheimers Disease


nose 6 Phosphate  Parkinson’s Disease
 I cell disease lack the golgi located NAcetyl Glucosa-  Huntington’s Disease
mine phosphotransferase.  Fronto Temporal Dementia
 Dementia with Lewy Bodies
49. Targeting sequence that direct Endoplasmic
 Amyloidosis
-        ˆ ) 
-  ZY `  Beta thalassemia.
a. KDEL ~H` 5%%  %   )        
b. KDAL  %     %   } ': H||ƒ(
c. DALK a. Proteases
d. KDUL b. Proteosomes
Ans. a. KDEL ( !"#0# c. Templates
-       % +: d. Chaperones
A number of proteins possess the amino acid sequence Ans. d. Chaperones ( ! "#0#$
"3 (Lys-Asp-Glu-Leu) at their carboxyl terminal.   %  !
 ª_ˆ
         !   GA in  Hsp 70
]^^ !    &     Hsp 90
KDEL receptor protein, which retains them transiently.  Hsp 40 [Cochaperone]
They then   Z      % -=
 BiP [Immunoglobulin heavy chain binding protein]
where they dissociate from the receptor, and are thus
 Glucose Regulated Protein [GRP-94]
retrieved. HDEL sequences (H = histidine) serve a similar
 Calreticulin
purpose. The above processes result in net localization of
certain soluble proteins to the ER lumen.  Calnexin.
2[W gene, which encodes a channel through which  9$     !
secretory proteins under construction pass into the  Protein Disulphide Isomerase
endoplasmic reticulum lumen  Peptidylprolylisomerase.

~|` 2 $     $ %9  ! ~]` :  -J 


 % = ^!
': 2   |~( ' H|WH(
a. Cytoplasm a. Many of them are known as heat shock proteins
b. Endoplasmic reticulum b. They use energy during the protein-chaperone
c. First in cytoplasm and then in Endoplasmic interaction
Reticulum c. Ubiquitin is one of the most important
d. First in endoplasmic reticulum and then in chaperone
cytoplasm d. They are present in wide range of species from
Ans. b. Endoplasmic reticulum bacteria to human
Proteins are synthesized in the Endoplasmic reticulum Ans. c. Ubiquitin is one of the most important chaperone
and free ribosome. ( ! "#0#$
Protein Folding
Protein Folding Proteins are conformationally dynamic molecule that can
~W` 5%%         } fold into functionally competent conformation
a. Prion Disease Auxillary Proteins assist Protein Folding, they are called
b. Alzheimer’s Disease Chaperones.
c. Beta Thalassemia Properties of Chaperone Proteins
d. Ehlers-Danlos Syndrome  Present in a wide range of species from bacteria to
Ans. d. Ehlers-Danlos Syndrome ( !"#0#$ humans.
%  -   " Q  Many are so-called heat shock proteins (Hsp).
Prion like changes underlie many neurodegenerative  Are inducible by conditions that cause unfolding of
 €   '*    newly synthesized proteins (e.g. elevated temperature
  9 and various chemicals).
Proteins 99

 They bind to predominantly hydrophobic regions ~~`  


 % ! '+ $ H|WH(
of unfolded proteins and prevent their aggregation. a. Belong to heat shock Proteins
 They act in part as a quality control or editing b. Wide range of Expression
mechanism for detecting misfolded or otherwise c. Present from bacteria to human
defective proteins. d. Ubiquitin is the most important chaperones
 Most chaperones show associated ATPase activity. e. Also known as Stress Proteins
Ubiquitin is a key Molecule in Protein Degradation Ans. a, b, c, e  !"#$%1&$3
Ubiquitin  Ubiquitin is the key molecule of protein degradation.
 Key molecule in Protein degradation.
~[` 5%%  %   )     %
 Small protein with 76 Amino Acids.
o-Glycoxylation linkage in oligosaccharide
 Highly conserved protein.
 ! ': H|WH(
 ? = #    a. Asparagine
is called Kiss of Death.
b. Glutamine
        =   # ˆ/  # c. Serine
the target protein hence it is a Pseudopeptide or
d. Cysteine
Isopeptide Bond.
Ans. c. Serine  !"#/"
 Minimum of four Ubiquitin molecules must be
Major Classes of Glycoproteins
 == =    
O-linked Glycoprotein
~…` :$     %
  !  O-glycosidic linkage between hydroxyl side chain
' H|WH( of serine or threonine and N-acetylgalactosamine
a. A naturally present protein in normal is present
individuals Y 6 +$  
b. Involves selectively blood vessels  N-glycosidic linkage between amide nitrogen of
c. Is visible by naked eyes as whitish cheesy asparagine and N–acetylglucosamine is present.
material +$ $% %$  Y %  '+ Y % =
 + Y 6( $   
d. A material which gets deposited in extra-
cellular spaces  Third major class of Glycoproteins.
 They link several proteins to Plasma membrane.
Ans. d. A material which gets deposited in extracellular
spaces  Linked to the carboxyl terminal amino acid of a
protein via a phosphorylethanolamine moiety joined
(Ref: Robbins and Cotran Pathologic
to an oligosaccharide (glycan), which in turn is linked
   $*0
via glucosamine to phosphatidylinositol (PI).
Amyloid Fibrils
%$   :$  ~€`  
  ! '  (
 X-ray crystallography and infrared spectroscopy a. Proteins
demonstrate a characteristic  Y?  sheet b. Glycoproteins
conformation. c. Proteoglycan
 Congo red staining shows apple-green birefringence d. Glycoside
under polarized light. Ans. b. Glycoproteins
 By electron microscopy amyloid is seen to be made Biologically Important GlycoproteinsQ
 /#       &  Plasma proteins (except albumin)
a diameter of approximately 7.5 to 10 nm.  Blood group substances
%   :$   Hormones (HCG, TSH, LH, FSHQ)
 žšŸ# =/=    #   Enzyme-Alkaline Phosphatase
 5% of the amyloid material consists of P component  Structural Protein-Collagen
and other glycoproteins.  Transport Proteins-Ceruloplasmin, Transferrin.
e

e
m

m
m
co

3 Enzymes
m
co

Topics Included
• General Enzymology
• Clinical Enzymology
m

GENERAL ENZYMOLOGY Coenzyme


co

Thermostable, low molecular weight, nonprotein organic


Enzymes
substances are called Coenzymes. A coenzyme can bind
Enzymes are highly specialized proteins that can act as covalently or noncovalently to the enzymes. If covalently
catalyst in biological systems. The word enzyme was bound then it is called Prosthetic group. Involvement of
coined by Frederick Kuhne meaning ‘in Yeast’ coenzyme with substrate is so intimate that, coenzyme
• Substrate: The substance on which enzyme act is called co-substrates.Q
• Product: The substance produced by the action of
enzyme on the substrate. Enzymes are heat labile, Examples of Coenzymes
thermolabile and proteinaceous in nature Enzyme Coenzyme
• Exception to proteinaceous nature of enzymes are Kinases ATP/GTP
Ribozymes. Dehydrogenases NAD+/FAD
m

RibozymeQ is RNA with catalytic activity, e.g. Sn RNA in Spliceosome Pyruvate Dehydrogenase, Alpha TPP, Lipoic Acid, CoA, FAD,
co

• Ribonuclease P Keto Dehydrogenase, Branched NAD+


• Peptidyl Transferase Chain Ketoacid Dehydrogenase
• RNase H Transketolase Thiamine Pyrophosphate
Abzyme are antibodies with catalytic activity
Transaminase Pyridoxal Phosphate
Carboxylases Biotin
Enzymes can be of two types:
1. Simple Enzyme: Consists of only proteins.
2. A complex enzyme consists of: Cofactor and Prosthetic Group
‒ Protein part: Apoenzyme Prosthetic groups are nonprotein part tightly integrated
‒ Nonprotein part to the enzyme structure by covalent forces.
Apoenzyme + Nonprotein part (Prosthetic group/Cofactor/
• Metals are the most common prosthetic group
Coenzyme) = Holoenzyme • Enzymes which have tightly bound to metal is called
metalloenzyme.
m

Nonprotein part can be Cofactors associate reversibly with Enzymes or Substrates.


co

• Coenzyme • The most common cofactors are also metals


• Cofactor • Enzymes which have metal as cofactors, e.g. loosely
• Prosthetic group. bound to them are termed Metal activated Enzymes.
e

e
m

m
m
co

Enzymes | 101

Metal as Cofactors and Prosthetic Group Contd...

Metal Enzymes Enzyme Enzyme Type of reaction catalyzed


Class No class with examples
ZincQ Carbonic anhydraseQ, Carboxypeptidase, Alcohol
V Isomerases Enzymes that catalyze geometric
Dehydrogenase Q, Alkaline Phosphatase, ALA
or structural changes within a
Dehydratase Lactate Dehydrogenase
molecule.
Magnesium Phosphotransferase, (Hexokinase, Phospho- • Phosphohexose isomerase
m

fructokinase) • Mutase
Mutase, Enolase,
co

• Racemase
Glucose 6 Phosphatase
VI Ligases Enzymes that catalyze the
Copper Tyrosinase, Cyt C Oxidase, Lysyl Oxidase, joining together (ligation) of two
Superoxide Dismutase, Amino Acid Oxidase molecules in reactions coupled
Molybdenum Xanthine Oxidase, Sulfite Oxidase to the hydrolysis of ATP.
• Acetyl CoA carboxylase
Manganese Enolase, Arginase, Phosphotransferase
Arginosuccinate synthetase
(Hexokinase, Phosphofructokinase)
Mitochondrial Super Oxide Dismutase • PRPP synthetase
• Carbamoyl phosphate
Iron Succinate Dehydrogenase Synthetase
Calcium Lipase, Lecithinase • Glutamine synthetase

OxidoreductasesQ
m

IUBMB CLASSIFICATION–ENZYME CLASSES Subclassified into


co

As per this system of nomenclature, Enzymes are • Dehydrogenase


classified into six main classes: • Oxygenase
Enzyme Enzyme Type of reaction catalyzed ‒ Monooxygenase
Class No class with examples ‒ Dioxygenase
I Oxidoreductases Enzymes that catalyze oxidation • Oxidase
of one substrate with reduction
of another substrate (described • Catalase
later) • Peroxidase.
II Transferases Transfer functional group
other than hydrogen from one Dehydrogenase
substrate to another. • Catalyze removal of hydrogen from the substrate
• Kinases • They cannot use oxygen as a hydrogen as the
m

• Transaminase
acceptor.
co

• Transmethylase
Examples are:
III Hydrolases Enzymes that catalyze hydrolytic
cleavage of C—C, C—O, C—N • Alcohol Dehydrogenase
and other covalent bonds. • Lactate Dehydrogenase
• Lipase • Succinate Dehydrogenase.
• Arginase
• Pepsin Oxidases
• Esterases • Removal of hydrogen from a substrate with oxygen
IV Lyases Enzymes that catalyze cleavage as the acceptor of Hydrogen.
of C—C, C—O, C—N and
other covalent bonds by atom
Examples are:
elimination, generating double • Mono Amino Oxidase
bonds. • Cytochrome C Oxidase
m

• Aldolase
• Xanthine Oxidase.
co

• Fumarase
• HMG CoA Lyase Oxygenases
• Argininosuccinase • Catalyze the direct transfer and incorporation of oxygen
Contd... into a substrate molecule.
e

e
m

m
m
co

102 |  
Self Assessment and Review of Biochemistry

Monooxygenases [Mixed Function Oxidases or Hydro- Michaelis–Menten Theory


xylase] (Enzyme–Substrate Complex Theory)
Incorporate one atom of molecular oxygen into the Enzyme combines with a substrate to form a transient
substrate. Examples are: Enzyme-Substrate Complex which immediately break
• Phenylalanine Hydroxylases into Enzyme and products.
• Tyrosine Hydroxylase
m

• Tryptophan Hydroxylase
co

• 7 alpha Hydroxylases
• Cytochrome p450.

Dioxygenases
Incorporate both atoms of molecular Oxygen into the
substrate. The basic reaction is shown below:
A + O2 → AO2
Examples of Dioxygenase
• Homogentisate oxidase
• Tryptophan Pyrrolase (Dioxygenase)
m

• Nitric Oxide Synthase.


co

Fig. 3.2: Enzyme substrate complex


MECHANISM OF ENZYME ACTION Fischer’s Template Theory
Explained by different theories: • Lock and key model
• Lowering of activation energy • Three dimensional structure of the active site of
• Michaelis–Menten theory unbound enzyme is complementary to the substrate
• Fischer’s Template theory • Thus Enzyme and substrate fit each other
• Failed to explain dynamic changes that accompany
• Koshland’s Induced Fit theory.
catalysis.
Lowering of Activation Energy
Koshland’s Induced Fit Theory
Enzymes lower the activation energy. Activation energy is
• Binding of substrate to specific part of enzyme
defined as the energy required to convert all molecules of
m

induce conformational changes in the active site of


a reacting substance from ground state to transition state.
co

the enzyme
• Enzyme changes shape during or after binding with
the substrate
• Can explain the dynamic changes that accompany
catalysis.

FACTORS AFFECTING ENZYME ACTIVITY


Temperature
• Raising the temperature increases the rate of both
uncatalyzed and enzyme-catalyzed reactions by
increasing the kinetic energy and the collision
m

frequency of the reacting molecules


co

• Bell shaped curve is obtained by plotting temperature


against velocity of reaction
• The optimum temperature for most human enzymes
Fig. 3.1: Lowering of activation energy is between 35 and 40°C
e

e
m

m
m
co

Enzymes | 103

• Human enzymes start to denature at temperatures Enzyme Concentration


above 40°C In the beginning velocity of enzyme reaction is directly
• Enzymes from humans generally exhibit stability at proportional to the enzyme concentration.
temperatures up to 45–55°C.
Temperature Coefficient (Q10)
The temperature coefficient (Q10) is the factor by which the rate of a
m

biologic process increases for a 10°C increase in temperature. For the


temperatures over which enzymes are stable, the rates of most biologi-
co

cal processes typically double for a 10°C rise in temperature (Q10 = 2)

Fig. 3.5: Effect of enzyme concentration


m
co

Substrate Concentration
• For a fixed Enzyme concentration, rate of reaction is
directly proportional to the substrate concentration
up to certain concentration of substrate, but later
Fig. 3.3: Effect of temperature there is no further increase in velocity
Hydrogen Ion Concentration • Hyperbolic curve is obtained.
• The rate of almost all enzyme-catalyzed reactions
exhibits a significant dependence on hydrogen ion
concentration
• Most intracellular enzymes exhibit optimal activity
at pH values between 5 and 9
m

• The relationship of enzyme activity to hydrogen ion


concentration gives a bell shaped curve.
co

Fig. 3.6: Effect of substrate concentation

Michaelis-Menten Equation
Vmax X S
m

Vi =
Km + S
co

• Where Vi is the initial velocity


• Vmax is the maximal velocity
• Km is the Michaelis Constant
Fig. 3.4: Effect of pH • S is the substrate concentration
e

e
m

m
m
co

104 |  
Self Assessment and Review of Biochemistry

MICHEALIS CONSTANT (Km)Q In Lineweaver burk Plot


• X intercept is –1/Km
• Substrate concentration required to produce half-
• Y intercept is Km/Vmax
maximal velocity (½ Vmax)
• Slope is Km/Vmax
Dixon Plot
Alternative to Lineweaver Burk Plot
m

1/V is measured at different concentration of inhibitor[I], but at same


substrate concentration[S].
co

ENZYME INHIBITION
Types of Enzyme Inhibition
• Competitive inhibition
• Noncompetitive inhibition
• Suicide inhibition
• Allosteric inhibition
• Feedback inhibition.
Competitive Inhibition
A type of inhibition in which the inhibitor compete
Fig. 3.7: Michaelis constant
m

directly with a normal substrate for an enzyme’s substrate


Characteristics of Michealis Constant
co

binding site.
• Independent of enzyme concentration
• Unique for each enzyme–substrate pair
• Constant for an enzyme—Substrate pair, hence called
signature of the enzyme
• Denotes affinity of enzyme to substrate. Lower the
Km higher will be the affinity of the substrate
• Km helps to understand the natural substrate of an enzyme
• Substrate with lower Km will be the natural substrate
of the enzyme.
Lineweaver Burk Plot
m

A graphical representation of 1/V on y axis and 1/S on x axis


is called Lineweaver Burk Plot or Double Reciprocal plot.
co

Fig. 3.9: Competitive inhibition

Features of competitive inhibition


• Inhibitor will be structural analogue of substrate
• Reversible
• Excess substrate abolishes inhibition
• Vmax remains the same
• Km increases.
Examples of Competitive Inhibition
m

Competitive Inhibitors of enzymes are mostly drugs


co

Drug Enzyme inhibited


Statins HMG CoA Reductase

Fig. 3.8: Line weaver burk plot Contd...


e

e
m

m
m
co

Enzymes | 105

Contd...
Noncompetitive Inhibitor Enzyme
Drug Enzyme inhibited
CyanideQNBE pattern Cytochrome C Oxidase
Dicoumarol Vitamin K Epoxide
Iodoacetate Glyceraldehyde 3 Phosphate
Methotrexate Dihydrofolate Reductase
Fluoride Enolase
Succinyl Choline Acetyl Choline Esterase
Disulfiram(Antabuse) Aldehyde Dehydrogenase
m

Some competitive inhibitors which are not drugs are British Anti-Lewisite (Dimercaprol) -SH group of several enzymes
co

Arsenite Alpha Ketoglutarate


Enzyme Substrate Competitive Inhibitor
Dehydrogenase
Lactate Dehydrogenase Lactate Oxamate
Fluoroacetate Aconitase
Aconitase Cisaconitate Transaconitate
Di Isopropyl Flourophosphate Serine Proteases
Succinate Dehydrogenase Succinate Malonate
HMG CoA Reductase HMG CoA HMG Remember
Almost all inhibitors of Electron Transport Chain are examples of
irreversible Noncompetitive inhibition.
Noncompetitive InhibitionQ
A type of inhibition in which the inhibitor bind to a site Suicide Inhibition
distinct from the substrate binding site. Two different • Otherwise called mechanism based inactivation
types are: • Special class of irreversible inhibition
1. Reversible Noncompetitive Inhibition (Only few • Inhibitors are relatively unreactive, until they bind
m

Noncompetitive inhibition are reversible) with the active site of specific enzyme
co

2. Irreversible Noncompetitive Inhibition (Most of • Once the inhibitor binds to the enzyme, by the action
Noncompetitive are irreversible). of the enzyme, it is converted to a potent inhibitor
• Irreversibly bind to the enzyme and inhibit the
enzyme.

Examples of Suicide Inhibition


• Allopurinol inhibit Xanthine Oxidase
‒ Allopurinol converted to alloxanthine which
irreversibly inhibit the enzyme.
• Treatment of Trypanosomiasis by Difluoromethyl
ornithine (DFMO) inhibit Ornithine Decarboxylase
• Aspirin acetylates the active site of Cyclooxygenase,
m

there by inhibiting prostaglandin synthesis.


co

Feed Back Inhibition


• Also called end-product inhibition
• The activity of the enzyme is inhibited by the final
product of the pathway, e.g. AMP inhibits first step
Fig. 3.10: Noncompetitive inhibition
in purine synthesis.
Features of noncompetitive inhibitionQ
• Inhibitor have no structural resemblance to substrate REGULATION OF ENZYMES
• Mostly Irreversible (Except a few reversible non- Regulation of Enzyme Quality (Intrinsic Catalytic
competitive inhibition) Efficiency):
• Excess substrate do not abolish the inhibition • Allosteric Regulation
• Km remains the same • Covalent modification.
m

• Vmax decreases. Regulation of Enzyme Quantity:


co

• Control of Enzyme Synthesis (By Induction and


Examples of Irreversible Noncompetitive Inhibition Repression)
• Are mostly poisonous agents • Control of Enzyme Degradation.
e

e
m

m
m
co

106 |  
Self Assessment and Review of Biochemistry

Allosteric Regulation Contd...

Allosteric Enzymes have a catalytic site where the sub- Enzyme Activator Inhibitor
strate binds and a separate site where a modifier binds. Aspartate Transcarbamoylase ATP CTP
• If the modifier is an inhibitor it is called allosteric HMG CoA Reductase Cholesterol
inhibition Phosphofructokinase Fructose 2,6 Citrate
• If the modifier is an activator it is called allosteric Bisphosphate
m

activation. Pyruvate Carboxylase Acetyl CoA ADP


co

Features of allosteric regulation Acetyl CoA Carboxylase Citrate Acyl CoA


• The modifier need not be a structural analogue of Citrate Synthase ATP
the substrate Carbamoyl Phosphate N Acetyl
• Partially reversible when excess substrate is added Synthetase I Glutamate (NAG)
• Most allosteric enzymes have a quaternary structure Carbamoyl Phosphate PRPP UTP
and made up of subunits Synthetase II
• Allosteric enzymes occupy key regulatory positions
in metabolic pathways, called key enzymes or rate Covalent Modification
limiting enzymes • Method of regulation of Enzyme activity
• Hill’s Equation describes the behavior of enzymes • Addition or removal of a group by making or
that exhibit cooperative binding of the substrate breaking a covalent bond
• Sigmoid shape of the curve is due to Cooperative • By covalent modification enzyme activity is either
m

Binding. increased or decreased.


co

Cooperative BindingQ Two types of covalent modification


Binding of substrate to one site increases the affinity of binding of the
substrate to substrate binding site in other subunits. 1. Irreversible: Partial proteolysis/zymogen activationQ
2. Reversible: Addition/removal of a particular group.
Two types of allosteric enzymes Common methods of reversible covalent modificationQ
1. K Series: In K series allosteric enzyme, Km is raised • Phosphorylation/dephosphorylation [Most Common
but Vmax remains the same. covalent modification]
2. V series: In V series allosteric enzyme, Vmax decreases
• Methylation
and Km remains the same.
• Adenylation
Comparison of Allosteric regulation and Non-
competitive Inhibition Similarities • ADP ribosylation
• Modifier/Inhibitor is not a structural analogue of the • Acetylation.
m

substrate Examples of covalent modifications


• Modifier/inhibitor binds to a site distinct from Usually enzymes are in phosphorylated state when body
co

substrate binding site is fasting under the influence of Glucagon. The enzymes
• V series of Allosteric Enzymes and Noncompetitive that are active in phosphorylated state are
inhibition, the Km remains the same and V max • Glycogen Phosphorylase
decreases. • Key enzymes of Gluconeogenesis
Differences • Citrate Lyase
Noncompetitive Inbhibition Allosteric Inhibition • Phosphorylase Kinase
Follow Michaelis-Menten Kinetics Does not follow Michaelis-Menten
• HMG CoA Reductase Kinase.
Kinetics Usually enzymes are in dephosphorylated state when the
Effect of Substrate concentration Effect of Substrate concentration body is in well fed state under the influence of Insulin.
give a hyperbolic curve give a sigmoid curve The enzymes that are active in dephosphorylated state are
• Glycogen synthase
m

Examples of Allosteric EnzymesQ • Key enzymes of Glycolysis


co

Enzyme Activator Inhibitor • Acetyl CoA carboxylase


ALA Synthase Heme • Pyruvate dehydrogenase
Contd... • HMG CoA reductase.
e

e
m

m
m
co

Enzymes | 107

Protein Acetylation: A Ubiquitous Covalent Modification of • They differ in Km value.


Metabolic Enzymes e.g.: Glucokinase (an isoenzyme of Hexokinase) has
• In recent years, it is found that thousands of other mammalian high Km and but Hexokinase has low Km.
proteins are subject to modification by covalent acetylation,
including nearly every enzyme present in key metabolic pathways
• Acetylation-deacetylation, on the other hand, targets multiple Functional Enzymes and
proteins in a pathway, unlike other covalent modifications. Nonfunctional EnzymesQ
m

• It has been hypothesized that the degree of acetylation of metabolic


enzymes is modulated to a large degree by the energy status of
Functional EnzymesQ
co

the cell. Enzymes which have specific function in the plasma.


• In a well-nourished cell, high level of Acetyl CoA would promote
lysine acetylation. Examples of functional enzymes
• When nutrients are lacking, acetyl CoA levels drop and the ratio • Coagulation Factors
of NAD+/NADH rises, favoring protein deacetylation.
• Lipoprotein Lipase.
Acetylation
• Lysine acetyltransferases catalyze the transfer of the acetyl group
of acetyl-CoA to the ε-amino groups of lysyl residues, forming Nonfunctional Enzymes
N-acetyl lysine. • No specific function in the plasma
• In addition, some proteins, particularly those in the mitochondria, • Comes out from the tissue as a result of normal wear
become acetylated by reacting
• With acetyl-CoA directly, i.e. without the intervention of an enzyme
and tear
catalyst. • Their level is very low in the serum
Deacetylation • But during tissue injury their level rises in the serum
m

Two classes of protein deacetylases have been identified:


• Hence they help to diagnose the site of tissue injury.
co

• Histone deacetylases and sirtuins.


• Histone deacetylases catalyze the removal by hydrolysis of acetyl Examples of nonfunctional enzymes
groups, regenerating the unmodified form of the protein and
acetate as products. • LDH, Creatine Kinase, Alkaline Phosphatase
• Sirtuins, on the other hand, use NAD+ as substrate, which yields • Does not follow Michaelis–Menten hyperbolic
O-acetyl ADP-ribose and nicotinamide as products in addition to kinetics, instead gives sigmoid Curve.
the unmodified protein.
Isoenzymes of Lactate Dehydrogenase (LDH)
There are five isoenzymes for LDH. It is a tetramer made
ISOENZYMES
up of two types of subunits, H and M.
• Physically distinct forms of the same enzymes, but Name of the isoenzyme Subunit Tissue localization
catalyse the same reaction LDH-1 H4 Heart Muscle
m

• Different molecular forms of the same enzyme LDH-2 H3M1 RBC


co

synthesized from same/various tissues. LDH-3 H2M2 Brain

Features of Isoenzymes LDH-4 HM3 Liver and Skeletal Muscle

• They catalyse the same chemical reactions. They LDH-5 M4 Liver and Skeletal Muscle
differ in heat stability.
But LDH-5 predominates in Liver
e.g.: heat stable ALP and heat labile ALP.
LDHx
• They differ in electrophoretic mobility. A sixth atypical LDH found in male genital tissues.
e.g.: CK-I moves faster than CK-3
• They differ in the susceptibility to an inhibitor.
Isoenzymes of Creatine Kinase (CK)
e.g.: tartarate labile Acid Phosphatase and tartarate
There are three isoenzymes for Creatine Kinase, made up
stable Acid Phosphatase.
of two types of subunits, M and B.
• They differ in subunits they are made up of.
m

Name of the isoenzyme Subunit Tissue localization


e.g.: LDH-1 (H4), LDH 5 (M4)
co

CK-1 BB Brain
• They differ in tissue localisation.
CK-2 MB Heart
e.g.: LDH-1 located in the heart and LDH-5 located
CK-3 MM Skeletal Muscle
in the Muscle.
e

e
m

m
m
co

108 |  
Self Assessment and Review of Biochemistry

Two atypical Creatine Kinases are • Aminotransferases (transaminases) are sensitive


1. CK Macro (Macro–CK): Formed by aggregation of indicators of liver cell injury
CK-BB with immunoglobulins like IgG. • Are most helpful in recognizing acute hepatocellular
2. CK-Mi (Mitochondrial CK): Found in the exterior diseases such as hepatitis
surface of Inner mitochondrial membrane of muscle, ‒ Aspartate aminotransferase (AST) (is found
live and brain. in liver, cardiac muscle, skeletal muscle,
m

Isoenzymes of Alkaline Phosphatase (ALP) kidney, brain, pancreas, lungs, leukocytes, and
co

erythrocytes)
Isoenzyme forms of ALP Tissue of origin
‒ Alanine aminotransferase (ALT) (is found
Alpha-1 ALP Synthesized by epithelial cells of Biliary primarily in the liver and is therefore a more
Canaliculi
specific for Liver Disease than AST)
Alpha-2 Heat labile ALP Produced by Hepatic Cells
• Enzymes whose elevation in serum reflects cholestasis
Alpha-2 Heat stable ALP Produced by Placenta
Inhibited by Phenylalanine ‒ Alkaline phosphatise (ALP)
Pre beta ALP Produced by Osteoblast ‒ 5’-nucleotidase
Gamma ALP Produced by Intestinal Cells ‒ γ glutamyltransferase (Transpeptidase) (GGT).
Leukocyte ALP By leukocytes
Remember
GGT
Regan Isoenzyme
m

• Named after the first patient from which the enzyme isolated • GGT elevation in serum is less specific for cholestasis than are
co

• An isoenzyme of ALP closely resemble alpha-2 heat stable ALP elevations of alkaline phosphatase or 5’-nucleotidase
• Otherwise called Carcino Placental Isoenzyme • GGT is used to identify occult alcohol use.
• Elevated in Carcinoma of Lung, Liver, intestine.
Nagao Isoenzyme ALP
• A variant of Regan isoenzyme • Less than three fold elevation in ALP can be seen in any type of
• Inhibited by L-leucine. liver disease
• More than four fold elevation of ALP is seen in cholestasis
Cardiac Biomarkers • ALP elevation is not helpful in distinguishing between intrahepatic
and extrahepatic cholestasis.
• Creatine Kinase [CKMB]
5’ Nucleotidase
• Cardiac Troponin I [CTnI]
Specific for cholestasis than ALP and GGT.
• Cardiac Troponin T [CTnT]
• Brain Natriuretic Peptide [BNP]: Marker of cardiac
failure not a marker of Myocardial Infarction Nonpathologic elevations of ALP seen in:
m

• Myoglobin • Patients over age 60 years


co

• Lactate Dehydrogenase [LDH]: Not used nowadays • Type O and Type B blood group
• Aspartate amino Transferase [AST]: Not used nowadays. • After eating a fatty meal (due to efflux of gamma ALP)
Flipped Pattern of LDHQ • In children due to rapid bone growth
Normally LDH-2 is present in higher concentration than LDH-1. But
this pattern is reversed in MI. This limited diagnostic importance. • Late normal pregnancy.

AST/ALT Ratio.Q
New Cardiac Biomarkers AST:ALT ratio < 1
• Ischemia Modified Albumin Any condition causing hepatocellular damage ratio <1 is seen as ALT
level rises above AST level. This is because ALT is more specific for
• Glycogen Phosphorylase BB Isoenzyme. hepatocellular damage than AST.
• Pregnancy Associated Plasma Protein A (PAPP-A) • Chronic viral hepatitis
• Myeloperoxidase (MPO).
m

• Nonalcoholic fatty liver disease


co

• Toxic hepatitis
ENZYME PROFILE FOR LIVER DISEASES • Paracetamol toxicity.
• Enzymes whose elevation in serum reflects damage AST: ALT ratio >2:1 is suggestive, while a ratio >3:1
to hepatocyte • Highly suggestive of alcoholic liver disease.
e

e
m

m
m
co

Enzymes | 109

Contd...
Remember
Aminotransferases (ALT and AST) • Alkaline Phosphatase
• ALT is more specific for hepatocellular damage than AST • α-Glutathione S-Transferase
• In hepatocellular disease ALT elevation is slightly higher than or • γ Glutamyltranspeptidase
equal to AST, so AST/ALT ratio is less than 1
• β2Microglobin
• If cirrhosis develop the ratio becomes more than 1
• α-1-Macroglobin
• Minimal ALT elevation less than 300 IU/L is nonspecific. Most likely
m

explanation is Fatty liver • Retinol Binding Protein


• Cystatin C
co

• Level > 1000 IU/L in extensive hepatocellular injury


• In alcoholic liver disease, alcohol-induced deficiency of Pyridoxal • Microalbumin
Phosphate causes reduced level of transaminases (ALT and AST) • Osteopontin
ALT level is often normal and AST is rarely > 300 IU/L. • Liver Fatty Acid Binding Protein
• Sodium–Hydrogen Exchanger Isoform
ENZYME PROFILE IN PROSTATE CANCER • Exosomal Fetuin.

Tartrate Labile Acid Phosphatase, Prostate specific


antigen (PSA) are enzyme markers of Prostate Cancer. MARKERS OF BONE DISEASES
Prostate specific antigen
• Otherwise called Kallikrein related Peptidase 3 Bone Formation
(KLK3) • Serum Bone-specific Alkaline Phosphatase (BAP)
• It is a Serine Protease • Serum Osteocalcin
m

• Secreted by epithelial cells of Prostate • Serum Propeptide of type I Procollagen.


co

• This is Prostate specific, but not prostate cancer


specific Bone Resorption
• Commonly used cut point for Prostate cancer is PSA • Urine and serum cross linked N-telopeptide
level > 4 ng/mL • Urine and Serum cross linked C-telopeptide
• But actually there is no PSA level below which risk • Urine total free deoxypyridinoline.
of Prostate cancer is Zero
• So PSA level estimation should be accompanied by
Prostate Biopsy. SERINE PROTEASES
• Proteolytic Enzymes with Serine at their active site
ENZYME PROFILE PANCREATITIS
• Amino Acid triad in the active site of Serine Proteases-
They are: Ser, His, Asp.
m

• Amylase
co

• Lipase Examples of Serine Proteases


Serum Amylase is not specific for Pancreatic disease as • Chymotrypsin
its level is increased in Parotitis also. • Trypsin
A serum lipase level measurement can be instrumental • Elastase
in differentiating a pancreatic or nonpancreatic cause for • Thrombin
hyperamylasemia.
• Plasmin
Apart from serum Amylase level can be estimated in
• Complements
urine also.
• Factor X and XI
Biomarkers of Acute Kidney Injury • Prostate Specific Antigen.
Novel Biomarkers of Acute Kidney InjuryQQQ
• Kidney Injury Molecule-1 (KIM-1) Serine Proteases Differ in Substrate SpecificityQ
m

• Neutrophil Gelatin associated Lipocalin (NGAL)


• Trypsin cleave Basic Amino Acid
co

• IL-18
• Alanine Amino Peptidase • Chymotrypsin cleave hydrophobic bulky amino Acid
• Clusterin • Elastase cleave Small neutral amino Acids like
Contd... Alanine, Glycine.
e

e
m

m
m
co

110 |  
Self Assessment and Review of Biochemistry

BI-BI REACTION Contd...


Membrane/Organelle Marker enzymes
• Kinetic behavior for two-substrate, two-product
Lysosomes Cathepsin
reactions termed ‘Bi-Bi’ reactions
Cytoplasm Lactate Dehydrogenase
• Most Bi-Bi reactions follow Michaelis-Menten
kinetics.
Bi-bi reactions can be divided into ENZYMES AS DIAGNOSTIC REAGENTS
m

• An ordered Bi-Bi reaction


co

For example: NAD (P) H-dependent oxidoreductases Enzyme Diagnostic test done

• A random Bi-Bi reaction Urease Urea Estimation

For example: Many kinases and some dehydrogenases. Uricase Uric Acid Estimation
Glucose Oxidase Glucose
• A ping-pong reaction
Hexokinase Glucose
For example: Aminotransferases and Serine
proteases.Q Peroxidase Q
Glucose, Cholesterol
Cholesterol Oxidase Cholesterol

ENZYMES AS MARKERS OF ORGANELLE Creatininase Creatinine

AND MEMBRANESQQQ Lipase Triglycerides

Membrane/Organelle Marker enzymes


m

Plasma Membrane 5’- Nucleotidase Adenylyl Cyclase ENZYMES IN OTHER BODY FLUIDSQ
co

Na+-K+ ATPase
Enzyme Clinical use
Endoplasmic reticulum Glucose-6-phosphatase
Lactate dehydrogenase in csf, Suggestive of malignant tumor
Golgi Complex Galactosyl TransferaseQ pleural fluid, ascitic fluid but not confirmatory
Inner Mitochondrial Membrane ATP Synthase Adenosine deaminase in pleural Suggestive of tuberculous pleural
Peroxisome Catalase, Urate Oxidase fluid effusion
Amylase in urine Suggestive of pancreatitis
Contd...

REVIEW QUESTIONS
m

GENERAL ENZYMOLOGY • Involved in prostanoid (prostaglandin. Thromboxane


co

and Prostacyclin) synthesis


Classification of Enzymes, Enzyme Kinetics
• This enzyme has two activities, a cyclooxygenase
1. Suicidal Enzyme is: (AIIMS May 2013) and peroxidase
a. Lipoxygenase • COX is present as two isoenzymes, COX-1 and
b. Cyclooxygenase COX-2.
c. Thromboxane Synthase
d. 5’ Nucleotidase Drugs acting as inhibitors COX
Ans. b. Cyclooxygenase. (Ref: Harper 30/e p241) • NSAID
Cyclooxygenase is a ‘Suicide Enzyme’ ‒ Aspirin: Inhibits COX-1 and COX-2.
‘Switching off’ of prostaglandin activity is partly achieved ‒ Indomethacin and ibuprofen-inhibit cyclooxyge-
by a remarkable property of cyclooxygenase—that of nases by competing with arachidonate.
self-catalyzed destruction; i.e. it is a ‘suicide enzyme.’
m

‒ Coxibs: Selectively inhibit COX-2


co

Quick review-cyclooxygenase ‒ Some coxibs have been withdrawn or suspended


• Cyclooxygenase (COX) (also called prostaglandin from the market due to undesirable side effects
H synthase) and safety issues.
e

e
m

m
m
co

Enzymes | 111

• Anti-inflammatory Corticosteroids • Catalase


‒ Transcription of COX-2—but not of COX-1—is • Peroxidase.
completely inhibited by corticosteroids. Dehydrogenase
Catalyse removal of hydrogen from the substrate. They
2. Which of the following is a Lyase? (JIPMER 2014) cannot use oxygen as a hydrogen acceptor.
a. Aldolase B
Oxidases
b. Acetyl Co ASynthetase
m

Removal of hydrogen from a substrate with oxygen as


c. Fatty Acyl CoA Dehydrogenase
co

the acceptor of hydrogen.


d. Acetyl CoA Carboxylase • Monoamine Oxidase
Ans. a. Aldolase B. • Cytochrome Oxidase
Some examples of Lyases are: • Xanthine Oxidase.
• HMG CoA Lyase Oxygenases
• Argininosuccinate Lyase Catalyze the direct transfer and incorporation of oxygen
• ATP Citrate Lyase into a substrate molecule.
• Aldolase Monooxygenases [Mixed Function Oxidases or
• Fumarase. Hydroxylase]
Incorporate one atom of molecular oxygen into the
3. All are true about oxygenases, except: substrate:
(AIIMS Nov 2011) • Phenylalanine Hydroxylases
m

a. Can incorporate 2 atoms of O2 in a substance • 7 alpha Hydroxylases


co

b. Can incorporate 1 atom of O2 in a substance • Cytochrome p450.


c. Important in hydroxylation of steroids Dioxygenases
d. Catalyse carboxylation of drugs Incorporate both atoms of molecular oxygen into the
Ans. d. Catalyse carboxylation of drugs. substrate. The basic reaction is shown below:
(Ref: Harper 30/e p118, 119) A + O2 → AO2
• Oxygenases can be monooxygenase or dioxygenase Examples of Dioxygenase
• Monooxygenase incorporate 1 atom of Oxygen • Homogentisate oxidase
molecule to the substrate • Tryptophan Pyrrolase (Dioxygenase)
• Dioxygenase incorporate both the atoms of Oxygen • Nitric Oxide Synthase.
molecule to the substrate 5. Enzyme which cleave C-C bond:
• Phase 1 Xenobiotic reactions, hydroxylation, cata- a. Lyase
m

lyzed mainly by members of a class of enzymes b. Oxidoreductase


co

referred to as monooxygenases or cytochrome P450s. c. Ligase


4. All of the following enzymes are involved in d. Isomerase
oxidation-reduction, except: (AI 2009) Ans. a. Lyase. (Ref: Harper 30/e p61)
a. Dehydrogenases 6. Velocity at Km is:
b. Hydrolases a. Half the substrate concentration
c. Oxygenases b. Same as V max
d. Peroxidases c. Quarter the V max
Ans. b. Hydrolases. (Ref: Harper 30/e p118, 119) d. Half the V max
Oxidoreductases Ans. d. Half the V max.
Can be: 7. Coenzyme in decarboxylation reaction:
• Dehydrogenase a. Niacin
m

• Oxygenase b. Biotin
co

• Monooxygenase c. Pyridoxine
• Dioxygenase d. Riboflavin
• Oxidase Ans. c. Pyridoxine. (Ref: Harper 30/e p39, 40)
e

e
m

m
m
co

112 |  
Self Assessment and Review of Biochemistry

• Coenzyme for carboxylation reaction—Biotin c. High Km low affinity


• Coenzyme for decarboxylation—Pyridoxine d. Low Km low affinity
• Coenzyme for decarboxylation reactions of Alpha Ans. c. High Km and lower affinity.
Ketoglutate dehydrogenase and branched chain Significance of Km (Michaelis Constant):
Ketoacid dehydrogenase—Thiamine. • Km is substrate concentration at 1/2 Vmax
Enzyme Inhibition • Constant for an enzyme substrate pair
m

• It is called signature of the enzyme


co

8. The type of enzyme inhibition in which Succinate • Higher the Km, lower is the affinity of the enzyme
dehydrogenase reaction is inhibited by malonate towards the substrate
is an example of: (AIIMS May 2006) • Lower the Km, higher is the affinity of the enzyme
a. Noncompetitive towards the substrate.
b. Uncompetitive
Features of competitive inhibition:
c. Competitive
• Km increases, hence the affinity is lowered
d. Allosteric
• Vmax remains the same.
Ans. c. Competitive. (Ref: Harper 30/e p78)
Examples of competitive inhibition Features of noncompetitive inhibition:
Competitive inhibitors of enzymes are mostly drugs • Km remains the same
Drug Enzyme inhibited • Vmax decreases.
m

Statins HMG CoA Reductase 11. Noncompetitive enzyme inhibition leads to:
co

Dicoumarol Vitamin K Epoxide. a. Vmax ↑


Methotrexate Dihydrofolate Reductase b. Vmax ↓
Succinyl Choline Acetyl Choline Esterase c. Vmax unchanged
d. Km ↑
Some competitive inhibitors which are not drugs are e. Km ↓
Enzyme Substrate Competitive inhibitor Ans. b. Vmax ↓ . (Ref: Harper 30/e p81)
Lactate Dehydrogenase Lactate Oxamate Features of Noncompetitive Inhibition
Aconitase Cisaconitate Transaconitate • Inhibitor have no structural resemblance to substrate
Succinate Dehydrogenase Succinate Malonate • Irreversible
• Excess substrate do not abolish the inhibition
HMG CoA Reductase HMG CoA HMG
• Km remains the same
m

• Vmax decreases.
co

9. Features of competitive inhibition is/are:


a. Vmax increases (PGI May 2014) 12. True about competitive inhibition of enzyme:
b. Km increases a. ↑ Km (PGI May 2010)
c. Vmax decreases b. ↓ Km
d. Km decreases c. ↑ Vmax
e. Vmax constant d. No change in Km and Vmax
Ans. b. Km increases, e. Vmax constant. e. Vmax remain same
Features of competitive inhibition are: Ans. a. ↑ Km, e. Vmax remain same. (Ref: Harpers 30/e p81)
• Vmax remains a constant
13. Noncompetitive reversible inhibitors:
• Km increases a. Raise Km
• Reversible. b. Lower Km
m

c. Lower Vmax
co

10. Which is true about enzyme kinetics for


competitive inhibition: (JIPMER 2014) d. Raise both Vmax and Km.
a. Low km high affinity e. Do not affect either Vmax or Km
b. High km high affinity Ans. c. Lower Vmax . (Ref: Harper 30/e p79)
e

e
m

m
m
co

Enzymes | 113

14. True about competitive antagonism: c. ADP Ribosylation


a. Vmax increased (PGI June 2009) d. Glycosylation
b. Substrate analogue Ans. d. Glycosylation. (Ref: Harper 29/e p89)
c. Reversible Reversible covalent modifications
d. Km increased • Phosphorylation Dephosphorylation
e. Vmax decreased • ADP Ribosylation
m

Ans. b, c, d. (Ref: Harper 30/e p78-79) • Methylation


co

15. Km changes and Vmax remains the same. What is • Acetylation.


the type of Enzyme inhibition? Irreversible Covalent Modification
a. Competitive Inhibition • Zymogen activation/ Partial Proteolysis.
b. Noncompetitive Inhibition
18. The following affect enzyme activity except:
c. Uncompetitive inhibition
a. Methylation
d. Suicide Inhibition
b. Acetylation
Ans. a. Competitive Inhibition. (Ref: Harper 30/e p80)
c. Induction
• Competitive inhibition—Km increases and Vmax
d. Phosphorylation
remains the same
Ans. c. Induction. (Ref: Harper 30/e p89, 90)
• Noncompetitive inhibition—Km remains the same
Induction is method of regulation of enzyme quantity.
m

and Vmax decreases


• Uncompetitive inhibition—Both Km and Vmax dec- Regulation of Enzymes
co

reases. Can be classified as:


Regulation of Enzyme Quality (Intrinsic Catalytic
Enzyme Regulation Efficiency)
16. Which enzymes catalytic activity is by • Allosteric Regulation
dephosphorylation (PGI November 2009) • Covalent modification.
a. HMG CoA reductase Covalent modification
b. Glycogen Phosphorylase • Phosphorylation/dephosphorylation [Most Common
c. Citrate Lyase covalent modification]
d. Glycogen Synthase • Methylation
Ans. a. HMG CoA reductase, d. Glycogen Synthase. • Adenylation
m

(Ref: Harper 30/e p91, Table 9.1) • ADP ribosylation


co

Activity state • Acetylation.


Enzyme Low High
Regulation of enzyme quantity
Acetyl-CoA carboxylase EP E
Control of Enzyme Synthesis
Glycogen synthase EP E
By Induction and Repression
Pyruvate dehydrogenase EP E
Control of Enzyme Degradation Enzymes are degraded
HMG-CoA reductase EP E
by Ubiquitin—Proteasome pathway.
Glycogen phosphorylase E EP
Citrate lyase E EP 19. Chymotrypsinogen is a:
Phosphorylase b kinase E EP a. Zymogen
HMG-CoA reductase kinase E EP b. Carboxypeptidase
c. Transaminase
m

17. All of the covalent modification regulate enzyme d. Exopeptidase


co

kinetics except: Ans. a. Zymogen.


a. Phosphorylation Zymogen activation is an example of irreversible covalent
b. Acetylation modification.
e

e
m

m
m
co

114 |  
Self Assessment and Review of Biochemistry

Serine Proteases c. Phospholipase


20. Chymotrypsin cleaves carbonyl terminal of: d. Elastase
a. Phenylalanine (PGI May 2011) Ans. a. Serine Protease. (Ref: Harper 30/e p63)
b. Arginine
c. Lysine ENZYME AS MARKERS OF
d. Tryptophan ORGANELLE AND MEMBRANE
m

e. Tyrosine
co

Ans. a, d, e. (Ref: Harper 30/e p64) 24. Markers of plasma membrane is/are:
Serine Proteases a. Galactosyltransferase (PGI June 2009)
• Proteolytic Enzymes with Serine at their active Site b. 5’nucleotidase
• Amino acid triad in the active site of Serine Proteases: c. Adenyl cyclase
Ser, His, Asp. d. ATP synthetase
Examples of serine proteases e. Tyrosine
• Chymotrypsin Ans. b. 5’nucleotidase, c. Adenylyl cyclase.
• Trypsin (Ref: Harper 30/e, Table 40.2)
• Elastase Enzymes as markers of organelle and membranes
• Thrombin Enzymatic markers of different membranesa
• Plasmin
m

Membrane Enzyme
• Complements
co

Plasma 5”-Nucleotidase
• Factor X and XI.
Adenylyl cyclase
Serine proteases differ in substrate specificity
Na+-K+-ATPase
• Trypsin cleave Basic Amino Acid, like Arg, Lys
Endoplasmic reticulum Glucose-6-Phosphatase
• Chymotrypsin cleave Hydrophobic Bulky Amino
Golgi apparatus
Acid like Trp, Tyr, Phe
Cis GlcNAc transferase I
• Elastase cleave Small neutral AminoAcids like
Alanine, Glycine. Medial Golgi mannosidase II
Trans Galactosyl transferase
21. Trypsin cleaves: (PGI May 2010) Trans Golgi Network Sialyl transferase
a. Arginine
Inner mitochondrial membrane ATP synthase
b. Glutamate
m

c. Lysine 25. Marker enzyme for Golgi apparatus:


co

d. Proline a. Galactosyltransferase
Ans. a. Arginine, c. Lysine. (Ref: Harper 30/e p63) b. Glucose 6 Phosphatase
22. A common feature of all serine proteases is: c. 5’ Nucleotidase
(AI 2006) d. Catalase
a. Autocatalytic activation of zymogen precursor Ans. a. Galactosyltransferase.
b. Tight binding of pancreatic trypsin inhibitor
Clinical Enzymology
c. Cleavage protein on the carboxyl site of serine
residues 26. True about isoenzymes is: (AIIMS Nov 2011)
d. Presence of Ser-His-Asp catalytic triad at the a. Catalyse the same reaction
active site b. Same quaternary structure
Ans. d. Presence of Ser-His-Asp catalytic triad at the c. Same distribution in different organs
m

active site. (Ref: Harper 30/e p62, 63)


d. Same enzyme classification with same number
co

23. Trypsin is a: and name


a. Serine protease Ans. a. Catalyse the same reaction.
b. Lecithinase (Ref: Harper 30/e p63)
e

e
m

m
m
co

Enzymes | 115

• Isoenzymes catalyse the same reaction. For example, Contd...


LDH1-5 all convert Pyruvate to lactate Enzyme Diagnostic test done
• They have different quaternary structure. For Cholesterol Oxidase Cholesterol
example, the subunits in LDH-1 is different from Creatininase Creatinine
LDH-2
Lipase Triglycerides
• Tissue distribution of each isoform is different
m

• Enzyme name and number can be different. 29. Which of the following estimates blood creatinine
co

level most accurately: (AIIMS May 2006)


27. Nonfunctional enzymes are all except: a. Jaffe method
a. Alkaline phosphatase (AIIMS Nov 2008)
b. Kinetic Jaffe method
b. Acid phosphatese
c. Technicon method
c. Lipoprotein lipase
d. Enzyme assay
d. Gamma glutamyltranspeptidase
Ans. d. Enzyme Assay
Ans. c. Lipoprotein lipase.
(Ref: Varley’s Practical Clinical Biochemistry 6/e p352)
(Ref: Vasudevan and Sreekumari 7/e p301)
Estimation of blood creatinine
Two Methods:
FUNCTIONAL ENZYMES AND
NONFUNCTIONAL ENZYMES Chemical Method-Based of Jaffe’s Test
In alkaline medium creatinine form a red colored tautomer
Functional enzymes
m

of Creatinine picrate which is measured colorimetrically.


• Enzymes which have specific function in the plasma.
co

This method can be automated in autoanalysers and


Examples of functional enzymes Kinetic method can be used.
• Coagulation Factors • Kinetic Jaffe more accurate than Jaffe’s Method.
• Lipoprotein Lipase. Enzymatic method
• By employing two enzymes Creatininase or Creatinine
Nonfunctional enzymes Deaminase
• No specific function in the Serum • More specific
• Comes out from the tissue as a result of normal wear • No interference by Ketones, Bilirubin or Glucose
and tear
• Hence measure Creatinine accurately.
• There level is very low in the serum
• But during tissue injury their level rises in the serum 30. Not raised in liver disorder is/are:
a. Lipase (PGI may 2013)
• Hence they help to diagnose the site of tissue injury.
m

b. Urease
Examples of nonfunctional enzymes are LDH, Creatine
co

Kinase, Alkaline Phosphatase. c. ALP


d. AST
28. Peroxidase enzyme is used in estimating: e. ALP
a. Hemoglobin (AIIMS Nov 2007) Ans. a. Lipase, b. Urease.
b. Ammonia (Ref: Harper 29/e p721, 722, Table 48.6)
c. Creatinine
Aspect of Liver
d. Glucose Test Major Utility
Function Assessed
Ans. d. Glucose. Serum bilirubin Indicator of the ability Aids in the differential
(Ref: Vasudevan and Sreekumari 7/e p309) levels (total and of the liver to conjugate diagnosis of jaundice
conjugated) and excrete bilirubin
Enzyme Diagnostic test done (conjugation and
Urease Urea Estimation excretory function)
m

Total serum Measure of the Indicator of severity of


Uricase Uric Acid Estimation
protein and biosynthetic function chronic liver disease
co

Glucose Oxidase Glucose albumin of the liver, as the


liver is the primary site
Hexokinase Glucose
of synthesis of most
Peroxidase Glucose, Cholesterol plasma proteins.
Contd... Contd...
e

e
m

m
m
co

116 |  
Self Assessment and Review of Biochemistry

Contd... c. Muscle
Test
Aspect of Liver
Major Utility
d. RBC
Function Assessed
Ans. a. Liver. (Ref: Harper 29/e p66)
Prothrombin Measure of the Indicator of severity of
Lactate Dehydrogenase (LDH) is a tetrameric enzyme
time biosynthetic function acute liver disease
of the liver, as several consisting of two monomer types: H (for heart) and M
coagulation factors are (for muscle) that combine to yield five LDH isozymes:
m

synthesized in the liver HHHH (I1), HHHM (I2), HHMM (I3), HMMM (I4), and
co

Serum enzymes: MMMM (I5). Tissue-specific expression of the H and M


a. Aspartate Serves as marker of Activities of serum genes determines the relative proportions of each subunit
transaminase injury to hepatocytes AST and ALT are in different tissues. Isozyme LDH-I predominates in heart
(AST) that contain AST in early indicators of liver
abundance damage. They also tissue, and isozyme LDH-5 in the liver. Thus, tissue injury
help in monitoring releases a characteristic pattern of LDH isozymes that can
response to treatment be separated by electrophoresis.
b. Alanine Serves as marker of
transaminase injury to hepatocytes 32. Which of the following LDH is having fastest
(ALT) that contain ALT in electrophoretic mobility? (CMC Ludhiana 2014)
abundance
a. LDH-1
c. Alkaline Serves as marker of Aids in diagnosis of
phosphatase biliary obstruction obstruction of the
b. LDH-2
(ALP) biliary tract c. LDH-3
m

Blood ammonia Indicator of the ability Levels are elevated in d. LDH-5


co

of the liver to detoxify cirrhosis of liver with Ans. a. LDH-1.


ammonia portal hypertension
and in disorders of the • LDH having fastest electrophoretic mobility is LDH-1
urea cycle • LDH having least electrophoretic mobility is LDH-5
• Creatine Kinase having fastest electrophoretic
31. LDH-5 level elevated in which cell injury: mobility is CK-1
a. Liver • Creatine Kinase having least electrophoretic mobility
b. Heart is CK-3.
m
co
m
co
e

e
m

m
2
m
co
m
co

Section Carbohydrates
m
co

C H A P T E R S

4. Chemistry of Carbohydrates
m
co

5. Metabolism of Carbohydrates
m
co
co co co co co
m m m m m
m
e

m
e

m
e

m
e
e

e
m

m
m
co

4 Chemistry of Carbohydrates
m
co

Topics Included
• Classification of Carbohydrates • Reactions of Carbohydrates
• Glycosaminoglycans • Isomerism in Carbohydrates
• Mucopolysaccharidoses
m
co

Carbohydrates are the most abundant organic molecules Depending on the no. of carbon atoms, monosaccharides
in nature. The word ‘carbohydrate’ literally means are subclassified into:
hydrate of carbon.
Number of carbon atoms Generic name
3 Trioses
DEFINITION 4 Tetroses
Aldehyde or Keto derivatives of Polyhydric Alcohols or 5 Pentoses
compounds which yield these derivatives on hydrolysis. 6 Hexoses

General Formula for CarbohydratesQ is Cn(H2O)n , where n = no. 7 Heptoses


of carbon atoms 9 Nanoses

Main (Primary) source of energy for human beings


m

Depending on the functional group, monosaccharides


is Carbohydrates, as Glycogen. (45–65% of total are classified into:
co

energy)(PGI June 08)


• Aldoses with Aldehyde group
• Ketoses with Keto group.
CLASSIFICATION OF CARBOHYDRATES Important monosaccharides based on functional group
are:
Carbohydrates are classified into:
• Monosaccharides Generic name Aldoses Ketoses
• Disaccharides Triose Glyceraldehyde Dihydroxyacetone
• Oligosaccharides
Tetrose Erythrose Erythrulose
• Polysaccharides
Pentose Ribose Ribulose
Xylose (Epimer of Ri- Xylulose (Epimer of
Monosaccharides [Cn(H2O)n] bose) Arabinose Ribulose]
m

• Sugars which cannot be further hydrolyzed. They Hexose Glucose, Galactose, Fructose
co

contain one sugar unit Mannose

• Building blocks of all carbohydrates. Heptose Sedoheptulose


e

e
m

m
m
co

120 |  
Self Assessment and Review of Biochemistry

High-yielding Points—Monosaccharides Organs whose Major Energy Source is Glucose


• Simplest Carbohydrates of biological interest are: Glyceraldehyde • Brain
and Dihydroxyacetone. • Renal Medulla
• Cornea
• Pentoses which constitute a part of Nucleic Acid are Riboses and • Retina
Deoxyriboses • Testis
• Nanoses with biologic significance is Neuraminic Acid. • RBC
m

Remember
co

Sialic Acid • The only metabolic fuel for mature erythrocytes in fed state
and starving state is GlucoseQ AIIMS May 2015
• N-Acyl or O-Acyl derivative of Neuraminic Acid
• N-Acetyl Neuraminic Acid (NANA) is the predomi- Galactose
nant Sialic Acid
• Constituent of LactoseQ (Milk sugar)
• Constituents of both Glycoprotein and Ganglioside. • Synthesized in the mammary gland for synthesis of
lactose
RING STRUCTURES OF MONOSACCHARIDES • Part of Glycoprotein, Glycosaminoglycan in Proteo-
glycans and GlycolipidsQ.
Monosaccharide molecules of 4, 5 or 6 carbon atoms are
quite flexible, and this flexibility brings aldehyde or keto Mannose
group in close proximity to other hydroxyl groups on the • Isolated from plant mannans, hence the name
m

same molecule. • Occurs in Glycoproteins and Mucoproteins.


co

• The reaction of Ketone with hydroxyl group forms


Hemiketal Ring Structure
Fructose
• Constituent if Sucrose, the common sugar
• The reaction of Aldehyde with hydroxyl group forms
Hemiacetal Ring Structure. • Present in fruit juices, honeyQ and sugarcane
• Present in the seminal fluidQ.
If the ring structure formed by cyclization is six-sided
(made of 5 carbons and 1 oxygen), it is called a Pyranose Remember
All the Hexoses have a free functional group, hence they are
ring; if it is five-sided (made of 4 carbons and one oxygen),
reducing sugars.
it is called a furanose ring.
• Fructose exists predominantly as Furanose ring
structure (Fructofuranose) DISACCHARIDES [Cn(H2O)n-1]
• Glucose exists predominantly as Pyranose ring Two monosaccharide units are linked by a glycosidic
m

structure (Glucopyranose). bond or yield 2 monosaccharide units on hydrolysis.


co

Depending on their reducing property, they are divided


into:
• Nonreducing disaccharides: The functional groups are
involved in the glycosidic bond formation, hence free
functional groups are not available
• Reducing disaccharides: Free functional groups are
Fig. 4.1: Pyran and furan ring available.

Biologically Significant Hexoses Reducing Disaccharides–Free


Functional Group Present (DNB 1994)
Glucose
Disaccharide Sugar UnitsQ Linkage
• Most predominant sugar in human body
m

Q
1. Maltose αDGlucose + αDGlucose α1α4 linkage
• Main source of metabolic fuel of mammals (α14 linkage)
co

• Glucose is dextrorotatory, hence otherwise called 2. Isomaltose αDGlucose + αDGlucose α1α6


DextroseQNBE model 2012 Linkage (α16 linkage)
• Universal fuel of fetus. Contd...
e

e
m

m
m
co

Chemistry of Carbohydrates  | 121

Contd... POLYSACCHARIDES
Disaccharide Sugar UnitsQ Linkage
Q Condensation product of more than 10 monosaccharide
3. Lactose (Milk DGalactose +
Q
β1β4 linkage
Sugar) β DGlucose units or yield > 10 monosaccharide units on hydrolysis.
4. Lactulose αDGalactose + α1β4 linkage Also called Glycans.
βDFructose Depending on the type of Monosaccharide units.
Polysaccharides are classified into:
m

Nonreducing DisaccharidesQ —No Free


• Homoglycans [Homopolysaccharide] Contain only
co

Functional Group
one type of Monosaccharide unit
Disaccharide Sugar Units Linkage • Heteropolysaccharides [Heteroglycans] contain
Trehalose (Sugar of insect he- αDGlucose α1 → α1 Link- different types of Monosaccharide units.
molymph, yeast and fungi) + αDGlucose age
SucroseQ (Cane SugarQ) αDGlucose α1 → β2 Link- Homoglycans [Homopolysaccharide]
+βDFructose age
Glycogen
Lactulose
• Osmotic Laxative • Storage form of Glucose in animals, hence called
• Mainly Synthetic [small amount in heated milk] animal starch(Kerala 2006)
• Not hydrolyzed by intestinal bacteria • Made up of αD Glucose
• But fermented by intestinal bacteria.
• Branched polymer of Glucose
• α 1,4 Linkage at the linear part and α 1,6 Linkage at
m

OLIGOSACCHARIDES
branches
co

Condensation product of 3 to 10 monosaccharides, or • In Muscle Glycogen, granules called β particle are


yield 3–10 monosaccharide units on hydrolysis: present. These contain 60,000 glucose residues
• Blood group substances are oligosaccharides • In Liver apart from β particle, rosettes of glycogen
• Usually found in association with Proteins (Glyco- granules, which are aggregated β particles, are also
proteins and Proteoglycans) and lipids (Glycolipids) present.
m
co
m
co

Fig. 4.2: Structure of glycogen


e

e
m

m
m
co

122 |  
Self Assessment and Review of Biochemistry

Starch Contd...
• Homopolysaccharides made up only of Glucose Substances used are:
• Storage form of Carbohydrates in plants • Human albumin
• Dextran
• Also called glucan or glucosan.
• Hydroxyethyl starch (Hetastarch)(AI-94, AI-2002)
Two main constituents are: • Degraded gelatin polymer.
• Amylose (13–20%) which has a nonbranching helical
m

structure Compare and Study


co

• Amylopectin (80–87%) which consist of branched Q


Dextrose Glucose being dextrorotatory is known as Dextrose
chains with 24–30 glucose residues. in clinical practice
Starch [Glucosan] Dextrin Are intermediates in the hydrolysis of starch
Mixture of Amylose 15–20% and Amylopectin 80–85% Dextran Homopolysaccharide made up of Glucose
(i) Amylose Glucose α1 4 Linkage
Soluble Unbranched Lactose Disaccharide made up of Galactose and Glucose

(ii) Amylopectin Glucose α1 4 Linkage Lactase Enzyme which hydrolyze Lactose to Galactose and
Insoluble Branched Glucose
α 16 Linkage
[at Branches] Lactate Product obtained from Pyruvate during anerobic
Glycolysis

Chitin Lactulose Disaccharide made up of Galactose and Fructose

• Found in exoskeleton of crustaceans and insects and


m

in mushrooms HETEROPOLYSACCHARIDES
co

• Made up of N Acetyl D Glucosamine joined by β1 (HETEROGLYCANS)


→ 4 linkage. • Glycosaminoglycans (Mucopolysaccharides)
Cellulose • Pectins
• Chief constituents of plant cell wall • Agarose
• Homopolysaccharide of βGlucose in β 1, 4 linkage • Agar.
• Insoluble
• Major component of dietary fiber [Source of bulk in
Mucopolysaccharides (GlycosaminoglycansQ)
the diet] • Glycosaminoglycans are unbranched hetero-
• Humans lack enzymes that hydrolyse β 1, 4 glycosidic polysaccharideQ chains composed of Disaccharide
bonds.Q Hence cannot digest cellulose. repeat units
• Each disaccharide repeats unit composed of an
m

Inulin (Fructosan) aminosugar and uronic acid


co

• Homopolysaccharide of Fructose in β 2 → 1 linkage • They were first isolated from mucin, hence called
• Found in roots of dahlias, chicory, onion, garlic, mucopolysaccharides
dandelions • Major component of extracellular matrix.
• Belongs to a class of fibers
• Readily soluble in water Properties of GAG
• Used in clearance test to determine GFR • Carry large number of negative charges [COO –,
• Not hydrolyzed by human digestive enzymes. Acetyl, Sulphate], these chains tend to repel each
other
Dextran • Hence slippery consistency of mucous secretion and
• α Glucose in different linkage (α 1, 6 and α 1, 4 and synovial fluid
α 1, 3) • When water squeezes out, they occupy small volume.
• Used as Plasma Volume Expander.
m

When compression is released, they spring back to


original hydrated volume because of repulsion of
co

Plasma Expanders
High mol wt substances that exert colloid osmotic pressure and retain negative charges
fluid in the vascular component. • Hence, resilient nature of synovial fluid and vitreous
Contd... humor
e

e
m

m
m
co

Chemistry of Carbohydrates  | 123

• Special ability to bind to large amounts of water. Heparin


Hence forming major component of extracellular • Consists of Glucosamine and either of two uronic
matrix. acids
Disaccharide repeat unit in glycosaminoglycans (GAG) • Vast majority of the uronic acid residues are Iduronic
acid
• Initially, all are Glucuronic acid, but 90% of GlcUA
m

is converted to IdUA by a 5’ Epimerase


co

• Heparin is an anticoagulant
• Heparin specifically binds to Lipoprotein Lipase
present in capillary walls, causing release of this
enzyme into circulation
• Heparin is found in the granules of mast cells, also
liver, lung, and skin.
Biologically Important GAGs Heparan sulfate
Disaccharide • Present on many cell surfaces as proteoglycan
GAG Repeat Unit Location • Predominant uronic acid is GlcUA unlike Heparin
Hyaluronic N-Acetyl Glucos- Skin, Synovial fluid, bone,
• They act as receptors
Acid amine + Gluc- cartilage, vitreous humor,
(HyaluronanQ uronic AcidQ loose connective tissue, um- • Mediate cell growth and cell to cell communication
m

bilical cord • Found in kidney basement membrane along with


co

Chondroitin N-Acetyl GaIac- Cartilage, bone, CNS Type IV collagen and laminin
sulfate tosamine + Gluc-
uronic Acid
• In kidney basement membrane, it plays a role in
Keratan N-Acetyl Glucos- CorneaQ charge selectiveness of glomerular filtration.
sulfate I and II amine, Galac- Cartilage Dermatan sulfate
tose Loose connective tissue • Widely distributed GAG
Heparin Glucosamine, Mast cellsQ • The main GAG of skin.
Iduronic Acid Liver, lung, skin
Points to Ponder—GAGs
Heparan sul- Glucosamine, Skin
fate (HS) Glucuronic Acid • GAG with no Uronic AcidQ: Keratan Sulfate
Kidney basement membrane
• GAG with no Sulfate group: Hyaluronic Acid
Dermatan N-Acetyl Ga- Skin,
• GAG not covalently linked to Protein: Hyaluronic Acid
sulfate (DS) lactosamine, Wide distribution • GAG found in bacteria: Hyaluronic Acid
Iduronic Acid/
• GAG which is an anticoagulant: Heparin
m

Glucuronic Acid
• Most abundant GAG: Chondroitin Sulfate
co

• Site of Synthesis of GAG: Endoplasmic Reticulum and Golgi


Important Points of Glycosaminoglycans • Shape of Proteoglycan monomer: Bottle Brush
Hyaluronic acid • Glycosaminoglycans are Polyanions
• GAG that helps in cell migration: Hyaluronic acid
• Present in bacteria and ECM of nearly all animals • GAGs that have a role in compressibility of cartilage in weight-
• Play an important role in permitting cell migration bearing are Hyaluronic acid and Chondroitin Sulfate
during morphogenesis and wound repair. • GAGs that play a role in corneal transparency: Keratan Sulfate I
Chondroitin sulfate and Dermatan Sulfate
• GAGs that determine charge selectiveness of Renal Glomerular
• Join with a protein by the Xylulose-Serine O membrane: Heparan Sulfate
Glycosidic bond • GAGs are generally extracellular
• Major component of cartilage • Some intracellular GAG are Heparin in mast cells and Heparan
• Located at sites of calcification in endochondral bone. sulfate in synaptic and other vesicles.

Keratan Sulfate I and II


m

• Keratan Sulfate I is originally isolated from cornea Mucin Clot Test (Rope Test)Q
co

• Keratan Sulfate II is isolated from cartilage • To detect hyaluronate in the synovial fluid
• In eye, keratan sulfate lies between the collagen fibrils • Normal synovial fluid forms tight ropy clot on
and play a critical role in corneal transparency. addition of acetic acid.
e

e
m

m
m
co

124 |  
Self Assessment and Review of Biochemistry

PROTEOGLYCAN GAGs results in IntralysosomalNBE patternQ2012accumulation


of GAGs.
• GAGs are covalently attached to a protein (termed
Core Protein) to form hybrid molecules, called
Proteoglycan
• Linking of Polysaccharide chain to core protein
occurs by a Core trisaccharide, Gal-Gal-Xyl
m

• GAG-Core Trisaccharide-Core Protein.


co

Remember
• An exception in which GAG not attached to a core protein is
Hyaluronic Acid. Fig. 4.3: Simplified causation of mucopolysaccharidosis

Muco-
Enzyme Urinary
Proteoglycan Monomer and Proteoglycan polysac- Inheritance
Defect Metabolite
chridosis
Aggregate
MPS I H Hurler Autosomal L-Iduroni- Dermatan
• Proteoglycan molecules attached to core protein diseaseQ Recessive daseQ Sulfate
forming proteoglycan monomer Heparan
Sulfate
• The shape of Proteoglycan monomers is Bottle brush
MPS I S Scheie Autosomal L-Iduroni- Dermatan
• Several Proteoglycan monomer associate noncova- disease Recessive dase Sulfate
lently to a Hyaluronic Acid by a link protein to form
m

MPS II Hunter X-linked Re- Iduronate Dermatan


Proteoglycan Aggregate. diseaseQ cessive Sulfata- Sulfate
co

seQ Heparan
Proteoglycan Glycoprotein Sulfate
>95% Carbohydrate <5% Carbohydrate MPS III A Sanfilippo Autosomal Heparan Heparan
Long linear unbranched Short highly branched Oligosac- A disease Recessive Sulfate N Sulfate
Oligosaccharides charide chains Sulfatase

Disaccharide Repeats No repeating units MPS III B Sanfilippo Autosomal N-Acetyl Heparan
B disease Recessive Glucosa- Sulfate
minidase
GAG AND DISEASES
Tumor cell migration MPS III C Sanfilippo Autosomal Glucosa- Heparan
C disease Recessive minide Sulfate
• Tumor cells induce fibroblast to synthesize Hyaluronic acid
N Acetyl
• Hyaluronic acid permit tumor cells to migrate through ECM Transfer-
• Some tumor cells have less heparan sulfate at their surfaces, ase
and this may play a role in the lack of adhesiveness that these
MPS III D Sanfilippo Autosomal N Acetyl Heparan
cells display.
m

D disease Recessive Glucos- Sulfate


GAG and atherosclerosis amine
co

Sulfatase
• Dermatan Sulfate appears to be the major GAG synthesized by
arterial smooth muscle cells MPS IV A Morquio A Autosomal Galactos- Keratan
• These cells proliferate in atherosclerotic lesions in arteries Recessive amine 6 Sulfate
Sulfatase Chondroitin
• Dermatan sulfate binds plasma low-density lipoproteins
6 Sulfate
• Because of this, dermatan sulfate may play an important role in
development of the atherosclerotic plaque. MPS IV B Morquio B Autosomal Beta Keratan
Recessive Galactosi- Sulfate
GAG and osteoarthritis dase
• In arthritis, proteoglycans may act as autoantigens MPS VI Marote- Autosomal N Acetyl Dermatan
• The amount of chondroitin sulfate in cartilage diminishes with aux- Recessive Galactos- Sulfate
age, whereas the amounts of keratan sulfate and hyaluronic acid Lamy amine 4
increase Sulfatase
• These changes may contribute to the development of osteoarthritis. (Aryl Sul-
fatase B)
m

MPS VII Sly Dis- Autosomal Beta - Dermatan


ease Recessive Glucoroni- Sulfate
co

MUCOPOLYSACCHARIDOSES (MPS) dase Heparan


Sulfate
Hereditary progressive disease caused by mutation of
genes coding for Lysosomal Enzymes needed to degrade NB: The important MPS are given in bold letters. Please do learn them.
e

e
m

m
m
co

Chemistry of Carbohydrates  | 125

Recognition Pattern of MucopolysaccharidosisQ2013-14


Clinical features MPS IH MPS IS MPS II MPS III MPS IV MPS VI MPS VII
Common name Hurler Scheie Hunter San Filippo Morquio Maroteaux-Lamy Sly Disease
Mental deficiency + – + + – – ?
Coarse facial features + (+) + – – + ?
Corneal clouding + + – - (+) + ?
m

Visceromegaly + (+) (+) – – + +


co

Short stature + (+) + + + + +


Joint contractures + + + – – + +
Dysostosis multiplex + (+) (+) + + + +
Leucocyte inclusions + (+) + + – + +
Mucopolysacchariduria + + + + + + +

SOME IMPORTANT enzyme activity associated with a milder form of the


MUCOPOLYSACCHARIDOSES disease.
Clinical features
Mucopolysaccharidosis-I H (Hurler’s Disease) • Similar to MPS IH
m

Biochemical defect • MPS I-S is a comparatively mild disorder characterized


Homozygous or double heterozygous nonsense mutations by joint stiffness, aortic valve disease, corneal
co

IDUA gene on Chr 4p encoding α-L-Iduronidase clouding, and mild dysostosis multiplex
Clinical features of MPS I H (Hurler’s Disease) • Onset of significant symptoms is usually after the
• Progressive disorder with multiple organ and tissue age of 5 years, with diagnosis made between 10 and
involvement that results in premature death, usually 20 years of age
by 10 years of age • Patients with Scheie disease have normal intelligence
• An infant with Hurler’s syndrome appears normal and stature but have significant joint and ocular
at birth, but inguinal hernias are often present. involvement.
Diagnosis is usually made between 6 and 24 mo of age
• Hepatosplenomegaly, coarse facial features, corneal Mucopolysaccharidosis II (Hunter Disease)
clouding, large tongue, prominent forehead, joint Biochemical defect
stiffness, short stature, and skeletal dysplasia • X-linked disorder caused by the deficiency of
m

• Acute cardiomyopathy has been found in some iduronate-2-sulfatase (IDS)


co

infants < 1 year of age • Point mutations of the IDS gene mapped to Xq28 have
• Most patients have recurrent upper respiratory tract been detected in about 80% of patients with MPS II
and ear infections, noisy breathing, and persistent • Hunter disease manifests almost exclusively in
copious nasal discharge males; it has been observed in a few females and
• Valvular heart disease with incompetence, notably this is explained by skewed inactivation of the
of the mitral and aortic valves, regularly develops, X chromosome carrying the normal gene.
as does coronary artery narrowing
Clinical features
• Obstructive airway disease, notably during sleep, may
• MPS II have features similar to those of Hurler
necessitate tracheotomy. Obstructive airway disease,
disease except for the lack of corneal clouding and the
respiratory infection, and cardiac complications are
somewhat slower progression of somatic and central
the common causes of death.
nervous system (CNS) deterioration
m

Scheie Disease (MPS-IS) • Coarse facial features, short stature, dysostosis


co

Biochemical defect multiplex, joint stiffness, and mental retardation


A missense mutations in IDUA gene on Chr 4p encoding manifest between 2 and 4 years of age
α-L-Iduronidase more likely to preserve some residual • Grouped skin papules are present in some patients.
e

e
m

m
m
co

126 |  
Self Assessment and Review of Biochemistry

Natowicz syndrome (MPS-IX) Contd...


• A genetic defect in hyaluronidase causes MPS IX, a Stem Cell Enzyme
Transplanta- Replace-
lysosomal storage disorder in which hyaluronic acid
MPS Type tion (SCT) ment Remarks
accumulates in the joints
III (Sanfilip- No No Experimental: Sub-
• Joint pains and short stature are the clinical features. po) strate reduction by
Flavinoids
Laboratory Diagnosis of Mucopolysaccharidoses
m

IVA No Preclini- Recombinant Galac-


• Urinalysis for presence of increased amounts of
co

(Morquio) cal tosamine Sulfatase


GAGs (GALNS) in course

• Assays of suspected enzymes in white blood cells, VI (Marote- Yes Nagla- Sustained improve-
aux Lamy zyme ment
fibroblasts, or possibly serum Disease)
• Tissue biopsy with subsequent analysis of GAGs by VII (Sly) Questionable ? Single SCT attempt
electrophoresis without neurological
• Use of specific gene tests improvement

• Prenatal diagnosis can now be performed in at least


certain cases using amniotic fluid cells or chorionic DERIVED SUGARS
villus biopsy.
Monosaccharides whose structure cannot be represented
Points to Ponder—MPS
by general formula or which have some unusual features.
m

• Impaired degradation of Heparan Sulfate associated with Mental


They are:
co

Deterioration
• Impaired degradation of DS, CS, KS associated with mesenchymal • Acid Sugars (by oxidation of sugars)
abnormalities • Sugar Alcohols (by reduction of sugars)
• All MPS are Autosomal Recessive except Hunter Disease
• Deoxy Sugars
• Most common MPS is Sanfilippo followed by Hunter and Hurler.
MPS with no Mental Retardation • Amino SugarsQ
• Scheie Disease • Glycosides
• Morquio Disease [Keratan Sulphate and Chondroitin Sulphate] • Furfural Derivative.
• Maroteaux Lamy Disease
MPS with no corneal clouding Acid Sugars
• Hunter’s Disease Formed by oxidation of aldehyde carbon atom, hydroxyl
• Sanfilippo Disease carbon atom or both of monosaccharides.
MPS with no visceromegaly:
Under mild oxidation conditions
m

• Morquio Disease
Aldehyde group is oxidised to produce Aldonic Acid.
co

Same enzyme deficiency associated with two diseases:


• Hurler Disease • Glucose to Gluconic Acid (TNPGEE 2002)
• Scheie Disease • Mannose to Mannonic Acid
• Galactose to Galactonic Acid.
Clinical application of glucose to gluconic acid
NEWER MODALITIES OF TREATMENT OF MPS
• During Glucose oxidase method for estimation
Stem Cell Enzyme of blood glucose, Gluconic AcidQ is formed from
Transplanta- Replace- Glucose
MPS Type tion (SCT) ment Remarks
• When aldehyde group is protected and last Carbon
I (Hurler, Yes Aldura- Transplantation be-
Scheie) zyme fore age 2. Enzyme
atom is oxidised, then uronic acid is the product
replacement before • Glucose to Glucuronic Acid:
and after transplan-
‒ Iduronic Acid is an epimer of Glucuronic Acid
m

tation
‒ Constituent of Glycosaminoglycans (GAGs)
co

II (Hunter Questionable Elaprase Lack of neurological


Disease) improvement after ‒ Used for conjugation of Bilirubin
SCT • Mannose to mannuronic acid galactose to galacturonic
Contd... acid.
e

e
m

m
m
co

Chemistry of Carbohydrates  | 127

Under strong oxidation condition AMINO SUGARS (HEXOSAMINES)


• Both first and last carbon atoms are oxidised to
Aminogroup replaces the hydroxyl group present in the
produce saccharic acid
second carbon atom of monosaccharides to form Amino
‒ Glucose to glucosaccharic acid Sugars.
‒ Mannose to mannaric acid Important amino sugars are:
‒ Galactose to mucic acid • Glucosamine
m

Clinical application—galactose to mucic acid • Galactosamine (Chondrosamine)


co

Mucic acid forms insoluble crystal forms basis for mucic • Mannosamine.
acid test for the identification of galactose. Remember
An unusual Amino sugar with 9 carbon atom is Sialic Acid. The
SUGAR ALCOHOLS principal Sialic Acid found in human body is N Acetyl Neuraminic
Acid (NANA)
• Monosaccharides are reduced at their carbonyl group
Points to Ponder—Metabolism of Amino Sugars
to yield corresponding polyhydroxyalcohols • Glucose is the precursor of Amino Sugar.
• Aldoses undergo reduction to form corresponding • The immediate precursor of Glucosamine is Fructose 6
Alcohol PhosphateQ 2012.
• Amino group is donated by Glutamine.
• Ketoses form two alcohols because of appearance of • NANA is derived from N Acetyl Mannosamine.
new asymmetric carbon atom.
m

Biochemical significance of amino sugars


Glucose Sorbitol
• They are components of Glycoproteins, Gangliosides
co

Mannose Mannitol
and Glycolipids
Galactose Dulcitol/Galactitol
• Antibiotic which contains amino sugar is Erythro-
Fructose Sorbitol and Mannitol mycin.

Clinical Applications of Sugar Alcohols GLYCOSIDES


• Mannitol is used to reduce intracranial pressure by When the monosaccharide is condensed with an alcohol,
forced diuresis phenol or sterol by O-Glycosidic linkage to form Glycoside.
• Osmotic effect of Dulcitol and Sorbitol causes cataract The noncarbohydrate group is called Aglycone.
in Galactosemia and Diabetes respectively Clinical Importance of Glycosides
• Polyol pathway is responsible for the development • Cardiac Glycosides—Action on heart
of Diabetic cataract. ‒ Digitalis (Steroid is the Aglycone)
m

‒ Quabain.
co

DEOXY SUGARS • Antibiotics


‒ Streptomycin, Puromycin.
Hydroxyl group of sugars is replaced by hydrogen atom.

Biochemical Importance of Deoxy Sugars ISOMERISM IN CARBOHYDRATES

Deoxyribose Different compounds having same molecular formula


are called isomers of one another.
• Oxygen is removed from 2nd position (TNPGEE 04)
• Is an important component of DNA Asymmetric Carbon Atom
• Feulgen staining is specific for 2 deoxy sugars (and The carbon atom to which four different substituent
DNA) in the tissues. groups are attached is called a chiral or asymmetric
carbon atom.
m

L–Fucose Lebervon’t Hoff Rule


co

• Deoxy Sugar present in the Blood group antigens. The relationship between the number of Asymmetric Carbon
atom and the number of Stereoisomers possible.
2-Deoxy Glucose Number of isomers = 2 n
• Experimentally an inhibitor of Glucose metabolism. where n is the number of Asymmetric Carbon atom.
e

e
m

m
m
co

128 |  
Self Assessment and Review of Biochemistry

• In Open chain structure of Glucose, there are Mutarotation


4 asymmetric Carbon atoms (C-2, C-3, C-4, C-5) • Mutarotation is change in optical rotation of plane polarized light
• But in solution 99.5% Glucose exists in Pyranose form, with time
• Mutarotation is a property of Anomeric Carbon atom
then first carbon atom also becomes an asymmetric
• Mutarotation is studied by measuring the rotation of plane polarized
Carbon atom. Hence, the number of isomers possible light
for Glucose is 25 is 32. • The optical rotation of α D Glucose is + 112°
m

• The optical rotation of β D Glucose is + 19°


Types of Isomers in Carbohydrates • Both undergo mutarotation over a period of a few hours, an
co

The presence of asymmetric carbon atom imparts two equilibrium is attained


important properties: • At equilibrium the optical rotation is + 52°.

• Stereoisomerism
• Opticalisomerism. EPIMERISM (DIASTEREOISOMERISM)
Stereoisomerism Difference in orientation of H and OH group around
Compounds having the same molecular formula but Carbon atoms other than Anomeric Carbon and
different spatial configuration of H and OH group around Penultimate Carbon results in isomerism referred to as
the asymmetric carbon atoms. Epimerism.
Epimers of GlucoseQ
D and L Isomerism [Enantiomers] Difference in orientation of H and OH at C2 C3 and C4
Difference in the orientation of H and OH group around for Glucose:
m

penultimate carbon atom results in two mirror images • 2nd Epimer of Glucose – Mannose
co

called D and L isomers (Enantiomers). • 3rd Epimer of Glucose – Allose


The penultimate carbon atom is called Reference • 4th Epimer of Glucose – Galactose.
Carbon atom.
• The penultimate carbon atom in Glucose and
OPTICAL ISOMERISM
Fructose is C-5
• Most of the naturally occurring Monosaccharides • When a beam of plane polarized light is passed
are D isomers (Unlike Amino Acids, which are L through a solution of carbohydrates, it rotates the
isomers). light either to right or to left. Depends on the direction
Examples of EnantiomersQ of rotation, two optical isomers possible
• D Glucose and L Glucose • Dextrorotatory (represented by d or +)
• D Fructose and L Fructose • Rotate plane polarized light to right (Clock-wise)
• Levorotatory (represented by l or –)
m

• D Mannose and L Mannose


• D Glyceraldehyde and L Glyceraldehyde. • Rotate plane polarized light to left (Anticlockwise)
co

• D Glucose is dextrorotatory (i.e. why glucose is also


ANOMERISM called Dextrose) but D Fructose is levorotatory.

• Formation of ring structure in monosaccharides Racemic Mixture (AI 03)


• Equimolar mixture of optical isomers which has no net rotation
results in creation of an additional asymmetric carbon of plane polarized light.
called anomeric Carbon atom
• The carbon atom with functional group forms the Sucrose is Otherwise Called Invert Sugar
anomeric carbon atom • Sucrose is dextrorotatory. On hydrolysis of Sucrose yield a
mixture of dextrorotatory Glucose and levorotatory Fructose.
• In Glucose C-1 and in fructose C-2 form the anomeric Because of strong levorotatory nature of fructose, Sucrose
carbon atom on hydrolysis is levorotatory. Hence, Sucrose is called invert
• Difference in orientation of H and OH group around sugar
the anomeric carbon atom results in Anomerism
m

Points to remember—Isomerism
• The resulting isomers are called α and β anomers.
co

• Monosaccharide with no Asymmetric Carbon atom-Dihydroxy-


Examples of Anomerism: acetone
• α D Glucose and β D Glucose • Ketoses have 1 asymmetric carbon atom less than aldoses

• α D Fructose and β D Fructose. Contd...


e

e
m

m
m
co

Chemistry of Carbohydrates  | 129

Contd... Reaction in glucose oxidase-peroxidase method


• No. of Isomers possible is 2n where n is the no. of Asymmetric Principle of glucose oxidase-peroxidase
carbon atom (Leber von’t Hoff rule)
• All D isomers need not be dextrorotatory and vice versa
• Glucose and Fructose are Aldo-Keto Isomers

SHAPES OF OSAZONES
m

DIETARY FIBERSQ
co

Shape Sugar
Needle-shaped/Broomstick/ Glucose, Fructose, Mannose Complex Carbohydrates not digested by human digestive
Sheaves of Corn enzyme. Otherwise called Nonstarch Polysaccharide.
Pincushion with pins/Hedgehog/ Lactose Include:
Flower of Touch-me-not
• Insoluble Fibers
Sunflower Petal-shaped Maltose
‒ Cellulose
NB: Sucrose does not form osazones ‒ Hemicellulose
‒ Lignin
TESTS FOR CARBOHYDRATES • Soluble Fibers
‒ Pectin
General test for all Carbohydrates
‒ Gums
• Molisch test
m

‒ Mucilage
Test for Reducing Substances
co

RDA 40 g per 2000 kcal per day is desirable. Supply of


• Benedict’s Test energy from dietary fibers is 2 kcal/g.
Test to differentiate Monosaccharides and Disaccharides
Beneficial Effects of Dietary Fibers
• Barfoed’s Test
• Prevent constipation
• Moore’s test
• Maintain normal motility of GIT
• Fehling’s Test
• Eliminate bacterial toxin
Test to differentiate Aldoses and Ketoses
• Fiber absorbs large quantity of water and toxic
• Seliwanoff’s Test compounds by intestinal bacteria
• Rapid furfural Test • Increase bulk of the stool
• Foulger’s Test • Reduce the stool transit time
Test to detect Deoxy Sugar • Decrease GI Cancers—Colon and Rectum
m

• Feulgen Staining • Slow Gastric Emptying


co

Test for Pentoses • Improve Glucose Tolerance by decreasing rate of


• Bial’s Test absorption of glucose
Test for Galactose • Reduce Plasma Cholesterol
• Mucic Acid Test. • Decrease absorption of dietary cholesterolQ
• Bind the bile salt and decrease enterohepatic
circulation of bile salts and increase excretion of bile
METHODS FOR ESTIMATION OF GLUCOSE
salt, the excretory form of cholesterol
Reductometric Methods • Give sensation of stomach fullness.
• Nelson Somogyi Method A High-fiber diet is associated with reduced incidence ofQ:
• Folin-Wu Method • Diverticulosis
• O-Toluidine Method • Cancer Colon
• Cardiovascular Disease
Enzymatic Method
m

• Diabetes Mellitus
• Hexokinase Method
co

Dietary Fiber neither digested nor fermented—LigninQ


• Glucose Oxidase Peroxidase Method (GOD–POD) Digestion of dietary fibers by colonic bacteria in herbivores give rise
• Highly Specific Method to small chain fatty acids (Acetate, Propionate and ButyrateQ)
AIIMS Nov 2001
• Used in dry analysis technique like Glucometer.
e

e
m

m
m
co

130 |  
Self Assessment and Review of Biochemistry

DIGESTION OF CARBOHYDRATES Glucose Transporters


• Passive process down the concentration gradient
By two sets of enzymes:
• Bidirectional
1. Alpha Amylases
• Facilitative Diffusion
2. Disaccharidases.
• Ping Pong Mechanism
Action of Alpha Amylase (AIIMS Dec 94) • Sodium-independent.
m

• Cleaves alpha 1,4 glycosidic bond


co

• Present in saliva, Pancreatic juice and Intestinal juice


• Yielding dextrins, then a mixture of glucose, maltose,
and maltotriose and small branched dextrins (from
the branchpoints in amylopectin).

Action of Disaccharidases
Fig. 4.4: Glucose transporters—Ping pong mechanism
• They are maltase, sucrase-isomaltase (a bifunctional
enzyme catalyzing hydrolysis of sucrose and Transporter Tissue Location Function
isomaltose), lactase, and trehalase GLUT 1 Brain, Kidney, Co- Basal Glucose Uptake
lon, Placenta, RBCs,
• They are located on the brush border of the intestinal Retina
mucosal cells
GLUT 2 Liver Sinusoid In liver, removal of excess
m

• Yield the corresponding monosaccharides, which membrane β cells glucose from blood; In pan-
co

are absorbed. of Pancreas, Sero- creas, regulation of insulin


sal side (basolateral release.
Lactase Deficiency and Sucrase DeficiencyQ side) of Intestinal Low affinity and higher Km.
Cell Basolateral
Lactase Deficiency Sucrase Deficiency (AI 04) membrane of PCT in
Kidney
Clinical manifestation following Clinical manifestation following
ingestion of milk, which contain ingestion of dairy products which GLUT 3 Neurons, Placenta, High affinity for Glucose
Lactose contain Sucrose Kidney
Watery Diarhea Bloating Watery Diarhea Bloating Failure GLUT 4 Heart, Skeletal Mus- Insulin-dependent
Failure to thrive to thrive cle, Adipose Tissue Glucose Uptake
GLUT 5 Small Intestine, Tes- Primarily Fructose Trans-
tis, Sperm porterQ
ABSORPTION OF CARBOHYDRATES GLUT 6 Spleen, Leukocytes Possibly no transporter
m

function
By two sets of transporters:
GLUT 7 Liver Endoplasmic Glucose Transporter in the
co

1. Sodium-dependent Glucose transporters (SGLT). reticulum Endoplasmic Reticulum


2. Sodium-independent Glucose transporters (GLUTs).
Other Glucose transporters are:
Sodium-dependent Glucose Transporters Glucose Tissues where
• Secondary Active Transport Transporter expressed Functions
• Unidirectional GLUT 8 Testis, Blastocyst, Insulin-responsive
brain glucose transporter of
• SGLT is coupled with Na-K+ ATPase pump. Sodium-
Blastocyst. Glucose
dependent Glucose Transporters (SGLT). transporter to mature
spermatozoa
SGLT1 Small Intestine, Renal Tubules Absorption of Glucose
GLUT 9 Liver, Kidney Urate transporter
SGLT2 Renal Tubules Absorption of Glucose
GLUT 10 Liver, Pancreas
m

Clinical Correlation—Renal Glycosuria GLUT 11 Heart, Skeletal muscle Fructose transporter


Isolated glucosuria in the presence of a normal blood glucose con-
co

centration is due to mutations in SLC5A2, the gene that encodes the GLUT 12 Prostate, Heart, Mam- Insulin responsive
high-capacity sodium-glucose co-transporter SGLT2 in the proximal mary gland, White
renal tubule adipose tissue
e

e
m

m
m
co

Chemistry of Carbohydrates  | 131

Inhibitors of Glucose Transporters • They are carried by the same transport protein
• Phlorizin (Phloretin 2 β Glucoside)Q(PGI Dec 2008) is an (SGLT 1) and compete with each other for intestinal
inhibitor of Sodium-dependent glucose transporter absorption
by competing with D Glucose-binding sites of the • Fructose absorbed down their concentration gradient
carrier. SGLT2 > SGLT1 by GLUT 5
• Phloretin (Aglycone of Phlorizin) is an inhibitor of • All the sugars exit from intestinal cells via GLUT 2.
m

facilitated diffusion by GLUT-1 or GLUT-4.


co

Points to Ponder—Glucose Transporters


• Widely distributed Glucose transporter-GLUT-1
• Most abundant Glucose transporter in RBC-GLUT-1
• Major glucose transporter of Brain-GLUT-1
• Major glucose transporter of neurons-GLUT-3
• Major glucose transporter of Placenta-GLUT-1
• Glucose transporter of blastocyst-GLUT-8
• Insulin-dependent Glucose transporters-GLUT-4, GLUT-8,
GLUT-12
• GLUT-3 is present in neurons, whereas GLUT-1 is not present
in neurons
m

• Urate transporter is GLUT-9


co

ABSORPTION OF MONOSACCHARIDES
• Glucose and galactose are absorbed by a sodium-
dependent process. Fig. 4.5: Absorption of monosaccharides

REVIEW QUESTIONS

CLASSIFICATION OF CARBOHYDRATES Noncarbohydrates that give Benedict’s test positive are:


(INCLUDES TESTS AND REACTIONS OF • Homogentisic acid
m

CARBOHYDRATES) • Glucuronic acid


co

• Salicylates
1. In Benedict’s test, red color is/are produced by: • Ascorbic acid
(PGI Nov 2014) • Uric acid
a. Sucrose • Glutathione
b. Inositol • Creatinine in very high amount
c. Fructose Nonreducing disaccharides, like Sucrose and Trehalose
d. Lactose do not give positive Benedict’s test. Tests for reducing
e. Maltose substances are Benedict’s test and Fehling’s test.
Ans. c. Fructose, d. Lactose, e. Maltose. 2. Which of the following is not an aldose?
(Ref: Varleys: Practical Clinical Biochemistry 4/e p110) a. Glucose (PGI 2012)
Benedict’s test is a test for reducing substances in urine. b. Mannose
m

Carbohydrates that give positive Benedict’s test are: c. Fructose


co

• Pentoses, Fructose, Glucose, Galactose d. Galactose


• Reducing disaccharides like Lactose, Maltose, e. Glycerol
Isomaltose Ans. c. Fructose, e. Glycerol.
e

e
m

m
m
co

132 |  
Self Assessment and Review of Biochemistry

Generic name Aldose Ketose Galactose Tolerance test assesses the metabolic capacity
Triose Glyceraldehyde Dihydroxy Acetone
of liver
Tetrose Erythrose Erythrulose
Galactose is almost entirely metabolized in the liver.
Pentose Ribose Ribulose 5. Cn(H2O) n is the formula for: (Kerela 2009)
Xylose (Epimer of Xylulose (Epimer of a. Monosaccharide
Ribose) Ribulose]
b. Disaccharide
m

Arabinose
c. Polysaccharide
co

Hexose Glucose, Galac- Fructose


tose, Mannose d. Oligosaccharide
Heptose Sedoheptulose Ans. a. Monosaccharide.
(Ref: Vasudevan and Sreekumari 7/e p69)
3. Glucose detection can be done by all except:
6. All are reducing sugars except (Kerala 2009)
(Kerela 2010)
a. Sucrose
a. Glucose oxidase
b. Lactose
b. Ferric Chloride test c. Glucose
c. Dextrostix d. Fructose
d. Folin-Wu method Ans. a. Sucrose. (Ref: Vasudevan and Sreekumari 7/e p77)
Ans. c. Ferric Chloride Test. All monosaccharides have free functional group and
m

Ferric Chloride test is the test done in Alkaptonuria and hence they are reducing sugars.
co

Phenyl Ketonuria. Reducing Disaccharides–Free Functional group present


Methods for estimation of glucose (DNB 1994)
• Reductometric Methods Disaccharide Sugar UnitsQ Linkage
• Nelson-Somogyi Method 1. Maltose Q
αDGlucose + αD α 1 → α4
• Folin-Wu Method G lucose Linkage
• O-Toluidine Method (α1 → 4 linkage)

Enzymatic method 2. Isomaltose αDGlucose + αD α 1 → α6


Glucose Linkage
Hexokinase Method-Glucose Oxidase-Peroxidase (α1 → 6 linkage)
Method (GOD –POD) (AIIMS Nov 2007) 3. LactoseQ DGalactose + βD β1 → β 4 LinkageQ
• Highly Specific Method (Milk Sugar) Glucose
• Used in dry analysis technique like Glu-cometer. 4 Lactulose αDGalactose + βD α1 → β4 Linkage
m

Fructose
Reaction
co

Principle of Glucose Oxidase Peroxidase Nonreducing Disaccharides Q—No Free Functional


group
Disaccharide Sugar Units Linkage
Trehalose Glucose + Glucose α1β → 1 Linkage
SucroseQ Glucose + Fructose α1 → β2 Linkage
4. Which of the following carbohydrate meta-
bolisms is used for liver function assessment: Isomerism in Carbohydrates
(Kerela 2010)
7. Which of the following are enantiomers:
a. Galactose tolerance test (PGI May 2011)
b. Sucrose tolerance test a. D-Galactose and L-Glucose
m

c. Glucose tolerance test b. d- Galactose and I-Glucose


co

d. Lactose tolerance test c. D-Mannose and L-Mannose


Ans. a. Galactose tolerance test. d. d-Mannose and I-Mannose
(Ref: Vasudevan and Sreekumari 7/e p355) e. D-glucose and L-Glucose
e

e
m

m
m
co

Chemistry of Carbohydrates  | 133

Ans. c. D-Mannose and L-Mannose, Ans. a. Keratan Sulfate.


   e. D-Glucose and L-Glucose. (Ref: Harper 30/e p637)
(Ref: Harper 29/e p133, Harper 30/e p153) • KS-I Originally isolated from CorneaQ
D and L Isomerism [Enantiomers] • KS-II from Cartilage
Difference in the orientation of H and OH group around • The composition of both Keratan sulfate are same
Penultimate Carbon atom results in two mirror images (N-acetyl Glucosamine and Galactose)
m

called D and L isomers (Enantiomers). • No Uronic acid in Keratan Sulfate


co

The penultimate carbon atom is called Reference • In the eye, they lie between collagen fibril and play
Carbon atom. The penultimate carbon atom in Glucose a critical role in corneal transparency.
and Fructose is C-5.
Most of the naturally occurring Monosaccharides are 10. Which deposition results in cataract?
D isomers [Unlike Amino Acids, which are L isomers.] (NBE pattern Qn)
Examples of EnantiomersQ a. Glucose
D Glucose and L Glucose b. Galactose
D Fructose and L Fructose c. Sugar amines
D Mannose and L Mannose d. Sugar alcohols
D Glyceraldehyde and L Glyceraldehyde. Ans. d. Sugar alcohol. (Ref: Harper 30/e p205)
• In Diabetes mellitus, in the lens by polyol pathway
8. Epimer combination(s) is/are: Glucose converted to Sorbitol by the enzyme Aldose
m

(PGI May 2010) (Nov 2009) reductase


co

a. D-glucose and D-fructose • In galactosemia, Dulcitol or Galactictol is responsible


b. D-mannose and D-talose for cataract
c. D-glucose and D-mannose • Sorbitol, Dulcitol are sugar alcohols.
d. D-glucose and D-gulose
11. Cellulose is: (NBE Pattern Q)
e. D-galactose and D-glucose
a. Complex Lipoprotein
Ans. c. D-glucose and D-mannose,
b. Starch Polysaccharide
     e. D-galactose and D-glucose. (Ref: Harper 30/e p153)
c. Nonstarch Polysaccharide
Epimerism [Diastereoisomerism]
d. Complex Glycoprotein
Difference in orientation of H and OH group around
Ans. c. Nonstarch Polysaccharides.
Carbon atoms other than Anomeric Carbon and
Penultimate Carbon results in isomerism refered to as (Ref: Harper 30/e p153)
m

Epimerism. Foods contain a wide variety of other polysaccharides that


co

Epimers of GlucoseQ are collectively known as nonstarch polysaccharides; they


are not digested by human enzymes, and are the major
Difference in orientation of H and OH at C2 C3 and C4
components of dietary fiber.
for Glucose:
Examples are cellulose from plant cell walls (a glucose
• 2nd Epimer of Glucose – Mannose
polymer) and inulin, the storage carbohydrate in some
• 3rd Epimer of Glucose – Allose
plants (a fructose polymer).
• 4th Epimer of Glucose– Galactose.
12. A 4-year-old boy with mental retardation,
Glycosaminoglycans and dysostosis multiplex, coarse facial features, clear
Mucopolysaccharidoses cornea. What is the diagnosis?
9. Glycosaminoglycans present in cornea: a. MPS Type IV
(NBE pattern Qn) b. Hunter’s Disease
m

a. Keratan Sulfate c. Hurler’s Disease


co

b. Hyaluronic acid d. Zellweger Syndrome


c. Chondroitin Sulfate Ans. b. Hunter’s Disease.
d. Dermatan Sulfate (Ref: Nelson 20/e Tables 82.1 and 82.2)
e

e
m

m
m
co

134 |  
Self Assessment and Review of Biochemistry

Recognition pattern of mucopolysaccharidoses • Heparin is a Glycosaminoglycan


Clinical features MPS IH MPS IS MPS II • Glycosaminoglycans are heteropolysaccharides
Common name Hurler Scheie Hunter • Disaccharide repeat unit is Glucosamine and Iduronic
Mental deficiency + – + acid
Coarse facial features + (+) + • Initially uronic acid present is Glucuronic acid,
Corneal clouding + + – 5 epimerase convert 90% of GlcUA to IdUA
m

Visceromegaly + (+) (+) • Heparin is found in the granules of mast cells, also
co

Short stature + (+) +


in lung, liver and skin. Heparin specifically binds to
lipoprotein lipase present in capillary walls and cause
Joint contractures + + +
its release into circulation
Dysostosis multiplex + (+) (+)
• Heparin is an anticoagulant.
Leucocyte inclusions + (+) +
Mucopolysacchariduria + + + 16. Glycogenin is a: (NBE pattern Qn)
a. Polypeptide
b. Polysaccharide
13. Mucopolysaccharide that doesn’t contain Uronic
acid residue is: (JIPMER 2015) c. Lipid
a. Heparan Sulfate d. Glycosaminoglycan
b. Heparin Ans. a. Polypeptide. (Ref: Harper 30/e p177)
m

c. Chondroitin Sulfate • Glycogenin is a protein


co

d. Keratan Sulfate • 37 K Da protein


Ans. d. Keratan Sulfate. • Glycogenin catalyzes transfer of 7 glucosyl residues
GAG with no Uronic AcidQ-Keratan Sulfate from UDPGlucose, in alpha 1 → 4 linkage
GAG with no Sulfate group-Hyaluronic Acid • Glucosyl residues are added on specific tyrosine
GAG not covalently linked to Protein-Hyaluronic Acid. residues of Glycogenin
• Glycogenin remains at the core of Glycogen granule.
14. Mucopolysacchridoses, which are a lysosomal
storage disease, occur due to abnormality in: 17. Complex polysaccharides are converted to glucose
(PGI May 2015) and absorbed by the help of: (Kerala 2007)
a. Hydrolase enzyme a. Na+ K+ ATPase
b. Dehydrogenase enzyme b. Sucrase
c. Lipase enzyme c. Enterokinase
m

d. Phosphatase d. Carboxypeptidase
co

e. Acetyl CoA Carboxylase Ans. b. Sucrase. (Ref: Harper 30/e p538)


Ans. a. Hydrolase enzyme. The cells of brush border of intestine contain the enzyme,
(Ref: Nelson 20/e Chapter Defects in the Sucrase, Maltase, Iso-maltase and lactase hydrolyse
metabolism of lipids/lysosomal storage disorder) corresponding disaccharides to monosaccharides which
The lysosomal storage diseases are diverse disorders each are absorbed.
due to an inherited deficiency of a lysosomal hydrolase
leading to the intralysosomal accumulation of the Glucose Transporters
enzyme’s particular substrate.
18. After overnight fasting, levels of glucose
15. Heparin is a: (NBE Pattern Qn) transporters are reduced in: (May 2010)
a. Glycosaminoglycan a. Brain cells
m

b. Polysaccharide b. RBCs
co

c. Proteoglycan c. Adipocyte
d. Carbohydrate d. Hepatocyte
Ans. a. GAG. (Ref: Harper 30/e p637) Ans. c. Adipocytes. (Ref: Harper 29/e p158, 30/e p192)
e

e
m

m
m
co

Chemistry of Carbohydrates  | 135

GLUT-4 and Insulin c. GLUT-3: placenta


Glucose uptake into muscle and adipose tissue is d. GLUT-2: beta cell glucose sensor
controlled by insulin, which is secreted by the islet cells Ans. None. (Ref: Harper 29/e p158, 30/e p191)
of the pancreas in response to an increased concentration Location of glucose transporters
of glucose in the portal blood.
Transporter Tissue Location
In the fasting state, the glucose transporter of muscle
m

and adipose tissue (GLUT-4) is in intracellular vesicles. GLUT 1 Brain, Kidney, Colon, Placenta, RBCs, Retina
co

An early response to insulin is the migration of these GLUT 2 Liver, β cells of Pancreas, Serosal side of Intes-
tinal Cell
vesicles to the cell surface, where they fuse with the
Basolateral membrane of PCT in Kidney.
plasma membrane, exposing active glucose transporters.
GLUT 3 Neurons, Placenta, Kidney
These insulin-sensitive tissues only take up glucose
GLUT 4 Heart, Skeletal Muscle, Adipose Tissue
from the bloodstream to any significant extent in the
presence of the hormone. GLUT 5 Small Intestine, Testis, Sperm Kidney

As insulin secretion falls in the fasting state, so that GLUT 6 Spleen, Leukocytes
the receptors are internalized again, reducing glucose GLUT 7 Liver Endoplasmic reticulum
uptake. GLUT 8 Testis, Blastocyst, Adipose tissue, Brain
GLUT 9 Liver, Kidney
19. Glucose transporter in myocyte stimulated by
GLUT 12 Heart, Prostate, mammary gland, White adipose
insulin is: (AIIMS Nov 2009) tissue
m

a. GLUT-1
co

b. GLUT-2
22. Facilitated transport of glucose that is insulin
c. GLUT-3
insensitive (non-dependent) takes place in:
d. GLUT-4
a. Skeletal muscle
Ans. d. GLUT-4. (Ref: Harper 29/e p158, 30/e p191)
b. Liver
Glucose transporters which are insulin-responsive, are
c. Adipose tissue
GLUT 4, GLUT 8 and GLUT 12.
d. Heart
• GLUT 4 is present in the Heart, skeletal muscle,
adipose tissue Ans. b. Liver. (Ref: Harper 29/e p158, 30/e p191)
• GLUT 8 is present in the Testis, blastocyst Insulin responsive Glucose transporters are
• GLUT 12 is present in Heart, prostate, white adipose • GLUT 4-Heart, Skeletal Muscle, Adipose tissue
tissue, mammary gland. • GLUT 8-Testis, Blastocyst, Brain, Adipose tissue
• GLUT 12-Heart, Prostate, White adipose tissue,
m

20. Defect in renal glucosuria: (NBE Pattern Q) mammary gland.


co

a. GLUT 1
b. GLUT 2 23. Glucose transporter present in the RBC:
c. SGLT 1 (NBE pattern Qn)
d. SGLT 2 a. GLUT-1
Ans. d. SGLT2. (Ref: Harrison 19/e p299) b. GLUT-2
Renal Glucosuria c. GLUT-3
Isolated glucosuria in the presence of a normal blood d. GLUT-4
glucose concentration is due to mutations in SLC5A2, Ans. a. GLUT-1. (Ref: Harper 29/e p158, 30/e p191)
the gene that encodes the high-capacity sodium-glucose • Highest level of GLUT1 is present in the RBC
co-transporter SGLT2 in the proximal renal tubule • Major Glucose transporter in brain is GLUT1 (not
present in neurons)
m

21. Regarding glucose transporters [GLUT], incorrect


match is: • Major Glucose transporter in the Placenta is GLUT 1
co

a. GLUT-5: intestines and kidney • Major Glucose transporter in the RBC is GLUT1
b. GLUT-4: adipose tissue • Major neuronal Glucose transporter is GLUT3
e

e
m

m
m
co

136 |  
Self Assessment and Review of Biochemistry

• Insulin-responsive glucose transporter is GLUT4, c. Enterokinase


GLUT 8 and GLUT 12 d. Carboxypeptidase
• Fructose transporter GLUT 5 (mainly) and GLUT 11 Ans. b. Sucrase. (Ref: Harper 30/e p538)
• Urate transporter is GLUT 9 • The cells of brush border of intestine contain
• Glucose transporter in blastocyst is GLUT 8. the enzyme, Sucrase, Maltase, Iso-maltase and
lactase hydrolyse corresponding disaccharides to
Digestion and Absorption of Carbohydates
m

monosaccharides which are absorbed.


co

24. Inulinlike fructosans are used as prebiotics as


Digestion of Carbohydrates
they are nondigestible. Resistance to digestion in
the upper GI tract results from: (AI 2010) Amylases Catalyze the Hydrolysis of Starch
a. Absence of digestive enzyme in the upper GIT The hydrolysis of starch is catalyzed by salivary and
b. Beta configuration of anomeric C2 pancreatic amylases, which catalyze random hydrolysis
c. Low pH of the stomach of (α1 → 4) glycoside bonds, yielding dextrins, then
d. Presence of α-Glycosidic linkage a mixture of glucose, maltose, and maltotriose and
Ans. b. Beta configuration of anomeric C2. small branched dextrins (from the branchpoints in
(Ref: Harper 29/e p137, 30/e p156) amylopectin).
• Inulin is a polysaccharide of fructose (and hence a Disaccharidases are Brush Border Enzymes
fructosan) The disaccharidases, maltase, sucrase-isomaltase
m

• Inulin consists of fructose polymer linked β 2 → 1 (a bifunctional enzyme catalyzing hydrolysis of sucrose
co

• Found in tubers and roots of dahlias, artichokes, and and isomaltose), lactase, and trehalase are located on the
dandelions brush border of the intestinal mucosal cells. They act on
• It is readily soluble in water and is used to determine corresponding disaccharides and monosaccharides are
the glomerular filtration rate formed.
• It is not hydrolyzed by intestinal enzymes as Enzyme Monosaccharides formed
mammals lack any enzyme that hydrolyzes the β1 Maltase Glucose + Glucose
→ 4 bonds.
Sucrase-Isomaltase Glucose + Fructose
25. Complex polysaccharides are converted to glucose Glucose + Glucose
and absorbed with the help of: (Kerala 2007) Lactase Galactose + Glucose
a. Na+ K+ ATPase Trehalase Glucose + Glucose
b. Sucrase
m
co
m
co
e

e
m

m
m
co

5 Metabolism of
Carbohydrates
m
co

Topics Included
• Major metabolic Pathways • Minor Metabolic Pathways
– Glycolysis – HMP Shunt Pathway
– Uronic Acid Pathway
– Gluconeogenesis
– Polyol Pathway
– Glycogenesis – Fructose Metabolism
– Glycogenolysis – Galactose Metabolism
m
co

GLYCOLYSIS (EMBDEN-MEYERHOF PATHWAY)


m
co
m
co

Fig. 5.1: Glycolysis


e

e
m

m
m
co

138 | Self Assessment and Review of Biochemistry

Biochemical Significances of Glycolysis • In the liver, function of Glucokinase is to remove


• Principal route for Carbohydrate metabolism glucose from portal vein following a meal
• Pathway taking place in all the cells of the body • In the beta cells of Pancreas, function of Glucokinase
• Only pathway which can operate aerobically and anaerobically is to release Insulin.
• The ability to operate glycolysis in the absence of oxygen is
important in Skeletal muscle Fates of Glucose 6 Phosphate
• Skeletal muscles can survive anoxic episodes because of • HMP Shunt Pathway
m

anaerobic glycolysis
• Gluconeogenesis
co

• Defect in muscle Phosphofructokinase manifest as fatigue because


of its significance in muscle • Glycogenesis (Major fate in well fed state) (AIIMS May 93)
• Heart muscle has relatively low glycolytic activity, hence poor • Glycogenolysis.
survival under conditions of ischemia
• Mature Erythrocyte which lack mitochondria are completely reliant Step II
on Glucose as their metabolic fuelQ AIIMS May 2015
• Isomerization of Glucose 6 Phosphate to Fructose 6
• Defect in Glycolytic enzyme like Pyruvate Kinase manifest
as hemolytic anemia, because of its significance in mature
Phosphate by Phosphohexose Isomerase
erythrocytes. • Involves Aldose–Ketose Isomerism

Step III
Overview of Glycolysis • Fructose 6 Phosphate is phosphorylated to Fructose
1, 6 Bisphosphate by Phosphofructokinase I (PFK-I)
• Second irreversible step
m

• Major regulatory step of GlycolysisQ


co

• CommittedQ step of Glycolysis because once Fructose


1, 6 Bisphosphate is formed it should undergo
glycolysis
• Otherwise called bottle neck of Glycolysis
• 1 ATP is utilized.

Step IV
• Fructose 1, 6 Bisphosphate split to 2,3 Carbon
Fig. 5.2: Overview of gycolysis compounds Glyceraldehyde 3 Phosphate and
Dihydroxyacetone phosphate
Steps of Glycolysis
• By the Enzyme Aldolase
Site–Cytoplasm
• Aldolase is a LyaseQ.
m

Step 1-Hexokinase/Glucokinase
co

Step V
Hexokinase • Dihydroxyacetone Phosphate isomerized to Glyceral-
• Transfer Phosphate group from ATP to Glucose dehyde 3 Phosphate by Triose Phosphate Isomerase.
• Has high affinity for glucose (Or Lower Km)
Step VI
• Mg2+ is the cofactor
• Glyceradehyde 3 Phosphate is oxidized to a high
• Irreversible step
energy compound, 1,3 bisphosphoglycerate
• ATP is utilized.
• By the enzyme Glyceraldehyde 3 Phosphate Dehy-
Glucokinase drogenase
• Present in Liver and Pancreatic β cells • NADH is generated
• Has low affinity for Glucose. (High Km) • An inorganic Phosphate is addedNBE patternQ2013
• Hence acts only when blood glucose is very high • NADH generated in this step enter into mitochondria
m

(> 100 mg/dl) by Malate –Aspartate Shuttle or Glycerophosphate


co

• Induced by Insulin following a mealQ shuttle under aerobic conditions


• Play an important role in regulation of blood • But in anaerobic conditions NADH is utilized by
glucose.Q Lactate Dehydrogenase, NAD + is regenerated.
e

e
m

m
m
co

Metabolism of Carbohydrates  | 139

Step VII Substrate Level PhosphorylationQ


• 1,3 Bisphosphoglycerate to 3 Phosphoglycerate • Phospho Glycerate Kinase [1,3 Bisphosphoglycerate
• By the enzyme 1,3 Bisphosphoglycerate Kinase to 3 Phosphoglycerate]
• Only Kinase in Glycolysis which is reversible • Pyruvate Kinase [Phosphoenol Pyruvate to Pyruvate].
• ATP is formed NB: Learn the enzyme and the reaction. Question can be
• An example of Substrate level PhosphorylationQ. asked in either ways.
m

Step VIII Inhibitors of Glycolysis


co

• 3 Phosphoglycerate to 2 Phosphoglycerate • Iodoacetate inhibit Glyceraldehyde 3 Phosphate


• By Phosphoglycerate Mutase. Dehydrogenase
• Fluoride inhibit Enolase
Step IX ‒ Application: For Estimation of Blood Glucose:
• 2 Phosphoglycerate to Phosphoenol Pyruvate Sodium Fluoride: Potassium Oxalate Mixture
• This step involves dehydration is used
• By the enzyme Enolase • Arsenite is toxic to 1,3 Bisphosphoglycerate Kinase
• Enolase is dependent on Mn2+ and Mg2+ step.
• Fluoride inhibit Enolase.
CLINICAL CORRELATION–GLYCOLYSIS
Step X
• Inherited Aldolase A and pyruvate Kinase deficiency
m

• Phosphoenol Pyruvate to PyruvateQ


in erythrocyte leads to Hemolytic anemia
co

• By Pyruvate Kinase
• Muscle Phosphofructokinase deficiency leads to
• Second substrate level Phosphorylation
Exercise Intolerance.
• ATP is generated
• Irreversible step. Energetics of Aerobic GlycolysisQ
Reducing equivalents/ ATP per mol-
ANAEROBIC GLYCOLYSIS Enzyme ATP from the step ecule of Glucose
Glyceraldehyde 3 NADH = 2.5 ATPs 2 NADH = 5ATPs
• Pyruvate to Lactate by the enzyme Lactate Dehy- Phosphate Dehy-
drogenase drogenase
• NADH is utilized in this step 1, 3 Bisphospho- 1 ATP by substrate level 2 ATPs
• NAD+ is regenerated. Glycerate Kinase Phosphorylation

Remember Pyruvate Kinase 1 ATP by Substrate level 2 ATPs


m

Phosphorylation
Tissues that derive much of their energy from Glycolysis and produce
co

lactate or in other words, tissues that depend mainly on Glucose as The number of ATPs generated 9 ATPs
metabolic fuel Consumption of ATPs in the Hexokinase and -2ATP
• White fibers of Skeletal muscle Phosphofructokinase
• Mature Erythrocytes
No of ATPs from Aerobic Glycolysis 9–2 = 7 ATPs
• Brain
• Gastrointestinal Tract
• Renal Medulla Energetics of Anaerobic Glycolysis
• Skin
• Many Cancer cells. Reducing equivalents/ ATP per mole-
Enzyme ATP from the step cule of Glucose
1,3 Bisphosphoglyc- 1 ATP by Substrate level 2 ATPs
Irreversible Steps of GlycolysisQ erate Kinase Phosphorylation
• Hexokinase Pyruvate Kinase 1 ATP by Substrate level 2 ATPs
• Phosphofructokinase Phosphorylation
m

• Pyruvate Kinase. The number of ATPs generated 4ATPs


co

Remember Consumption of ATPs in the HexoKinase and -2ATP


Phosphofructokinase
All the Kinases are irreversible except 1,3 Bisphosphoglycerate
Kinase which is reversible. No of ATPs from Anaerobic Glycolysis 4–2 = 2 ATPs
e

e
m

m
m
co

140 | Self Assessment and Review of Biochemistry

Energy yield from 1 mol of Glucose under aerobic Glucagon inhibit glycolysis
condition • Increasing cAMP dependent Protein Kinase A
Source No of ATPs generated • By Phosphorylating key Enzymes of Glycolysis.
From Aerobic Glycolysis 7 ATPs Allosteric RegulationQ DNB 2000
From Pyruvate Dehydrogenase (as 2 2 NADH = 5ATPs
Pyruvates from I mol of Glucose) Enzyme Allosteric activator Allosteric inhibitor
Hexokinase Glucose-6-phosphate
m

From TCA Cycle (As 2 Acetyl CoA from 2 × 10 = 20 ATPs


I mol of Glucose) PFK-1 5’ AMP ATPQ
co

Fructose-6- Phosphate, CitrateQ Low pH


Net ATPs from 1 mol of Glucose under 7 + 5 + 20 = 32 ATPs Fructose 2,6 Bisphosphate.
aerobic condition Pyruvate Kinase ATP
Net ATPs from 1 mol of Glucose under 4 – 2 = 2 ATPs
anaerobic condition Remember
• PFK-1 plays a key role in regulation of Glycolysis
Remember • Glucokinase play a key role in regulating blood glucose level
• The number of ATPs produced from 1 NADH if Malate shuttle is following a meal.
used for transport of NADH into mitochondria is 2.5 ATPs.
• The number of ATPs produced from 1 NADH if Glycerophosphate RAPAPORT-LEUBERING CYCLE
shuttle is used for transport of NADH into mitochondria is only
1.5 ATPs. (2,3 BPG SHUNT)
• If Muscle Glycogen is used for anaerobic glycolysis, then 3 ATPs
are produced instead of 2 ATPs because, as there is no Glucose • Site: Mature erythrocytes
6 Phosphatase in muscle Glucose 6 Phosphate directly enter into • The reaction catalyzed by phosphoglycerate kinase
Glycolysis. Hence 1 ATP for Hexokinase step is not needed. So may be bypassed
m

out of 4 ATPs produced only 1 is utilized, resulting in 3 ATPs from


• 1,3-bisphosphoglycerate is converted to 2,3-bispho-
co

1 mol of Glucose.
sphoglycerate by bisphosphoglycerate 2,3- bisphos-
Key Concept of regulation of all metabolic pathways phoglycerate
Hormonal regulation • 2,3 Bisphosphoglycerate is hydrolyzed to 3-phos-
• Insulin generally favor all pathways which decrease blood glucose
phoglycerate and Pi by 2,3-bisphosphoglycerate
level by dephosphorylating the regulatory enzymes of these
pathways phosphatase mutase
• In other words enzymes active under the influence of insulin is • No ATP is generated by this step
active in the dephosphorylated state • 2 ATPs at Pyruvate Kinase step is generated but that
• Glucagon generally favor all pathways which increase blood
glucose level by phosphorylating the regulatory enzymes of is used for Hexokinase and Phosphofructokinase
these pathways • So no net yield of ATPs 2,3 BPG shunt pathway
• In other words enzymes active under the influence of glucagon is • Serve to provide 2,3-bisphosphoglycerateQ
active in the phosphorylated state.
Allosteric Regulation • 2,3 BPG shifts the oxygen Dissociation curve to right.
m

• Substrate favor forward reaction


• Product inhibits forward reaction.
co

REGULATION OF GLYCOLYSIS
Glycolysis is regulated at three physiologically irreversible
steps
1. Hexokinase/Glucokinase
2. Phosphofructokinase-I (Occupies a key position in
the regulation of Glycolysis)
3. Pyruvate Kinase.

Hormonal Regulation
m

Insulin favor glycolysis


co

• By dephosphorylating key enzymes of Glycolysis


• By Inducing Glucokinase
• Glucokinase play a key role in regulating blood
glucose level after a meal. Fig. 5.3: Rapaport-Leubering cycle
e

e
m

m
m
co

Metabolism of Carbohydrates  | 141

Points to Ponder
Net no of ATPs produced from 1 mol of Glucose by
• Anaerobic Glycolysis-2 ATPs
• Aerobic Glycolysis-7 ATPs
• Aerobic oxidation-32 ATPs
• Rapaport-Leubering Cycle-0
m
co

Cancer Cells and Metabolic Reprogramming


A comparison of Warburg effect and recent data about Clinical application of metabolic reprogramming
altered metabolism in cancer cells Warburg effect
• Early detection of cancer: Blood and urine mass
In 1924, Otto Warburg and his colleagues observed spectrometry to look for altered metabolic profile
that cancer cells take up large amount of glucose and helps in the early detection of cancer
metabolize it to lactic acid even in presence of oxygen.
• Development of anticancer drugs: Chemicals that inhibit
This observation is called Warburg effect. glycolysis will selectively kill cancer cells.
The hypothesis made by him based on this data were:
Compound Enzyme inhibited
• Increased ratio of glycolysis when compared to
aerobic respiration was likely due to defect in 3 Bromo-pyruvate Hexokinase II

mitochondrial respiratory chain 2 Deoxy d glucose Hexokinase I


m

• Enhanced glycolysis help cancer cells to preferentially Dichloroaceate Pyruvate Dehydrogenase Kinase
co

proliferate in reduced oxygen tension Iodoacetate Glyceraldehyde 3 Phosphate dehydrogenase


• He also argued that switch from aerobic to anaerobic
glucose metabolism was the driver of tumorigenesis.
FATES OF PYRUVATE
Recent data-Metabolic reprogramming of cancer cells
Rather than overt defect in mitochondrial respiratory • To Glucose (Gluconeogenesis)
chain, a metabolic reprogramming is typically observed • To Lactate (Lactate Dehydrogenase)
in tumor cells. • To Oxaloacetate (Pyruvate Carboxylase)
Metabolic Enzyme reprogramming • To Acetyl CoA (Pyruvate Dehydrogenase)
Genetic changes in specific metabolic enzyme encoding • To Alanine (Alanine Amino Transferase).
gene, preferentially express mRNA splice variants. The
mutated enzymes produce oncometabolites, that regulate
m

gene expression by epigenetic mechanism.


co

Metabolic reprogramming of Glycolysis result in


• Less shuttling of glucose –derived chemical energy
by Glycolysis into production of ATP
• Shunting of glucose derived chemical energy for
building up of cellular biomass of proteins, lipids, etc.
These explain the increased rate of glycolysis in tumor
cells.
Increased anaerobic Glycolysis in tumor cells despite
Fig. 5.4: Fates of pyruvate
angiogenesis can be explained by:
• Despite angiogenesis there is local areas of poor
PYRUVATE DEHYDROGENASE (PDH)
m

blood supply
COMPLEX
co

• This results in low oxygen tension


• Induction of Hypoxia-inducible factor-1 (HIF-1) • Pyruvate formed in the cytosol enter the mitochondria
• Upregulate the genes controlling glycolysis. by a symporter
e

e
m

m
m
co

142 | Self Assessment and Review of Biochemistry

• Pyruvate is oxidatively decarboxylated to Acetyl CoA Regulation of Pyruvate Dehydrogenase


• The oxidation of Pyruvate to Acetyl-CoA is the By end product inhibition and covalent modification
irreversible route from glycolysis to the citric Acid End products that inhibit PDH Complex are
Cycle • Acetyl CoA
• No alternate pathway to circumvent this step • NADH
• Pyruvate dehydrogenase complex is analogous to the By Covalent modification
m

α-ketoglutarate dehydrogenase
PDH is active in dephosphorylated state and inactive in
co

• Multienzyme complex associated with the inner phosphorylated state


mitochondrial membrane.
PDH is phosphorylated by a PDH Kinase
PDH Kinase is activated by increase in:
• ATP/ADP
• Acetyl CoA/CoA
• NADH/NAD+
PDH is dephosphorylated by PDH Phosphatase
• Insulin favor PDH Phosphatase, hence dephospho-
rylate PDH Complex and PDH is active.
Significance of PDH Complex
m

• Thiamine deficiency affect PDH. Hence complete oxidation of


co

Glucose
• PDH defect can lead to Lactic AcidosisQ
• Fat cannot be converted to Glucose because of the irreversible
nature of PDH
• Acetyl CoAQ cannot be converted to glucose.
Exception to fat cannot be converted to Glucose
• Glycerol part of Triacyl Glycerol
• Odd Chain Fatty Acid oxidation which forms Propionyl CoA.

Fate of Acetyl CoA (AI 09, DNB 08, 09, AIIMS May 03)
• Fatty Acid Synthesis
• Ketone Body Synthesis
m

• Cholesterol Synthesis
• TCA Cycle
co

Fig. 5.5: Pyruvate dehydrogenase complex Acetyl CoA cannot be converted to glucose.Q

PDH complex consist of 3 Enzymes and 5 Coenzymes


Three enzymes are
• Pyruvate Dehydrogenase bound to Thiamin
Diphosphate
• Dihydrolipoyl Transacetylase, the prosthetic group
is oxidised Lipomide
• Dihydrolipoyl Dehydrogenase, contains FAD.
CoenzymesQ (Very important)
• Thiamine Diphosphate (TDP)
m

• Lipomide
co

• Coenzyme A
• FAD
• NAD+ Fig. 5.6: Cori’s cycle
e

e
m

m
m
co

Metabolism of Carbohydrates  | 143

Cori’s Cycle (Glucose-Lactate Cycle) Substrates for GluconeogenesisQ


(Lactic Acid Cycle) (very important)
Lactate, formed by glycolysis in skeletal muscle and • Glucogenic Amino Acid [Alanine Q is the major
erythrocytes, is transported to the liver and kidney where contributor]
it reforms glucose, which again becomes available via the • Lactate
circulation for oxidation in the tissues. This process is • Glycerol
known as the Cori cycle, or the lactic acid cycle.
m

• Propionyl CoA.
Uses
co

• Prevents Lactate accumulation in the muscle Sites of GluconeogenesisQ


• Reutilize lactate from muscle and erythrocyte for LiverQ (60–90%) KidneyQ (10–40%)
Gluconeogenesis. OrganelleQ-Cytoplasm and Mitochondria
Cori’s cycle involves Remember
• Liver and Kidney Key Gluconeogenic enzymes are expressed in the small intestine
but it is unclear that significant Gluconeogenesis in intestine during
• Muscle starvation.
• RBC.

Glucose Alanine Cycle (Cahill Cycle) Biomedical Significance of Gluconeogenesis


In the fasting state, there is a considerable output of • Provides a major contribution to blood glucose after
alanine from skeletal muscle formed by transamination overnight fast, once Glycogen stores are depleted
m

of pyruvate produced by glycolysis of muscle glycogen, • A supply of glucose is essential for erythrocytes and
co

and is exported to the liver, where, after transamination brain


back to pyruvate, it is a substrate for gluconeogenesis. • Glucose is important for maintaining the intermediates
This is glucose-alanine cycle. It provides an indirect way of Citric Acid Cycle
of utilizing muscle glycogen to maintain blood glucose • Gluconeogenesis clears the lactate produced in
in the fasting state. the erythrocytes and skeletal muscle and glycerol
produced in the adipose tissue
• Excessive gluconeogenesis occur in critically ill
patients in response to injury and infection
• Excessive gluconeogenesis is contributory factor to
hyperglycemia in type II Diabetes mellitus
• Involves Glycolysis, the Citric Acid Cycle, plus some
m

special reactions.
co

GLUCONEOGENESIS–STEPS
1. Pyruvate and Phosphoenolpyruvate
Reversal of the reaction catalyzed by pyruvate kinase in
Fig. 5.7: Glucose alanine cycle glycolysis involves two endothermic reactions.
Uses of glucose alanine cycle • Pyruvate Carboxylase
• Carries amino group to the Liver • Phosphoenolpyruvate Carboxykinase (PEPCK).
• Alanine as a substrate for Gluconeogenesis during Pyruvate Carboxylase
starvation • Mitochondrial pyruvate carboxylase catalyzes the
• Amino acid increased in blood during starvation is carboxylation of Pyruvate to Oxaloacetate
Alanine.(AIIMS Nov 2011) • It is an ATP-requiring reaction
m

• Biotin is the coenzyme


co

GLUCONEOGENESIS • The resultant oxaloacetate is reduced to malate,


Definition: The process of formation of glucose or exported from the mitochondrion into the cytosol
Glycogen from noncarbohydrate precursors. and there oxidized back to oxaloacetate.
e

e
m

m
m
co

144 | Self Assessment and Review of Biochemistry


m
co
m
co
m
co
m
co

Fig. 5.8:Gluconeogenesis
e

e
m

m
m
co

Metabolism of Carbohydrates  | 145

Phosphoenolpyruvate Carboxykinase • Then undergoes isomerization to succinyl-CoA


• Catalyzes the decarboxylation and phosphorylation catalyzed by methylmalonyl-CoA mutase, vitamin
of oxaloacetate to phosphoenolpyruvate using GTP B12 – dependent enzyme.
as the phosphate donor.
Sources of Propionyl CoA in Humans
2. Fructose 1,6-Bisphosphate and Fructose • β oxidation of Fatty acid
6-Phosphate
m

• Oxidation of Isoleucine
Fructose 1,6 Bisphosphatase
co

• Side chain of Cholesterol.


• The conversion of fructose 1,6-bisphosphate to
fructose 6-phosphate, for the reversal of glycolysis,
is catalyzed by fructose 1,6-bisphosphatase.

3. Glucose 6-Phosphate and Glucose


Glucose 6 Phosphatase
The conversion of glucose 6-phosphate to glucose is
catalyzed by glucose 6-phosphatase.
It is present in liver and kidney, but absent from
muscle and adipose tissue, which, therefore,
cannot export glucose into the bloodstream.
m

Fig. 5.9: Entry of propionyl CoA to gluconeogenesis


Glucose 1-Phosphate and Glycogen
co

Glycogen synthesis involves a different pathway via


uridine diphosphate glucose and glycogen synthase.
Clinical Correlation
Methylmalonic aciduria
Summary of Key Enzymes of Gluconeogenesis Methylmalonyl-CoA mutase is a vitamin B12-dependent
Irreversible steps of Key enzymes to bypass the irre- enzyme, and in deficiency methylmalonic acid is excreted in
glycolysis versible steps in gluconeogenesis the urine (methylmalonic aciduria)
1. Pyruvate Kinase Pyruvate Carboxylase (Mitochondria)
• Enzymes Common to Glycolysis and Gluco-
Convert Phosphoenol Convert Pyruvate to Oxaloacetate
neogenesis(AI 97, Kerala 2007)
Pyruvate to Pyruvate Oxaloacetate transported to the
Cytosol by Malate ShuttleQ • All the enzymes other than the irreversible enzymes
Phosphoenol Pyruvate Carboxy in the glycolysis
Kinase (PEPCK) (Cytosol) Convert • One mol of lactate converted to Glucose-6 ATPs are
m

Oxaloacetate to Phosphoenol utilized.


co

Pyruvate
2. Phosphofructokinase Fructose 1,6 Bisphosphatase
[Cytosol] RECIPROCAL REGULATION OF
3. Hexokinase/Glucokinase Glucose 6 Phosphatase [Cytosol] GLUCONEOGENESIS AND GLYCOLYSIS
Since Glycolysis and Gluconeogenesis share the same
Entry of Propionyl CoA to Gluconeogenesis pathway but in opposite directions, they must be
By three enzymes regulated reciprocally.
1. Propionyl CoA Carboxylase Three mechanisms are responsible for regulating
2. Methyl Malonyl CoA Racemase the activity of enzymes concerned in carbohydrate
3. Methyl Malonyl CoA Mutase metabolism:
• Propionyl-CoA is carboxylated to D-methyl-malonyl-
1. Changes in the Rate of Enzyme Synthesis—By
m

CoA, catalyzed by propionyl-CoA carboxylase, a


biotin-dependent enzyme Induction and Repression
co

• Methylmalonyl-CoA racemase catalyzes the conver- • Insulin, secreted in response to increased blood
sion of D-methylmalonyl-CoA to L-methylmalonyl- glucose, enhances the synthesis of the key enzymes
CoA in glycolysis
e

e
m

m
m
co

146 | Self Assessment and Review of Biochemistry

• Pyruvate Carboxylase is repressed by


Insulin (DNB 09, AIIMS May 2013
• Insulin also antagonizes the effect of the
glucocorticoids and glucagon-stimulated cAMP,
which induce synthesis of the key enzymes of
gluconeogenesis.
m

Fig. 5.10: Action of bifunctional enzyme


that regulate glycolysis and gluconeogenesis
co

2. Covalent Modification by Reversible


Phosphorylation–By means of Hormones
In Well Fed State
Epinephrine and Glucagon • Insulin dephosphorylate the tandem enzyme
• Increasing the concentration of cAMP • PFK-II part is active and F2, 6 BPase is inactive
• This in turn activates cAMP-dependent protein • Level of Fructose 2,6 Bisphosphate rises
kinase • This favors Glycolysis
• Leading to the phosphorylation and inactivation of • But Gluconeogenesis is inactive
pyruvate kinase. • Decreases the blood Glucose.
Insulin
In the Fasting State
• Decrease the concentration of cAMP
• Glucagon phosphorylate the tandem enzyme by
m

• Dephosphorylate the key enzymes of Gluconeogenesis cAMP dependent Protein Kinase


co

and become inactive.


• PFK-II is inactive and F2,6 BPase is active
3. Allosteric Modification • Level of Fructose 2,6 Bisphosphate falls
• This favors Gluconeogenesis
By Acetyl CoA and Fructose 2,6 Bisphosphate
• But Glycolysis is inactive
Acetyl CoA • Increases the blood Glucose.
• Acetyl-CoA as an allosteric activatorQQQ of Pyruvate
Fructose 2, 6 Bisphosphate reciprocally regulate
Carboxylase.
Glycolysis and Gluconeogenesis
• This ensures provision of Oxaloacetate, so that Acetyl
Phosphofructo- Fructose 2,6
CoA can be oxidised by Citric acid cycle. Characteristics kinase-II Bisphosphatase
Fructose 2,6 Bisphosphate Reaction Fructose 6 Phos- Fructose 2,6
m

First we learn about the tandem enzyme that synthesize phate to Fructose 2,6 Bisphosphate to
co

Bisphosphate Fructose 6 Phos-


Fructose 2,6 Bisphosphate.
phate Remember
the reverse reaction
Tandem Enzyme (Bifunctional Enzyme) of PFK-II
• Single polypeptide with two enzyme activity Hormonal Favored by Insulin Favored by Glu-
• Two enzyme activities are Phosphofructokinase–II regulation cagon
(PFK-II) and Fructose 2,6 Bisphosphatase (F2,6 Covalent Modifi- Active in dephosphor- Active in Phosphor-
BPase). cation ylated state ylated state.
Dietary regulation Active in well fed state Active in fasting
Action of the tandem enzyme
state.
• PFK-II Convert Fructose 6 Phosphate to Fructose 2,6
Reciprocal regula- Fructose 2,6 Bisphos- Decreases the
Bisphosphate
tion of Glycolysis phate, the product of level of Fructose 2,6
• F2, 6 BPase Convert Fructose 2,6 Bisphosphate to
m

& Gluconeogen- PFK-II favor Glycoly- Bisphosphate, there


Fructose 6 Phosphate esis sis. inhibit Gluconeo- by favor Gluconeo-
co

• Fructose 2,6 Bisphosphate, the product of PFK-II, is genesis genesis inhibit Gly-
colysis.
an allosteric activator of PFK-I
e

e
m

m
m
co

Metabolism of Carbohydrates  | 147

Contd...
Features Liver Muscle
Regulation of blood Contributes Does not directly contri-
glucose to blood glu- bute to blood glucose.
cose But serves as a source of
energy to muscle itself
Glucose 6 Phosphatase Present Absent Q
m
co

Remember
After 12–18 hour of fasting, liver glycogen is almost totally
depleted.Q

Muscle Glycogen and Gluconeogenesis


Muscle does not contribute directly to blood Glucose but Pyruvate formed
by Glycolysis, is transaminated to Alanine. This is transported to Liver,
which is used for Gluconeogenesis. This is Glucose Alanine Cycle.

GLYCOGENESIS
• Occurs mainly in Muscle and Liver
• Organelle-Cytosol
• Rate Limiting Enzyme: Glycogen Synthase.
m

Glycogenesis–Steps
co

Synthesis of UDP glucose


• Glucose converted to Glucose 6 Phosphate by
Hexokinase in muscle/Glucokinase in liver
• Glucose 6 Phosphate isomerized to Glucose 1
Phosphate Phosphoglucomutase
• Glucose 1 Phosphate react with UTP to form UDP
Glucose and Pyrophosphate catalyzed by UDP
Fig. 5.11: Reciprocal regulation of glycolysis and Glucose PyroPhosphorylase
gluconeogenesis
• UDP Glucose is the Glucose donor for Glycogen
GLYCOGEN METABOLISM Synthesis.
Initiation of glycogen synthesis
• Glycogen Synthesis (Glycogenesis) • Glycogen Synthase is the enzyme that joins Glucose
m

• Glycogen Degradation (Glycogenolysis) residues by α 1,4 Linkage (C1 of UDP Glucose and
co

‒ Glycogen is the major storage carbohydrate in C- 4 of the terminal glucose residue in the Glycogen,
animals liberating UDP)
‒ Glycogen is present mainly in liver and muscle, • But Glycogen Synthase can do this only on a pre-exist-
with modest amount in brain. ing Glycogen molecule or a primer called Glycogenin
• Glycogen Synthase adds Glucose residue on this
Structure of Glycogen Glycogen Primer.
• Branched homopolysaccharide made up of α D
Glycogenin
Glucose.
Glycogenin is a 37 kDa proteinQNBE pattern that is glucosylated
Linkage on a specific tyrosine residue by UDPGlc.
• α 1,4 linkage at unbranched points Formation of branch points
• α 1,6 linkage at branching points. • When the chain is at least 11 glucose residues long,
Differences between liver glycogen and muscle glycogen branching enzyme acts
m

• Branching enzyme transfers at least six glucose


co

Features Liver Muscle residues to a neighboring chain to form a α 1,6


Total Glycogen content Less (1.8 Kg) Highest (35 Kg) linkage, establishing a branch point
Percentage by tissue weight Highest (5.0) Less (0.7) • Branches grow by further addition of α 1 → 4 glucosyl
Contd... units.
e

e
m

m
m
co

148 | Self Assessment and Review of Biochemistry


m
co
m
co

Fig. 5.12: Pathways of glycogenesis and glycogenolysis


m
co
m
co

Fig. 5.13: Synthesis of glycogen


e

e
m

m
m
co

Metabolism of Carbohydrates  | 149

GLYCOGENOLYSIS • Second part is a α1,6 Glucosidase (Amylo 1,6


Glucosidase)
• Occurs in the Muscle and Liver
‒ Hydrolyse the branching point
• Organelle Mainly Cytoplasm [Small proportion in ‒ Releases free Glucose NOT Glucose 1 Phosphate.
the Lysosomes]
Conversion of Glucose 1 Phosphate to free Glucose
• Rate Limiting Enzyme-Glycogen Phosphorylase
• Glucose 1 Phosphate to Glucose 6 Phosphate by
m

• PLP is a CoenzymeQ of Glycogen Phosphorylase. Phosphoglucomutase


co

Isoenzymes of Glycogen Phosphorylase • Glucose 6 Phosphate to Glucose by Glucose 6


• Present in Muscle, Liver and Brain. Phosphatase
• Glycogen Phosphorylase BB is a Cardiac Biomarker. • Glucose 6 Phosphatase is present in the smooth
endoplasmic reticulum
• A transporter is required for the transport of Glucose
Steps of Glycogenolysis 6 Phosphate from SER to cytoplasm
Breaking of α 1, 4 linkage • Defect in the Glucose 6 Phosphate transporter lead
• Glycogen Phosphorylase cleave the α 1,4 linkage to Type Ib Glycogen Storage disorder.
• Releases Glucose 1 Phosphate NOT free Glucose
Minor Pathways of Glycogenolysis
• Glycogen Phosphorylase stops its action when it is
• Taking place inside lysosomes
atleast 4 glucose residues from a branch point.
• By the enzyme Acid maltase
m

Removal of Branches
• Glycogen is hydrolyzed to Glucose
co

By a bifunctional enzyme: • This is important in glucose homeostasis in neonates.


• First part is a α-1, 4 α 1,4 Glucan transferase • Genetic lack of Lysosomal acid Maltase lead to Type
‒ Transfer trisaccharide residue to another forming II Glycogen Storage Disorder (Pompe’s Disease or
a new α 1, 4 linkage. Type II GSD)
m
co

Fig. 5.14: Glycogenolysis

Remember • Glucagon and Epinephrine favor Glycogenolysis by


Enzyme common to Glycogenesis and Glycogenolysis is Phospho- phosphorylating Glycogen Phosphorylase.
glucomutaseQ (AIIMS May 2014) • Glycogen Phosphorylase active in the phosphoryla-
ted state.
Regulation of Glycogen Metabolism • Phosphorylase a: Active state
Basic Concepts • Phosphorylase b: Inactive state.
m

• Insulin favor Glycogenesis by dephosphorylating,


co

Rate Limiting Steps


Glycogen Synthase.
• Glycogenesis: Glycogen Synthase
• Glycogen Synthase active in dephosphorylated
state. • Glycogenolysis: Glycogen Phosphorylase.
e

e
m

m
m
co

150 | Self Assessment and Review of Biochemistry

Hormonal Regulation Contd...


Insulin • In an exercising muscle, epinephrine favor Glycogenolysis for
supplying energy to the muscle
• Insulin dephosphorylate Glycogen Synthase and
• Epinephrine has action only in the Muscle.
Glycogen Phosphorylase
• Glycogen Synthase is active in the dephosphorylated
state Differences between muscle and liver in the regulation
of glycogen metabolism
m

• Glycogen Phosphorylase is inactive in the dephos-


• Epinephrine acts in Muscle and Liver where as
co

phorylated state
• So Glycogen is synthesized. Glucagon acts only in the Liver
Glucagon (In Liver) and Epinephrine (In liver and Muscle) • In the muscle there is cAMP-independent activation
• Phosphosphorylate Glycogen Phosphorylase and of glycogenolysisQ
Glycogen Synthase ‒ By the stimulation of a Ca2+/calmodulin-sensitive
• Glycogen Phosphorylase active in the phosphoryla- phosphorylase kinase
ted state ‒ Phosphorylate Glycogen Phosphorylase in the
• Glycogen Synthase inactive in the phosphorylated muscle
state ‒ Favor glycogenolysis.
• So, Glycogen is degraded. • Muscle phosphorylase can be activated without
Remember phosphorylation.
m

• Well fed state under the influence of Insulin store excess ‒ Muscle Phosphorylase has a binding site for
carbohydrate as Glycogen
co

• In fasting state under the influence of Glucagon, Glycogenolysis 5’AMP. 5’AMP is an allosteric activator without
takes place in the liver to supply Glucose phosphorylation
Contd... ‒ Favor Glycogenolysis.
m
co
m
co

Fig. 5.15: cAMP dependent and cAMP independent mechanism regulation of glycogenphosphorylase
e

e
m

m
m
co

Metabolism of Carbohydrates  | 151

Mechanism of action of Glucagon and Contd...


Epinephrine on Glycogen Metabolism
• Glucagon/Epinephrine bind to its receptor
• Inactive Adenylyl Cyclase is converted to Active
Adenylyl Cyclase
• Adenylyl Cyclase convert ATP to cAMP
m

• cAMP activate inactive Protein Kinase A to active


co

Protein Kinase A
• Phosphorylate Phosphorylase Kinase
• Phosphorylase Kinase b (Inactive) is now Phosphory-
lase Kinase a (Active)
Fig. 5.16: Allosteric regulation of glycogen
• This Phosphorylate Glycogen Phosphorylase metabolism in muscle and liver
• Glycogen Phosphorylase b (Inactive) is now Glycogen
Allosteric regulators of Glycogen synthase and Glyco-
Phosphorylase a (Active)
gen phosphorylase
• Glycogen is degraded (Ref. Figure 5.11).
Allosteric in-
Organ Enzyme Allosteric Activator
hibitor
Mechanism of Action of Insulin on Glycogen Liver Glycogen Syn- Glucose 6 Phos- ----
Metabolism
m

thase phate
co

• Insulin increases the activity of Phosphodiesterase, Glycogen ----- Glucose


which hydrolyses cAMP to 5’ AMP Phosphorylase Glucose 6
Phosphate, ATP
• Thus insulin terminate the action of cAMP
Muscle Glycogen Glucose 6 Phos- -----
• Increase the activity of Protein Phosphatase Synthase phate
• This dephosphorylate Glycogen Synthase and Glycogen Ca++ AMP Glucose 6
Phosphorylase Kinase Phosphorylase Phosphate, ATP

• So Glycogen Synthase is active


• Glycogen Phosphorylase is inactive GLYCOGEN STORAGE DISORDERS
• Hence, glycogen Synthesis takes place. (VERY IMPORTANT TOPIC)
Group of inherited disorders characterized by deposition
ALLOSTERIC REGULATION OF of an abnormal type or quantity of glycogen in tissues,
m

GLYCOGEN SYNTHASE or failure to mobilize glycogen.


co

Common Glycogen Storage Disorders


Liver Glycogen Storage Disorder
Enzyme effi-
Type Name ciency Characteristics
0 —– Glycogen syn- Early morning drowsiness
thase and fatigue, fasting
hypoglycemia, and keto-
sis; early death [No Hepa-
tomegaly]
Ia Von Glucose 6-phos- Glycogen accumulation
Gierke’s phataseQ in liver and renal tubule
disease cells (Kidney Enlarged)
m

hypoglycemia; elevated
co

blood lactate, cholesterol,


triglyceride, and uric acid
Fig. 5.16 (Contd...) levels

Contd...
e

e
m

m
m
co

152 | Self Assessment and Review of Biochemistry

Contd... Clinical Presentation


Enzyme effi- Most commonly present at 3–4 months of age with
Type Name ciency Characteristics
• Doll like facies with fat cheeks
Ib —– Endoplasmic re- Same as type Ia, with
ticulum glucose additional findings of
• Relatively thin extremities
6-phosphate neutropenia and impaired • Short stature, Protuberant abdomen
transporter neutrophil function, • Massive Hepatomegaly
m

Recurrent Bacterial Infec-


tion, Inflammatory Bowel • Kidneys are also enlarged
co

Disease • No Splenomegaly
III Limit dex- Liver and mus- Fasting hypoglycemia • Plasma may be milky due to associated hypertri-
trinosis, cle debranching hepatomegaly in in-
Forbe’s enzyme (Amylo fancy accumulation of
glyceridemia.
or Cori’s 1,6 Glucosi- characteristic branched Type Ib has additional features of recurrent bacterial
disease dase) polysaccharide (limit dex- infection due to neutropenia and impaired neutrophil
trin) muscle weakness,
dysfunction.
elevated transaminase
levels; liver symptoms The biochemical hallmarks are
can progress to liver fail- • Hypoglycemia
ure later in life • Lactic Acidosis
IV Amylo- Branching en- Hepatosplenomegaly • Hyperlipidemia
pectinosis, zyme Accumulation of polysac- • Hyperuricemia
Ander- charide with few branch
m

sen’s dis- points


ease
Long term effects of Type I GSD are PCOD, Pancreatitis,
co

Failure to thrive, hypoto-


nia, hepatomegaly, sple- Hepatic Adenomas, Pulmonary hypertension, Osteopenia,
nomegaly, progressive Renal disease.
cirrhosis (death usually
before 5th yr), elevated Type III GSD (Limit Dextrinosis)
transaminase levels
• Autosomal Recessive
VI Hers’ dis- Liver phosphory- Hepatomegaly
ease lase
• The gene located on chr 1p21.
VIII Liver phosphory- Biochemical Defect
lase kinase
Fanconi Glucose trans- Failure to thrive, rickets,
• Debranching enzyme is defective
Bickel porter 2 hepatorenomegaly, proxi- • Abnormal glycogen with short outer branch chain
Syndrome (GLUT-2) mal renal tubular dysfunc- resembling limit dextrin accumulate.
QNBE pattern
tion, impaired glucose
and galactose utilization
m

Clinical Features
• Hypoglycaemia, Hepatomegaly, hyperlipidemia,
co

Type Ia GSD-Von Gierke’s Disease short stature, variable skeletal muscle myopathy
• Most common Glycogen Storage Disorder in • Kidneys are not enlarged
childhood • Splenomegaly may be present
• Autosomal Recessive • Progressive liver cirrhosis and failure occurs.
• The gene for glucose 6 phosphatase is located on
chr 11q23 Definite Diagnosis
• Muscle not affected because Glucose 6 Phosphate • Enzyme assay in liver, muscle, or both
absent in the muscles • Mutation analysis can provide a noninvasive method
• Structure of Glycogen normal. for diagnosis and subtype assignment in the majority
of patients.
Biochemical Defect
m

• Type Ia GSD-Glucose 6 Phosphatase absent or Type IV Glycogen Storage Disease


(Amylopectinosis, or Andersen Disease)
co

deficient in liver, kidney and intestinal mucosa


• Type Ib GSD-Translocase that transport Glucose 6 • Autosomal recessive
Phosphate across endoplasmic reticulum membrane • The glycogen branching enzyme gene is located on
is defective. chromosome 3p21.
e

e
m

m
m
co

Metabolism of Carbohydrates  | 153

Biochemical Defect Contd...


• Deficiency of Branching Enzyme activity results in Type Name Enzyme defect Characteristics
accumulation of an abnormal glycogen with poor V McArdle’s Muscle phos- Poor exercise
solubility syndrome phorylaseQ tolerance mus-
cle glycogen
• The disease is referred to as type IV GSD or amy- abnormally high
lopectinosis because the abnormal glycogen has a VII Tarui’s disease Muscle and Poor exercise
m

structure resembling amylopectin. erythrocyte phos- tolerance; Hem-


co

phofructokinase 1 olytic anemia


Clinical Manifestations myoglobinuria
• This disorder is clinically variable
• The most common and classic form is characterized Pompe Disease (Type II GSD)
by progressive cirrhosis of the liver and is manifested Autosomal recessive
in the 1st 18 mo of life as hepatosplenomegaly and
failure to thrive Biochemical Defect
• The cirrhosis progresses to portal hypertension, • Deficiency of acid α-1, 4-glucosidase (acid maltase),
ascites, esophageal varices, and liver failure that an enzyme responsible for the degradation of
usually leads to death by 5 years of age. glycogen in lysosomes
• The gene for acid α-glucosidase is on chromosome
Diagnosis 17q
m

• Tissue deposition of amylopectin-like materials • Lysosomal glycogen accumulation in multiple tissues


co

can be demonstrated in liver, heart, muscle, skin, and cell types, with cardiac, skeletal, and smooth
intestine, brain, spinal cord, and peripheral nerve muscle cells being the most seriously affected.
• The hepatic histologic findings are characterized by
micronodular cirrhosis Clinical Picture
Present in the 1st few months of life
• Electron microscopy shows accumulation of the
fibrillar aggregations that are typical of amylopectin. • Hypotonia
• A generalized muscle weakness with a ‘floppy infant’
The definitive diagnosis
appearance
• Demonstration of the deficient branching enzyme
• Feeding difficulties
activity in liver, muscle, cultured skin fibroblasts,
or leukocytes • Macroglossia
• Identification of disease-causing mutations in the • Hepatomegaly
m

glycogen branching enzyme (GBE) gene • Hypertrophic cardiomyopathy


co

• Prenatal diagnosis is possible by measuring the • Followed by death from cardiorespiratory failure or
enzyme activity in cultured amniocytes, chorionic respiratory infection usually by 1 year of age. Juvenile
villi, or mutation analysis. and adult-onset disease (late-onset Pompe disease) is
characterized by a lack or absence of severe cardiac
Muscle Glycogen Storage Disorders involvement and a less severe short-term prognosis.
Type Name Enzyme defect Characteristics
Laboratory Diagnosis
II Pompes Disease Lysosomal α1,4 Cardiomegaly,
and α1,6 glu- hypotonia,
• Serum creatine kinase, aspartate aminotransferase,
(Belongs to ly-
sosomal storage cosidase (acid hepatomegaly; lactate dehydrogenase, acid Phospahatase
disorder) maltase)Q cardiorespira- • Chest x-ray showing massive cardiomegaly
tory failure lead-
ing to death by • Electrocardiographic findings include a high-voltage
age 2 year QRS complex and a shortened PR interval. Echocar-
m

Danon disease Lysosome- Hypertrophic diography reveals thickening of both ventricles and/
co

associated mem- cardiomyopathy or the intraventricular septum and/or left ventricular


brane protein 2 Rare X linked outflow tract obstruction
(LAMP2)
• Muscle biopsy shows the presence of vacuoles that
Contd... stain positively for glycogen
e

e
m

m
m
co

154 | Self Assessment and Review of Biochemistry

• Electron microscopy reveals glycogen accumulation • Lack of an increase in blood lactate levels and exag-
within the membranous sac and in the cytoplasm. gerated blood ammonia elevations indicate muscle
Electromyography reveals myopathic features with glycogenosis
excessive electrical irritability of muscle fibers and • Phosphorus magnetic resonance imaging (31P MRI)
pseudomyotonic discharges allows for the noninvasive evaluation of muscle
• Enzyme assayin muscle, cultured skin fibroblasts metabolism
m

• Gene sequencing. • Enzyme Studies.


co

Treatment Treatment
Specific enzyme replacement therapy (ERT) with recom- • Avoidance of strenuous exercise
binant human acid α-glucosidase (alglucosidase alfa, • Glucose or sucrose given before exercise or injection
(Myozyme) is available for treatment of Pompe disease. of glucagon can markedly improve tolerance in these
Glycogen Storage Diseases Mimicking Hypertrophic Cardio- patients
myopathy • Vitamin B6 supplementation reduces exercise
• Lysosomal-associated membrane protein 2 (LAMP2, also called
intolerance and muscle cramps.
Danon disease)
• AMP-activated protein kinase γ2 (PRKAG2)
Type VII Glycogen Storage
– Both result in accumulation of glycogen in the heart and skeletal
muscle. Disease (Tarui Disease)
• Autosomal recessive disorder
m

Type V Glycogen Storage Disease • The gene for muscle phosphofructokinase is located
co

(McArdle Disease) on chromosome 12q13.3.


• Type V GSD is an autosomal recessive disorder. Biochemical Defect
Biochemical Defect Deficiency of muscle phosphofructokinase (M Isoenzyme
form), which catalyzes the ATP-dependent conversion of
• Muscle Phosphorylase Defect
fructose-6-phosphate to fructose-1,6-diphosphate.
• The gene for muscle phosphorylase (PYGM) has been
Isoforms of PFK and defect in Tarui Disease
mapped to chromosome 11q13
• Phosphofructokinase is composed of 3 isoenzyme
• Lack of this enzyme limits muscle ATP generation
subunits (M [muscle], L [liver], and P [platelet]) that
by glycogenolysis, resulting in muscle glycogen
are encoded by different genes and differentially
accumulation, and is the prototype of muscle energy
expressed in tissues
disorders.
m

• Skeletal muscle contains only the M subunit, and red


co

Clinical Manifestations blood cells contain a hybrid of L and M forms


• Symptoms usually 1st develop in late childhood or as • Type VII disease is due to a defective M isoenzyme,
an adult and are characterized by exercise intolerance which causes a complete enzyme defect in muscle
with muscle cramps and pain and a partial defect in red blood cells.
• Many patients experience a characteristic ‘second
Clinical Manifestations
wind’ phenomenon. If they slow down or pause
briefly at the 1st appearance of muscle pain, they can • Exercise intolerance
resume exercise with more ease • Compensated hemolysis
• About 50% of patients report burgundy-colored • Hyperuricemia is common and exaggerated by
urine after exercise, which is the consequence of muscle exercise
exercise-induced myoglobinuria secondary to • An abnormal polysaccharide is present in muscle
fibers; it is periodic acid–Schiff positive but resistant
m

rhabdomyolysis.
to diastase digestion
co

Diagnosis • Exercise intolerance is particularly acute after meals


• Ischemic exercise test offers a rapid diagnostic that are rich in carbohydrates because glucose cannot
screening for patients with a metabolic myopathy be utilized in muscle and because glucose inhibits
e

e
m

m
m
co

Metabolism of Carbohydrates  | 155

lipolysis and thus deprives muscle of fatty acid and HMP Shunt Pathway has Two Phases
ketone substrates Three molecules of Glucose 6 phosphate give rise to
• There is no spontaneous second-wind phenomenon three molecules of CO2 and three 5 Carbon sugars. They
because of the inability to metabolize blood glucose. are rearranged to regenerate two molecules of Glucose
6 Phosphate and one molecule of Glyceraldehyde 3
Diagnosis Phosphate. This is taking place in two phases.
m

• Enzymatic defect demonstrated in muscle 1. Oxidative Phase


co

• The absence of the M isoenzyme of phosphofruc- 2. Nonoxidative Phase


tokinase can also be demonstrated in blood cells and
fibroblasts. Characteristics of the two phases of HMP shunt pathway
Oxidative Phase—Characteristics and Organs
Treatment Sites: Liver, Adipose Tissue, Adrenal Cortex, Erythrocytes,
• There is no specific treatment Gonads, Thyroid, Lactating Mammary Glands (Not in
• Avoidance of strenuous exercise is advisable to non lactating mammary glands)
prevent acute attacks of muscle cramps and myo- • Irreversible
globinuria. • Takes place in sites where NADPH is required
Glycogen Storage Disorders (GSDs) at a glance • Glucose 6 Phosphate undergoes dehydrogenation
• Most common GSD in adolescent and adults: Type V GSD
and decarboxylation to Ribulose 5 Phosphate
(McArdles Disease)
• Oxidation is achieved by dehydrogenation using
m

• Liver GSD disorder causes fasting hypoglycemia and hepato-


megaly NADP+, not NAD+, as the hydrogen acceptor
co

• GSDs associated with liver cirrhosis: Type III, Type IV, Type IX • Produce NADPH(TNPGEE 2008) DNB 2012
GSDs
• GSD associated with renal dysfunction: Type I GSD
• Liver GSD with myopathy: Type III GSD and Type IV GSD
• Liver GSD with neurological (brain and anterior horn cells)
involvement: Type II GSD

MINOR METABOLIC PATHWAYS


• HMP Shunt Pathway
• Uronic Acid Pathway
• Polyol Pathway
m

• Galactose Metabolism
co

• Fructose Metabolism.
Fig. 5.17: Oxidative phase of HMP pathway
HMP Shunt Pathway
• Other names: Pentose Phosphate Pathway, Dicken Uses of NADPHQ
Horecker Pathway, Phosphogluconate Pathway • Used to prevent oxidative Damage—RBC, Lens by
• Organelle-Cytosol keeping glutathione in the reduced state
• Rate limiting step-Glucose 6 Phosphate Dehydro- • Reductive biosynthesisQ of fatty acids and Steroids.
genase. Nonoxidative Phase regenerate Glucose 6 Phosphate
Sites: In rapidly dividing cells bone marrow, skin,
Biochemical Significances of HMP Pathway intestinal mucosa and virtually in all tissues.
• Alternative route for metabolism of Glucose • Reversible
m

• Complete oxidation of Glucose • Produce Pentoses


co

• More complex pathway than Glycolysis • Ribulose 5-phosphate is converted back to glucose
• Major function is to generate NADPH and Riboses 6-phosphate mainly by two transketolase reaction
• No ATP is generated by this pathway. NBE Pattern QAI-08) and one transaldolase.
e

e
m

m
m
co

156 | Self Assessment and Review of Biochemistry


m
co
m
co

Fig. 5.18: Nonoxidative phase of HMP pathway

Transketolase Contd...
Two transketolase reactions • Ribose 5 Phosphate
• Transfers the two-carbon unit comprising carbons • Glyceraldehyde 3 Phosphate
1 and 2 of a ketose onto the aldehyde carbon of an • Sedoheptulose 7 Phosphate
aldose sugar • Fructose 6 phosphate
• Erythrose 4 Phosphate
• It therefore affects the conversion of a ketose sugar
into an aldose with two carbons less and an aldose Key points to remember in HMP (Pentose Phosphate) Pathway
m

sugar into a ketose with two carbons more • Main source of NADPHQ2012 and Pentoses.
co

• Require thiamine as coenzyme • Two transketolation and one transaldolation reactions are involved.
• No ATP is producedQ
• Erythrocyte transketolaseQ is a measure thiamine • CO2 is produced in this pathway and not in glycolytic PathwayQ
status of the body. DNB 01)
• Deficiency of Glucose 6 phosphate dehydrogenase is a major
Transaldolase cause of acute hemolysis in erythrocytes
One transaldolase reaction • NADPH is used for reductive biosynthetic pathways, like fatty
acid synthesis, steroid synthesis, Amino acids by Glutamate
Transfer the three carbon unit of a 7 carbon keto dehydrogenase
sugar (sedoheptulose 7 phosphate) to a 3 C aldosugar • NADPH is required for regeneration of reduced Glutathione, that
(Glyceraldehyde 3 Phosphate) to form a 6 carbon Keto clears free radicals from erythrocytes and lens.
Sugar (Fructose 6 Phosphate) and 4 carbon aldo Sugar.
Clinical Correlation—HMP Pathway
No cofactor for this enzyme.
Glucose 6 Phosphate Dehydrogenase Deficiency
Metabolites in HMP Shunt Pathway Most common enzyme deficiency in human beings.
m

• Glucose 6 Phosphate X-linked recessive


co

• 6 Phosphogluconate Manifest as
• Ribulose 5 Phosphate • Hemolytic Anemia
• Xylulose 5 Phosphate • Methemoglobinemia
Contd... Contd...
e

e
m

m
m
co

Metabolism of Carbohydrates  | 157

Contd... • L-Xylulose excreted in urine


Due to decrease in NADPH which clears the free radicals in the • Gives Benedict’s Test Positive
RBCs and keep the iron in the hemoglobin in the reduced state.
• Bial’s Test Positive.
These people are sensitive to certain drugs like
• Primaquine, Aspirin, Sulfadrugs Various drugs increase the rate at which glucose enters
• Consumption of favabeans (vicia fava) can also precipitate the uronic acid pathway.
hemolysis. (Favism) For example, administration of barbital or chlorobutanol
m

to rats results in a significant increase in the conversion


co

Uronic Acid Pathway of glucose to glucuronate, L-gulonate, and ascorbate.


• An alternative oxidative pathway for glucose Aminopyrine and antipyrine increase the excretion of
• UDP Glucose converted to UDP Glucuronic Acid L-xylulose in pentosuric subjects.
(Glucuronate) by using NAD+(DNB 93) (TNPGEE 89) Pentosuria also occurs after consumption of relatively
• No ATP is formed large amounts of fruits such as pears that are rich sources
• Site-liver of pentoses (alimentary pentosuria).
• Organelle-cytosol
Polyol/Sorbitol Pathway
• To produce Fructose from Glucose
• Responsible for occurrence of Fructose in seminal
fluid
m

• May be responsible for the pathogenesis of Diabetic


co

Cataract
• The pathway is increasingly seen as glucose concen-
tration rises in those tissues which are not insulin sen-
sitive, like the lens, peripheral nerves, renal glomeruli
• Glucose is reduced to Sorbitol by Aldose Reductase.
• Sorbitol is oxidized to Fructose by Sorbitol dehy-
drogenase
• Sorbitol is responsible for Diabetic cataract because,
it cannot pass through cell membrane so accumulate,
causing osmotic damage.

Metabolism of Galactose
m

Biochemical Role
Galactose is seen in:
co

• To Produce Glucuronic Acid


• Lactose
‒ Source of Glucuronate for the synthesis of
Glycosaminoglycans and Proteoglycans • Glycolipids
‒ For Phase II Conjugation reaction (Conjugation • Glycoproteins
of Bilirubin and many drugs). • GAG (Keratan Sulfate) (TNPGEE 2010)
• To produce Ascorbic Acid Galactose is derived from intestinal hydrolysis of the
‒ But not in higher primates and mammals disaccharide lactose, the sugar of milk. It is readily
‒ Because they lack L-Gulonolactone OxidaseQ converted in the liver to glucose.
• To Produce Pentoses Conversion of Galactose to Glucose
Clinical Correlation: Uronic Acid Pathway Rate limiting Step-Galactose 1 Phosphate uridyl Trans-
Essential Pentosuria a defect in Uronic Acid PathwayQ ferase
m

• Benign condition Site: Galactose is metabolized exclusively in the Liver,


co

• One among the Garrod’s tetrad (Pentosuria, Albi- hence Galactose Tolerance Test is a Liver Function Test.Q
nism, Cystinuria, Alkaptonuria) • Galactokinase catalyzes the phosphorylation of
• Due to deficiency of Xylitol DehydrogenaseQ 2012 galactose, using ATP as phosphate donor
e

e
m

m
m
co

158 | Self Assessment and Review of Biochemistry

• Galactose 1-phosphate reacts with UDPGlc to form Diagnosis


uridine diphosphate galactose (UDPGal) and glucose • Demonstrating a reducing substance in several urine
1-phosphate, in a reaction catalyzed by galactose specimens collected while the patient is receiving
1-phosphate uridyl transferase human milk, cow’s milk, or any other formula
• The conversion of UDPGal to UDPGlc is catalyzed containing lactose
by UDPGal 4-epimerase • Benedict’s test positiveQ
m

• The UDPGlc is then incorporated into glycogen • Chromatography for presence of Galactose in urine.
co

• In the synthesis of lactose in the mammary gland, • Clinistix urine test results are usually negative
UDPGal condenses with glucose to yield lactose, because the test materials rely on the action of
catalyzed by lactose synthase. glucose oxidase, which is specific for glucose and is
nonreactive with galactose (Glucose Oxidase Test is
negative)
• Mucic Acid Test Positive
• Galactose Tolerance Test is contraindicated
• Direct enzyme assay using erythrocytes establishes
the diagnosis.
Treatment
• Lactose free diet till 4-5 years of age
m

• Because galactose-1-phosphate pyrophosphorylase


becomes active by 4–5 years. This enzyme reduces
co

the level of galactose-1-phosphate


• Breast milk is avoidedQ
Fig. 5.19: Metabolism of galactose
Metabolism of Fructose
Galactosemia (Very Important Topic)
Enzyme deficiency (AIIMS Nov 2011, May 2013)
• Galactose-1-phosphate uridyltransferase [Classic
Galactosemia]
• Galactokinase
• UDP Hexose 4 Epimerase.
The newborn infant receives high amounts of lactose,
m

which consists of equal parts of glucose and galactose.


co

Without the galactose-1-phosphate uridyltrasferase


enzyme, the infant is unable to metabolize galactose-1-
phosphate, the accumulation of which results in injury
to kidney, liver, and brain.
Clinical features
• Classic galactosemia is a serious disease with onset
of symptoms typically by the 2nd half of the 1st
week of life
Fig. 5.20: Metabolism of fructose
• With jaundice, vomiting, seizures, lethargy, irritabil-
ity, feeding difficulties, poor weight gain or failure Key Points in the Metabolism of Fructose
to regain birth weight Fructose undergoes more rapid glycolysis in the liver
m

• Hepatomegaly Oil drop cataracts, Hepatic failure than does glucose, because it bypasses the regulatory
co

• Mental retardation step catalyzed by phosphofructokinase.


• Patients with galactosemia are at increased risk for This allows fructose to flood the pathways in the
Escherichia coli neonatal sepsis. liver, leading to enhanced fatty acid synthesis, increased
e

e
m

m
m
co

Metabolism of Carbohydrates  | 159

esterification of fatty acids, and increased VLDL secretion, Essential Fructosuria (Benign Fructosuria)
which may raise serum triacylglycerols and ultimately • Autosomal recessive. Benign condition
raise LDL cholesterol concentrations. • It is an accidental finding usually made because the
Fructokinase, in liver, kidney, and intestine, catalyzes asymptomatic patient’s urine contains a reducing
the phosphorylation of fructose to fructose-1-phosphate. substance.
Unlike glucokinase, its activity is not affected by fasting Biochemical defect
m

or by insulin, which may explain why fructose is cleared


• Fructokinase defect
co

from the blood of diabetic patients at a normal rate


• The gene encoding fructokinase is located on
Fructose-1-phosphate is cleaved to D-glyceraldehyde chromosome 2p23.3
and dihydroxyacetone phosphate by aldolase B. • Fructokinase catalyzes the 1st step of metabolism of
Clinical Correlations dietary fructose: conversion of fructose to fructose-1-
phosphate
Hereditary fructose intolerance
• Without this enzyme, ingested fructose is not
Autosomal Recessive metabolized. Its level is increased in the blood, and
Biochemical defect it is excreted in urine because there is practically no
• Deficiency of fructose-1,6-bisphosphate Aldolase renal threshold for fructose.
(Aldolase B) Lab diagnosis
• The gene for aldolase B is on chromosome 9q22.3 • Rapid furfural test and Seliwanoff’s test positive
m

• Deficiency of this enzyme activity causes a rapid • Clinistix for reducing sugars
co

accumulation of fructose-1-phosphate and initiates • Urine chromatography for fructose.


severe toxic symptoms when exposed to fructose. Loading of the Liver with Fructose may Potentiate Hypertriacyl-
Clinical manifestations glycerolemia, Hypercholesterolemia, and Hyperuricemia
• In the liver, fructose increases fatty acid and triacylglycerol synthesis
• Patients with HFI are asymptomatic until fructose or and VLDL secretion, leading to hypertriacylglycerolemia—and
sucrose (table sugar) is ingested (usually from fruit, increased LDL cholesterol—which can be regarded as potentially
fruit juice, or sweetened cereal) atherogenic.
• Acute loading of the liver with fructose, as can occur with
• Symptoms may occur early in life, soon after birth intravenous infusion or following very high fructose intakes, causes
if foods or formulas containing these sugars are sequestration of inorganic phosphate in fructose-1-phosphate and
introduced into the diet diminished.
• Early clinical manifestations resemble galactosemia
and include jaundice, hepatomegaly, vomiting,
METABOLISM OF CARBOHYDRATES
m

lethargy, irritability, and convulsions, hypoglycemia ATP synthesis. As a result, there is less inhibition of de
co

• Acute fructose ingestion produces symptomatic novo purine synthesis by ATP, and uric acid formation
hypoglycemia. If the intake of the fructose persists, is increased, causing hyperuricemia, which is the cause
hypoglycemic episodes recur, and liver and kidney of gout.
failure progress, eventually leading to death Since fructose is absorbed from the small intestine by
• Chronic ingestion results in failure to thrive. (passive) carrier-mediated diffusion, high oral doses may
lead to osmotic diarrhea.
Laboratory diagnosis
Fructose and Sorbitol in the Lens Are Associated with Diabetic
• Prolonged clotting time, hypoalbuminemia, elevation Cataract
of bilirubin and transaminase levels • Both fructose and sorbitol are found in the lens of the eye in
• Proximal tubular dysfunction increased concentrations in diabetes mellitus and may be involved
in the pathogenesis of diabetic cataract. The sorbitol (polyol)
• Definitive diagnosis is made by assay of fructaldolase pathway (not found in liver) is responsible for fructose formation
m

B activity in the liver from glucose


co

• Gene-based diagnosis for the mutation. • The glucose concentration rises in those tissues that are not insulin-
sensitive, i.e. the lens, peripheral nerves, and renal glomeruli
Treatment • This increases activity of Sorbitol pathway
Complete exclusion of all sources of fructose. Contd...
e

e
m

m
m
co

160 | Self Assessment and Review of Biochemistry

Contd... • Uptake of Glucose by GLUT-2 is insulin independent


• Glucose is reduced to sorbitol by aldose reductase, followed But Glucokinase is an inducible enzyme by insulin,
by oxidation of sorbitol to fructose in the presence of NAD+ and when Glucose is excess
sorbitol dehydrogenase (polyol dehydrogenase)
• Decreased Gluconeogenesis.
• Sorbitol does not diffuse through cell membranes, but accumulates,
causing osmotic damage. Simultaneously, myoinositol levels fall Fat metabolism
• In experimental animals, sorbitol accumulation and myoinositol
• Favor LipogenesisQ
m

depletion, as well as diabetic cataract, can be prevented by aldose


reductase inhibitors. • Increased Fatty Acid Synthesis
co

• Increased Triacyl Glycerol Synthesis.


INTEGRATION OF METABOLISM In adipose tissue
• Favor the transport of GLUT-4 from intracellular
Major Metabolic Fuel of Major OrgansQ vesicle to the cell surfaceQ
Organ Major metabolic fuels • Favor the uptake of glucose
Liver Free fatty acids, glucose (in fed state), lactate, • Inhibit hormone sensitive LipaseQ-Inhibit Lipolysis
glycerol, fructose, amino acids, alcohol
Favor
Brain Glucose, amino acids, ketone bodies in pro-
longed starvation • Fatty acid synthesis. Favor Triacylglycerol Synthesis.
HeartQ Ketone bodies, free fatty acids, lactate, chylo- In the muscle
micron and VLDL triacylglycerol, some glucose
• Glucose uptake by insulin dependent GLUT-4
m

Adipose tissue Glucose, chylomicron and VLDL triacylglycerol


• Favor Glycolysis
co

Fast Twitch Muscle Glucose, glycogen


• Favor Glycogenesis.
Slow Twitch Muscle Ketone bodies, chylomicron and VLDL tria-
cylglycerol Post-absorptive Phases
Kidney Free fatty acids, lactate, glycerol, glucose No fuel from gut. Plasma insulin Decreases, Glucagon
Erythrocyte Glucose level begin to rise. Increased cAMP level–Increased cAMP
dependent Protein Kinase A.
Most of the enzyme active in postabsorptive phase are
STARVE FEED CYCLE
in Phosphorylated state.
Divided into 5 stages: Glucagon
• Well-fed state (1–4 hours after food) • Activate Adenylyl Cyclase
• Early fasting (4–16 hours after food) • Increase cAMP level.
m

• Fasting (16–48 hours after food)


Insulin
co

• Starvation (2–3 days without food)


• Favor Phosphodiesterase which convert cAMP to
• Prolonged starvation (> 5 days without food).
5’AMP
Stage I Well-Fed State • Decrease cAMP level.(DNB 2011, AIIMS May 95)
• Exogenous dietary supply of Fuel from the Intestine
Stage II Early Fasting (4–16 Hours after Food)
• Glucose is the major fuel
Fuel from the gut stopped
• Insulin is the hormone: Most of the enzyme regulated
On Carbohydrate metabolism
by covalent modification is in the Dephosphorylated
• Increased break down of liver glycogen (not Muscle)
State
• Decreased Glycogenesis.
• Favor Glucose utilization and storage of excess
glucose.Q On Lipid Metabolism
Decreased Lipogenesis
m

In the Liver
co

On Carbohydrate Metabolism Stage III Fasting (16–48 Hours After Food)


• Favor Glycolysis • No fuel from the gut
• Favor Glycogenesis • By 12–18 hours liver GlycogenQ stores are depleted
e

e
m

m
m
co

Metabolism of Carbohydrates  | 161

• Hepatic Gluconeogenesis favored Stage V Prolonged Starvation (after 5 days)


• Muscle Proteins are degraded to supply amino group Increased Proteolysis: Muscle Wasting Cachexia
to Pyruvate to form Alanine
• This Alanine reaches the liver used for gluconeogenesis.
This is Glucose Alanine Cycle.
m
co

Stage IV Starvation or Prolonged Fasting


(2–4 Days Without Food)
• No fuel from the gut
• Decreased Gluconeogenesis
• Increased activity of hormone sensitivity lipase
• Increased lipolysis, Free Fatty Acid level rises
• Increased Ketone Bodies synthesis
• Ketone Bodies inhibit Muscle proteolysis. Fig. 5.21: Biochemical changes in fasting

REVIEW QUESTIONS
m
co

Glycolysis Contd...

1. Irreversible steps of Glycolysis are catalyzed by: Metabolic Pathway Site


(AIIMS May 2013) Glycogenolysis Cytoplasm and some in Lysosomes
a. Hexokinase, Phosphofructokinase, Pyruvate HMP Pathway Cytoplasm
Kinase Pyruvate Dehydrogenase Mitochondria
b. Glucokinase, Pyruvate Kinase, Glyceraldehyde Krebs Cycle Mitochondria
3 Phosphate Dehydrogenase
c. Hexokinase, Phospho Glycerate Kinase, Pyru- 3. Irreversible step(s) in Glycolysis is/are:
vate Kinase (PGI May 2012)
d. Pyruvate Kinase, Fructose 1,6 Bisphosphatase, a. Enolase
Phosphofructokinase b. Phosphofructokinase
m

Ans. a. Hexokinase, Phosphofructokinase, Pyruvate c. Pyruvate Kinase


co

Kinase (Ref: Harper 29/e p171-174, 30/e p170, 172) d. Glyceraldehyde 3 Phosphate Dehydrogenase
Remember e. Hexokinase
All the Kinases are irreversible except 1,3 Bisphosphoglycerate Ans. b, c and e. (Ref: Harper 29/e p171, 172, 30/e p170-172)
Kinase which is reversible.
Irreversible Steps of GlycolysisQ
• Hexokinase
2. Glycolysis occurs in: (AIIMS May 2007) • Phosphofructokinase
a. Cytosol • Pyruvate Kinase.
b. Mitochondria
Remember
c. Nucleus All the Kinases are irreversible except 1,3 Bisphospho Glycerate
d. Lysosome Kinase which is reversible.
Ans. a. Cytosol (Ref: Harper 29/e p171, 30/e p170)
Substrate Level PhosphorylationQ
m

Metabolic Pathway Site


• Phosphoglycerate Kinase (1,3 Bisphosphoglycerate
co

Glycolysis Cytoplasm
to 3 Phosphoglycerate)
Gluconeogenesis Cytoplasm and Mitochondria
• Pyruvate Kinase (Phosphoenolpyruvate to Pyruvate).
Glycogen Synthesis Cytoplasm NB: Learn the enzyme and the reaction. Question can be asked in
Contd... either ways.
e

e
m

m
m
co

162 | Self Assessment and Review of Biochemistry

4. Enzyme catalyzing reversible step in glycolysis is: Contd...


(PGI Nov 2010) Enzyme Inducer Repressor Activator Inhibitor
a. Phosphofructokinase Pyruvate Insulin GlucagonQ Fructose ATP
b. Enolase kinase 1,6-bispho- alanine
sphate, in- glucagon
c. Pyruvate kinase sulin norepi-
d. Phosphoglycerate mutase nephrine
m

e. Glyceraldehyde-3-Phosphate Dehydrogenase Pyruvate Insulin GlucagonQ CoA, NAD+, Acetyl


co

Ans. b, d and e. (Ref: Harper p171, 172, 30/e p170) dehydroge- insulinQ, CoAQ,
nase ADP, pyru- NADH,
5. In which of the following steps ATP is released? vate ATP
(Kerala 2008) (fatty
acids,
a. Phosphoenol pyruvate to pyruvate ketone
b. Glyceraldehyde 3 phosphate to 1,3 bisphos- bodies)
phoglycerate
c. Fructose 6 phosphate to fructose 1,6 bisphos- Gluconeogenesis
phate Repres-
Enzyme Inducer Activator Inhibitor
d. Glucose to Glucose 6 phosphate sor

Ans. a. Phosphoenol pyruvate to pyruvate Pyruvate car- Glucocor- Insulin Acetyl ADP
boxylase ticoids, CoAQ
(Ref: Harper 30/e p170, 172)
m

Glucagon,
Steps releasing ATP at the level of substrate Epinephrine
co

• 1, 3 Bisphosphoglycerate to 3 Phosphoglycerate (1, 3 Phosphoenol- Glucocor- Insulin


Bisphosphoglycerate Kinase) pyruvate car- ticoids,
boxykinase Glucagon,
• Phosphoenolpyruvate to Pyruvate. (Pyruvate Kinase) Epinephrine
• Succinyl CoA to Succinate (Succinate Thiokinase) Glucose 6- Glucocor- Insulin
phosphatase ticoids,
6. What activate Kinases of glycolysis? Glucagon,
a. ATP (NBE Pattern Q) Epinephrine
b. c AMP
c. Insulin 7. True statement about glycolysis is: (PGI Dec 98)
d. Glucagon a. Occurs in mitochondria
Ans. c. Insulin (Ref: Harper 30/e p188 Table 19-1) b. Complete break down of glucose
m

Regulation of Carbohydrate Metabolism c. Conversion of glucose to 3C units


co

(NB: This table is an important topic for all exams) d. 3 ATPs are used in anaerobic pathway
Enzyme Inducer Repressor Activator Inhibitor Ans. c. Conversion of Glucose to 3 C units
Glycogen Insulin Glucagon Insulin, glu- Glucagon (Ref: Harper 30/e p171)
synthase cose 6-phos- Option a. Glycolysis occur in cytosol
phate
Option b. Complete break down of Glucose happens
Hexokinase Glucagon Glucose
6-phos-
when Pyruvate formed by Glycolysis undergo Pyruvate
phate Dehydrogenase reaction, followed by TCA Cycle.
Glucokinase Insulin Glucagon Option d. In anaerobic Glycolysis
Phospho- Insulin GlucagonQ 5’AMP, Citrate, • Number of ATPs produced is 4
fructoki- fructose ATP, • Number of ATPs used is 2
nase-1 6-phosphate, glucagon
• Net ATP yield by anaerobic glycolysis is 2.
m

fructose 2,
6-bisphos-
co

phateQ
8. Compound that joins glycolysis with glycogenesis
Inorganic and glycogenolysis: (JIPMER 04)
Phosphate a. Glucose 1,6 bisphosphate
Contd... b. Glucose 1 PO4
e

e
m

m
m
co

Metabolism of Carbohydrates  | 163

c. Glucose 6 PO4 Regulatory steps of Glycolysis are


d. Fructose 1,6 bisphosphate • Hexokinase/Glucokinase
Ans. c. Glucose 6 Phosphate (Ref: Harper 30/e p170) • Phosphofructokinase
Fates of Glucose 6 Phosphate • Pyruvate Kinase.
• Can undergo Glycolysis Harper says Phosphofructokinase occupy a key position
• Can enter into Glycogenesis in regulating Glycolysis and is also subject to feed back
m

• Can be used for gluconeogenesis control.


co

• Is an intermediate in Glycogenolysis
12. Cancer cells derive nutrition from:
• Can enter in to HMP Pathway.
a. Glycolysis (AIIMS Nov 2001)
9. Key glycolytic enzymes: (PGI 98) b. Oxidative phosphorylation
a. Phosphofructokinase
c. Increase in mitochondria
b. Hexokinase
d. From a fast food joint
c. Pyruvate kinase
Ans. a. Glycolysis (Ref: Harper 30/e p738)
d. Glucose 1, 6 bisphosphatase
In 1924, the biochemist Otto Warburg and his colleagues
Ans. a, b and c. (Ref: Harper 30/e p170)
made the discovery that cancer cells take up large
Irreversible steps of Glycolysis are:
amounts of glucose and metabolize it to lactic acid, even in
• Hexokinase/Glucokinase the presence of oxygen. This observation was termed the
• Phosphofructokinase Warburg effect. Based on these data, Warburg made two
m

• Pyruvate Kinase. hypotheses: first, that the increased ratio of glycolysis


co

10. In glycolysis the first committed step is catalyzed to aerobic respiration was likely due to defects in the
by: (AIIMS Dec 97) mitochondrial respiratory chain; and second, that this
a. 2,3-DPG enhanced glycolysis enabled cancer cells to preferentially
b. Glucokinase proliferate in the reduced oxygen tension often seen in
c. Hexokinase tumors. Furthermore, Warburg argued that the switch
d. Phosphofructokinase from aerobic to anaerobic glucose metabolism was the
driver of tumorigenesis.
Ans. d. Phosphofructokinase (Ref: Harper 30/e p173)
• First Committed step is catalyzed by Phosphofruc- 13. True statements about glucokinase is/are:
tokinase (PGI Dec 2003)
• This is otherwise called the bottle neck of Glycolysis. a. Km value is higher than normal blood sugar
Hexokinase/Glucokinase b. Found in liver
m

• Convert Glucose to Glucose 6 Phosphate c. Glucose 6 phosphate inhibit it


co

• Glucose 6 Phosphate has different fates, not only d. Has both glucose 6 phosphatase and kinase
Glycolysis. activity
Fates of Glucose 6 Phosphate e. Glucose enter into cells through GLUT-2
• Can undergo Glycolysis Ans. a, b and e. (Ref: Harper 30/e p170, 191)
• Can enter into Glycogenesis
• Glucokinase is important in regulating blood
• Can be used for gluconeogenesis glucose after a meal
• Is an intermediate in Glycogenolysis • Glucokinase has a considerably higher Km (lower
• Can enter in to HMP Pathway. affinity) for glucose, so that its activity increases
11. The rate-limiting enzyme in glycolysis is: • With increases in the concentration of glucose in the
(AI 2000) hepatic portal vein
a. Phosphofructokinase
m

• Found in liver cells and Pancreatic Beta islet cells


b. Glucose 6-dehydrogenase
co

• Glucose 6 Phosphate inhibit Hexokinase not


c. Glucokinase Glucokinase
d. Pyruvate kinase • Glucose enter into liver cells and Pancreatic beta cells
Ans. c. Phosphofructokinase (Ref: Harper 30/e p170, 188) through GLUT2.
e

e
m

m
m
co

164 | Self Assessment and Review of Biochemistry

14. Within the RBC, hypoxia stimulates glycolysis by • 2,3 Bisphospho Glycerate is hydrolyzed to 3-phos-
which of the following regulating pathways: phoglycerate and Pi by 2,3-bisphosphoglycerate
(AI 2007) phosphatase mutase
a. Hypoxia stimulates pyruvate dehydrogenase • No ATP is generated by this step
by increased 2,3-DPG • But 2 ATPs are generated by Pyruvate Kinase
b. Hypoxia inhibits hexokinase • As 2 ATPs are utilized by Hexokinase and PFK-1, No
c. Hypoxia stimulates release of all glycolytic
m

net ATPs are generated by this pathway.


enzymes from Band 3 on RBC membrane
co

d. Activation of the regulatory enzymes by high 17. Enzyme responsible for complete oxidation of
pH glucose to CO2 and water is present in:
(AIIMS May 2007)
Ans. c. Hypoxia stimulates release of all glycolytic
enzymes from Band 3 on RBC membrane a. Cytosol
b. Mitochondria
15. All except occurs on decrease in blood glucose c. Lysosomes
level: (AI 2012) d. Endoplasmic reticulum
a. Inhibition of PFK-II Ans. b. Mitochondria (Ref: Harper 29/e p173, 30/e p172)
b. Activation of Fructose 2,6 Bisphosphatase • Under aerobic conditions, pyruvate is taken up into
c. Increase in glucagon mitochondria, and after oxidative decarboxylation to
d. Increase in Fructose 2,6 Bisphosphate acetyl-CoA is oxidized to CO2 by the citric acid cycle
Ans. d. Increase in Fructose 2, 6 Bisphosphate
m

• Under anaerobic conditions, pyruvate is reduced


(Ref: Harper 30/e p191) by the NADH to lactate, catalyzed by lactate
co

• On Decreasing Blood Glucose Level dehydrogenase.


• Glucagon is released from β cells of Pancreas
18. Substrate level phosphorylation is by:
• Increases the Blood Glucose Level by
(NBE Pattern Q)
• Inhibiting Glycolysis
a. Pyruvate kinase
‒ By phosphorylating the key enzymes of Glycolysis
b. Phosphofructokinase
‒ By decreasing the level of Fructose 2, 6 Bis-
c. Hexokinase
phosphate, the product of PFK-II
d. ATP Synthase
• Favor Gluconeogenesis
Ans. a. Pyruvate Kinase. (Ref: Harper 30e p171)
‒ By phosphorylating Key enzymes of Gluconeo-
genesis 19. The enzyme not involved in substrate level
‒ By favoring Fructose 2, 6 Bisphosphatase, which phosphorylation: (JIPMER 2014)
m

decreases the level of Fructose 2, 6 Bisphosphate, a. Pyruvate kinase


co

a potent activator of PFK-I. b. Phosphofructokinase


16. The number of ATPs produced by Rapaport c. Succinate thiokinase
leubering Cycle in RBC from Glucose? d. Phosphoglycerate kinase
(NBE pattern Qn) Ans. b. Phosphofructokinase. (Ref: Harper 30/e p169)
a. 1
Gluconeogenesis
b. 2
c. 3 20. Which of the following does not contribute to
d. 4 glucose by gluconeogenesis? (AIIMS Nov 2015)
Ans. b. 2 (Ref: Harper 30/e p173, 174) a. Lactate
• Rapaport Leubering cycle (2,3 BPG Cycle) takes place b. Acetyl CoA
in the erythrocytes c. Pyruvate
m

• The reaction catalyzed by phosphoglycerate kinase d. Oxaloacetate


co

may be bypassed Ans. b. Acetyl CoA (Ref: Harper 30/e p185, 186)
• 1,3-bisphosphoglycerate is converted to 2,3-bispho- Lactate from muscle and RBC are converted to glucose in
sphoglycerate by bisphosphoglycerate 2,3- bisphos- the liver (Coris Cycle) Lactate and Alanine is converted to
phoglycerate Pyruvate which can enter into Gluconeogenesis.
e

e
m

m
m
co

Metabolism of Carbohydrates  | 165

Oxaloacetate is converted to Phosphoenolpyruvate by Contd...


PEPCK and enter into Gluconeogenesis. Enzyme Inducer Repressor Activator Inhibitor
Glucokinase Insulin Glucagon
21. True about gluconeogenesis is: (PGI May 2013)
a. Prevent hypoglycemia during prolonged fasting Phosphofruc- Insulin GlucagonQ 5’ AMP, Citrate,
tokinase-1 fructose ATP, gluca-
b. Occur both in muscle and liver
6-phos- gon
c. Fructose 2, 6 Bisphosphate stimulate it
m

phate,
d. Excess acetyl CoA causes stimulation fructose
co

e. Carbon skeleton of amino acids are involved in 2,6-bispho-


sphateQ, Pi
gluconeogenesis
Ans. a, d and e. Pyruvate Insulin GlucagonQ Fructose ATP, ala-
kinase 1,6-bispho- nine, gluca-
• Gluconeogenesis occur in liver and Kidney
sphate, gon, nor-
• Fructose 2, 6 Bisphosphate activate PFK-I, hence insulin epinephrine
stimulate Glycolysis.
Pyruvate Insulin GlucagonQ CoA, NAD+, Acetyl
dehydroge- insulinQ, CoAQ,
22. A child with low blood glucose is unable to do
nase ADP, pyru- NADH, ATP
glycogenolysis or gluconeogenesis. Which of the vate (fatty acids,
following enzyme is missing in the child? ketone
(AIIMS Nov 2012) bodies)
a. Fructokinase
m

b. Glucokinase Gluconeogenesis
co

c. Glucose 6 Phosphatase Enzyme Inducer Repressor Activator Inhibitor


d. Transketolase Pyruvate Glucocor- Insulin Acetyl ADP
Ans. c. Glucose 6 Phosphatase carboxylase ticoids, CoAQ
(Ref: Harper 29/e p179, 181 Table 19.2, 189; 30/e p177) Glucagon,
Epinephrine
The answer should be an enzyme common to
Phospho- Glucocor- Insulin
Glycogenolysis and Gluconeogenesis.
enolpyruvate ticoids,
carboxyki- Glucagon,
23. In fasted state gluconeogenesis is promoted by nase Epinephrine
which enzyme: (AIIMS May 2012)
Glucose 6- Glucocor- Insulin
a. Acetyl CoA induced stimulation of Pyruvate phosphatase ticoids,
Carboxylase Glucagon,
b. Citrate induced stimulation of Acetyl CoA Epinephrine
m

Decarboxylase
co

c. Fructose 2,6 bisphosphate induced stimulation 24. During prolonged fasting, rate of gluconeogenesis
of Phosphofructokinase-1
is determined by: (AIIMS May 2012)
d. Stimulation of Pyruvate kinase by Fructose 1,6 a. Essential Fatty Acid in liver
Bisphosphate
b. Alanine in liver
Ans. a. Acetyl CoA induced stimulation of Pyruvate
Carboxylase c. Decreased c GMP
(Ref: Harper 29/e p190 Table 20-1; 30/e p188 Table 19.1) d. ADP in liver
Regulation of Carbohydrate Metabolism (NB: This table Ans. b. Alanine in the liver
is an important topic for all exams) (Ref: Harpers 29/e p160; 30/e p186)
Enzyme Inducer Repressor Activator Inhibitor Major Substrates for GluconeogenesisQ (AI 97)
Glycogen Insulin Glucagon Insulin, Glucagon • Glucogenic Amino Acid (Alanine Q is the major
m

synthase glucose 6- contributor)


co

phosphate
• Lactate
Hexokinase Glucagon Glucose 6-
phosphate • Glycerol
Contd...
Propionate (Major contributor in Ruminants).
e

e
m

m
m
co

166 | Self Assessment and Review of Biochemistry

25. True about gluconeogenesis is/are: (PGI May 2013) 28. Not a substrate for gluconeogenesis:
a. Prevent hypoglycemia during prolonged (NBE Pattern Q)
fasting a. Acetyl CoA
b. Occur in both muscle and liver b. Lactate
c. Fructose 2,6 bisphosphate stimulate it c. Glycerol
d. Excess of acetyl CoA stimulate it d. Propionyl CoA
m

e. Carbon skeleton of amino acid is involved Ans. a. Acetyl CoA (Ref: Harper 29/e p185, 186)
co

Ans. a, d and e. (Ref: Harper 29/e p187-191; 30/e p187) Substrates for GluconeogenesisQ
• Gluconeogenesis occur in liver and kidney not in • Glucogenic Amino Acid (Alanine Q is the major
the muscle contributor)
• Gluconeogenesis prevent hypoglycemia in prolonged • Lactate
fasting • Glycerol
• Fructose 2,6 Bisphosphate stimulate Glycolysis • Propionyl CoA.
• Excess Acetyl CoA is an allosteric activator of Pyruvate
Carboxylase, a key enzyme of Gluconeogenesis 29. Which of the following reactions takes place in
two compartments? (NBE Pattern Q)
• Carbon skeleton of gluconeogenic amino acid are
a. Gluconeogenesis
involved in gluconeogenesis.
b. Glycolysis
26. Common enzyme for gluconeogenesis and
m

glycolysis is: (Kerala 2007) c. Glycogenesis


d. Glycogenolysis
co

a. Glyceraldehyde 3 PO4 dehydrogenase


b. Hexokinase Ans. a. Gluconeogenesis (Ref: Harper 30/e p188, 189)
c. Pyruvate kinase • Gluconeogenesis takes place in cytosol and mito-
chondria
d. Pyruvate carboxylase
• The Mitochondrial step is Pyruvate Carboxylase
Ans. a. Glyceraldehyde 3 Phosphate dehydrogenase
reaction.
• An enzyme that catalyze a reversible step in Glycolysis
Pathways taking place in two compartments are
is common to both Glycolysis and Gluconeogenesis.
• Heme Synthesis
27. Phosphofructokinase-I is activated by all except: • Urea Cycle
a. 5’AMP (NBE Pattern Q) • Gluconeogenesis.
b. Fructose 2, 6 Bisphosphate
30. Glyconeogenic capability of cell is determined by
m

c. Fructose 6 Phosphate
the presence of: (PGI Dec 2005)
co

d. Citrate
a. Pyruvate dehydrogenase
Ans. d. Citrate. (Ref: Harper 30/e p188 Table 19-1)
b. Glucose-6-phosphatase
Enzyme Inducer Repressor Activator Inhibitor c. Pyruvate carboxylase
Hexoki- Glucagon Glucose 6- d. Fructose 1,6- bisphosphatase
nase phosphate
e. Pyruvate carboxykinase
Glucoki- Insulin Glucagon
nase Ans. b, c and d. (Ref: Harper 30/e p188, 189)
Phospho- Insulin Glucagon 5’ AMP, Citrate, • Glyconeogenic capacity is determined by the
fructoki- fructose ATP, glu- presence of key enzymes of Gluconeogenesis.
nase-1 6-phosphate, cagon
fructose 2,6- • Gluconeogenesis is the process of synthesizing
bisphosphate, glucose or glycogen from noncarbohydrate precursors
Inorganic
• Pyruvate Dehydrogenase and Pyruvate carboxykinase
m

Phosphate
Pyruvate Insulin Glucagon Fructose 1, 6- ATP,
are not enzymes of gluconeogenesis.
co

kinase bisphosphate, alanine,


insulin glucagon, 31. Step of Gluconeogenesis is: (NBE pattern Qn)
norepi- a. Pyruvate to Lactate
nephrine
b. Glucose 6 Phosphate to Fructose 6 Phosphate
e

e
m

m
m
co

Metabolism of Carbohydrates  | 167

c. Pyruvate to Acetyl CoA • Acetyl CoA is NEVER a substrate for gluconeogenesis


d. Oxaloacetate to Phosphoenolpyruvate • Acetyl CoA is the starting material for Fatty acid and
Ans. d. Oxaloacetate to PEP (Ref: Harper 30/e p188, 189) Cholesterol synthesis
• Pyruvate to lactate is a step in Anerobic Glycolysis • Acetyl CoA is an intermediate in Ketone body
• Glucose 6 Phosphate to Fructose 6 Phosphate is a synthesis.
step in Glycolysis
36. A genetic disorder renders fructose
m

• Pyruvate to Acetyl CoA is a step in aerobic oxidation 1,6-bisphosphatase in liver less sensitive to
co

of Glucose regulation by fructose 2,6-biphosphate. All of the


• Oxaloacetate to PEP catalyzed by PEPCK is a step in following metabolic changes are observed in this
gluconeogenesis. disorder except: AI 2004
32. Major contribution towards gluconeogenesis is a. Level of fructose 1,6-biphosphate is higher
by: AI 92 than normal
a. Lactate b. Level of fructose 1,6-biphosphate is lower than
b. Glycerol normal
c. Ketones c. Less pyruvate is formed
d. Alanine d. Less ATP is generated
Ans. d. Alanine (Ref: Harper 30/e p188, 189) Ans. b. Level of fructose 1, 6-biphosphate is lower than
normal
Alanine is the principal gluconeogenic amino acid
• The action of Fructose 1,6 Bisphosphatase on Fructose
m

33. Glucose can be synthesized from all except: (AI 96) 1,6 Bisphosphatase is decreasing its activity
co

a. Amino acids • Here as the control of Fructose 1,6 BP on Fructose 1,6


b. Glycerol Bisphosphatase is lost
c. Acetoacetate • The enzyme is more active
d. Lactic acid • So the level of Fructose 1,6 Bisphosphate is lower
Ans. c. Acetoacetate (Ref: Harper 30/e p188, 189) than normal.
Substrates for Gluconeogenesis
Glycogen Metabolism and
• Glucogenic amino acids Glycogen Storage Disorders
• Lactate
37. Why is Glucose 6 Phosphate in the cytoplasm
• Glycerol
of hepatocyte not acted upon by Glucose 6
Acetyl CoA, Acetoacetate are not substrates for gluco- Phosphatase as soon as it is formed?
neogenesis. (AIIMS Nov 2015)
m

a. Thermodynamically possible only when


co

34. Gluconeogenesis does not occur significantly


from ………………… in humans: (AIIMS 92) gluconeogenesis occurs
a. Lactate b. Needs Protein Kinase for its activation
b. Fatty acids c. Enzyme is present in SER, Glucose 6-phosphate
c. Pyruvate needs to be transported into SER
d. Aminoacid d. Steric inhibition of Phosphatase by albumin
Ans. b. Fatty acids (Ref: Harper 30/e p188, 189) Ans. c. Enzyme is present in SER, Glucose 6-phosphate
Glycerol part of fat and Propionyl CoA from odd chain     need to be transported into SER
fatty acid oxidation are Gluconeogenic part of fat. (Ref: Harper 30/e p178)
• Glucose-6-phosphatase is in the lumen of the smooth
35. Acetyl CoA can be converted into all of the endoplasmic reticulum
following except: (AI 2009) • Glucose-6-phosphate is transported to SER by a
m

a. Glucose transporter called translocase to be acted by Glucose-


b. Fatty acids
co

6-phosphatase
c. Cholesterol • Genetic defects of the Glucose-6-phosphate trans-
d. Ketone bodies porter can cause a variant of type I glycogen storage
Ans. a. Glucose (Ref: Harper 30/e p188,189) disease.
e

e
m

m
m
co

168 | Self Assessment and Review of Biochemistry

Other options: Enzyme


• Glycogenolysis provides blood glucose before Type Name efficiency Characteristics
Gluconeogenesis sets in in fasting state 0 —– Glycogen syn- Early morning drowsi-
thase ness and fatigue, fast-
• Glucose-6-phosphatase does not need Protein Kinase, ing hypoglycemia, and
but Glycogen Phosphorylase needs Protein Kinase ketosis; early death [No
Hepatomegaly]
for its activation.
m

Ia von Gierke’s Glucose-6-phos- Glycogen accumulation


38. The reasons for ketosis in von Gierke’s disease disease phataseQ in liver and renal tubule
co

cells (Kidney Enlarged)


are all except: (AIIMS Nov 2013) hypoglycemia; elevated
a. Hypoglycemia blood lactate, choles-
terol, triglyceride, and
b. Oxaloacetate is necessary for gluconeogenesis uric acid levels
c. Low blood glucose less than 40 mg% Ib —– Endoplasmic re- Same as type Ia, with
d. Fatty acid mobilization is low ticulum glucose- additional findings
6-phosphate of neutropenia and
Ans. d. Fatty acid mobilization is low transporter/ impaired neutrophil
translocase function
Glucose-6-phosphatase deficiency leading to hypogly- Recurrent Bacterial
cemia. Glucose-6-phosphate converted to Pyruvate. This Infection, Inflammatory
is converted to Acetyl CoA. As oxaloacetate is depleted Bowel Disease

because of using up of Oxaloacetate for Gluconeo- III Limit dextrino- Liver and mus- Fasting hypoglycemia
m

sis, Forbe’s or cle debranching hepatomegaly in infancy


genesis, Acetyl CoA enter in to Ketone body synthesis. Cori’s disease enzyme(Amylo accumulation of charac-
co

Hence Ketosis. 1,6 Glucosidase) teristic branched poly-


saccharide (limit dex-
Glucose-6-phosphate also enters into HMP shunt trin), muscle weakness,
pathway which leads to more production of Pentoses. elevated transaminase
levels; liver symptoms
Therefore more purine synthesis. Purines degrade to Uric can progress to liver
Acid. Hence there is Hyperuricemia. failure later in life
As there is hypoglycemia, fat is mobilized. This also IV Amylo- Branching en- Hepatosplenomegaly
pectinosis, zyme Accumulation of poly-
leads to more Acetyl CoA by Fatty acid oxidation. This Andersen’s saccharide with few
increases Ketone body synthesis. Hence Ketosis. disease branch points
Failure to thrive, hypoto-
39. A child with low blood glucose is unable to do nia, hepatomegaly, sple-
nomegaly, progressive
glycogenolysis or gluconeogenesis. Which of the cirrhosis (death usually
following enzymes is missing in the child?
m

before 5th yr), elevated


(AIIMS Nov 2012) transaminase levels
co

VI Hers’ disease Liver phosphory- Hepatomegaly


a. Fructokinase lase
b. Glucokinase
VIII Liver phosphory-
c. Glucose-6-phosphatase lase kinase
d. Transketolase Fanconi Bick- Glucose trans- Failure to thrive, rick-
el Syndrome porter 2 (GLUT- ets, hepatorenomegaly,
Ans. c. Glucose-6-phosphatase QNBE pattern 2) proximal renal tubular
(Ref: Harper 29/e p179, 181 Table 19.2, 189; 30/e p177) dysfunction, impaired
glucose and galactose
The answer should be an enzyme common to utilization
Glycogenolysis and Gluconeogenesis.
Glycogen Storage Disorders 40. In which of the following tissues is glycogen
Inborn errors of metabolism of Glycogen associated with incapable of contributing directly to blood
m

accumulation or altered function of Glycogen in various glucose: (AI-2008)


co

organs are concerned with Glycogen metabolism. a. Liver


Liver Glycogen Storage Disorder. b. Muscle
e

e
m

m
m
co

Metabolism of Carbohydrates  | 169

c. Both c. Hydrolyses Starch to Monosaccharides


d. None d. Convert maltose to glucose
Ans. b. Muscle (Ref: Harper 29/e p180; 30/e p177, 178) Ans. c. Hydrolyses Starch to limit dextrins.
Differences between Liver Glycogen and Muscle (Ref: Harper 30/e p538)
Glycogen.
44. A 5 years old boy presents with hepatomegaly,
Features Liver Muscle
hypoglycaemia, ketosis. The diagnosis is
m

Total Body Glycogen content Less Highest


a. Mucopolysaccharidosis
co

Percentage by tissue weight Highest Less b. Glycogen storage disorder


Regulation of blood glucose Contributes to Does not contrib- c. Lipopolysaccharidosis
blood glucose ute to blood glu-
cose d. Diabetes mellitus
Ans. b. Glycogen storage disorder
41. In humans carbohydrates are stored as: (Ref: Nelson 20/e Chapter 715 Defects in
(Kerala 2006) metabolism of Carbihydrates)
a. Glucose • Patients with type I GSD may present in the neonatal
b. Glycogen period with hypoglycemia and lactic acidosis
c. Starch • These children often have doll-like faces with fat
d. Cellulose cheeks, relatively thin extremities, short stature,
Ans. b. Glycogen (Ref: Harper 30e p154) and a protuberant abdomen that is due to massive
m

• Glycogen is the storage polysaccharide in animals hepatomegaly; the kidneys are also enlarged,
co

and is sometimes called animal starch. whereas the spleen and heart are normal
• The biochemical hallmarks of the Type I a GSD (von
42. Glycogen is released from the muscle due to Gierke’s) disease are hypoglycemia, lactic acidosis,
increased cAMP due to: hyperuricemia, and hyperlipidemia.
a. Epinephrine
b. Thyroxine 45. Glycogen phosphorylase can be regulated by all
c. Glucagon the following except: (AIIMS Nov 2015)
a. cAMP
d. Growth hormone
b. Calmodulin
Ans. a. Epinephrine (Ref: Harper 30/e p180, 181)
c. Protein Kinase A
Differences between Muscle and Liver in the regulation
of Glycogen Metabolism d. Glycogenin
Ans. d. Glycogenin (Ref: Harper 30/e p180-182)
m

• Epinephrine acts in Muscle and Liver whereas


Glucagon acts only in the Liver Regulation of Glycogen metabolism at Glycogen Phos-
co

• In the muscle there is cAMP-independent activation phorylase


of glycogenolysisQ • cAMP activates Glycogen Phosphorylase by cAMP-
• By the stimulation of a Ca2+/calmodulin-sensitive dependent Protein Kinase
phosphorylase kinase • cAMP-independent Calcium/Calmodulin-sensitive
• Phosphorylate Glycogen Phosphorylase in the muscle Phosphorylase. Kinase also activates Glycogen
• Favor glycogenolysis Phosphorylase.
• Muscle phosphorylase can be activated without Glycogenin is a protein on which initial glucose is added
phosphorylation in the synthesis of Glycogen.
• Muscle Phosphorylase has a binding site for 5’AMP 46. Cofactor for Glycogen Phosphorylase:
• 5’AMP is an allosteric activator without phospho- (AIIMS Nov 2015)
rylation a. Thiamine Pyrophosphate
m

• Favor Glycogenolysis. b. Pyridoxal Phosphate


co

43. Pancreatic alpha amylase: c. Citrate


a. Convert starch to glycogen d. FAD
b. Hydrolyses starch to limit dextrins Ans. b. Pyridoxal Phosphate. (Ref: Harper 30/e p178)
e

e
m

m
m
co

170 | Self Assessment and Review of Biochemistry

47. Pompe’s disease is due to deficiency of: c. 24


(NBE Pattern Q) d. 48
a. Debranching enzyme Ans. b. 18 hours (Ref: Harper 30/e p149)
b. Muscle phosphorylase • Liver and Muscle Glycogen exhausted by 18 hours
c. Acid maltase of fasting.
d. Branching enzyme
50. In the fed state, major fate of glucose-6-phosphate
m

Ans. c. Acid maltase. (Ref: Harper 30/e p178) in tissues is: (AIIMS may 93)
co

LIVER GLYCOGEN STORAGE DISORDERS a. Storage as fructose


Type Name Enzyme efficiency b. Storage as glyceraldehyde-3-phosphate
0 —– Glycogen synthase c. Enters HMP shunt via ribulose-5-phosphate
Ia von Gierke’s disease Glucose-6-phosphataseQ d. Storage as glycogen
Ans. d. Storage as glycogen (Ref: Harper 30/e p148)
• The uptake of glucose into the liver GLUT 2 is
Ib —– Endoplasmic reticulum glucose- independent of insulin
6-phosphate transporter • In the well fed state, the concentration of glucose
III Limit dextrinosis, Liver and muscle debranching entering the liver increases, so does the rate of
Forbe’s or Cori’s dis- enzyme (Amylo 1,6 Glucosidase) synthesis of glucose-6-phosphate
ease
• This is in excess of the liver’s requirement for
m

IV Amylopectinosis, Branching enzyme energy-yielding metabolism. So it is used mainly for


Andersen’s disease
synthesis of glycogen
co

VI Hers’ disease Liver phosphorylase


• In both liver and skeletal muscle, insulin acts to
VIII Liver phosphorylase kinase stimulate glycogen synthetase and inhibit glycogen
Fanconi Bickel Syn- Glucose transporter 2 (GLUT-2) phosphorylase.
dromeQNBE pattern
MUSCLE GLYCOGEN STORAGE DISORDER 51. Which of the following is a debranching enzyme:
(AIIMS 90)
II Pompes’s Disease (Be- Lysosomal α1,4 and α1,6 glucosi-
longs to lysosomal stor- dase (acid maltase)Q a. Glycogen synthetase
age disorder) b. Glucose-6-phosphatase
c. Amylo 1,6 glucosidase
Danon disease Lysosome-associated membrane
protein 2 (LAMP2)
d. Amylo 1,4-1,6 transglycosylase
Ans. c. Amylo 1,6 Glucosidase. (Ref: Harper 30/e p178)
V McArdle’s syndrome Muscle phosphorylaseQ
m

VII Tarui’s disease Muscle and erythrocyte phospho- 52. Sequence of events in glycogenolysis:
co

fructokinase 1 (PGI June 97, Dec 96)


a. Phosphorylase, glucan transferase, debranch-
48. Glycogen storage disorder(s) is/are: ing, phosphorylase
a. Niemann-Pick Disease (PGI Nov 2014) b. Debranching, phosphorylase, transferase, phos-
b. Gaucher Disease phorylase
c. Tay-Sacks disease c. Transferase, phosphorylase, debranching, phos-
phorylase
d. Pompe’s disease
d. Any of the above
e. McArdle’s disease
Ans. a. Phosphorylase, glucan transferase, debranching,
Ans. d. Pompe’s disease, e. McArdle’s disease.
phosphorylase (Ref: Harper 30/e page 180,181)
(Ref: Nelson 20/e Chapter Defects in the
Steps of Glycogenolysis
Metabolism of Carbohydrates)
m

Breaking of α 1,4 linkage


49. How many hours for depletion of glycogen: • Glycogen phosphorylase cleave the α 1,4 linkage
co

(NBE pattern Qn) • Releases Glucose 1 Phosphate, NOT free Glucose


a. 9 • Glycogen Phosphorylase stops its action when it is at
b. 18 least 4 glucose residues from a branch point.
e

e
m

m
m
co

Metabolism of Carbohydrates  | 171

Removal of Branches c. Cori’s disease


By a bifunctional enzyme: d. Anderson’s disease
• First part is a α-1, 4 α 1,4 Glucan transferase e. Pompe’s disease
‒ Transfer trisaccharide residue to another forming Ans. b. von Gierke’s Disease
a new α 1, 4 linkage (Ref: Nelson 20/e Defects in the metabolism of Carbohydrates)
• Second part is α1, 6 Glucosidase (Amylo 1,6 Clinical Picture of von Gierke’s Disease
m

Glucosidase). Hydrolyse the branching point Clinical presentation


co

‒ Releases free Glucose, NOT Glucose 1 Phosphate. • Most commonly present at 3–4 months of age with
Conversion of Glucose 1 Phosphate to free Glucose • Doll-like facies with fat cheeks
• Glucose 1 Phosphate to Glucose-6-phosphate by • Relatively thin extremities
Phosphoglucomutase • Short stature, Protuberant abdomen
• Glucose-6-Phosphate to Glucose by Glucose-6- • Massive Hepatomegaly
phosphatase • Kidneys are also enlarged
• Glucose-6-phosphatase is present in the smooth • No Splenomegaly
endoplasmic reticulum • Plasma may be milky due to associated hypertri-
• A transporter is required for the transport of Glucose- glyceridemia.
6-phosphate from SER to cytoplasm Type Ib has additional features of recurrent bacterial
• Defect in the Glucose-6-phosphate transporter lead infection due to neutropenia and impaired neutrophil
m

to Type Ib Glycogen Storage disorder. dysfunction.


co

The biochemical hallmarks are:


53. Muscles are not involved in which glycogen
• Hypoglycemia
storage disease: (PGI Dec 97)
a. I • Lactic Acidosis
b. II • Hyperlipidemia
c. III • Hyperuricemia.
d. IV Long-term effects of Type I GSD are PCOD, Pancreatitis,
Hepatic Adenomas, Pulmonary hypertension, Osteopenia,
Ans a. I
Renal disease.
(Ref: Nelson 20/e Defects in the metabolism of Carbohydrates)
Glycogen Storage Disorder II (Limit dextrinoses) and 55. Glycogen storage diseases include all the
GSD IV (Anderson disease) are liver GSD with muscle following except: (PGI Dec 01)
involvement. a. von Gierke’s disease
m

Muscle Glycogen Storage Disorders b. Fabry’s disease


co

Type Name Enzyme defect c. McArdle’s disease


II Pompe’s Disease Lysosomal α1,4 and α1,6 gluco-
d. Fragile X syndrome
(Belongs to lysosomal sidase (acid maltase)Q e. Krabbe’s disease
storage disorder) Ans. b, d, e.
Danon disease Lysosome-associated membrane (Ref: Nelson 20/e Defects in the metabolism of Carbohydrates)
protein 2 (LAMP2)
• Fabry’s Disease and Krabbe’s Disease are Sphin-
V McArdle’s syndrome Muscle phosphorylaseQ golipidoses.
VII Tarui’s disease Muscle and erythrocyte phospho-
fructokinase 1 56. The cause of hyperuricemia and gout in Glucose-
6-phosphatase deficiency is: (AIIMS Nov 01)
54. An infant has hepatosplenomegaly, hypoglyce- a. More formation of pentose
b. Decreased availability of glucose to tissues
m

mia, hyperlipidemia, acidosis and normal struc-


tured glycogen deposition in liver. What is the c. Increased accumulation of sorbitol
co

diagnosis: (PGI June 01) d. Impaired degradation of free radicals


a. Her’s disease Ans. a. More formation of pentoses
b. von Gierke’s disease (Ref: Nelson 20/e Defects in the metabolism of Carbohydrates)
e

e
m

m
m
co

172 | Self Assessment and Review of Biochemistry

Biochemical defect in von Gierke’s 59. Reduced NADPH is produced from which
Glucose 6 Phosphatase deficiency → Glucose 6 Phosphate pathway: (NBE Pattern Q)
accumulate → channeled to Pentose Phosphate pathway a. Krebs cycle
→ more Pentoses → To Purine Synthesis → So Uric acid b. Anerobic glycolysis
the degradation product Purine accumulate. c. Uronic acid pathway
d. Hexose monophosphate pathway
HMP Pathway
m

Ans. d. HMP pathway. (Ref: Harper 30/e p197)


co

57. Metabolites in HMP Shunt are all except: (AI 95)


60. Which of the following metabolic pathways does
a. Glycerol-3-phosphate
not generate ATP: (AI 2008)
b. Sedoheptulose-7-phosphate a. Glycolysis
c. Glyceraldehyde-3-phosphate b. TCA Cycle
d. Xylulose-5-phosphate c. Fatty Acid Oxidation
Ans. a. Glycerol 3 Phosphate (Ref: Harper 30/e p197) d. HMP Pathway
Metabolites in HMP Shunt Pathway Ans. d. HMP Pathway. (Ref: Harper 29/e p199; 30/e p200)
• Glucose-6-phosphate Metabolic pathway Net ATP yield
• 6-Phosphogluconate Anerobic Glycolysis 2
• Ribulose-5-Phosphate
Aerobic Glycolysis 7
• Xylulose-5-Phosphate
m

Aerobic oxidation of 1 mol of glucose 32


• Ribose-5-Phosphate
co

• Glyceraldehyde-3-Phosphate Palmitic Acid Oxidation 106

• Sedoheptulose-7-Phosphate Citric Acid Cycle 10


• Fructose-6-phosphate Rapaport-Luebering Cycle 0
• Erythrose 4 Phosphate. HMP Pathway 0

58. NADPH is produced by: (AIIMS Nov 2003) 61. Severe thiamine deficiency is associated with:
a. Glycolysis (JIPMER 2013)
b. Citric acid cycle a. Decreased RBC transketolase activity
c. HMP shunt b. Increased clotting time
d. Glycogenesis c. Decreased RBC transaminase activity
Ans. c. HMP shunt (Ref: Harper 30/e p197) d. Increased xanthic acid excretion
m

Key points to remember in HMP (Pentose Phosphate) Ans. a. Decreased RBC transketolase
co

Pathway (Ref: Harper 30/e p203, 204)


• Main source of NADPHQ2012 and Pentoses Thiamine is a coenzyme of Erythrocyte transketolase so
• Two transketolation and one transaldolation its activity is decreased if Thiamine deficiency.
reactions are involved
Other Metabolic Pathways of Glucose
• No ATP is producedQ
• CO2 is produced in this pathway and not in glycolytic 62. Products of uronic acid pathway in human beings
pathwayQ DNB 01) are all except: (NBE pattern Qn)
• Deficiency of Glucose-6-phosphate dehydrogenase a. Vitamin C
is a major cause of acute hemolysis in erythrocytes b. Glucuronic acid
• NADPH is used for reductive biosynthetic pathways, c. Pentoses
like fatty acid synthesis, steroid synthesis, Amino d. NADH
m

acids by Glutamate dehydrogenase Ans. a. Vitamin C (Ref: Harper 30/e p202, 203)
co

• NADPH is required for regeneration of reduced • Uronic acid pathway cannot synthesize Vitamin C
Glutathione, that clears free radicals from erythrocytes in humans and higher primates because of lack of
and lens. L-Gulanolactone Oxidase.
e

e
m

m
m
co

Metabolism of Carbohydrates  | 173

63. Uronic acid pathway is not involved in: Clinical manifestations


(NBE Pattern Qn) • Patients with HFI are asymptomatic until fructose or
a. Conjugation of bilirubin sucrose (table sugar) is ingested (usually from fruit,
b. GAG Synthesis fruit juice, or sweetened cereal)
c. Vitamin C Synthesis • Symptoms may occur early in life, soon after birth
d. Biotransformation if foods or formulas containing these sugars are
introduced into the diet
m

Ans. c. Vitamin C Synthesis (Ref: Harper 30/e p202, 203)


• Early clinical manifestations resemble galactosemia
co

• Glucuronic acid is involved in the conjugation of


and include jaundice, hepatomegaly, vomiting,
Bilirubin
lethargy, irritability, and convulsions, hypoglycemia
• Uronic acid and Amino Sugar are the repeating
• Acute fructose ingestion produces symptomatic
disaccharide unit in the Glycosaminoglycans
hypoglycemia. If the intake of fructose persists,
• Glucuronidation is a Phase II Xenobiotic reactions hypoglycemic episodes recur, and liver and kidney
(Biotransformation) failures progress, eventually leading to death
• Uronic acid pathway cannot synthesize Vitamin C • Chronic ingestion results in failure to thrive.
in humans and higher primates because of lack of
L-Gulonolactone Oxidase. Galactose Metabolism and Disoders
Fructose Metabolism and Disoders 66. Galactosemia enzyme defect:
a. Fructokinase
m

64. Hereditary Fructose Intolerance is due to b. Glucokinase


deficiency of: (JIPMER 2014) c. Galactose-1-Phosphate Uridyl Transferase
co

a. Aldolase B d. Glucose-6-Phosphatase
b. Aldolase A Ans. c. Galactose 1 Phosphate Uridyl Transferase
c. Fructokinase (Ref: Harper 29/e p206)
d. Sucrase Galactosemia
Ans. a. Aldolase B (Ref: Nelson 20/e Chapter Defects Enzyme Deficiency (AIIMS Nov 2011, May 2013)
in metabolism of carbohydrates) • Galactose-1-Phosphate Uridyl Transferase [Classic
• Hereditary Fructose Intolerance due to deficiency Galactosemia]
of Aldolase B • Galactokinase
• Essential Fructosuria due to deficiency of Fructoki- • UDP Hexose 4 Epimerase
nase. The newborn infant receives high amounts of lactose,
which consists of equal parts of glucose and galactose.
m

65. False about hereditary fructose intolerance: Without the Galactose-1-Phosphate Uridyl Trasferase
co

(AIIMS June 98) enzyme, the infant is unable to metabolize galactose-1-


a. Deficiency of fructose 1-phosphate aldolase phosphate, the accumulation of which results in injury
b. Accumulation of fructose 1-phosphatein to kidney, liver, and brain.
tissues Clinical features
c. Hyperglycemia • Classic galactosemia is a serious disease with onset
d. Liver and kidneys are involved of symptoms typically by the 2nd half of the First
Ans. c. Hyperglycemia. week of life
(Ref: Nelson 20/e Chapter Defects in • With jaundice, vomiting, seizures, lethargy, irritabil-
metabolism of carbohydrates) ity, feeding difficulties, poor weight gain or failure
Biochemical defect of Hereditary Fructose Intolerance to regain birthweight
• Deficiency of Fructose-1,6-Bisphosphate Aldolase • Hepatomegaly oil drop cataracts due to accumulation
of Galactictol/dulcitol
m

(Aldolase B)
• Hepatic failure
co

• The gene for aldolase B is on chromosome 9q22.3


• Deficiency of this enzyme activity causes a rapid • Mental retardation
accumulation of fructose-1-phosphate and initiates • Patients with galactosemia are at increased risk for
severe toxic symptoms when exposed to fructose. Escherichia coli neonatal sepsis.
e

e
m

m
m
co

d. Defect in galactose 1-phosphate uridyl


transferase
Ans. a, c, d.
(Ref: Nelson 20/e Chapter Defects in
metabolism of carbohydrates)

Integration of Metabolism
m

(Fed State and Fasting State)


co

70. Which is used for energy: (PGI May 2013)


a. Ketone bodies
b. Glucose
c. Free fatty acids
d. Creatinine
e. Collagen
Ans. a, b, c.
(Ref: Harper 29/e p161 Table 16.3; 30/e p150 Table 14.3)
Metabolic Fuels for Different Organs
Organ Major metabolic fuels
m

Liver Free fatty acids, glucose (in fed state), lactate,


glycerol, fructose, amino acids, alcohol
co

Brain Glucose, amino acids, ketone bodies in pro-


longed starvation
HeartQ Ketone bodies, free fatty acids, lactate, chylo-
micron and VLDL triacylglycerol, some glucose
Adipose tissue Glucose, chylomicron and VLDL triacylglycerol
Fast Twitch Muscle Glucose, glycogen
Slow Twitch Muscle Ketone bodies, chylomicron and VLDL triac-
ylglycerol
Kidney Free fatty acids, lactate, glycerol, glucose
Erythrocyte Glucose

71. All the following are increased in starvation


m

except: (PGI Nov 2012)


co

a. Lipolysis
b. Ketogenesis
c. Gluconeogenesis
d. Glycogenesis
e. Glycogenolysis
Ans. d. Glycogenesis
Glycogenesis is increased in well fed state.
72. Which enzyme is active when insulin glucagon
ratio is low? (AIIMS Nov 2013)
(Ref: Nelson 20/e Chapter Defects
a. Glucokinase
in metabolism of carbohydrates)
b. Hexokinase
m

69. True regarding galactosemia: (PGI Dec 01) c. Glucose-6-phosphatase


co

a. Mental retardation occurs d. Pyruvate Carboxylase


b. Absent disaccharidase in intestine Ans. c. Glucose-6-phosphatase, d. Pyruvate Carboxylase
c. Defect in epimerase (Ref: Harper 30/e p188 Table 19.1)
e

e
m

m
m
co

Metabolism of Carbohydrates  | 175

Insulin: Glucagon ratio is more when body is in the fasting Contd...


state. Key Enzymes of Gluconeogenesis will be active. Organ Major metabolic fuels
So, Pyruvate Carboxylase and Glucose 6 Phosphatase Slow Twitch Ketone bodies, chylomicron and VLDL triac-
are active. Muscle ylglycerol
Kidney Free fatty acids, lactate, glycerol, glucose
73. Substrate used by RBC in fasting state is:
Erythrocyte Glucose
(May AIIMS 2015)
m

a. Glucose
co

b. Amino Acids 74. Lactic acidosis in thiamine deficiency is due to


c. Ketone body which enzyme dysfunction? (AIIMS May 2015)
d. Fatty acid a. Phosphoenol Pyruvate Carboxykinase
Ans. a. Glucose (Ref: Harper 30/e p150 Table 14.3) b. Pyruvate Dehydrogenase
Organ Major metabolic fuels c. Pyruvate Carboxylase
Liver Free fatty acids, glucose (in fed state), lactate, d. Aldolase
glycerol, fructose, amino acids, alcohol Ans. b. Pyruvate Dehydrogenase (Ref: Harper 30/e p174)
Brain Glucose, amino acids, ketone bodies in pro- In Thiamine deficiency, PDH reaction is defective. So
longed starvation Pyruvate is converted to lactic acid.
HeartQ Ketone bodies, free fatty acids, lactate, chylo- Causes of inhibition of PDH leading to lactic acidosis:
micron and VLDL triacylglycerol, some glucose
• Inherited PDH deficiency
m

Adipose tissue Glucose, chylomicron and VLDL triacylglycerol • Thiamine deficiency


co

Fast Twitch Muscle Glucose, glycogen • Alcoholics due to thiamine deficiency


Contd... • Arsenite and Mercury poisoning.
m
co
m
co
e

e
m

m
m
co

176 | Self Assessment and Review of Biochemistry


m
co
m
co
m
co
m
co
e

e
m

m
3
m
co
m
co

Section Lipids
m
co

C H A P T E R S

6. Chemistry of Lipids and Biomembranes


m
co

7. Metabolism of Lipids
m
co
co co co co co
m m m m m
m
e

m
e

m
e

m
e
e

e
m

m
m
co

6 Chemistry of Lipids and


Biomembranes
m
co

Topics Included
• Classification of Lipids • Phospholipids
• Fatty Acids • Glycolipids
• Triacylglycerol • Biomembranes
m
co

DEFINITION
Lipids are heterogenous group of compounds relatively
insoluble in water and freely soluble in nonpolar organic
solvents—ether, chloroform.
Unlike Carbohydrates and Proteins they are not
chemically related, but they are physically related. Fig. 6.1: Diagrammatic representation of triacylglycerol
• Carbohydrates: Polymer of Monosaccharide
• Proteins: Polymer of Amino Acids. Waxes
Lipids are not true polymers, but mixture of chemically They are esters of fatty acid with higher molecular weight
unrelated substances. monohydric alcohol other than glycerol, e.g. Bee wax,
m

Lanolin.
CLASSIFICATION OF LIPIDS
co

Fatty Acid + High molecular weight alcohol other than glycerol


Bloor’s Classification
Simple Lipids
They are esters of fatty acid with alcohol Complex Lipids
They are esters of fatty acid with alcohols having
Fatty Acid + Alcohol = Esters of fatty acid
additional groups like Phosphoric acid, Carbohydrates,
They are divided into Proteins etc.
• Fats and oils Fatty Acid + Alcohol (Glycerol/Sphingosine) + additional groups
• Waxes.
Depending on the additional groups present they are
Fats and oils
again classified into
They are esters of fatty acid with alcohol, glycerol.
m

• Phospholipids
Fatty Acid + Glycerol
co

• Glycolipids
Fats and oils are the same except that fats are solid at room • Other complex lipids like Sulfolipids, Lipoproteins,
temperature, and oils are liquid at room temperature. Amino lipids.
e

e
m

m
m
co

180 |  
Self Assessment and Review of Biochemistry

Phospholipids • ∆9 indicates a double bond between carbons 9 and


Lipids containing phosphoric acid residue and nitrogenous 10 of the fatty acid counting from the carboxyl end
base in addition to glycerol and fatty acid. • ω9 indicates a double bond on the ninth carbon
Fatty Acid + Alcohol (Glycerol/Sphingosine) + Phosphate + counting from the ω-carbon.
Nitrogenous base

Phospholipids divided into


m

Fig. 6.2: Nomenclature of number and


• Glycerophospholipids: The alcohol in this group is
co

position of double bond of unsaturated fatty acid


Glycerol, e.g. Lecithin, Cephalin
• Sphingophospholipids: The alcohol in this group is an Classification of Fatty Acid
amino alcohol Sphingosine, e.g. Sphingomyelin. Depending on the Chain Length
Glycolipids or Glycosphingolipids Q • Short Chain Fatty Acid (C2–C6)
• Lipids containing carbohydrate apart from Fatty acid • Medium Chain Fatty Acid (C8–C14)
and Alcohol (usually Sphingosine) • Long Chain Fatty Acid (≥ C16).
• Fatty Acid + Alcohol (Sphingosine) + Carbohydrate, Depending on the Presence of Double Bond
e.g. Cerebroside, Ganglioside. • Saturated fatty acid: No double bond in the Hydrocarbon
Lipoproteins chain
• Lipids complexed with proteins, e.g. Low density • Unsaturated fatty acid: Double bonds are present in
m

Lipoprotein (LDL), High Density Lipoprotein (HDL). the Hydrocarbon chain.


co

Depending on the number of double bonds present


Derived Lipids or Precursor Lipids unsaturated fatty acid is again classified into:
Compounds which are derived from the above group of • Monounsaturated Fatty Acid: Only one double bond
Lipids, e.g. Fatty acids, Glycerol, Cholesterol. is present
• Polyunsaturated Fatty Acid: More than one double
Miscellaneous Lipids
bonds are present.
Vast number of lipids which are not classified under any
of the above groups, e.g. Squalene, Carotenoids, Vitamin Common saturated fatty acids and their sources
E, Vitamin K. Saturated fatty acid Source
Acetic Acid (2C) Vinegar
FATTY ACIDS Butyric Acid (4C) Butter
m

Valeric Acid (5C) Butter


They are aliphatic carboxylic acid.
co

Caproic Acid (6C) Butter and Coconut milk


General Structural Formula of Fatty acid—R-COOH
• R is the Aliphatic Hydrocarbon chain. Lauric Acid (12C) Coconut milk
• R group accounts for the nonpolar nature of fat. Myristic Acid (14C) Coconut milk
Palmitic Acid (16C) Body Fat

Numbering the Carbon Atoms in Fatty Acids Stearic Acid (18C) Body Fat

• Carbon atoms are numbered from the carboxyl


carbon (carbon no. 1) Common Unsaturated Fatty Acids
• The carbon atoms adjacent to the carboxyl carbon Number of C
(nos. 2, 3, and 4) are also known as the α, β, and γ atoms and num-
ber and posi-
carbons, respectively, and the terminal methyl carbon tion of common Common
is known as the ω-or n-carbon. double bonds Family name OccurrenceQ
m

Monoenoic acids (one double bond)


Conventions Used to Indicate the
co

16:1; 9 ω7 Palmitoleic In nearly all fats


Positions of Double Bonds
18:1; 9 ω9 Oleic High in olive oil
• Various conventions use for indicating the number
and position of the double bonds Contd...
e

e
m

m
m
co

Chemistry of Lipids and Biomembranes  | 181

Contd... • Alpha Linolenic Acid


Number of C • Arachidonic Acid is considered as semi essential fatty
atoms and num-
Acid as it can be synthesized from Linoleic Acid.
ber and posi-
tion of common Common
double bonds Family name OccurrenceQ Functions of Essential Fatty Acid
18:1; 9 ω9 Elaidic Hydrogenated and • They are integral components of membrane structure,
m

ruminant fats often in the 2 position of phospholipids


co

Dienoic acids (two double bonds) • Eicasanoids are synthesized from Arachidonic Acid.
18:2; 9, 12 ω6 Linoleic Corn, peanut, cotton- Essential fatty acids are needed for the synthesis of
seed, soybean Arachidonic Acid
• Lower the risk of Cardiovascular Diseases
Trienoic acids (three double bonds)
• Lower the risk of Fatty liver.
18:3; 6, 9, 12 ω6Q γ-Linolenic Oil of evening prim-
(GLA) rose, borage oil; lin- Deficiency of Essential Fatty Acid
seed oil
• Skin: Acanthosis and Hyperkeratosis
18:3; 9, 12, 15 ω3Q α-Linolenic Linseed oil • Fatty liver
• Swelling of mitochondrial membrane and reduction
in efficiency of oxidative phosphorylation
• Decrease in fibrinolytic activities.
m

Tetraenoic Acid (four double bondsQ)


co

20:4; 5, 8, 11, 14 ω6 Arachidonic Found in animal fats;


important compo- OMEGA CLASSIFICATION OF FATTY ACID
nent of phospholip-
ids in animals In this type of classification position of double bond is
Pentaenoic acids (five double bonds) counted from the methyl end (ω carbon atom). Depending
20:5; 5, 8, 11, 14, ω3 Timnodonic Important compo- on the position of first double bond from the terminal
17 (Eicosapen- nent of fish oils, e.g. end, the fatty acids are again classified into.
taenoic ) cod liver, mackerel,
ω3 Series • α Linolenic Acid
menhaden, salmon
oils • Timnodonic Acid (Eicosa pentaenoic Acid)
• Cervonic Acid (Docosa Hexaenoic Acid) (DHA)
Hexaenoic acids (six double bonds)
ω6 Series • Linoleic Acid
22:6; 4, 7, 10, 13, ω3 Cervonic Fish oils, phospho-
• Linolenic Acid (GLA)
16, 19 (Docosa- lipids in brain, Breast
• Arachidonic Acid
m

hexaenoic) milkQ
(DHA) ω9 Series • Oleic Acid
co

• Elaidic Acid
NB: This table is very important, Numerous questions can be asked.
Significance of Medium Chain Fatty Acid Significance of ω3 Fatty Acid
• Absorbed directly into the Blood
• Decrease the risk of Cardiovascular Disease
• Do not need Carnitine for transport into Mitochondria
• No effect on Atherosclerosis. • Appear to replace arachidonic acid in platelet
membranes
• Lower the production of Thromboxane and tendency
ESSENTIAL FATTY ACID of the platelet aggregation
The fatty acids that are required by humans, but are not • Decrease Serum Triglycerides
synthesized in the body hence need to be supplied in the • Important for Infant Development
m

diet are known as essential fatty acid (EFA). Humans lack • Lower the risk of various mental illness (Depression,
the enzymes that can introduce double bond beyond 9th ADHD)
co

Carbon. • Lower the risk of chronic degenerative diseases such


They are Polyunsaturated Fatty Acid namely: as Cancer, Rheumatoid Arthritis, and Alzheimer’s
• Linoleic Acid Disease.
e

e
m

m
m
co

182 |  
Self Assessment and Review of Biochemistry

Contd...
DOCOSA HEXAENOIC ACID (DHA)
• Sources: Human milk, Fish liver oils, Algal oils SFA MUFA Linoleic Alpha Linole-
• Synthesized in the body from α Linolenic acid. Fats/Oil (%) (%) acid (%) nic acid (%)
• Highest concentration of DHA found in retina, cerebral cortex, Butter/Ghee 68 29 2 1
sperms.
High SFA and MUFA
• Functions: Needed for the development of fetal brain and retina
• DHA is supplied transplacentally and through breast milk. Palmolein 39 46 11 < 0.5
m

• Clinical significance: Low DHA is associated with increased risk High MUFA and Moderate Linoleic acid
co

of Retinitis Pigmentosa.
Ground nut 19 41 32 < 0.5
Rice bran 17 43 38 1

ISOMERISM IN FATTY ACIDS Sesame 16 41 42 < 0.5


High linoleic acid
Cis-trans Isomerism Cotton seed 24 29 48 1
When similar groups occur on same side of the double Corn 12 35 50 1
bond it is called cis isomer and when similar group occur Safflower 9 13 75 —
on opposite side it is called trans isomer.
Sunflower 12 22 62 —
Biological Significance of Cis isomers High Linoleic acid and alpha linolenic acid
All naturally occurring amino acids are cis isomers. Soybean 14 24 53 7
Cis isomers increases the fluidity of biological membranes.
m

Canola 6 60 22 10
Mustard/Rapeseed 4 65 15 14
co

Trans fatty Acids


Flax-Seed 10 21 16 5
• Present in dairy products and partially hydrogenated
High trans fatty acid
edible oils (e.g. Margarine)
Vanaspati 46 49 4 —
• They are used in food industry to improve the shelf
life Dietary Sources of Fatty Acids
• Trans fatty acids are present in high amounts in • Highest content of MUFA in Mustard/Rapeseed oil
processed foods, fast foods and bakery items and • Highest content of Medium chain fatty acid in Coconut oil
fried foods • Highest content Linoleic acid in Safflower oil
• Trans fatty acid compete with essential fatty acid, • Highest content alpha linolenic acid in Flax seed oil
hence exacerbate essential fatty acid deficiency • Highest amount of PUFA is present in Safflower oil
• Consumption of trans fatty acid for long terms may • Second highest source of PUFA is Sunflower oil
• Least source of PUFA is Coconut oil
m

raise the risk factor for cardiovascular diseases like


• Richest source of medium chain fatty acid is Coconut oil
Atherosclerosis and Coronary Artery disease and
co

• Fatty acid present in human milk is Docosa Hexaenoic Acid


Diabetes mellitus.
(Cervonic Acid).
• Fatty acid present in the fish oils are
– Timnodonic Acid
– Clupanodonic Acid
– Cervonic Acid.

Fig. 6.3: Cis and trans isomer of elaidic acid


TRIACYLGLYCEROL (TAG)
DIETARY SOURCES OF FATTY ACID

SFA MUFA Linoleic Alpha Linole-


Fats/Oil (%) (%) acid (%) nic acid (%)
m

High Medium chain and Short chain fatty acid


co

Coconut 92 6 2 —
Palm kernel 83 15 2 —

Contd... Fig. 6.4: Structure of triacylglycerol


e

e
m

m
m
co

Chemistry of Lipids and Biomembranes  | 183

• Main storage form of lipids in the body PHOSPHOLIPIDS


• Stored in the adipose tissues
Compound lipids composed of fatty acid, alcohol,
• Otherwise called Neutral fat.
phosphoric acid and a nitrogenous base. Based on the
Physical Properties of Triacylglycerol alcohol present phospholipids are divided into.
• Glycerophospholipid: They contain alcohol, Glycerol
Saponification and Saponification Number • Sphingophospholipid: They contain alcohol, Sphingo-
m

The hydrolysis of triacylglycerol by alkali into glycerol sine.


co

and soap is called Saponification.


Saponification Number: The mg of KOH required to Glycerophospholipids
saponify 1 gm of fat or oil completely. This is a measure
of average molecular weight and chain length of the fatty
acid present.
• It is inversely proportional to the chain length of fatty
acid present in the fats.
Type of fat Saponification number
Fig. 6.5: Diagramatic representation of glycerophospholipid
Human Fat 195–200
Butter 230–240
Glycerophospholipid Contain
Coconut oil 250–260
• Glycerol
m

• Fatty acid esterified to the first two carbon atoms


co

Iodine number
• Nitrogenous base
• The number of grams of Iodine absorbed by 100 g
of fat or oil • Phosphoric Acid.
• Iodine number is used to assess the degree of Phosphatidic Acid
unsaturation of fat Simplest Phospholipid.
• It is directly proportional to the degree of unsaturation Does not contain any nitrogenous base.
of fatty acid.
Phosphatidic acid contains.
Type of fat/oil Iodine number
• Glycerol
Butter 25–28
• Fatty acid esterified to the first two carbon atoms
Human fat 65–70
• Phosphoric Acid.
Linseed oil 170–200
All glycerophospholipids are derived from Phosphatidic
m

acid.
co

Reichert-Meissl (RM) Number i.e. Phosphatidic acid + Nitrogenous base


The number of 0.1 N KOH required to neutralize the Glycerophospholipids are divided into
volatile fatty acids distilled from 5 g of fat. Nitrogen Containing
Assess the purity of fats having more volatile fatty acid. • Lecithin (Phosphatidylcholine)
• Cephalin (Phosphatidylethanolamine)
Rancidity of Fat • Phosphatidyl serine
• Rancidity refers to unpleasant smell and taste for Nonnitrogen Containing
fats and oils • Phosphatidylinositol
• Phosphatidylglycerol
• Hydrolytic rancidity due to partial hydrolysis of
• Diphosphatidylglycerol (Cardiolipin)
Triacylglycerol due to hydrolytic enzymes present
in naturally occurring fats and oils
• Oxidative rancidity due to partial oxidation of Lecithin (Phosphatidylcholine)
m

unsaturated fatty acid Glycerophospholipid with Choline as nitrogenous base.


co

• Vegetable oils with high content of Poly Unsaturated i.e. Phosphatidic Acid + Choline.
Fatty Acid are easily oxidized. Hence vegetable oils Most abundant phospholipid of cell membrane.
are preserved with antioxidants. Largest body store of choline.
e

e
m

m
m
co

184 |  
Self Assessment and Review of Biochemistry

Importance of choline Action of Phospholipase C on Phosphatidylinositol 4,


• Acetylcholine in nerve transmission 5 Bisphosphate (PiP2)
• Transmethylation reaction. • Diacyl Glycerol and Inositol Tris phosphate IP3 are
Dipalmitoyl Lecithin (Di Palmitoyl Phosphatidyl Choline) formed and both act as second messengers.
is a major constituent of lung surfactant. Phospho
Phosphatidylinositol 4,5- Lipase C Diacyl Glycerol + Inositol
Lung Surfactants
m

bisphosphate (PiP2) Tris Phosphate (IP3)


Consist of Dipalmitoyl Lecithin, Phosphatidyl Glycerol, Cholesterol
co

and Surfactant protein A, B, C.


Ether Lipids
Lecithin Sphingomyelin Ratio (L/S ratio)
Before 28 weeks fetal lung synthesizes Sphingomyelin. But as lung When fatty acid is attached to C1 of Glycerol in Glycerophospholipids
matures more Lecithin is synthesized. L/S ratio indicates lung maturity. by an ether linkage instead of usual Ester linkage, they are called
Ratio of 2 indicate full lung maturity. Ether lipids.
Respiratory Distress Syndrome Two biologically important Ether Lipids are:
In premature infants due to decreased lung surfactants. 1. Plasmalogen.
2. Platelet Activating Factor (PAF).
Plasmalogen
Cephalin (Phosphatidyl Ethanolamine)
On C1 instead of fatty acid an unsaturated alkyl is attached by an
Glycerophospholipid with Ethanolamine as nitrogenous ether linkage. Components are:
base. i.e. Phosphatidic Acid + Ethanolamine. Glycerol + Unsaturated Alkyl residue + Fatty Acid + Phosphoric Acid
• Component of cell membrane + Ethanolamine.
m

• Play a role in Blood coagulation. • Plasmalogens occur in Brain and heart


• The Plasmalogen may have a protective effect against reactive
co

Cardiolipin oxygen species.


• First isolated from Cardiac muscle and hence the Platelet Activating Factor
It has an ether linkage at C1 to an Alkyl residue and Ester linkage at
name
C2 to Acetyl group Components are:
• Made up of two molecules of Phosphatidic Acid linked Glycerol + Alkyl residue + Acetic Acid + Phosphoric Acid + Choline.
by a molecule of Glycerol, i.e. Diphosphatidylglycerol
• It is a major lipid of Inner Mitochondrial Membranes. Sphingophospholipid
• Only Phospholipid which possess antigenicity The second group of phospholipid which contain
• Recognized by antibodies raised against Treponema- sphingosine as the backbone alcohol.
Q
pallidum, that causes syphilis Only Sphingophospholipid is Sphingomyelin .
• Decreased Cardiolipin levels or alteration in its struc-
ture or metabolism cause mitochondrial dysfunction.
m

Cardiolipin associated mitochondrial dysfunction is seen in


co

• Aging
• Heart failure
• Barth Syndrome (Cardioskeletal Myopathy)
• Hypothyroidism

Fig. 6.6: Diagrammatic representation of sphingophospholipids


Phosphatidylserine
• Nitrogenous base is Serine Sphingomyelin
• Plays an important role in Programmed cell death. Structure
Phosphatidylinositol • The Amino Alcohol, Sphingosine is attached to a fatty
• Phosphatidylinositol present in the cell membrane acid by an amide linkage forming Ceramide
• The Inositol is present as its stereo isomer, Myoinositol • Ceramide is further linked to a phosphoryl and
m

nitrogenous base (Choline) to form Sphingomyelin.


• Play an important role in cell signalling and mem-
co

brane trafficking Sphingosine + Fatty Acid = Ceramide


Ceramide + Phosphoryl group + Nitrogenous base
• Phosphatidyl inositol is a precursor of second
= Sphingomyelin
messenger in hormonal pathways.
e

e
m

m
m
co

Chemistry of Lipids and Biomembranes  | 185

• Sphingomyelins are found in outer leaflet of cell Glycolipids or Glycosphingolipids


membrane bilayer • Cerebroside
• Particularly abundant in specialised areas of plasma • Globoside
membrane called lipid rafts • Ganglioside.
• They are also particularly abundant in the myelin
sheath that surrounds nerve fibers Cerebroside
m

• They play a role in cell signalling and apoptosis. • Ceramide + Monosaccharide


co

• Glucocerebroside is Ceramide + Glucose


• Galactocerebroside is Ceramide + Galactose
• Galactocerebroside found in the brain and other
neural tissues
• The fatty acid present in Galactocerebroside is
characteristically Cerebronic Acid (C-24)
• Sulfogalactosylceramide present in high amounts
in the myelin
• Glucocerebroside found in the non-neural tissues
and some amount in brain.
Globoside
m

• Ceramide + Oligosaccharide.
co

Ganglioside
• Ceramide + Oligosaccharide containing N Acetyl
Fig. 6.7: Structure of sphingomyelin Neuraminic Acid [NANA] or Sialic Acid).
Ganglioside is named as GMn where
GLYCOLIPIDS (GLYCOSPHINGOLIPIDS) • G represent Ganglioside
• Complex lipids which contain carbohydrates, but no • M represent Monosialo, as it contain Sialic Acid
phosphoric Acid • n stands for number assigned on the basis of
• The alcohol in glycolipid is always Sphingosine hence chromatographic migration. Gangliosides are present
called Glycosphingolipid in the brain in high concentration.
• Glycosphingolipids are found in the outer leaflet of They act as receptors for bacterial toxins and for hormones
m

Plasma membrane GM3 Ganglioside


co

• Widely distributed in all tissues especially in nervous • The simplest ganglioside found in tissues is GM3
tissue like brain. • Ceramide + Glucose + galactose + NANA
GM1 Ganglioside
Basic Structure of Glycosphingolipid • More complex than GM3 Ganglioside
• Derived from GM3 Ganglioside
• This is known to be receptor for Cholera toxin in human intestine.

SPHINGOLIPIDOSIS
Sphingolipidoses are a group of lysosomal storage
disorder characterized by an inherited deficiency of a
Fig. 6.8: Diagrammatic representation of glycolipids lysosomal hydrolase leading to intralysosomal storage
m

of lipid substrates resulting from defective catabolism of


co

Three Types Glycolipids the sphingolipids comprising cellular membranes. The


Ceramide (Sphingosine + Fatty Acid) attached to different lipid substrates share a common structure that includes
carbohydrate to form three types of Glycolipids. a ceramide backbone (2-N-acylsphingosine).
e

e
m

m
m
co

186 |  
Self Assessment and Review of Biochemistry

General Features of Sphingolipidosis • By the end of the first year of life, most patients are
Progressive lysosomal accumulation of glycosphingolipids blind and deaf, with severe neurologic impairment
in the central nervous system leads to neurodegeneration, characterized by decerebrate rigidity
storage in visceral cells can lead to organomegaly, • Death usually occurs by 3–4 years of age.
skeletal abnormalities, pulmonary infiltration, and other
Diagnosis
manifestations.
Demonstration of the deficiency of β-galactosidase
m

Enzyme deficiencies in sphingolipidosis activity in peripheral leukocytes.


co

Sphingolipidoses Enzyme deficiency GM2 Gangliosidoses


Farber disease Ceramidase This includes:
Fabry disease α-galactosidase • Tay–Sachs disease
GM1 gangliosidosis β-galactosidase • Sandhoff disease.
GM2 gangliosidosis
1. Tay–Sachs disease, β-hexosaminidases A
Biochemical Defect
2. Sandhoff disease β-hexosaminidases A and B
• Autosomal Recessive
Gaucher’s Disease Glucocerebrosidase/ β Glucosidase
• Deficiency of β-Hexosaminidase
• Lysosomal accumulation of GM2 Ganglioside,
Niemann-Pick Disease Sphingomyelinase
particularly in CNS.
Metachromatic leukodys- Arylsulfatase A
m

trophy Sphingolipid Activator Protein (SAP-1) Concept of enzyme defect in GM2 Gangliosidoses
• β-hexosaminidase has two isoforms
co

Krabbe’s disease β-galactosidase/


• β-hexosaminidase A, composed of 1 α and 1 β subunit
β-galactocerebrosidase
• β-hexosaminidase B, composed of 2 β subunits
• Mutation in α subunit causes Tay–Sachs Disease, so only
β-hexosaminidase A
IMPORTANT SPHINGOLIPIDOSIS • Mutation in β subunit causes Sandhoffs Disease, so both
β-hexosaminidase A and B defect as β subunit is common to
GM1 Gangliosidosis both isoforms.
Autosomal recessive trait
Clinical Features
Biochemical Defect
Tay–Sachs Disease
• Deficient activity of β-galactosidase, a lysosomal
• The infantile form of Tay–Sachs disease have clinical
enzyme encoded by a gene on chromosome 3
manifestations in infancy
• Accumulation of GM1 gangliosides in the lysosomes
m

• Loss of motor skills


of both neural and visceral cells
co

• Increased startle reaction (hyperacusis)


• A mucopolysaccharide accumulate in GM1 Gangli-
osidoses is Keratan Sulfate, in the liver and excreted • Macular pallor and retinal cherry red spots
in urine. • Macrocephaly
• Seizures develop which may be refractory to anti-
Clinical Presentation convulsant therapy
• Most frequently presents in the first 6 months of life • Neurodegeneration is relentless
with developmental delay followed by progressive • Death occurring by the age of 4 or 5 years.
psychomotor retardation and the onset of tonic-clonic
seizures Sandhoff Disease
• Hepatosplenomegaly, edema, and skin eruptions Similar to Taysachs but infants with Sandhoff disease
(angiokeratoma) have hepatosplenomegaly, cardiac involvement, and mild
• A typical facies is characterized by low-set ears, bony abnormalities.
m

frontal bossing, a depressed nasal bridge, and an Diagnosis


co

abnormally long philtrum Definitive diagnosis is made by determination of


• Fifty percent of patients have a macular cherry red β-hexosaminidase A and B activities in peripheral
spot leukocytes.
e

e
m

m
m
co

Chemistry of Lipids and Biomembranes  | 187

Gaucher’s Disease Two other disorders with Gaucher cells


Most common Lysosomal Storage Disorder. • Granulocytic Leukemia
Autosomal Recessive. • Myeloma.

Biochemical Defect • Determination of the acid β-glucosidase activity in


Deficient activity of the lysosomal hydrolase, acid isolated leukocytes or cultured fibroblasts.
m

β-glucosidase (β-glucocerebrosidase). This is encoded Treatment


co

by a gene located on chromosome 1q. Enzyme replacement therapy (ERT)


The enzymatic defect results in the accumulation • Mannose terminated Recombinant Human Acid Beta
of undegraded glycolipid substrates, particularly Glucosidase (Imiglucerase)
glucocerebroside (glucosylceramide), in cells of the
‒ Most symptoms including organomegaly,
reticuloendothelial system. hematological manifestation, bone pain improves.
Clinical Manifestations • Two additional Enzyme Preparation approved by
Type I Gaucher Disease FDA are
‒ Velaglucerase alfa, which is produced from
• Variable age of onset (early childhood to late adult
human fibro sarcoma cells
hood)
‒ Taliglucerase alfa, which is produced in carrot
• Hematological features: Pancytopenia, bleeding
cells.
manifestation, chronic fatigue
m

• Hepatosplenomegaly Oral Substrate reduction agents


co

• Bone Pain (In a pseudo-osteomyelitis pattern) and • Miglustat


pathological fractures of long bones ‒ To decrease glucosylceramide by chemical
• Bone crises with severe pain and swelling can occur inhibition of glucosylceramide Synthase.
• No cherry red spot in the macula Bone marrow transplantation
• No mental deterioration. Concept of clinical manifestations and diagnosis of Gaucher
Disease
Type 2 Gaucher Disease • Accumulation of Glucocerebroside in the reticuloendothelial
• Rapid neurodegenerative course system, so there is Hepatosplenomegaly
• Infiltration into the bone marrow, so pancytopenia which includes
• With extensive visceral involvement thrombocytopenia, which causes bleeding manifestation and
• Death within the first years of life. anemia
• Skeletal manifestation like bone pain and pathological fractures.
m

Type 3 Gaucher Disease Diagnosis


co

• Clinical manifestations are intermediate to those seen • Gaucher disease should be considered in the differential diagnosis
of patients with unexplained organomegaly, who bruise easily, have
in types 1 and 2 bone pain, or have a combination of these conditions.
• Presents in childhood and death by age 10–15 years.
Niemann-Pick Disease
Diagnosis Autosomal recessive
• X-ray Femur: Erlenmeyer Flask Deformity
• Bone marrow examination. Biochemical Defect
‒ The pathologic hallmark of Gaucher disease is • Deficient activity of acid sphingomyelinase, a lysoso-
the Gaucher cell particularly in the bone marrow. mal enzyme encoded by a gene on chromosome 11
• Accumulation of sphingomyelin and other lipids in
Gaucher cell the monocyte-macrophage system.
• They have characteristic wrinkled paper appearance
m

resulting from the presence of intracytoplasmic Clinical Features


co

substrate inclusions • Failure to thrive


• The cytoplasm of the Gaucher cell reacts strongly • Hepatosplenomegaly
positive with the periodic acid–Schiff stain. • Rapidly progressive neurodegenerative course.
e

e
m

m
m
co

188 |  
Self Assessment and Review of Biochemistry

Treatment Renal transplantation and long-term hemodialysis are


• Orthotopic liver transplantation lifesaving procedures for patients with renal failure.
• Amniotic cell transplantation Enzyme Replacement Therapy
• Bone marrow transplantation • Recombinant α-galactosidase (Agalzidase β or
• Miglustat Fabrazyme) is a safe and effective enzyme replacement
• A phase I trial of enzyme replacement therapy for therapy of choice for Fabry disease produced from
m

type B NPD. Chinese hamster ovary cells approved by FDA.


co

• Agalsidase α (Replagal)
Fabry Disease
X-linked recessive condition. Krabbe Disease
• Also called globoid cell leukodystrophy
Biochemical Defect • Autosomal recessive.
• Mutations in the α-galactosidase
• A gene located on the long arm of the X. Chromosome Biochemical Defect
(Xq22) • Deficiency of the enzymatic activity of galactocer-
• The enzymatic defect leads to the systemic accu- ebrosidase (Beta Galactosidase)
mulation of neutral glycosphingolipids, primarily • Accumulation of galactosylceramide in the white
globotriaosylceramide, particularly in the plasma matter of brain
and lysosomes of vascular endothelial and smooth • Galactocerebroside is normally found almost
m

muscle cells. exclusively in the myelin sheath


co

• The galactocerebrosidase gene is on chromosome 14.


Clinical Features
• Angiokeratomas (telangiectatic skin lesions. Charac- Clinical Features
teristically, the lesions are most dense between the • The infantile form of Krabbe disease is rapidly
umbilicus and knees, in the ‘bathing trunk area,’) progressive and patients present in early infancy with
• Hypohidrosis irritability, seizures, and hypertonia
• Corneal and lenticular opacities • Optic atrophy is evident in the 1st year of life
• Acroparesthesias • Mental development is severely impaired
• Pain is the most debilitating symptom in childhood • As the disease progresses, optic atrophy and severe
and adolescence developmental delay become apparent; affected
children exhibit Opisthotonus and die before 3 years
• Fabry crises, lasting from minutes to several days,
of age.
m

consist of agonizing, burning pain in the hands, feet,


and proximal extremities and are usually associated
co

Diagnosis
with exercise, fatigue, fever Demonstration of the specific enzymatic deficiency in
• Red cell casts and lipid inclusions with characteristic white blood cells or cultured skin fibroblasts.
birefringent “Maltese crosses” appear in the urinary
sediment. Farber Disease
• Mitral insufficiency is the most common valvular Autosomal recessive disorder.
lesion.
Biochemical Defect
Lab Diagnosis • Deficiency of the lysosomal enzyme acid ceramidase
• α-galactosidase A activity in plasma, isolated • The accumulation of ceramide in various tissues,
leukocytes, or cultured fibroblasts or lymphoblasts. especially the joints.

Treatment Clinical Features


m

Phenytoin and/or carbamazepine • Symptoms can begin as early as the 1st year of life
co

• Decrease the frequency and severity of the chronic with painful joint swelling and nodule formation
acroparesthesias and the periodic crises of excruciat- over the joints
ing pain. • Mimicks Rheumatoid Arthritis.
e

e
m

m
m
co

Chemistry of Lipids and Biomembranes  | 189

Diagnosis Contd...
Ceramidase activity should be determined in cultured Sphingolipidoses with no Hepatosplenomegaly
skin fibroblasts or peripheral leukocytes. • Fabry’s Disease
• Metachromatic Leukodystrophy
Wolman Disease and Cholesterol Ester Storage • Krabbe’s Disease
Disease (CESD) Sphingolipidoses with Corneal Clouding
m

• Autosomal recessive condition • Fabry’s Disease


• GM1 Gangliosidoses
co

• Lysosomal storage diseases.


Sphingolipidoses with angiokeratoma
• GM1 Gangliosidoses
Biochemical Defect
• Fabrys Disease
• Deficiency of acid lipase
• Accumulation of cholesterol esters and triglycerides
in histiocytic foam cells of most visceral organs BIOMEMBRANES
• The gene for lysosomal acid lipase is on chromosome 10. Properties of Biomembranes
Clinical Features • Membranes are sheet like structures, that form closed
boundaries between different compartments. The
• Presents in 1st week of life
thickness of most membranes is between 60 Å (6 nm)
• Failure to thrive
and 100 Å (10 nm)
m

• Relentless vomiting • Membranes consist mainly of lipids and proteins. The


co

• Abdominal distention mass ratio of lipids to proteins ranges from 1:4 to 4:1.
• Steatorrhea Membranes also contain carbohydrates that are linked
• Hepatic dysfunction and fibrosis may occur to lipids and proteins
• Calcification of the adrenal glands is pathognomonic • Membrane lipids are small molecules that have
for the disorder both hydrophilic and hydrophobic moieties. These
lipids spontaneously form closed bimolecular sheets in
• Death usually occurs within 6 months.
aqueous media. These lipid bilayers are barriers to the
Cholesterol ester storage disease is a less severe disorder flow of polar molecules
that may not be diagnosed until adulthood. • Specific proteins mediate distinctive functions of mem-
• Hepatomegaly can be the only detectable abnormality. branes. Proteins serve as pumps, channels, receptors,
energy transducers, and enzymes
Diagnosis
m

• Membranes are noncovalent assemblies. The cons-


• Measuring acid lipase activity in peripheral leukocytes
co

tituent protein and lipid molecules are held


or cultured skin fibroblasts
together by many noncovalent interactions, which act
• Mutations detection in acid ceramide gene. cooperatively
Treatment • Membranes are asymmetric. The two faces of
biological membranes always differ from each other.
There is no specific therapy available for either disorder,
although pharmacologic agents to suppress cholesterol • Membranes are fluid structures
synthesis, in combination with cholestyramine and diet • Lipid molecules diffuse rapidly in the plane of the
modification. membrane, as do proteins, unless they are anchored
by specific interactions. This is lateral diffusion
Quick review-Sphingolipidoses
• In contrast to lipid molecules, proteins do not rotate
Sphingolipidosis with no Cherry red spot on macula-Gauchers Type I,
Fabry’s Disease across the membrane. The movement of proteins is
called transverse diffusion or flip-flop movement.
m

Most Common Lysosomal Storage Disorder- Gaucher Type I


• Most cell membranes are electrically polarized, such
co

Sphingolipidosis with no mental Deterioration-Gaucher Type I and


Fabry’s Disease that the inside is negative [typically 260 millivolts
Sphingolipidosis with X linked recessive inheritance-Fabry’s Disease (mV)]. Membrane potential plays a key role in
Contd... transport, energy conversion, and excitability.
e

e
m

m
m
co

190 |  
Self Assessment and Review of Biochemistry

Sterol
• The most common sterol in animal cell is cholesterol
• Cholesterol is not present in plants.

Fluid Mosaic Model of BiomembranesQ


• Proposed by Singer and Nicolson in 1972
m

• The membrane consist of bimolecular lipid bilayer


with proteins inserted in it or bound to either surface
co

• This model is likened to integral membrane protein


‘icebergs’ floating in a sea of predominantly fluid
phospholipid molecules.
m

Fig. 6.9: Lateral diffusion and flip-flop diffusion


co

Components of Membranes
Membranes are complex structures composed of lipids,
proteins, and carbohydrate-containing molecules.
Ratio of Protein to Lipid in Different Membranes Fig. 6.10: Fluid mosaic model of plasma membrane
Proteins equal or exceed the quantity of lipid in nearly all
membranes. The outstanding exception is myelin (Protein Membrane Proteins
to lipid ratio is 0.23). Membranes Contain Integral and Peripheral Proteins.
Integral Protein
The Major Lipids in Mammalian Membranes
• Deeply embedded (to both hydrophilic and hydro-
• Phospholipids
phobic portions) in the lipid bilayer
• Glycosphingolipids
• Usually globular and are themselves amphipathic
m

• Sterols.
• Trans membrane integral protein are the proteins
Membrane lipids are amphipathic.
co

that span the whole lipid bilayer.


Phospholipid Peripheral Proteins
Two Major Phospholipid Classes Present in Membranes • Attached to the Hydrophilic portions of Plasma
are Glycerophospholipid and Sphingomyelin membrane
• Glycerophospholipid are the most common phos-
pholipid (Lecithin is the most common Glycerophos-
pholipid)
• Choline-containing phospholipids (phosphatidyl-
choline and sphingomyelin) are located mainly in
the outermolecular layer
• Aminophospholipids (phosphatidylserine and phos-
phatidylethanolamine) are preferentially located in
m

the inner leaflet.


co

Glycosphingolipids
• Glycosphingolipids are cerebrosides and gangliosides
• The back bone of GSL is ceramide. Fig. 6.11: Peripheral proteins and integral proteins
e

e
m

m
m
co

Chemistry of Lipids and Biomembranes  | 191

• Do not interact directly with the hydrophobic cores Caveolae


of the phospholipids in the bilayer. Flask shaped indentation of the cell membrane facing the
cytosol. Contain the protein caveolin-1.
Fluidity of Membranes
The temperature at which the structure undergoes the Functions
transition from ordered to disordered (i.e. melts) is called • Signal transduction system (e.g. the insulin receptor
m

the ‘transition temperature’ (Tm). and some G proteins), the folate receptor, and
co

endothelial nitric oxide synthase (eNOS)


Factors Affecting the Melting Temperature
• Transport of macromolecules [IgA]
The properties of fatty acids and lipids derived from them
• Endocytosis of Cholesterol containing lipoprotein.
are markedly dependent on chain length and degree of
saturation.
• The longer and more saturated fatty acid chains
cause higher values of Tm. So decreases the fluidity
of membranes
• Unsaturated fatty acid increases the fluidity (i.e. they
lower Tm)
• Cholesterol modifies the fluidity of membranes.
‒ At temperatures below the Tm increases fluidity
m

‒ At temperatures above the Tm decreases fluidty.


co

At normal body temperature (37°C) the lipid bilayer is in a


fluid state.

SPECIALIZED REGION OF PLASMA


MEMBRANE Fig. 6.13: Caveolae

Lipid RaftsQ
Intercellular Connections
Lipid rafts are specialized areas of the exoplasmic Intercellular junctions that form between the cells in
leaflet of the lipid bilayer: Enriched in cholesterol, tissues can be broadly split into two groups:
sphingolipids. Contain certain GPI-linked proteins • Junctions that fasten the cells to one another and to
m

(outer leaflet) and acylated and prenylated proteins surrounding tissues


co

(inner leaflet). Important for signal transduction and other


• Junctions that permit transfer of ions and other
processes. molecules from one cell to another.

Group I: The Types of Junctions That Tie Cells


Together
• Tight junctions [the zonulaoccludens]
‒ Prevent the diffusion of macromolecules between
cells.
‒ Are composed of various proteins, including
occludin, various claudins, and junctional
adhesion molecules [JAMs]
‒ Absence of Tight junction implicated in loss of
m

contact inhibition
co

• Desmosome and zonula adherens also help to hold


Fig. 6.12: Lipid rafts cells together
e

e
m

m
m
co

192 |  
Self Assessment and Review of Biochemistry

• Hemidesmosome and focal adhesions attach cells to Self-oriented structures formed by amphipathic lipids
their basal laminas. • Amphipathic lipids self-orient at Oil: Water interfaces
• They form Membranes, Micelles, Liposomes, and
Emulsions.
Lipid bilayer
A bilayer of such amphipathic lipids is the basic structure
m

in biologic membranes.
co

Micelle
When a critical concentration of these lipids is present in
an aqueous medium, they form micelles. Aggregation of
bile salts into micelles and liposomes and the formation
of mixed micelles with the products of fat digestion are
important in facilitating absorption of lipids from the
intestine.
Liposomes
• Are formed by sonicating an amphipathic lipid in an
aqueous medium
• They consist of spheres of lipid bilayers that enclose
m

part of the aqueous medium.


co

Clinical uses of liposomes


Fig. 6.14: Intercellular connections
• As carriers of drugs in the circulation, targeted to
specific organs, for example, in cancer therapy
Group II: Junctions That Permit Transfer of Ions and
• They are used for gene transfer into vascular cells as
Other Molecules from One Cell to Another
carriers for topical and transdermal delivery of drugs
Gap Junctions and cosmetics.
• Cytoplasmic ‘tunnel’ for diffusion of small molecules
(< 1000 Da) between two neighbouring cells
• At gap junctions, the intercellular space narrows
from 25 to 3 nm, and units called connexons in the
membrane of each cell are lined up with one another
m

• Each connexon is made up of six protein subunits


co

called connexins.
m
co

Figs 6.15A and B: Gap junction Fig. 6.16: Self-oriented structures formed by amphipathic lipids
e

e
m

m
m
co

Chemistry of Lipids and Biomembranes  | 193

REVIEW QUESTIONS

Lipids of Physiological Significance d. ↑ LDL


e. ↑ HDL
1. Essential fatty acid is/are: (PGI May 2013)
Ans. a, b, d, e. (Ref: Harper 30/e p241)
a. Palmitic acid
m

b. Linoleic acid Trans fatty acids


co

c. Linolenic acid • Present in dairy products and partially hydrogenated


d. Oleic acid edible oils
e. Free fatty acid • They are used in food industry to improve the shelf
Ans. b. Linoleic acid, c. Linolenic Acid. life
• Trans fatty acids are present in high amounts in
Essential fatty acid
processed foods, fast foods and bakery items
The fatty acids that are required by humans, but are not
• Consumption of trans fatty acid for long-terms may
synthesized in the body hence need to be supplied in the
raise the risk factor for cardiovascular diseases like
diet are known as essential fatty acid (EFA). Humans lack
the enzymes that can introduce double bond beyond 9th Atherosclerosis and Coronary Artery disease and
Carbon. Diabetes mellitus
They are polyunsaturated fatty acid namely • Exacerbate essential fatty acid deficiency by compet-
m

ing for essential fatty acid.


• Linoleic Acid
co

• Linolenic Acid 4. PUFA (more than 50%) content is seen in:


Linoleic acid is the most essential fatty acid arachidonic (PGI Nov 2009)
Acid is considered as Semi essential fatty Acid as it can a. Groundnut oil
be synthesized from Linoleic Acid. b. Safflower oil
Functions of essential fatty acid c. Corn oil
• They are integral components of membrane structure, d. Sunflower oil
often in the 2 position of phospholipids
Ans. b, c, d. (Ref: Park 23/e p611 Table 2)
• Eicasanoids are synthesized from Arachidonic Acid.
• Highest content of MUFA in Mustard/Rapeseed oil
Essential fatty acids are needed for the synthesis of
Arachidonic Acid • Highest content of Medium chain fatty acid in
• Lower the risk of Cardiovascular Diseases. Coconut oil
m

• Highest content Linoleic acid in Safflower oil


co

2. True statement about fatty acid: (PGI Nov 2011) • Highest content alpha linolenic acid in Flax seed oil
a. PUFA is essential for membrane structure
SFA MUFA Linoleic Alpha linolenic
b. Biologically arachidonic acid is essential to life Fats/Oil (%) (%) acid (%) acid (%)
c. Hydrogenated vegetable oils contains trans High Medium chain and Short chain fatty acid
fatty acid
Coconut 92 6 2 –
d. Most of the naturally occurring unsaturated
Palm kernel 83 15 2 –
FA exist as trans isomer
Butter/Ghee 68 29 2 1
Ans. a, b, c. (Ref: Harper 30/e p241)
High SFA and MUFA
Most of the naturally occurring UFA exist in cis form.
Palmolein 39 46 11 < 0.5
3. True about trans fatty acid: (PGI Nov 2010) High MUFA and Moderate Linoleic acid
a. Fried rice have high content of Trans Fatty
m

Ground nut 19 41 32 < 0.5


Acid
co

Rice bran 17 43 38 1
b. Partial Hydrogenation increases Trans Fatty
Sesame 16 41 42 < 0.5
Acid
c. Refining decreased TFA Contd...
e

e
m

m
m
co

194 |  
Self Assessment and Review of Biochemistry

Contd... 6. Which among the following is not a saturated


SFA MUFA Linoleic Alpha linolenic fatty acid? (Kerala 2009)
Fats/Oil (%) (%) acid (%) acid (%) a. Myristic acid
High linoleic acid b. Stearic acid
Cotton seed 24 29 48 1 c. Palmitic
Corn 12 35 50 1 d. Linoleic acid
m

Safflower 9 13 75 – Ans. d. Linoleic acid. (Ref: Harper 30/e p213)


co

Sunflower 12 22 62 –
Common Saturated Fatty Acids and Their Sources
High Linoleic acid and alpha linolenic acid
Saturated fatty acid Source
Soyabean 14 24 53 7
Acetic Acid (2C) Vinegar
Canola 6 60 22 10
Butyric Acid (4C) Butter
Mustard/Rape- 4 65 15 14
Valeric Acid (5C) Butter
seed
Caproic Acid (6C) Butter and Coconut milk
Flax-Seed 10 21 16 5
Lauric Acid (12C) Coconut milk
High trans fatty acid
Myristic Acid (14C) Coconut milk
Vanspati 46 49 4 –
Palmitic Acid (16C) Body Fat
Stearic Acid (18C) Body Fat
m

5. Which among the following is a cardio protective


co

fatty acid? (Kerala 2011)


7. Most essential fatty acid is: (Kerala 2008)
a. Palmitic acid
a. Linolenic acid
b. Stearic acid
b. Linoleic acid
c. Oleic acid
c. Arachidonic acid
d. Omega-3 fatty acid
d. Eicosapentaenoic acid
Ans. d. Omega-3 fatty acid. (Ref: Harper 30/e p248)
Ans. b. Linoleic Acid. (Ref: Harper 30/e p 222, 241)
Significance of ω3 Fatty Acid
• Linoleic Acid is the Most Essential Fatty Acid
• Decrease the risk of Cardiovascular Disease
Arachidonic Acid is considered as Semi essential fatty
• Appear to replace arachidonic acid in platelet Acid as it can be synthesized from Linoleic Acid.
membranes
• Lower the production of thromboxane and tendency Functions of essential fatty acid
m

of the platelet aggregation • They are integral components of membrane structure,


co

• Decrease Serum Triglycerides often in the 2 position of phospholipids


• Important for Infant Development • Eicasanoids are synthesized from Arachidonic Acid.
Essential fatty acids are needed for the synthesis of
• Lower the risk of various mental illness (Depression,
Arachidonic Acid
ADHD)
• Lower the risk of Cardiovascular Diseases.
• Lower the risk of chronic degenerative diseases such
as Cancer, Rheumatoid Arthritis, and Alzheimers 8. All are true except: (PGI Dec 2002)
Disease. a. Linoleic acid is found in Soyabean oil
Omega Classification of Fatty Acids b. Linolenic and linoleic acids are cis derivatives
ω3 Series α Linolenic Acid containing double bonds
Timnodonic Acid (EicosaPentaenoic Acid) c. Arachidonic acid contain five double bonds
m

Cervonic Acid (DocosaHexaenoic Acid) (DHA) d. Monoenoic acids contain one double bond at
co

ω6 Series Linoleic Acid 9th position


γLinolenic Acid (GLA) Arachidonic Acid
Ans. c. Arachidonic acid contain five double bonds.
ω9 Series Oleic Acid Elaidic Acid
(Ref: Harper 30/e p212, 238)
e

e
m

m
m
co

Chemistry of Lipids and Biomembranes  | 195

• Fifty-three percent of Soyabean oil is Linoleic acid 12. An example Omega 6 Fatty acid is:
• Almost all unsaturated fatty acids have cis configu- (CMC Ludhiana 2014)
ration a. Cervonic Acid
• Arachidonic acid has 4 double bonds b. αLinolenic Acid
• First double bond is usually added in 9th position by c. Arachidonic Acid
a delta 9 desaturase. d. Timnodonic acid
m

Ans. c. Arachidonic Acid. (Ref: Harper 30/e p214)


9. Maximum source of linoleic acid is:
co

(AIIMS Jun 97) 13. Which is not present in plants?


a. Coconut oil (CMC Ludhiana 2014)
b. Sunflower oil a. Cholesterol
c. Palm oil b. Linolenic acid
d. Vanaspati c. Linoleic Acid
Ans. b. Sunflower oil. d. Lauric Acid
• Highest content of MUFA in Mustard/Rapeseed oil Ans. a. Cholesterol.
• Highest content of Medium chain fatty acid in
Coconut oil
Phospholipids and Glycolipids
14. Gangliosides contain: (PGI May 2015)
• Highest content Linoleic acid in Safflower oil
a. Phosphate
• Highest content alpha linolenic acid in Flax seed oil.
m

b. Galactose
co

10. Which of these fatty acids is found exclusively in c. Sulfate


breast milk: (AI 2001) d. Serine
a. Linolaete e. Sialic Acid
b. Linolenic Ans. b. Galactose, e. Sialic Acid.
c. Palmitic (Ref: Harper 30/e p218)
d. Docosahexanoic acid Ganglioside
Ans. d. Docosa Hexaenoic acid. • Contain Sphingosine + Fatty acid + Oligosaccharide
(Ref: Harper 30/e p214 Table 22) that contain one or two molecules of sialic acid.
• Docosahexaenoic acid is present in fish oils, phos-
pholipids of brain, algal oils 15. Glycosphingolipid is made up of:
(PGI Nov 2010)
• DHA is otherwise called Cervonic acid
m

a. Glucose
• It belongs to ω3 fatty acid.
b. Glycerol
co

11. The following fatty acid does not belong to ω6 c. Sphingosine


series linoleic acid: d. Fatty acids
(PGI June 2000, AIIMS Dec 90) e. Thromboxane A2
a. Arachidonic acid Ans. a, c, d. (Ref: Harper 30/e p219)
b. Gamma-linoleic acid
Phospholipids
c. Alpha-linolenic acid
Lipids containing phosphoric acid and nitrogenous base
d. Timnodonic acid in addition to glycerol and fatty acid. Fatty Acid + Alcohol
Ans. d. Alpha linolenic acid. (Ref: Harper 30/e p214) (Glycerol/Sphingosine) + Phosphate + Nitrogenous base
ω3 FA ω6 FA ω7 FA ω9 FA
Glycerophospholipids
Alpha-linolenic acid Linoleic acid Palmitoleic Oleic
The alcohol in this group is Glycerol, e.g. Lecithin,
m

acid acid
Cephalin.
co

Timnodonic acid (Eicosa- Gamma-lino-


pentaenoic acid) lenic acid Sphingophospholipids
Cervonic Acid Arachidonic The alcohol in this group is Sphingosine, e.g. Sphingo-
(Docosahexaenoic acid) acid
myelin.
e

e
m

m
m
co

196 |  
Self Assessment and Review of Biochemistry

Glycolipids or Glycosphingolipids Sphingolipidosis


Lipids containing carbohydrate apart from Fatty acid and 19. A child presents with hepatosplenomegaly and
Alcohol (usually Sphingosine). pancytopenia. Bone marrow shows ‘crumbled
Fatty Acid + Alcohol (Sphingosine) + Carbohydrate, e.g. tissue paper appearance’. It is due to accumulation
Cerebroside, Ganglioside. of: (AIIMS May 2013)
a. Glucocerebroside
16. Alcoholic group is found in: (PGI June 98)
m

a. Ganglioside b. Sphingomyelin
co

b. Sphingomyelin c. Ganglioside
c. Cerebroside d. Galactocerebroside
d. Ceramide Ans. a. Glucocerebroside.
Ans. a, b, c, d. (Ref: Nelson 20/e Chapter 86 4 Lipidoses)
• Ceramide = Sphingosine (Amino alcohol) + Fatty acid Enzyme Sphingolipid
Sphingolipidoses Deficiency accumulated
• Ganglioside = Ceramide + Oligosaccharide that
Farber disease Ceramidase Ceramide
contains N Acetyl Neuraminic acid
• Cerebroside = Ceramide +Monosaccharide Fabry disease α-galactosidase Globotriosylceramide

• Sphingomyelin = Glycerol + 2 Fatty acid + PO4 + GM1 gangliosi- β-galactosidase GM1 ganglioside
dosis
Choline.
GM2 Gangliosidosis
17. Which of the following is a glycolipid?
m

Tay-Sachs disease, β-hexosaminidases A GM2 ganglioside


(JIPMER 2012)
co

Sandhoff disease β-hexosaminidases GM2 Ganglioside


a. Cerebroside A and B
b. Plasmalogen Gaucher disease Glucocerebrosidase/ Glucosylceramide
c. Sphingomyelin β-glucosidase

d. Lecithin Niemann-Pick Sphingomyelinase Sphingomyelin


Disease
Ans. a. Cerebroside. (Ref: Harper 30/e p218)
Metachromatic Arylsulfatase A Sulfogalactosylce-
Glycolipids or Glycosphingolipids leukodystrophy Sphingolipid Activa- ramide
• Cerebroside tor Protein (SAP-1)
• Globoside Krabbe disease β-galactosidase Galactosylceramide
• Ganglioside. β-galactocerebro-
sidase
18. Second messenger is produced from:
m

a. Phosphatidylinositol Gaucher’s Disease


co

b. Phosphatidylserine • Most common Lysosomal Storage Disorder


c. Phosphatidylcholine • Glucocerebrosidase defect
d. None • Lysosomes filled with Glucocerebroside
Ans. a. Phosphatidylinositol. (Ref: Harper 30/e p216) • No cherry red spot in the macula
Phosphatidylinositol • No mental deterioration (Type I)
• Phosphatidyl Inositol present in the cell membrane • Hematological features: Pancytopenia, bleeding
manifestation
• The inositol is present as its stereo isomer, Myoinositol.
• Hepatosplenomegaly
• Play an important role in cell signalling and
membrane trafficking • Bone Pain and Pathological Fractures of long bones
• Phosphatidylinositol is a precursor of second • X-ray Femur Erlenmeyer Flask Deformity
messenger in hormonal pathways. • Bone Marrow Biopsy Gaucher cell with Wrinkled
m

paper appearance/crumbled tissue paper appearance.


Action of Phospholipase C on Phosphatidylinositol 4,5
co

Bisphosphate (PiP2) 20. Which is/are sphingolipidosis? (PGI May 2013)


Diacylglycerol and Inositol Tris phosphate IP3 are formed a. Tay-Sachs disease
and both act as second messengers. b. Fabry’s disease
e

e
m

m
m
co

Chemistry of Lipids and Biomembranes  | 197

c. Krabbe’s disease I cell Disease


d. Sandhoff’s disease • Due to defective synthesis of recognition marker –
e. Wolman’s disease Mannose 6-phosphate
Ans. a, b, c, d. (Ref: Nelson 20/e Chapter 86.4 Lipidoses) • Serum level of Lysosomal enzymes raised.
Sphingolipidosis Enzyme defect 23. Which of the following disease occurs due to the
deficiency of glucocerebrosidase? (Kerala 2008)
m

Fabry disease α-galactosidase


a. Gaucher disease
co

Farber disease Ceramidase


Galactosialidosis β-galactosidase and sialidase
b. Pompe disease
c. Fabry disease
GM1 gangliosidosis β-galactosidase
d. Krabbe disease
GM2 gangliosidosis
Ans. a. Gaucher Disease.
Tay-Sachs disease β-hexosaminidases A
(Ref: Nelson 20/e Chapter 86.4 Lipidoses)
Sandhoff’s disease β-hexosaminidases A and B
Gaucher Type I Glucocerebrosidase 24. Accumulation of sphingomyelin in phagocytic
Gaucher Type II Glucocerebrosidase cells is feature of: (AIIMS Nov 02)
a. Tay–sachs disease
Gaucher Type III Glucocerebrosidase
b. Gaucher’s disease
Niemann-Pick Type A Sphingomyelinase
c. Niemann-Pick disease
Niemann-Pick Type B Sphingomyelinase
m

d. Down’s syndrome
Metachromatic leukodystrophy Arylsulfatase A or Sphingolipid
Ans. c. Niemann-Pick Disease.
co

Activator Protein (SAP-1)


Krabbe disease β-galactocerebrosidase
Niemann-Pick Disease Autosomal Recessive Biochemical
/β-galactosidase defect
• Deficient activity of acid sphingomyelinase, a
lysosomal enzyme encoded by a gene on chromosome
21. Sphingomyelinase Deficiency is seen in: 11
(AI 2010)
• Accumulation of sphingomyelin and other lipids in
a. Niemann-Pick disease the monocyte-macrophage system.
b. Farber’s disease
Clinical features
c. Tay-Sachs disease
• Failure to thrive
d. Krabbe’s disease
• Hepatosplenomegaly
Ans. a. Niemann-Pick disease. • Rapidly progressive neurodegenerative course.
m

(Ref: Nelson 20/e Chapter 86.4 Lipidoses)


co

Treatment
Other options
• Orthotopic liver transplantation
• Farber’s Disease—Ceramidase
• Amniotic cell transplantation
• Tay-Sachs Disease-β-hexosaminidase A
• Bone marrow transplantation
• Krabbe’s Disease-β-galactosidase.
• Miglustat
22. Deficiency of phosphorylating enzymes for the • A phase I trial of enzyme replacement therapy for
formation of which of the following recognitior type B NPD.
marker leads to I- cell disease? (Kerala 2008) 25. Tay-Sachs disease is due to accumulation of:
a. GM2 ganglioside (JIPMER 2012)
b. Mannose 6-phosphate a. GM2 ganglioside
c. Galactose b. GM1 ganglioside
m

d. Globoside c. Glucocerebroside
co

Ans. b. Mannose 6-phosphate. (Ref: Harper 30/e p586) d. Galactocerebroside


Disorders associated with defect in import of Proteins Ans. a. GM2 gangliosidoses.
to Lysosomes. (Ref: Nelson 20/e Chapter 86.4 Lipidoses)
e

e
m

m
m
co

198 |  
Self Assessment and Review of Biochemistry

Biomembranes • Membranes are Complex Structures Composed


of Lipids, Proteins, and Carbohydrate-Containing
26. Eukaryotic plasma membrane is made up of all Molecules
except: (May 2009) • The Major Lipids in Mammalian Membranes Are
a. Carbohydrates Phospholipids, Glycosphingolipids and Cholesterol
b. Triglycerides • Two major phospholipid classes present in mem-
c. Lecithin
m

branes, phosphoglycerides are the more common


d. Cholesterol
co

• Among the Phosphoglycerides, lecithin is the most


Ans. b. Triglycerides (Ref: Harper 30/e p478) abundant.
m
co
m
co
m
co
7 Metabolism of Lipids

Topics Included
 Digestion and Absorption of Lipids  Metabolism of Fatty Acids
 Metabolism of Simple Lipids  Metabolism of Ketone Bodies
– Triacylglycerol  Metabolism of Cholesterol
 Metabolism of Compound Lipids  Metabolism of Lipoproteins
– Phospholipids
– Glycolipids

DIGESTION AND ABSORPTION OF LIPIDS  R!   



   
 !"#    
       $
The major lipids in the diet are triacylglycerols (>90%) and
   
   
the rest is made of phospholipids, cholesterol, cholesterol
 

    Pancreatic esterase
 25% of monoacylglycerols are hydrolyzed to glycerol
Enzymes for Digestion of Lipids  
   
Lingual lipase and gastric lipase  Cholesterol esters and other lipid esters are also
 Hydrolysis of triacylglycerols is initiated by lingual  & '
and gastric lipases Phospholipases
 Site: Stomach  *   +#+  
 Action: Hydrolysis of 3rd ester bond of Triacylglycerol $ 

  
 30% of total Triacylglycerol digestion takes place in Absorption of Lipids
 
Role of Bile Salt/Bile Acids
Pancreatic lipase  * & 
 Requires a further pancreatic protein, colipase, for  - .  
    At physiological pH bile acids are seen ionized
 Colipases prevents the inhibition Lipases by bile (anions) so bile acids and bile salts (anions) are
    .  
 This is the major enzyme of Triacylglycerol  The products of lipid digestion are hydrophobic
hydrolysis  
   
   /4/   
  
1 and 3 in triacylglycerol 
   
200
Self Assessment and Review of Biochemistry

 The minimal concentration of bile acids necessary Activation of Fatty Acid


for micelle formation is called Critical Micellar  Enzyme: Acyl CoA Synthetase or Thiokinase
6 .7  Reaction: ?     6# 
 Micelles are less than 1 μm in diameter and are
. Activation of Glycerol
 They allow the products of digestion, to be transported  Enzyme: Glycerol Kinase
through the aqueous environment of the intestinal  Reaction: Glycerol is phosphorylated to Glycerol 3
lumen to come into close contact with the brush 
border of the mucosal cells, allowing uptake into the In Muscle and Adipose Tissue
  Glycerol Kinase is absent in muscle and white adipose
 The fat-soluble vitamins, A, D, E and K also 
    =  @ + 
 
  ' 
# + =  
Within the Intestinal Epithelial Cells
 !      &  
    
  
 * 
      
   
 ;   
 Glycerol released in the intestinal lumen is absorbed Fig. 7.1: Activation of fatty acid
 
 < 
       !   
    
 <! 
    !  
reacylated to triacylglycerols via the monoacylglycerol
Fig. 7.2: Activation of glycerol
; 
 6 . 6# 
 
   
 
   
 The newly synthesized Triacylglycerol, Cholesterol Triacylglycerol
ester and phospholipids are incorporated into  Enzyme: Acyltransferase
6    Reaction: Transfer of 3 Acyl moiety to Glycerol
 Chylomicrons are secreted into the lymphatics, enter   
.      Synthesis of Triacylglycerol in the Intestine-Mono-
acylglycerol Pathway
METABOLISM OF TRIACYLGLYCEROL In the intestinal Mucosa, Monoacylglycerol Transferase
converts Monoacylglycerol to 1,2 Diacylglycerol in the
Synthesis of Triacylglycerol Monoacylglycerol Pathway
Triacylglycerol are the predominant form of simple lipids,
;   
=  ?  Degradation of Triacylglycerol (Lipolysis)
    Triacylglycerols are hydrolyzed by a lipase to their
Site: Almost all tissues but predominantly in Liver and 
     
#     Much of this hydrolysis (lipolysis) occurs in adipose
Orgenelle: Majority Endoplasmic Reticulum, a few in 
    -

    ;
 
  . ;.
Three steps for TAG synthesis  -

   F  
 # 
? #   heart, kidney, muscle, lung, testis, and adipose tissue,
 # 
=  but not readily by brain), where they are oxidized or
@ B 

 #  =  ! 
Metabolism of Lipids 201

 The utilization of glycerol depends upon whether


such tissues have the enzyme glycerol kinase
 -= 
  
in liver, kidney, intestine, brown adipose tissue, and
   
NB: Brown Adipose Tissue contains Glycerol Kinase
unlike White Adipose Tissue which lack Glycerol
Kinase.

Fig. 7.4: Action of hormone sensitive lipase

Regulation of Hormone Sensitive Lipase


 The activity of this enzyme is under the control of
 
  
 It is active in phosphorylated state
     
Hormone Sensitive lipase is activated by:
 Glucagon
 Catecholamines (Epinephrine and Norepinephrine)
 ACTH
 TSH
 Glucocorticoids
 Thyroid hormones
 Growth hormone
 X Y" *UF"*UJ
 Z
Mechanism of activation of Hormone Sensitive LipaseQ
 By stimulating the activity of adenylyl cyclase, the
&  #-+ #"+
 #"+ .   cAMP-dependent protein
kinase, activates hormone-sensitive lipase by
 
 But Glucocorticoids promote lipolysis via synthesis of
;.  #"+!  ; 
Fig. 7.3: Synthesis of triacylglycerol Hormone sensitive Lipase is inactivated by
 Insulin
LIPOLYSIS IN ADIPOSE TISSUE  Nicotinic Acid
 + B
 * 
-  F-#=J   
Mechanism of Inactivation of Hormone Sensitive Lipase
 By special enzyme: hormone sensitive Lipase (hSL)
-    [  
         
P QU<?    prostaglandin E1 are accounted for by inhibition of the
from 1st and 3rd Carbon of TAG to form Diacylglycerol,   
 #"+          
 "  V  =
 - U  <   ?  Insulin also stimulates phosphodiesterase inactivates
acid from 2nd Carbon atom which is removed by 2- !*?\7


"  < 
202
Self Assessment and Review of Biochemistry

Fig. 7.5: Regulation of hormone sensitive lipase

METABOLISM OF FATTY ACIDS  ?   *  6 


 #  6#   [ " !
 ?    
drial membrane
 ?   ' 
 Ketone body metabolism

De Novo Fatty Acid Synthesis (Lipogenesis)


"$ .   . ? <  
the pathway is otherwise called Lynen’s Spiral
SiteQ: Liver, kidney, brain, lung, lactating mammary
   
Organelle:Q By an extramitochondrial system [in the
^
CofactorQ V  _#+U#-+"2+,
Biotin, and HCO3– (as a source of CO2)
Acetyl-CoAQ     .  .  
 ? 
#  

Sources of Acetyl CoA


 #.  =   `+   #   6# . 
+  F+UJ  ^
 ? #  {' F  J

Transport of Acetyl CoA


 ? 
#   6# .   .  
    Fig. 7.6: Transport of acetyl CoA
Metabolism of Lipids 203

 *#  6#  .   


 Acetyl CoA condenses with Oxaloacetate within the
  
6
 Citrate is translocated into cytosol via the Tricar-
boxylate Transporter
 Citrate undergoes cleavage to acetyl-CoA and
oxaloacetate catalyzed by ATP Citrate Lyase.

Fatty Acid Synthesis


By two enzyme system
 Acetyl CoA Carboxylase
Fig. 7.8: Fatty acid synthase complex
 ? #  * 6'

Acetyl CoA Carboxylase


 Converts Acetyl CoA (2C) to Malonyl CoA in the
 
#-+
 Acetyl-CoA carboxylase has a requirement for the B
'/
 & 
 This is the rate limiting step Q   ?  #  
* 
 Acetyl CoA carboxylase is not a part?#*6'Q

Fig. 7.7: Acetyl CoA carboxylase

Fatty Acid Synthase (FAS) Multienzyme ComplexQ


 The complex is a homodimer of two identical
polypeptide monomers in which six enzyme activities
   F#6+J
 #6+Q contains the vitamin Pantothenic acid in the
form of 4’-phosphopantotheine
 X-ray crystallography of the three-dimensional
structure, shown that the complex is arranged in an
X shape

        
 Fig. 7.9: De novo fatty acid synthesis
are
 Ketoacyl Synthase Sources of NADPH
 Malonyl-Acetyltransacylase  -  *  
 _#+U
   
@ Hydratase U"++; F+++; J
~ Enoyl Reductase  Malic enzyme F_#+  J
7 Ketoacyl Reductase  The extra mitochondrial isocitrate dehydrogenase
 Thioesterase (Deacylase)  
204
Self Assessment and Review of Biochemistry

Reactions of Fatty Acid Synthase Complex Allosteric regulation


By three stages  +# # # 
 Condensation of Acetyl CoA Carboxylase by CitrateQ
 Reduction  Citrate promotes the conversion of Acetyl CoA
@ P
?   Carboxylase from an Inactive dimer to active
  

Condensation Reactions
 Malony/Acetyl Transacylase: A priming molecule of
  !6# .; €*U
 While malonyl-CoA combines with the adjacent
*U~ƒ!
#6+


 These reactions are catalyzed by malonyl acetyl
transacylase, to form acetyl (acyl)-malonyl
enzyme
 Ketoacyl Synthase: The acetyl group attacks the
methylene group of the malonyl residue, catalyzed
by 3-ketoacyl synthase, and liberates CO2, forming Fig. 7.10: Allosteric regulation of acetyl CoA carboxylase
3-ketoacyl enzyme (acetoacetyl enzyme) (reaction 2),

 €*U Inactivation promoted by long chain Acyl CoA by:
 ? 
#  6#6.' 
Reduction reactions
 By inhibiting the Tricarboxylic transporter that
 Ketoacyl Reductase: The 3-ketoacyl group is reduced
6
  6 
to Hydroxyacyl group by Ketoacyl Reductase
 Hydratase: Hydroxyacyl group is dehydrated to   
 
Unsaturated Acyl (Enoyl) group by Hydratase  By phosphorylation-dephosphorylation by hormones
 Enoyl Reductase: Unsaturated Acyl (Enoyl) group is  Acetyl CoA Carboxylase is active in dephosphorylated
   # . B P   stateQ     
     Glucagon and epinephrine
These reactions of Condensation and reduction repeated Glucagon and Epinephrine inactivate Acetyl CoA
several times till the desired acyl group is assembled on 6.' .  & 
the enzyme
Mechanism of Action of Glucagon and Epinephrine
Thioesterase  = 4B  #"+
 ?   F# J. 
&   #"+
 #"+  +…
complex by the activity of the sixth enzyme in the  - #"+……F#"+……J
complex, thioesterase (deacylase)  #"+……   #"+ … F#"+…J . 
Regulation of Fatty Acid Synthesis + 
Rate limiting Step: Acetyl CoA Carboxylase  #"+… #  6#6.' 
Insulin
Long Term Regulation
Insulin activate Acetyl CoA Carboxylase, by depho-
Control of Enzyme Synthesis by regulation of gene
sphorylating the enzyme.
'
Mechanism of Action of Insulin
Short Term Regulation     #"+ … .  !
 Allosteric regulation  
 6"    This inturn dephosphorylate Ac 6# .' 
Metabolism of Lipids 205

Synthesis of Unsaturated Fatty Acids


 { B   
 / &   ? # 6#
 -     ‡9 Desaturase, a
' 
 - ..    ; ˆ9

 * 
   
   
Humans cannot introduce additional double bond in the fatty acid
 

9  
 
!

 
 
"#$

%&
'  


9,12(

'  

9,12,15) become es
 
* 
 

Oxidation of Fatty Acid


- . ; 
       
; .  #  6#  
 
  
   :
 Y{' 
? #  
 {' 
Z < ? #  
 {' 
‰ ? #  
 {' 
{  ?   
Fig. 7.11: 
  

 

  X{' 
? #  
 Š{' 
?   
Activation of acetyl CoA Inactivation of acetyl CoA
carboxylase carboxylase  
  
  
Citrate Acyl CoA  "  

   ' Q
 Two carbon at a time are cleaved from Carboxyl end
Insulin Glucagon, Epinephrine

  ?   #  6#Q
Dephosphorylation Phosphorylation
 - .;X Y .  
Fates of Acyl CoA
/' 

 

   Site-Organs-Liver, Adipose Tissue, Muscle

Chain Elongation to produce very long chain fatty acid  Organelle-Mitochondria

Desaturation to produce unsaturated Fatty acid   
   


  
 
 
 
 # 

 #  
 - 
   
    
 6  
Elongation of Fatty Acid Chains   
 Occurs in the Endoplasmic Reticulum (the  P 
.' 
†  ƒJ         
 / ? #  B   Site: Cytoplasm
 B   
  !6#  Enzyme-Acyl CoA Synthetase/Thiokinase
(from C10 upward) by two carbons  -    

 
 Malonyl CoA donates 2 Carbon atoms in stepwise   V 
manner  Two inorganic phosphatesQ  
 ?   * 6'
in the Cytosol
 _#+UV ;  
 Elongation reaction are particularly increased in
brain during myelination to provide C22 and C24

   
  Fig. 7.12: Activation of fatty acid
206
Self Assessment and Review of Biochemistry

Transport of Acyl CoA into Mitochondria  Carnitine Acyl Carnitine Translocase


 <!6 ?  #       P6. 6 
Mitochondrial Membrane as CarnitineQ 
Reactions of betaoxidation
 *6 "  ? #   
Involves sequence of four reaction successively cleave
 6

2 Carbons as Acetyl CoA
Carnitine

+ 
% 
- 
.  
  #
+#             

/ 
  # 
$ #


# Reducing equivalents

0  1

' 

2   


  Reaction Enzyme produced
Oxidation Acyl CoA Dehydro *3%4
5
6
.7
      
 genase
 Carnitine Acyltransferase I Hydration Enoyl CoA Hydratase —
 #  6+ -
! Oxidation Hydroxy Acyl CoA De 83%
5
46
.7
`6+-!J 
      hydrogenase
+    Cleavage Thiolase —
 <  {  .
 Transfer acyl group present in the Acyl CoA to
6
# 6
    


 Carnitine Acyl Carnitine Translocase -.
#-+.   .

6.
   
 Acyl Carnitine is translocated across the inner
  . From Beta-oxidation of Palmitic acid (C-16)
 Carnitine Acyltransferase II ?  ;  
.!' `F4J!^
 #   6 +  -
! where n = no of carbon atoms
F6+-!J   
   \ 
.' 
 Located in the Inner mitochondrial membrane ? 
.!' 
 Converts Acyl Carnitine to Acyl CoA  ?#U7#-+
 _#U7#-+
 -#-+
 
.!' ~#-+
?\ 
.!' 
 \Ž_
#-+
 
 \Ž~28 ATPs
Second calculate how many 2 Carbon Acetyl CoA from
16 Carbon Palmitic Acid
 (n/2) where n = no of carbon atoms
* 
+ # 
 (16/2) = 8 Acetyl CoA
 ?#  6#. -6#6 ’#-+
 ?“#  6#
 “Ž’80 ATPs
-#-+
+ #  “”“’’“
#-+& 
 
? #  So net
ATPs from Palmitic Acid = 108 – 2 = 106 ATPs
From Stearic Acid (18C)
 “6 
/!' ”•#  6#
 F“Ž~J”F•Ž’J#-+
 @”•’#-+
Fig. 7.13: Transport of fatty acid Net ATPs produced from stearic acid is 122 – 2 = 120 ATPs
Metabolism of Lipids 207


  

  +mary Treatment is IV Glucose
Controlled by CPT-I Gateway  +  . 
V
  ; 
In the Fed State .  ;
 
   4=      
 ! 
 Increased Acetyl CoA Carboxylase, Malonyl CoA is  {
#-+
=   . 
    .' 
?   
 Malonyl CoAQ .
6+-!  If MCAD deficiency fatty acid oxidation do not
 *   /{'    #-+= 
In the fasting state U 
  
 Decreased Insulin/Glucagon ratio Sudden Infant Death Syndrome (SIDS) common in
 Decreased Acetyl CoA Carboxylase activity, Malonyl ! 
CoA is not increased  
;"6#   

 

 # 6+-! or more especially in the nights, hypoglycemia sets
 *  /'    
Concept—Regulation of Beta oxidation of Fatty acid

/
 

  
9
$

  


  &

Jamaican Vomiting Sickness




# 

 

$   Ackee fruit that grows in Jamaica and West Africa

In fasting state, ATP can be provided by fatty acid oxidation as '  U  


 
 U     . 
 ?  #  6#

:

  &
 

 


 

  
 
 
 Severe Hypoglycemia if ingested as it inhibit beta
' 

   
 Characterized by sudden onset of vomiting 2–6 hrs
after ingestion, followed by convulsion, coma and



   !  
Can occur particularly in the newborn—and especially

   
Clinical features
 Hypoglycemia, which is a consequence of impaired

   ' 
 Muscular weakness, due to lipid accumulation
 Hence they have a vitamin-like dietary requirement

 
Fig. 7.14: Steps of beta oxidation  Treatment is by oral supplementation with carnitine

Clinical Correlations "#$ !  


       #[     
   
"  6 #  6#      '  ;  
F"6#  J
"#$$ !  
 Fasting Hypoglycemia
 #[    ;
 No Ketone Bodies

 Vomiting, Coma and Death
 C8-C10 Acyl Carnitine in the blood Sulfonyl Urea in Type II Diabetes Mellitus
 Episodes may be provoked by overnight fast in an  The sulfonylurea drugs (glyburide [glibenclamide]
Infant and tolbutamide), used in the treatment of type 2
208
Self Assessment and Review of Biochemistry

diabetes mellitus   
     '  .   Infantile Refsum disease (IRD)
inhib6+-!  P&     FP6+J
 Hence it reduces Gluconeogenesis, there by preven-
Lorenzo’s oil therapy
U  
 Treatment for Adrenoleuko Dystrophy
 "   ##   Lorenzo’s oil (4:1 mixture of glyceryltrioleate and
 Defects in long chain 3-hydroxyacyl-CoA dehy- glyceryltrierucate)


  * 

 
  

  % &  '
 
    { .   Y!' +; 
 /   Y' ;    
 ?? #  š620, C22  Till the double is reached, normal beta oxidation will
 Takes place in the Peroxisomes till Octanoyl CoA  
 Oxidation in peroxisome produces Acetyl CoA and  An additional Isomerase and a Reductase helps to
H2O2Q F 
?#U2) 
 .. 
 _#-+  -
  ?#   #  6#  !
 ?' 
#  6# {  6# .  
takes place in the mitochondria.  ?#U2
 
 +'    
     -   . ' 
‰ ? 
.  {  6# Acid is 1.5 ATP less per double bondQ
 +' {'       

6.  
 
   '
 
  
 +' 
6  -   
   B=    {'  
  ? #   ;    . 

carbon atoms yields Acetyl CoA and a molecule of
Clinical Correlation
Propionyl-CoA
Defects in the oxidation of VLCFA in the Peroxisomes  -  
 ! 
   
!  "#    

     
The peroxisomal diseases are genetically determined
disorders caused either by the failure to form or maintain
the peroxisome or by a defect in the function of a single
&    
The basic defect is
 The proteins that are destined to the peroxisomes
  V   '
V F+-*J
 
 +-* +' 
Peroxisomal ghost
 Absence or reduction in the number of peroxisome Fig. 7.15: Conversion of propionyl CoA to succinyl CoA
is pathognomonic for disorders of peroxisome
. +  
 "
',
  
  
 In most disorders there are membranous sacs that $%&    
contain peroxisomal integral membrane proteins,  *QB    +'
which lack the normal complement of matrix
 P
 .
X6.
›+'†ƒ

Some peroxisomal disorders are:  ?' 
/   ? #  
 Zellweger’s syndrome  . 
 Neonatal adrenoleukodystrophy (NALD)  _  
Metabolism of Lipids 209

 ? #      Ketone Body Synthesis


 ‰ 
' 
+    $   Site: Exclusively Liver MitochondriaQ
  
         Acetoacetyl CoA from Beta oxidation is the starting
Refsum’s disease 
 
 #' 
+  #  F+    HMG CoA Synthase 
Acid Oxidase) (phytanoyl CoA hydroxylase) in the  HMG CoA Synthase is common to Cholesterol
+' *  …/ * 
 The manifestation of classic Refsum’s Disease
includes impaired vision from retinitis pigmentosa, Steps of Ketone Body Synthesis
ichthyosis, peripheral neuropathy, ataxia, and,  -;   !6#  
   Y!' 
       condense to form acetoacetyl-CoA by a reversal of
 Classic Refsum disease often does not manifest the thiolase  
until young adulthood, but visual disturbances  Condensation of acetoacetyl-CoA with another
such as night blindness, ichthyosis, and peripheral molecule of acetyl-CoA by Hydroxy-3-
neuropathy may already be present in childhood methylglutaryl-CoA synthase forms 3-hydroxy-3-
     methylglutaryl-CoA (HMG-CoA).
 Restrict dietary dairy products and Green Leafy
 Hydroxy-3-methylglutaryl-CoA lyase then causes
Z.
  !6#[
U"=!6#
'%&    
  
 { B  P   Acetoacetate continually undergoes spontaneous
 {'  Š   .'    
 Involves Hydroxylation at the terminal methyl group  #      Y U ' .  . 
Š . 6 +~7’F"' 
 {' J     &  Y hydroxybutyrate
 Resulting in Short chain Dicarboxylic Acid (double dehydrogenase
  
   J
Quick Review—Sites of Oxidation of Fatty acids
Beta oxidation of fatty acid Mitochondria
Beta oxidation of unsaturated Mitochondria
fatty acids
Beta oxidation of very long chain Peroxisomes up to Octanoyl CoA,
fatty acid then rest in mitochondria
Alpha oxidation of fatty acid Peroxisomes, Smooth Endoplas
mic reticulum
Omega oxidation of fatty acid Smooth Endoplasmic Reticulum
Activation of fatty acid Cytosol

Ketone Bodies
Ketogenesis occur in metabolic conditions associated with


   ' 
 + …/ #  
 Secondary Ketone bodies are Acetone and Beta
U ' / 
 Concentration of Ketone Bodies in the blood does not Fig. 7.16: Ketone body synthesis
 '  ’4<
 In normal persons the ratio of beta hydroxy butyrate The Pathways where HMG CoA is an Intermediate are:
to acetoacetate is 1:1 
Ketone Body Synthesis
 In Ketosis the ratio of beta hydroxy butyrate to 
Cholesterol Synthesis
  Q 
'# 
   
210
Self Assessment and Review of Biochemistry

Energetics of Fatty acid oxidation if Ketone bodies are the end Quick Review — Ketone bodies
products 
7 
@ 
    

4;


.7

$ #

   



$ # 
0 
@ 
  &
+  #  

4


< #  



$ #  
8# 
@ 
  

@ 
 
 
#"
#" 

0 

  

@ 
 ' 
  

J"
 


#  1
9 
 ' &
>+
Utilization of Ketone Bodies 
> 
 "
 $

@ 
 
  %2-

0 
Ketone Bodies serve as a fuel for extrahepatic tissues. 
@ 
 




- 
 +  #  


 
Almost all the organs utilize Ketone bodies with the 
@ 
 




 
> 



exception of Liver and RBCsQ - 


 +  #  
< &…/ 
Steps of utilization of Ketone Bodies
   !  
  
 
  

 In extrahepatic tissues, acetoacetate is activated to Organ Fed Fasting Starvation

acetoacetyl-CoA by succinyl-CoA-acetoacetate CoA Brain Glucose Glucose Ketone Bodies


transferase or Thiophorase. Heart Fatty Acid Fatty Acid Ketone Bodies

 CoA is transferred from succinyl-CoA to form Liver Glucose Fatty Acid Amino Acid
acetoacetyl-CoA Muscle Glucose Fatty Acid Fatty Acid/Ketone Bodies

 The acetoacetyl-CoA is split into two acetyl-CoAs RBC Glucose Glucose Glucose
by thiolase
 #  6#' &       CLINICAL CORRELATION: FATTY LIVERQQQ
 #   ' & 
Lipid mainly as triacylglycerolQ can accumulate in the
'& 
  
 
*     # /*4) is the most
  ; ; 

Stages of progression of NAFLD


Nonalcoholic steatohepatitis (NASH), which can progress
to liver diseases including cirrhosis, hepatocarcinoma, and
liver failure.
6
? <Q
 -.  

 . .;
rate of Triacyl Glycerol synthesis and its export in

? <
; 
 Due to raised levels of plasma free fatty acids: The
production of VLDL does not keep pace with the
 ž'  


   
allowing triacylglycerol to accumulate, which in turn

 
This can be due to
 Starvation
Fig. 7.17: Utilization of ketone bodies  The feeding of high-fat diets
Test for Ketone Bodies in urine  In uncontrolled diabetes mellitus

- =
* 
 
. 

     Twin lamb disease

> =
8 $# 
. 

   


Acetone  …  



8+?8


 

+  #  

 Due to a metabolic block in the production of plasma
$  
9 
 

@   lipoproteins, thus allowing triacylglycerol to accumulate
Metabolism of Lipids 211

This may be due to: Cholesterol Synthesis


 A block in apolipoprotein synthesis as in Kwashiorkar Major Sites: All tissues containing nucleated cells are
 A block in the synthesis of the lipoprotein from lipid capable of Cholesterol Synthesis especially in Liver,
and apolipoprotein Adrenalcortex, Testes, Ovaries, Intestine
 A failure in provision of phospholipids that are found Orgenelle: Smooth Endoplasmic Reticulum and
in lipoproteins Cytoplasm
*"Q#  6#
 A failure in the secretory mechanism itself
 Orotic acid  
  Steps of Cholesterol Synthesis
 The antibiotic puromycin, ethionine, carbon Formation of HMG CoA
tetrachloride, chloroform, phosphorus, lead, and  Two molecules of Acetyl CoA (2C) condense to form
  
  #   6#F~6J. & -
 Lack of Lipotropic FactorsQ  Acetoacetyl CoA condense with a third molecule of
Acetyl CoA to form HMG CoA by the enzyme HMG
Lipotropic Factors are: 6#* 

Choline, Betaine

Vitamin E-#$$  
 
Synthesis of Mevalonate

Methionine in S Adenosyl Methionine,trap the available adenine  HMG CoA (6C) converted to Mevalonate (6C) by

$

  

.7 HMG CoA Reductase

Selenium  This is the rate limiting stepQQQ

 
 
 &
"
  
  - B   

Pyridoxine, and Pantothenic acid  _#+UV 
 Statins are competitive inhibitor of HMG CoA
Alcoholic Fatty Liver P  
    #alcoholic liver Generation of Isoprenoid Units (5C)
disease (ALD) which is caused by alcoholism and  Mevalonate on decarboxylation and phosphorylation
ultimately leads to cirrhosis. 
 
The fat accumulation in the liver is caused by a Condensation of 5 Carbon isoprenoid units to form
combination of impaired fatty acid oxidation and Squalene (30C)
    Two 5C unit condense to form 10C compound–
Oxidation of ethanol by alcohol dehydrogenase, = + 
Aldehyde Dehydrogenase leads to excess production  ’6= +   ;

_#U 76 
76  !? 
This results in increased NADH/NAD”- 
 Increased esterification of fatty acids to form  -;? F76J  

@’6  !*V
  
 
 Increased (lactate)/(pyruvate), resulting in hyperlac- Formation of Cholesterol
ticacidemia, which decreases excretion of uric acid,  Linear 30C molecule cyclises to form a structure that
aggravating gout.  .    <
 <  
    

6F\6J
CHOLESTEROL  The intermediates in the conversion of Lanosterol to
Cholesterol are
 Exclusive animal sterol never seen in plants
 14 Desmethyl Lanosterol
 "$ 
+.
 Zymosterol
 Made up of Steroid nucleus called ‘Cyclopentanop-
 Desmosterol
 F6++Jƒ
Lanosterol
 -.
 .6\ 
First Cyclical Compound formed
 #  
First Steroid Compound formed
212
Self Assessment and Review of Biochemistry

 Feedback regulation: Cholesterol repress the


 

U"=6#P  
 This acts via Sterol Regulatory element binding
F*PB/+J
 Hormonal Regulation: By Covalent modification-
+   
 HMG CoA Reductase is active in dephosphorylated
  
 Insulin and Thyroxine increase the activity of
HMG CoA Reductase
 Glucagon and Glucocorticoids decrease the
activity of HMG CoA Reductase
Fig. 7.18: Synthesis of cholesterol
Mechanism of regulation by Insulin
 = 
Compare–Cholesterol Synthesis ?U"=6#P  .   
and Ketone Body Synthesis & 
Ketone body By two mechanisms
Characteristics synthesis Cholesterol synthesis First method
Site Mitochondria Cytoplasm/Smooth En   ++
doplasmic reticulum
 -   #"+ …   #"+
HMG CoA as an Yes Yes …
intermediate  *U"=6#P   .  
 H e n c e H M G C o A R e d u c t a s e i s i n a c t i ve
HMG CoA Yes, the regulatory Yes   
Synthase step
Second Method
HMG CoA No Yes, the rate limiting step  Insulin converts HMG CoA Reductase directly to
Reductase     
HMG CoA Yes No
Glucagon and Glucocorticoids
Lyase
 Inhibit HMG CoA Reductase by phosphorylating
Remember & 

Cytoplasmic HMG CoA Synthase is for Cholesterol synthesis  /  #"+   ; .+

Mitochondrial HMG CoA Synthase is for Ketone body synthesis +
 *#"+… ;  U"=
Uses of Isoprenoid units in Farnesyl and 6#P  
Geranyl Diphosphate  U"=6#P   
 +     Dolichol and
Ubiquinone 
 
?  Key points Cholesterol regulation
 Prenylation 
+Q 
3  
 
 


  

$ "
%2-


> # 
1 
 +-"  

3

!!
"

 
 
#




 =-+/ +   $$  


!V
W'

#
 
 ?    
     
%2-

 # 

 

$$ 
  
. 
:# 

. 

 
  
 ? +-  
-#"

-#  
 
 
  

Regulation of Cholesterol Synthesis Tests for Cholesterol


Rate limiting Enzyme is HMG CoA Reductase.  <./  
 Feed back inhibition: HMG CoA Reductase is inhibited  *;ƒ-
. " 6    ƒP 
Metabolism of Lipids 213

Fig. 7.19: Regulation of cholesterol synthesis

Cholesterol cannot Generate Energy Steps of Bile Acid Synthesis



Unlike other biomolecules, cholesterol does not degrade to  Synthesis of Primary Bile Acids—In the Liver
" 
"
First step:
The Fates of Cholesterol

8

 

+ 
 
 6  \U  . \

0 
 



 

$  XU '  6 +~7’& 

$ 


$  $
 


  6¡+\#

Rest serve as precursors of Vitamin D and Sex hormones,  A typical monooxygenase requires oxygen,
Corticosteroids _#+U Z6
Specialized Products of Cholesterol  -Q

Bile Acids (Excretory form of Cholesterol)

Vitamin D

Sex Hormones

Corticosteroids

BILE ACID SYNTHESIS


Starting material-CholesterolQ

Bile Acids and its Site of Synthesis


Primary Bile acids—Liver
They are:
 Cholic Acid( Most abundant bile acids in mammals)
 Chenodeoxycholic Acid or Chenic acid

Secondary Bile acids-Intestine


They are:
 Deoxycholic Acid
 Lithocholic Acid Fig. 7.20: Bile acid synthesis
214
Self Assessment and Review of Biochemistry

Further multiple steps: LIPOPROTEINS (VERY IMPORTANT)


 \         ;
subpathways leading to synthesis of Cholic acid Denition
 6 ' 6     Lipoproteins are compound lipids formed as a
 6$
+ /#   .
 ;
 + .   $ ;=   -
  
-  U 
 6$ '
Structure of lipoprotein
 In humans the ratio of Glycine to Taurine
 Lipoproteins consist of a nonpolar core and a single
$@Q
surface layer of amphipathic lipids
With glycine With taurine  The nonpolar lipid core consists of mainly
Glycocholic Acid Taurocholic acid triacylglycerolQ and cholesteryl esterQ
Glycochenodeoxycholic acid .#  
   It is surrounded by a single surface layer of
     
Bile Salts

:
9 
 
$%
[;

\](
##
^#" 
 
 
  

 These are oriented so that their polar groups face


as sodium and Potassium salts outward to the aqueous medium, as in the cell

0



 
  .

+ 
 
 
 
 
#"
 
Major Classes of Lipoproteins
 Synthesis of Secondary Bile Acids—In the Intestine Based on ultracentrifugation, in the ascending order
Intestinal bacterial enzymes deconjugate and of density is
dehydroxylate primary bile acids to form  Chylomicrons (Least Density)
   .  
 Very Low Density Lipoproteins (VLDL)
 Cholic acid to Deoxycholic acid
 Low Density Lipoproteins (LDL)
 Chenodeoxycholic acid to Lithocholic acid
 Intermediate density lipoproteins (IDL)
 Enterohepatic Circulation
 High Density Lipoproteins (HDL)
 +  *   .  .. 
Based on electrophoretic separation
'  
 98–99% of absorbed bile acids are returned to From cathode to anode the order of Lipoprotein in an
the liver via portal circulation (Enterohepatic electrophoretogram is (See Fig. 7.21)
6 J  Chylomicron
 Lithocholic acid is the bile acid which undergo  <<FY<J
     Z<<F+Y<J
 <F/ Y<J
Regulation of Bile Acid Synthesis
 U<FX<J
 The principal rate-limiting step in the biosynthesis

 .        \X!  ' 
(CYP7A1) reaction.
 The activity of the enzyme is feedback regulated via
the nuclear bile acid-binding receptor, farnesoid X
receptor (FXR).
 When the size of the bile acid pool in the enterohepatic
    ?¢P      
 
 \X! ' 
 
 Chenodeoxycholic acid is particularly important in
 ?¢P
 6\! '     
.  
     Fig. 7.21: Structure of lipoprotein
Metabolism of Lipids 215

S # #   #  Lipoproteins and its function


on the protein content
Lipoprotein Function
 Higher the protein content faster the mobility of
    Chylomicron * 


:  
Carry dietary Triacylglycerol (Exogenous TAGs)
 ChylomicronQ with least protein content remains 

 
at the origin and HDL with highest protein content
VLDL * 


 

 Carry endogenous Triacylglycerol
 An exception is VLDL and IDL with less protein
IDL * 

_'3'
 
<< (VLDL Remnant) '3'

 

:3'

LDL 3 

_'3'
  
Deliver Cholesterol and cholesterol ester to
 $ 
#


 

HDL * 


 

  
Deliver cholesterol from peripheral tissues to liver

 
  " 
#Q

Apolipoproteins
 The protein part of Lipoprotein is apolipoprotein
Fig. 7.22: Electrophoretic separation of lipoproteins .. 
 - F/J;  .
removed to other lipoprotein or peripheral proteins
F6 BJ
 -$U<#
 The major apolipoprotein in LDL and VLDL is apo
/’’
Fig. 7.23: Size of lipoproteins  Chylomicron contain a truncated apolipoprotein
/~“

Lipoproteins—Composition and Apolipoproteins Apolipoproteins and its Function


Apolipoprotein Function
Composition
Protein Lipid $
: Activates Lecithin Cholesterol Acyl Transferase
(LCAT)
Lipoprotein Apolipoproteins (%) (%)
Chylomicron $
+]\ `4 j\`jj $
:: Inhibits Lipoprotein Lipase
$
:&::&::: $
_ Promote lipoprotein lipase mediated Triacyl
apo E -
 $ 
$
:&::&:_ $
+!!  

_'3'


 
VLDL $
+!! [`! j!`jV Act as ligand for the LDL receptor and LDL re
$
:&::&::: $ 
 
$  
'>7(

#$ 9

'3'
apo E $
+]\  

 


  
IDL $
+!!  \j $
: Inhibit Cholesterol Ester Transfer Protein (CETP)
(VLDL Remnant) apo E
$
:: Activates lipoprotein Lipase
LDL $
+!! 4 79
$
::: Inhibit Lipoprotein Lipase
HDL $
:&::&:_
V4`6[ ]V`;\
$
:&::&::: Apo E Act as ligand for LDL receptor for uptake of
apo D & E Chylomicron remnant and VLDL remnant (IDL)
216
Self Assessment and Review of Biochemistry

More about Apolipoproteins HDL


$


 

"    Alpha lipoprotein
$
3

  
 
# 
# " 
  Least Diameter
Isoforms of apo E
 Maximum Electrophoretic Mobility

$

"

$ $ 

|#

. 
#

$ 


 
 
 "'+6

$
4&
$
V&
$
]  Carry Cholesterol from peripheral tissues to liver and

$V


  other steroidogenic tissues

:  #


$
]


$

$
 This is called Reverse Cholesterol transport
1 
  This makes HDL Cholesterol ‘the good cholesterol’

$
4



 

'3'$ 
 The major role of HDL is to acts as the repository

:  #
 "

$
4


$

$
$
:::

hyperlipoproteinemia (Familial Dysbeta Lipoproteinemia) for apo C and apo E required for the metabolism of
Z<< 6  
Lipoproteins at a Glance Lp (a)
Chylomicron  #<<
 "'  #FJ   /’’.   . 
 <   "$
  
<FJ
 ".   Strongly associated with Atherosclerosis and
"   
 
 <+ 
 *  ;+Q
 "'-  
  
 ;   
 + 
 Least Electrophoretic Mobility [Remain at the point 

 ^  U .   
 <

  * . .
 Carry exogenous (dietary Triacylglycerol from
+J LpxNBE patternQ
 Carry dietary cholesterol and cholesterol ester into  6'  .  .
     .;+ 
 
<¢
VLDL  U  '
 
 +.
 Carry endogenous TAG from Liver to peripheral LIPOPROTEIN METABOLISM
Tissues Chylomicron Metabolism
IDL Step I—Formation of Nascent Chylomicron
 Otherwise called VLDL remnant Assembly of nascent chylomicron in the intestine
 .   
 Broad Beta Lipoprotein
Step II—Formation of Mature Chylomicron
LDL Remodelled to mature chylomicron by receiving apo C-II
 /<  B
U<
 "'6 6 Remember
 \’£ 
 <<         @’£ 
 << %3'
 


$ 

$


$

  '  Step III—Formation of Remnant Chylomicron
 << .
  Apo C-II activates Lipoprotein Lipase
 The degradation of LDL in extra hepatic tissue  Lipoprotein lipase that is located on the walls of
is responsible for deposition of cholesterol and blood capillaries, anchored to the endothelium by
'  negatively charged proteoglycan chains of Heparan
 -<< †.  ƒ *

Metabolism of Lipids 217

 Lipoprotein Lipase hydrolyses TAG in mature VLDL Metabolism


chylomicron to fatty acid and glycerol, to form Step I—Formation of Nascent VLDL
6    Assembly of nascent VLDL from the liver which carry
 ?     
  -  
 -6  
  Step II—Formation of Mature VLDL
Step IV-Uptake of Remnant Chylomicron.  Nascent VLDL remodelled to mature VLDL by
 6! B
U<
 Chylomicron remnant is taken up in the liver by
      Step III—Formation of Remnant VLDL (IDL)
 Uptake is mediated by apo E via two apo E dependent  apo C-II activates Lipoprotein Lipase
recptors,LDL receptor and LDL receptor related  Lipoprotein lipase that is present in the walls of
!F<P+!J the capillaries that lines the extrahepatic tissues
  -#=? #   = 
 Hepatic Lipase hydrolyse remnant triacylglycerol
 ?     

  
 -Z<<F<J
 
Step IV—FATES OF IDL
 VLDL remnant (IDL) is taken up in the liver by apo
BP 
 Z<<
 << -
  <   ; Z<<
Z<<F<J<<

Fig. 7.26:
' $$  
 
$ 

Step V—Uptake of LDL by tissues


 LDL is metabolized by LDL receptor via receptor
    
 apo B100Q   
<< 
 Approximately 30% of LDL is degraded in
Fig. 7.24: Metabolism of chylomicron '  \’£
LDL Receptor

'3'
>$ 


-$  

' "

'3'
$ 

$
+!!

$


Present in hepatic and extrahepatic tissuesQ

Occur on the cell surface in the pits coated on the cytosolic side



 
 

$  

Clathrin.

LDL is taken intact by Receptor mediated Endocytosis.

 
#$ 9

'3'&

$ 





#

.
}#

 
 


suppress the synthesis
of LDL receptor 
0>+7
 
"# 
    "

$  (
$ 

 
 "


PUFA and MUFA is thought to be
due to upregulation of LDL-receptor, that increases the catabolic
 

 
 
'3'

HDL Metabolism and Reverse


Cholesterol Transport
 Nascent HDL is synthesized and secreted from
Fig. 7.25: Metabolism of VLDL     _  U<     
218
Self Assessment and Review of Biochemistry

shape, consist of phospholipid bilayer, Cholesterol Enzymes Responsible Reverse


 # Cholesterol Transport
 Lecithin Cholesterol Acyl Transferase (LCAT) binds
Lecithin Cholesterol Acyl Transferase (LCAT)
 U<
 ## <6#-
 ##! <6#-
 *  ;U<
 LCAT converts Cholesterol to nonpolar cholesterol
  Responsible for virtually all plasma Cholesteryl esters

Lecithin Cholesterol Acyl
Cholesterol Transferase (LCAT) Cholesterol Ester  B
 U<
+ +
Lecithin Lysolecithin
 Cholesterol ester being nonpolar moves into the core

U<
 6* 
generated forming a spherical HDL (HDL3)with sur-  Thus create a concentration gradient and draws

 
    

 HDL3 accepts Cholesterol from tissues by Class B  Thus enabling HDL to function in reverse cholesterol
*  P  /! F*P!/J   #-+!.  
 #F#/6#J =!F#/6=!J Cholesterol Ester Transfer Protein
 LCAT acts on the Cholesterol in HDL3, convert it into  *  ;U<
   ? 

  
U<
 Thus less dense HDL2
  other lipoproteins,VLDL, IDL, LDL in exchange of
 HDL2 delivers Cholesterol and cholesterol ester -  
to liver via SR-B1 receptor or transport it to  This relieves the product inhibition by cholesteryl
steroidogenic tissues or acted upon by hepatic lipase <6#-
 <  6  ; . <<<
 Thus HDL3
  
#! 
 ?#!
   +YU< Receptors Responsible for Reverse
 HDL3 +/U<    Cholesterol Transport
¤'
 Class B scavenger receptor B1 (SR-B1)
 Thus $%  ##  & #  #  "  '  6  U< 
and transport it to liver and steroidogenic tissuesQ  ##!  
 
(&  ## )  & #  #    SR-BI has dual role in HDL metabolism
 In liver and steroidogenic tissues it helps in the
delivery of cholesterol and cholesterol ester to the

U<
 In liver excess cholesterol is converted to bile
   '  
 In steroidogenic tissues, cholesterol is used for
 
 
 Whereas from peripheral organs SR-BI facilitates
¤'U< 
 ;<%>  # # #
 #-+!.    # F#/6#J
  #-+!.    =
F#/6=J
 +   '   
 
¤'
 

 ABCAI preferentially transport cholesterol to
   +YU<#
Fig. 7.27: Metabolism of HDL  #/6= U<
Metabolism of Lipids 219

HDL fractions Cholesterol Balance in the Tissues


Nascent HDL or Discoidal HDL The factors that increase the Cholesterol in the tissues

Contain phospholipid bilayer, cholesterol and apo AI  Uptake of cholesterol-containing lipoproteins by

Spherical HDL or HDLV  

Contain cholesterol, cholestetryl ester, phospholipid, Apo A I, LCAT
 6 
Spherical less dense HDL or HDL2
 Hydrolysis of cholesteryl esters by the enzyme

Contain cholesterol, cholesteryl ester, phospholipid, Apo AI
  
Prebeta HDL

Most potent HDL

}#
of cholesterol from tissues
The factors that decrease the Cholesterol in the tissues

Contain apo AI, cholesterol and phospho $   B¤'
 
.U<

ABCAI preferentially transfer cholesterol to Prebeta H3' #/6##/6=*P!/
HDL Cycle  B  
  .  #6#- F !6#Q

The interchange of HDL2 and H3'V is called HDL
  
J
 Utilization of cholesterol for synthesis of other
  .  
Transport of Cholesterol between Tissues
     6   DISORDERS OF LIPOPROTEIN METABOLISM
 95% of Chylomicron cholesterol is delivered to liver (DYSLIPOPROTEINEMIAS)
6  
 -   
Z<< Hyperlipoproteinemias
 Most of the cholesterol secreted in VLDL is retained  . 

  0   1

<  << ?   <    


 LDL cholesterol is taken up by liver and extrahepatic according to the lipoprotein particles that accumulate in
 . 

Frederick- Genetic Trans- Estimated


Nomenclature Molecular Defect
son Type mission Incidence
Type I Familial Chylomicronemia Syndrome (FCS) Lipoprotein Lipase or Apo CII AR W!!!!!!
Type IIa Familial Hypercholesterolemia (FH) LDL receptor AD W46!W6!!
(Most common)
Familial Defective Apo B (FDB) Autosomal Dominant $
+!! AD €W6!!
Hypercholesterolemia Type II (ADH Type II)
Autosomal Dominant Hypercholesterolemia Type III PCS K9 AD €W!!!!!!
(ADH Type III)
Autosomal Recessive Hypercholesterolemia (ARH) LDL Receptor Adapter Protein AR €W!!!!!!
(LDLRAP)
Sitosterolemia +-6

+%\ AR €W!!!!!!
Type II b Familial Combined Hyperlipidemia (FCHL)  AR €W!!!!!!
Type III Familial Dysbeta lipoproteinemia (FDBL) Apo E AR W!!!!
Type IV Familial Hypertriglyceridemia (FHTG) $
_ AR €W!!!!!!
Type V Familial Hypertriglyceridemia (FHTG) $
_

-7:%+7
 AR €W!!!!!!

4#  "


"    # ?#  ## 
 
Phenotype I IIa IIb III IV V
Lipoprotein, elevated Chylomicrons LDL LDL and VLDL Chylomicron VLDL Chylomicrons
(predominant) and VLDL and VLDL
VLDL remnants
Triglycerides  N    
Contd...
220
Self Assessment and Review of Biochemistry

Contd...
Phenotype I IIa IIb III IV V
Cholesterol (total)     N/ 
'3'       
%3'   N/  N  
Plasma appearance Lactescent Clear Clear Turbid Turbid Lactescent
Xanthomas Eruptive Tendon, None Palmar, None Eruptive
tuberous tuberoeruptive
Pancreatitis +++ ! ! ! ! +++
Coronary atherosclerosis ! +++ +++ +++ +/– +/–
Peripheral atherosclerosis ! + + ++ +/– +/–

Primary Hyperlipoproteinemias Causing Molecular sequencing of gene


Hypertriglyceridemia New advances in treatment of Familial Chylomicronemia
Familial Chylomicronemia Syndrome (Type I Hyper- Syndrome
lipoproteinemia) A gene therapy approach-Alipogene tiparvovec

2# $
  ##
^ 


 
  
 
 

Biochemical abnormalities encoding again of function LPL variant, leading to skeletal myocyte
 <<F<+<J#6 
  $ 

'7'
 Lipoprotein Lipase is required for hydrolysis of TGs 
$$

#$
6   Z<<#6 
 

<< Familial Hypertriglyceridemia (FHTG) (Type IV and V
 Lipoprotein accumulated is Chylomicron and VLDL, Hyperlipoproteinemia)
.     Biochemical abnormalities
 ?-  š’’’4 < %@ 
 ?6    Apo A-V facilitate association of VLDL and
  ;<+<
Clinical presentation
 +   ; .   Loss of function mutation of Apo A-V causes
due to acute pancreatitis  
6   Z<<
 On fundoscopic examination opalescent retinal blood J<Z\<%Z 
vessels (lipemia retinalis)  =+U/+!=   + U<
 Lactescent plasma / +!
 Eruptive xanthoma (small yellowish white papules  "=+U/+!  
 

appear in clusters on backs, buttocks, extensor <+<Z  

  
     -     6   
lesions may become pruritic)  -     
 Hepatosplenomegaly
 +6U

?6* Primary Hyperlipoproteinemias Causing
Hypercholesterolemia
Diagnosis
Familial Hypercholesterolemia (FH)
Assaying triglyceride lipolytic activity in post heparin
plasma (IV heparin injection to release the endothelial- Also known as Autosomal Dominant Hypercholester-
bound LPL) olemia Type I (ADH Type I)
Observation Biochemical abnormalities
 <+<  
     .<+<   <

 << 
6!    Can be
 ;6!   &
  Homozygous or receptor negative called as
the addition of normal plasma (providing a source & ?U

6!J  U&   
 
Metabolism of Lipids 221

 P     
<<
    +6*…•        .    <<
 As LDL receptor is responsible for clearance of IDL, receptor and is redirected to the lysosome and
<<  
<     
 So lipoprotein accumulated is LDL and lipid  So in ADH 3 there is accelerated degradation of LDL
  6  
 LDL-Cholesterol >400-1000 mg/dL in homozygous  *<<  
  6 ?U
 -    Autosomal Recessive Hypercholesterolemia (ARH)
Clinical presentation Biochemical abnormalities
 ?  
6U  Due to mutations in a protein LDL Receptor adaptor
 Corneal arcus  <<P#+    <<  ¥
    
 + 
 . 
<<P#+<<. <<
 No pancreatitis
receptor but the lipoprotein-receptor complex fails
 Tendon xanthomas particularly dorsum of hands and .& 
Achilles tendon  6 #PU?U
   
    
Sitosterolemia
Diagnosis Biochemical abnormalities
 _    " #-+!.   F#/6J 

 LDL receptor assay transporter family, ABCG5 and ABCG8, expressed
 LDL receptor gene sequencing in enterocytes and hepatocytes which pumps plant
Recent advances in the treatment of Homozygous Familial Hy- sterols such as sitosterol and campesterol, and animal
percholesterolemia (FH) sterols, predominantly cholesterol, into the gut lumen
.
$
#"
$
  .

Lomitapide: Small molecule inhibitor of microsomal triglyceride  Hence intestinal absorption of sterols is increased and

$  &
 


$ # 

_'3'.

'3'
$ # 

_'3'
biliary excretion of the sterols is reduced, resulting
4
Mipomersen:  
 "# 

$
+ in increased plasma and tissue levels of both plant
  
Familial Defective apo B100 (FDB)  Increase in hepatic sterol level results in transcriptional
Also known as Autosomal Dominant Hypercholestero- suppression of the expression of LDL receptors, which
lemia Type II (ADH Type II) results in reduced uptake of LDL and substantially
Biochemical abnormalities   <<!6
 "  /’’    Clinical presentation
<< ¥.  
/!’’  ‰
?U  
Clinical presentation like tendon xanthoma, premature atherosclerotic
   
 #& ?U
 Anisocytosis, poikilocytosis of erythrocytes,
 +
<<!6;& 
megathrombocytes due to incorporation of plant
?U
sterols into cell membrane
 This is because, IDL clearance is not impaired unlike  Episodes of hemolysis and splenomegaly are
?U/’’.  
<<  distinctive features
 [  
 Severe hypercholesterolemia not responding to statins
Diagnosis .     &.
 *V  . 
/ Diagnosis
Autosomal Dominant Hypercholesterolemia Type III  *. 
(ADH Type III)
Treatment
Biochemical abnormalities  Bile acid sequestrants
 =

 
+6*…•  6..
222
Self Assessment and Review of Biochemistry

Primary Hyperlipoproteinemias Causing Both Hypolipoproteinemias


Hypertriglyceridemia and Hypercholesterolemia Abetalipoproteinemia
Type III Hyperlipoproteinemia Autosomal recessive disease
Familial Dysbetalipoproteinemia (FDBL) Or Familial Biochemical defect
Broad Beta Disease or Remnant Removal Disease  Loss-of-function mutations in the gene encoding
Autosomal Recessive    
F"-+J
"--+
Biochemical abnormalities  "-+ 
          
 Due to genetic variations in apoE, especially apo Z<<   
E2 that interfere with its ability to bind lipoprotein  So Chylomicron Q, VLDL Q and hence LDL Q not
     
 B  +  
       
' ;  
Apo E gene Polymorphism
 Chylomicrons, VLDLs, LDLs, and apo B are
 APOE gene is polymorphic in sequence, resulting in
  .
the expression of three common isoforms
Clinical presentation
 apoE3, which is the most common; and apoE2 and
 +    ;  

apoE4
to thrive due to fat malabsorption
 #B   ;  
  <<  ›
 Neurologic manifestations—loss of deep-tendon
therefore, chylomicron and VLDL remnants ž'
; .     ;' 
containing apoE2 are removed from plasma at a vibratory and proprioceptive sense, dysmetria,
slower rate ataxia, and the development of a spastic gait
 Individuals who are homozygous for the E2 allele  +  
(the E2/E2 genotype) comprise the most common  Acanthocytes
.
;?/<  "      B       
 +;B~     
        #   … F
!#&ƒ  to defective fat absorption and hence the fat soluble
Clinical presentation JQ
Treatment consists of a low-fat, high-caloric, vitamin-
 +;?/<   ;
enriched diet accompanied by large supplemental doses
    

B
 +    

 

   Q


Most clinical manifestations of abetalipoproteinemia result from
 Lipoprotein elevated is Chylomicron and VLDL  


$ 

$

 #
  
 
Vitamin E and retinyl esters are normally transported from
  


 

 
 <  6 -   
Vitamin E is dependent on VLDL for transport out of the liver and
Xanthomas in FDBL  

 # 

.
  
$

  &
##$ 

$ &

As apo B containing lipoproteins are not formed these patients,



*3+'
$   there is, marked deficiency of Vitamin E, mild to moderate
 

_  


@

Tuberoeruptive xanthomas begin as clusters of small papules


&
9&

# 9


"


 1


Bleeding manifestation due to defective absorption of fat and hence

#
  
 
"$

Palmar xanthomas (alternatively called xanthomata striata
$  (

"
 






Tangier Disease
$ 

   Autosomal codominant

+ 

  

  #
$ "  

*3+' Biochemical defect
 Mutations in the gene encoding ABCA1, a cellular
Diagnosis
transporter that facilitates efflux of unesterified
 Very high level of remnant lipoprotein   
 #!
 Lipoprotein electrophoresis-Broad beta band  In the absence of ABCA1, the nascent HDL, poorly
 #B    
Metabolism of Lipids 223

 ##!    


   Contd...
 Extremely low circulating plasma levels of HDL-C Cause Biochemical defect
 #! Obesity The increase in adipocyte mass and accompanying
and Insulin 
# 
  
  
 
 

 Cholesterol accumulates in the reticuloendothelial resistance 


# $
 

 $
   
  
More free fatty acids are delivered from the
$ 
 $
#


 
Clinical presentation 
The increased insulin levels promote fatty acid
 Hepatosplenomegaly   


 
 B   ;F+- 
Increased Insulin resistance causes reduced
'7'
  
 
#
 #



nomonic Clinical feature)  



_'3'
 #  F . 
# 

$  "  
multiplex) Nephrotic 
 
#

 
_'3'
$ # 
Syndrome 

#
#9
#
  # 

$
 +..   ;   
+
buminemia leading to increased hepatic protein
atherosclerotic disease, although association is not   
.'   
Increased VLDL production, hence increased LDL

_'3'
&# !   Cushings Exogenous or endogenous glucocorticoids lead to
-; 

<6#-    Syndrome 
_'3'
$ # 

   /# ## #  %*( +
) Secondary causes of reduced hepatic uptake of
or Norum’s Disease lipoproteins
6 +
Cause Biochemical defects
 +      Hypothyroidism 
Thyroid hormone increases the hepatic


   $ 

'3'
$ 
 Hemolytic anemia 
So hypothyroidism reduction in hepatic LDL
$ &
 #


'3'
 +B*P

 
'3'
Biochemical abnormalities Chronic Kidney 
# # 

_'3'&

 
 $
disease $  


$  "  
 Very low plasma levels of HDL-C

Due to reduced TG lipolysis and remnant
 Rise in free cholesterol  $$  

 Rise in lecithin
Liver disorders and secondary hyperlipoproteinemias
 ?<  6U< 
Liver is the principal site of formation and clearance of lipoproteins
< #  4;  /  %  ^ 
0
 



$ 
 $



  


Hepatitis due to infection, drugs, or alcohol
Same features as classic LCAT deficiency, but no 
  
 

_'3'
  



  

Hemolytic Anemia and do not progress to ESRD $  "  


 
Severe hepatitis and liver failure
 # +<6#-   
  
 
  
 # 

$ 
 


 " 
#

 #
 $$  
   
$ 
 *V <6#- Cholestasis

  
 
$  &
 




Secondary Hyperlipoproteinemia 

^
$ 

 
 

 


 

Secondary causes of increased VLDL production is via secretion into bile, either directly or after conversion to bile
 &

  
9
 
 
 
$ 
Cause Biochemical defect
In cholestasis, free cholesterol, coupled with phospholipids, is
High Car Excess dietary carbohydrate is converted to fatty acid secreted into the plasma as a constituent of a lamellar particle
bohydrate 

 &
  


. 
"&
 
called LP-X.
diet from liver as VLDL
Alcohol The most common effect of alcohol is to increase Estrogen and Secondary Hyperlipoproteinemia
$ 
 " 
 
 "
    

  
 

_'3'



Alcohol consumption inhibit the hepatic oxidation %3'
  


 
 &
 

$  
hepatic 
Resulting in elevated plasma levels of both triglycerides and
triglyceride synthesis and VLDL secretion %3'

This lipoprotein pattern is distinctive since the levels of plasma
Contd...  " 

%3'

$ 

 
224
Self Assessment and Review of Biochemistry

Calculation of Lipid Fractions Cholesterol Absorption Inhibitors



:
 "& 

 
$  


 $$   
 Ezetimibe
VLDL, LDL, and HDL
– Total Cholesterol = HDL Cholesterol + VLDL Cholesterol + LDL
Cholesterol absorption inhibitor that binds directly to
Cholesterol  ._+6< . .

_'3'
  
 


#
 " 

 "


 
 
– 0
_'3'
 
5
. " W6 Bile Acid Sequestrants (Resins)

. 
 #
 
9

 " 

€
]!!
"W '

*   
 #

'3' 

#

"W '(Q  Cholestyramine, Colestipol, colesevelam

'3'
 
5
. 
 %3'
 ol
 Bile acid sequestrants bind bile acids in the intestine
Triglycerides
– and promote their excretion rather than reabsorption
6

 To maintain the bile acid pool size, the liver diverts
PHARMACOLOGIC TREATMENT OF .   
LIPOPROTEIN DISORDERS
 The decreased hepatic intracellular cholesterol
For severe Hypertriglyceridemia content results in upregulation of the LDL receptor
and enhanced LDL clearance from the plasma
Fibric Acid Derivatives (Fibrates)
?.    
++#P  LDL Apharesis
  
 . 
Treatment of hypercholesterolemia
 * <+<    F     
Patients blood is passed through a column that selectively removes

'3'
hydrolysis)
Specialized drugs for homozygous Familial Hypercholesterol-
 Reduce apoC-III synthesis (enhancing lipoprotein emia.
remnant clearance) 
2.7
  ' $ 
 +.!' 

    
$
+
  2 $ 
 P  Z<<    
ATP III Guidelines for Ideal Level of
Fibrates are the most effective drugs available for
LDL, HDL and Cholesterol
reducing triglyceride levels and also raise HDL-C levels
modestly Biochemical parameter Values mg/dL Risk
 =. LDL- €!! Optimum
Cholesterol
 ?. !!`4j Near or above
Optimum
1
3 
   4' 5 V!`6j Borderline high
-;   Š!@+‰?#

 ;!`\j High
hyperlipidemias ƒ
j! Very High
 B    FB+#J Total-Cholesterol €
4!! Desirable
  '   FU#J 4!!`4Vj Borderline High
ƒ
4]! High
Nicotinic Acid (Niacin)
HDL- €
]! '
Niacin suppress lipolysis in the adipocytes due to its
Cholesterol ƒ
;! High
[ =+P’•#    
   . Predictors of Coronary Artery Diseases

hs CRP
For Hypercholesterolemia 
Total Cholesterol/HDL ratio
HMG-CoA Reductase Inhibitors (Statins) 
Apo B/Apo A ratio
HMG-CoA reductase is a key enzyme in cholesterol 
LDL

Non HDL Cholesterols
biosynthesis, and inhibition of this enzyme decreases

%3' 
 
Metabolism of Lipids 225

REVIEW QUESTIONS
TAG Synthesis c #  6#6.' 
1. True about ac.etyl CoA: (PGI Nov 2011)  …# * 
 + 
 
   Ans. c. Acetyl CoA Carboxylase (Ref: Harper 30/e p234)
steroids
      /^!" 


. ?…. 
 *
 

    The complex is a homodimer of two identical
 #
=   polypeptide monomers in which six enzyme
Ans. a, b, c, d. (Ref: Harper 30/e p226, 273, 234) activities and the acyl carrier protein (ACP)
Fates of Acetyl CoA  #6+    Pantothenic acid in the
 * 
6   form of 4’-phosphopantetheine
 * 
?     X-ray crystallography of the three-dimensional
 * 
…/  structure, shown that the complex is arranged in an
 B-6#6  X shape
2. Regarding synthesis of triacylglycerol in adipose  #  6# .' ?#*6'Q
tissue, all of the following are true except: (AI 07)
5. Mitochondria is involved in A/E: (AI 2012)
 * 
 '  
. B&  =        ? #  * 
role . _#* 
 B& = @    ? #  {' 
plays an important role  +* 
 +   & Ans. a. ? #  *  (Ref: Harper 30/e p233)
Ans. b. Enzyme Glycerol kinase plays an important role Mitochondria is involved in mitochondrial DNA
In Muscle and Adipose Tissue *  +* F  . " 
 Glycerol Kinase is absent in muscle and white adipose   J

*     /4^
 =  @ + 
 
  ' 
# + =    "$ .     .  ?  < 
3. The storage Triacylglycerol are hydrolyzed by: hence the pathway is otherwise called Lynen’s Spiral
(JIPMER 2012)  Site: Liver, kidney, brain, lung, lactating mammary
# +  < gland, and adipose tissue
/ <<  Organelle: By an extra mitochondrial system [in the
6 < < cytosol]
 U< Cofactor requirements include
Ans. d. Hormone sensitive lipase (Ref: Harper 30/e p263)  _#+U#-+"2+, Biotin, and HCO (as a source
3–
 +    &  -= of CO2)
 Lipoprotein lipase to hydrolyze TGs in lipoprotein  #  !6# . . 
? 
in the blood
#  
 Lysosomal hydrolase to act on TGs in lysosomes
 Hormone sensitive lipase hydrolyze stored TGs in  €{        
   }
  enzymes except: (Kerala 2010)
Fatty acid Synthesis  B   
 `{ |  }   #     . …   
Synthase Complex? (AIIMS Nov 2013)  #  6# .' 
 … P     
. B P   Ans. c. Acetyl CoA Carboxylase (Ref: Harper 30/e p234)
226
Self Assessment and Review of Biochemistry

Six enzym        


 #  Reduction
 Ketoacyl Synthase  P
?  
 Malonyl-Acetyl Transacylase Condensation Reactions
 Hydratase  Malonyl Acetyl Transacylase
 Enoyl Reductase
 A priming molecule of acetyl-CoA combines with
 Ketoacyl Reductase  €*U
 -F J  While malonyl-CoA combines with the adjacent
7. NADPH is required for: (AI 1998) €*U   ~ƒ! 
#6+ 

 =  
. =    These reactions are catalyzed by malonyl acetyl
transacylase, to form acetyl (acyl)-malonyl
 ?    
enzyme
 =  
 Ketoacyl Synthase
Ans. c.?     (Ref: Harper 30/e p235)
 -           
 { ;#       ‚ of the malonyl residue, catalyzed by 3-ketoacyl
(AI 1996) synthase, and liberates CO2, forming 3-ketoacyl
 #  6# .'  enzyme (acetoacetyl enzyme) (reaction 2), freeing
. ©¥U ' 6#    €*U
 #     Reduction reactions
 +   Ketoacyl Reductase
Ans. a. Acetyl CoA Carboxylase (Ref: Harper 30/e p235)  The 3-ketoacyl group is reduced to Hydroxyacyl
      group by Ketoacyl Reductase
By two enzyme system  Hydratase
 Acetyl CoA Carboxylase  Hydroxyacyl group is dehydrated to Unsaturated
 ? #  * 6' Acyl (Enoyl) group by Hydratase
 Enoyl Reductase
Acetyl CoA Carboxylase
 Unsaturated Acyl (Enoyl) group is reduced to
 Converts Acetyl CoA (2C) to Malonyl CoA in the # . B P  
 
#-+
 Acetyl-CoA carboxylase has a requirement for the B     
. These reactions of Condensation and reduction repeated
 &  several times till the desired acyl group is assembled on
 This is the rate limiting stepQ? #  * - & 
 Thioesterase
 #  6# .' ?#*6'Q  ?   F# J. 
& 
complex by the activity of the sixth enzyme in the
 ƒ{ Z        &2 loss occurs in which
'F  J
step: (PGI Dec 2006)
 U  †‡{  #> 
      ‚
.   (AIIMS Nov 91)
 6      ~  
 P   . ~    
Ans. c. Condensation reaction (Ref: Harper 30/e p235)      
       
  ~  ~  
By three stages Ans. b. 4 in Ist cycle and 2 in IInd cycle
 Condensation (Ref: Harper 30/e p235)
Metabolism of Lipids 227

 ? 6  
 ?#* 6'  6
#   6# 14. Acetyl CoA acts as a substrate for all the enzymes
condenses with 2 Carbon atom of Malonyl CoA, by except: (AIIMS May 03)
liberating 1 CO2*~6.    U"=!6# 
    '~6.
   . " & 
 6.  . … *   " 6# 
11. True about mitochondrial chain elongation of  ?    
   ˆ #‚ /<JZ‰"†ƒƒƒ^ Ans. b. Malic enzyme 

    
 { .     "  B&   "  +  . 
. {.  liberating CO2
 6;
15. Acetyl CoA Carboxylase is activated by:
 _ ; /*\Š #Ž^
 +  '!+ _#+UV   " 6#
Ans. b. Operates aerobically, d. Not a common pathway . 6
Š       + 6#
 Occurs in the Endoplasmic Reticulum (the  #  
†  ƒJ    Ans. b. Citrate (Ref: Harper 30/e p237)
 / ? #  B  Allosteric regulation of Acetyl CoA carboxylase
 B   
  !6#  +# # # 
(from C10 upward) by two carbons of Acetyl CoA Carboxylase by CitrateQ
 Malonyl CoA donates 2 Carbon atoms in stepwise  Palmitoyl CoA is an inhibitor of Acetyl CoA Car-
manner boxylase
 ?   * 6'
Oxidation of Fatty Acid and Disorders
in the Cytosol
 _#+UV ;   16. Number of ATP formed by oxidation of one
molecule of palmitic acid (16 c): (Kerala 2009)
 Elongation reaction are particularly increased in
 ~
brain during myelination to provide C22 and C24
   #{/  

^ . ’


 @7
†Œ{ <*%            
  @~
accepts: (PGI Dec 2000, AIIMS Dec 94)
Ans. b. 106 (Ref: Harper 30/e p226)
 #  6#
Energetics of Beta Oxidation
. " 6#
The number of ATPs obtained depends on the number of
 + 6#
*,   "  & 
  "    
 #
!#"  -*./
Ans. b. Malonyl CoA (Ref: Harper 30/e p235)
?  ;  
.' `F4J¥^
 Cys–SH group accept Acetyl CoA where n = no of carbon atoms
 +!*U " 6#   
   \ 
.' 
13. In which organelle(s) of hepatocyte, the elonga-  ? 
.' 
       ‚  ?#U7#-+
(PGI Dec 08)  _#U7#-+
 B     -~#-+
. =.  From 7 cycles of beta oxidation
 "    \Ž_
#-+
 
 <   \Ž~28 ATPs
 P.  
+ #  F4J“#  6#
Ans. a. Endoplasmic reticulum, c. Mitochondria  ?#  6#. -6#6 ’#-+
(Ref: Harper 30/e p236)  ?“#  6#
228
Self Assessment and Review of Biochemistry

 “Ž’80 ATPs . # 6# 


 Total ATPs from Palmitic Acid = 28 + 80 = 108  -
 0 (! '#1   #    
  -
   (!  " !#"  2 3450 2 3. (!  Ans. a.+  #{' 
17. Beta Oxidation in Peroxisome generate: (Ref: Dinesh Puri 3/e p210;
/*\Š< #Ž^ Nelson 20/e Chapter Defects in metabolism of lipids)
 _#+U
20. Adrenoleukodystrophy is associated with:
. U2O2
(CMC Vellore 2014)
 <6
  
 # 
  
   
 ?#U2
. # 
  
  
Ans. b. H2O2 (Ref: Harper 30/e p226)
   +
#  

Y!' 
 '
and leads to the formation of acetyl-CoA and H2O2 (from    +  # 
 ž!    J ;   Ans. a.# 
Z <6? # 
. ;.  -   >#
 ># #  

 ##
peroxisomes is not linked directly to phosphorylation and peroxisome biogenesis

#-+- 
 '  
Peroxisomes absent to reduced in number

  
   F620, C22J-&  
Catalase in cytosol
'    
   › 
3 
  

 #
#


$ "
Y!' V    !6# 
Defective oxidation and abnormal accumulation of very long chain
fatty acids
18. All are features of Refsum’s disease except: 
3 
  

" $ 
# # 

$  

(PGI Nov 2014) acid


   
 '  
Defects in certain steps of bile acid formation and accumulation
of bile acid intermediates
. 
 
.' 

3 

  

# # 

$ $ 

 # 
+    
Increased urinary excretion of dicarboxylic acids
 +
 - . +      Π{ %  
    ‚ /<JZ ‡^
from diet  @  6#
Ans. a, c, d, e. . •#-+
Refsum’s Disease  @#-+
 Defect in Alpha oxidation of Phytanic Acid (Phytanic    6#
Acid Oxidase) (phytanoyl CoA hydroxylase) in the  •#-+
     
+' Ans. b. •#-+ e.•#-+
     
 The manifestation of classic Refsum’s Disease -V.    _#U
includes impaired vision from retinitis pigmentosa, @#-+?#U#-+*
ichthyosis, peripheral neuropathy, ataxia, and,
 +   ' \  “#  6#
      
  
.'   @”7#-+
 Classic Refsum disease often does not manifest
until young adulthood, but visual disturbances  \   @7#-+
such as night blindness, ichthyosis, and peripheral  “#  6#. -6#   “Ž•#-+
neuropathy may already be present in childhood  *#-+•”@7@#-+
      #-+
 
 Restrict dietary dairy products and Green Leafy  *#-+   
+   @¥
Z. •#-+
19. Enzyme defect in Refsum’s disease:  According to new calculation
/*\Š #Ž^  ’ #-+
   
 +     .  .
 +  #{'  ' 
Metabolism of Lipids 229

ŒŒ{ % #


?#  #
  Skeletal Muscle
that occurring in mitochondria by: (PGI June 03)  P6'
 #  6# Ans. a. RBC (Ref: Harper 30/e p227)
. U2O2 formed  …/ *?
B' -
 [& 
  [  Almost all the organs utilize Ketone bodies with the
 _#UV exception of Liver and RBCs.
Ans. b. H2O2 formed (Ref: Harper 30/e p226)
26. Rothera’s test used for detection of: (Kerala 2010)
& @# 4      +
 /   Y!' ; . = 
 ?? #  š620,C22  ? # 
 Takes place in the Peroxisomes till Octanoyl CoA  …
 Oxidation in peroxisome produces Acetyl CoA and Ans. d. Ketone Bodies
H2O2F 
?#U2) Test for Ketone Bodies
 _#-+   = ƒ? 6 -Q #  
?' 
#  6# {  6#  Rothera’s Nitroprusside Test: Detect Acetoacetate
place in the mitochondria and Acetone
23. One of the following is obtained in the by beta  _
.  / ' . 
      ‚(JIPMER 2013) Name of the test Compound detected
 #  6#”#  6# Rothera’s Test Ketone Bodies, Branched chain
Ketoacids
. #  6#”+ 6#
Hay’s test Bile salt
 + 6#”+ 6#
Liebermann Burchard reaction Cholesterol
 #  6#
Ans. b.#  6#”+ 6# 099 =
>  Cholesterol

(Ref: Harper 30/e p225) 27. Which organ does not utilize ketone bodies:
& &     (AIIMS Sep 96)
 -     <
 {'  
  ? #   ;    . 
 . /
carbon atoms yields Acetyl CoA and a molecule of  * 
Propionyl-CoA  6   
 -  
 ! 
    Ans. a. Liver (Ref: Harper 30/e p227)
 

     
28. The immediate precursor in the formation of
Ketone Bodies acetoacetate from acetyl CoA in the liver is:
(PGI June 99)
24. Which of the following organs do not utilize
 "
ketone bodies? (PGI May 2014)
 / . U"=6#
. P/6  #   6#
 @! ' !.  6#
 " 
Ans. b. HMG CoA (Ref: Harper 30/e p228)
 U
Hydroxy-3-methylglutaryl-CoA lyase then causes
 <
  !6#[
U"=!6#

Ans. b. RBC, e. Liver
  
25. Ketone bodies can be utilized by all, except: 29. In a well fed state, acetyl CoA obtained from diet
(AIIMS May 2013) is least used in the synthesis of: (AI 2002)
 P/6  + 6#
. / . 6
230
Self Assessment and Review of Biochemistry

 # oacetate Derivatives of Cholesterol


 {'   Bile acids
Ans. c. Acetoacetate  Vitamin D
 ;
 
   F+   J   Corticosteroids
is active  Sex Hormones
 TCA cycle also takes place, Citrate and oxalosuccinate
are intermediates in TCA Cycle 32. Which of the following does not have cholesterol?
 Oxalosuccinate is an intermediate in the reaction of (Kerala 2006)
Isocitrate Dehydrogenase of TCA Cycle  Z
 Acetoacetate, a primary ketone body is synthesized . B

  # 
 +
Cholesterol Synthesis Ans. c. Adrenaline (Ref: Harper 30/e p267)
30. Common enzyme in cholesterol and Ketone body
metabolism: (AI 2012) Specialized Products of Cholesterol
 U"=6#    Bile Acids [Excretory form of Cholesterol]
. U"=6#   Vitamin D
 U"=6#   Sex Hormones
 -  Corticosteroids
Ans. b. HMG CoA Synthase, d. Thiolase 33. Which coenzyme act as reducing agent in anabolic
(Ref: Harper 30/e p228, 267) reaction?
Compare–cholesterol synthesis and ketone body synthesis  ?#U
Ketone body Cholesterol . ?"_U
Characteristics synthesis synthesis  _#+U
Site Mitochondria Cytoplasm  _#U
HMG CoA as an Yes Yes Ans. c._#+U
intermediate
*<    #"  >      
HMG CoA Synthase Yes, the regula Yes acids, Cholesterol and steroid hormones.
tory step
HMG CoA No Yes, the rate 34. Enzyme common for synthesis of both ketone
Reductase limiting step bodies and Cholesterol: (JIPMER Nov 2015)
HMG CoA Lyase Yes No  U"=6#P  
Thiolase Yes Yes . U"=6#* 
 #  6#6.' 
 -   &     …/  
 U"=6#< 
- #   6##  6#
Ans. b. HMG CoA Synthase (Ref: Harper 30/e p237, 267)
 In Cholesterol synthesis, two mols of acetyl CoA
 
#   6#. - Ketone Body
 Cytoplasmic HMG CoA Synthase for Cholesterol Characteristics Synthesis Cholesterol Synthesis
  Site Mitochondria Cytoplasm/Smooth
 "  U"=6#* 
…/  Endoplasmic reticulum
HMG CoA as an Yes Yes
31. All are derived from cholesterol except: intermediate
(Kerala 2011) HMG CoA Yes, the regulatory Yes
 Z Synthase step
. / HMG CoA No Yes, the rate limiting step
 / Reductase
 * HMG CoA Yes No
Ans. c./+ (Ref: Harper 30/e p267) Lyase
Metabolism of Lipids 231

Bile Acids  LDL receptor is used for the uptake of LDL and other
remnant lipoprotein (VLDL remnant)
35. Bile acids are derived from: (AI 1994)
 ?     LDL receptor has ligand binding site for both apo E
and apo B100
. 6
 ?
<</’’  

 /.
LDL receptor
 +
 ?
B 
Ans. b. Cholesterol (Ref: Harper 30/e p267)  
<< 
 /  '  

 
LDL Receptor
36. Bile acids synthesized in liver (primary bile acids)  ?   
 <<    
(PGI Dec 2000) B< 
<< /’’
 <     B
. 6    +< ' F 
 6 '    U J
 '     High level of cholesterol upregulate LDL receptor,
causing an increase in the uptake LDL
 -   
 Mechanism of uptake is receptor mediated uptake
Ans. b. Cholic acid, c. Chenodeoxycholic acid,
or absorptive pinocytosis.
‘‘‘‘{Taurocholic acid (Ref: Harper 30/e p273)
 Vesicles formed during absorptive pinocytosis are
Primary Bile acids—Liver derived from invaginations (pits) are coated on the
They are:   ; 
 Cholic Acid (Most abundant bile acids in mammals)     
 Chenodeoxycholic Acid or Chenic acid  ?
<</’’  

Secondary Bile acids—Intestine Clathrin


They are:  Three-limbed structure (called a triskelion)
 Deoxycholic Acid  With each limb being made up of one light and one
  
 
 Lithocholic Acid
Conjugation of Primary Bile Acids 38. Triglycerides are maximum in: (AIIMS May 2007)
 +  .     $  ; =    6  
- . Z<<
 6$ '  <<
 In humans the ratio of Glycine to Taurine conjugates  U<
@Q Ans. a. Chylomicrons 

   
With Glycine With Taurine Protein Lipid Main lipid
Glycocholic Acid Taurocholic acid Lipo- content Content compo- Apolipo-
protein Source (%) (%) nent proteins
Glycochenodeoxycholic acid .#  
 
Chylo Intes `4
j\`jj Triacylg :&
::&

microns tine (Mini (Maxi lycerol :_&

Lipoprotein Metabolism mum) mum) +]\&


:&
::&

37. Which is the ligand for receptors present in liver :::&



for uptake of LDL: (May 2009) VLDL Liver [`! j!`jV Triacylg +!!&

 B (intes lycerol :&


::&

tine) 
:::
. # B
IDL VLDL  \j Triacyl +!!&

 B /’’
glycerol,
 /’’ cholesterol
Ans. d. apo B100 (Ref: Harper 30/e p271) Contd...
232
Self Assessment and Review of Biochemistry

Contd...  LDL cholesterol is taken up by liver and extrahepatic


Protein Lipid Main lipid tissues (peripheral tissues).
Lipo- content Content compo- Apolipo-
protein Source (%) (%) nent proteins 41. Which of the following lipoproteins does not
LDL VLDL 4 79 Choles +!! move towards charged end in electrophoresis:
terol  Z<< (AI 2010)
HDL Liver, V4
;\ Phos :&
::&
. <<
intes (Maxi (Mini pholipids, :_&
:&
 U<
tine mum) mum) Choles ::&
:::&

terol D,E  6  


Ans. d. Chylomicron  !" #$#
39. Increased level of lipoprotein (a) predisposes to: Electrophoretic Separation of Lipoproteins
 <  (AIIMS May 2007)
. # 
 _   
 + 
Ans. b. Atherosclerosis
(Ref: Vasudevan and Sreekumari 7/e p179)
Lp (a)
 #<<
 #   /’’.   . 
 Strongly associated with Atherosclerosis and Myo-     6 F+    
 
 
  major determinants of charge) content
 *  ;+  Higher the protein content faster the mobility of
  
 ;   
 +     
  Chylomicron remains at the origin
 U .     U<

 * . . From Cathode to Anode the Order of Lipoprotein
Lpx  Chylomicron
 6'  .  .  <<FY<J
    .;+   Z<<F+Y<J
 
<¢  <F/ Y<J
 U  '
   U<FX<J
40. Main transporter of cholesterol to peripheral
42. All of the following statements about Lipoprotein
tissue: (PGI Nov 2011)
Lipase are true, Except: (AI 2009)
 U<
 ?  
. <<
. ?   
 Z<<
        
 <
 V6 
 
 6  
Ans. d. Does not require CII as cofactor
Ans. b. LDL (Ref: Harper 30/e p272, 273)
(Ref: Harper 30/e p257)
Transport of Cholesterol between tissues
     6   Option a and b
 95% of Chylomicron cholesterol is delivered to liver  Lipoprotein lipase is not present in the myocyte and
6    
 -   
Z<<  It is actu  

 Most of the cholesterol secreted in VLDL is retained  /   ;
<  << ;
Metabolism of Lipids 233

Lipoprotein Lipase Hormone Sensitive Lipase


 Is lo  ;
.    The activity of this enzyme is under the control of
 Is anchored to the endothelium by negatively charged  
  

  +adipose tissue
 It has been found in heart, adipose tissue, spleen,  Involved in the metabolism of triacylglycerol stored
lung, renal medulla, aorta, diaphragm, and lactating  
      U .&

   
 
 It is not normally found in blood; however, following   .
injection of heparin, lipoprotein lipase is released
43. All of the following statements about apoproteins
from its heparansulfate binding sites into the
are true Except: (AI 2008)
 
 ##! <6#-
Both phospholipids and apo C-II are required as
. #6! 
cofactors for lipoprotein lipase activity, while apo A-II
and apo C-III  .  #6!.
 #6! 
Reaction Catalyzed by Lipoprotein Lipase Ans. c. Apoprotein C-II inhibits lipoprotein lipase
 Triacylglycerol is hydrolyzed progressively through (Ref: Harper 30/e p255)
       
Apoliporoteins and its function
??# 
Apolipoprotein Function
 /
??#   
$
: Activates Lecithin Cholesterol Acyl Transferase
 Thus it helps in the hydrolysis of Chylomicron and
(LCAT)
VLDL to Chylomicron remnant and VLDL remnant
$
:: Inhibits Lipoprotein Lipase
  
$
_ Promote lipoprotein lipase mediated Triacyl
Lipoprotein Lipase in Heart During Starvation -
 $ 
 Heart lipoprotein lipase has a low Km for triacylgly- $
+!!  

_'3'


 
cerol, about one-tenth of that for the enzyme in Act as ligand for the LDL receptor and LDL re
  $ 
 
$  
'>7(

#$ 9

'3'
 This enables the delivery of fatty acids from $
+]\  

 


  
triacylglycerol to be redirected from adipose tissue $
: Inhibit Cholesterol Ester Transfer Protein(CETP)
to the heart in the starved state when the plasma $
:: Activates lipoprotein Lipase
    
$
::: Inhibit Lipoprotein Lipase
 Lipoprotein Lipase in Lactating Mammary Gland
Apo E Act as ligand for LDL receptor for uptake of Chy
 A similar redirection to the mammary gland occurs lomicron remnant and VLDL remnant (IDL)
during lactation, Allowing uptake of lipoprotein
  
   
milkfat   44. Which of the following types of Hypertriglyceri-
Lipoprotein Lipase and Insulin demia is associated with an increase in chylomi-
cron and VLDL remnants? (AI 2007)
 In adipose tissue, insulin enhances lipoprotein
 - 
lipase synthesis in adipocytes and its translocation

 
    . - 
 Deficiency of Lipoprotein lipase lead to Type I  - 
U   -¡+BZ
Hepatic Lipase Ans. c. Type III 

%!#&' *
   
 .  
 
  45. The human plasma lipoprotein containing the
  .  highest percentage of triacylglycerol by weight is:
 This enzyme, however, does not react readily with  Z<< (AI 2006)
  Z<< . 6  
 But is involved in chylomicron remnant and HDL  U<
.  <<
234
Self Assessment and Review of Biochemistry

Ans. b. Chylomicrons 



   loss of apo C (which returns to HDL) but not apo E, which
 Lipoprotein with highest TAG content is Chylomicron  -chylomicron remnant is about half
 Lipoprotein with highest Cholesterol and Cholesterol the diameter of the parent chylomicron and is relatively
 << enriched in cholesterol and cholesteryl esters because

   
    *    
46. Cholesterol from dietary sources is transported to VLDL, with the formation of VLDL remnants (also called
the peripheral tissue by: (Kerala 2012) *
>= *=%*? !
!*GI
 6  
. Z<< 49. In Coronary artery disease the cholesterol level
 << /
ˆ^# 

‚ /*\Š< #Ž^


 U<  /;’’
Ans. c. LDL (Ref: Harper 30/e p272) . ª7’
Transport of Cholesterol between tissues  ª’
     6    ª“’
 95% of Chylomicron cholesterol is delivered to liver Ans. a. Below 200
6    +-<= >* %!& &'>%*
? @
 -   
Z<< Biochemical parameter Values mg/dL Risk
 Most of the cholesterol secreted in VLDL is retained '3'
  €
!! Optimum
<  << !!`4j Near or above
 LDL cholesterol is taken up by liver and extrahepatic Optimum
tissues (peripheral tissues). V!`6j Borderline high
;!`\j High
47. LDL level in NonDiabetics should be below what
ƒ
j! Very High
 "
ˆ’ /*\Š< #Ž^
. 

  €
4!! Desirable
 ’’
4!!`4Vj Borderline High
. \7
ƒ
4]! High
 7’ %3'  €
]! High
 @’ > 60 Low
Ans. a. 100 mg/dl
 +-<= >* %!& &'>%*
? @ ‡{ 4#   #‚ /*\Š< #Ž^
LDL Goals for different risk categories of atherosclerosis  # 
Risk category LDL Goal mmol/L(mg/dl) . 6
Very high €
\

[!(  U
High €
4;

!!(  "   
 
Moderately high €
4;

!!(
Ans. b. Cholestasis
Moderate €
V]

V!(
' €
]

;!(
In cholestasis, free cholesterol, coupled with phospho-
lipids, is secreted into the plasma as a constituent of a
`{  #  ‚ /*\Š< #Ž^    <+!¢-   
 -
 in skinfolds, producing lesions resembling those seen in
. +< #  ;?/<F'J+
and eruptive xanthomas can also be seen in patients with
 -
6  

 -
U<
Ans. a. To form remnant lipoprotein 51. Which is the lipoprotein with lowest density?
(Ref: Harper 30/e p257)  U<
The Action of Lipoprotein Lipase Forms Remnant . <<
Lipoproteins  Z<<
Reaction with lipoprotein lipase results in the loss of  <
\’¥•’£
  
    the Ans. c. VLDL (Ref: Harper 30/e p214 Table 25.1)
Metabolism of Lipids 235

* "     %  


Composition
Diameter Density
Lipoprotein Source (nm) (g/mL) Protein (%) Lipid (%) Main lipid components Apolipoproteins
Chylomicrons Intestine j!`!!! €
!j6 `4 j\`jj Triacylglycerol :&
::&
:_&
+]\&
:&

::&
:::&

Chylomicron Chylomicrons ]6`6! €
!!; ;`\ j4`j] Triacylglycerol, +]\&

remnants phospholipids, cholesterol
VLDL Liver (intestine) V!`j! !j6`!!; [`! j!`jV Triacylglycerol +!!&
:&

::&
:::
IDL VLDL 46`V6 !!;`!j  \j Triacylglycerol, cholesterol +!!&

LDL VLDL 4!`46 !j`!;V 4 79 Cholesterol +!!
HDL Liver, intestine, 7$ :&
::&

VLDL, lipids, :_&
:&
::&

chylomicrons cholesterol :::&


3&
4

HDL 4!`46 !j`!;V V4 ;\

52. Which of the following has highest electrophoretic  Transport of LDL to liver and Extrahepatic tissues
mobility and least lipid content: (PGI June 01) . /’’
 6ylomicrons
54. Which of the following is an activator of LCAT:
. U<
(JIPMER 2002)
 <<
 #/’’
 Z<<
. #/~“
 <
 #B
Ans. b. HDL 

   
 ##!
HDL Ans. d. Apo A-I (Ref: Harper 30/e p255)
 Alpha lipoprotein
 Least Diameter 55. Cholesterol present in LDL: (AIIMS May 03)
 P      .
 Maximum Electrophoretic Mobility
 
 
 "'+6
. /         [
 Least lipid content   .
 Carry Cholesterol from peripheral tissues to liver and  {  .!
other steroidogenic tissues 
<< 
 This is called Reverse Cholesterol transport  «   
 !
 This makes HDL Cholesterol ‘the good cholesterol’ CoA: cholesterol acyltransferase ACAT
 The major role of HDL is to acts as the repository Ans. c. On accumulation in the cell inhibits replenishment
for apo C and apo E required for the metabolism of ¬¬¬¬¬¬
<<  (Ref: Harper 30/e p271)
Z<< 6    Option a: LDL cholesterol is primarily from other
lipoproteins where as HDL cholesterol is from
53. Which helps in the transport of chylomicrons

from intestine to liver: (AI 2000)
 #/  Option b: Mechanism of uptake of LDL by LDL
receptor
. ##
 LDL receptors occur on the cell surface in pits
 #6
that are coated on the cytosolic side of the cell
 #B
membrane with a protein called clathrin.
# #B (Ref: Harper 30/e p257)
 The LDL receptor spans the membrane, the
 Transport of Chylomicron remnant to liver by Apo E B-100 binding region being at the exposed amino
 Transport of IDL or VLDL remnant to liver by Apo E  
236
Self Assessment and Review of Biochemistry

 After binding, LDL is taken up intact by  Heparin is an anticoagulant and it is a Glycosamino


endocytosis Glycan.
 The apoprotein and cholesteryl ester are then  It releases Lipoprotein lipase not hormone sensitive
hydrolyzed in the lysosomes, and cholesterol is lipase.
    58. Lipoprotein a resembles: (JIPMER 2014)
 -    
   +
 Option b: LDL is taken up intact, and not cholesterol . +
 [ .  -.
Cholesterol balance in tissues  +.
-ž'
 . 
 Ans. a.+ &' "
J  =U'=# #
genes encoding Lp(a)
 HMG-CoA synthase, HMG-CoA reductase, and other  #<<
enzymes involved in cholesterol synthesis  #FJ   /’’.   . 
 -<< 
*PB/+;  "$
  
<FJ
Thus coordinately suppresses cholesterol synthesis and  Strongly associated with Atherosclerosis and
 "   
 
 In addition, ACAT activity is stimulated, promoting  *  ;+Q
    
 ;   
 + 
Recent advance 
In addition, recent research has shown that the protein  U .   
proprotein convertase subtilisin/kexin type 9 (PCSK9)  * . .
regulates the recycling of the receptor to the cell surface
.   
   /     Dysbetalipoproteinemia
Cholesterol balance is maintained within normal limits 59. Which of the following is increased in lipoprotein
     ‚ /ZZ!*Œ‡‡‡^
 Z<<
56. A person on a fat free carbohydrate rich diet
. <<
continues to grow obese. Which of the following
 U<
lipoproteins is likely to be elevated in his blood?
(AI 2004)  6  
 6   Ans. a. VLDL, d. Chylomicron
. Z<< Phenotype I IIa IIb III IV V
 << Lipoprotein, Chylo- LDL LDL Chylo VLDL Chy
elevated microns and micron lomi
 U<
(pre- VLDL and crons
Ans. b. VLDL domi- VLDL and
Excess Carbohydrates are converted to Acetyl CoA, nant) rem VLDL
VLDL nants
;      ?          
Triglycerides  N    
B -=Z<<    -=*
Z<< Cholesterol     N/ 
(total)
57. Which of the following is false about heparin? '3'      
(JIPMER May 2014) terol

 P %3'  N/  N  


terol
. P
   60. Familial hypercholesterolemia is: (PGI Dec 98)
 =     << 
Ans. b. Releases hormone sensitive lipase .  U< 
Metabolism of Lipids 237

 U"=6#      Hyperthyroidism


  Z<<   Drugs: niacin toxicity
Ans. a. << 
62. A patient was diagnosed with isolated increase in
Frederickson
type Nomenclature Molecular defect
LDL. His father and brother had the same disease
Type I Familial Chylomicronemia Lipoprotein Lipase or with increased cholesterol. The likely diagnosis
Syndrome (FCS) Apo CII is: (AIIMS May 2009)
Type IIa Familial LDL receptor  ?  
Hypercholesterolemia (FH) . #.
Familial Defective Apo B $
+!!  ?<+<   F J
(FDB)
Autosomal Dominant
 << 
Hypercholesterolemia Ans. d. LDL receptor mutation
Type II (ADH Type II) (Ref: Harrison 18/e Table 356.3)
Autosomal Dominant PCS K9
Hypercholesterolemia
?  ƒ6 
U mia
Type III (ADH Type III)
Phenotype Type I Type II Type III
Autosomal Recessive LDL Receptor Adapt
Hypercholesterolemia er Protein (LDLRAP) Molecular Lipoprotein LDL receptor apo E defect
(ARH) Defect Lipase apo CII defect apo
+!!
 
Sitosterolemia +-6

+%\
Type II b Familial Combined Hyper  Genetic No- Familial Chy Familial Hy Familial Dys
lipidemia (FCHL) menclature lomicronemia percholesterol   $$ 
Syndrome emia inemia
Type III Familial Dysbeta lipopro Apo E
teinemia (FDBL) Familial Defec
tive apo B
Type IV Familial Hypertriglyceride $

_
mia (FHTG) Clinical fea- Eruptive Xan Tendon Xan Tuberoeruptive
tures thoma Pancre thoma Xanthoma
Type V Familial Hypertriglyceride Apo A –V and
atitis Lactescent Tuberous Xan No Pancre
mia (FHTG) -7:%+7
 Plasma thoma atitis
No Coronary/ No Pancreatitis Coronary ath
61. Hypertriglyceridemia not seen in: Peripheral Ath No Lactescent erosclerosis
(CMC Vellore 2014) erosclerosis plasma Coro +++ Peripheral
 U    nary athero Atherosclero
sclerosis +++ sis +
. - ."
Peripheral
 6ƒ*   Atherosclero
 U sis +
Ans. a. Hypothyroidism 

%!@   Lipid el- Triacyl Glycerol Cholesterol Cholesterol


evated NB: Cholesterol NB: Triacyl Triacyl Glyc
Conditions associated with increased LDL (Increased Normal Glycerol erol
Cholesterol) Normal
 Hypothyroidism Lipoprotein Chylomicron LDL Chylomicron
elevated remnant VLDL
 Nephrotic syndrome remnant
 Cholestasis
 #    63. Abetalipoproteinemia result in absence of:
 Anorexia nervosa  6   (PGI Dec 02)
 Hepatoma . <<
 Drugs: thiazides, cyclosporin, tegretol  Z<<
Conditions associated with decreased LDL  U<
 Severe liver disease  -=
 Malabsorption Ans. a, b, c, e.
 Malnutrition (Ref: Harrison 19/e Chapter Disorders
 Gaucher’s disease associated with metabolism of Lipoproteins)
238
Self Assessment and Review of Biochemistry

Abetalipoproteinemia c #B


 Autosomal recessive disease  ##
 Biochemical Defect Ans. b. Apo B48 (Ref: Harper 30/e p254)
 Loss-of-function mutations in the gene encoding  #/~“
 '     
   
F"-+J  The only one apolipoprotein in LDL is apo B100
"--+ 67. Both Triglycerides and HDL increased:
 "-+ 
            * (JIPMER Nov 2015)
Z<<   
. #
 So Chylomicron Q, VLDL Q and hence LDL Q not
 *
   
 # 
 +  
       
' ;   # # 
 Chylomicrons, VLDLs, LDLs, and apo B are (Ref: Harrison 19/e Disorders of lipoprotein metabolism)
  .  P  .[ 
 
64. Absence of this apo lipoprotein is responsible for  -   [  
      
the genetic disorder, familial type III hyperlipo-   
# 
  /*\Š< #Ž^  Alcohol consumption stimulates hepatic secretion of
 #/’’ VLDL, possibly by inhibiting the hepatic oxidation
. #/~“ of free fatty acids, which then promote hepatic
 #B     Z<< 
 #6  Regular alcohol use also raises plasma levels of
Ans. c. Apo E U<!6
65. Patient with abetalipoproteinemia frequently 68. In prolonged fasting Glycerol formed from Trigly-

  }    > { ;  ceride. Which of the following statement is true
due to inability to: (JIPMER 2012) regarding Glycerol: (PGI 2013)
 +       ‰  
   
. +  Z<< .     . 
  
 * &  
    
          
 -
 * &
    mone sensitive lipase
Ans. a. +      b.+  Z<<  =  .
 
Fat soluble vitamins are absorbed in Chylomicrons,  
 .     
<
from liver it is secreted out by VLDL Lipase

 

   Q Ans. c. It is formed due to increased activity of hormone

Most clinical manifestations of abetalipoproteinemia result from sensitive lipase, d. Glycerol acts as a substrate for
 


$ 

$

 #
   gluconeogenesis

Vitamin E and retinyl esters are normally transported from
  


 

 
In prolonged fasting Gluconeogenesis sets in and Glycerol

Vitamin E is dependent on VLDL for transport out of the liver and liberated from stored TGs in adipose tissue by the action
 

 #  


As apo B containing lipoproteins are not formed these patients,
there is, marked deficiency of Vitamin E, mild to moderate 69. Full form of LCAT: (PGI May 2014)
 

_  


@
 < 6# -


Bleeding manifestation due to defective absorption of fat and hence

#
   . < 6# -

 < 6# -

66. Apolipoprotein of Chylomicron is:  < 6# -

 #/’’ (JIPMER Nov 2015)  < 6#-

. #/~“ Ans. a.< 6# -

e

e
m

m
4
m
co
m
co

Section Molecular Genetics


m
co

C H A P T E R S

8. Chemistry and Metabolism of Nucleotides


9. Structure and Organization of DNA
10. DNA Replication
11. Transcription
m
co

12. Translation
13. Regulation of Gene Expression
14. Molecular Biology Techniques and Recent
Advances in Molecular Biology
m
co
co co co co co
m m m m m
m
e

m
e

m
e

m
e
e

e
m

m
m
co

8 Chemistry and Metabolism


of Nucleotides
m
co

Topics Included
• Nucleic Acids • Metabolism of Purines and Disorders
• Components of Nucleotides • Metabolism of Pyrimidines and Disorders
m

NUCLEIC ACIDS ‒ Guanine-2-amino, 6-oxopurine.


co

Adenine and Guanine are present in both DNA and RNA.


Nucleic acids are polymers of nucleotides joined by 3'–5'
Minor Purine Bases are:
phosphodiester bond.
‒ Hypoxanthine-6 oxo purine
Two Types of Nucleic Acid ‒ Xanthine-2, 6 dioxopurine
• Deoxyribonucleic acid (DNA) ‒ Uric Acid-2, 6, 8 trioxopurine.
• Ribonucleic acid (RNA). 2. Pyrimidine Bases are Cytosine, Uracil and Thymine
General Properties of Nucleotides ‒ Cytosine-2 oxo 4 amino Pyrimidine
• Nucleotides are polyfunctional acids. Nucleotides ‒ Uracil-2, 4 dioxo Pyrimidine
bear a negative charge at physiological pH. ‒ Thymine-2, 4 dioxo 5 methyl Pyrimidine.
• Absorbs UV light at a wavelength 260 nm, at pH Modified Nitrogenous bases
7.0. The conjugated double bond of Purine and ‒ Dihydrouracil
m

Pyrimidine nucleotide is responsible for it. ‒ Pseudouridine


co

Ultraviolet Rays are Mutagenic ‒ 5-Methyl Cytosine


The mutagenic effect of ultraviolet light is due to its absorption of ‒ Dimethyl amino adenine
UV light by nucleotides in DNA that result in chemical modifications.
‒ 7-methyl Guanine.
NUCLEOTIDES Remember
• Cytosine is present in both DNA and RNA
Composition of Nucleotides • Uracil is present only in the RNA
The components of nucleotides are: • Thymine is present only in the DNA.
A nitrogenous base + A pentose Sugar + Phosphate
Group
Nitrogenous Base
They are nitrogen containing heterocyclic ring structures.
m

Two types of Nitrogenous Bases


co

• Purine
• Pyrimidine.
1. Purine BasesQ are Adenine and Guanine
‒ Adenine-6 Amino Purine Fig. 8.1: Structure of purine and pyrimidine ring
e

e
m

m
m
co

242 |  
Self Assessment and Review of Biochemistry

Methylated Xanthine derivative present in the Nucleic Acid


Coffee-Caffeine (Trimethylxanthines) Polymers of nucleotides joined by 3'–5' phosphodiester
Tea-Theophylline (Dimethylxanthine) bond.
Cocoa-Theobromine (Dimethylxanthine) They are RNA and DNA.
Vitamin that contains pyrimidine ring is Thiamine 3’–5’ Phosphodiester bond
3’ hydroxyl group of sugar of first mononucleotide linked
m

Pentose Sugar to 5’ phosphoryl group of the second mononucleotide by


co

There are two types of Pentose Sugar. a 3’–5’ phosphodiester bond.


• D-Ribose Sugar in RNA
Nucleic Acid Exhibit Polarity
• 2’ deoxy D-Ribose Sugar in DNA.
• Nucleotides are linked by 3'–5' phosphodiester bond,
Composition of Nucleoside there is a free phosphoryl group at 5' of the first
• Nitrogenous base + Pentose Sugar nucleotide and free hydroxyl group at the 3' end of
• C1 of ribose or deoxyribose sugar to N1 of Pyrimidine last nucleotide.
or N9 of Purine by β N Glycosidic linkage • Hence, nucleic acid exhibits polarity.
• So Nucleosides are N Glycosides. • The base sequence is usually written from 5' end to
3’ end (Fig. 8.2).
Composition of Nucleotides
• Nucleoside + Phosphoryl groups
m

• Usually phosphoryl group is attatched to 5’ hydroxyl


co

group of pentose sugar by an ester bond.


• Additional phosphoryl group are attached by an acid
anhydride bond to form nucleoside diphosphates
and triphosphates.
Ribonucleotides
Nitrogenous Pentose Nucleoside = Nucleotide =
Base Sugar Nitrogenous Nucleoside +
Base Phosphate Group
Adenine Ribose Adenosine Adenosine Mono-
phosphate (AMP)
Guanine Ribose Guanosine Guanosine Mono- Fig. 8.2: Nucleic acid
m

phosphate (GMP)
Remember
co

Cytosine Ribose Cytidine Cytidine Mono-


Most abundant free nucleotide in mammalian cell is ATP.
phosphate (CMP)
Uracil Ribose Uridine Uridine Mono- Functions of Nucleotides and Nucleotide
phosphate (UMP)
Derivative
Hypoxanthine Ribose Inosine Inosine Mono-
• They are building blocks of nucleic acid.
Phosphate (IMP)
• Adenosine Tri-Phosphate (ATP) is the principal
Xanthine Ribose Xanthosine Xanthosine Mono-
Phosphate (XMP) biological transducer of free energy.
Deoxyribonucleotides • cAMP and cGMP are second messengers in hormonal
pathways.
Adenine Deoxy- dAdenosine dAdenosine Mono-
ribose phosphate (dAMP) • S Adenosyl Methionine (SAM) is an important methyl
Guanine Deoxy- dGuanosine dGuanosine Mono- donor.
ribose phosphate (dGMP) • Adenosine 3' Phosphate -5' phosphosulphate (PAPS)
m

Cytosine Deoxy- dCytidine dCytidine Mono- is an important Sulfate donor.


co

ribose phosphate (dCMP) • UDP-Sugar derivative in Glycogen synthesis and


Thymine Deoxy- dThymidine Thymidine Mono- oligosaccharides of Glycoproteins.
ribose Phosphate
• UDP-Glucuronic acid in conjugation of bilirubin.
e

e
m

m
m
co

Chemistry and Metabolism of Nucleotides  | 243

• Many coenzymes incorporate nucleotides in its De Novo Synthesis of Purine Nucleotides


structure, like NAD+, NADP, FMN, FAD+ • Site of synthesis—Most of the tissues but majority
Nucleotide Derivatives as Coenzymes in the liver
Biologically active forms of Niacin are NAD + and NADP + • Organelle—Cytoplasm.
NAD+ is Nicotinamide Adenine Dinucleotide.
• It consists of two nucleotides Sites where de novo synthesis do not take place:
• Brain
m

• First nucleotide with nicotinamide as base and second nucleotide


with adenine as base
• Erythrocytes
co

• Nicotinamide-D Ribose-P-P-D Ribose-Adenine


• Polymorphonuclear leukocytes
NADP+ is Nicotinamide Dinucleotide Phosphate
• To the Adenine of NAD+ there is Phosphoric group • Bone marrow.
Biologically active form of Riboflavin are FAD + and FMN.
FMN is Flavin Mononucleotide (FMN).
It consists of Flavin-Ribityl-P, where ribityl is an alcohol.
FAD+ is Flavin adenine nucleotide
It consists of Flavin-Ribityl-P-P-Ribose-Adenine.

Comparison of DNA and RNA


DNA RNA
Mostly seen in the Nucleus Mostly seen in the Cytoplasm
m

Pyrimidine bases are Thymine Pyrimidine bases are Uracil and


co

and Cytosine Cytosine


Sugar is deoxyribose Sugar is Ribose
Pyrimidines = Purines Pyrimidines not equal to Purines
Fig. 8.3: Source of purine ring
Not destroyed by Alkali Destroyed by Alkali
Usually Double Stranded Single Stranded Sources of Purine Ring
• C 4, C 5, N7 by Glycine
Pseudouridine in tRNA • N3, N 9 by Amide Nitrogen of Glutamine
• Seen in the Ribothymidine Pseudouridine Cytidine • N 1 by Aspartate
(TψC) arm of tRNA. • C2 by N10 Formyl THFA
• C8 by N5 N10 Methenyl THFA
• Formed by modification of UMP on preformed tRNA.
• C6 by Respiratory CO2
• D-Ribose of pseudouridine linked to C-5 of Uracil,
m

by a carbon to carbon bond rather than by β N Key Points of De Novo Purine Biosynthesis
co

Glycosidic bond. • Rate limiting Step in Purine Synthesis—PRPP Glutamyl


• Pseudouridine is excreted unchanged in urine. Amidotransferase
• Purine ring is built up on a ribose-5-phosphate. Hence, nucleotides
Thymidine in tRNA are the products of the de novo synthesis
• UMP in preformed tRNA is methylated by S-adenosyl • First Nucleotide formed in Purine Synthesis-Inosine Mono
Methionine (SAM) to form TMP. Phosphate (IMP)
• tRNA is the RNA with thymidine. • From IMP Adenosine Monophosphate (AMP) and Guanosine
monophosphate (GMP) is formed.
• Ribithymidine is also seen in Pseudouridine Cytidine
(TψC) arm of tRNA.
Conversion of IMP to AMP and GMP
PURINE METABOLISM • Amino group of AMP is contributed by Aspartic Acid
Metabolic Pathways of Purine • Amino group of GMP is contributed by Glutamine.
m

• De novo synthesis of Purine Nucleotides Mnemonic


co

• Salvage pathways of Purine Nucleotides A for A (Aspartic Acid for AMP)


• Degradation of Purine Nucleotides. G for G (Glutamine for GMP)
e

e
m

m
m
co

244 |  
Self Assessment and Review of Biochemistry

• Phosphorylation of Purine Nucleoside


Adenosine Kinase
– Adenosine + ATP --------------> Adenosine Mono-
phosphate + ADP

CATABOLISM OF PURINE NUCLEOTIDES


m

• Adenine nucleotides catabolism—Liver, Heart


co

muscle, Skeletal muscle, GIT mucosa


• Guanine nucleotide catabolism—Liver, spleen,
Kidneys Pancreas, GIT mucosa
Fig. 8.4: Overview of de novo purine synthesis • The end product of Purine catabolism is in humans
Uric AcidQ
Regulation of De Novo Purine Synthesis • The end product of purine catabolism in mammals
• The regulatory enzymes of Purine Synthesis are PRPP other than higher primates is Allantoin
Synthase and PRPP Glutamyl Amidotransferase • Uricase enzyme convert uric acid to water soluble,
• The rate limiting step of de novo Purine Synthesis is Allantoin.
PRPP Glutamyl amidotransferase Steps of Catabolism of Purine Nucleotides
• The overall determinant of de novo purine synthesis Adenosine Deaminase
m

is level of PRPP
• Adenosine converted to inosine by Adenosine
co

• This activates PRPP Synthase


Deaminase.
• The rate of PRPP Synthesis depends on the availability
of ribose 5 Phosphate Purine Nucleoside Phosphorylase
• AMP, ADP, GMP and GDP the end products of • Inosine is converted to Hypoxanthine
de novo Purine synthesis feedback regulate PRPP • Guanosine is converted to Guanine.
Synthase and PRPP Glutamyl Amidotransferase. Xanthine Oxidase
• Convert Hypoxanthine and Guanine to Xanthine.
SALVAGE PATHWAY OF PURINE
• Same enzyme catalyse conversion of Xanthine to
NUCLEOTIDES
Uric acid.
Recycling of degraded purines and purine nucleosides to
their corresponding mononucleotides.
• Less energy consuming
m

• Effective recycling of degraded nucleotides


co

• Important in organs where de novo synthesis do not


take place.
Salvage pathway reactions are:
1. Phosphoribosylation of Purine bases (Pu) by PRPP.
2. Phosphorylation of Purine Nucleosides (PuR) by ATP.
Salvage Pathway Reaction of Purines
• Phosphorylation of Purine (Pu) by PRPP
Adenine Phosphoribosyl Transferase (APRTase)
‒ Adenine + PRPP --------------> Adenosine
Monophosphate + PPi
Hypoxanthine Guanine Phosphoribosyl
m

Transferase (HGPRTase)
co

‒ Hypoxanthine + PRPP --------------> Inosine


Monophosphate + PPi
‒ Guanine + PRPP --------------> Guanosine
Monophosphate + PPi Fig. 8.5: Catabolism of purine nucleosides
e

e
m

m
m
co

Chemistry and Metabolism of Nucleotides  | 245

CLINICAL CORRELATIONS—PURINE Treatment includes


METABOLISM • High fluid intake
• Dietary purine restriction
Lesch–Nyhan SyndromeQ
• Allopurinol, which inhibits the conversion of adenine
X-linked Recessive DisorderQ
to 2,8-dihydroxyadenine
Biochemical Defect
• Alkalinization of the urine is to be avoided, because, unlike
m

• Complete deficiency of HGPRTase that of uric acid, the solubility of 2,8-dihydroxyadenine


co

• Purine accumulates does not increase up to a pH of 9


• Purines degraded to uric acid • Shock-wave lithotripsy has been reported to be
• Uric acid level increases successful.
Clinical Features
• Hyperuricemia, Intellectual Disability Gout
• Dystonic Movement, Choreoathetosis, Dysarthric Group of disorders presented with
Speech 1. Hyperuricemia
• Compulsive self-mutilationQ 2. Uric Acid Nephrolithiasis
• Megaloblastic anemia also can occur in the Lesch– 3. Acute Inflammatory Arthritis.
Nyhan syndrome, in which regeneration of purine
Two Types of Gout
nucleotides is blocked.
• Primary Gout—Due to defect in the enzymes that
m

Diagnosis lead to overproduction of purine nucleotides


co

• Hyperuricemia ‒ Superactivity PRPP Synthetase (X linked Dis-


• HGPRTase enzyme activity in the erythrocytes is order)
deficient.
‒ Superactivity PRPP Amidotransferase
Treatment ‒ Deficiency of HGPRTase (Lesch–Nyhan synd-
• Allopurinol rome)
• Alkalanization of urine ‒ Glucose 6 Phosphatase Deficiency (Type I
• High fluid intake. Glycogen Storage Disorder).
Kelley–Seegmiller Syndrome • Secondary Gout
• Partial deficiency of HGPRTase with >1.5–2% enzyme activity.
Increased production of uric acid
• Associated with hyperuricemia and variable neurologic dysfunction.
Leukemia, Lymphoma.
Adenine Phosphoribosyltransferase (APRTase) Decreased excretion rate
m

Deficiency Renal failure, Thiazide diuretics, Lactic acidosis.


co

Autosomal recessive.
Clinical Features
Biochemical Defect • Acute Gouty Arthritis
• Deficiency APRTase Typically in the metatarsophalangeal joint of the big
• Adenine accumulates toe.
• Adenine oxidized by xanthine dehydrogenase to • Chronic Cases
2, 8-dihydroxyadenine, which is extremely insoluble. TophiQ deposits of monosodium urate crystals in the
Clinical manifestations subcutaneous tissue.
• Urinary calculus formation with crystalluria
Diagnosis
• The presence of brownish spots on the infant’s diaper
Aspiration and examination of synovial fluid
or of yellow-brown crystals in the urine is suggestive
• Negatively birefringent Q needle shaped mono-
m

of the diagnosis.
sodium urate crystals using polarized light micro-
co

Laboratory findings
scopy.
• Urinary levels of adenine, 8-hydroxyadenine, and
2, 8-dihydroxyadenine are elevated Treatment
• Plasma uric acid is normal. • Colchicine, an anti-inflammatory agent
e

e
m

m
m
co

246 |  
Self Assessment and Review of Biochemistry

• Uricosuric agents, such as probenecid or sulfinpy- Sources of Pyrimidine Ring


razone
• Allopurinol.
Allopurinol is converted in the body to oxypurinol
(alloxanthine), which inhibits xanthine oxidase.

Severe Combined Immunodeficiency (SCID)


m

Adenosine Deaminase (ADA) defect is one of the causes.


co

• Both B cells and T cells are affected


• First disorder to be treated by Gene TherapyQ
• Enzyme Replacement Therapy with Polyethylene
glycol modified bovine adenosine deaminase (PEG-
ADA).
Father of Gene Therapy ---- French Anderson Sources of Pyrimidine Ring
• N3 by Amide Nitrogen of Glutamine
Immunodeficiency in SCID is due to
• C2 by CO2
• Deficiency of ADA
• C4, C5, C6, N1 by Aspartate
• Adenosine accumulate
• Adenosine converted to its ribonucleotides and deoxyribonucleotides
m

(dATP) Steps of Pyrimidine Biosynthesis


• dATP inhibit ribonucleotidereductase
co

This pathway is similar to urea cycle.


• Decreases production of all deoxyribose containing nucleotides
• Hence, DNA synthesis decreased.
• Decrease in both T-cells and B-cells, Hence, immunodeficiency. Enzymes of Pyrimidine Synthesis
Purine Nucleoside Phosphorylase Defect • Catalyzed by multifunctional enzymes (Single
• Severe deficiency of T cells but apparently normal B-cell function polypeptide with more than one enzyme activity).
isQ.
• This is an enzyme in Purine Catabolism.
i. CPS II, Aspartate Transcarbamoylase, Dihydro
Orotase (CAD) in the Cytoplasm
Comparison of Immunodeficiency in SCID and Purine ii. Dihydroorotate Dehydrogenase
Phosphorylase deficiency ‒ Only mitochondrial step in Pyrimidine Synthesis.
iii. Orotate Phosphoribosyltransferase and OMP
Decarboxylase (Orotidylic acid Decarboxylase)
m

‒ Together called UMP Synthase


co

‒ Seen in the Cytoplasm.

Important Points of Pyrimidine Synthesis


Fig. 8.6: Immunodeficiency in SCID and purine phosphorylase In de novo Purine Synthesis amphibolic intermediates are
deficiency added on Ribose 5 Phosphate. Hence, Purine Nucleotide
is the end product. But in Pyrimidine Synthesis,
Xanthine Oxidase Deficiency amphibolic intermediates are not added on a ribose 5
• Genetic defect in Xanthine Oxidase enzyme phosphate. Hence, Pyrimidine base is synthesized rather
• Associated with Hypouricemia than Pyrimidine nucleotide.
• Xanthine crystals in urine On Pyrimidine base Ribose and Phosphate is added
• Xanthine lithiasis. by Orotate Phosphoribosyl Transferase (OPRTase), OMP
m

is formed.
PYRIMIDINE BIOSYNTHESIS
co

Synthesis of Uridine Nucleotides


• Site mainly liver • OMP is decarboxylated to UMP, the first Pyrimidine
• Organelle Cytoplasm and Mitochondria. nucleotide by OMP Decarboxylase.
e

e
m

m
m
co

Chemistry and Metabolism of Nucleotides  | 247

Synthesis of Cytidine Nucleotides • dUDP is converted to dUMP


• UTP converted to CTP by CTP synthase • dUridine MonophosphateQ (dUMP) to Thymidine
• Amino group is donated by Glutamine. Monophosphate (TMP) by Thymidylate Synthetase
• N5 N10 Methylene THFA donates the methyl group
Synthesis of Thymine Nucleotides for thymidine Monophosphate (TMP)
• First UDP is converted to dUDP by ribonucleotide • The only reaction in pyrimidine Synthesis where a
m

reductase (explained later) Tetrahydrofolate derivative is needed.


co
m
co
m
co

Two Anticancer Drug Inhibit the Synthesis of • Dihydrofolate Reductase convert Dihydrofolate to
m

TMP Tetrahydrofolate
co

• Hence, TMP Synthesis is affected.


Methotrexate 5 Fluorouracil
• Inhibit Dihydrofolate Reductase • Competitively inhibit Thymidylate Synthase.
e

e
m

m
m
co

248 |  
Self Assessment and Review of Biochemistry

Regulation of Pyrimidine Synthesis Clinical Correlation: Pyrimidine Metabolism


• CPS II, Aspartate Transcarbamoylase, Dihydro Orotic Acidurias
orotase (CAD) is the primary focus of regulation of • Rare autosomal recessive condition
Pyrimidine Synthesis • The most common metabolic error in the de novo
• Expression of CAD gene is controlled at genetic level synthesis of pyrimidines.
• CPS II is activated by PRPP and is feedback inhibited Biochemical defect
m

by UTP. • Type I Orotic Aciduria


co

Deficiency of both orotate phosphoribosyltransferase


Differences between CPS-I and CPS-II and OMP decarboxylase (together called UMP
CPS-I CPS-II Synthase).
Cellular location Mitochondria Cytosol
• Type II Orotic Aciduria
Deficiency only of orotidylate decarboxylase
Pathway involved Urea cycle Pyrimidine synthesis
Clinical Features of Orotic Acidurias
Source of nitrogen Ammonia Amide nitrogen of
glutamine Manifest in the first year of life and is characterized by
Allosteric regulators Activator-N-Acetyl Activator-PRPP
growth failure, developmental retardation, megaloblastic
Glutamate Inhibitor-UTP anemia, and increased urinary excretion of orotic acid.
No inhibitors Diagnosis of Orotic Aciduria
• The diagnosis of this disorder is suggested by the
m

Conversion Ribonucleotides to presence of severe megaloblastic anemia with normal


co

Deoxyribonucleotides serum B12 and folate levels and no evidence of TC-II


• Forms deoxyribonucleoside diphosphates (dNDPs) deficiency.
from ribonucleoside diphosphateQ • A presumptive diagnosis is made by finding
increased urinary orotic acid.
• Ribonucleotide reductase complex catalyses the
reaction • Confirmation of the diagnosis, however, requires
assay of the transferase and decarboxylase enzymes
• Reduction requires thioredoxin, Qthiore-doxin in the patient's erythrocytes.
re-ductase, and NADPH.
• The anemia is refractory to vitamin B12 or folic acid
but responds promptly to administration of uridine.
Reye Syndrome and Orotic Aciduria
• Inability of severely damaged mitochondria to utilize carbamoyl
phosphate,
m

• Exits to the cytosol, where it stimulates pyrimidine nucleotide


co

biosynthesis.
Urea Cycle Disorder and Orotic Aciduria
• Deficiency in liver mitochondrial ornithine transcarbamoylase .
• Excess carbamoyl phosphate in the mitochondria
• Exits to the cytosol, where it stimulates pyrimidine nucleotide
biosynthesis.

Drugs may Precipitate Orotic Aciduria


Fig. 8.7: Formation of deoxyribonucleosides • Allopurinol is an alternative substrate for orotate
phosphoribosyltransferase competes with orotic acid.
Catabolism of Pyrimidines • 6-Azauridine, following conversion to 6-azauridylate,
• Cytosine and Uracil to Beta Alanine CO2, NH3 also competitively inhibits orotidylate decarboxylase
• Thymine to β–aminoisobutyrate CO2, NH3 enhancing excretion of orotic acid and orotidine.
m

‒ Unlike the end products of purine catabolism, Antimetabolites used in Cancer Chemotherapy
the end products of pyrimidine catabolism are
co

Compounds with structural similarity to precursors


the highly water soluble. of purines or pyrimidines (Synthetic Nucleotides), or
‒ Hence, pyrimidine overproduction results in few compounds that interfere with purine or pyrimidine
clinical signs or symptoms. synthesis are called antimetabolites.
e

e
m

m
m
co

Chemistry and Metabolism of Nucleotides  | 249

• Methotrexate inhibits dihydrofolate reductase • 5-Fluorouracil (5FU)Q inhibits thymidylatesynthetase


competitively. • 6 Aza-cytidine—inhibits thymidylate synthetase
• Hydroxyurea inhibits ribonucleotide reductase • 6 Aza-uridine—inhibits thymidylate synthetase
• 6-Mercaptopurine-Purine analog • Cytosine Arabinoside—Ribose is replaced by
• 6-Thioguanine-Nucleotide analog Arabinose.
• 6-Azaguanine—Nucleotide analog
m

• Synthetic nucleotide used in the treatment of Gout—Allopurinol.


• Aza Serine—Glutamine Analog • Synthetic nucleotide used in the treatment of AIDS—Zidovudine.
co

• Diazenorleucine—Inhibits PRPP Glutamyl Amido- • Synthetic nucleotide used in the treatment of Herpetic Keratitis—
transferase 5-iodo-deoxyuridine.
• Mycophenolic AcidQ—Potent Reversible uncompeti- • Synthetic nucleotide used to suppress immunologic rejection
tive inhibitor of IMP Dehydrogenase. during organ transplantation—Azathioprine.

REVIEW QUESTIONS

CHEMISTRY OF NUCLEOTIDES Ribonucleotides

1. Nucleoside is made up of: (PGI Nov 2010) Nitrogenous Pentose Nucleoside Nucleotide
Base Sugar
a. Pyrimidine
m

Adenine Ribose Adenosine Adenosine Mono-


b. Histone
co

phosphate (AMP)
c. Sugar Guanine Ribose Guanosine Guanosine Mono-
phosphate (GMP)
d. Purine
Cytosine Ribose Cytidine Cytidine Mono-
e. Phosphate phosphate (CMP)
Ans. a, c, d. Pyrimidine, Sugar, Purine Uracil Ribose Uridine Uridine Mono-
(Ref: Harper 30/e page 340) phosphate (UMP)
Hypoxanthine Ribose Inosine Inosine Mono-
Composition of Nucleoside Phosphate (IMP)
• Nitrogenous base + Pentose Sugar Xanthine Ribose Xanthosine Xanthosine Mono-
• C1 of ribose or deoxyribose sugarto N1 of Pyrimidine Phosphate (XMP)
or N9 of Purine by β N Glycosidic linkage. Deoxyribonucleotides
Adenine Deoxy- dAdenosine dAdenosine Mono-
m

Composition of Nucleotides ribose phosphate (dAMP)


• Nucleoside + Phosphoryl groups
co

Guanine Deoxy- dGuanosine dGuanosine Mono-


• Usually phosphoryl group is attatched to 5’ hydroxyl ribose phosphate (dGMP)
group of pentose sugar by an ester bond. Cytosine Deoxy- dCytidine dCytidine Mono-
ribose phosphate (dCMP)
• Additional phosphoryl groups are attached by
Thymine Deoxy- Thymidine Thymidine Mono-
an Acid anhydride bond to form Nucleoside ribose Phosphate
diphosphates and triphosphates
3. Apart from occurring in nucleic acid, pyrimidines
2. The followings correctly arranged: are also found in: (AIIMS Nov 05)
(PGI June 2009) a. Theophylline
a. GMP-Guanine monophosphate b. Theobromine
b. UMP-Uracil monophosphate c. Flavin mononucleotide
c. TMP-Thymine monophosphate d. Thiamine
m

d. CMP-Cytidine monophosphate Ans. d. Thiamine


co

e. AMP-Adenine monophosphate • Vitamin with pyrimidine ring is Thiamine


Ans. d. CMP-Cytidine monophosphate • Theophylline and Theobromine has Purine ring
(Ref: Harper 30/e page 341) • FMN also contains Purine ring.
e

e
m

m
m
co

250 |  
Self Assessment and Review of Biochemistry

4. Which of the following is not a nitrogenous base? Reduction of the 2’-hydroxyl of purine and pyrimidine
a. Adenine ribonucleotides, catalyzed by the ribonucleotide reductase
b. Guanosine complex, provides the deoxyribonucleoside diphosphates
c. Cytosine (dNDPs) needed for both the synthesis and repair of DNA.
d. Thymine
Ans. b. Guanosine (Ref: Harper 30/e p 341 table 32-1)
m

• Guanosine is nucleoside.
co

5. Which is not found in DNA: (AI 1994)


a. Adenine
b. Thymine
c. Guanine
d. Uracil
Ans. d. Uracil (Harper 30/e page 360)
• Uracil is found only in the RNA.

6. At the physiological pH, the DNA molecules are:


9. What is involved in formation of d-TMP from
(AIIMS Nov 02)
d-UMP? (PGI June 07)
a. Positively charged
a. N5, N10-methylene tetrahydrofolate
m

b. Negatively charged
b. From iminofolate
co

c. Neutral
c. N5 formylfolate
d. Amphipathic
d. Dihydrofolate
Ans. b. Negatively charged
Ans. a. d. N5N10 methylene THF and DHF
• DNA is negatively charged because of Phosphate
(Harper 30/e page 353)
group.
When TMP is formed from dUMP, N5 N10 Methylene
Metabolism of Purines and Pyrimidines THFA is converted to Dihydrofolate
7. End product of purine metabolism in non-primate 10. Inosinic acid is biological precursor:
mammals is: (AIIMS May 2008) (Nimhans 97, JIPMER 04)
a. Uric acid a. Uracil and thymine
b. Ammonia b. Purines and thymine
c. Urea
m

c. Adenylic acid and guanylic acid


d. Allantoin
co

d. Orotic acid and uridylic acid


Ans. d. Allantoin (Ref: Harper 30/e page 354) Ans. c. Adenylic acid and Guanylic acid
• Humans convert adenosine and guanosine to uric (Harper 30/e page 350)
acid
• AMP and GMP is derived from IMP (Inosinic acid)
• In mammals other than higher primates, uricase
converts uric acid to the water-soluble product 11. False regarding gout is: (AI 2001)
allantoin. a. Due to increased metabolism of pyrimidines
• Humans lack uricase, the end product of purine b. Due to increased metabolism of purines
catabolism in humans is uric acid. c. Uric acid levels may not be elevated
d. Has a predilection for the great toe
8. Deoxyribonucleic acid is formed from:
a. Ribonuclease Ans. a. Due to increased metabolism of pyrimidines.
Remember Uric acid level may not be elevated always in gout.
m

b. Ribonucleotide monophosphate
co

c. Ribonucleotide diphosphate 12. The enzyme deficient in Lesch-Nyhan syndrome


d. Rubonucleotide triphosphate is: (PGI June 99)
Ans. c. Ribonucleotide diphosphate a. GTRT
(Ref: Harper 30/e page 352) b. Glutaminase
e

e
m

m
m
co

Chemistry and Metabolism of Nucleotides  | 251

c. Transcarboxylase c. Adenosine deaminase


d. HGPRTase d. APRtase
Ans. d. HGPRTase (Harper 30/e page 350) Ans. b. Xanthine oxidase (Harper 30/e page 355)
Hypouricemia
13. A 10-year-old child presents with history of rashes
self mutilation family history positive which of • Hypouricemia and increased excretion of hypox-
the following investigations do you think may be anthine and xanthine are associated with xanthine
m

suggestive of valuable for diagnosis: (AI 2012) oxidase deficiency.


co

a. Lead Lesch–Nyhan Syndrome


b. Alkaline phosphatase • The Lesch–Nyhan syndrome, an overproduction
c. L D H hyperuricemia characterized by frequent episodes
d. Uric acid of uric acid lithiasis and a bizarre syndrome of self-
mutilation, reflects a defect in hypoxanthine-guanine
Ans. d. Uric acid (Harper 30/e page 350)
phosphoribosyltransferase, an enzyme of purine
Lesch-Nyhan Syndrome salvage.
X-linked Recessive Disorder Adenosine Deaminase Deficiency
Biochemical Defect • Adenosine deaminase deficiency is associated
• Complete deficiency of HGPRTase with an immunodeficiency disease in which both
• Purine accumulates thymus-derived lymphocytes (T cells) and bone
m

• Purines degraded to uric acid marrow-derived lymphocytes (B cells) are sparse


and dysfunctional. Patients suffer from severe
co

• Uric acid level increases.


immunodeficiency.
Clinical Features
Purine Nucleoside Phosphorylase Deficiency
• Hyperuricemia, Intellectual disability, Dystonic
movement, Choreoathetosis, Dysarthric speech. • Purine nucleoside phosphorylase deficiency is
associated with a severe deficiency of T cells but
• Compulsive self-mutilation.
apparently normal B cell function.
Diagnosis
16. Choose the incorrect statement. Lesch-Nyhan
• Hyperuricemia
Syndrome: (Kerala 2015)
• HGPRTase Enzyme activity in the Erythrocytes is
a. Affects young boys
deficient.
b. Presents with gouty arthritis
Treatment c. The enzyme defect enhances the reutilization
• Allopurinol. of purine bases
m

d. Bizarre behavior of self-mutilation


co

14. A ten-year-old child with aggressive behavior and


poor concentration is brought with presenting Ans. c. The enzyme defect enhances the reutilization of
complaints of joint pain and reduced urinary purine bases (Harper 30/e page 354)
output. Mother gives history of self-mutilate his Lesch-Nyhan Syndrome is characterized by
finger. Which of the following enzymes is likely • X-linked recessive inheritance
to be deficient in this child: (AI 2009) • Over production hyperuricemia
a. HGPRTase • Frequent episodes of uric acid lithiasis
b. Adenosine deaminase • Bizarre syndrome of self-mutilation.
c. APRTase • Defective reutilzation of Purine bases because of
d. Acid maltase absence of HGPRTase enzyme.
Ans. a. HGPRTase (Harper 30/e page 350) 17. Hyperuricemia is not found in:
m

15. A patient with increased Hypoxanthine and a. Cancer


b. Psoriasis
co

Xanthine in blood with hypouricemia which


enzyme is deficient? c. Von Gierke’s disease
a. HGPRTase d. Xanthinuria
b. Xanthine oxidase Ans. d. Xanthinuria (Harper 30/e page 355)
e

e
m

m
m
co

9 Structure and Organization


of DNA
m
co

Topics Included
• Structure of DNA • Supercoiling of DNA
• Organization of DNA • Central Dogma of Molecular Biology
m

STRUCTURE OF DNA Different Types of DNA


co

There are 6 types of DNA.


Watson, Crick, and Wilkins to propose a model of a DNA
in 1953 based on the X-ray diffractionQ photographs of • A, B, C, D, E -are right handed
DNA taken by Rosalind Franklin. • Z is left handed
Characteristics A-DNA B-DNA Z-DNA
Salient Features of Watson–Crick Model of DNA
Number of base pairs 11 10 12
1. Right-handed double stranded DNA helix per turns
2. Base Pairing Rule Morphology Broad and Longer and Elongated
‒ Adenine always pairs with Thymine short thinner and thin
‒ Guanine pairs with Cytosine. Base pair tilts the axis 20º tilt Base pair 9º tilt
of helix perpendicular
3. Two strands are antiparallel. to helix
The polarity of DNA is such that Screw sense Right handed Righ handed Left handed
m

‒ One strand runs in 5’ to 3’ direction


co

‒ Other strand runs in 3’ to 5’ direction.


A-DNA
4. Hydrogen Bonding
• X-ray diffraction studies on dehydrated DNA fibers
‒ Adenine pair with Thymine by 2 Hydrogen
revealed A form, called A-DNA.
bonds. (A = T)
• A-DNA is found in conditions of low humidity and
‒ Guanine pair with Cytosine by 3 Hydrogen
high salt concentration.
bonds (G = C)
• Right-handed double helix like B-DNA.
5. Grooves of the DNA
They are two types: B-DNA
1. Major groove • Physiologically most common form
2. Minor groove • Right-handed double helix
- Grooves often act as sites of DNA–Protein • B-DNA is found in conditions of high humidity and
interaction needed for regulation of gene
m

low salt concentration


expression.
co

• Highly flexible.
- The DNA Protein interaction is via hydropho-
bic interaction and ionic bond. Z–DNA
Chargaff’s Rule—The number of Purines = The number of Pyrimidines • Phosphodiester backbone assume a zig-zag form
e

e
m

m
m
co

Structure and Organization of DNA   | 253

• Left-handed double helix • Poly (dA) and Poly (dT) strands combine to form
• Seen in the 5’ end of chromosomes triple stranded DNA.
• Longer and thinner than B-DNA
• 12 bp per turn
Four Stranded DNA
• Particularly seen in sequence of alternating purine • Four Stranded Structure formed in DNA high in
and pyrimidine guanine content
m

• Particularly in d(GC)n sequence • Ends of eukaryotic chromosomes (telomeres) contain


Guanine-rich sequences
co

• Methylation of Guanine and Cytosine residues


stabilizes Z-form • A base pairing scheme for parallel four-stranded
• Sequences that are not strictly alternating purine and DNA, referred to as a G-quartet DNAQ
pyrimidine also form Z DNA on methylation • Hoogsteen pairs are seen in four stranded DNA also.
• Z-DNA influences gene expression and regulation. Methods to separate DNA
• DNA can be separated by HPLC, thin layer chromatography (TLC),
Points to Ponder
Paper Chromatography and Gel electrophoresis.
• Physiologically most common is B-DNA.Q
• Under low salt and high degree of hydration B-DNA is usually found
• Under high salt concentration and low degree of hydration A-DNA Denaturation of DNA (Melting of DNA)
is usually found.
• The process by which two strands are separated into
• The distance spanned by one turn of B DNA is 3.4 nm (34 A0)
• The width of the double helix in B-DNA is 2 nm (20A0) component strands.
m

Features of Denaturation
co

• Breaking of Hydrogen bonds


• Phosphodiester bond is not broken
• Primary structure not altered, only secondary and
tertiary structure altered
• Viscosity decreasesQ
• Increase in the optical absorbance of UV light by Purine
and Pyrimidine bases, called hyperchromicity.Q
Melting Temperature (Tm)
The strands of a given molecule of DNA separate over
a range of temperature. The midpoint is called melting
m

temperature.
co

Factors Influencing Tm
1. Base Composition.
‒ More GC pairs more the Tm.
2. Salt concentration.
‒ 1 0 - f o l d i n c r e a s e o f m o n o va l e n t c a t i o n
concentration increases the Tm by 16.6°C.
Fig. 9.1: Structure of DNA
3. Formamide Q destabilize hydrogen bond, hence
NONCANONICAL DNA STRUCTURES decreases Tm.
Application of Denaturation of DNA
Triple-stranded DNAQ
• Measurement of increased optical absorbance at
• Triple-stranded DNA is generated by the hydrogen
m

bonding of a third strand into the major groove of 260 nm is an indication of denaturation of DNA
co

B-DNA • In recombinant DNA Technology


• The third strand forms hydrogen bonds with • Renaturation following denaturation obeying base
another surface of the double helix through so-called pairing rule is applicable in various hybridization
Hoogsteenpairs.Q and blotting techniques.
e

e
m

m
m
co

254 |  
Self Assessment and Review of Biochemistry

ORGANIZATION OF DNA Nucleosome at a Glance


• No of turns of DNA on Histone octamer—1.75 turns
• Genome in the prokaryotes are loosely packed
Q
• Direction of turn of the DNA over Histone—left handed.
• In eukaryotes DNA is well organized inside the • No of base pairs in 1.75 turn of DNA—145-150 bp
nucleus. • Diameter of nucleosome—10 nm
• No of base pairs in the linker DNA—30 bp
Levels of Organization of DNA
m

• DNA double helix


• 10 nm chromatin fibril HISTONES
co

• 30 nm chromatin fibril
• Nuclear Scaffold form (Interphase Chromosome) • Most abundant chromatin Protein
– Noncondensed loop • Small family of closely related basic proteins
– Condensed loop • The carboxyl terminal two-third is hydrophobic,
• Metaphase Chromosome while amino terminal one-third is rich in basic amino
acids like arginine and lysine
DNA Double Helix • Core histones are subject to at least six types of post-
• First level of organization of DNA translational modifications
• The characteristics are same as that of Watson-Crick • They are highly conserved among the species.
model of DNA Histones are divided into:
• Diameter is 2 nm. 1. Core Histones
2. Linker Histones
m
co

Core Histones
• Core histones are H2A, H2B, H3, and H4
• They form histone octamer
Fig. 9.2: DNA double helix
• H3 and H4 forms tetramer, while H2A and H2B form
dimers
10 nm Chromatin Fibril • (H 3-H 4) 2 tetramer associate with two (H 2A-H 2B)
• Consist of nucleosomes separated by linker DNA dimers to form histone octamer
• Nucleosome is a nucleoprotein complex • H2A and H2B are ArginineQ rich
• DNA double helix is wrapped nearly twiceQ (exactly • H3 and H4 are Lysine rich.
1.75 times) over a histone octamer in left-handed helix
Linker Histones
to form a disk like structure
• H1 histone which is seen in the linker region
• Individual nucleosome are linked together by 30 bp
m

• This is loosely boundQ to nucleosome.


segment called linker
co

• This gives a Beads on a String appearanceQ on Nonhistone Proteins


electron microscopy. • They include enzymes involved in DNA replication
and repair, RNA synthesis and processing
• Unlike histones, nonhistone proteins are acidic
• They are larger than histone proteins.
30 nm Chromatin Fibril (Solenoid)
Groups of nucleosome form ‘DNA fibril’.
6 such DNA fibrils form 30 nm chromatin fibril.
m
co

Fig. 9.3: Structure of nucleosome Fig. 9.4: Structure of chromatin fibril


e

e
m

m
m
co

Structure and Organization of DNA   | 255

Nuclear Scaffold Associated Form or Interphase Negative Supercoils


Chromosome Circular DNA twisted in the direction opposite from the
• 30,000 to 100,000 bp loops or domains anchored in clockwise turns of the right-handed double helix creates
a scaffolding or supporting matrix, called nuclear negative supercoils or right-handed helix. Such DNA is
matrix. said to be underwound.
• Loops can be a condensed loop or non-condensed
Functions of Supercoils
m

loops.
• Supercoiling promotes packing of DNA into compact
co

EuchromatinQ and HeterochromatinQ structures


Euchromatin • Helps to generate regions with broken hydrogen
• Chromatin is less densely packedQ bonds which facilitate DNA strand separation and
• Transcriptionally activeQ facilitate replication, repair and recombination of
• Chromatin stains less densely.Q the DNA.

Heterochromatin Topoisomerase
• Chromatin is densely packedQ
• Nicking Resealing Enzyme
• Transcriptionally inactiveQ
• Enzymes that can relax or insert supercoils
• Chromatin stains densely.Q
• Enzymes that relieve torsional strains in the DNA.
Two types of Heterochromatin
m

1. Constitutive Heterochromatin Topoisomerases can be of two types: Type I and Type II


co

‒ Always condensed Topoisomerase Type I


‒ Essentially inactive • Make transient single strandedQ break in a negatively
‒ Seen in Centromere and chromsomal ends of the supercoiled DNA double helix
telomere. • Phosphodiester bond is interrupted
2. Facultative Heterochromatin
• Topoisomerase remain bound to the phosphoryl
‒ It is at times condensed, but at other times it is
group at incision site by a covalent bond, till the nick
uncondensed and actively transcribed.
is resealed
For example: One of the X chromosome in mammalian
female. The heterochromatic X chromosome • ATP is not needed.Q
decondenses during gametogenesis. There are two classes of Topoisomerases I
• Type IA and Type IB Topoisomerase
m

SUPERCOILING OF DNA
Type IA the tyrosine residue of the Topoisomerase is
co

• DNA can be in relaxed or Superhelical bound to 5’ phosphoryl group of nucleotide.


• Linear B-DNA is relaxed
Type IB the tyrosine residue of the Topoisomerase
• This is thermodynamically most favored, but is bound to 3’ phosphoryl group of nucleotide.
biological activity of relaxed DNA, like replication,
Topoisomerase Type II
repair, etc. is reduced
• The biologically active form is superhelical, but it is • Dimeric enzyme that bind to double stranded DNA,
topologically strained isomer. make breaks in both the strandsQ of DNA
• Can insert and remove supercoils
Supercoils can be divided into:
1. Positive Supercoils • ATP is needed.Q
2. Negative Supercoils.
Remember
m

Positive Supercoils • Bacterial DNA GyrasesQ is a subset of Topoisomerases type II


co

Circular DNA twisted in the direction same as that • All Topoisomerases type II relaxes supercoils in the DNA
original rotation creates positive supercoils or left-handed • Bacterial DNA Gyrases are the only subset of type II topoisomerases
Superhelix. Such DNA is said to be overwound. that can add negative supercoils.
e

e
m

m
m
co

256 |  
Self Assessment and Review of Biochemistry

Some Important Prokaryotic Topoisomerases • Daunorubicin


• Idarubicin
Prokaryotic topoiso-
merases Types Functions
E coli Topoisomerase I IA Relaxes negatively supercoiled CENTRAL DOGMA OF MOLECULAR BIOLOGY
DNA
Three process involved in Central Dogma of molecular
E coli Topoisomerase III IA Relaxes negatively supercoiled
biology are:
m

DNA
1. DNA replication (DNA to DNA)
co

E coli Topoisomerase IV II Relaxes negatively supercoiled


DNA 2. Transcription (DNA to RNA)
E coli DNA Gyrase II Introduces negative supercoils. 3. Translation (RNA to protein)
Relaxes either positive or nega-
tive supercoils

Some Important Eukaryotic Topoisomerases


Eukaryotic
topoisomerase Types Functions
Eukaryotic DNA Topoi- IB Relaxes either negatively or
somerase I positively coiled DNA
Eukaryotic DNA Topoi- II Relaxes either negatively or
m

somerase II positively coiled DNA


co

Eukaryotic DNA Topoi- IA Possible role in recombination


somerase III

Remember
The numbering given in the name of topoisomerase and the number-
ing in the types of Topoisomerases are different: For example, E coli
Topoisomerase III belongs to Type IA Topoisomerase. Fig. 9.5: Central dogma of molecular biology
• Almost all E coli Topoisomerases relaxes negative supercoils.
• Only E coli DNA Gyrase introduces negative supercoils. Current Central Dogma of Molecular Biology
• All Eukaryotic DNA Topoisomerases relax negative and positive With the advances made by Human Genome Projects, the
supercoils. central dogma of molecular biology is changed.
m

Bacterial Topoisomerases Inhibitors


co

1. Nalidixic Acid • Genome: The complete set of genes of an organism.


2. Fluoroquinolones • Transcriptome: The complete set of RNA transcripts
3. Novobiocin. produced by the genome of an organism.
• Proteome: The complete complement of proteins of
Human Topoisomerase Inhibitors
an organism.
Used in Cancer Chemotherapy.
Human Topoisomerases that inhibit Type I Topoisomerases Numerals in Molecular Genetics
• Irinotecan • Total number of chromosome in humans is 46 (23 pairs)
• Topotecan • The number of base pairs in haploid set of chromosome is 3.0 ×
109 bp (3 billion bp)
Human Topoisomerases that inhibit Type II Topoiso- • Percentage of exons in human genome is approximately 1.14%
merases
m

(1.5–2%)
• Etoposide • The number of protein coding genes in human genome is 20,687.
co

• Adiramycin (Doxorubicin) • The genes account for 10–15% of DNA.


e

e
m

m
m
co

Structure and Organization of DNA   | 257

REVIEW QUESTIONS

STRUCTURE OF DNA Hydrogen Bonding:


• Adenine pair with Thymine by 2 Hydrogen bonds.
1. Which of the following is false? (PGI 2002) (A = T)
m

a. Ratio of A:T and G:C is approximately equal to • Guanine pair with Cytosine by 3 Hydrogen bonds
co

1:1 (G = C)
b. Ratio of A:G and T:C is approximately equal to Grooves of the DNA
1:1
There are two types:
c. A + T = G + C • Major groove
d. A + C = G + T • Minor groove
e. A + G = C + T Grooves act as sites of DNA-Protein interaction needed for
Ans. b, c, d. Ratio of A:G and T:C ..., A + T = G + C, regulation of gene expression.
A+C=G+T
3. If a sample of DNA if adenine is 23% what will be
Based on Chargaff’s rule, Purines = Pyrimidines Adenine
the amount of guanine present (PGI May 2013)
pair with Thymine and Guanine with Cytosine, hence,
a. 23%
ratio of A:T & G:C is approximately equal to 1:1.
m

b. 25%
co

2. True about DNA structure: (PGI Nov 2010) c. 46%


a. Purines are adenine and guanine and d. 27%
pyrimidines are uracil and cytosine e. 54%
b. Watson and Crick discovered structure in 1973 Ans. d. 27%
c. Deoxyribose–phosphate backbone with bases Based on Chargaff’ rule:
stacked inside • Purines = Pyrimidine
d. Mainly consists of left-handed helix • No. of Adenine = No. of Thymine
e. 5’-3’ Phosphodiester bonding is present • So Adenine + Thymine = 46%
Ans. c. Deoxyribose-phosphate backbone with bases • Guanine + Cytosine = 54%
stacked inside (Ref: Harper 30/e page 359-362) • No. of Guanine = No. of Cytosine
Structure of DNA • So the amount of Guanine = 54/2 = 27%
m

Elucidated by Watson and Crick based on the X-ray


co

4. True statements about DNA structure:


diffraction picture taken by Rosalind Franklin in 1953 (PGI June 06)
and got Nobel prize in 1962.
a. All nucleotides are involved in linkage
Salient Features of Watson–Crick Model of DNA b. Antiparallel
Right-handed Double stranded DNA helix. c. Parallel
Purines are Adenine and Guanine and Pyrimidines d. Bases are perpendicular to DNA
are Cytosine and Thymine. e. Attached by hydrogen l bond
Based on Pairing Rule: Ans. a, b, d and e. All nucleotides are …, Antiparallel,
• Adenine always pairs with Thymine Bases are perpendicular … attatched by hydrogen bond
• Guanine pairs with Cytosine (Harper 30/e page 360)
Two strands are antiparallel.
m

5. The two strands of DNA are held together by:


co

The polarity of DNA is such that: (AIIMS Feb 97)


• One strand runs in 5’ to 3’ direction a. van der Waals bond
• Other strand runs in 3’ to 5’ direction. b. Hydrogen bond
e

e
m

m
m
co

258 |  
Self Assessment and Review of Biochemistry

c. Covalent bond c. A–G


d. Ionic interaction d. C–T
Ans. b. Hydrogen bond (Harper 30/e page 360) Ans. b. C-G
• Three hydrogen bonds between C and G
6. Which form of DNA is predominantly seen: • Two Hydrogen bonds between A and T.
(AI 1996)
10. Thermo-stability in DNA is contributed mostly
m

a. A
by: (AI 1996)
co

b. C
a. A = T
c. B
b. G = C
d. Z
c. Molecular base
Ans. c. B DNA (Harper 30/e page 360) d. Parallel arrangement
• Physiologically most common is B-DNAQ
Ans. b. G = C
• Under low salt and high degree of hydration B-DNA
Factors Influencing Tm
is usually found
1. Base Composition.
• Under high salt concentration and low degree of
More GC pairs more the Tm
hydration A-DNA is usually found
2. Salt concentration.
• The distance spanned by one turn of B-DNA is
10-fold increase of monovalent cation concentration
3.4 nm (34 A0)
increases the Tm by 16.6°C.
m

• The width of the double helix in B-DNA is 2 nm 3. Formamide Q destabilize hydrogen bond, hence
co

(20A0). decreases Tm.


7. Chargaff rule state that: (Bihar 98, MP 04, UP 03) 11. At the physiological pH, the DNA molecules are:
a. A + G = T + C (AIIMS Nov 02)
b. A/T = G/C a. Positively charged
c. A = U = T = G = C b. Negatively charged
d. A + T = G + C c. Neutral
Ans. a. A + G = T + C (Harper 30/e page 360) d. Amphipathic
Chargaff’s rule is total number of Purines = Total number Ans. b. Negatively charged
of Pyrimidines in a double stranded DNA. DNA is negatively charged because of Phosphate group.

8. A nucleic acid was analyzed and found to contain 12. Total number of genes in a human being is:
m

32% adenine, 18% guanine, 17% cytosine and 33% (Kerala 2001, CMC 04, WB 98)
co

thymine. The nucleic acid must be: a. 800,000


(AIIMS May 06) b. 50,000
a. Single-stranded RNA c. 100,000
b. Single-stranded DNA d. 30,000
c. Double-stranded RNA Ans. d. 30,000
d. Double-stranded DNA This is an old question, Now it is found that number of
Ans: d. ds DNA (Harper 30/e page 360) genes is approx 20,000.
The above nucleic acid is obeying Chargaff’s rule as the 13. Triplex DNA is due to: (AIIMS May 2011)
number of Purines (32 + 18) = No of Pyrimidines (17 + a. Hoogsteen pairing
33). Double stranded RNA is not the answer as it does b. Palindromic sequences
not contain Thymine.
c. Large no. of guanosine repeats
m

9. Triple bonds are found between which base pairs: d. Polypyrimidine tracts
co

(AI 2001) Ans. a. Hoogsteen pairing


a. A–T (Ref: Textbook of Biochemistry with clinical correlation
b. C–G Thomas M Devlin 7/e page 43)
e

e
m

m
m
co

Structure and Organization of DNA   | 259

NONCANONICAL DNA STRUCTURES • Current estimates predict 20,687 protein coding genes
Triple-stranded DNA • Exome constitutes 1.14% of genome
Triple-stranded DNA is generated by the hydrogen • SNPs estimated is 10 million.
bonding of a third strand into the major groove of B-DNA
17. Proteins seen in chromosomes are called:
Commonly seen in Polynucleotides, Poly (dA) and
(Ker 2006)
Poly (dT).
m

a. Nucleotides
The third strand forms hydrogen bonds with another
co

b. Histones
surface of the double helix through so-called Hoogsteen
pairs. c. Apoproteins
d. Glycoproteins
14. About DNA which of the following is true Ans. b. Histones (Ref: Harper 30/e page 371)
(Jipmer 2014)
Histones are the most abundant histone proteins.
a. The nucleotide of one strand form bonds with
nucleotide of opposite strand. 18. Euchromatin is the region of DNA that is
b. Cytosine and Uracil differ by one ribose sugar relatively: (AI 2006)
c. The information from DNA is copied in the a. Uncondensed
form of tRNA b. Condensed
d. Each nucleotide pair includes two purines. c. Overcondensed
m

Ans: a. The nucleotide of one strand form bonds with d. Partially condensed
co

nucleotide of opposite strand Ans. a. Uncondensed (Ref: Harper 30/e page 374)
• One strand af DNA join with other strand of DNA by Euchromatin and Heterochromatin
means of Hydrogen bond between the bases.
Euchromatin
• Cytosine and uracil differ by amino group.
• Chromatin is less densely packed
• The information in the DNA is copied in the form
• Transcriptionally active
of mRNA
• Chromatin stains less densely
• Each nucleotide pair includes one purine and one
pyrimidine. Heterochromatin
• Chromatin is densely packed
15. Which model of DNA was discovered by Watson • Transcriptionally inactive
and Crick? (NBE Pattern Q)
• Chromatin stains densely
a. A-DNA
m

Two Types of Heterochromatin


b. B-DNA
co

1. Constitutive Heterochromatin:
c. C-DNA
‒ Always condensed
d. Z-DNA
‒ Essentially inactive
Ans. b. B-DNA
‒ Seen in centromere and chromsomal ends of the
Organization of DNA telomere.
16. Total number of base pairs in human haploid set 2. Facultative Heterochromatin:
of chromosome (Ker 2007) ‒ Is at times condensed, but at other times it is
a. 3 million uncondensed and actively transcribed, e.g. one
of the X chromosome in mammalian female.
b. 3 billion
‒ The heterochromatic X chromosome decondenses
c. 33 billion
during gametogenesis.
d. 5 million
m

Ans. b. 3 billion (3 × 109) (Harper 30/e page 377) 19. The long and short arms of chromosomes are
co

(Harrison 19/e page 427) designated respectively as (AI 2006)


• Human haploid genome of each cell consist of 3 × 109 a. P and q arms
bp (3 billion bp) b. M and q arms
e

e
m

m
m
co

260 |  
Self Assessment and Review of Biochemistry

c. q and p arms DIFFERENT GROUPS OF CHROMOSOME


d. I and s arms
GROUPS CHROMOSOME DESCRIPTION
Ans. c. q and p arms
A 1–3 Largest; 1 & 3 Metacentric
(Emery’s Elements of Medical Genetics 13/e page 31) 2 -Submetacentric
B 4,5 Large Submetacentric
20. True about Histone Proteins (PGI May 2012)
m

a. Ribonucleoprotein C 6–12, X Medium size, Submetacentric


co

D 13–15 Medium size, Acrocentric with satellite


b. Present inside the nucleus
E 16–18 Small; 16 is metacentric but 17 & 18
c. Acidic are Submetacentric
d. Basic F 19–20 Small Metacentric
e. Glycoprotein G 21, 22, Y Small Acrocentric
Ans. b, d. Present inside .., basic
(Ref: Harper 29/e page 371)
Points to Ponder
• X Chromosome is Submetacentric (C)
Histones
• Y Chromosome is Small Acrocentric (G)
• Most abundant Chromatin Protein • Most common is Submetacentric
• Small family of closely related basic Proteins • Humans lack Telocentric Chromosome
There are 5 classes of Histones:
m

22. Nucleosome consists of: (PGI May 2010)


• H1, H2A, H2B, H3, and H4 a. Histone
co

Core Histones are: b. DNA


• H3 + H4 + H2A + H2B c. RNA
• They form Histone Octamer. d. DNA and RNA both
e. Carbohydrate
Linker Histones
Ans. a, b. Histone, DNA (Ref: Harper 30/e page 371)
• H1 histone which is seen in the linker region.
Nucleosome
• Loosely bound to nucleosome.
• Nucleoprotein complex
Nonhistone proteins
• DNA double helix is wrapped nearly twice over a
• Most of which are acidic and larger than histones histone octamer in left handed helix to form a disk
• The nonhistone proteins include enzymes involved like structure.
m

in DNA replication and repair, and the proteins • Individual nucleosome are linked together by 30 bp
co

involved in RNA synthesis, processing, and transport segment called linker. This gives a Beads on a String
to the cytoplasm. appearance on electron microscopy.

21. Y-chromosome is: (AIIMS May 2008) 23. Component of chromosome are: (PGI Dec 03)
a. DNA
a. Metacentric
b. tRNA
b. Submetacentric
c. mRNA
c. Acrocentric d. rRNA
d. Longer than the X-chromosome e. Histones
Ans. c. Acrocentric Ans. a, e. DNA, Histones (Harper 30/e page 371)
(Ref: Emery’s Elements of Medical
24. The protein rich in basic amino acids, which func-
Genetics page 30, 31)
m

tions in the packaging of DNA in chromosome, is:


Karyotype (AI 2003)
co

Arrangement of chromosome in the decreasing order of a. Histones


length. b. Collagen
e

e
m

m
m
co

Structure and Organization of DNA   | 261

c. Hyaluronic acid binding protein that is retained in the nucleus, where it ‘coats’ the X
d. Fibrinogen chromosome that it is transcribed from and initiates a gene-
Ans. a. Histones (Harper 30/e page 371) silencing process by chromatin modification and DNA
methylation. The XIST allele is switched off in the active X.
25. Random inactivation of X chromosome is: 26. In the entire genome, the coding DNA constitutes
a. Lyonization how much: (AIIMS 2014 May)
m

b. Allelic Exclusion a. 0.1


co

c. Randomization b. 0.02
d. Genomic imprinting c. 0.25
Ans. a. Lyonization d. 0.4
Two factors that are peculiar to the sex chromosomes: Ans. b. 0.02 In the entire genome, the coding DNA
(1) lyonization or inactivation of all but one X chromosome constitutes 1.5–2% (approx 1.14% according to Harrison
and (2) the modest amount of genetic material carried by 19/e and Harper 30/e).
the Y chromosome.
27. True about DNA hyperchromatism
In 1961, Lyon outlined the idea of X-inactivation, now (PGI Nov 2013)
commonly known as the Lyon hypothesis. It states that a. It is increase of absorbance
(1) only one of the X chromosomes is genetically active,
b. Measured by absorbance at 260 nm (in a
(2) the other X of either maternal or paternal origin
spectrophotometer)
undergoes heteropyknosis and is rendered inactive,
m

(3) inactivation of either the maternal or paternal X occurs c. It occurs when the DNA duplex is denatured
co

at random among all the cells of the blastocyst on or about d. Double stranded DNA is more hyperchromic
day 16 of embryonic life, and (4) inactivation of the same than ssDNA
X chromosome persists in all the cells derived from each Ans. a, b and c. It is increase of absorbance, Measured by
precursor cell. absorbance at 260 nm (in a spectrophotometer), it occurs
The inactive X can be seen in the interphase nucleus as when the DNA duplex is denatured.
a darkly staining small mass in contact with the nuclear During denaturation of DNA, there is increased in
membrane known as the Barr body, or X chromatin. absorbance at 260 nm, measured by Spectrophotometry.
The molecular basis of X inactivation involves a unique This is called hyperchromicity.
gene called XIST, whose product is a noncoding RNA ss DNA is more hyperchromic than ds DNA.
m
co
m
co
e

e
m

m
m
co

10 DNA Replication
m
co

Topics Included
• Definition • Steps of DNA Replication
• Enzymes Involved in the DNA Replication • DNA Repair Mechanisms
• DNA Polymerases
m
co

DNA REPLICATION • Helicase: ATP driven processive unwinding of DNA


• Single Strand Binding (SSB) protein prevents
Definition premature reannealing of ds DNA.
The process by which copying of base sequence present • DNA Primase:
in the parent strand to daughter strand, thereby passing
‒ Initiates synthesis of RNA primers
the genetic information from parent to progeny is called
Special class of DNA dependent RNA Polymerase
Replication.
• DNA Polymerase: Catalyse the chemical reaction of
Salient Features of DNA Replication DNA Polymerization. Synthesize DNA only in 5’ to
• Occurs in the S Phase of the cell cycle 3’ direction
• DNA strands separate and each acts as template • DNA Ligase: Seals the single strand nick between
strand on which complementary strand is synthesized the nascent chain and Okazaki fragments on lagging
m

• Base pairing rule is obeyed strand.


co

• Semiconservative nature-Proved by Meselson and


Stahl Experiment DNA POLYMERASESQ
‒ Half of the parent strand is conserved in the • Enzymes that catalyse Deoxyribonucleotide
daughter DNA polymerization
• New Strand is synthesized always in 5’ to 3’ direction • The initiation of DNA synthesis by DNA Polymerase
• Overall DNA replication is bidirectional always require priming by a short length of RNA,
• Synthesis of DNA in both strands are not similar called Primer.
‒ Leading Strand: The strand which DNA is
Three important properties of DNA Polymerase
continuously polymerized
Complex
‒ Lagging Strand: The strand which is DNA is
1. Chain elongationQ: Chain elongation accounts for
discontinuously polymerized.
the rate (in nucleotides per second; nucleotides/s) at
m

Enzymes Involved in the DNA Replication which polymerization occurs.


co

• Topoisomerases Rate of chain elongation of DNA Pol III is 20-50


‒ Relieve torsional strain that results from helicase- nucleotides/second.
induced unwinding of DNA 2. Processivity Q: Processivity is an expression of
‒ Nicking Resealing Enzyme the number of nucleotides added to the nascent
e

e
m

m
m
co

DNA Replication   | 263

chain before the polymerase disengages from the • DNAP γ


template. Processivity of DNA Pol III is 100 to >50,000 • DNAP δ
nucleotides. • DNAP ε
3. ProofreadingQ: The proofreading function identifies
Eukaryotic DNA polymerase Function
copying errors and corrects them.
DNAP alpha Primase
• Proofreading function needs 3’ to 5’ exonuclease activity
DNAP beta DNA repair
m

• Repair function needs 5’ to 3’ exonuclease activity.


DNAP gamma Mitochondrial DNA synthesis
co

DNAP delta Lagging strand synthesis


Prokaryotic DNA Polymerase
DNAP epsilon Leading strand synthesis
Three types of Prokaryotic DNA Polymerase
• Pol I Remember
• Eukaryotic DNA Polymerase involved in DNA repair are DNA
• Pol II
Polymerase delta (δ) and eta (η).
• Pol III
Prokaryotic DNA
polymerase Function
Steps of DNA Replication
• Identification of the origins of replication
Pol I Removal of Primers and Gap filling on lagging
strand Fixed points on the chromosome where replication
DNA Proofreading begins are called Ori
m

DNA Repair ‒ In E coli-ori C


Recombination
co

‒ In Bacteriophage λ-ori λ
Pol II DNA proofreading and repair
‒ In Yeast-Autonomous Replicating Sequence
Pol III Processive, leading strand synthesis (ARS)
Synthesis of Okazaki fragments
‒ In humans similar to Yeast
Single ori in bacteria
Role of DNA Sliding Clamp
Multiple ori present in eukaryotes
DNA Polymerase III associate with two identical β
subunits of DNA Sliding ‘clamp’ which increases the There is an AT rich sequence adjacent to ori facilitating
Pol III–DNA stability, processivity and rate of chain DNA unwinding.
elongation. In eukaryotes ~80 bp AT rich sequence called DNA
Bacterial DNA Polymerase—at a Glance
UNWINDING ELEMENT (DUE).
• Main replication DNA Polymerase is DNA Polymerase III • Ori+ ds DNA binding Protein (DNA A) opens the
m

• DNA Polymerase with highest rate of chain elongation (Most DNA duplex
co

Processive)Q is Pol III ‒ Unwinding (denaturation) of ds DNA to provide


• DNA Polymerase with proofreading activityQ: Pol I, Pol II and Pol III an ssDNA template
• DNA Polymerase with repair activity: Pol I and Pol II
‒ Ori + ds binding protein causes local denaturation
• DNA Polymerase which fills the gap in the lagging strand is Pol I
• DNA Polymerase which polymerises Okazaki fragments Pol III
of DNA.
• DNA Polymerase which synthesizes leading strand Pol III ‒ This facilitates the further unwinding of DNA
• Kornberg’s enzyme is DNAP I, as it is discovered by Arthur by Helicase.
Kornberg
• Arthur Kornberg described the existence of DNA Polymerase I
Role of Single Strand Binding Protein (SSB)
in E coli • Prevents the re annealing of the separated DNA
• Klenow FragmentQ: DNA Polymerase in which 5’ to 3’ exonuclease strands.
activity is removed.
• Human SSBs are called Replication Protein A (RPA)
• Formation of the replication fork
m

Eukaryotic DNA Polymerase ‒ Unwinding of DNA forms replication bubble


co

Mainly five types of Eukaryotic DNA Polymerase ‒ A pair of replication fork is replication bubble
• DNAP α Synthesis of RNA primer by Primase synthesize
• DNAP β 100–200 length Ribonucleotides.
e

e
m

m
m
co

264 |  
Self Assessment and Review of Biochemistry

‒ DNA G is the primase in case of Prokaryotes Contd...


‒ DNAP α Q has primase activity in case of Primosome
eukaryotes Mobile complex between helicase and primase.
• Initiation of DNA synthesis and elongation
Two strands are synthesized in different mannerQ TelomereQ and TelomeraseQ
• On the 5’ end of the newly synthesized linear DNA,
On 3’ ----> 5’ Strand (Leading Strand) (Continuous
m

RNA primer is removed by RNAase H


Strand) (Forward Strand)
• This leaves a gap at the 5’ end of daughter strand.
co

Synthesized in continuousQ manner on the 3’ hydroxyl


• In other words 3’ end of the leading strand is not
end of RNA primer in 5’ to 3’ direction
synthesized.
‒ In Prokaryotes by DNA Polymerase III
• This results in shortening of DNA with each cell
‒ In Eukaryotes by DNA Polymerase ε
division.
On 5’----> 3’ Strand (Lagging Strand) (Discontinuous • This is prevented by presence of TelomereQ and
Strand) (Retrograde Strand) Telomerase.Q
Synthesized in discontinuousQ manner.
Telomeres
• Small fragments of DNA are added in short spurts
• The ends of chromosome contain structures called
called Okazaki FragmentQ synthesized in 5’ ---- 3’
telomeres
direction.
• Telomeres consist of short T-G repeats
‒ By DNAP III in prokaryotes
m

• Human telomeres have variable number of tandem


‒ By DNAP δ in eukaryotes
repeats of the sequence 5’ TTAGGG-3’.
co

‒ Length of Okazaki fragments in Prokaryotes is


1000 to 2000 nucleotides Telomerase (Telomere Terminal Transferase)
‒ Length of Okazaki fragments in Eukaryotes is • Enzyme which prevents shortening of DNA
100 to 250 nucleotides. • Has an intrinsic RNA primerQ
• Removal of RNA Primers and gap filling of the • Has Reverse TranscriptaseQ (RNA Dependent DNA
lagging strand. Polymerase) activity.
‒ In Prokaryotes—Removal of RNA Primer and • Present in Germ lineQ, stem cellsQ, most cancer cellsQ.
Gap filling by DNA polymerase-I • Absent from most somatic cellsQ.
‒ In Eukaryotes-RNAse H removes the primer
‒ DNA P δ fills the gap, where RNA Primer is Clinical Significance of Telomerase
removed. • Absence of Telomerase lead to premature ageing
• Sealing the nick following gap filling. • In cancer cells increased Telomerase activity
m

‒ DNA ligase: Seals the single strand nick between • Telomere shortening is associated with ageing,
co

the nascent chain and Okazaki fragments on malignancy


lagging strand.Q • Telomerase has become an attractive target for cancer
• Length of Okazaki fragments in Prokaryotes is 1000 to 2000
chemotherapy and drug development.
nucleotides
• Length of Okazaki fragments in eukaryotes is 100 to 250
nucleotides
• Time taken for replication in bacteria is 30 minutes.
• Time taken for replication of entire human genome is 9 hours.

Replisome
Multimeric proteins that assemble in the replication fork are called
Replisome.
It includes:
m

• DNA helicase
co

• Primase
• DNA polymerase
• Single strand binding proteins
Contd... Fig. 10.1: DNA replication
e

e
m

m
m
co

DNA Replication   | 265

Contd...
Reverse Transcriptase
• Temin and Baltimore isolated this enzyme in 1970 DNA Repair
• They are RNA Dependent DNA Polymerase damaging Defects in mecha- Disorder associ-
• Synthesize new DNA strand with RNA as template agents DNA nism ated
• Thus, they reverse Central dogma of molecular genetics Werner Syndrome
• These enzymes are important in RNA Viruses like Retroviruses (WS)
• Telomerase has reverse transcriptase activity. Rothmund-Thomson
m

Syndrome (RTS)
co

DNA REPAIR MECHANISMS Breast Cancer Sus-


ceptibility (BRCA 1,
• DNA is subjected to a huge array of chemical, BRCA 2)
physical, and biological assaults on a daily basis 1. UV lightQ 1. Bulky Ad- Nucleotide Xeroderma Pigmen-
• Repair of damaged DNA is critical for maintaining 2. Chemi- ducts Excision tosaQ (XP)
cals 2. Pyrimidine Repair Cockayne Syn-
genomic integrity and thereby preventing the
DimersQ (NER)Q drome (SS)
propagation of mutations
Trichothiodystrophy
• Eukaryotic cells contain five major DNA repair (TTD)
pathways.
1. Oxygen 1. Abasic Sites Base MUTYH–associated
The mechanisms of DNA repair include: radicals 2. Single Excision Polyposis (MAP)
2. Alkyl- strand Repair
• Nucleotide Excision Repair (NER)
ating breaks (BER)
m

• Mismatch Repair (MMR) agents 3. 8 oxogua-


co

• Base Excision Repair (BER) nine lesions


• Homologous Recombination (HR) 1. Replica- 1. Bases mis- Mismatch Hereditary non-
• Nonhomologous End-Joining (NHEJ) repair. tion er- match repair polyposis
rors 2. Insertion (MMR) Colorectal Cancer
DNA Repair (HNPCC)Q (Lynch
damaging Defects in mecha- Disorder associ- 3. Deletion
syndrome)
agents DNA nism ated
1. Ionizing 1. Double Nonhomol- Severe Combined
Radia- Strand ogous End Immunodeficiency Double Strand Break Repair Mechanisms (DSB)
tions BreaksQ Joining (SCID) • They are Homologous recombination (HR) and non
2. X-raysQ 2. Single (NHEJ)
Strand
Homologous End Joining Repair (NHEJ)
3. Anti-
tumor Breaks Nonhomologous end joining
drugs 3. Intrastrand Homolo- Ataxia Telangecta- Homologous recombination repair
m

cross links gous Re- sia Like Disorder Major mechanism of DSB repair Major mechanism of DSB repair
co

4. Interstrand combina- (ATLD) in yeast mammals


cross links tion (HR) Nijimen break Syn-
drome (NBS) Takes place between homolo- Does not need a homologous
Bloom’s Syndrome gous chromosomes Chromosome
(BS) Takes place before cell enter Takes place before cell enter
mitosis (S and G2/M phase) mitosis (G0/G1 phase)
Contd...

REVIEW QUESTIONS

DNA REPLICATION c. DNA replication proceeds in one direction


d. Lagging strand stick by RNA primase
1. False statements is/are: (PGI May 2011)
m

a. In leading strands DNA is synthesized e. DNA polymerase III-processive leading strand


co

continuously synthesis
b. Multiple origins of replication are possible for Ans. b, c, d. (Harper 30/e page 381-387)
bacteria • Multiple ori in eukaryotes
e

e
m

m
m
co

266 |  
Self Assessment and Review of Biochemistry

• DNA replication is bidirectional 3. Incorrect statements are: (PGI Nov 2010)


• Lagging strand stick by DNA Ligase a. T4 DNA polymerase has 3’->5’ exonuclease
activity
DNA Replication
b. Klenow fragment of DNA polymerase I function
Salient Features of DNA Replication is almost similar to T4 DNA polymerase
• Occurs in the S Phase of the cell cycle c. Restriction endonuclease cut DNA chains at
m

• Each DNA Stand separate and each acts as template specific location
strand on which complementary strand is synthesized
co

d. Endonuclease cut DNA at 5’ terminus


• Base pairing rule is obeyed e. Right-handed helix of DNA is more common
• Semi conservative nature-Proved by Meselson and Ans. d. Endonuclease cut DNA at 5’ terminus.
Stahl Experiment Endonuclease cut the DNA from within
• New Strand is synthesized in the 5’ to 3’ direction Exonuclease cuts the DNA from ends
• Synthesis of DNA in both strands are not similar
T4 DNA Polymerase similar to Klenow polymerase.
‒ Leading strand: The strand which DNA is
continuously polymerized 4. Which DNA polymerase is involved in repair of
mammalian DNA: (PGI June 2009)
‒ Lagging Strand: The strand which is DNA is dis-
a. Alpha
continuously polymerized (Semi discontinuous)
b. Beta
• Replication proceeds from multiple origins in each
c. Gamma
m

chromosome in eukaryotes including humans (a total


d. Epsilon
co

of as many as 100 in humans)


• Replication obeys polarity e. Delta
• Replication occurs in both directions along all of Ans. b. Beta (Ref: Harper 30/e page 383)
the chromosomes, i.e. bidirectional in prokaryotes Eukaryotic DNA Polymerase
and eukaryotes Mainly five types of Eukaryotic DNA Polymerase
• Both strands are replicated simultaneously • DNAP α
• Replication process generates ‘replication bubbles’. • DNAP β
• DNAP γ
• DNAP δ
• DNAP ε
Eukaryotic DNA polymerase Function
m

DNAP alpha Primase


co

DNAP beta DNA repairQ


DNAP gamma Mitochondrial DNA synthesis
Fig. 10.2: Direction of replication DNAP delta Lagging strand synthesis
DNAP epsilon Leading strand synthesis
2. True about Eukaryotic DNA replication compared
to prokaryotic: (PGI May 2013) Prokaryotic DNA Polymerase
a. Conservative Three types of Prokaryotic DNA Polymerase
b. Semi conservative • Pol I
c. Unidirectional • Pol II
d. Bidirectional • Pol III
e. Semi discontinuous Prokaryotic DNA
m

Ans. b, d, e. Semi conservative, Bidirectional, Semi Polymerase Function


co

discontinuous. (Harper 30/e page 381-387) Pol I Gap filling following DNA replication, repair,
and recombination
This questions means the common features between
eukaryotic and prokaryotic DNA replication. Contd...
e

e
m

m
m
co

DNA Replication   | 267

Contd... • This results in shortening of DNA with each cell


Prokaryotic DNA division
Polymerase Function • This is prevented by presence of Telomere and
Pol II DNA proofreading and repair Telomerase.
Pol III Processive, leading strand synthesis, Telomeres
Okazaki fragment synthesis
• Are tandem repeats of simple sequence on 3’ end of
m

5. The gaps between segment of DNA on the the parent DNA


co

lagging stand produced by restriction enzymes • In humans repeats are (TTAGG)n.


are joined/sealed by: (AI 2009) Telomerase (Telomere Terminal Transferase)
a. DNA Ligases • Enzyme which prevents shortening of DNA
b. DNA Helicase • Has an intrinsic RNA primer
c. DNA Topoisomerase • Has Reverse Transcriptase (RNA Dependent DNA
d. DNA Phosphorylase Polymerase) activity
Ans. a. DNA Ligase (Ref: Harper 30/e page 381) • Present in Germ line, stem cells, most cancer cells
• Absent from most somatic cells.
6. During replication of DNA, which one of the
following enzymes polymerizes the Okazaki Clinical Significance of Telomerase
fragments? (AI 2006) • Absence of Telomerase lead to premature aging
m

a. DNA Polymerase I • In Cancer cells increased Telomerase activity


co

b. DNA Polymerase II • Telomerase has become an attractive target for cancer


c. DNA Polymerase III chemotherapy and drug development.
d. RNA Polymerase I
8. DNA Polymerase with both replication and repair
Ans. c. DNA Polymerase III (Harper 30/e page 381-387) function is: (Ker 2009)
DNA Synthesis a. I
On 5’----> 3’ Strand (Lagging Strand) (Discontinuous b. II
Strand) (Retrograde Strand) c. III
Synthesized in discontinuous manner d. None of the above
Small fragments of DNA are in short spurts of 100–250 Ans. a. I and b. II (Harper 30/e page 383)
nucleotide (1000–2000 bp in prokaryotes) called Okazaki 9. Radiolabelled DNA was allowed to replicate
Fragment synthesized in 5’ --- 3’ direction.
m

twice in a nonradioactive environment. Which of


• By DNAP III in prokaryotes.
co

the following is true? (Ker 2008)


• By DNAP δ in eukaryotes. a. All the strands will have radioactivity
b. Half of the DNA will have no radioactivity
7. All of the following cell types contain the enzyme
telomerase which protects the length of telome- c. No strands will have radioactivity
rase at the end of chromosomes, except: (AI 2006) d. Three–fourth of the DNA replicated will have
a. Germinal radioactivity.
b. Somatic Ans. b. Half of the DNA will have no radioactivity
c. Hemopoietic (Harper 30/e page 381-387)
d. Tumor • Semi conservative nature of DNA Replication proved
by Meselson and Stahl states that half of the parent
Ans. b. Somatic (Harper 30/e page 374)
strand is conserved during replication in the daughter
m

Telomere and Telomerase strand


• On the 3’ end of the linear DNA, RNA primer is
co

• After one replication all the DNA will have radio-


removed by RNAaseH, which leaves a gap at the activity
3’ end • After two replication half of the DNA will have
• Thus 5’ end of the new strand is not synthesized radioactivity.
e

e
m

m
m
co

268 |  
Self Assessment and Review of Biochemistry

10. In which of the following phase, DNA doubling 13. Action of telomerase is: (CUPGEE 2011)
occurs: (Ker 2006) a. DNA repair
a. Gl phase b. Longevity of cell
b. S phase c. Breakdown of telomere
c. G2 phase d. None
d. M phase Ans. b. Longevity of cell-aging (Harper 30/e page 374)
m

Ans. b. S Phase (Harper 30/e page 381-387)


co

14. Ends of chromosomes replicated by: (PGI Dec 06)


11. Unwinding Enzyme in DNA synthesis: a. Telomerase
a. Helicase b. Centromere
b. Primase c. Restriction endonuclease
c. DNA Polymerase d. Exonuclease
d. Transcriptase Ans. a. Telomerase (Harper 30/e page 374)
Ans. a. Helicase (Ref: Harper 30/e page 382)
15. Highly repetitive DNA is seen in: (PGI June 3)
Classes of Proteins involved in DNA Replication
a. Cloning of DNA
Protein Function
b. Microsatellite DNA
DNA polymerases Deoxynucleotide polymerization
c. Telomere
Helicases Processive unwinding of DNA
m

d. Centromere
Topoisomerases Relieve torsional strain that results from
co

helicase-induced unwinding Ans. c, d. Telomere and Centromere


DNA primase Initiates synthesis of RNA primers (Harper 30/e page 377)
Single-strand binding Prevent premature reannealing of dsDNA • In human DNA, at least 30% of genome consist of
proteins repetitive sequence
DNA ligase Seals the single strand nick between the • These sequences are clustered in the centromere and
nascent chain and Okazaki fragments on
and telomere
lagging strand
• They are transcriptionally inactive
• They are mostly having structural role in the
12. True about telomerase or telomere is/are:
chromosome.
(PGI Dec 03)
a. They are present at the ends of eukaryotic 16. Which enzymatic mutation is responsible for
chromosome immortality of cancer cells: (AIIMS Nov 01)
m

b. Increased telomerase activity favors cancer a. DNA reverse transcriptase


co

cells b. RNA polymerase


c. DNA dependent RNA polymerase c. Telomerase
d. DNA polymerase d. DNA polymerase
Ans. a, b. They are present at the ends of …, increased Ans. c. Telomerase (Robbins 9/e page 288)
telomerase… (Harper 30/e page 374) Telomerase is one of several factors that contribute to
• Telomeres are present in the ends of eukaryotic the endless replicative capacity (the immortalization) of
chromosomes. cancer cells.
• Telomeres consist of TG repeats.
• Telomere shortening has been associated with 17. Okazaki fragments are formed during the
malignant transformation and aging synthesis of: (AI 08)
• Telomerase, multisubunit RNA template containing a. dsDNA
m

RNA Dependent DNA Polymerase (Reverse b. ssDNA


co

Transcriptases). c. mRNA
• Enzyme responsible for Telomere synthesis and d. tRNA
maintaining the length of telomere. Ans. a. ds DNA
e

e
m

m
m
co

DNA Replication   | 269

18. Correct sequence of enzymes required for DNA DNA Repair


formation is: (PGI June 01) 21. Xeroderma pigmentosa is due to: (Ker 2006)
a. DNA polymerase → protein unwinding a. Base excision defect
enzyme → DNA ligase → DNA Isomerase → b. Nucleotide excision defect
Polymerase I
c. SOS repair defect
b. Protein unwinding enzyme → polymerase
d. Cross linking defect
I → DNA ligase → DNA isomerase → DNA
m

polymerase Ans. b. Nucleotide excision defect (Harper 30/e page 389)


co

c. RNA polymerase → DNA polymerase III → DNA damag- Defects in Repair Disorder
DNA polymerase I → DNA ligase ing agents DNA Mechanism associated
d. RNA polymerase → DNA polymerase III • Ionizing Ra- • Double Nonhomolo- Severe Com-
→ DNA ligase → exonuclease → DNA diations Strand gous End Join- b i n e d I m m u-
polymerase I • X-rays Breaks ing (NHEJ) nodeficiency
(SCID)
Ans. c. RNA Polymerase → DNA polymerase III → DNA • Antitumor • Single Strand
Breaks
polymerase I → DNA ligase (Harper 30/e page 381) drugs
Homologous Ataxia Tel-
• Intrastrand
• The correct sequence of enzymes is Helicase, Primase, Recombination angiectasia
cross links
DNA Polymerase III, DNA Polymerase I and on (HR) Like Disorder
• Interstrand Nijmen Break
lagging strand, DNA ligase cross links Syndrome
• Helicase, Primase, DNA Polymerase III on leading Bloom’s Syn-
m

strand. drome, Werner


co

Syndrome,
19. True about DNA polymerase in eukaryotes: Rothmund-
(PGI June 08) Thomson
Syndrome
a. Components are α, β, γ, δ, ε
Breast Cancer
b. β associated with repair Susceptibil-
c. γ associated with repair ity (BRCA 1,
BRCA 2)
d. δ associated with synthesis of mitochondria
DNA • UV light • Bulky Ad- Nucleotide Ex- Xeroderma
ducts cision Repair Pigmentosa
e. α is abundant amount • Chemicals
(NER) Cockayne Syn-
• Pyrimidine
Ans. a, b. Components are ..., β associated with ... Dimers
drome
Trichothio-
(Harper 30/e page 381)
dystrophy
m

Eukaryotic DNA polymerase Function


• Oxygen radi- • Abasic Sites Base Excision MUTYH –
co

DNAP alpha Primase cals Repair (BER) associated


• Single strand
DNAP beta DNA repair Polyposis
• Alkylating breaks
DNAP gamma Mitochondrial DNA synthesis agents • 8 oxoguanine
DNAP delta Lagging Strand Synthesis lesions
DNAP epsilon Leading Strand Synthesis • Replication • Bases mis- Mismatch Hereditary
errors match repair (MMR) nonpolyposis
20. DNA polymerase have: (PGI June 03) • Insertion Colorectal
Cancer
a. 3’–5’ polymerase activity • Deletion (HNPCC)
b. 5’–3’ polymerase activity
c. 3’–5’ exonuclease activity
22. UV light damage to the DNA leads to:
d. 5’–3’ exonuclease activity
(PGI Dec 05)
Ans. b, c, d. 5’–3’ polymerase activity, 3’–5’ exonuclease
m

a. Formation of pyrimidine dimers


activity, 5’–3’ exonuclease activity Harper 30/e page 382
co

b. No damage to DNA
DNA Polymerase have 5’ to 3’ Polymerase activity, 3’–5’
exonuclease (Proofreading) and 5’–3’exonuclease activity c. DNA hydrolysis
(repair) activity. d. Double stranded breaks
e

e
m

m
m
co

270 |  
Self Assessment and Review of Biochemistry

Ans. a. Formation of pyrimidine dimers • This is repaired by Nucleotide excision repair (NER)
(Harper 30/e page 390) • Defect in NER leads to Xeroderma Pigmentosa.
DNA lesions formed by UV light damage are Bulky 25. Which of the following is true regarding DNA
adducts and Pyrimidine Dimers. double-strand breaks repair pathway:
a. Homologous recombination require a long
23. Excessive ultraviolet (UV) radiation is harmful to
homologous sequence to guide repair
m

life. The damage caused to the biological system


b. Nonhomologous end-joining does not require
co

by ultraviolet radiation I by: (AIIMS May 04)


a. Inhibition of DNA synthesis a long homologous sequence to guide repair
b. Formation of thymidine dimers c. Homologous recombination repairs DNA
before the cell enters mitosis
c. Ionization
d. Nonhomologous end-joining repairs DNA
d. DNA fragmentation
before the cell enters mitosis
Ans. b. Formation of thymidine dimers e. Nonhomologous end-joining is prominent
(Harper 30/e page 390) DSB repair mechanism in mammals.
24. The primary defect in Xeroderma pigmentosa is: Ans. a, b, c, d, e.
(AI 2000) Double Strand Break Repair Mechanisms (DSB). They are
a. Formation of thymidine dimers Homologous recombination (HR) and Nonhomologous
End Joining Repair (NHEJ).
m

b. Poly ADP ribose polymerase is defective


co

c. Exonuclease is defective Nonhomologous end joining


d. Formation of adenine dimers Homologous recombination repair
Major mechanism of DSB repair Major mechanism of DSB repair
Ans. a. Formation of Thymidine dimers
in yeast mammals
(Harper 30/e page 390) Takes place between homolo- Does not need a homologous
• UV light radiation causes Bulky adducts and gous chromosomes Chromosome
Pyrimidine dimers (Most common is Thymidine Takes place before cell enter Takes place before cell enter
dimers) mitosis (S & G2/M phase) mitosis (G0/G1 phase)
m
co
m
co
e

e
m

m
m
co

11 Transcription
m
co

Topics Included
• Definition and Salient Features • Post-transcriptional Modification
• Steps of Transcription • Different Classes of RNA

TRANSCRIPTION • Multisubunit Enzyme


m

• Core Enzyme + σ subunit = Holoenzyme (E σ)


co

Definition
• Core Enzyme consist 2 α and 1β and1 β’ and ω subunit
The process by which RNA is synthesized from the DNA
• σ subunitQ help RNA polymerase to bind to the
is called Transcription.
promoterQ site
Salient Features of Transcription • β subunitQ is the catalytic subunit
Template Strand and Coding Strand • β subunitQ binds the Mg2+ ions.
The strand that is transcribed or copied to the mRNA is
referred to as Template strand or Nonsense strand.
The opposite strand is referred to as Coding strand or
Non-template strand or Sense Strand.
m
co

The nontemplate strand is called coding strand Fig. 11.1: Prokaryotic RNA polymerase
• Primary transcript is complementary to the template strand.
• Hence, the coding strand contains the same base sequence in the Eukaryotic RNA Polymerases
nascent mRNA except in the case of Thymine replaced by Uracil
There are threeQ types of Eukaryotic RNA Polymerase.
• Hence, nontemplate strand is called coding strand.
They are more complex than prokaryotic RNA polymerase
RNA Polymerase (RNAP) with a number of subunits.
They are DNA dependent RNA Polymerase. Form of RNA Sensitivity to Major Products
Differences between DNAP and RNAP Polymerase α-Amanitin of RNAP
m

• No primer is needed in RNAP RNA Polymerase I Insensitive rRNA


co

• No proofreading activity in RNAP. RNA Polymerase II High sensitivity mRNA, miRNA,


SnRNA, lnc RNA
Prokaryotic RNA Polymerase
RNA Polymerase III Intermediate sensitivity tRNA, 5s rRNA
• Only one typeQ of Prokaryotic RNA Polymerase
e

e
m

m
m
co

272 |  
Self Assessment and Review of Biochemistry

Promoters of Transcription Enhancers and Repressors


Defined as the short conserved sequence in the coding Certain DNA elements facilitate or enhance initiation
strand of the DNA that specifies start site of the at the promoter and hence are termed enhancers and
transcription. those which repress initiation at the promoter are called
Repressors or silencers.
Bacterial Promoters are:
Enhancer elements, typically contain multiple binding
• TATA Box (5’ TATAAT 3’) or Pribnow Box
m

sites for transactivator proteins.


10 bp upstream of the start site of transcription
co

(–10 bp) Properties of EnhancersQ


• TGG Box (5’ TGTTGACA 3’) • Can be located upstream or downstream of the
35 bp upstream of the start site of transcription transcription site
(–35 bp). • Work when located long distances from the promoter
• Work when upstream or downstream from the
Eukaryotic Promoters are: promoter
• Golberg Hogness Box Q (similar to TATA Box) • Work when oriented in either direction
(5’ TATAAAG 3’)
• Can work with homologous or heterologous
25 to 35 bp upstream of the start site of transcription promoters
(–25 to –35 bp)
• Work by binding one or more proteins
• CAAT Box • Work by facilitating binding of the basal transcription
m

70 to 80 bp upstream of the start site of transcription complex to the cis-linked promoter.


co

• GC-rich region (GC box)


Cis–acting Elements
• Apart from this additional elements like Inr (Initiator
• Promoters and enhancers that are located on the same
sequence) and DPE (Downstream promoter Element)
DNA are called cis-acting elements.
can serve as promoters.
Trans-acting Elements
Remember
• Usually promoters are located upstream of the start site of • Trans-acting proteins are the products of a separate
transcription gene that interacts with the cis-acting elements.
• But promoters for RNA Polymerase III (transcribe tRNA) are
located downstream within the gene The Transcription Cycle
• The promoter sequence (e.g. TATAAT) is on the coding strand
of the DNA Transcription can be described in six steps:
• Complementary sequence to the above promoter sequence is • Template binding and closed RNA polymerase-
seen on the template strand
m

promoter complex formation:


• σ subunit of RNA Polymerase bind to promoter on the template
co

strand. RNA polymerase (RNAP) binds to DNA and then


locates a promoter (P) by means of sigma subunit.
• Open promoter complex formation:
Once bound to the promoter, RNAP melts the two
DNA strands to form an open promoter complex.
This complex is also referred to as the pre-initiation
complex or PIC.
Strand separation allows the polymerase to access the
coding information in the template strand of DNA.
• Chain initiation:
Using the coding information of the template RNAP
m

catalyzes the coupling of the first base (often a purine)


Figs 11.2 A and B: Promoters of transcription
to the second, to form a dinucleotide.
co

Transcription Unit • Promoter clearance:


The region of DNA that includes signals of transcription After RNA chain length reaches ~10–20 nucleotides,
initiation, elongation and termination. the polymerase undergoes a conformational change,
e

e
m

m
m
co

Transcription   | 273

then it moves away from the promoter, transcribing Differences between Replication and Transcription
down the transcription unit. Replication Transcription
• Chain elongation: Deoxyribonucleotides are added Ribonucleotides are added
Successive residues are added to the 3’ –OH terminus A is paired with T on the parent U replaces T as the complemen-
of the nascent RNA molecule until a transcription strand tary base for A in RNA
termination signal (T) is encountered. Both the strands of DNA act as Only one strandQ of the DNA
m

template acts as the Template


• Chain termination and RNAP release:
co

Entire genome must be copied Only portions of the genome are


By two methods: vigorously transcribed or copied
‒ r (rho) factor dependent terminationQ into RNA

Termination signal for transcription in the A primer is involved as DNA A primer is not involved as RNA
Polymerase cannot initiate DNA polymerases have the ability to
template strand is identified by r factor. Synthesis de novo initiate synthesis de novo
r factor is an ATP dependent RNA-DNA helicase Highly active proofreading mech- No highly active proofreading
that disrupts the nascent RNA-DNA complex. anism mechanism

‒ Intrinsic (Spontaneous) or r (rho) factor DNA dependent DNA Polymerase DNA dependent RNA
is the enzyme. Polymerase is the enzyme
independent termination
RNA polymerase identifies the termination signal
on the template strand without the aid of r factor. Post-Transcriptional Modifications of mRNA or
RNA Processing
m

But for this termination the nascent RNA should


In Prokaryotes
co

have certain pre-requisites.


• mRNA are not subjected to post-transcriptional
i. GC rich region that forms a hairpin turn
processing.
ii. U rich region after the GC rich region. The • Translation is started simultaneous with transcription.
binding of A-U is weak, hence, facilitate ter- • Exception: tRNA and rRNA of prokaryotes undergo
mination of transcription. post transcriptional modification.

Similarities between Transcription and In eukaryotes


Replication • The RNA molecule synthesized by RNA polymerase
is known as Primary Transcript or Heteronuclear
• The general steps of initiation, elongation, and
RNA (hn RNA)
termination with 5’–3’ polarity is present in both.
• hnRNA undergoes extensive post-transcriptional
• Large, multicomponent initiation complexes are modification.
m

involved in both.
Site: The primary transcripts are extensively modified
co

• Both obey Watson–Crick base-pairing rules. in the nucleus after transcription.

Post-Transcriptional Processing of Primary


Transcript of mRNA
• 7-methylguanosine capping at 5’ end
• Addition of a poly-A tail at 3’ end
• Removal of introns and joining of Exons called
Splicing
• Methylations
• Alternative RNA Processing
1. 7-methylguanosine capping at 5’ end
m

This takes place in two steps:


co

a. Guanosine Triphosphate is attached to the 5’ end


of hnRNA
Fig. 11.3: Transcription cycle - By an enzyme Guanylyltransferase
e

e
m

m
m
co

274 |  
Self Assessment and Review of Biochemistry

- By an unusual 5’-5’ triphosphate linkage.


- Takes place inside the nucleus.
b. Methylation of Guanosine Triphosphate
- By Guanine 7 methyltransferase.
- S-adenosyl Methionine is the methyl donor.
- Takes place inside the Cytosol.
m

Functions of 5’ capping
co

Helps in
‒ The initiation of translation
‒ Helps to stabilize the mRNA
‒ Prevents the attack of 5’ to 3’ exonuclease.
2. Addition of a poly-A tail at 3’ end
‒ Poly A tail is added to the 3’ end of the hnRNA
‒ Polyadenylate Polymerase is the enzyme
‒ Takes place in the nucleus
‒ Length of Poly A tail is up to 200 Adenine bases. Fig. 11.4: Mechanism of splicing
Functions of Poly A tail at the 3’ end.
m

‒ Stabilize the mRNA Mechanism of Splicing


co

‒ Prevents the attack of 3’ to 5’ exonuclease • The binding of snRNP brings the sequences of the
‒ Facilitate their exit from the nucleus neighboring exons into the correct alignment for
‒ Poly A tail and its binding protein PAB-1 splicing.
are required for efficient initiation of Protein • The splicing start with a cut in the 5’ splice donor site.
Synthesis. • The 2′ –OH group of an adenosine (A) residue (known
3. Removal of introns and joining of Exons called as the branch site) in the intron attacks the phosphate
Splicing at the 5′ -end of the intron, forming an unusual
Intron: Intervening sequence that do not code for 2′ → 5′ phosphodiester bond and creating a ‘lariat’
amino acid structure.Q
ExonQ: Amino acid coding sequence • The newly freed 3′ –OH of exon 1 attacks the
Molecular machinery that carry out splicing is called 5′-phosphate at the splice acceptor site, forming a
m

Spliceosome. phosphodiester bond that joins exons 1 and 2.


co

Spliceosomes • The excised intron is released as a lariat, which is


Consist of the primary transcript, five small nuclear RNAs (U1, U2, degraded.
U4, U5, and U6) and more than 60 protein.
Spliceosome = snRNA + RNP + hnRNA (or mRNA precursor) • After introns have been removed and exons joined,
Small Nuclear RNA (Sn RNA) the mature mRNA molecules leave the nucleus and
• Uracil rich RNA which can act as enzymes, i.e. Ribozyme. pass into the cytosol through pores in the nuclear
• U1, U2, U4, U5, U6 involved in mRNA processing. membrane.
• U6 is certainly essential, as yeast deficient in this Sn RNA is not
viable Self SplicingQ
• U7 snRNA is involved in production of correct 3’ ends of histone • Certain hnRNA itself has splicing activity
mRNA that lacks Poly A tail.
SnRNP complex (Snurps) (Small nuclear ribonucleoprotein) • Because of Ribozyme activity.
• SnRNP (Snurps) = Sn RNA + Ribonucleo Protein (RNP)
Effect of Splice site Mutations
m

Clinical Correlation
Systemic lupus erythematosus results from an autoimmune response Mutations at splice sites can lead to improper splicing
co

in which the patient produces antibodies against host proteins, includ- (faulty splicing) and the production of aberrant
ing snRNP (Snurps)
proteins.
e

e
m

m
m
co

Transcription   | 275

For example, mutations that cause the incorrect RNA Editing


splicing of β-globin mRNA are responsible for some cases • mRNA editing is an exception to central dogma of
of β-thalassemia. molecular genetics
4. Methylations
• Current estimate suggest that 0.01% of mRNA is
‒ Methylation of N7 of Adenine and 2’ hydroxyl edited in this fashion
group of ribose
m

• The central dogma states that for a given gene and


‒ Takes place in the cytoplasm.
gene product there is a linear relationship between
co

5. Alternative processing of mRNA precursor or


the coding sequence in DNA, the mRNA sequence,
Alternative Splicing
and the protein sequence
‒ A mechanism for producing a diverse set of
proteins from a limited set of genes. • Changes in the DNA sequence should be reflected in
a change in the mRNA sequence and, depending on
‒ The pre-mRNA molecules from same genes can
codon usage, in protein sequence
be spliced in two or more alternative ways in
different tissues. • mRNA editing is the process by which coding
information is changed at the mRNA level by
The mechanisms for alternative processing of mRNA precursors
• Selective Splicing—selective inclusion or exclusion of exons chemical modification of the nitrogenous bases
• Alternative 5’ donor site—5’ donor site of certain exons is changed. present in the codons
• Alternative 3’ acceptor site—3’ acceptor site of certain exons is • Hence, the linear relationship between the coding
m

changed.
• Alternative Polyadenylation site—Different site is used for Poly- sequence in DNA, the mRNA sequence, and the
co

adenylation. protein sequence is altered


• Thus, it is an exception to Central Dogma of
Applications of Alternative mRNA Processing
molecular genetics.
• Generation of membrane bound or Secretory IgG by
alternative polyadenylation sites.Q
• Production of several tissue specific isoforms of An example is the apolipoprotein B (apoB) gene and
tropomyosin from single mRNA transcript. mRNA.
In the liver,
• The single apoB gene is transcribed into an mRNA
that directs the synthesis of a 512kDa protein, apo
B100 with 4536 amino acid residues.
m
co

In the intestine,
• The same gene directs the synthesis of the primary
transcript
• A cytidine deaminase converts a CAA codon in the
mRNA to UAA at a single specific site
• Rather than encoding glutamine, this codon becomes
a termination signal, and a truncated 242kDa protein
(apo B48) with 2512 amino acid residues is the
result.

Other examples of RNA editing


m

• Glutamate Receptor (Glutamine changed to Arginine)


co

Fig. 11.5: Methods of alternative splicing • Trypanosome mitochondrial DNA.


e

e
m

m
m
co

276 |  
Self Assessment and Review of Biochemistry

DIFFERENT CLASSES OF RNA • Ribosomal Assembly is ribosomal RNAs associated


with certain proteins (i.e. rRNA + Proteins).
CLASSES OF RNAs Ribosomal Assembly in ProkaryotesQ
RNA exists in two major classes: 70S Ribosome = 30S + 50S Subunits
In 30S Subunit 16S rRNA + Proteins
In 50S subunit 23S rRNA+ 5S rRNA + Proteins
m

Ribosomal Assembly in EukaryotesQ


80S Ribosome = 40S Subunit + 60S Subunit
co

60S Subunit = 28S rRNA + 5.8S r RNA + 5S rRNA + ~50 proteins


40S =18S rRNA + ~30 proteins
m

Fig. 11.6: Classes of RNA


co

RNA Types Abundance Stability


Protein coding RNAs
Fig. 11.7: Ribosomal assembly in prokaryotes and eukaryotes
Messenger ~105 Different 2–5% of total Unstable to
(mRNA) species very stable Transfer RNA (tRNA)
Large ncRNAs • RNA which transfer amino acid from the cytoplasm
Ribosomal 28S, 18S, 80% of total Very stable to the ribosomal protein synthesizing machinery
(rRNA) 5.8S, 5S
lnc RNA ~1000s ~1%–2% Unstable to
• Clover leaf shape in the secondary structure
very stable • L shaped tertiary structure
Small noncoding RNA (Small ncRNA) • Single tRNA contains 74-95 nucleotides
Transfer ~60 Different ~15% of total Very stable
• Cytoplasmic translation system possess 31 tRNA
(tRNA) species
species
m

Small nuclear ~30 Different 1% of total Very stable


(snRNA) species • Mitochondrial translation system possess 22 tRNA
co

Micro/Silencing 100s–1000 <1% of total Stable species.


(mi/SiRNA)
tRNA contain significant proportion of nucleosides with unusual
bases
Messenger RNA (mRNA) They are
• Most heterogenous RNA • DihydroUridine (contain Dihydrouracil)
• Pseudouridine
• Function as messenger conveying information to
• Inosine (contain Hypoxanthine)
translation machinery • Ribothymidine
• 2-5% of total cellular RNA
• 5’ end capped by 7 Methylguanosine triphosphate Arms of tRNA
• 3’ end nongenetically encoded by polymer of 20–250 • Acceptor arm
adenylate residues.
‒ Site of attachment of the amino acid
m

Ribosomal RNA (rRNA) ‒ 3’ end of the tRNA


co

• Most abundant RNA is rRNA (80% of total RNA) ‒ Has 3 unpaired nucleotide, CCA
• Function-Forms Protein Synthesising Machinery ‒ Carboxyl group of the amino acid is attached to
called Ribosome the 3’ hydroxyl group of the adenosyl moiety.
e

e
m

m
m
co

Transcription   | 277

• Anticodon arm Post-transcriptional modification of tRNA precursor


‒ Has the triplet nucleotide sequence complemen- • Standard bases (A, U, G, C) undergo methylation,
tary to the codon of the amino acid which the reduction, deamination, rearranged glycosidic bond
tRNA carries • Cleavage and attachment of C C A terminal at 3’ end
‒ The sequence is read from 3’ to 5’ direction takes place in cytoplasm
‒ Codon is read from 5’ to 3’ direction • Methylation of tRNA takes place in nucleus.
m

‒ Codon of mRNA and anticodon in tRNA are Small Nuclear RNA (SnRNA)
co

antiparallel in their complementarity.


• They belong to small RNAs of size 90 to 300
• DHU arm nucleotides
‒ Contain Dihydrouracil residue • 1% of total RNAs
‒ Acts as the recognition site for specific aminoacyl • They have ribozyme activity.
tRNA synthetase.
Function
• Pseudouridine arm (TψC arm)
• mRNA Processing (U1, U2, U4, 5, 6 and U7 as a part
‒ TψC stands for Ribothymidine, Pseudouridine,
of spliceosome)
Cytidine
• rRNA processing
‒ Involved in the binding of aminoacyl tRNA to
• Gene regulation.
the Ribosomal surface.
• Extra arm (Variable arm) miRNA and siRNA
m

‒ Between Pseudouridine and Anticodon arm • Small noncoding single stranded RNAs which are
co

‒ The most variable feature of tRNA. 21-22 nucleotide length


‒ Different classes of tRNA is based on the extra • Main functionQ: Post-transcriptional regulation of
arm. gene expression by targeting mRNA by several
distinct mechanism.
We can discuss the following
• Generation of miRNA and SiRNA
• Post-transcriptional modification of miRNA and
siRNA
• Regulation of gene expression by miRNA and siRNA
• Clinical correlations.
m

Micro RNA (miRNA)


co

• Generation of miRNA
‒ Transcribed by RNA Polymerase II from miRNA
encoding genes to Pri miRNA.
Post-transcriptional modification of miRNA
Pri miRNA undergo extensine post-transcriptional
processing as follows:
‒ Pri-miRNA is subject to processing by DROSHA-
DGCR8 nucleases, which trims 5’ cap and 3’ Poly
A tail to generate Pre-miRNA
‒ The double stranded Pre miRNA is transported
to cytoplasm through nuclear pore, Exportin-5
m

‒ Pre-miRNA is further trimmed by Dicer nuclease


co

(TRBP-Dicer) to form 21-22 nucleotide miRNA


Fig. 11.8: Structure of tRNA duplex.
e

e
m

m
m
co

278 |  
Self Assessment and Review of Biochemistry
m
co
m
co

Fig. 11.9: Generation of miRNA and SiRNA

‒ One of the strand of duplex miRNA is selected. Regulation of gene expression by miRNA
‒ The selected strand is loaded to RNA-induced • Binding of miRNA to mRNA by normal base pairing
Silencing complex (RISC) • All mRNAs contain a seed sequence in their
‒ Mature functional 21–22 nucleotide mi RNA is 3’untranslated region (UTR) that determines the
thus produced. specificity of miRNA binding and gene silencing
• If miRNA-mRNA base pairing has one or more mis-
Silencing RNA or Small Interfering (SiRNA)
matches. Translation of cognate mRNA is inhibited
Generation of SiRNA
m

• If miRNA-mRNA base pairing is perfect,


• Functional SiRNA is generated endogenous or
co

corresponding mRNA is degraded.


exogenous double stranded RNA
• Extracellular sources include RNA Viruses. miRNA modulate the function of target mRNA by three
methods
Post-transcriptional modification of SiRNA
• Translation repression by targeting 5’ methyl cap
• Double stranded RNA are processed by Dicer binding factor, eIF4 or ribosome directly
nuclease
• mRNA destabilzation by mRNA Poly A tail
• One strand is selected and loaded to RISC.
deadenylation
RISC in miRNA is composed of Argonuate Proteins (Ago 1 → 4) • Promoting mRNA degradation directly.
RISC in SiRNA is composed of Argonuate-2 Proteins
Regulation of gene expression by SiRNA
Specific Nucleases Involved in the Post Transcriptional
Modification of miRNA and SiRNA • Induces mRNA cleavage which inactivate target
mRNA.
m

• Drosha-DGCR8Q Nucleases
• Dicer Nucleases
co

RNA Interference or RNAi


• The Regulation Gene Expression by mi RNA and • Functional Consequence of Translation Arrest and
si RNA mRNA degradation by miRNA/SiRNA is Silencing
‒ By altering mRNA function. the gene expression or Gene Silencing
e

e
m

m
m
co

Transcription   | 279

• This is otherwise called RNA Interference Regulation of gene expression by lnc RNA
• RNAi by miRNA/SiRNA is an example of gene knock • Facilitate transcription factor binding and thus
down. promote gene activation
P Bodies • Bind to transcription factors and thus prevent gene
• Nontranslating mRNA form ribonucleoprotein transcription, e.g. Decoy lnc RNA
particles and they accumulate in cytoplasmic ‒ The best known example of a repressive function
m

organelle called P bodies involves XIST, which is transcribed from the


co

• Ribonucleoprotein or mRNP are mRNA, bound by X Chromosome and plays an essential role in
specific packaging Proteins physiologic X chromosome inactivation
• P bodies are sites of translation repression and mRNA • Facilitate Histone and DNA modification by directing
decay methylases or acetylases
• P bodies contain mRNA decapping enzymes, RNA
• Act as scaffolding and stabilize secondary or tertiary
helicases, RNA exonucleases, etc. for mRNA quality
structures and multisubunit complexes that influence
control
chromatin structure.
• A portion of miRNA driven mRNA modulation takes
place in P bodies. Some recent facts about lncRNA
In 2006, Craig Mello and Andrew Fire were awarded Nobel Prize for • lncRNAs may exceed coding mRNAs by 10- to 20-fold
silencing gene expression by mi RNA. • It has been recently appreciated that many enhancers
m

miRNA and Cancer


are sites of lncRNA synthesis, often increase
co

• Can prevent cancer by degrading mRNA of an oncogene. They transcription


are called Tumor Suppressive miRs • Emerging studies are exploring the roles of lncRNAs
• Can cause cancer degrading m RNA of a tumour suppressor gene in various human diseases, from atherosclerosis to
called oncogenic mRNA (oncomir)
miRNA associated with cancer cancer.
Oncomirs
• Mir-21 is one of the widely studies oncogenic miRNA.
• miR-200 promote epithelial-mesenchymal transitions which is
important in invasiveness and metastasis of tumor.
• miR-155 is overexpressed in many human B cell lymphomas and
indirectly upregulates genes like MYC.
Tumor suppressive miRs
• Deletions affecting certain tumor suppressive miRs, such as
miR-15 and miR-16, are among the most frequent genetic lesions
m

in chronic lymphocytic leukemia


co

• Rare ovarian and testicular tumors, associated with germline


defects in DICER, a gene that encodes an endonuclease
miRNA in DNA repair
• mir-34
• p53, the molecular policeman activate the expression of mir-34.

Application of miRNA/SiRNA
SiRNA/miRNA and Transgenic mice
• Synthetic SiRNA targeted against specific mRNA can be
experimentally introduced into cell to study gene function by Gene
knock down technology.
• SiRNA can be used as possible therapeutic agents to silence
pathogenic genes, such as oncogenes involved in neoplastic
transformation.
m

Long Noncoding RNA (lnc RNA)


co

• Transcribed by RNA Polymerase II


• They are >200 nucleotide length
• They modulate gene expression in many ways. Figs 11.10A to D: Regulation of gene expression by lncRNA
e

e
m

m
m
co

280 |  
Self Assessment and Review of Biochemistry

hnRNA (Heteronuclear RNA) piwi-interacting RNAs (piRNAs),


• Primary transcript mRNA formed from DNA template • The most common type of small noncoding RNA
• It underges post transcriptional modification to form • Function: They have a role in post-transcriptional
mature mRNA.
gene silencing (like miRNAs).
Newly Described Noncoding RNAs
Small Nucleolar RNA (SnoRNA) Long intervening noncoding RNAs (linc RNAs)
m

• RNA involved in eukaryotic rRNA Processing and • Function: Regulate the factors that modify histones
co

assembly of ribosomes (epigenetic modifications) and thereby control gene


• Present in the nucleolus. expression.

REVIEW QUESTIONS

TRANSCRIPTION Ans. e. 5’-GUACGUAA 3’.


• Here question is to find the RNA, so all options with
1. True about coding strand of DNA: (PGI Dec 03) Thymine is not the answer. So answer is E
a. Template strand
• The exact method is to find the complementary
b. Minus strand
m

sequence for the given DNA and at the places of T,


c. Runs at 5’–3’ direction replaced by U
co

d. Runs at 3’–5’ direction • 5’ TTACGTAC 3’ is read in 3’ to 5’ direction


e. Plus strand • So RNA is 5’ GUACGUAA 3’.
Ans. c, d, e. Runs in 5’–3’ direction, Runs at 3’–5’ direction,
Plus strand. (Harper 30/e page 395) 3. Immunoglobulin molecule is synthesized by in
• Template strand is in 3’ to 5’ direction, or 5’ to 3’ mixed or separate due to: (AIIMS May 2012)
direction, Minus strand, Antisense strand a. Co-dominance
• Coding strand is in 5’ to 3’ direction, or 3’ to 5’ b. Gene switching
direction, Plus strand, Sense strand c. Allele exclusion
• Whatever is the direction of template strand, for d. Differential RNA processing
transcription, that strand is read always in 3’ to 5’ Ans. d. Differential RNA processing.
direction. (Ref: Harper 30/e page 449)
m
co

Alternative RNA Processing, which includes Alterna-


tive Polyadenylation site in the μIg heavy chain primary
transcript, results in two μ protein, μm and μs. One
remains membrane bound to B lymphocyte (μm) and
the other secreted (μs).
This figure from Harper clearly shows template or coding
strand can be either 5’ to 3, or 3’ to 5’. But the direction 4. A four-year-old child is diagnosed with Duchenne
of transcription is 5’ to 3’. The template strand is read muscular dystrophy, an X-linked recessive dis-
always in 3’ to 5’ direction. order, Genetic analysis shows that the patient’s
gene for the muscle protein dystrophin contains a
2. 5’ TTACGTAC 3’ after transcription what will be mutation in its promoter region. What would be
the RNA: (PGI June 08) the most likely effect of this mutation?
a. 5’-TTACGTAC 3’ (Nov 2010)
m

b. 3’-TTACGTAC 5’ a. Tailing of dystrophin mRNA will be defective


co

c. 5’-CATGCATT 3’ b. Capping of dystrophin mRNA will be defective


d. 3’-CATGCATT 5’ c. Termination of dystrophin transcription will
e. 5’-GUACGUAA 3’ be deficient
e

e
m

m
m
co

Transcription   | 281

d. Initiation of dystrophin transcription will be • Methylation of Guanosine Triphosphate


deficient ‒ By Guanine 7 methyltransferase
Ans. d. Initiation of dystrophin transcription will be ‒ S-adenosyl Methionine is the methyl donor
deficient. (Ref: Harper 30/e page 397-400) ‒ Takes place inside the Cytosol.
The fidelity and frequency of transcription is controlled
by proteins bound to certain DNA sequences. These Addition of a Poly-A Tail at 3’ End
m

regions are termed promoters, and it is the association • Poly A tail is added to the 3’ end of the hnRNA
co

of RNAP with promoters that ensures accurate initiation • Polyadenylate Polymerase is the enzyme
of transcription. • Takes place in the nucleus
Promoters are responsible for initiation process of • Length of Poly A tail is up to 200.
transcription.
Removal of Introns and Joining of Exons called Splicing
5. Splicing activity is a function of: • Intron-Intervening Sequence that does not code for
(AIIMS Nov 2010) amino acid
a. mRNA • Exon-Amino Acid Coding Sequence
b. snRNA • Molecular machinery that carry out splicing is called
c. tRNA Spliceosome.
d. rRNA
m

Ans. b. sn RNA. (Harper 30/e page 367,411) 7. Noncoding RNAs are: (PGI May 2012)
co

Small Nuclear RNA (Sn RNA) a. siRNA


Uracil rich RNA which can act as enzymes, i.e. Ribozyme. b. miRNA
U1, U2, U4, U5, U6 involved in mRNA processing. c. tRNA
U7 in processing histone mRNA. d. mRNA
e. rRNA
6. Post-transcriptional modification includes:
(PGI Nov 2012) Ans. a, b, c, e. (Harper 30/e page 395)
a. All RNA undergo post-transcriptional mod- Noncoding RNAs (ncRNA)
ification • Transfer RNA (tRNA)
b. Capping of pre mRNA involves the addition of • Ribosomal RNA (rRNA)
7-methylguanosine to the 5’ end
m

• Sno RNA
c. Poly A tail occur at 3’ end d.
co

• gRNA (Guide RNA)


Intron excision by spliceosome e.
• miRNA
Primarily occurs in the cytoplasm
• siRNA
Ans. a, b, c, d. All RNA undergo post ........., Capping
• lncRNA.
of pre mRNA ........., Poly A tail occur at 3’ end, Intron
excision by spliceosome.
8. Not a product of transcription (PGI May 2011)
Post-Transcriptional Processing Includes:
a. tRNA
Primarily occurs in the nucleus
b. mRNA
7-methylguanosine Capping at 5’ End
c. rRNA
This takes place in two steps.
d. cDNA
• Guanosine Triphosphate is attached to the 5’ end of
m

e. New strand of DNA


hnRNA
co

‒ By an enzyme Guanylyltransferase Ans. d, e. cDNA, New strand of DNA.


‒ By an unusual 5’-5’ triphosphate linkage (Ref: Harper 30/e page 395-398)
‒ Takes place inside the nucleus. cDNA is produced by reverse transcription.
e

e
m

m
m
co

282 |  
Self Assessment and Review of Biochemistry

New DNA strand produced by DNA replication. They are:


Form of RNA Sensitivity to Major Products of • Dihydro uridine (contain Dihydrouracil)
Polymerase α-Amanitin transcription • Pseudouridine
RNA Polymerase I Insensitive rRNA • Inosine (contain Hypoxanthine)
RNA Polymerase II High sensitivity mRNA, miRNA,
• Ribothymidine.
SnRNA, lnc RNA
m

RNA Polymerase III Intermediate tRNA, 5s rRNA 11. The sigma (s) submit of prokaryotic RNA poly-
co

sensitivity
merase: (AI 2006)
a. Binds the antibiotic rifampicin
9. Reverse transcriptase is: (PGI May 2011)
b. Is inhibited by a-amanitin
a. DNA dependent RNA polymerase
c. Specifically recognizes the promoter site
b. RNA dependent DNA polymerase
d. Is part of the core enzyme
c. DNA dependent DNA polymerase
d. RNA dependent RNA polymerase Ans. c. Specifically recognizes the promoter site.
e. RNA polymerase (Ref: Harper 30/e page 399)
Ans. b. RNA dependent DNA polymerase. Prokaryotic RNA Polymerase
(Ref: Harper 30/e page 363) Only one type of Prokaryotic RNA Polymerase
Reverse Transcription Multisubunit Enzyme
m

• RNA converted to DNA is reverse transcription • Core Enzyme + σ subunit = Holoenzyme (E σ)


co

• The enzyme is called Reverse Transcriptase • Core Enzyme consist 2 α and 1β and1 β’ and ω
• This is otherwise RNA dependent DNA Polymerase. subunit
Remember • σ subunit help RNA polymerase to bind to the
• DNA Polymerase is DNA dependent DNA promoter site
Polymerase • β subunit is the catalytic subunit
• Reverse Transcriptase is RNA dependent DNA • β subunit binds the Mg2+ ions.
Polymerase
• Primase is DNA Dependent RNA Polymerase. 12. The base sequence of the strand of DNA used as a
template has the sequence 5’ GATCTAC 3’. What
10. Which type of RNA has the highest percentage of would be the base sequence of RNA product?
modified base? (AI 2006) (Ker 2012)
m

a. mRNA a. 5’ CTAGATG 3’
co

b. tRNA b. 5’ GAUCUAC 3’
c. rRNA
c. 5’ GTAGATC 3’
d. snRNA
d. 5’ GUAGAUC 3’
Ans. b. tRNA. (Ref: Harper 29/e page 411)
Ans. d. 5’ GUAGAUC 3’
Transfer RNA (tRNA)
Read the strand in 3’ to 5’ direction. Write the
• RNA which transfer amino acid from the cytoplasm
complementary sequence in 5’ to 3’ direction obeying
to the ribosomal protein synthesizing machinery
base pairing rule, except in the case of T replaced by U.
• Clover leaf shape in the secondary structure
• L-shaped tertiary structure 13. Most common RNA is: (Ker 2011)
• Single tRNA contains 74-95 nucleotides a. rRNA
m

• Cytoplasmic translation system possess 31 tRNA b. mRNA


species
co

c. tRNA
• Mitochondrial system possess 22 tRNAs.
d. hnRNA
Contain significant proportion of nucleosides with
unusual bases. Ans. a. rRNA. (Harper 30/e page 395)
e

e
m

m
m
co

Transcription   | 283

RNA Types Abundance Stability c. DNA polymerase III


Ribosomal (rRNA) 28S, 18S, 80% of total Very stable d. RNA polymerase
5.8S, 5S e. Primase
Messenger ~105 Different 2–5% of total Unstable to Ans. d. RNA Polymerase. (Harper 30/e page 395)
(mRNA) species very stable
Primase, DNA Polymerase & DNA Ligase for replication.
Transfer (tRNA) ~60 Different ~15% of total Very stable
m

species
18. RNA polymerase does not require: (AI 2004)
co

Small RNAs
a. Template (ds DNA)
Small nuclear ~30 Different 1% of total Very stable
(snRNA) species b. Activated precursors (ATP, GTP, UTP, CTP)
Micro (miRNA) 100s–1000 <1% of total Stable c. Divalent metal ions (Mn2+, Mg2+)
d. Primer
14. DNA dependent RNA polymerase is seen in: Ans. d. Primer. (Harper 30/e page 395, 397)
(Ker 2008) • RNA Polymerase does not require Primer.
a. Primase The following is a generalized diagram of typical
b. DNA polymerase I eukaryotic gene: (AIIMS May 06)
c. DNA polymerase III
Promoter Region Polypeptide Coding Region
d. DNA gyrase
m

Ans. a. Primase. (Ref: Harper 29/e page 366) INTRON


co

Primase enzyme synthesize RNA primer on the DNA


Direction of transcription
strand, hence, it is DNA dependent RNA Polymerase.

15. Strand of DNA from which mRNA is formed by 19. What is the most likely effect of a 2 bp insertion
transcription is called: (Ker 2006) in the middle of the intron?
a. Template a. Normal transcription, altered translation
b. Anti-template b. Defective termination of transcription, normal
c. Coding translation
d. Transcript c. Normal transcription, defective mRNA
Ans. a. Template. (Ref: Harper 30e page 395) splicing
d. Normal transcription, Normal translation
16. On which of the following tRNA acts specifically.
m

a. ATP Ans. d. Normal transcription, Normal translation.


co

b. Golgi body • As the insertion is in the middle of the noncoding


c. Specific amino acid region, intron
d. Ribosome • Transcription is normal
• As the intron is spliced out and only exons join
Ans. c. Specific amino acid. (Harper 29/e page 397)
together, translation is also normal.
At Least one Species of Transfer RNA (tRNA) Exists for
Each of the 20 Amino Acids 20. In a DNA, the coding region reads 5’-CGT-3’.
• tRNA molecules have extraordinarily similar This would code in the RNA as: (AIIMS May 03)
functions and three-dimensional structures. The a. 5’-CGU-3’
adapter function of the tRNA molecules requires b. 5’-GCA-3’
the charging of each specific tRNA with its specific
c. 5’-ACG-3’
amino acid.
m

d. 5’-UGC-3’
17. In conversion of DNA to RNA, enzyme required:
co

Ans. a. 5’-CGU-3’ (Harper 30/e page 395)


(PGI June 08) As the question specifies that it is coding region, so the
a. DNA-polymerase RNA product is same as coding region except in the case
b. DNA Ligase of T replaced by U.
e

e
m

m
m
co

284 |  
Self Assessment and Review of Biochemistry

21. Cytoplasmic process during processing is: Spliceosomes


(SGPGI 05, CMC 03) Consist of the primary transcript, five small nuclear
a. 5’ capping RNAs (U1, U2, U4, U5, and U6) and more than 60 protein.
b. Poly A tailing Spliceosome = snRNA + RNP + hnRNA (or mRNA
precursor).
c. Methylation of tRNA
d. Attachment of CCA in tRNA 24. A segment of eukaryotic gene that is not
m

Ans. d. Attachment of CCA in tRNA. represented in the mature mRNA is known as:
co

(Harper 30/e page 411) (AI 2004)


Sites of post-transcriptional processing a. Intron
b. Exon
tRNA c. Plasmid
• Cleavage and attatchment of CCA terminal at 3’ end d. TATA box
takes place in cytoplasm
Ans. a. Intron. (Harper 30/e page 409)
• Methylation of tRNA takes place in nucleus.
25. An enzyme that makes a double stranded DNA
mRNA
copy from a single stranded RNA template
• Poly A tail in Nucleus molecule is known as: (AI 2004)
• 5’ MeGTP capping in nucleus and cytoplasm a. DNA polymerase
• Methylation of some residues in Cytoplasm.
m

b. RNA polymerase
co

rRNA c. Reverse transcriptase


• Most common site is nucleolus. d. Phosphokinase
Ans. c. Reverse transcriptase.
22. All are the processing reaction in tRNA, except:
(WB 03, Delhi 04, UP 05) 26. Thymidylated RNA present in: (PGI June 01)
a. CCA tailing a. mRNA
b. Methylation of bases b. rRNA
c. tRNA
c. Poly A tailing
d. 16-s-RNA
d. Trimming of 5’ end
Ans. c. tRNA. (Harper 30/e page 409)
Ans. c. Poly A tailing.
Poly A tailing is a processing step of mRNA primary Pseudouridine Arm (TψC arm) of tRNA
m

transcript. TψC stands for Ribothymidine, Pseudouridine, Cytidine.


co

23. Introns are exised by: (PGI Dec 05) 27. Function of Pseudouridine arm of tRNA
a. RNA splicing (JIPMER 2015 Nov)
b. RNA editing a. Helps in initiation of translation
c. Restriction endonuclease b. Serves as the recognition site of amino acyl
tRNA synthetase
d. DNAase
c. Recognises the triple nucleotide codon present
e. Helicase
in the mRNA
Ans. a. RNA splicing d. Helps in initiation of transcription
Removal of introns and joining of Exons called Splicing Ans. a. Helps in initiation of translation.
Intron: Intervening sequence that do not code for (Harper 30/e page 409)
amino acid. Arms of tRNA
m

ExonQ-Amino acid coding sequence. 1. Acceptor Arm


co

Molecular machinery that carries out splicing is called Site of attachment of the Amino Acid
Spliceosome. 3’ end of the tRNA
e

e
m

m
m
co

Transcription   | 285

Has 3 unpaired nucleotide, CCA 30. Which of the following is true regarding trans-
Carboxyl group of the amino acid is attached to the cription except: (PGI)
3’ hydroxyl group of the adenosyl moiety. a. mRNA formed
2. Anticodon arm b. DNA polymerase enzyme is used
Has the triplet nucleotide sequence complementary to c. RNA polymerase enzyme is used
the codon of the amino acid which the tRNA carries. d. Eukaryotes possess 3 different types of RNA
m

The sequence is read from 3’ to 5’ direction polymerase


co

Codon is read from 5’ to 3’ direction Ans. b. DNA Polymerase enzyme is used.


Codon of mRNA and anticodon in tRNA are antiparallel (Harper 30/e page 395-397)
in their complementarity. • DNA Polymerase is the enzyme of replication.
3. DHU Arm 31. Apo B48 and Apo B100 is synthesized from the
Contain Dihydrouracil residue. mRNA; the difference between them is due to:
Acts as the recognition site for specific aminoacyl (AIIMS May 2011)
tRNA synthetase. a. RNA splicing
4. Pseudouridine Arm (TψC arm) b. Allelic exclusion
c. Deamination of cytidine to uridine
‒ TψC stands for Ribothymidine, Pseudouridine,
d. Upstream repression
Cytidine
Ans. c. Deamination of Cytidine to Uracil.
‒ Involved in the binding of aminoacyl tRNA to the
m

Ribosomal surface. This helps in the formation of (Ref: Harper 30/e page 393)
co

initiation complex. mRNA editing


• The central dogma states that for a given gene and
28. True about 3’ exonuclease: (PGI May 2014) gene product there is a linear relationship between
a. Cleave 3’ end of DNA the coding sequence in DNA, the mRNA sequence,
b. Cleave 5’ end of DNA and the protein sequence
c. Cleave 3’ end of RNA • Changes in the DNA sequence should be reflected in
a change in the mRNA sequence and, depending on
d. Cleave 5’ end of RNA
codon usage, in protein sequence
Ans. a. Cleaves 3’ end of DNA. • mRNA editing is the process by which coding
information is changed at the mRNA level by
29. Which is a reverse transcriptase: (Jipmer 2014)
chemical modification of the nitrogenous bases
a. Topoisomerase present in the codons
m

b. Telomerase • Hence, the linear relationship between the coding


co

c. RNA polymerase II sequence in DNA, the mRNA sequence, and the


d. DNA polymerase alpha protein sequence is altered
Ans. b. Telomerase. • Thus, it is an exception to Central Dogma of
molecular genetics.
Reverse transcriptase is RNA dependent DNA Polymerase:
An example is the apolipoprotein B (apoB) gene and
• Option a: Topoisomerase is a nicking resealing
mRNA.
enzyme, not a reverse transcriptase
• In the liver, the single apoB gene is transcribed into
• Option b: Telomerase has reverse transcriptase an mRNA that directs the synthesis of a 100-kDa
activity protein, apoB100
• Option c: RNA polymerase II is an DNA dependent • In the intestine, the same gene directs the synthesis
RNA Polymerase, not a reverse transcriptase of the primary transcript
• Option d: DNAP alpha is a eukaryotic DNA • A cytidinedeaminase converts a CAA codon in the
m

dependent DNA Polymerase. But also has Primase mRNA to UAA at a single specific site
co

activity • Rather than encoding glutamine, this codon becomes


Primase is DNA dependent RNA Polymerase, not a a termination signal, and a 48-kDa protein (apoB48)
reverse transcriptase. is the result.
e

e
m

m
m
co

12 Translation
m
co

Topics Included
• Translation
• Codon and Genetic Code
• Inhibitors of Protein Synthesis
m
co

TRANSLATION Genetic Code


• The whole set of codons representing all the amino
Definition acids is called Genetic Code
• The process by which message in the genetic code in • Cracking of Genetic Code was done by Marshal
the mRNA is translated into sequence of amino acids Nirenberg and Har Gobind Khorana.
in the proteins.
Salient Features of Genetic Code
Codon • Triplet Codon: Each amino acid is represented by
The triplet nucleotide sequence present in the mRNA triplet sequence
representing specific amino acid. • Degenerate (Redundant)Q
• If 1 base represent 1 amino acid only 4 amino acids ‒ More than 1 codon represent a single amino acid
• If 2 base represent 1 amino acid 42 Aminoacidsie 16 ‒ Degeneracy of the codon lies in the 3rd Base.Q
m

Amino acids Two amino acids with single codon


co

• If 3 base 43, i.e. 64 amino acids ‒ AUG --- Methionine


• If 4 bases 44, i.e. 256 amino acidsQ ‒ UGG ---- Tryptophan
tRNA as an Adapter Molecule • Nonoverlapping
• The language of nucleotide is translated to language of amino Reading of genetic code does not involve overlapping
acid in translation
sequence.
• The codon has no affinity towards the amino acid that it codes
• tRNA acts as the adapter molecule between the codon and the Amino acids with maximum number of codons (Six codons)
specific amino acid • Serine, Arginine, Leucine
• This is possible by two reasons
• Nucleotide sequence of codon is complementary to the anticodon • Unambiguous
in the tRNA ‒ Any specific codon can represent only one amino
• DHU arm recognize the specific Amino acyl tRNA synthetase, so acid.
that specific amino acid bind to the Acceptor arm of the tRNA.
m

For example:
• Universal
co

• UUU is the codon for Phenylalanine ‒ A specific codon represent a specific amino acid
• The tRNA that carries Phenylalanine has GGG in the anticodon in all the species
arm ‒ Exception to this rule–Codons of Mitochondrial
• DHU arm recognizes the Phenylalanyl tRNA Synthetase. DNA.
e

e
m

m
m
co

Translation  | 287

• Initiator codon • The charging reaction has an error rate of less than
‒ In eukaryotes ---- AUG codes for Met 10–4
‒ In Prokaryotes ---- AUG codes for N-Formyl • Hence, aminoacyl tRNA Synthetase is considered as
Methionine the proofreading mechanism of translation
• Terminator Codons • 2 inorganic Phosphates are used in the charging of
‒ UAG ---- Amber the tRNA.
m

‒ UGA ---- Opal


co

‒ UAA ---- Ochre


Exceptions:
• UGA can be recoded to Selenocysteine
• UAG can be recoded to Pyrrolysine
• UGA codes for Tryptophan in mitochondrial DNA.

Wobbling Phenomenon
The base pairing at the 3rd nucleotide between the anticodon in the
tRNA and Codon in the mRNA is not stringently regulated. This is Fig. 12.1: Charging of tRNA
called Wobbling phenomenon.
For example: 2. Initiation
• Two codons for Arginine are AGA and AGG can bind with same
tRNA having UCU in the anticodon arm Identification of initiator codon.
m

• Base pairing at the third nucleotide is not always obeying base- By Marker Sequence—Consensus sequence that helps in
co

pairing rule the identification of initiator codon.


• Thus, it is said degeneracy lies in the third base • Prokaryotes: Shine-Dalgarno sequence
• This explains how 31 tRNA species can bind with 61 coding
codons. • Eukaryotes: Kozak sequence.
The first AUG sequence after the marker sequence is
CistronQ defined as the start codon.
It is the smallest unit of genetic expression which code for a polypep-
tide chain. Monocistronic–one cistron for one Polypeptide AUG codon binds with met tRNA1.
e.g. Eukaryotic mRNA Initiation
Polycistronic—One cistron represents more than one polypeptide
• Is a multistep process
e.g. Prokaryotic mRNA
• Mnemonic: P for P (Prokayotic is Polycistronic). • Facilitated by accessory proteins called Initiation
factors
m

Polarity of Transcription • In case of eukaryotes it is called eukaryotic Initiation


• The message in the mRNA is decoded from 5' end to 3' end Factors (eIF)
co

• The codon in the 5' end of mRNA corresponds to N Terminal amino


acid of the polypeptide. 1. Dissociation of the Ribosome into its 40S and 60S
Subunits
‒ Two initiation factors delays the association of
Steps of Protein Synthesis
40S Subunit and 60S Subunit
1. Charging of tRNA
‒ They are eukaryotic Initiation Factor -3 (eIF-3)
2. Initiation
and eIF-1A.
3. Elongation
2. Formation of 43S Pre-initiation Complex
4. Termination
Three steps:
1. Charging of tRNA ‒ First step involves binding of GTP by eIF-2 to
• The process by which specific amino acid is attached form Binary Complex
m

to the acceptor arm by specific aminoacyl tRNA ‒ Second step involves binding of binary complex
co

synthetase to met tRNAi to form Ternary Complex


• Specific aminoacyl tRNA Synthetase enzyme is ‒ This ternary complex binds to 40S Subunit of
identified by DHU arm Ribosome to form 43S Pre-initiation complex.
e

e
m

m
m
co

288 |  Self Assessment and Review of Biochemistry

In Stressful conditions kinases are activated. 3. Elongation of Polypeptide in Translation


eIF2α is phosphorylated and protein synthesis is arrested during Multistep Process
stressful conditions.
This explains how protein synthesis is decreased in glucose starva- Catalyzed by proteins called Elongation Factors.
tion, viral infection etc.
It involves 4 steps:
3. Formation of 48S Initiation Complex
i. Binding of Aminoacyl tRNA to the A site.
‒ Binding of 43S Pre-initiation complex to the
m

mRNA forms 48S Initiation Complex ii. Peptide Bond Formation.


co

‒ Formation of 48S Initiation Complex require ATP iii. Translocation of ribosome on the mRNA.
Hydrolysis. iv. Expulsion of deacylated tRNA from P and E site.
Factors that facilitate the binding of mRNA to 40S Binding of Aminoacyl tRNA to the A site
Subunit
• tRNA carrying the specific amino acid binds to the
• 5’ methyl Guanosine cap and Cap binding Complex
A site
(Described below)
• 3’ Poly A tail and Poly A tail Binding Protein (PAB-1). • Elongation factor EF-1 helps in the binding of tRNA
aa

Cap Binding Complex


• GTP is hydrolyzed to GDP.
eIF 4F Complex
Peptide Bond Formation
• Consist of eIF-4E and eIF4G - eIF4A
• This cap binding complex bind to 7meG cap though 4E • The alpha amino group of the incoming amino acid
m

• This complex is very important in controlling the rate of translation in the A site forms peptide bond with the COOH
group of the peptidyl tRNA in the P (Peptidyl or
co

• 4E responsible for recognition of mRNA cap structure, is a rate


limiting step of translation.
Polypeptide) site
Insulin and other mitogenic growth factors through AKT/PI3 Kinase • Enzyme is Peptidyltransferase, a ribozyme which is
pathway. a component of 28Sr RNA of 60S Subunit
This phosphorylates 4E binding protein, this makes 4E free.
This facilitates 4F complex to bind mRNA cap through 4E. • NO ENERGY IS REQUIRED FOR THIS PEPTIDE
Thus, Insulin increases the rate of initiation of translation. BOND FORMATION STEP
4. Formation of 80S Initiation Complex • The growing peptide chain is now in the A site.
48S Initiation Complex + 60S subunit of the ribosome Translocation of the ribosome on the mRNA
form 80S Initiation Complex. • The ribosome move forwards, then whole mRNA is
Involves GTP Hydrolysis. shifted by a distance of one codon
Three sites are present in the 80S Ribosome:
m

• Peptidyl tRNA is translocated to P site from A site


i. A site-where the new Aminoacyl tRNA binds.
co

• A site is free to receive the next incoming tRNAaa


ii. P site-where the growing peptidyl chain present.
iii. E site-where the deacylated tRNA is present. • Deacylated tRNA is on E site
• Translocation requires EF-2 and GTP.

Termination of Protein Synthesis


• Releasing factor helps in the termination
• Stop codon is in the A site now
• Releasing factor-1 (RF-1) recognises the stop codon
in the A site
• RF-1 is bound by RF-3 and GTP
m

• This complex with peptidyl transferase promote


co

hydrolysis bond between polypeptide chain and


tRNA
Fig. 12.2: Initiation complex • This involves hydrolysis of GTP to GDP.
e

e
m

m
m
co

Translation  | 289

Eukaryotic Initiation Factors (eIF) Energetics of Peptide Bond Synthesis


eIF-3 and Bind to newly dissociated 40S ribosomal subunit. This Charging of tRNA to tRNAaa – 2 Inorganic Phosphates
eIF-1A allows translation initiation factor to associate with the = 2ATPs
40S subunit
EF1-binding of tRNAaa to A Site- 1GTP
eIF-2 Involved in the formation of binary complex, of 43S
preinitiation complex EF2-Translocation-1GTP
eIF-4F Cap binding complex No Energy for actual peptide bond synthesis.
m

Important in 48S Initiation complex Even though the actual peptide bond formation does
co

eIF-5 Hydrolysis of GTP bound to eIF-2 facilitate 60S as- not require energy, for the formation of one peptide bond
sociation
hydrolysis of 4 inorganic phosphates is required. (2 ATP
for charging of the amino acid 1GTP for EF-1 and 1 GTP
for EF-2 Translocation).
Regulation of Translation
The two control points of translation are:
1. eIF 4E of 4F complex.
2. eIF 2.
Rate of Protein Synthesis
Prokaryotes-18 Amino acids per second.
Eukaryotes-6 Amino acids per second
m

Bacterial Protein synthesis Inhibitors


co

Reversible: Bacteriostatic
• Tetracyclins
• Chloramphenicol
• Erythromycin and Clindamycin.
Irreversible Inhibitors-Bactericidal
• Streptomycin and other aminoglycosides.
Mammalian Protein Synthesis Inhibitors
Inhibitor Mechanism of Action
PuromycinQ Structural analog of tyrosinyl tRNA
Cycloheximide Inhibit peptidyltransferase in the 60S ribosomal
subunit
m

Diphtheria toxinQ Exotoxin of Corynebacterium diphtheriae


co

Catalyzes ADP ribosylation of:


Elongation Factor-2 on the unique amino acid
diphthamide in mammalian cell
This inactivates EF-2, specifically inhibits
mammalian protein synthesis
RicinQ From castor bean inactivates eukaryotic 28S
ribosomal RNA

Post–translational Modifications
• Covalent modifications of aminoacyl residues
• Gamma carboxylation
• Hydroxylation
• Methylation
m

• Glycosylation
co

• Zymogen activation.
Polysome or Polyribosome
• Multiple ribosome on the same mRNA are called Polysome or
Fig. 12.3: Elongation of translation Polyribosome
e

e
m

m
m
co

290 |  Self Assessment and Review of Biochemistry

REVIEW QUESTIONS

1. A codon consists of: (AIIMS 90, UP 99, WB 02) 5’ end of anticodon suggesting the binding of 3rd
a. One molecule of aminoacyl-tRNA nucleotide of codon and anticodon is not strictly obeying
b. Two complementary base pairs the Watson–Crick base pairing rule. This is called Wobble.
c. 3 consecutive nucleotide units It is between nucleotide in the 3’ end of codon and 5’ end
m

d. 4 individual nucleotides of anticodon.


co

Ans. c. 3 consecutive nucleotides.


4. Components of 50s subunit is/are:
(Ref: Harper 30/e page 415)
(PGI May 2011)
• Each codon consists of a sequence of three nucleotides,
i.e. it is a triplet code. a. 23S
b. 28S
2. All are true of genetic code except: c. 5S
a. Degenerate (DNB 2001, Delhi 98, TN 95) d. 5.8S
b. Universal
e. 16S
c. Punctuation
Ans. a, c. 23S, 5S. (Ref: Harper 30/e page 367)
d. Nonoverlapping
Ans. c. Punctuation. (Ref: Harper 30/e page 415) Ribosomal Assembly in Prokaryotes
Features of the Genetic Code 70S Ribosome = 30S + 50S Subunits
m

• Degenerate In 30S Subunit 16S rRNA + Proteins


co

• Unambiguous
In 50S rRNA 23S rRNA + 5S rRNA + Proteins.
• Nonoverlapping
• Not punctuated Ribosomal Assembly in Eukaryotes
• Universal. 80S Ribosome = 40S Subunit + 60S Subunit
3. Wobble hypothesis – regarding the variation true 60S Subunit = 28S rRNA + 5.8S rRNA + 5S rRNA + ~50
is: (PGI Dec 07) proteins
a. 3-end of anticodon 40S = 18S rRNA + ~30 proteins.
b. 5-end of anticodon
5. Ribosome 60S Subunit contains: (PGI Nov 2009)
c. mRNA
a. 5.8S subunit
d. tRNA
Ans. b. 5-end of anticodon. (Harper 30/e page 416) b. 23S subunit
c. 28S subunit
m

d. 16S subunit
co

e. 18S subunit
Ans. a, c. 5.8S Subunit, 28S Subunit.
(Ref: Harper 30/e page 367)

6. There are 20 amino acids with three codons in spite


of the number of amino acids could be formed is
64 leading to that an amino acid is represented by
more than one codon is called: (AIIMS May 2012)
a. Transcription
b. Degeneracy
c. Mutation
d. Frame shift
m

Ans. b. Degeneracy. (Ref: Harper 30/e page 415)


co

Genetic Code
The whole set of codons representing all the amino acids
is called Genetic Code.
e

e
m

m
m
co

Translation  | 291

Cracking of Genetic Code was done by Marshal c. Mutation


Nirenberg and Har Gobind Khorana d. Frameshift
Two amino acids with single codon Ans. b. Degeneracy. (Ref: Harper 30/e page 415)
AUG ---- Methionine
UGG ---- Tryptophan. 8. The polypeptide from poly (A) is:
(AIIMS Nov 2012)
Salient Features of Genetic Code a. Polylysine
m

1. Triplet Codon: Each amino acid is represented by b. Polyglycine


co

triplet sequence. c. Polyproline


2. Degenerate (Redundant) d. Polyalanine
More than 1 codon represent a single aminoacid. Ans. a. Polylysine. (Ref: Harper 30/e page 415)
Degeneracy of the codon lies in the 3rd Base. Poly A codes for Lysine
3. Nonoverlapping Poly C codes for Proline
Reading of genetic code does not involve overlapping Poly G codes for Arginine
sequence.
Poly U codes for Phenylalanine.
Amino Acids with maximum number of codons (Six
codons). 9. If constitutive sequence of 4 nucleotide codes
Serine, Arginine, Leucine for 1 amino acid, how many amino acids can be
4. Unambiguous theoretically formed? (Nov 2012)
m

A codon can represent only one amino acid. a. 4


co

5. Universal b. 64
A specific codon represent a specific amino acid in c. 16
all the species. d. 256
Exception to this rule–Codons of Mitochondrial DNA Ans. d. 256. (Ref: Harper 30/e page 415)
6. Initiator codon Codon
In Eukaryotes --- AUG codes for Met The triplet nucleotide sequence present in the mRNA
In Prokaryotes --- AUG codes for N-Formyl representing specific amino acid
Methionine. If 1 base represent 1 amino acid only 4 amino acid.
7. Terminator Codons If 2 base represent 1 amino acid 4 2 Amino acids,
UAG --- Amber i.e. 16 amino acids.
UGA --- Opal If 3 base 43, i.e. 64 amino acids.
m

UAA----Ochre. If 4 bases 44, i.e. 256 amino acids.


co

Exceptions: 10. False about eukaryotic protein synthesis:


‒ UGA can be recoded to Selenocysteine (AIIMS May 2009)
‒ UAG can be recoded to Pyrrolysine a. N formyl Met is the first tRNA to come into
‒ UGA codes for Tryptophan in mitochondrial action
DNA. b. mRNA read from 3’ to 5’
8. Wobbling Phenomenon c. eIF-2 shifts between GDP to GTP
The base pairing at the 3rd nucleotide between the d. Capping helps in attachment of mRNA to 40S
anticodon in the tRNA and Codon in the mRNA is ribosome
not stringently regulated. Ans. Both a and b. (Ref: Harper 30/e page 422)
Q. 7. Genetic code has triplet of nucleotides each for N formyl Met is the first tRNA to come into action
one amino acid. When an amino acid is specified mRNA read from 3’ to 5’.
m

by more than one codon, it is called: Explanation


co

(AIIMS Nov 2012) Option a


a. Transcription First tRNA to come into action is methionyl tRNA in
b. Degeneracy eukaryotes and N Formyl tRNA in prokaryotes.
e

e
m

m
m
co

292 |  Self Assessment and Review of Biochemistry

Option b 13. Part of eukaryotic DNA contributing to poly-


The translation of the mRNA commences near its 5’ peptide synthesis (PGI May 2011)
terminal with the formation of the corresponding amino a. Exon
terminal of the protein molecule. b. Enhancer
The message is read from 5’–3’ for translaton. c. Leader sequence
d. tRNA
Option d
m

e. ncRNA
4E subunit of eIF–4F that is bound to the cap forms a
co

Ans. a. Exon.
circular structure that helps direct the 40S ribosomal
subunit to the 5’ end of the mRNA. 14. Stop codons are: (PGI Nov 2010)
The poly (A) tail stimulates recruitment of the 40S a. UAA
ribosomal subunit to the mRNA. b. UAG
Both Poly A tail and 5’ cap is needed for the attachment c. UGA
of mRNA to the 40S Subunit. d. UAC
e. UCA
11. Termination process of protein synthesis is Ans. a, b, c. UAA, UAG, UGA.
performed by all except: (PGI May 2010) (Ref: Harper 30/e page 415)
a. Releasing factor
Terminator Codons/Stop Codons/Nonsense Codons
b. Stop codon
UAG --- Amber
m

c. Peptidyltransferase UGA --- Opal


co

d. UAA codon UAA --- Ochre.


e. AUG codon Exceptions
Ans. e. AUG codon. (Ref: Harper 30/e page 423) UGA can be recoded to Selenocysteine.
Termination of Protein Synthesis UAG can be recoded to Pyrrolysine.
Stop codons specifies the sites to stop translation. UGA codes for Tryptophan in mitochondrial DNA.
Releasing factor-1 along with Releasing factor 3 helps in 15. Met-tRNA would recognize: (PGI Nov 2009)
the termination a. AUG
Hydrolysis of GTP to GDP b. GCA
Peptidyltransferase also helps in the hydrolysis of the c. GUA
polypeptide chain. d. UAC
m

12. True about Ribozyme (AIIMS Nov 2012) e. GAC


(Ref: Harper 30/e page 415)
co

a. Peptidyltransferase activity Ans. a. AUG.


b. Cuts DNA at specific site The first AUG sequence after the marker sequence is
defined as the start codon. AUG codon binds with Met
c. Participate in DNA synthesis
tRNA1.
d. GTPase activity
Ans. a. Peptidyltransferase activity. 16. Which of the following statement is true?
(Ref: Harper 30/e page 411) (Ker 2008)
Ribozyme is RNA with catalytic activity. a. N-formylmethionine is the precursor of
eukaryotic polypeptide synthesis
• E.g.: Sn RNA in Spliceosome–Takes part in splicing
of exons and removal of introns b. Eukaryotic ribosomes are smaller than
prokaryotic
• Ribonuclease P- Cuts the RNA
c. Identification of 5’ cap of mRNA by IF4E is the
• Peptidyltransferase: Peptide Bond formation.
m

rate limiting step


Other options
co

d. Elongation factor 2 shuttles between ADP and


Cuts the DNA at specific site—Restriction ATP
Endonuclease Ans. c. Identification of 5’ cap of mRNA by IF4E is the
DNA Synthesis—DNA Polymerase. rate limiting step. (Ref: Harper 30/e page 422)
e

e
m

m
m
co

Translation  | 293

Rate limiting steps of Protein Translation are: c. Initiating codon


eIF 2 d. Codon for more than one amino acids.
eIF 4F (specifically saying 4E of 4 F complex) Ans. b. Stop codon (Ref: Harper 30/e page 415)
Option a: Methionine is the first amino acid in • Amber is UAG stop codon
eukaryotic protein synthesis • Ochre is UAA stop codon
Option b: Eukaryotic Ribosome is bigger than • Opal is UGA stop codon.
m

Prokaryotic ribosome
co

Option d: Elongation Factor 2 is associated with 20. Shine-Dalgarno sequence in bacterial mRNA is
hydrolysis of GTP not ATP. near: (AI 2004)
a. AUG codon
17. RNA polymerase differs from DNA polymerase b. UAA codon
(Ker 2007)
c. UAG codon
a. It edits and synthesis
d. UGA codon
b. Synthesize RNA primers
Ans. a. AUG codon. (Ref: Harper 30/e page 415)
c. Synthesis only in 5 to 3 direction
d. Uses RNA templates Shine-Dalgarno sequence is the marker sequence. The
first AUG codon after Shine-Dalgarno sequence is the
Ans. b. Synthesize RNA primer.
start codon in bacteria. Similarly in eukaryotes there is
(Ref: Harper 30/e page 415)
Kozak sequence.
• Option a: RNA Polymerase do not have editing
m

function 21. True regarding aminoacyl tRNA synthetase is


co

• Option c: Both DNA Polymerase and RNA Polymerase A/E: (SGPGI 06)
synthesize in 5’ to 3’ direction a. Is accepting tRNA
• Option d: Both use DNA as template b. Implement genetic code
• Option b: RNA primer is synthesized by RNA c. Attachment of amino group to 5’ end of tRNA
Polymerase. d. Editing function
Comparison between DNA Polymerase and RNA Polymerase Ans. c. Attachment of amino group to 5’ end of tRNA.
DNA Polymerase RNA Polymerase (Devlin 7/e page 216)
Involved in DNA Synthesis Involved in RNA Synthesis The reaction of Aminoacyl tRNA synthetase
(Replication) (Transcription)
Amino acid + ATP + Enzyme → Amino acid-AMP-
Needs a primer Do not need a primer
Enzyme complex + PPi
Synthesize in 5’ to 3’ direction Synthesize in 5’ to 3’ direction
m

Amino acid - AMP - Enzyme complex + tRNA →


Has DNA repair and proof- Do not have DNA repair and
Aminoacyl tRNA + AMP + Enzyme.
co

reading activity proofreading activity


• Aminoacyl tRNA synthetase first accept ATP and
Amino acid
18. The cellular component for protein synthesis is:
(AI 97) • Then the enzyme accepts the specific tRNA
a. Smooth endoplasmic reticulum • Amino acid is attached to 3’ hydroxyl adenosyl end
b. Rough endoplasmic reticulum of tRNA
c. Ribosomes • Two inorganic phosphates are used
d. Mitochondria • This enzyme is a part of editing mechanism of
Ans. c > b. Ribosome > RER. translation.
Rough endoplasmic reticulum is ER studded with 22. In translation process, proofreading of mRNA is
Ribosome, there also in Ribosome protein synthesis done by: (AIIMS Dec 97)
m

takes place. a. RNA polymerase


co

19. Amber codon refers to: (AIIMS May 01) b. Aminoacyl tRNA synthetase
a. Mutant codon c. Leucine zipper
b. Stop codon d. DNA
e

e
m

m
m
co

294 |  Self Assessment and Review of Biochemistry

Ans. b. Aminoacyl tRNA synthetase. esterified carboxyl group of the peptidyl-tRNA


(Ref: Harper 30/e page 415) occupying the P site (peptidyl or polypeptide site).
• Amino acyl tRNA synthetase which recognises the At initiation, this site is occupied by the initiator
specific tRNA is a proofreading mechanism. met-tRNAi
• This reaction is catalyzed by a peptidyltransferase,
23. Which enzyme involved in translation is often a component of the 28S RNA of the 60S ribosomal
referred to as ‘Fidelity enzyme’: (AI 1998)
m

subunit
a. DNA polymerase
co

• This is another example of ribozyme activity.


b. RNA polymerase
c. Aminoacyl tRNA synthetase 26. Termination is caused by all except:
d. Aminoacyl reductase (AIIMS Dec 90)
Ans. c. Aminoacyl tRNA synthetase. a. RF-1
(Ref: Harper 30/e page 415) b. UAA
c. Peptidyltransferase
24. The hydrolytic step leading to release of poly-
d. 48S complex
peptide chain from ribosomes is catalyzed by:
Ans. d. 48S complex. (Ref: Harper 30/e page 419)
(PGI JUNE 02)
Termination is helped by RF-1, RF-3, stop codon and
a. Stop codons
peptidyltransferase.
b. Peptidyl transferase
m

c. Releasing factors 27. True about translation of protein is: (PGI Dec 98)
co

d. AUG codon a. It has 3 steps: initiation, elongation, termination


e. Dissociation of ribosomes b. IF-2 prevent reassociation of ribosomal subunit
Ans. b, c. Peptidyl transferase, Releasing factors. c. IF-3 and 1A cause binding of initiation factors
(Ref: Harper 30/e page 423) d. IF 2 has α and β units
• Releasing factor RF1 recognises that a stop codon Ans. a, c. It has 3 steps …, IF-3 and 1A cause binding …
resides in the A site (Ref: Harper 30/e page 419)
• RF1 bound by a complex consisting of releasing factor • Like transcription, protein synthesis can be described
RF3 bound by GTP in three phases, Initiation, Elongation and Termination
• This complex, along with peptidyltransferase • eIF3 and eIF 1A bind to newly dissociated 40S
catalyses the hydrolysis of bond between peptide ribosomal unit and prevent its reassociation with
and tRNA occupying the P site 60S subunit till translation initiation factors are all
m

• Stop codon is not catalysing the hydrolytic cleavage, associated


co

so it cannot be the answer. • eIF-2 is a part of binary complex


25. About peptidyltransferase true is: (JIPMER 2000) • eIF-2 consist of α, β and γ subunits
a. Used in elongation and cause attachment of • eIF is one of the control points of translation.
peptide chain to A- site of tRNA
28. 43S preinitiation complex include all except:
b. Used in elongation and cause attachment
(AIIMS Dec 93)
peptide chain to P site
a. IF-3
c. Used in initiation and cause 43S complex
formation b. IF-1A
d. Used in initiation and cause 48S complex c. IF-2
formation d. IF-4F
Ans. a. Used in elongation and cause attachment of Ans. c. IF-2 (Ref: Harper 30/e page 419)
m

peptide chain to A-site of tRNA. 43 S Preinitiation complex consist of


co

(Ref: Harper 30/e page 422) • GTP-eIF2-tRNAi + 40S subunit


• The α amino group of the new aminoacyl-tRNA in • This 43 S Preinitiation complex is stabilized by eIF-3
the A site carries out a nucleophilic attack on the and eIF-1A.
e

e
m

m
m
co

Translation  | 295

29. IF 4F include all except: (PGI June 97, JIPMER 99) d. 3


a. 4A e. 4
b. 4G Ans. a. 0. (Ref: Harper 30/e page 422)
c. 4E For peptide bond formation no energy is required as the
d. 4S aminoacyl tRNA is already activated.
Ans. a, b, c. 4A, 4G, 4E.
31. Vitamin required for post-translational modifi-
m

(Ref: Harper 30/e page 422)


cation of coagulants is: (AI 97)
co

• Capbinding complex consists of 4A, 4G, 4E. a. Vitamin A


30. For 1 peptide bond formation how many high b. Vitamin C
energy phosphate bonds are required? (PGI Dec c. Vitamin B6
07) d. Vitamin K
a. 0 Ans. d. Vitamin K.
b. 1 Gamma carboxylation of Clotting factors require
c. 2 Vitamin K.
m
co
m
co
m
co
e

e
m

m
m
co

13 Regulation of Gene
Expression
m
co

Topics Included
• Regulation of Gene Expression at Different Levels • Mitochondrial DNA
• Lac Operon • Mutation
• Epigenetics • Patterns of Inheritance
• miRNA and SiRNA • DNA Polymorphisms
m
co

Gene Expression • Noncoding RNA-induced regulation (Described in


• Organisms adapt to the environmental changes by Chapter Transcription)
altering gene expression • Epigenetic modifications.
• Most common regulation is modulation of trans- At the level of DNA
cription. • Gene Amplification
Housekeeping Gene (Constitutive Gene) • Gene Rearrangement
• Genes which are expressed at a constant rate in • Transposition of DNA.
almost all the cells of the body
Operon Concept
• Required for the basal cellular function, e.g. Enzymes
• Lactose Operon by Francois Jacob and Jacques
for glycolysis.
m

Monod in 1961
Inducible Gene
co

• Operon is the linear array of genes involved in a


• Genes which are expressed under special circum- metabolic pathway, e.g. Lac operon concerned with
stances. lactose metabolism in prokaryotes.
• Increase in response to an activator or inducer. Lac Operon Comprises of
• Decrease in response to a repressor. i. Three structural genes coding for 3 proteins:
Regulation of Gene Expression at Different ‒ Lac z—Beta galactosidase
Levels ‒ Lac y—Permease-a carrier protein that helps
permeation of lactose to cell
At the level of Transcription
‒ Lac a—Thiogalactoside Transacetylase – function
• Induction and Repression (Explained by Operon
not known.
Concept)
ii. Regulatory genes comprises of:
• Alternate mRNA Processing (Described in Chapter
‒ Lac promoter
m

Transcription)
‒ Lac operator
co

• RNA Editing (Described in Chapter Transcription)


‒ Lac I encodes lac operon repressor protein.
• mRNA Stability
e

e
m

m
m
co

Regulation of Gene Expression  | 297


m
co
m
co
m
co

Fig. 13.1: Lac operon

Role of Catabolite Activator Protein (CAP) in • CAP is complexed with cAMP


m

Lac Operon • CAP-cAMP Complex facilitates binding of RNA


co

CAP is a Positive RegulatorQ of Lac Operon Polymerase to promoter


• If glucose concentration low-cAMP level is high • Structural genes expressed.
e

e
m

m
m
co

298 |  
Self Assessment and Review of Biochemistry

When Glucose Present and Lactose Absent GENE AMPLIFICATION


• Lac I ----> Repressor Protein active
• Gene amplification is the process by which number
• Binds to the Operator Site of gene available for transcription is increased
• RNA Polymerase cannot move to Promoter site (Fig. 13.2), e.g. Person who is on Methotrexate
• CAP is inactive as cAMP level is low develop resistance to MTX by increasing the number
• Lac Operon Repressed, Structural genes are not of genes for Dihydrofolate Reductase.
m

transcribed.
co

When Glucose Absent and Lactose Present


• Lac I ---> Repressor protein
• The repressor molecule, which has got affinity to
lactose
• This brings a conformational change in the repressor
molecule and it can no more bind to the operator site
• CAP is active as cAMP level is high
• Hence, structural genes are transcribed (De-
repression).
Fig. 13.2: Gene amplification
When Both Glucose and Lactose Absent
m

• CAP inactive as cAMP


co

GENE REARRANGEMENT
• Hence, structural gene not transcribed
• Repression of Lac operon. • Different gene segments are brought together in
different combination is called Gene rearrangement
Gratuitous Inducer
(Fig. 13.3).
• Lactose analogs capable of inducing Lac operon,
e.g. Isopropyl Thiogalactoside (IPTG).
• The IgG light chain is composed of variable (VL),
joining (JL), and constant (CL) domains or segments.
Concept • For particular subsets of IgG light chains, there are
• Whenever Glucose is present irrespective of the presence or roughly 300 tandemly repeated V L gene coding
absence of lactose, the lac operon is switched off. segments, 5 tandemly arranged JL coding sequences,
• Whenever Glucose is absent lac operon is on.
and roughly 10 CL gene coding segments.
m
co
m
co

Fig. 13.3: Gene rearrangement in immunoglobulin


e

e
m

m
m
co

Regulation of Gene Expression  | 299

• During B-Lymphocyte development, different VDJ 4. Stem loop structure in 3’ end region prevents
segments combine to form unique variable region in exonuclease attack in histone mRNA which lack Poly
immunoglobulin. A tail
• This allows generation of 109-1011 different immu- 5. Stem loop structure in 3’ end is critical for regulation
noglobulin from single gene. of mRNA encoding transferring receptor
6. Presence AU rich region in the 3’ UTR of certain
m

TRANSPOSONS (JUMPING GENE) mRNA shortens its half life of certain mRNA
co

7. Seed sequence in the 3’ UTR determine the specificity


• DNA sequences that move to new positions within of binding of miRNA to mRNA.
the genome
• Discovered by Barbara McClintock. EPIGENETICSQ
• Transposase enzyme help in this process.
Reversible heritable chemical modificationQ of DNA or
Retroposons histone or nonhistone proteins that does not alter DNA
• DNA sequence move from one segment to another sequence itself.
through an RNA Intermediate • The term epigenetics means‚ above genetics’ as the
• DNA segment is converted to RNA, RNA moves to nucleotide sequence is unaltered
another location, where it is reversely transcribed • This is one of the recently discovered method of
to a DNA. regulation of gene expression.
m

Epigenome: Constellation of covalent modification of


GENE SWITCHING DNA and histones that impact chromatin structure and
co

modulation of gene expression.


The process by which one gene is switched off while
a closely related gene take up its function, e.g. during Epigenetic modifications include
primary immune response genes for IgM is active but • DNA Methylation
during secondary immune response genes for IgG is • Post-translational modification of Histones.
active.
Functional Consequences of Chemical
Modification of DNA
GENE SILENCING
DNA Methylation and demethylations
• The process by which a gene is switched off
• DNA methylation is usually restricted to Cytosine
• By various epigenetic mechanisms (described later) residues in CpG dinucleotide of CpG islands
• By RNA interference (RNAi) by nc RNAs (described
m

• Enzyme responsible for methylation is Methyl


in chapter Transcription). transferases
co

• DNA methylation generally decreases the gene


REGULATION OF mRNA STABILITY expression or gene silencing
• mRNA exist in cytoplasm as Ribonucleoprotein • Acute DNA demethylation, increases the rate of
particle (RNP) transcription.
• Much of the mRNA metabolism likely to occur in Gene promoters and regulation by methylation-demethylations
P bodies. • Majority of gene promoters have high CG content.
• CpG islands are the common sites of methylation-demethylations
RNP can help mRNA in two ways: • Hence, methylation-demethylations plays an important role in
• Proteins protect mRNA from Nuclease regulation of gene expression.
• CpG islands of promoters are typically unmethylated, it favors
• It can promote nuclease attack transcription.
mRNA stability is determined by
m

1. 5’ cap that prevents 5’ exonuclease attack Histone Covalent Modifications


co

2. 3’ Poly A tail prevent 3’ exonuclease attack • Also known as “The Histone code.”
3. Other structures in coding region, 5’ Untranslated • Wide range of Post translational modifications.
region, 3’ Untranslated regions of mRNA • They are dynamic and reversible.
e

e
m

m
m
co

300 |  
Self Assessment and Review of Biochemistry

Histone Acetylation and Deacetylation Biochemical Functions of Epigenetic


• Histone acetylation and deacetylation is the best ModificationQ
understood modification of histones 1. Regulation of tissue specific gene expression
• Acetylation occurs on lysine residues in the amino 2. X chromosome inactivation (Facultative Heterochro-
terminal tails of histone molecules matin) one of the two X-chromosomes in every cell
• Histone acetylation increases the gene transcription of a female
m

• Histone Acetyltransferase (HAT) acetylate the 3. Genomic imprinting: Gene inactivation on selected
co

histones chromosomal regions of autosomes is called Genomic


Imprinting. This is the cause of preferential expres-
• Histone deacetylases (HDAC) deacetylate the
sion of one of the parental allele.
histones.
Maternal imprinting refers to transcriptional silencing
Mechanism of histone acetylation and deacetylation of the maternal allele.
• Histone acetylation reduces the positive charge of amino terminal Paternal imprinting implies that the paternal allele is
tails of histone molecules.
inactivated.
• Decreases the binding affinity for the negatively charged DNA.
Imprinting occurs in the ovum or the sperm, before
• This increases euchromatin formation.
fertilization.
• Transcription factors can bind to the promoters.
• Histone deacetylation has the opposite effect, i.e. decreases 4. Ageing Process
transcription and increases heterochromatin formation.
Epigenetic Changes Causing Pathological
m

Alteration
co

Histone methylation
1. Fragile X syndrome
• Addition of methyl group to specific lysine residues,
‒ Promoter site hypermethylation causing FMR-1
but rarely to arginine residues
gene silencing
• Histone methyltransferases is the enzyme.
2. Cancer
Functional consequence—Alter chromatin configuration, ‒ DNA methylation and histone modifications dictate
which favor or decrease transcription. which genes are expressed, which in turn deter-
Histone phosphorylation mines the lineage commitment and differentia-
• Serine residues can be modified by phosphorylation; tion state of both normal and neoplastic cells
• Functional consequences—Depending on the specific ‒ Local promoter hypermethylation of tumor
residue the DNA may be opened up for transcription suppressor gene leads decreased expression of
or condensed to become inactive. the tumor suppressor gene.
m

3. DNA methylation is considered as a defence


co

Other Histone Modification mechanism that minimise the expression of retroviral


incorporated sequences
Possible role of the modifica-
Histone modification tion
4. Genomic Imprinting
Acetylation of core histonesQ Increases transcription and in- Prader-Willi and Angelman Syndrome
creased expression of the gene The molecular basis of these two syndromes lies in
Phosphorylation of histone H1 Condensation of chromosomes the Genomic Imprinting.
during the replication cycle.
Prader- Willi Syndrome
ADP-ribosylation of histones DNA repair
• A gene or set of genes on maternal chromosome 15q12
Methylation of histones Activation and repression of gene
is imprinted (and hence silenced)
transcription.
• The only functional allele (s) are provided by the
Monoubiquitylation Gene activation, repression, and
m

heterochromatic gene silencing.


paternal chromosome.
co

Sumoylation of histones Transcription repression Causes of Prader-Willi Syndrome


(SUMO; small ubiquitin-relat- • Paternal deletion of Prader-Willi locus on located on
ed modifier)
Chr 15 (Most common cause)
e

e
m

m
m
co

Regulation of Gene Expression  | 301

• Maternal uniparental disomy CHROMATIN IMMUNOPRECIPITATION (ChIP)


• Rarely due to imprinting defect in paternal chromo- This method allows precise localization of a particular protein or modi-
some. fied protein on the DNA, e.g. acetylated, phosphorylated histones,
etc.) on a particular DNA sequence element in living cell.
Angelman Syndrome
RNA IMMUNOPRECIPITATION (RIP)
• A distinct gene that also maps to the same region • An RNA immunoprecipitation method, performed like ChIP, which is
of chromosome 15 is imprinted on the paternal used to score specific binding of a protein to a specific RNA in vivo.
m

chromosome. • RIP uses formaldehyde cross linking to induce covalent attachment


co

• Only the maternally derived allele of this gene is of proteins to RNA


normally active. • CLIP: A method that uses UV cross-linking to induce covalent
attachment of distinct proteins to specific RNAs in vivo.
Causes of Angelman Syndrome
• Maternal deletion of corresponding locus on located Remember
on Chr 15 (Most common cause) • ChIP is Chromatin immunoprecipitation
• Chip is hybridization on DNA Chip or Microarray.
• Paternal uniparental disomy
• Rarely due to imprinting defect in maternal
chromosome. Therapeutic Application of Epigenetic
Some Examples of Tumor Suppressor Gene Silencing by
Modification
Epigenetic Mechanism Unlike DNA mutations, epigenetic changes are potentially
• CDKN2A, a complex locus that encodes two tumor suppressors, reversible by drugs that inhibit DNA or histone-
p14/ARF and p16/INK4a p14/ARF is epigenetically silenced modifying factors.
m

• Colon and Gastric cancers.p16/INK4a is silenced in a wide variety


Thus, there is considerable interest in treating cancers
co

of cancers like
• Bladder cancer, Head and Neck Cancer, ALL, Cholangiocarcinoma with drugs that correct epigenetic abnormalities in cancer
• BRCA-1 Silencing–Cancer Breast cells, with some encouraging early results.
• VHL Silencing-Renal cell Cancer 1. Drugs that inhibit DNA Methyltransferases (DNMT
• MLH-1 Silencing-Colon Cancer. inhibitor)
‒ Azacytidine
Molecular Methods to Detect Epigenetic ‒ 5-aza-2’-deoxycytidine
Modifications in the Genome ‒ Decitabine
Traditional Sanger sequencing alone cannot detect 2. Drugs that inhibit histone Deacetylases (HDAC
epigenetic modification. inhibitor)
1. Methylation-specific PCR can detect DNA ‒ Vorinostat
methylations. ‒ Valproic acid
m

2. DNA Chromatin Immunoprecipitation (ChIP)


co

followed by Microarray hybridization analysis or MITOCHONDRIAL DNA


Direct Sequencing (ChIP-Chip or ChIP-Seq) detect
histone modifications. • 1% of cellular DNA is mitochondrial DNA.
3. Bisulphite Sequencing. • Mitochondria possess its own DNA and protein
4. Methylation Sensitive Restriction Endonuclease synthesizing machinery.
Digestion. • Human mitochondria contains 2–10 copies of a small
circular ~16 kbp dsDNA molecule.
Principle of Bisulphite Sequencing
• Composed of Heavy (H) and Light (L) chain or
• Sodium bisulphite convert unmethylated cytosine to strands.
uracil, which function like thymine in base pairing. • Contain 16,569 bp.
• But methylated cytosines are protected from
modification, hence remain unchanged. Mitochondrial DNA Encodes 37 Structural Genes
m

Procedure They incude


co

• Treat genomic DNA with Sodium Bisulphite. • 2 rRNAs


• Thus, unmethylated (modified) DNA is discriminated • 22 mitochondrial tRNAs, large 16S rRNA and small
from the methylated (unmodified) DNA on the basis 12S rRNA.
of sequence analysis. • 13 protein subunits of respiratory chain.Q
e

e
m

m
m
co

302 |  
Self Assessment and Review of Biochemistry

‒ Seven subunits of NADH Dehydrogenase ii. TransversionQ: A purine base replaced by another
(Complex I) pyrimidine, or pyrimidine replaced by another
‒ Cytochrome b of Complex III purine.
‒ Three subunits of Cytochrome C Oxidase Effects of Base Substitution
(Complex IV) I. Silent mutation
‒ Two subunits of ATP synthase. ‒ If a mutation does not alter the polypeptide
product of the gene
m

Mitochondria has Unique Genetic Code


‒ Also called synonymous mutation
co

CodonsQ Nuclear DNA Code Mitochondrial DNA Code II. Missense mutation
AUA Isoleucine Methionine ‒ The alteration in the nucleotide may result in the
UGA Stop codon Tryptophan incorporation of a different amino acid
AGA, AGG Arginine Stop codon A missense may be:
i. Acceptable—No clinical symptoms
Unique Features of Mitochondrial DNA e.g. Hb Hikari
1. Mutation rate is very high because ii. ii. Partially acceptableQ
‒ No Introns e.g. Hb SQ
‒ No protective histones iii. iii. Unacceptable
‒ No effective repair enzymes e.g. HbM
‒ It is exposed to oxygen free radicals generated Another classification of Mis-sense Mutation
m

by oxidative phosphorylation. 1. Conservative mutation


co

2. Non-Mendelian type of inheritance A missense mutation in which one amino acid is


Cytoplasmic inheritance or Matrilinear inheritance. replaced by a similar amino acid
(Described below) 2. Non-conservative mutation
Mitochondrial disease with no maternal inheritance A missense mutation in which one amino acid
‒ Pearson syndrome is replaced by an amino acid with different
‒ Kearns-Sayre syndrome (KSS) characteristics
3. Heteroplasmy e.g. HbS
‒ Heteroplasmy is defined as the presence of III. Nonsense mutation
normal and mutant DNA in different proportions ‒ A coding codon mutated to a nonsense codon
in different cells result in premature termination of polypeptide
‒ Mitochondrial disease with homoplasmy chain.
‒ Leber’s hereditary optic neuropathy IV. Splice site mutation
m

‒ Sensorineural deafness ‒ Mutation at the splice site results in faulty splicing


co

V. Promoter site mutation


MUTATION ‒ Results in altered gene expression
VI. Frame shift mutation
Definition ‒ Due to insertion or deletion of nucleotides that
Any permanent change in the primary nucleotide are not a multiple of three results in frameshift
sequence regardless of its functional consequences. mutation
Mutation rate ~10–10/bp per cell division. ‒ Reading frame is garbled
Types of the Mutation VII. Null mutation
• Point Mutation ‒ A mutation that leads to no functional gene
product is called null mutation
Single base changes in the nucleotide sequence in the
VIII. Constitutive mutation
gene.
‒ Mutation in which a inducible gene mutated to
m

Base Substitution housekeeping gene or constitutive gene


co

Replacement of a single nucleotide by another. These are IX. tRNA suppressor mutation
the most common type of mutation. ‒ The effect of mutation on mRNA can be
i. TransitionQ: A purine base replaced by a purine base suppressed by a mutant tRNA which has a
or a pyrimidine replaced by another pyrimidine. mutant anticodon sequence
e

e
m

m
m
co

Regulation of Gene Expression  | 303

‒ These mutant tRNA which can suppress the effect Contd...


of mutation are called Supressor tRNAs. • Genetic locus: Location of a particular gene on the Chromosome.
Classes of Mutation in Cystic Fibrosis • Alleles
Various mutations can be grouped into six “classes” based on In diploid organism there are two sets of chromosomes.
their effect on the CFTR protein: Therefore, there are 2 copies of each gene
Class I: Defective protein synthesis. These mutations are associ- The different forms of the same gene that are found at the same
ated with complete lack of CFTR protein at the apical surface of locus are called alleles
m

epithelial cells. One allele is received from the father and the other allele from
co

Class II: Abnormal protein folding, processing, and trafficking. the mother.
These mutations result in defective processing of the protein from They are responsible for alternate or contrast character.
the endoplasmic reticulum to the Golgi apparatus; the protein does • Genotype represents the set pattern of gene present in the cell
not become fully folded and glycosylated and is instead degraded • Phenotype is the observed character expressed by the gene
before it reaches the cell surface. The most common class II muta- • Homozygous: Both allele are defective.
tion is a deletion of three nucleotides coding for phenylalanine
• Heterozygous: One allele is normal and the other allele is
at amino acid position 508 (ΔF508). Worldwide, this mutation can
defective.
be found in approximately 70% of cystic fibrosis patients. Class II
mutations are also associated with complete lack of CFTR protein at • Recessive mode of transmission: Phenotypic expression of the
the apical surface of epithelial cells. disease only in the homozygous state.
Class III: Defective regulation. Mutations in this class prevent activa- • Carrier state: In recessive mode of inheritance if the person
tion of CFTR by preventing ATP binding and hydrolysis, an essential carries one abnormal gene it is not phenotypically expressed.
prerequisite for ion transport. Thus, there is a normal amount of CFTR Biochemically it is called Trait.
on the apical surface, but it is nonfunctional. • Dominant mode of transmission: Phenotypic expression even
Class IV: Decreased conductance. These mutations typically occur when one allele is abnormal or heterozygous state.
m

in the transmembrane domain of CFTR, which forms the ionic pore • Autosomal means defective gene is located in the autosomes.
(Somatic)
co

for chloride transport. There is a normal amount of CFTR at the apical


membrane, but with reduced function. This class is usually associated • Sex linked means defective gene is located in the Sex
with a milder phenotype. chromosome.
Class V: Reduced abundance. These mutations typically affect
intronic splice sites or the CFTR promoter, such that there is a reduced Some Common Pedigree Symbols
amount of normal protein.
Class VI: Altered regulation of separate ion channels. Mutations in
this class affect the regulatory role of CFTR. In some cases, a given
mutation affects the conductance by CFTR as well as regulation of
other ion channels. For example, the ΔF508 mutation is both a class
II and class VI mutation.

New Mutation and Gonadal Mosaicism


In some autosomal dominant disorders, phenotypically normal par-
ents have more than one affected children.
m

This is a violation of Mendelian Inheritance.


co

This is an example of single gene disorder-Non Classic Inheritance.


The sudden unexpected occurrence of a condition due to mutation
of a gene is called new mutation.
New mutation occurs post zygotically during early (embryonic)
development.
If the mutation affects only cells destined to form the gonads, the
gametes carry the mutation, but the somatic cells of the individual
are completely normal. Such an individual is said to exhibit germ line
or gonadal mosaicism.
Thus, a phenotypically normal parent who has germ line mo-
saicism can transmit the disease-causing mutation to the offspring
through the mutant gamete, e.g. Achondrodysplasia, Marfan’s syn-
drome, Neurofibromatosis.
m

Some Terms used in Genetics


• Genome: The complete complement of genetic information in a
co

living organism.
• Chromosome: Physical Division of Genome.
• Genes: Functional Division of Genome.
Contd...
e

e
m

m
m
co

304 |  
Self Assessment and Review of Biochemistry

The Methods to Find the Inheritance Pattern from


Pedigree Chart
1. Autosomal Dominant Inheritance
Disorder or trait which is manifested in the
heterozygous state.
m
co

Fig. 13.6: Autosomal recessive with pseudodominance


3. X-linked Recessive Inheritance
Mutant allele present in the X chromosome.
Fig. 13.4: Autosomal dominant inheritance
The characteristics of an autosomal dominant inheri-
tance.
• Males and females are affected in equal proportion.
• Traced through many generation in the family tree.
Hence called vertical transmission.
• Usually one of the parents is affected (Exception is
m

new mutation already explained).


co

• Genetic risk is 50%, i.e. 50% of the progeny will be Fig. 13.7: X-linked recessive inheritance
affected. The characteristics of X-linked recessive inheritance.
Skipping generation • Males are usually affected
Incomplete penetrance in autosomal dominant inheritance • Females are usually carriers
is called skipping generation, i.e. Some individuals inherit
• Affected males will have only carrier females
the mutant gene but are phenotypically normal.
• Carrier females will have affected males
2. Autosomal Recessive Inheritance
• Male to male transmission is never seen.
Disorder or trait which manifest in homozygous state.
Knight move pattern of inheritance
• In X-linked recessive inheritance affected male
transmit the mutant allele to carrier female and never
to a male
m

• Carrier female transmit the mutant allele to the male


co

who is affected
• This pattern of transmission is called Knight move
pattern or diagonal inheritance.
Fig. 13.5: Autosomal recessive inheritance
The difference between hemizygous and homozygous
The characteristics of an autosomal recessive inheritance.
• Males and females are affected in equal proportion • In X-linked or Y-linked inheritance, males with
mutant allele does not have alternative allele in the
• Affected individuals are usually same generation
homologous chromosome, as there is only one X and
(Hence called horizontal transmission)
one Y chromosome
• Consanguineous marriage common
• Hence, male with mutant allele on X or Y Chromosome
• You can find unaffected parents with affected
is called Hemizygous
progeny.
m

• Homozygous means mutant allele is present on both


Pseudodominance in autosomal recessive inheritance
homologous chromosome
co

• If an individual who is homozygous for an autosomal recessive


disorder marry a heterozygous carrier, 50% chance of being 4. X-linked Dominant Inheritance
affected Manifest in heterozygous females and hemizygous
• Resemble an autosomal dominant Pedigree. males.
e

e
m

m
m
co

Regulation of Gene Expression  | 305


m
co

Fig. 13.10: Mitochondrial inheritance


Fig. 13.8: X-linked dominant inheritance
The Characteristics of Mitochondrial inheritance
The characteristics of X-linked Dominant Inheritance. • Females transmit the disease to all her offsprings
• Resemble autosomal dominant pedigree pattern as • Males usually never transmit the disease
50% chance of being affected • This is called Matrilinear inheritance
• Affected males transmit the disease to all females of • Other name is Cytoplasmic inheritance.
next generation Most common inheritance is Autosomal Dominant 65%
• Male to male transmission is never seen Followed by Autosomal Recessive 25% and X-linked Recessive 5%
• This is because the son will receive only the Y
chromosome from the father. DNA POLYMORPHISM
m

5. Y-Linked Inheritance
co

• Normal variation in DNA sequence that have a


‒ Mutant allele is present in the Y chromosome. frequency of at least 1% of population
‒ Other name for Y-linked inheritance is Holandric • Any two individuals share 99.9% of DNA sequence.
inheritance. • DNA variation lies in this 0.1% (1 in 500-1000 bp)
The characteristics of Y-linked inheritance • They are genetic markers that determines the uni-
• Only males are affected queness of an individual (i.e. like your Fingerprints).
• Only male to male transmission is seen Different Types of Polymorphism
• The explanation is simple only Y chromosome carries • Single Nucleotide Polymorphism
the mutant allele.
• Repeat length Polymorphism or Short Tandem
Repeats (Variable Number Tandem repeats)
‒ Microsatellite repeats
m

‒ Minisatellite repeats
co

• Copy Number Variations


• Restricted Fragment Length Polymorphism

Single Nucleotide Polymorphism


• DNA variation in single base pair are called SNPs
(or Snips)
Fig. 13.9: Y-linked inheritance • Most common polymorphism Q (~90% of total
polymorphisms)
Y Chromosome carries the genes: • No of SNPs in human genome is 10 million
• SRY (Sex Reversal Y) codes, Testis determining • Occurrence is one nucleotide in every stretch of
factors. Mutated cause, Sex reversal approximately 100-300 base pairs
• DAZ (Deleted in Azoospermia)
m

• SNPs that are in close proximity are inherited


• AZF (Azoospermic factor gene) Mutated cause together (are linked) is called haplotype
co

Azoospermia or oligospermia • Haplotype map information is referred to as Hap map


6. Mitochondrial Inheritance • May occur anywhere in the genome within the exon,
Mutant allele present in the mitochondrial DNA. within the intron (most common)
e

e
m

m
m
co

306 |  
Self Assessment and Review of Biochemistry

• Used as genetic markers in Linkage and Association Microsatellite repeat length polymorphism
Studies. • The genetic polymorphism ideal for differentiating between two
SNP Genotyping Arrays individuals
• The genetic polymorphism ideal to follow the genetic marker
• Newer types of genomic arrays are designed to transmitted from parent to child
identify single nucleotide polymorphism (SNP) sites • Extensively used for determining paternity and criminal
genome-wide investigation
m

• This technology is the mainstay of genome wide • Used for the detection and quantification of transplant chimerism
in allogeneic hematopoietic stem cell transplant patients (host
co

association studies. and donor).


Repeat length Polymorphism or Short Tandem
Repeats or Variable Number Tandem Repeats Method to Detection of Repeat length Polymorphism
• Short repetitive sequence in human DNA is called • By allele specific PCR
Repeat length Polymorphism
Restricted Fragment Length Polymorphism
• Most common one is dinucleotide repeat involve AC
to TG on other strand. • Special type of Polymorphism (Described later in
Chapter 13)
Depending on the repeat size it is divided into:
• DNA variations that create or abolish a restriction site
Microsatellite: Repeat size of 2-6 bp.
Total length the repeats extend is usually Copy Number Variation (CNV)
<1 kbases • Insertion or Deletion of a segment of genome (involve
m

Mini satellite: Repeat size of 15-70 bp 1 kb to several Mbs)


co

Total length the repeats extend is usually 1 to 3 • About 1500 CNVs detected so far
kbases. • Involve substantial regions of the genome, not single
Application of Repeat Length Polymorphism nucleotide
• Useful as genetic markers in Linkage and Association • De novo CNVs observed among monozygotic twins
Studies • More recently detected
• Familial diagnosis of disease like Polycystic Kidney • 50% occur in the coding regions
Disease • Responsible for human phenotypic diversity.
• Cancer genetics VUS (Variant of Unknown Significance)
• Paternity testing • Sequence alteration which are unclear whether it is a mutation or
• Forensic medicine. polymorphism are called Variant of Unknown Significance.
m
co

REVIEW QUESTIONS

Operon Concept • Work when oriented in either direction


1. Which of the following are situated away from • Can work with homologous or heterologous
the coding region: (PGI June 06) promoters
a. Promoter • Work by binding one or more proteins
b. Enhancer • Work by facilitating binding of the basal transcription
c. Operator complex to the cis-linked promoter.
d. Structural gene
Ans. b. Enhancer (Ref: Harper 30/e page 401) 2. Housekeeping genes are: (JIPMER 02, WB 03)
Properties of EnhancersQ a. Inducible
m

• Can be located upstream or downstream of the b. Required only when inducer is present
c. Mutant
co

transcription site
• Work when located long distances from the promoter d. Not regulated
• Work when upstream or downstream from the Ans. d. Not Regulated
promoter (Ref: Harper 30/e page 430)
e

e
m

m
m
co

Regulation of Gene Expression  | 307

• The expression of some genes is constitutive, meaning Lac operon is induced if


that they are expressed at a reasonably constant rate • Lactose alone is present.
and not known to be subject to regulation. These are
often referred to as housekeeping genes 6. All of the following statements about Lambda
• An inducible gene is one whose expression increases phage are true, except: (AI 2009)
in response to an inducer or activator, a specific a. In Lysogenic phase it fuses with host
positive regulatory signal. chromosome and remains dormant
m

b. In Lytic phase it fuses with host chromosome


co

3. True among all is (PGI 91) and replicates


a. Repressor is dimer and a positive regulator c. Both Lytic and Lysogenic phase occur together
b. CRP is gratuitous inducer d. In Lytic phase it causes cell lysis and releases
c. Lactose is positive regulator virus particles
d. De-repression is due to presence of glucose Ans. c. Both Lytic and Lysogenic phase…
e. Catabolite repression is mediated by CRP (Ref: Harper 30/e page 432)
Ans. c, d, e. Lactose is positive regulator, De-repression
When lambda infects an organism of that species, it injects
is due to presence of glucose, Catabolite repression is
its 45,000-bp, double-stranded, linear DNA genome into
mediated by CRP (Harper 30/e page 432, 433)
the cell.
• Repressor is a tetramer
Depending upon the nutritional state of the cell, the
• Isopropyl Thiogalactoside (IPTG) is a gratuitous lambda DNA
inducer
m

1. Will either integrate into the host genome (lysogenic


• Although Lactose is present in the cell, as long as
co

pathway) and remain dormant until activated


glucose is present E coli does not activate lac operon.
2. It will commence replicating until it has made about
This is called Catabolite Repression. This is because
of catabolite gene activator protein complexed with 100 copies of complete, protein-packaged virus, at
cAMP. This complex is called cAMP Regulatory which point it causes lysis of its host (lytic pathway).
Protein (CRP) The newly generated virus particles can then infect
• Lactose alone is an inducer of lac operon. other susceptible hosts.
Poor growth conditions favor lysogeny while good
4. False statement is: (PGI 90, WB 03) growth conditions promote the lytic pathway of
a. Repressor binds operator gene lambda growth.
b. Regulator genes produce repressor subunits
c. IPTG is inducer but not substrate 7. True about transposons (PGI Nov 2013)
d. Regulator gene is inducible a. It has no effects on gene expression
m

Ans. d. Regulator gene is inducible b. Also called jumping genes


co

(Harper 30/e page 432, 433) c. Mediated by enzyme transposase


• Structural genes are inducible, not the regulator genes d. It is called retrotransposon when it involves an
• Lac I gene produces Repressor subunits RNA intermediate
• IPTG is an inducer of lac-operon, but itself is not a Ans. b, c, d. Also called jumping ...., mediated by enzyme
substrate, This is called gratuitous inducer. ..., It is called Retroposons ...
Transposons (Jumping Gene)
5. Lac operon transcription is induced by:
• DNA sequences that move to new positions within
(Delhi 03, TN 01)
the genome
a. Glucose
b. Glucose with inducer • Discovered by Barbara McClintock
c. Inducer without glucose • Transposase enzyme help in this process.
d. Both lactose and glucose Retroposons
Ans. c. Inducer without glucose • DNA Sequence move from one segment to another
m

(Ref: Harper 30/e page 433) through an RNA Intermediate


co

Lac operon is repressed when, the cell of E coli contains: • DNA segment is converted to RNA, RNA moves to
• Glucose alone another location, where it is reversely transcribed
• Glucose and Inducer (lactose) to a DNA.
e

e
m

m
m
co

308 |  
Self Assessment and Review of Biochemistry

Mitochondrial DNA 11. True about polymorphism is: (PGI Dec 06)
8. All are true about mitochondrial DNA except a. Single locus → multiple normal alleles
(PGI 2014) b. Single locus → multiple abnormal alleles
a. Contains 37 gene c. Single phenotype: Single locus → multiple
b. Transmit from mother to offsprings normal alleles
c. Transmit in classical mendelian fashion d. Single phenotype: Single locus → multiple
abnormal alleles
m

d. Cause Leber hereditary optic neuropathy


Ans. a, c.
co

Ans. c. Transmit in classical mendelian fashion


Mitochondrial DNA Encodes Structural Genes • Normal variations in nucleotide sequence which
occur in at least 1% of population is Polymorphism
For 2 rRNAs
• so they are multiple normal alleles in same locus
22 mitochondrial tRNAs
• Polymorphism can be in introns or exons so can be
13 protein subunits of respiratory chain. same phenotype or different phenotype depending
This includes 37 genes. on the location.
9. Mitochondrial DNA is: (AI 2006)
12. Microsatellite sequence is: (AI 2006)
a. Closed circular
a. Small satellite
b. Nicked circular
b. Extra chromosomal DNA
c. Linear
c. Short sequence (2–5) repeat DNA
d. Open circular
d. Looped-DNA
m

Ans. a. Closed circular (Ref: Harper 30/e page 362)


Ans. c. Short sequence (2–5) repeat DNA
co

Mitochondrial DNA
(Ref: Robbins 9/e page 179)
• Mitochondria possess its own DNA and protein
synthesizing machinery Repeat length Polymorphism or Short Tandem Repeats
or Variable Number Tandem Repeats
• Human mitochondria contains 2–10 copies of a small
circular dsDNA molecule Depending on the repeat size it is divided into:
• Composed of Heavy (H) and Light (L) chain or Microsatellite -- Repeat size of 2–6 bp.
strands Mini satellite - Repeat size of 15–70 bp.
• Contain 16,569 bp Epigenetics
• That makes up approximately 1% of total circular
13. Genes in CpG Island is inactivated by:
DNA
(PGI Nov 2013)
DNA Polymorphism a. Methylation
m

10. The size of microsatellite repeat sequence is: b. Metrylation


co

(PGI Nov 2014) c. Ubiquitization


a. Less than 1 kb d. Acetylation
b. 2–6 bp Ans. a. Methylation (Robbins 9/e page 3-5)
c. 1–3 kb Gene promoters and regulation by methylation-
d. More than 3 kb demethylations
e. 5–20 bp • Majority of gene promoters have high CG content.
Ans. b. 2–6 bp (Ref: Robbins 9/e page 179) • CpG islands are the common sites of methylation-
Repeat length Polymorphism or Short Tandem demethylations
Repeats or Variable Number Tandem Repeats • Hence methylation-demethylations plays an
important role in regulation of gene expression
Short repetitive sequence in human DNA is called Repeat
length Polymorphism. • CpG islands of promoters are typically unmethylated,
it favors transcription.
m

Most common one is dinucleotide repeat involve AC to


TG on other strand.
co

14. All are true DNA methylation except:


Depending on the repeat size it is divided into: (PGI Nov 2014)
Microsatellite -- Repeat size of 2–6 bp. a. It usually occurs in the cytosine
Mini satellite - Repeat size of 15–70 bp. b. Can alter the gene expression pattern in cells
e

e
m

m
m
co

Regulation of Gene Expression  | 309

c. Role in genomic imprinting Reversible heritable chemical modificationQ of DNA or


d. No role in carcinogenesis histone or nonhistone proteins that does not alter DNA
e. Essential for normal development sequence itself is called epigenetics.
Ans. d. No role in carcinogenesis
(Robbins 9/e page 3-5) 16. Random inactivation of X chromosome is
a. Lyonization
Biochemical functions of Epigenetic ModificationQ
b. Allelic exclusion
m

1. Regulation of tissue specific gene expression


c. Randomization
co

2. X chromosome inactivation (Facultative Heterochro-


d. Genomic imprinting
matin) one of the two X-chromosomes in every cell
of a female Ans. a. Lyonization
3. Genomic imprinting: Two factors that are peculiar to the sex chromosomes: (1)
Gene inactivation on selected chromosomal regions of lyonization or inactivation of all but one X chromosome
autosomes is called Genomic Imprinting. This is the and (2) the modest amount of genetic material carried by
cause of preferential expression of one of the parental the Y chromosome.
allele. In 1961, Lyon outlined the idea of X-inactivation, now
Maternal imprinting refers to transcriptional silencing commonly known as the Lyon hypothesis. It states that
of the maternal allele. (1) only one of the X chromosomes is genetically active,
Paternal imprinting implies that the paternal allele is (2) the other X of either maternal or paternal origin
inactivated. undergoes heteropyknosis and is rendered inactive, (3)
m

Imprinting occurs in the ovum or the sperm, before inactivation of either the maternal or paternal X occurs at
co

fertilization. random among all the cells of the blastocyst on or about


4. Aging Process. day 16 of embryonic life, and (4) inactivation of the same
X chromosome persists in all the cells derived from each
Epigenetic Changes Causing Pathological precursor cell.
Alteration The inactive X can be seen in the interphase nucleus as
1. Fragile X syndrome a darkly staining small mass in contact with the nuclear
‒ Promoter site Hypermethylation causing FMR-1 membrane known as the Barr body, or X chromatin.
gene silencing. The molecular basis of X inactivation involves a unique
2. Cancer gene called XIST, whose product is a noncoding RNA
‒ DNA methylation and histone modifications that is retained in the nucleus, where it “coats” the X
dictate which genes are expressed, which chromosome that it is transcribed from and initiates a
in turn determines the lineage commitment
m

gene-silencing process by chromatin modification and


and differentiation state of both normal and
co

DNA methylation. The XIST allele is switched off in the


neoplastic cells.
active X.
‒ Local promoter hypermethylation of tumor
True about ‘X’ chromosome inactivation:
Suppressor gene leads decreased expression of
the tumor suppressor gene. a. XIST gene (PGI Dec 06)
3. DNA methylation is considered as a defence b. RNA interference
mechanism that minimise the expression of retroviral c. Seen in male
incorporated sequences. d. Seen in female
Ans. a, b, d. XIST gene, RNA interference, Seen in female.
15. All are true regarding epigenetics mechanisms
except: (PGI may 2014) 17. Histone acetylation cause: (AIIMS May 2011)
a. Noninheritable a. Increased heterochromatin formation
b. Acetylation of histone b. Increased ruchromatin formation
m

c. Hereditary c. Methylation of cystine


co

d. Methylation of DNA d. DNA replication


e. X chromosome inactivation Ans. b. Increased euchromatin formation
Ans. a. Noninheritable (Ref: Harper 30/e page 440) (Ref: Harrison 19/e page 421)
e

e
m

m
m
co

310 |  
Self Assessment and Review of Biochemistry

Euchromatin is transcriptionally active core particle, by acetylation of lysine, phosphorylation


Heterochromatin is transcriptionally inactive. of serine, or methylation of lysine and arginine residues.
According to Harrison Acetylation of histones by histone acetylases (HATs),
Covalent posttranslational modifications of histones for example, leads to unwinding of chromatin and
and other proteins play an important role in altering accessibility to transcription factors.
chromatin structure and, hence, transcription. Conversely, deacetylation by histone deacetylases
m

Histones can be reversibly modified in their amino- (HDACs) results in a compact chromatin structure and
co

terminal tails, which protrude from the nucleosome silencing of transcription.


m
co

Fig. 13.11: Epigenetic modifications

18. Differential expression of same gene depending Ans. a, b, c, d. Alteration in gene expression, Genetic
on parent of origin is referred to as: (AI 08) code remains intact, Role in carcinogenesis, Protective
m

a. Genomic imprinting mechanism (Ref: Harper 30/e page 440)


co

b. Mosaicism Epigenetics
c. Anticipation Reversible heritable chemical modification of DNA or
d. Nonpenetrance histone or nonhistone proteins that does not alter DNA
Ans. a. Genomic imprinting (Ref: Robbins 9/e page 172) sequence itself.
Studies over the past two decades have provided The term epigenetics means‚ “above genetics” as the
definite evidence that, at least with respect to some nucleotide sequence is unaltered.
genes, important functional differences exist between the This is one of the recently discovered method of
paternal allele and the maternal allele. These differences regulation of gene expression.
result from an epigenetic process called imprinting. This includes:
19. True about DNA methylation (PGI Nov 2010) • DNA Methylation at Cytosine residues of CpG
islands (Some consider this as a post-replicational
m

a. Alteration gene expression


modification)
co

b. Genetic code remains intact


• Post-translational modification of Histones.
c. Role in carcinogenesis
d. Protective mechanism against cleaving by 20. Epigenetics is a: (NBE Pattern Question)
restriction endonuclease a. Chemical modification of DNA
e

e
m

m
m
co

Regulation of Gene Expression  | 311

b. Irreversible modification of DNA If the miRNA-mRNA base pairing has one or more
c. Change in nucleotide sequence mismatches, translation of the cognate‚ target mRNA’
d. Normal variation of nucleotides is inhibited (TRANSLATIONAL ARREST)
Ans. a. Chemical modification of DNA ‒ If the miRNA-mRNA base pairing is perfect over
all 22 nucleotide
(Ref: Harper 30/e page 440)
‒ The corresponding mRNA is degraded inside
The term “epigenetics” means “above genetics” and
m

cytoplasmic organelle called P Bodies. (mRNA


refers to the fact that these regulatory mechanisms do DEGRADATION)
co

not change the underlying, regulated DNA sequence, but


‒ Functional Consequence of Translation Arrest
rather simply the expression patterns of this DNA.
and mRNA degradation by miRNA is Silencing
Epigenetics refers to reversible, heritable changes the gene expression or Gene Silencing
in gene expression that occur without mutation. Such ‒ This is otherwise called RNA Interference
changes involve post-translational modifications of
histones and DNA methylation, both of which affect gene 23. Normal role of Micro RNA is: (AI 2009)
expression. a. Gene regulation
b. RNA splicing
21. Methylation of Cytidine residues of DNA will
cause: (AIIMS 2014 May) c. Initiation of Translation
a. No Change d. DNA conformation change
b. Decrease gene expression Ans. a. Gene regulation (Ref: Harper 30/e page 447)
m

c. Mutation
co

Mutation
d. Increase in gene expression
Ans. b. Decrease in gene expression 24. Which of the following is/are most severe/
dangerous change in gene: (PGI May 2014)
(Ref: Harper 30/e page 440)
a. Deletion
• DNA Methylation generally cause decrease in gene
expression. b. Insertion
c. Mutation
MiRNA and SiRNA d. Translocation
e. Duplication
22. Function of miRNA is/are: (PGI May 2014)
a. Gene silencing Ans. a, b. Deletion, Insertion
b. Gene activation Deletion and insertion causes garbling of reading
frame. Hence, it is dangerous change in the polypeptide
m

c. Transcription inhibition
synthesized.
co

d. Translation arrest
e. Cleavage of messenger RNA 25. No loss of genetic material occur in:
Ans. a, d, e. Gene silencing, Translation arrest, Cleavage (AIIMS Nov 2012)
of messenger RNA. (Ref: Harper page 447) a. Deletion
miRNA and siRNA b. Insertion
Small noncoding single stranded RNAs which are 21-22 c. Substitution
nucleotide length. d. Inversion
Main function- Post-transcriptional regulation of gene Ans. d. Inversion (Ref: Robbins 9/e page 160)
expression by altering mRNA function. Structural Anomalies in Chromosome
The regulation gene expression by miRNA 1. Deletion
By altering mRNA function. Refers to loss of a portion of a chromosome.
m

In 2006 Craig Mello and Andrew Fire were awarded Most deletions are interstitial,
co

Nobel Prize for silencing gene expression by miRNA Rarely terminal deletions may occur.
Steps involved are: 2. Ring chromosome
Binding of the miRNA to the target mRNA Is a special form of deletion.
e

e
m

m
m
co

312 |  
Self Assessment and Review of Biochemistry

It is produced when a break occurs at both ends of a


chromosome with fusion of the damaged ends.
Ring chromosomes do not behave normally in meiosis
or mitosis and usually result in serious consequences
3. Inversion
Refers to a rearrangement that involves two breaks
m

within a single chromosome with reincorporation of


co

the inverted, intervening segment.


An inversion involving only one arm of the
chromosome is known as paracentric.
If the breaks are on opposite sides of the centromere,
it is known as pericentric.
Inversions are often fully compatible with normal
development.
No loss of genetic element.
4. Isochromosome (Ref: Harper 30/e page 417)
Break along the axis perpendicular to the axis of • Changes one codon for an amino acid into another
chromosome. codon for that same amino acid—Silent mutation
• Codon for one amino acid is changed into a codon of
m

One arm of a chromosome is lost


another amino acid—Missense mutation
co

The remaining arm is duplicated, resulting in a


chromosome consisting of two short arms only or of • Codon for one amino acid is changed into a
two long arms. translation termination codon—Non-sense mutation
Loss of genetic element. • Reading frame changes downstream to the mutant
site—Frameshift mutation.
5. Translocation
A segment of one chromosome is transferred to 28. Frame shift mutation is caused by: (Ker 2007)
another chromosome. a. Deletion
Balanced reciprocal translocation b. Point mutation
‒ There are single breaks in each of two c. Substitution
chromosomes, with exchange of material d. Transversion
‒ A balanced translocation carrier, however, is at
Ans. a. Deletion (Ref: Harper 30/e page 417)
increased risk for producing abnormal gametes
m

Frameshift Mutation
‒ No loss of genetic element
co

Robertsonian translocation (or centric fusion) Due to insertion or deletion of nucleotides that are not a
multiple of three results in frameshift mutation.
‒ A translocation between two acrocentric
chromosomes Reading frame is garbled.
‒ Typically the breaks occur close to the centromeres 29. Cystic fibrosis mutation causing the reduced
of each chromosome chloride conductance is: (NBE pattern Q)
‒ Transfer of the segments then leads to one very a. Class-1
large chromosome and one extremely small one. b. Class-2
Usually the small product is lost.
c. Class-3
26. True about Fragile X syndrome (PGI June 2009) d. Class-4
a. Trinucleotide repeat sequence disease Ans. d. Class-4
m

b. Chromosome-breakage Class IV: Decreased conductance. These mutations typically


co

c. X-chromosome defect occur in the transmembrane domain of CFTR, which


d. Point-mutation forms the ionic pore for chloride transport. There is a
e. Deletion normal amount of CFTR at the apical membrane, but
e

e
m

m
m
co

Regulation of Gene Expression  | 313

with reduced function. This class is usually associated 31. One of the following mutations is potentially
with a milder phenotype. lethal: (Delhi 96)
30. X-ray causes DNA mutation by: (PGI May 2014) a. Substitution of adenine for cytosine
a. Double strand break repair b. Substitution of methylcytosine for cytosine
b. Oxidation c. Substitution of guanine for cytosine
c. Pyrimidine Dimer d. Insertion of one base
m

d. Intrastrand cross links Ans. d. Insertion of base


co

Ans. a, d. Double strand break repair, Intrastrand cross Insertion or deletion of base can garble the reading frame,
links. resulting in a frame shift mutation.
m
co
m
co

32. Sickle cell anemia is the clinical manifestation of • Base substitution


homozygous genes for an abnormal hemoglobin • Nonconservative mutation.
molecule. The event responsible for the mutation
in the B chain is: (AIIMS 91, Kerala 90)
33. Null mutation is: (AI 2000)
a. Insertion
a. Mutation occurring in noncoding region
b. Deletion
b. Mutation that does not change the amino acid
c. Nondisjunction or end product
d. Point mutation
c. Mutation that codes for a change in progeny
Ans. d. Point mutation without a chromosomal change
m

The mutation in HbS is an example of: d. Mutation that leads to no functional gene
co

• Point mutation product


• Partially acceptable missense mutation Ans. d. Mutation that leads to no functional gene
• Transversion product
e

e
m

m
m
co

314 |  
Self Assessment and Review of Biochemistry

Null mutation 36. Which of the following can be a homologous


A mutation that leads to no functional gene product is substitution for valine in the hemoglobin?
called null mutation. (AI 2004)
34. A mutation in the codon which causes a change in a. Isoleucine
the coded amino acid, is known as: b. Glutamic acid
a. Mitogenesis (AIIMS May 02) c. Phenyl alanine
m

b. Somatic mutation d. Lysine


co

c. Missense mutation
Ans. a. Isoleucine
d. Recombination
Ans. c. Missense mutation (Ref: Harper 30/e page 411) Both are branched chain nonpolar amino acids.
I. Silent Mutation
‒ If a mutation does not alter the polypeptide 37. Pyrimidine dimers are seen in:
product of the gene. a. UV rays
‒ Also called Synonymous mutation. b. Xeroderma pigmentosa
II. Missense mutation c. Alkylating agents
The alteration in the nucleotide may result in the d. X-rays
incorporation of a different amino acid.
Ans. b. Xeroderma pigmentosa
35. In a mutation if valine is replaced by which of the
(Ref: Harper 30/e page 390)
m

following would not result in any change in the


• DNA damaging agent that results in pyrimidine
co

function of protein: (AIIMS May 02)


a. Proline dimer is U-V light chemicals
b. Leucine • DNA repair mechanism that repair pyrimidine dimer
c. Glycine is Nucleotide Excision repair
d. Aspartic acid • Defective Nucleotide excision repair mechanism
Ans. b. Leucine causes Xeroderma pigmentosum, Cockayne
Valine and Leucine are branched chain nonpolar amino Syndrome, Trichothiodystrophy
acids. • So pyrimidine dimers are seen in disorders caused
So homologous substitution. by defective nucleotide excision repair.
m
co
m
co
e

e
m

m
m
co

14 Molecular Biology Techniques


and Recent Advances in
m

Molecular Biology
co

Topics Included
• Recombinant DNA Technology • Hybridoma Technique
• Gene Library • RFLP
• Probes • Stem Cell Biology
• Amplification Techniques • Gene Therapy
• Hybridization and Blot Techniques • Human Genome Project
m

• Cytogenetic Techniques • Omics of Molecular Biology


co

• Mutation Detection Techniques • Bioinformatic and Genomic Sources


• DNA Sequencing Techniques • Techniques to Study Proteomics
• Next Generation Sequencing • Linkage and Association Studies
• Transgenic Animals

RECOMBINANT DNA TECHNOLOGY


Definition
In vivo amplification technique used to get a clone of
m

desired DNA fragment.


co

To learn about Recombinant DNA Technology we should


have knowledge about Vector, Restriction endonuclease,
Chimeric DNA.
Restriction Endonuclease
Fig. 14.1: Restriction endonucleases
• Endonuclease which cut the DNA at specific
palindromic sequence. Restriction Map
• Palindrome is a sequence of duplex DNA that is When a genomic DNA is treated with a specific
the same when the two strands are read in opposite Restriction endonuclease it cuts the DNA at specific sites
directions. to create a characteristic linear array of DNA. This is called
Examples: GATCC and CCTAG restriction map.
AATT and TTAA
m

• They are otherwise called molecular scissors Vectors used in Recombinant DNA Technology
Plasmids
co

• They cut the DNA to produce sticky or staggered


ends or blunt ends • Circular, double stranded DNAQ molecules seen in
• They are called Restriction endonucleases because bacteria (8-10 copies/cell).
they restrict the entry of phages into host DNA. • Extra chromosomal
e

e
m

m
m
co

316 |  Self Assessment and Review of Biochemistry

• Each plasmid contains an origin of replication and ‒ Sticky-end sites may not be available or
can replicate independently in a convenient position for the restriction
• They are episomes, i.e. a genome above or outside endonuclease
the bacterium 2. Problems with Blunt Ends
• Natural function is to confer antibiotic resistance ‒ Blunt ends ligation is not directional.
• Carry 0.01-10 kbp of DNA. Remember
m

Phages (Bacterial Viruses) The enzyme that helps in blunt end ligation is Bacteriophage T4
co

Enzyme DNA Ligase.


• Virus which infect the bacteria are called phages.
• They have a linear DNA molecule Procedure of Homopolymer Tailing
• Carry DNA fragments up to 10–20 kbp. • To circumvent the problems of blunt ends,
Cosmids homopolymer tailing is used
• They are plasmids which combine the features of • New synthetic sticky ends are added using the
Plasmid and Phages enzyme terminal transferase
• They contain special genes called cos site (needed for • Poly d(G) is added to the 3’ ends of the vector
packing lambda DNA into phage particles) • Poly d(C) is added to the 3’ ends of the foreign DNA
• Carry DNA fragments up to 30–50 kbp. using terminal transferase
• Then the two molecules can only anneal to each other
BAC, YAC and PAC
This procedure is called homopolymer tailing.
m

Are artificially created chromosomes that can carry large


co

DNA insert: Blue-white Screening


• BAC—Bacterial Artificial Chromosome • Is a quick and easy screening technique to detect
• YAC—Yeast Artificial Chromosome successful ligation of DNA of interest to the vector
• PAC—E coli bacteriophage P1-based vectors • Cells are grown in the presence of X-Galactose
DNA Insert Size: • If the ligation was successful, the bacterial colony
BAC and PAC ---- 50–250 kbp will be white
YAC ---- 500–3000 kbp • If not, the colony will be blue.

Chimeric DNA or Recombinant DNA Recombinases–An Adjunct to Restriction


DNA to be cloned + Vector DNA = Chimeric DNA Endonucleases
Passenger DNA Catalyse specific incorporation of two DNA fragments
that carry the appropriate recognition sequences and
m

DNA to be cloned is called Passenger DNA or Foreign


carry out homologous recombination, between relevant
co

DNA.
recognition site.
Procedure to Prepare Chimeric DNA
Examples of Recombinases and relevant recognition site
• Both the foreign DNA and vector DNA is treated
with same Restriction endonuclease Recombinase Recognition site

• This creates sticky or blunt ends CRE Recombinase Bacterial Lox P site
• This is religated using DNA Ligase λ phage encoded INT protein Bacteriphage λ TT site
• Thus, chimeric DNA is produced. Yeast FLP Recombinases Yeast FRT site

Homopolymer Tailing
Steps of Recombinant DNA Technology
Technique used to overcome the problems inherent to
sticky ends and blunt ends. • Isolation of specific DNA
• Selection of Vector
m

1. Problems with Sticky End


• Synthesis of chimeric DNA
co

‒ Sticky ends of a vector may reconnect with


themselves, with no net gain of DNA • Introduction of recombinant plasmid to bacteria
‒ Sticky ends of fragments also anneal so that • Screening for Recombinant Vectors
heterogeneous tandem inserts form • Selection of Specific DNA clones.
e

e
m

m
m
co

Molecular Biology Techniques and Recent


Advances in Molecular Biology  | 317
m
co
m
co

Fig. 14.2: Steps of recombinant DNA technology

Uses of Recombinant DNA Technology in Clinical Contd ...


Medicine • This CRISPR-RNA Cas 9 complex then inactivates and degrade
• To understand molecular basis of diseases target DNA.
• Preparation of vaccines and hormones • This system can be used in eukaryotic cells, including humans.
• Diagnosis of infectious diseases Application of CRISPR Cas 9
• Forensic medicine—reveal a criminal from specimens • Genomic editing
left on the scene of crime. • Targeted Mutagenesis—Gene knock out
m

• Modulation of gene expression


co

CRISPR cas 9 System


• Gene deletion.
• Clustered Regularly Interspersed Short Palindromic Repeats
associated gene 9.
• A prokaryotic ‘immune system’ conferring resistance to external
cDNAQ
genes from bacteriophage.
Mechanism of action DNA complementary to mRNA is called cDNA or Copy
In bacteria DNA or Complementary DNA.
CRISPR derived RNA combines with the Cas nuclease to target and
Procedure to Prepare cDNA
specifically cleave the DNA of invading phage, thereby inactivating
these invading genomes and protecting the bacterium from produc- • Isolate the mRNA
tive phage infection and lysis • By the action of Reverse Transcriptase RNA-DNA
In eukaryotes hybrid is synthesized
• This system now emerged as Novel DNA/genome editing or gene
• By RNAseH RNA is digested
m

regulatory system
• By DNA Polymerase double stranded DNA is
co

• CRISPR use an RNA based targeting to bring Cas 9 nuclease to


foreign DNA. produced
Contd ... • Thus, DNA complementary to mRNA is synthesized.
e

e
m

m
m
co

318 |  Self Assessment and Review of Biochemistry

Gene Library
A collection of recombinant DNA clones generated from
a specific source.
Two Types of Gene Library
1. Genomic DNA Library:
‒ Prepared from total genomic DNA of an organism
m

‒ By digestion of Genomic DNA by Restriction


co

Endonuclease
‒ Then recombinant clones of such digested DNA
Fig. 14.3: End labelling at 5’ end
is produced by recombinant DNA Technology.
2. c DNA library
‒ cDNA is prepared from mRNA by the action of
Reverse Transcriptase
‒ The recombinant clones for cDNA are produced
by Recombinant DNA Technology.
Advantages of cDNA over Genomic DNAQ
• Contains only coding sequences
• Represent the mRNA in a tissue
m

• Hence used to study gene expression. Fig. 14.4: End labelling at 3’ end
co

Probes Nick Translation


Pieces of DNA or RNA labelled by various techniques to The technique used to produce radiactive labelled DNA
detect a complementary sequence. probes.
Uses of Probes Procedure
• To detect DNA on Southern blot transfers • Single stranded nick created and the dNucleotide is
• To detect RNA on Northern blot transfers removed by DNAse I
• Can search libraries for a specific gene. • Gap is filled by radiolabelled dNucleotide by DNA
Polymerase
Labelling of DNA Probes
• Thus, radiolabelled DNA probe is created.
Two types of labels
Radioactive Labels Expression Vector
A vector in which the foreign DNA introduced by
m

• Most commonly by Radioactive Phosphorus (32P)


Recombinant DNA Technology synthesizes protein, i.e.
co

Nonradioactive Labels the gene is expressed.


• Biotin label
Uses of Expression Vectors
• Fluorescent labels
• To detect specific cDNA
Methods of Radiolabelling • To detect specific protein produced by a specific
1. End labelling at 5’ end or 3’ end of the probe. cDNA
2. Nick translation. • To produce proteins like Insulin in large quantities.
END LABELLING OF PROBES Enzymes used in Recombinant DNA Research
Enzyme Reaction Uses
At 5’ end Alkaline phospha- Dephosphorylates 5’ Removal of 5’-PO 4
Enzymes used are: tase ends of RNA and DNA groups prior to kinase
• Alkaline Phosphatase labeling; also used to
m

prevent self-ligation
• Polynucleotide Kinase
co

DNA ligase Catalyzes bonds Joining of DNA mol-


At 3’ end between DNA ecules
molecules
Enzyme used is:
• Terminal Transferase Contd...
e

e
m

m
m
co

Molecular Biology Techniques and Recent


Advances in Molecular Biology  | 319

Contd... The instrument that takes samples through the multiple


Enzyme Reaction Uses steps of changing temperature in PCR Cycle is called
DNA polymerase I Synthesizes double- Synthesis of double- Thermocycler.
stranded DNA from stranded cDNA;
single-stranded DNA Nick translation; Steps of PCR CycleQ
Generation of blunt 1. Denature the DNA: The DNA to be amplified is
ends from sticky ends heated to separate the double-stranded target DNA
m

Thermostable Synthesize DNA at Polymerase chain into single strands 94°C.


co

DNA polymerases elevated temperatures reaction (DNA syn-


(Taq Polymerase) (60–80°C) thesis)
2. Annealing of primers to ssDNA: The separated
strands are cooled and allowed to anneal to the two
DNase I Under appropriate condi- Nick translation
tions, produces single- Mapping of hyper- primers (one for each strand). By cooling to 55°C.
stranded nicks in DNA sensitive sites 3. Extension of the Primer: Synthesize new chains
Mapping protein-DNA complementary to the original DNA chains. By
interactions heating again to 72°C.
Exonuclease III Removes nucleotides D N A s e q u e n c i n g
from 3’ ends of DNA Mapping of DNA-
protein interactions
λ Exonuclease Removes nucleotides DNA sequencing
from 5’ ends of DNA
m

Polynucleotide Transfers terminal phos- P end-labeling of


32

kinase phate (ϒ position) from DNA or RNA


co

ATP to 5’-OH groups of


DNA or RNA
Reverse transcrip- Synthesizes DNA from Synthesis of cDNA
tase RNA template from mRNA; RNA (5’
end) mapping studies
S1 nuclease Degrades single-strand- Removal of ‘hairpin’
ed DNA in synthesis of cDNA;
RNA mapping studies
(both 5’ and 3’ ends)
Terminal transfer- Adds nucleotides to the Homopolymer tailing
ase 3’ ends of DNA
CRISPR-Cas9 R N A t a r g e t e d D N A Genome deiting and
directed Nuclease modulation of gene
m

expression
co

AMPLIFICATION TECHNIQUES
Polymerase Chain Reaction
• Revolutionary technique invented by Karry B MullisQ
in 1989
• He got Nobel Prize for this in 1993
• The polymerase chain reaction (PCR) is a test tube
method for amplifying a selected DNA sequence
• ExponentialQ amplification of the sample
• The number of samples after n number of cycles is 2n
m

• One cycle of PCR require <5 to 10 minutes


co

• 20 cycles result in million-fold amplification of the


target DNA
• Product obtained by amplification is called Amplicon. Fig. 14.5: The PCR cycle
e

e
m

m
m
co

320 |  Self Assessment and Review of Biochemistry

Pre-requisites of PCR 3. Nested PCR


• Sample DNA to be amplified When the DNA to be amplified is present in low
• Deoxy nucleotides concentration compared to total DNA in the sample,
• Thermostable Polymerase-Taq PolymeraseQ obtained this technique is used
from Thermus Aquaticus found in hot springs. Nested PCR uses two sets of amplification primers.
• Primer ‒ Outer primers
m

• MgCl2 and KCl. ‒ Inner primers.


co

Phusion 4. Multiplex PCR


• Newly developed DNA Polymerase for PCR ‒ Simultaneous amplification of many targets in
• High fidelity DNA Polymerase one reaction
• Enzyme constructed by fusion of unique DNA binding domain to
Pyrococcus like Proofreading Polymerase
‒ Uses more than one pair of primers.
• High Processivity 5. Arbitrary PCR (Arbitrarily Primed Polymerase
• Low error rate Chain Reaction) (AP PCR)
• Not inhibited by any quantity of blood, where as Taq Polymerase
is inhibited. Main Features:
‒ Uses short random primers (Usually 10 bases)
Variants of PCR ‒ Amplifies anonymous stretches of DNA
1. Reverse Transcriptase PCR (RT PCR) Applications of PCR
m

‒ It is the PCR amplification of a reverse 1. Used in Forensic Medicine


co

transcription product 2. To detect infectious agents, especially latent viruses


‒ RT PCR amplifies very small amounts of any 3. To make prenatal genetic diagnoses
kinds of RNA (mRNA, rRNA, tRNA etc.) 4. To detect allelic polymorphisms
‒ cDNA copies of mRNA generated by a retroviral 5. To detect mutation
reverse transcriptase 6. To establish precise tissue types for transplants
‒ This cDNA is amplified as in usual PCR 7. To study evolution, using DNA from archeological
‒ TthPolymerase (from Thermus thermophilus) samples
is used 8. For quantitative RNA analysis after RNA copying
‒ Method used to obtain relative expression of and mRNA quantitation by the so-called Reverse
gene in a cell. Transcriptase PCR method
2. Real time (Homogenous or Kinetic) PCR 9. To score in vivo protein: DNA occupancy using
m

‒ This is a type of Quantitative PCR chromatin immunoprecipitation assays (ChIP)


co

‒ Methods used to amplify and at the same time 10. To facilitate New Generation Sequencing.
quantitate amplified PCR products during the
exponential phase of PCR cycle (called Realtime). HYBRIDIZATION AND BLOT TECHNIQUES
It is not after the amplification is over.
1. Southern Blot
Methods used to quantitate the number of copies DNA ‒ Devised by Edward Southern in 1975
present within the genome.
‒ Technique to detect specific DNA Segment.
Intercalating Dyes:
Principle:
‒ Ethidium Bromide
‒ Based on specific base pairing rule of
‒ SYBR Green.Q complementary nucleic acid strands
Sequence Specific Probes: ‒ It is a DNA-DNA Hybridization.
m

‒ TaqMan Steps of Southern Blot


co

‒ Molecular beacon ‒ Duplex DNA isolated


‒ Fret Probes: Fluorescence Resonance Energy ‒ Treated with Restriction Endonuclease
Transfer Probes. ‒ DNA is fragmented
e

e
m

m
m
co

Molecular Biology Techniques and Recent


Advances in Molecular Biology  | 321

‒ Fragmented DNA separated by Agar gel 3. Western Blot (Immunoblot) analysis for Proteins
electrophoresis Technique to detect specific Protein in a sample.
‒ Treated with NaOH to denature the DNA Principle:
‒ Denatured DNA fragments transferred to ‒ Antigen antibody Interaction
Nitrocellulose membraneor nylon paper (Blot)
‒ Radioactive labeled antibody used.
‒ Add Radiolabelled or Fluorescent cDNA probes
Uses:
m

‒ Detection of Hybridization by imaging (DNA-


‒ Identification of a specific protein in sample
co

DNA Hybridization).
Uses of Southern Blot ‒ Detection of Viral pathogens by identifying viral
‒ Identification of specific viral or bacterial DNA proteins—HIV virus or Hepatitis B virus.
in the infected sample 4. South Western Blotting
‒ Screening test to detect inborn errors To examine Protein–DNA Interaction
‒ Detect point mutations 5. Dot blot Technique
‒ Can detect gene alteration, like deletion, insertion ‒ The step, blotting to nitrocellulose membrane is
etc. avoided
‒ In forensic medicine, to analyze DNAs from ‒ The sample is directly applied to slots on a
specimens at the scene of crime—blood, semen, specific blotting apparatus containing nylon
saliva etc. membrane. This is also called slot blot.
2. Northern Blot
m

‒ Technique used to detect specific RNA


co

CYTOGENETIC TECHNIQUES
‒ Principle: RNA- DNA hybridization technique
‒ Radioactive/fluorescent labelled cDNA Probes The word chromosome is a combination of two words
used. greek words, ‘Chroma’ means ‘color’ and ‘Somes’
Uses means ‘body’. So chromosomes are colored bodies.
‒ Used to detect specific gene expression in specific Arms of Chromosome
tissues. • p arm is the short arm. p stands for petite
• q arm is the long arm. q is the next letter after p
• q arm is also called g arm. g stands for grande

Cytogenetic Techniques
Can be divided into:
m

• Conventional Cytogenetic techniques-Chromosome


co

Banding
• Molecular Cytogenetic Techniques—Fluorescent in
Situ Hybridization (FISH)
• Array Based Techniques (Cytogenomic techniques)
‒ Comparative Genomic Hybridization (CGH)
array
‒ Single Nucleotide Polymorphism (SNP) array
Samples for chromosome analysis
• Prenatal (fetal) chromosome
• Amniocytes by Amniocentesis
m

• Chorionic villi by Chorionic Villus Sampling—


co

Allows early detection of anomalies <10 weeks


• Fetal Blood by Percutaneous Umbilical cord sampling
Fig. 14.6: Blot techniques (PUBS) in late second trimester.
e

e
m

m
m
co

322 |  Self Assessment and Review of Biochemistry

For preimplantation detection of anomalies • Bands similar to G banding


• Analysis of Blastomere • Fluorescent Bands are obtained.
Other Samples in adults are: Disadvantages of Conventional Banding Techniques
1. Cultured skin fibroblast • Deletions smaller than several million base pairs
2. Bone marrow are not routinely detectable by standard G-banding
3. Peripheral lymphocyte techniques
m

Timing for Chromosome Analysis • Chromosomal abnormalities with indistinct or novel


co

banding patterns can be difficult or impossible to


Artificially arrested in mitosis during Metaphase (or
interpret.
prometaphase)
• To carry out cytogenetic analysis, cells must be
• Metaphase arrest by - N deacetyl N-methyl Colchicine
dividing, which is not always possible to obtain (e.g.
(Colcemid)
in autopsy or tumor material that has already been
Conventional Cytogenetics—Banding Techniques fixed).
• Cytogenetic analysis is most commonly carried out
on cells in mitosis, requiring dividing cells MOLECULAR CYTOGENETIC TECHNIQUE—
• Halting mitosis in metaphase is essential, because FLUORESCENT IN SITU HYBRIDIZATION (FISH)
chromosomes are at their most condensed state Simple detection of specific genetic information in a
during this stage of mitosis morphologically intact tissue, cell or chromosome using
• The banding pattern of a metaphase chromosome fluorescent probes.
m

is easily recognizable and is ideal for karyotyping. To metaphase spread of chromosomes on a glass slide
co

Important Banding Techniques fluorescent probe is added.


G Banding Advantages of FISH
• Other name Gband Trypsin Giemsa (GTG) Banding • FISH permits determination of the number and
• The chromosomes are treated with Trypsin location of specific DNA sequences in human cells
• Stained by Giemsa • FISH can be performed on metaphase chromosomes,
• Produce dark (G+) and light (G–) as with G-banding, but can also be performed on cells
• Visualized under light Microscope not actively progressing through mitosis
• Most commonly used • FISH performed on nondividing cells is referred to
as interphase or nuclear FISH.
R (Reverse) Banding
• Stained by Giemsa, Acridine Orange Disadvantages of FISH
• FISH requires a preselection of an informative
m

• Yield light and dark band which are reverse of G


Banding molecular probe prior to analysis
co

• Visualized under light microscope • So a prior knowledge of the anomaly is needed.


• Used for detecting rearrangements at the terminal Uses of FISHQ
end of chromosome (telomeres). • Detection of numeric abnormalities of chromosomes
C Banding (aneuploidy)
• Centromeric Heterochromatin Banding • The demonstration of subtle microdeletions
• Pretreated with acid followed by alkali • Detection of complex translocations not detectable
by routine karyotyping
• Stained by Giemsa
• For analysis of gene amplification
• Visualized under light microscope
e.g. HER2/NEU in breast cancer or N-MYC
• Centromeric and Heterochromatic regions are
amplification in neuroblastomas
preferentially stained
• For mapping newly isolated genesQ of interest to their
m

• To study Chromosomal translocation in the


chromosomal loci.
centromere.
co

Q Banding Chromosome Painting


• Stained by Quinacrine Mustard • Is an extension of FISH
• Visualized under Fluorescence Microscope • Probes are prepared for entire chromosomes
e

e
m

m
m
co

Molecular Biology Techniques and Recent


Advances in Molecular Biology  | 323

• Different chromosomes are identified by different RNA Microarray


fluorescently labelled probes • To the known array of oligonucleotide, fluorescently
• The number of chromosomes that can be detected labelled cDNA prepared from unknown mRNA is
simultaneously by chromosome painting is limited by added
the availability of fluorescent dyes. • Computerized algorithms can then rapidly
‘decode’ the cDNA sequence based on fluorescent
Multicolor FISH or Spectral Karyotyping
m

hybridization pattern on the chip


• Similar to Chromosome painting in which probe is
co

• This technique is used for gene expression studies.


prepared for entire Chromosome
Protein microarray
• But unlike chromosome painting, a combination of
five fluorochromes are used • Immobilized known antibodies placed on the glass
slide
• 23 distinct mixtures of 5 fluorophores to create a
unique ‘color’ for each chromosome • Fluorescently tagged target protein added
• By appropriate computer-generated signals, the • By antigen: antibody interaction target protein
entire human genome can be visualized. detected
• This technique used in the study of proteomics.
ARRAY-BASED METHODOLOGIES Uses of microarraysQ
(CYTOGENOMICS) • To analyze a DNA sample for the presence of gene
Array-based methods were introduced into the clinical variations or mutations (genotyping)
m

lab beginning in 2003 and quickly revolutionized the field • To determine the patterns of mRNA production (gene
co

of cytogenetics They are: expression analysis)


1. CGH array • To study Proteins (Protein Microarray)
2. SNP array • To SNP genotyping.

Advantages of Array-based Techniques Array-Based Comparative Genomic Hybridization


• Permit analysis of many regions of the genome in a (Array CGH)
single analysis • This is also a hybridization technique done on a
• High resolution over standard cytogenetics. Microarray or DNA Chip, hence in the name array –
based
Before discussing array CGH, lets learn about Micro-
array techniques • Here two Genomes are compared, hence the name
Comparative Genomic Hybridization.
m

MICROARRAY TECHNIQUEQ Procedure


co

In array CGH the test DNA and a reference (normal)


• Other names are DNA Chip DNA are labelled with two different fluorescent dyes
• DNA microarrays contain thousands of known (most commonly Cy5 and Cy3, which fluoresce red and
immobilized DNA sequences organized in an area green, respectively).
no larger than a microscope slide The differentially labelled samples are added to a DNA
• The fluorescently tagged sample to be sequenced is Chip spotted with entire human genome (usually cover
added. all 22 autosomes and the X chromosome) at regularly
DNA Microarray or DNA Chip spaced intervals.
• To the DNA Chip containing known oligonucleotide • If the contributions of both samples are equal for a
sequence, fluorescently labelled unknown given chromosomal region
oligonucleotide is added ‒ Then all spots on the array will fluoresce yellow
• The computerized algorithms can decode the ‒ The result of an equal admixture of green and
m

unknown oligonucleotide, by detecting the location red dyes.


co

of fluorescent hybridization pattern on the chip • If the test sample shows an excess of DNA at any
• This technique is used for genotyping or genome given chromosomal region (such as resulting from
sequencing. an amplification)
e

e
m

m
m
co

324 |  Self Assessment and Review of Biochemistry

Uses of Array CGH


1. Detect gene amplification
2. Detect gene deletion
3. Detect copy number variations.
Hence used diseases of unknown etiology like Cancer,
Autism, Mental Retardation, Child with dysmorphic
m

features etc.
co

Remember
Array CGH cannot detect Balanced Translocations

SNP Array
SNP platforms use arrays to find out SNPs that are
distributed across the genome.
(Please see DNA Polymorphisms in Chapter Regulation
of Gene expression, to know about SNPs)
Uses
Fig. 14.7: Array CGH • Used in genome-wide association studies to identify
‒ There will be a corresponding excess of sig- disease susceptibility genes
nal from the dye with which this sample was
m

• To identify genomic deletions and duplications


labeled
co

• To detect regions of the genome that have an excess of


• If the test sample show deletion, homozygous genotypes and absence of heterozygous
‒ There will be an excess of the signal used for genotypes (e.g. CC and TT genotypes only, with no
labelling the reference sample (green). CT genotypes).
Comparison of Different Cytogenetic Techniques
Requires grow- Detects deletion Detects balanced struc- Detects Uniparental Lower limits of de-
Method ing cells and duplication tural rearrangements Disomy tection
G Banding Yes Yes Yes No 5–10 mb
Metaphase FISH Yes Yes Yes No 40–250 thousand mb
Interphase FISH No Yes Some No 40–250 thousand mb
CGH array No Yes No No Single Exon or Single
m

gene
co

SNP array No Yes No Some Single Exon or


Single gene

Methods to detect aneuploidy ‒ Special strains of Salmonella typhimurium have


• Conventional Karyotyping mutated histidine gene
• FISH ‒ Hence, they will grow only in medium containing
• Microarray based CGH Histidine gene
• Multiple Ligation dependent Probe Amplification (MLPA) ‒ This is called reverse mutation
• Real time PCR
• Quantitative Fluorescent PCR (QF–PCR)
‒ The number of colonies is proportional to the
• Multiplex amplifiable probe hybridization (MAPH). quantity of mutagens.
2. Site Directed Mutagenesis
‒ Michael Smith in 1993
m

MUTATION DETECTION TECHNIQUES ‒ An oligodeoxyribonucleotide whose sequence


co

is complementary to a part of known gene is


Test to Detect Mutations synthesized
1. Ame’s Test ‒ A specific deletion/insertion is produced in the
‒ Test to detect mutagenicity oligodeoxynucleotide
e

e
m

m
m
co

Molecular Biology Techniques and Recent


Advances in Molecular Biology  | 325

‒ It is then extended by DNAP Multiplex Ligation–Dependent Probe Amplification


‒ This altered gene is amplified and expressed by (MLPA)
insertion into a cloning vector • MLPA blends DNA hybridization, DNA ligation, and
‒ This allows the study of a particular mutation. PCR amplification
Uses
Techniques used to Detect Mutations with DNA
• To detect deletions and duplications of any size,
Sequence Alterations
m

including anomalies that are too large to be detected


I. First do a PCR amplification of the DNA, then
co

by PCR and too small to be identified by FISH.


different sequencing techniques can be used
‒ Sanger’s technique: 36 years after its Nobel– Procedure of MLPA
worthy invention by Frederick Sanger, Sanger • Each MLPA reaction uses a pair of probes that can
sequencing is still considered the ‘gold standard’ hybridize side–by–side to one strand of the target
for sequence determination DNA.
‒ Pyrosequencing • Once bound, the probes are covalently joined via a
ligase reaction
Principle:
• When a nucleotide is incorporated into a growing
DNA strand, there is release of pyrophosphate (PPi).
• In this technique, individual nucleotides (A, C, T, or
G) one at a time into the reaction
m

• If one or more nucleotides are incorporated into the


co

growing strand of DNA, pyrophosphate is released


• Released Pyrophosphate participate in a secondary
reaction
• A secondary reaction involving luciferase that
produces light, which is measured by a photo detector
Advantage of Pyrosequencing
• More sensitive than Sanger sequencing
• Allowing for detection of as little as 5% mutated
alleles in a background of normal alleles
• To analyze DNA obtained from cancer biopsies, in
which tumor cells are often‚ contaminated with large
m

numbers of admixed stromal cells


co

II. Restriction Fragment length Polymorphism:


‒ If a mutation affect a specific restriction site, this
technique can be used.
Techniques Used to Detect Mutations that Affect
Length of DNA
a. Amplicon Length Analysis
b. Real-time PCR
c. Multiple Ligation Dependent Probe Amplification.
Amplicon length analysis
• After doing PCR, the size of PCR products determined
by gel electrophoresis.
m

Real–time PCR
co

• Detect and quantify the presence of particular Fig. 14.8: Multiple ligation dependent probe amplification
nucleic acid sequences in “real time” (i.e. during the • In addition to the target sequence, the probes also
exponential phase of DNA amplification rather than contain additional sequences at their ends that can
post–PCR). be used as primer sequences in a PCR
e

e
m

m
m
co

326 |  Self Assessment and Review of Biochemistry

• The ligated probes thus create a template that can DNA SEQUENCING TECHNIQUES
then be amplified by PCR.
1. Sanger’s Technique (Controlled Chain Termination
Advantages of MLPA Method)
• Can be performed on very small amounts of genomic ‒ Chain terminators used 2’ 3’ Dideoxynucleotides
DNA ‒ DNA Polymerase used is Klenow Polymerase
• Each probe–set can be designed with identical primer
m

‒ This can be automated


sequences
co

• Many probe–sets can be applied and amplified in Principle:


one reaction tube. ‒ Introduction of 2’ 3’ dideoxy nucleotides
will terminate the DNA synthesis because
Other Mutation Detection TechniqueQ no free 3’OH group for the formation of next
Method Type of Mutation Detected phosphodiester bond.
Cytogenetic analysis Numerical or structural abnormalities 2. Maxam and Gilbert Chemical Cleavage Method
in chromosomes 3. Next Generation Sequencing (NGS)
Fluorescent in situ Numerical or structural abnormalities
hybridization (FISH) in chromosomes Next Generation Sequencing (NGS)
Southern blot Large deletion, insertion, rearrange- Term used to describe several newer DNA sequencing
ment, expansions of triplet repeat, technologies that are capable of producing large amounts
m

amplification of sequence data in a massively parallel manner.


co

Polymerase chain reaction Expansion of triplet repeats, variable


(PCR) number of tandem repeats (VNTR), Comparison of Sanger’s Sequencing and Next
gene rearrangements, transloca- Generation Sequencing
tions; prepare DNA for other mutation
methods Sanger’s Sequencing New Generation Sequencing
Reverse transcriptase PCR Analyze expressed mRNA (cDNA) High Cost Low cost
(RT–PCR) sequence; detect loss of expression
Single, simple homogenous No such requirement, sample
DNA sequencing Point mutations, small deletions and template DNA from almost any source can be
insertions used
Restriction fragment Point mutations, small deletions and Provides an “average” result for Well suited for heterogenous
polymorphism (RFLP) insertions a DNA sample. Uninterpretable DNA Sample
results if sample of DNA is het-
Single–strand Point mutations, small deletions and erogenous
conformational insertions
m

polymorphism (SSCP)
co

Denaturing gradient gel Point mutations, small deletions and Three Basic Processes of NGS
electrophoresis (DGGE) insertions
Spatial separation
RNAse cleavage Point mutations, small deletions and
insertions At the beginning of the procedure, individual input DNA
molecules are physically isolated from each other in space.
Oligonucleotide specific Point mutations, small deletions and
hybridization (OSH) insertions The specifics of this process are platform–dependent.
Microarrays Point mutations, small deletions and Local amplification
insertions
After separation, the individual DNA molecules are
Genotyping of SNPs
amplified in situ using a limited number of PCR cycles.
Protein truncation test (PTT) Mutations leading to premature
truncations Parallel sequencing
Pyrosequencing Sequencing of whole genomes of The amplified DNA molecules are simultaneously
m

microorganisms, resequencing of sequenced by the addition of polymerases and other


co

amplicons
reagents, with each spatially separated and amplified
Multiplex ligation–dependent Copy number variations original molecule yielding a‚ “read” corresponding to
probe amplification (MLPA)
its sequence.
e

e
m

m
m
co

Molecular Biology Techniques and Recent


Advances in Molecular Biology  | 327

Some New Sequencing Techniques Different Strategies of Genetic Modification


CAGE (Cap analysis of gene expression) • Injection of the transgene into the male pronucleus
A method that allows the selective capture, amplification, cloning, of fertilized ovum
and sequencing of mRNAs via the 5’Cap structure
RNA–Sequence
• By homologous recombination in embryonic stem
mRNAs are converted to cDNAs using reverse transcription, and cell. This is called targeted mutagenesis
these cDNAs are amplified and directly sequenced. This method is • Forward Genetics
m

termed RNA–Sequence.
• Animal Cloning.
co

GRO–Sequence (Global Run–on Sequencing)


A method where nascent transcript are specifically captured and
Targeted Mutagenesis
sequenced using NGS sequencing.
This helps to map location of active transcription complexes. Basic Principle of Targeted Mutagenesis
NET–Sequence (Native elongating transcript sequencing)
The normal gene in a very low percentage of ES cells
This technique allows for sequencing of RNA within elongating RNA
polymerase–DNA–RNA ternary complexes. This helps in genome– may be replaced by the neo–disrupted recombinant gene
wide analysis of transcription in living cells. (transgene) by homologous recombination.
WES (Whole Exome Sequencing)
Targeted Mutagenesis can be of different types:
Since exons comprise only about 1% of the human genome, the
exome represents a much smaller and more tractable target than Gene Knock out: Endogenous gene is replaced by mutated trans-
the complete genome. gene by homologous recombination in embryonic stem cell.
Whole Exome sequencing has emerged as an alternative to whole Gene Knock in: Mutated endogenous gene is replaced by normal
genome sequencing as a means for diagnosing rare or cryptic transgene by homologous recombination in embryonic stem cell.
m

genetic diseases. Gene Knock down–si RNA or mi RNA induced gene silencing called
RNA interference or RNAi
co

Gene Knock up: Using transcription factors, transcription of gene


TRANSGENIC ANIMALS is increased.

Foreign genes can be introduced into fertilized egg. Steps for Generation of a Knock out Mouse
Animals that develop from such fertilized egg is called 1. Inactivating a recombinant purified gene
transgenic animal. 2. Culture embryonic stem (ES) cell from mice
The gene of interest is a cloned recombinant DNA with 3. Transfection of ES with cloned mutated non–
its own promoter and a different promoter which can be functional gene
selectively regulated. 4. A few cultured embryonic cells contain the non–
Transgenic Models of Animals functional gene through homologous recombination.
Several organisms have been studied extensively as 5. Isolate ES cells with altered gene
m

genetic models. 6. Microinjection of altered ES cells into mouse-


blastocyst
co

Musmusculus (mouse), Drosophila melanogaster (fruit


fly), Caenorhabditis elegans (nematode), Saccharomyces 7. Implantation of blastocyst into foster mother
cerevisiae (baker’s yeast), and Escherichia coli (colonic 8. Breed to many generation till offspring with altered
bacterium). gene in its germ cell.

Uses of Transgenic Animals Characteristic Features of Different Strategies of


• Study of DNA regulatory elements of a gene Transgenic Models
• Study of functions of gene (role of oncogene in 1. Transgenic: Pronuclear injection of transgene
induction of tumorigenesis) ‒ This technique is commonly used
• Study of disorders susceptible for Gene dosage ‒ Random integration of transgene
(overexpression or underexpression) ‒ Genomic DNA or cDNA constructs can be
• Used as a precursor to gene therapy injected
m

• For studying the physiologic effects of insertion or ‒ Variable copy numbers of transgene
co

deletion of a particular gene ‒ Variable expression in each individual founder


• Providing unique genetic models for doing animal ‒ Gain–of–function models produced due to over
experiments in pathology and Pharmacology. expression using tissue–specific promoters
e

e
m

m
m
co

328 |  Self Assessment and Review of Biochemistry

‒ Loss–of–function models produced using Characteristics of gene knock out


antisense and dominant negative transgenes • Tissue–specific knock–out possible
‒ Inducible expression possible • Absence of phenotype possible due to redundancy,
‒ Applicable to several species. may not be always due to knock out.
2. Targeted mutagenesis
By homologous recombination Characteristics of gene knock in
m

‒ Site specific integration of transgene • Predominantly used in mice


co

‒ Predominantly used in mice • Can accurately model human disease


m
co

Fig. 14.9: Generation of transgenic mice by transgenesis and targeted mutagenesis

Latest technique of gene modulation in targeted mutagenesis are: • Cloning of genetically identical individuals is possible
• miRNA/Si RNA-mediated Gene Silencing called as Gene Knock • May affect lifespan
m

down.
• Lots of Ethical concerns.
co

• CRISPR Cas 9-mediated gene editing (loss of function of gene


otherwise knock down or gain-of-function of genes) Successful Stories of Animal Cloning
Forward Genetics • Ian Wilmut and Keith Campbell cloned sheep named
• Mutations created randomly by ENU (N–ethyl–N– Dolly in 1996
nitrourea) • First lamb born to Dolly is Bonnie.
• A phenotype selected Antisense Nucleic Acid in Research and Therapy
• Then its genetic characterization done
• Antisense nucleic acid can be RNA or DNA
• Useful for identifying novel genes.
• It can be natural or synthetic
Animal Cloning
• It is complementary to mRNA
Nucleus of an oocyte is removed. This enucleated
• Si RNA can be used
oocyte is fused nucleus of a somatic cell. The fused cell
Antisense nucleic acid binds with target mRNA, it is
m

is implanted into the uterus of a surrogate mother. This


is also called Nuclear transfer. selectively destructed or inhibited
co

Characteristics of Animal Cloning Uses


• Successful in several mammalian species including • Gene knock down to study function of a gene
sheep, mice, cows, monkeys • Treat viral infection like HIV.
e

e
m

m
m
co

Molecular Biology Techniques and Recent


Advances in Molecular Biology  | 329

HYBRIDOMA Certain Mutation can also result in RFLP if it alters the


Restriction Site
• Technique to produce monoclonal antibodies in the
Sickle cell anemia is an example of a genetic disease
clinical laboratory
caused by a point mutation. The sequence altered by the
• Introduced by Georges Kohler and Cesar Milstein
mutation abolishes the recognition site of the restriction
in 1975
endonuclease Mst-ii.
• Monoclonal antibodies are antibodies against a
m

specific epitope of the antigen. Procedure of RFLP


co

• DNA is extracted from the cell


Steps
• DNA is cleaved by a specific Restriction endonuclease
1. Spleen cells from immunized animal fused with mice
myeloma cell to produce hybrid cell. (Polyethylene • DNA fragments obtained are separated by Agarose
GlycolQ, PEG –1500 is the fusing agent). Gel Electrophoresis
2. Grown in HAT (Hypoxanthine, Aminopterine and • DNA fragments are denatured
Thymidine) medium • Transferred to Nitrocellulose membrane (Southern
Unfused normal cells die as they lack multiplication blot)
potential. • Treated with Radiolabelled probe
Unfused Myeloma Cells Die • DNA variation detected by looking for hybridization
Aminopterine, a folate antagonist inhibit de novo by autoradiography.
purine synthesis and hence DNA synthesis as they
Uses of RFLP
m

lack HGPRTase enzyme.


Tracing chromosomes from parent to offspring
co

Fused Normal Cell and Myeloma Cell Survive


Prenatal diagnosis of diseases
As normal cells provide HGPRTase enzyme.
• Direct diagnosis of sickle cell disease using RFLP
Myeloma cell have multiplication potential.
• Indirect, prenatal diagnosis of phenylketonuria
3. Select Positive clones and expand.
4. Indefinite amount of antibodies harvested. Medicolegal uses
Application of Monoclonal antibodies Detect mutations, point mutation, insertion, deletion.
‒ Enumeration of lymphocyte subpopulation
DNA Fingerprinting
‒ Quantitative preparation of specific cells
• The use of normal genetic variation in the DNA (SNP
‒ Nephelometric assay of blood components
or VNTR or RFLP) to establish a unique pattern of
‒ To prepare Monoclonal antibodies for ELISA
DNA fragments for an individual
‒ Quantitative preparation of pure antigens.
m

• This is also called DNA Profiling


co

• Most commonly used is VNTR or repeat length


RESTRICTION FRAGMENT LENGTH polymorphism
POLYMORPHISM (RFLP)
• The process of DNA fingerprinting was invented by
• Commonly called as ‘rif lip’ Alec Jeffreys in 1985.
Definition
DNA Footprinting
An inherited difference in the pattern of restriction
• DNA with protein bound is resistant to digestion by
map produced by the digestion of a specific restriction
endonuclease is called Restriction fragment length DNase enzymes
polymorphism. • When a sequencing reaction is performed using such
This is because of certain DNA variations present in DNA, a protected area, representing the ‘footprint’
the genome that create a new restriction site (the site of the bound protein, will be detected
where the restriction endonuclease cleave) or abolish a • Because nucleases are unable to cleave the DNA
m

restriction site. directly bound by the protein.


co

Two type of DNA Variations that result in RFLP are: Chromosome Walking
1. Single Nucleotide Polymorphism (SNP) • Method to isolate and clone target DNA from a long
2. Variable Number Tandem Repeat (VNTR) segment of DNA
e

e
m

m
m
co

330 |  Self Assessment and Review of Biochemistry

• In Chromosome walking, a fragment representing Oligopotent cells


one end of a long piece of DNA is used to isolate • Can form more than one cell lineage but are more
another fragment, that overlaps the first but extends restricted than multipotent cells. Oligopotent cells
the first are sometimes called progenitor cells or precursor
• This process continued till the target DNA is isolated cells, e.g. Neural Stem cells.
• Cystic fibrosis (CF) gene was the first to be isolated
Unipotent cells or monopotent cells
m

solely by chromosome walking.


• Can form a single differentiated cell lineage, e.g.
co

Technique of Chromosome Walking spermatogonial stem (SS) cells.


• Gene X to be isolated from a long of DNA
Nuclear reprogramming
• The exact location of Gene X is also not known
• A probe directed against a fragment of DNA in the • The reversal of the terminally differentiated cells to
5’ end is available totipotent or pluripotent cells.
• The initial probe hybridize fragment 1 Trans differentiation
• This is used to detect a probe to detect fragment 2 • Lineage–committed multipotent cells, possessing the
• This process repeated until the fragment that contain capacity to differentiate into cell types outside their
Gene X is reached. lineage restrictions.
m

Types of Stem Cell


co

Name Source Properties


Embryonic stem Blastocysts or Maintain a stable
Fig. 14.10: Chromosome walking cells (ES, ESC) immuno–surgically euploid karyotype even
isolated inner cell after extensive culture
Chromosome Jumping mass (ICM) from and manipulation.
• Bypass regions difficult to clone, such as those blastocysts Can differentiate into a
variety of cell types
containing repetitive DNA, that cannot be easily
in vitro
mapped by chromosome walking.
Embryonic germ Primordial germ EG cells show
cells (EG, EGC) cells (PGCs) from essentially the same
STEM CELL BIOLOGY embryos pluripotency as ES cells
Trophoblast stem Trophectoderm TS can contribute
Stem Cell cells (TS, TSC) exclusively to all
m

trophoblast subtypes
The most widely accepted stem cell definition is a cell
co

when injected into


with a unique capacity to produce unaltered daughter blastocysts
cells (Self Renewal) and to generate specialized cell Extra embryonic ICM from XEN cells can contribute
types (potency). endoderm cells blastocysts only to the parietal
(XEN) endoderm lineage when
Totipotent cells injected into a blastocyst
• Can form an entire organism autonomously Embryonal Teratocarcinoma They can contribute
• Only a fertilized egg (zygote) possesses this feature. carcinoma cells to nearly all cell types
(EC) when injected into
Pluripotent cells blastocysts
• Can form almost all of the body’s cell lineages Mesenchymal Bone marrow, MS cells can
(endoderm, mesoderm, and ectoderm), including stem cells (MS, muscle, adipose differentiate into
germ cells, e.g. Embryonic Stem (ES) cells. MSC) tissue, peripheral mesenchymal cell
m

blood, and umbilical types, including


Multipotent cells cord blood adipocytes, osteocytes,
co

• Can form multiple cell lineages but cannot form all chondrocytes, and
myocytes
of the body’s cell lineages, e.g. Hematopoetic Stem
(HS) cells. Contd...
e

e
m

m
m
co

Molecular Biology Techniques and Recent


Advances in Molecular Biology  | 331

Contd... GENE THERAPY


Name Source Properties
Multipotent Bone marrow MAPCs can differentiate Novel Area of Therapeutics
adult stem cells mononuclear cells into all tissues in vivo Intracellular delivery of genes to generate a therapeutic
(MAPC) when injected into a
mouse blastocyst, and effect by correcting an existing abnormality.
can differentiate into Divided into
m

various cell lineages


of mesodermal, 1. Somatic cell gene therapy—Gene is introduced into
co

ectodermal, and somatic cells.


endodermal origin
in vitro
2. Germ cell gene therapy (Transgenic animal).
Spermatogonial Newborn testis SS cells can
stem cells (SS, reconstitute long–term
Methods of Gene DeliveryQ
SSC) spermatogenesis after Chemical methods of gene delivery
transplantation into
recipient testes and DEAE–dextran
restore fertility • Diethylaminoethyl–dextran (DEAE Dextran) is a
Germline stem Neonatal testis GS cells can
poly cationic derivative of the carbohydrate polymer,
cells (GS, GSC) differentiate into three
germlayers in vitro and dextran
contribute to a variety • Because of its positive charge, DEAE–dextran is able
of tissues, including
germline, when injected to bind to the anionic phosphodiester backbone of
m

into blastocysts DNA


co

Multipotent adult Adult testis maGSC can differentiate • The resultant complex maintains an overall cationic
germline stem into three germlayers
charge and is able to bind to negatively charged cell
cells (maGSC) in vitro and can
contribute to a variety membrane surfaces
of tissues, including • Subsequently, the complex is internalized, presumably
germline, when injected
into blastocysts by endocytosis.
Neural stem cells Fetal and adult NS cells can Calcium PhosphateQ
(NS, NSC) brain (subventricular differentiate into neuron
zone, ventricular and glia in vivo Mixing DNA with calcium chloride, and then carefully
zone, and and in vitro. Recently, adding this mixture to a phosphate buffered saline
hippocampus) the culture of solution followed by incubation at room temperature.
pure population of
symmetrically dividing This generates a DNA–containing precipitate, which
adherent NS cells is then dispersed onto cultured cells.
m

became possible
The precipitate is then taken into the cells via
co

Unrestricted Mononuclear USSCs can differentiate


somatic stem fraction of cord into a variety of cell endocytosis or phagocytosis.
cells (USSC)Q blood types in vitro and can Advantages of the calcium phosphate method are:
contribute a variety of
cells types in in vivo • Simplicity
transplantation • Low cost
experiments
• Applicability to a wide variety of cell types
Epistem cells Early postimplanta- EpiSCs can differentiate
(EpiSC) tion epiblast into three germlayers in • Generate stably transfected cell lines, allowing for
vitro and form teratomas long–term gene expression studies.
but cannot contribute
to normal tissues when Cationic Lipids (Lipofection)
injected into blastocysts
When lipids mixed with DNA in water, the lipids formed
Induced Variety of terminally A number of somatic cell
pluripotent stem differentiated cells types can be converted hollow spheres, called liposomes, with DNA entrapped
m

cells (iPS, iPSC) and tissue stem into iPS cells using in the aqueous center.
co

cells. different combinations


of transcription factors
When these liposomes were added to cells growing
and treatment with small in vitro, some of the liposomes would fuse with cellular
molecule plasma membranes and be taken up into the cells via
NB: Just have an overall idea about this table given in Harrison endocytosis.
e

e
m

m
m
co

332 |  Self Assessment and Review of Biochemistry

Polymers Contd...
• More recently, a variety of organic polymers have Viral vectors Main advantages Main disadvantages
been utilized for transfection Adenoviral Highly effective in Viral capsid elicits
• One of the most popular is the polycation, poly transducing various strong immune
ethylenimine (PEI) tissues responses

• PEI is an organic macromolecule that possesses a Adeno-Associated Elicits few Limited packaging
viral inflammatory capacity
m

high cationic charge density, sometimes known as a responses,


“proton sponge”
co

nonpathogenic
• It condenses DNA into positively charged particles Human foamy virus Persistent gene Need a stable
that interact with anionic cell surfaces and enter cells expression in both packaging system
dividing and non-
via endocytosis.
dividing cells
Physical Methods of Gene Delivery HSV–1 Large packaging Residual cytotoxicity
Microinjection capacity with with neuron specificity
persistent gene
It entails the direct injection of DNA into the nuclei of transfer
target cells using fine glass needles under microscopy.
Simian virus–40 Wide host cell Limited packaging
Conceptually, microinjection is the simplest gene range; lack of capacity
delivery method, but one of the most difficult to apply. immunogenicity
Tedious and time–consuming, typically allowing only Alpha viruses Limited immune Transduced gene
a few hundred cells to be transfected per experiment. responses against expression is
m

the vector transient


ElectroporationQ
co

Electroporation is a method of introducing nucleic acids Nonviral Vectors


into cells by exposing the cells to a rapid pulse of high– Nonviral Main
voltage current, causing pores in the cell membrane to Vectors Main advantages Disadvantages
open temporarily. Transposon/ 1. Undetermined, probably Early stage in
Transposase large packaging capacity. development
This allows exogenous DNA to pass through the pores system
and into the cytoplasm of the cells. 2. Transfects many cell types
with long–term gene ex-
Typically, the gene transfer efficiency is relatively low, pression
and electroporation frequently results in a high incidence Liposome 1. Undetermined, probably Expensive to
of cell death. large packaging capacity. produce
Gene GunQ 2. Transfects many cell types.
Plasmid DNA is coated onto metal microparticles and Large holding capacity to
m

enable a high number of


then blasted into cells using either electrostatic force or base pairs
co

gas pressure. Naked DNA 1. Undetermined, probably Transient


Some of the DNA becomes trapped by a few cells, and large packaging capacity. and low–level
may then be expressed to sufficient levels. 2. Efficient in gene transfer.
expression
This technique is fast, simple and safe, and it can 3. Limited immunogenicity
transfer genes to a wide variety of tissues. Site Specific 1. Undetermined, probably Early stage in
Also, there appears to be no limits to the size or number Integrase large packaging capacity. development
of genes that can be delivered. 2. Specific integration site
Viral Vectors in Gene Delivery
Strategies of Gene Transfer
Viral vectors Main advantages Main disadvantages
1. In vivo
Retroviral Persistent gene Theoretical risk
transfer in dividing of insertional Gene is transferred directly into the patient.
m

cells mutagenesis 2. Ex vivo


co

Lentiviral Persistent gene Might induce Removal of target cell.


transfer in oncogenesis in some
Gene transfer into target cells.
transduced tissues cases
Return of target cell into the patient after gene
Contd... transfer.
e

e
m

m
m
co

Molecular Biology Techniques and Recent


Advances in Molecular Biology  | 333

Application of Gene Therapy in Medicine • This approach most commonly involves transfer of
Gene therapy clinical trials addressed in: the missing gene to a physiologically relevant target
1. Cancer (67%)—Most common cell
2. Vascular Disease (8.9%) • First genetic disorder addressed by gene therapy is
3. Monogenic Disorders (8.6%) Severe Combined Immunodeficiency
• Father of Gene therapy is French Anderson.
m

Gene Therapy in Cancers


Gene Therapy for Other Disease
co

Local/Regional Approach
Suicide gene/prodrug • Parkinson’s disease: AAV vectors expressing
• Intratumoral injection of an adenoviral vector enzymes required for enhanced production of
expressing the thymidine kinase (TK) gene dopamine, or of the inhibitory neurotransmitter
• Cells that take up and express the TK gene are Gamma aminobutyric acid
susceptible to Gancyclovir. • In Alzheimer’s disease, fibroblasts are transduced
with a retroviral vector expressing nerve growth
Used for the treatment of:
factor.
• Glioblastoma multiforme
• Locally recurrent tumors of prostate, breast, and
HUMAN GENOME PROJECT
colon.
Suppressor oncogene In 1990, the United States launched a multibillion dollar
m

effort, the Human Genome Project, for the express


Adenoviral–mediated expression of the tumor suppressor
co

purpose of developing the automated high–throughput


gene for p53
techniques, instrumentation, and data mining software
• Squamous cell carcinoma of the head and neck
necessary to determine the entire DNA sequence of the
• Esophageal cancer
Homo sapiens genome.
• Non–small cell lung cancer
Bioinformatics
Oncolytic viruses
The discipline concerned with the collection, storage,
These selectively replicate in tumor cells and not in
and analysis of biologic data, mainly DNA and protein
normal cells.
sequences.
• Squamous cell carcinoma of the head and neck.
“Omics” of Molecular Biology
Systemic Gene Therapy
Genome: The complete set of genes of an organism
Immunomodulation
m

Genomics: In depth study of the structures and functions of genomes.


Immune–enhancing genes encoding cytokines, Transcriptome: The complete set of RNA transcripts produced by
co

chemokines, or co–stimulatory molecules. the genome during a fixed period of time.


Transcriptomics: The comprehensive study of transcriptome.
Anti–angiogenesis
Proteome: The complete complement of proteins of an organism.
• Constitutive expression of angiogenesis inhibitors Proteomics: The systematic study of structures and functions of
such as angiostatin and endostatin proteomes and their variation in health and disease.
• Use of siRNA to reduce levels of VEGF or VEGF Glycome and Glycomics: The glycome is the total complement of
receptor. simple and complex carbohydrates in an organism. Glycomics is the
systematic study of the structures and functions of glycomes such
Gene Therapy for Vascular Disease as the human glycome.
Lipidome and Lipdomics: The lipidome is the complete complement
Transgene for VEGF, fibroblast growth factor (FGF) and
of lipids found in an organism. Lipidomics is the in–depth study of the
hypoxia–inducible factor which increase blood flow. structures and functions of all members of the lipidome and of their
• Skeletal muscle (critical limb ischemia) interactions, in both health and disease.
m

• Cardiac muscle (angina/myocardial ischemia). Metabolome and Metabolomics: The metabolome is the complete
co

complement of metabolites (small molecules involved in metabolism)


Gene Therapy for Genetic Disorders present in an organism. Metabolomics is the in–depth study of their
structures, functions, and changes in various metabolic states.
• Gene transfer strategies for genetic disease generally
involve gene addition therapy Contd...
e

e
m

m
m
co

334 |  Self Assessment and Review of Biochemistry

Contd... modifications, etc. have been verified by manual


Nutrigenomics: The systematic study of the effects of nutrients on curation
genetic expression and of the effects of genetic variations on the
• TrEMBL, on the other hand, contains empirically
metabolism of nutrients.
Pharmacogenomics: The use of genomic information and technolo-
determined and genome derived protein sequences
gies to optimize the discovery and development of new drugs and whose potential functions have been assigned, or
drug targets annotated, automatically—solely on the basis of
m

Exome: The nucleotide sequence of the entire complement of mRNA computer algorithms
co

exons expressed in a particular cell, tissue, organ or organism.


Thus, while TrEMBL currently includes more than 80
million entries, Swiss–Prot contains slightly more than
TECHNIQUES TO STUDY PROTEOMICS 500,000.
First generation Proteomics Gen Bank
• To purify proteins sample–SDS PAGE or 2D
• The genetic sequence database
Electrophoresis
• The goal of GenBank, of the National Institutes
• To determine the amino acid sequence–End group
of Health (NIH), is to collect and store all known
analysis like Sanger’s reagent and Edman’s reagent.
biological nucleotide sequences and their translations
Second generation Proteomics in a searchable form.
To purify protein in the sample Nanoscale Chromato- PDB
m

graphic techniques
co

• The Protein Data Base (PDB) is a repository of the three


To determine amino acid sequence: dimensional structures of proteins polynucleotides,
• Mass Spectrometry and other biological macromolecules.
• Mud PIT (Multidimensional Protein Identification • The PDB presently contains over 95,000 three–
Technology) dimensional structures for proteins.
MudPIT (Multidimensional Protein Identification Technology) Tagged SNPs
Successive rounds of chromatography to resolve the peptides pro-
duced from complex biologic sample into simpler fractions, that can • When sets of SNPs localized to the same chromosome
be analyzed separately by Mass Spectrometry. are inherited together in blocks, the pattern of SNPs
in each block is termed a haplotype
BIOINFORMATIC AND GENOMIC RESOURCES • Tag SNPs, is a subset of the SNPs in a given block
m

sufficient to provide a unique marker for a given


The large collection of databases that helps in
co

haplotype.
contemporary molecular, biochemical, epidemiological,
• Selected regions are then subject to more detailed
and clinical research.
study to identify the specific genetic variations
UniProt KB that contri-bute to a specific disease or physiologic
The UniProt Knowledgebase, UniProtKB, is jointly response.
sponsored by the Swiss Institute of Bioinformatics and HapMap
the European Bioinformatics Institute.
• Haplotype Map (HapMap) Project, a comprehensive
UniProtKB’s stated objective is “to provide the effort to identify SNPs associated with common
scientific community with a comprehensive, high–quality human diseases and differential responses to
and freely accessible resource of protein sequence and pharmaceuticals
structural information”.
m

• The long–term goal of the project is to provide earlier


It is organized into two sections: and more accurate diagnosis of potential genetic risk
co

• Swiss–Prot contains entries whose assigned factors that lead to improved prevention and more
functions, domain structure, post–translational effective patient management.
e

e
m

m
m
co

Molecular Biology Techniques and Recent


Advances in Molecular Biology  | 335

ENCODE Linkage Analysis


• National Human Genome Research Institute • The genetic marker loci very close to the disease
(NHGRI) initiated the ENCODE (Encyclopedia of allele, that are transmitted through pedigrees called
DNA Elements) Project linkage disequilibrium
• ENCODE is a collaborative effort that combines • Assess the genetic marker loci in family members
laboratory and computational approaches to identify having the disease or trait of interest
m

every functional element in the human genome • With time it becomes possible to define a panel
co

• These include mapping sites of DNA methylation, of marker loci, all of which co–segregate with the
assessing local histone methylation etc. putative diseases
Entrez Gene • Hence, Linkage analysis facilitates localization and
cloning of the disease allele
Entrez Gene, a database maintained by the National
Center for Biotechnology Information (NCBI). • The genetic markers used are, SNPs and repeat–length
polymorphisms (minisatellite and microsatellite
• This provides a variety of information about
repeats).
individual human genes
• The information includes the sequence of the genome GWAS
in and around the gene, exon–intron boundaries, the • In GWAS large cohorts of patients with and without a
sequence of the mRNA (s) produced from the gene, disease (rather than families) are examined across the
and any known phenotypes associated with a given
m

entire genome for genetic variants or polymorphisms


mutation of the gene in question.
co

that are over–represented in patients with the disease


dbGAP • This identifies regions of the genome that contain a
• dbGAP, the Database of Genotype and Phenotype, variant gene or genes that confer disease susceptibility
is an NCBI database that complements Entrez Gene. • The causal variant within the region is then
• dbGAP compiles the results of research into the links provisionally identified using a “candidate gene”
between specific genotypes and phenotypes. approach
• By this approach, in the provisionally identified
LINKAGE AND ASSOCIATION STUDIES region, genes are selected based on how tightly they
are associated with the disease and whether their
Principle of Linkage analysis and Association studies biologic function seems likely to be involved in the
Based on the concept of linkage disease under study
• When two genes are close together on a chromosome,
m

• Thus, localize the causal gene or in some instances,


they are transmitted together unless a recombination
co

functional polymorphism associated with it.


event separates them
• Genes which are close together are less likely to be Comparison of Linkage and Association Studies
separated by a recombination event.
Linkage Study Association Study
Linkage and Association Studies Study conducted in the same Compare a population of
There are two primary strategies for mapping genes that family or same sibships affected individuals with a
cause or increase susceptibility to human disease: control population
Analysis of monogenic traits Suitable for identification of
1. Classic linkage analysis can be performed based
susceptibility genes in polygenic
on a known genetic model or, when the model is and multifactorial disorders
unknown, by studying pairs of affected relatives Difficult to obtain sufficient More statistical power for
2. Disease genes can be mapped using allelic association statistical power for complex complex multigenic disorder
m

studies (Genome Wide Association Studies, GWAS). traits


co
e

e
m

m
m
co

336 |  Self Assessment and Review of Biochemistry

REVIEW QUESTIONS

Recombinant DNA Technology In RFLP, DNA is cut using a Restriction endonuclease.


1. cre-cis regulatory elements bind to what site? They cut DNA at specific pallindromic sites. So it
(AIIMS May 2015) produces a characteristic restriction map.
m

a. RE site 4. Function of endonucleases (TN 97)


co

b. FTR site a. Cut DNA at specific DNA sequences


c. Lox b4 site b. To point out the coding regions
d. INT site c. Enhancers
Ans. c. Lox b4 site (Ref: Harper 30/e page 454) d. To find out antibiotic resistance
Recombinases Recognition site Ans. a. Cut DNA at specific DNA sequence
CRE Recombinase Bacterial Lox P site (Ref: Harper 30/e page 453)
λ phage encoded INT protein Bacteriphage λ tt site • Restriction endonuclease cut DNA at specific
Yeast Flp Recombinases Yeast FRT site palindromic sites
• These enzymes are isolated from bacteria
2. cDNA used in gene amplification in bacteria of • They restrict the entry of phages into the bacteria.
genomic DNA because: (PGI May 2014)
5. Enzymes used in DNA research programme are,
m

a. Easy to replicate
except: (PGI June 97)
co

b. Human genome has many introns that cannot a. Polymerase


be removed by bacteria
b. Exonuclease
c. Promoter are not found
c. Nuclease
d. Complete genome cannot be replicated
d. Alkaline phosphatase
Ans. a, b, d. Easy to replicate, Human genome has many
e. None
introns that cannot be removed by bacteria, Complete
Ans. e. None (Ref: Harper 30/e page 453)
genome cannot be replicated (Harper 30/e page 455)
Enzyme Reaction Uses
cDNA
DNA complementary to mRNA is called cDNA or Copy Alkaline phosphatase Dephosphorylates Removal of 5’-PO 4
5’ ends of RNA and groups prior to kinase
DNA or Complementary DNA. DNA labeling; also used to
Procedure to prepare cDNA prevent self-ligation
m

• Isolate the mRNa DNA ligase C a t a l y z e s b o n d s Joining of DNA mol-


co

between DNA mol- ecules


• By the action of Reverse Transcriptase RNA-DNA ecules
hybrid is synthesized DNA polymerase I Synthesizes double- Synthesis of double-
• By RNAse H RNA is digested stranded DNA from stranded cDNA; Nick
• By DNA Polymerase double stranded DNA is single-stranded DNA translation;
Generation of blunt
produced. ends from sticky ends
Advantages of cDNA over genomic DNA Thermostable DNA Synthesize DNA at Polymerase chain
• Contains only coding sequences polymerases elevated tempera- reaction (DNA syn-
(Taq Polymerase) tures (60–80° C) thesis)
• Represent the mRNA in a tissue
DNase I Under appropriate Nick translation
• Hence used to study gene expression. conditions, produces Mapping of hyper-
single-stranded nicks sensitive sites
3. Restriction Endonuclease is used in (JIPMER 2013)
in DNA M a p p i n g p r o te i n -
a. RFLP
m

DNA interactions
b. PCR
co

Exonuclease III Removes nucleo- D N A s e q u e n c i n g


c. FISH tides from 3’ ends Mapping of DNA-
d. SDS-PAGE of DNA protein interactions
Ans. a. RFLP Contd...
e

e
m

m
m
co

Molecular Biology Techniques and Recent


Advances in Molecular Biology  | 337

Contd... 8. Function of restriction II enzyme (AI 2012)


Enzyme Reaction Uses a. Prevents protein folding
λ Exonuclease Removes nucleo- DNA sequencing b. Removing formed DNA
tides from 5’ ends c. Cleaves DNA at palindromic recognition site
of DNA
d. Negative supercoiling
Polynucleotide Transfers terminal 32P end-labeling of
kinase phosphate (ϒ posi- DNA or RNA
Ans. c. Cleaves DNA at palindromic recognition site.
m

tion) from ATP to 5’- (Ref: Harper 30/e page 452)


co

OH groups of DNA
or RNA 9. After digestion by restriction endonucleases
Reverse transcrip- Synthesizes DNA Synthesis of cDNA
DNA strands can be joined again by:
tase from RNA template from mRNA; RNA (5’ (AIIMS May 2011) (Nov 2010)
end) mapping studies a. DNA polymerase
S1 nuclease Degrades single- Removal of ‘hairpin’ b. DNA ligase
stranded DNA in synthesis of cDNA;
RNA mapping studies
c. DNA topoisomerase
(both 5’ and 3’ ends) d. DNA gyrase
Terminal transferase Adds nucleotides to Homopolymer tailing Ans. b. DNA ligase (Ref: Harper 30/e page 453)
the 3’ ends of DNA
CRISPR-Cas9 RNA targeted DNA Genome deiting and
Enzymes Involved in the DNA Replication
directed Nuclease modulation of gene 1. Topoisomerases
m

expression ‒ Relieve torsional strain that results from helicase-


induced unwinding of DNA
co

6. In DNA transfer, the vectors used from smallest ‒ Nicking Resealing Enzyme
to largest is: (PGI Dec 07) 2. Helicase: ATP driven processive unwinding of DNA
a. Cosmids, Plasmids, Bacteriophage 3. Single Strand Binding Protein (SSB) Prevent
b. Plasmids, Bacteriophage, Cosmids premature reannealing of dsDNA
c. Bacteriophage, Cosmids, Plasmids 4. DNA Primase
d. Cosmids, Bacteriophage, Plasmids ‒ Initiates synthesis of RNA primers
e. Plasmids, Cosmids, Bacteriophage Special class of DNA dependent RNA Polymerase.
Ans. b. Plasmids, Bacteriophage, Cosmids 5. DNA Polymerase: Catalyse the chemical reaction of
(Ref: Harper 30/e page 455) DNA Polymerization. Synthesize DNA only in 5’ to
3’ direction.
The DNA insert size in ascending order is Plasmid
6. DNA Ligase: Seals the single strand nick between
m

< Phage < Cosmids < BAC/PAC < YAC Cloning capacity
the nascent chain and Okazaki fragments on lagging
co

of common cloning vectors


strand.
Vector DNA insert size (Kbp)
10. Starting material for production of insulin from
Plasmid 0.01–10
bacteria is: (AIIMS May 2011)
Lambda Phage 10–20 a. Genomic DNA of lymphocytes
Cosmid 35–50 b. mRNA of lymphocytes
BAC, PAC 50–250 c. Genomic DNA of beta cell of pancreas
YAC 500–3000 d. mRNA of beta cells of pancreas
Ans. d. mRNA of beta cells of pancreas
7. In gene cloning, largest fragment can be (Ref: Vasudevan and Sreekumari 7/e page 626)
incorporated in: (AIIMS Dec 95) 11. True statement about Restriction Endonuclease:
a. Plasmid (PGI May 2012)
m

b. Bacteriophage a. Palindromic Sequence Observed


co

c. Cosmid b. Protects bacteria from infection by virus


d. Retrovirus c. Present only in Eukaryotes
Ans. c. Cosmid (Ref: Harper 30/e page 455) d. Restrict replication of DNA
e

e
m

m
m
co

338 |  Self Assessment and Review of Biochemistry

Ans. a, b. Palindromic Sequence Observed (b) Protects Two types of Gene Library:
bacteria from infection by virus. 1. Genomic DNA Library:
(Ref: Harper 30/e page 452) Prepared from total genomic DNA of an organism.
Restriction Enzymes By digestion of Genomic DNA by Restriction
• Recognizes and cleaves a specific palindromic Endonuclease.
double-stranded DNA sequence that is typically Then recombinant clones of such digested DNA is
m

4–7 bp long. produced by recombinant DNA technology.


co

• These DNA cuts result in blunt ends (e.g. HpaI) 2. cDNA library
overlapping (sticky or cohesive) ends (e.g, BamHI) cDNA is prepared from mRNA by the action of
• Are a key tool in recombinant DNA research. Reverse Transcriptase
• These enzymes were called restriction enzymes The recombinant clones for cDNA are produced by
because their presence in a given bacterium restricted Recombinant DNA Technology.
the growth of certain bacterial viruses called
bacteriophages. Advantages of cDNA over Genomic DNA
• Each enzyme recognizes and cleaves a specific • Contains only coding sequences.
double-stranded DNA sequence that is typically • Represent the mRNA in a tissue.
4–7 bp long. • Hence, used to study gene expression.
• Restriction endonucleases are present only in cells
13. Restriction enzymes: (PGI Nov 2009)
m

that also have a companion enzyme, site-specific


a. Prevent elongation of DNA
co

DNA methylases, that site-specifically methylates


the host DNA, rendering it an unsuitable substrate b. Break DNA to create sticky end
for digestion by that particular restriction enzyme. c. Cuts at palindromic sites
Thus, prevent the digestion of host DNA. d. Restriction sites are not specific
e. Breaks at sugar-phosphate bond
Restriction Enzymes are Named after the Bacterium
from which They are Isolated Ans. a, b, c, e. Prevent elongation of DNA, Break DNA
to create sticky end, Cuts at palindromic sites, breaks at
For example, EcoRI is from Escherichia coli, and BamHI is
sugar-phosphate bond. (Ref: Harper 30/e page 455)
from Bacillus amyloliquefaciens.
The first three letters in the restriction enzyme name 14. Correct statements regarding restriction endonu-
consist of the first letter of the genus (E) the first two clease is/are: (PGI Dec 03)
letters of the species (co). These may be followed by a a. Restriction endonuclease recognizes specific
m

strain designation (R) sites of DNA sequence


co

A roman numeral (I) to indicate the order of discovery b. Restriction endonuclease recognizes short
(e.g. EcoRI and EcoRII). sequence of DNA
c. It acts at 5’ – 3’ direction
12. True about Gene Library: (PGI Nov 2009)
d. It acts at 3’ – 5’ direction
a. Library of Gene books
Ans. a. Restriction endonuclease recognizes specific sites
b. Plasmid with copies of different genes
of DNA sequence. (Ref: Harper 30/e page 457)
c. Computer database with all gene knowledge
d. Collection of gene copies of one organism as 15. True about the function of Restriction endonu-
completely as possible in bits and pieces. clease: (PGI Dec 06)
e. DNA fragments. a. Cut both the strands of ds DNA
Ans. d, e. Collection of gene copies of one organism as b. The cut ends produced are sticky
completely as possible in bits and pieces, DNA fragments. c. The cut ends produced are blunt
m

(Ref: Harper 30/e page 455) d. Cuts single strand of DNA


co

Gene Library Ans. a, b, c. Cut both the strands ..., The cut ends produced
A collection of recombinant DNA clones generated from are sticky. The cut ends produced are blunt.
a specific source. (Ref: Harper 30/e page 457)
e

e
m

m
m
co

Molecular Biology Techniques and Recent


Advances in Molecular Biology  | 339

Restriction endonuclease cut both strands of DNA. They • Primer.


produce sticky ends or blunt ends depending on which • MgCl2, KCl
restriction endonuclease act on the DNA.
19. For PCR which of the following is not required:
PCR (AIIMS May 2007)
a. Taq polymerase
16. Real-time PCR is used for: (AIIMS May 2013)
b. d-NTP
m

a. Multiplication of RNA
c. Primer
co

b. Multiplication of specific segment of DNA


c. Multiplication of Protein d. Radiolabelled DNA probe
d. To know how much amplification has occurred Ans. d. Radiolabelled DNA Probe
Ans. d. To know how much amplification has occurred (Ref: Tietz Textbook of Clinical
(Ref: Tietz Textbook of Clinical Chemistry 4/e page 1446)
Chemistry 4/e page 1446) 20. SYBR Green Dye is used for: (AIIMS May 2008)
• Real-Time (Homogeneous, Kinetic) Polymerase a. HPLC
Chain Reaction b. Immunofluorescence
• Real-time PCR describes methods by which the c. PCR
target amplification and detection steps occur
d. ELISA
simultaneously in the same tube. Thus it is a method
Ans. c. PCR (Ref: Tietz Textbook of Clinical
of quantitative PCR or q PCR.
m

Chemistry 4/e page 1446)


• Quantitaion is done in the exponential phase of
co

amplification. Real-time PCR


• This is a type of Quantitative PCR.
17. Quantitative DNA analysis/estimation is done • Methods used to quantitate PCR Products in Real
by: (AIIMS May 2012) time PCR
a. pH meter • Intercalating Dyes—Ethidium Bromide, SYBR Green.
b. Sphymometer • Sequence Specific Probes—TaqMan, Molecular beacon
c. Spirometer • Fret Probes—Fluorescence Resonance Energy
d. Spectrometer Transfer Probes.
Ans. d. Spectrometer
• Absorbance of UV light at 260 nm can be used to 21. True about DNA polymerase used in PCR
estimate DNA (PGI May 2013)
a. Obtained from virus
m

• Can be done using spectrophotometer or simply


spectrometer. b. Obtained from bacteria
co

• Other options are self-explanatory. c. Used for joining two strands


d. It is heat stable
18. All are added to PCR, except: (AIIMS Nov 2012) e. Add nucleotide
a. Deoxynucleotide
Ans. b, d, e. Obtained from bacteria. It is heat stable, Add
b. Dideoxynucleotide nucleotide (Ref: Tietz Textbook of Clinical
c. Thermostat DNAP Chemistry 4/e page 1446)
d. Template DNA DNA Polymerase used in PCR is Taq Polymerase. This
Ans. b. Dideoxynucleotide is thermostable polymerase obtained from bacteria
(Ref: Tietz Textbook of Clinical Thermophilus Aquaticus. Taq Polymerase add nucleotide
Chemistry 4/e page 1446) during extention of PCR Cycle.
Pre-requisites of PCR
m

22. Enzyme(s) used in polymerase chain reaction is/


• Sample DNA to be amplified are: (PGI May 2011)
co

• Deoxynucleotides a. Restriction endonuclease


• Thermostable Polymerase-Taq Polymerase obtained b. DNA polymerase
from Thermus Aquaticus found in hot springs. c. Alkaline phosphate
e

e
m

m
m
co

340 |  Self Assessment and Review of Biochemistry

d. RNA polymerase c. Li +
e. Reverse transcriptase d. Na +
Ans. b. DNA Polymerase (Ref: Tietz Textbook of Clinical Ans. b. Mg++ (Ref: Tietz Textbook of Clinical
Chemistry 4/e page 1446) Chemistry 4/e page 1446)
Pre-requisites of PCR: 26. In PCR Acquaticus thermophilus is preferred
• Sample DNA to be amplified over E coli, because: (PGI Dec 07)
m

• Deoxy nucleotides. a. Thermostable at temperature at which DNA


co

• Thermostable Polymerase—Taq Polymerase obtained liquefies


from ThermusAquaticus found in hot springs. b. Proofreading done
• Primer c. Done in more precisely
• Mg Cl2 d. Does not require primer
e. Better DNA replication
23. True about PCR all except: (PGI June 2009)
a. Carried out by thermostable DNA-polymerase Ans. a. Thermostable at temperature at which DNA
liquefies (Ref: Tietz Textbook of Clinical
b. Exponential amplification
Chemistry 4/e page 1446)
c. Additive amplification
d. Specific amplification 27. Which of the following is not true about PCR:
e. Single-stranded DNA required (PGI 2003)
a. Thermostable enzyme is used
m

Ans. a. Additive amplification.


b. Annealing is done after DNA denaturation
co

Polymerase Chain Reaction


c. Specific primers are required
• Revolutionary technique invented by Karry B Mullis
in 1989. d. Required at least 1st week time for synthesis
• He got nobel prize for this in 1993. e. DNA polymerase has to be added to each cycle
• The polymerase chain reaction (PCR) is a test tube Ans. d, e. Required at least 1st week time for synthesis,
method for amplifying a selected DNA sequence. DNA polymerase has to be added to each cycle.
(Ref: Tietz Textbook of Clinical
Exponential amplification of the sample Chemistry 4/e page 1446)
• The number of samples after ‘n’ number of cycles
is 2n. 28. In PCR true is/are: (PGI June 08)
• One cycle of PCR require 20–30 seconds. a. Thermostable enzyme is needed
• 20 cycles result in million-fold amplification of the b. 2n copies formed after ‘n’ numbers of Cycle
m

target DNA. c. Non specific


co

• Product obtained by amplification is called Amplicon. d. Thermolabile enzyme


• The instrument that takes samples through the e. Primer is needed
multiple steps of changing temperature in PCR Cycle Ans. a, b, e. Thermostable enzyme is needed, 2n copies
is called Thermocycler. formed after ‘n’ numbers of Cycle, Primer is needed
24. PCR is used in: (Ker 2006) Blot Techniques
a. Medicolegal cases
b. Amplification of gene 29. Western blot detects (AIIMS Nov 2009)
c. Identification of organism a. DNA
d. All of the above b. RNA
Ans. d. All of the above (Ref: Tietz Textbook of Clinical c. Protein
Chemistry 4/e page 1446) d. mRNA
m

Ans. c. Protein (Ref: Harper 30/e page 457)


25. Which of the following is used in PCR?
co

(AIIMS Nov 07) Blot Techniques


a. Ca ++ Southern Blot
b. Mg ++ • Devised by Edward Southern in 1975
e

e
m

m
m
co

Molecular Biology Techniques and Recent


Advances in Molecular Biology  | 341

• Technique to detect specific DNA Segment Ans. b. Northern blot


• Principle: Based on specific base pairing rule of • Southern blot detect DNA by DNA-DNA
complementary nucleic acid strands hybridization
• It is a DNA-DNA Hybridization. • Northern blot detect RNA by RNA-cDNA
hybridization
Northern Blot
• Western blot detect Protein by Antigen antibody
• Technique used to detect specific RNA
m

Interaction.
• Principle: RNA-DNA hybridization technique
co

• Radioactive labeled cDNA Probes used.


Cytogenetic Techniques
Western Blot (Immunoblot) analysis for Proteins
• Technique to detect specific protein in a sample 32. Which method is used to locate a known gene
locus? (AIIMS May 2013)
• Antigen antibody interaction
a. FISH
• Radioactive labeled antibody used.
b. CGH
South Western Blotting c. Chromosome Painting
• To examine Protein–DNA Interaction d. RT-PCR
• Dot blot Technique The step, blotting to nitrocellulose Ans. a. FISH (Ref: Robbins 9/e page 177)
membrane is avoided Robbins 9th edition gives the following description:
m

• The sample is directly applied to slots on a specific • FISH uses DNA probes that recognize sequences
co

blotting apparatus containing nylon membrane. This specific to particular chromosomal regions.
is also called slot blot.
• FISH requires prior knowledge of the one or few
30. Confirmatory test for proteins are: specific chromosomal regions suspected of being
(PGI May 2014) altered.
a. Western Blot • Genomic abnormalities can also be detected without
b. ELISA prior knowledge of what these aberrations may be,
using a global strategy such as array CGH.
c. Chip assay
• From the above description it is concluded that
d. Dot blot
• To locate a known gene locus FISH can be used.
Ans. a, b, c, d. Western blot, ELISA, Chip assay and dot
blot is based on Antigen antibody interaction. Hence, • To locate an unknown gene locus, a global strategy
they are confirmatory test for proteins. Chip is the other like CGH is used.
m

name for Microarray. Just like DNA Chip, where DNA –


co

33. Light microscopy resolution to visualise chromo-


DNA Hybridization is done, there Protein Microarray or somes: (AIIMS May 2013)
Protein Chip where Antigen antibody interaction is done. a. 500 kb
Dot blot technique: In blotting technique, the blotting b. 5 mb
to nitrocellulose membrane is avoided, instead they
c. 50 mb
are arranged in different slots. This is called dot blot
d. 5 kb
technique. This is applied to detect protein.
Ans. b. 5 mb
31. Which is the test used to identify mRNA (Ref: Emery’s Elements of Medical
(JIPMER Nov 2015) Genetics 13th edition page 33)
a. Southern Blot G Banding generally provides high quality Chromosome
b. Northern Blot analysis with approximately 400-500 bands per haploid
m

c. Western Blot set. Each of these band corresponds on an average to


co

d. South Western Blot approximately 6000-8000 kilobases (i.e. 6-8 mb)


e

e
m

m
m
co

342 |  Self Assessment and Review of Biochemistry

Requires Detects deletion Detects balanced structural Detects Uniparental


Method growing cells and duplication rearrangements Disomy Lower limits of detection
G Banding Yes Yes Yes No 5–10 mb
Metaphase FISH Yes Yes Yes No 40–250 thousand mb
Interphase FISH No Yes Some No 40–250 thousand mb
CGH array No Yes No No Single Exon or Single gene
m

SNP array No Yes No Some Single Exon or Single gene


co

34. Test to differentiate in the chromosome of normal 36. Rapid method of chromosome identification in
and cancer cell: (AIIMS Nov 2012) intersex is: (AIIMS May 2008)
a. PCR a. FISH
b. Comparative Genomic Hybridization b. PCR
c. Western Blotting c. SSCP
d. Southern Blotting d. Karyotyping
Ans. b. Comparative Genomic Hybridization Ans. a. FISH (Robbins 9/e page 177, 178)
(Ref: Robbins and Cotran Pathologic Certain probes used in FISH hybridize to repetitive
basis of disease 9/e Page 178, 179) sequences located to the pericentromeric regions. These
Array-Based Comparative Genomic Hybridization probes are useful for the rapid identification of certain
m

(Array CGH) trisomies in interphase cells of blood smears, or even in


the rapid analysis of prenatal samples from cells obtained
co

• This is also a hybridization technique done on a


Microarray or DNA Chip, hence in the name array- through amniocentesis. Such probes are available for
based. chromosomes 13, 18, and 21 and for the sex pair X and Y.
• Here two Genomes are compared, hence the name Interphase FISHQ
Comparative Genomic Hybridization. • Allows rapid diagnosis within 24–48 hours.
Uses of array CGH • Especially useful for amniocytes.
• Detect Gene Amplification (e.g. Microduplication) 37. Which of following techniques is used for
• Detect Gene Deletion. (eg. Subtelomeric deletion, detection of variation in DNA sequence and Gene
Microdeletion) expression: (AI 2010)
• Detect Copy number variations a. Northern Blot
• Detect Aneuploidy b. Southern Blot
m

• Hence, used diseases of unknown etiology like c. Western Blot


co

Cancer, Autism, Mental Retardation, Child with d. Microarray


dysmorphic features etc.
Ans. d. Microarray (Robbins 9/e page 177, 178)
35. Karyotyping under light microscopy is done by: • DNA microarray can be used to detect DNA sequence
(AIIMS Nov 2009) variations
a. R-banding • RNA microarray can detect Gene expression.
b. Q-banding
38. Which of the following tests is not used for
c. G-banding
detection of specific aneuploidy (AI 2010)
d. C-banding a. FISH
Ans. c. G-Banding b. RT-PCR
(Ref: Emery’s Elements of Medical c. QF-PCR
m

Genetics 13/e page 30, 31)


d. Microarray
co

R-Banding, G-Banding and C-Banding are done under


Ans. b. RT-PCR
light microscopy.
As G-Banding is the most common method it is the (Henrys Clinical Diagnosis and Management
correct answer. by Laboratory Methods 22/e 1294.1297) Use of a
e

e
m

m
m
co

Molecular Biology Techniques and Recent


Advances in Molecular Biology  | 343

DNA method, QF-PCR, in the prenatal diagnosis of Test to Detect Mutations


fetal aneuploidies. J Obstet Gynaecol Can. 2011 Sep; 1. Ames Test, Test to detect mutagenicity
33(9):955-60. Special strains of Salmonella typhimurium have
• Quantitative Fluorescent PCR is a real-time PCR in mutated histidine gene.
which fluorescent dyes are used for quantitation Hence, they will grow only in medium containing
• RT–PCR is used to amplify a RNA target. Not a histidine gene.
m

method to detect aneuploidy. This is called reverse mutation.


co

Methods of aneuploidy detection The number of colonies is proportional to the quantity


• Conventional Karyotyping of mutagens.
• FISH 2. Site Directed Mutagenesis
• Microarray based CGH Michael Smith in 1993
An oligodeoxyribonucleotide whose sequence
• Multiple ligation dependent Probe Amplification
is complementary to a part of known gene is
(MLPA)
synthesized.
• Real-time PCR
A specific deletion/insertion is produced in the
• Quantitative Fluorescent PCR (QF-PCR)
oligodeoxynucleotide.
• Multiplex amplifiable probe hybridization (MAPH).
It is then extended by DNAP.
39. For isolating a gene of long DNA molecules (50- This altered gene is amplified and expressed by
m

100 KB) following is used: (AIIMS Dec 98) insertion into a cloning vector.
co

a. Chromosome walking This allows the study of a particular mutation.


b. Nitch Other Mutation Detection Technique
c. RFLP
Method Type of Mutation Detected
d. SSLP
Cytogenetic analysis Numerical or structural abnormali-
Ans. a. Chromosome walking ties in chromosomes
(Ref: Harper 30/e page 463) Fluorescent in situ hybridiza- Numerical or structural abnormali-
tion (FISH) ties in chromosomes
Chromosome Walking
Southern blot Large deletion, insertion, rearrange-
• Method to isolate and clone target DNA from a long ment, expansions of triplet repeat,
segment of DNA amplification
• In Chromosome walking, a fragment representing Polymerase chain reaction Expansion of triplet repeats, variable
m

(PCR) number of tandem repeats (VNTR),


one end of a long piece of DNA is used to isolate gene rearrangements, transloca-
co

another fragment, that overlaps the first but extends tions; prepare DNA for other muta-
the first. This process continued till the target DNA tion methods
is isolated. Reverse transcriptase PCR Analyze expressed mRNA (cDNA)
(RT-PCR) sequence; detect loss of expression
• Cystic fibrosis (CF) gene was the first to be isolated
solely by chromosome walking. DNA sequencing Point mutations, small deletions and
insertions
Mutation Detection Techniques Restriction fragment polymor- Point mutations, small deletions and
phism (RFLP) insertions
40. Techniques used to detect Gene Mutation is/are: Single-strand conformational Point mutations, small deletions and
(PGI May 2012) polymorphism (SSCP) insertions

a. RTPCR Denaturing gradient gel elec- Point mutations, small deletions and
trophoresis (DGGE) insertions
b. Denaturing Gradient Gel Electrophoresis
m

RNAse cleavage Point mutations, small deletions and


c. DNA Sequencing
co

insertions
d. Restriction Fragment Length Polymorphism Oligonucleotide specific Point mutations, small deletions and
e. Single Strand Conformational Polymorphism hybridization (OSH) insertions

Ans. a, b, c, d, e. Contd...
e

e
m

m
m
co

344 |  Self Assessment and Review of Biochemistry

Contd... c. DNA sequencing


Method Type of Mutation Detected d. Northern blot analysis
Microarrays Point mutations, small deletions and Ans. d. Northern blot analysis
insertions Genotyping of SNPs
Methods to detect point mutation
Protein truncation test (PTT) Mutations leading to premature
truncations Method Type of Mutation Detected
m

Pyrosequencing Sequencing of whole genomes of Southern blot Large deletion, insertion, rearrange-
microorganisms, resequencing of ment, expansions of triplet repeat,
co

amplicons amplification
Multiplex ligation-dependent Copy number variations DNA sequencing Point mutations, small deletions and
probe amplification (MLPA) insertions
Restriction fragment poly- Point mutations, small deletions and
41. Correct statement about Restriction fragment morphism (RFLP) insertions
gene: (PGI May 2011) Single-strand confor- Point mutations, small deletions and
mational polymorphism insertions
a. Detected by Southern blot
(SSCP)
b. Detected by Northern blot
Denaturing gradient gel Point mutations, small deletions and
c. Used for identification of gene for genomic electrophoresis (DGGE) insertions
mapping RNAse cleavage Point mutations, small deletions and
d. RFLP is DNA variation sequence insertions
m

Ans. a, c, d. Detected by Southern blot, Used for Oligonucleotide specific Point mutations, small deletions and
identification of gene for genomic mapping, RFLP is DNA hybridization (OSH) insertions
co

variation sequence (Ref: LippincottIllustrated Microarrays Point mutations, small deletions and
insertions Genotyping of SNPs
Biochemistry 6/e page 475)
The procedure of RFLP
44. DNA fingerprinting is based on possessing in
• DNA is extracted from the cell DNA of: (PGI Dec 08)
• DNA is cleaved by a specific restriction endonuclease a. Constant Tandem Repeat
• DNA fragments obtained are separated by Agarose b. Variable Number Tandem Repeat
Gel Electrophoresis c. Non-repetitive sequence
• DNA fragments are denatured d. Exon
• Transferred to Nitrocellulose membrane (Southern e. Intron in eukaryotes
blot)
Ans. b. Variable Number Tandem Repeat
• Treated with Radiolabelled probe
m

• DNA variation detected by looking for hybridization DNA Fingerprinting


co

by autoradiography. • The use of normal genetic variation in the DNA (SNP


or VNTR or RFLP) to establish a unique pattern of
42. DNA fingerprinting was found by: (PGI June 06) DNA fragments for an individual.
a. Watson • This is also called DNA Profiling.
b. Galton • Most commonly used is VNTR or repeat length
c. Jeffrey polymorphism.
d. Sanger • The process of DNA fingerprinting was invented by
e. Wilkins Alec Jeffreys in 1985.
Ans. c. Jeffrey • Primer is needed.

43. The following methods can be used to detect the 45. RFLP, true is/are: (PGI Dec 06)
point mutation in the beta globulin gene that a. Detects mutation
m

causes sickle cell anemia, except: (AI 06) b. Recognizes trinucleotide repeats
co

a. Polymerase chain reaction with allele—specific c. Detects deletion


oligonucleotide hybridization d. Blunt ends are produced
b. Southern blot analysis e. Always short ends are produced
e

e
m

m
m
co

Molecular Biology Techniques and Recent


Advances in Molecular Biology  | 345

Ans. a, c, d. Detects mutation, Detects deletion, Blunt Transfection: Introduction of gene through nonviral
ends are produced (Ref: Harper 30/e page 463) vectors.
Two type of DNA Variations that result in RFLP are: Transduction: Introduction of gene through viral vectors.
1. Single Nucleotide Polymorphism (SNP) Site directed recombination: The method by which
endogenous gene is replaced by passenger gene by
2. Variable Number Tandem Repeat (VNTR)
homologous recombination.
Uses of RFLP
m

‒ Tracing chromosomes from parent to offspring. Fluorescent In situ Hybridization (FISH)–Not a


co

‒ Prenatal diagnosis of diseases method of gene delivery.


- Direct diagnosis of sickle cell disease using Gene Therapy
RFLP 48. The first gene therapy (somatic enzyme) was
- Indirect, prenatal diagnosis of phenylke- successfully done in: (PGI Dec 07)
tonuria a. SCID
‒ Medicolegal uses b. Phenylketonuria
‒ Detect mutations, point mutation, insertion, c. Thalassemia
deletion.
d. Cystic fibrosis
Gene Therapy e. Alkaptonuria
46. Natural methods of horizontal gene transfer in Ans. a. SCID
Dr French Anderson did the first gene therapy to treat
m

bacteria: (PGI Nov 2009)


SCID.
co

a. Transformation
b. Transduction 49. Gene therapy methods are: (PGI June 03)
c. Conjugation a. Electroporation
d. Electroporation b. Intranuclear injection
e. Mutation c. Site directed mutagenesis
Ans. a, b, c. Transformation, Transduction, Conjugation d. Retrovirus
• Electroporation is an artificial method of gene Ans. a, b, c, d. Electroporation, Intranuclear injection, Site
delivery directed mutagenesis, Retrovirus.
• Trinucleotide ..., X chromosome ...
50. Purpose of gene therapy: (PGI June 03)
47. Methods of introducing gene in target cells are all
a. Replacement of abnormal gene by normal gene
except: (AIIMS Nov 2010)
b. Replacement of normal gene by abnormal gene
m

a. Electroporation
c. Knock out of abnormal gene
co

b. Transfection
d. Introduction of viral gene
c. Site directed recombination
Ans. a. Replacement of abnormal gene by normal gene.
d. FISH
Novel area of therapeutics
Ans. d. FISH
(Ref: lib.store.yahoo.net/.../How-to-Choose-the-Optimal- Gene Therapy
Gene-Delivery-Method.pdf) Intracellular delivery of genes to generate a therapeutic
Electroporation effect by correcting an existing abnormality
Electroporation is a method of introducing nucleic acids Divided into:
into cells by exposing the cells to a rapid pulse of high- 1. Somatic cell gene therapy—Gene is introduced into
voltage current, causing pores in the cell membrane to somatic cells.
open temporarily. 2. Germ cell gene therapy (Transgenic animal).
m

This allows exogenous DNA to pass through the pores


and into the cytoplasm of the cells.
Transgenic animals
co

Typically, the gene transfer efficiency is relatively low, 51. RNAi in gene expression denotes
and electroporation frequently results in a high incidence a. Knockdown (AIIMS May 2015)
of cell death. b. Knock up
e

e
m

m
m
co

346 |  Self Assessment and Review of Biochemistry

c. Knock in c. Inserting in transgenic mice


d. Knock out d. Inserting as a knock out gene
Ans. a. Knock down (Ref: Harper 30/e page 447) Ans. d> c. Inserting as a knock out gene > Inserting in
Gene Knock out: Endogenous gene is replaced by transgenic mice. (Ref: Text book of biochemistry
mutated transgene by homologous recombination in Thomas M Devlin 7/e page 292, 293)
embryonic stem cell • Inserting in transgenic mice site specific integration
m

Gene Knock in: Mutated endogenous gene is replaced is not possible.


by normal transgene by homologous recombination in
co

• Knock out transgenesis is by homologous


embryonic stem cell recombination. So site specific integration possible.
Gene Knock down: siRNA or miRNA induced gene So d>c.
silencing called RNA interference or RNAi
Gene Knock up: Using transcription factors, transcription Bioinformatics
of gene is increased. 54. The following are used to study pathological
52. True statement about transgenic mice: genome except:
(PGI Nov 2010) a. Genbank
a. Developed from DNA insertion into fertilized b. Entrez gene
egg c. Hapmap
b. Have same genome as parents except one or d. BLAST
more genes Ans. d. BLAST (Ref: Harper 30/e page 98, 99)
m

c. Identical genome to parent mice Bioinformatic and Genomic Resources


co

d. Produced by breeding over several generations The large collection of databases that have been
e. Homozygous are selected developed for the assembly, annotation, analysis and
Ans. a, b, d, e. Developed from DNA insertion into distribution of biological and biomedical data reflects
fertilized egg, Have same genome as parents except the breadth and variety of contemporary molecular,
one or more genes, Produced by breeding over several biochemical, epidemiological, and clinical research. The
generations. Homozygous are selected prominent bioinformatics resources: UniProt, GenBank,
(Textbook of biochemistry and the Protein Database (PDB) represent three of the
Thomas M Devlin 7/e page 292, 293) oldest and most widely used bioinformatics databases.
Different Strategies of Genetic Modification Uniprot: The world’s most comprehensive resource
Injection of the transgene into the male pronucleus of on protein structure and function
fertilized ovum. Genbank: The store all known biological nucleotide
m

Steps for generation of a Knock out Mouse sequences and their translations in a searchable form
co

• Inactivating a recombinant purified gene PDB (Protein Data Base) a repository of the three-
• Culture embryonic stem (ES) cell from mice dimensional structures of proteins, polynucleotides,
• Transfection of ES with cloned mutated non- and other biological macromolecules
functional gene HapMap is a comprehensive effort to identify SNPs
• A few cultured embryonic cells contain the non- associated with common human diseases and differential
functional gene through homologous recombination responses to pharmaceuticals.
• Isolate ES cells with altered gene ENCODE (Encyclopedia of DNA Elements) Project-
• Microinjection of altered ES cells into mouse Identification of all thefunctional elements of the genome
blastocyst will vastly expand our understanding of the molecular
• Implantation of blastocyst into foster mother events that underlie human development, health, and
• Breed to many generation till offspring with altered disease.
gene in its germ cell Entrez Gene provides a variety of information about
m

• Homogygous are selected. individual human genes. The information includes


co

the sequence of the genome in and around the gene,


53. The function of a gene is determined by: exon-intron boundaries, the sequence of the mRNA(s)
a. Southern blot (AI 2012) produced from the gene, and any known phenotypes
b. Western blot associated with a given mutation of the gene in question.
e

e
m

m
m
co

Molecular Biology Techniques and Recent


Advances in Molecular Biology  | 347

dbGAP (Database of Genotype and Phenotype), Linkage and Association Studies


compiles the results of research into the links between There are two primary strategies for mapping genes that
specific genotypes and phenotypes cause or increase susceptibility to human disease:
BLAST (Basic Local Alignment Search Tool) is a 1. Classic linkage can be performed based on a known
method to identify protein by homology. This is not a genetic model or, when the model is unknown, by
bioinformatic data base. studying pairs of affected relatives
m

55. Study of structure and products of gene is: 2. Disease genes can be mapped using allelic association
co

(PGI Dec 05) studies (Genome Wide Association Studies, GWAS)


a. Genomics Genome-wide association studies (GWAS) have
b. Proteomics elucidated numerous disease-associated loci and are
c. Bioinformatics providing novel insights into the allelic architecture of
d. Cytogenetics complex traits. These studies have been facilitated by
e. Pharmacogenomics the availability of comprehensive catalogues of human
Ans. a. Genomics (Ref: Harper 30/e page 98, 99) single-nucleotide polymorphism (SNP) haplotypes
generated through the HapMap Project.
• Bioinformatics: The discipline concerned with the
collection, storage, and analysis of biologic data, The data generated by the HapMap Project are greatly
mainly DNA and protein sequences facilitating GWAS for the characterization of complex
• Genome: The complete set of genes of an organism disorders. Adjacent SNPs are inherited together as blocks,
and these blocks can be identified by genotyping selected
• Genomics: In depth study of the structures and
m

marker SNPs, so-called Tag SNPs, thereby reducing cost


functions of genomes
co

and workload.
• Transcriptome: The complete set of RNA transcripts
produced by the genome during a fixed period of 58. Which of the following statement is true about
time. Linkage analysis? (AIIMS Nov 07)
• Transcriptomics: The comprehensive study of a. Detection of characteristic DNA polymorphism
transcriptome in a family associated with disorders
• Proteome: The complete complement of proteins of b. Useful to make pedigree chart to show affected
an organism and non-affected family members
• Proteomics: The systematic study of structures and
c. Used to make a pedigree chart to show non-
functions of proteomes and their variation in health
paternity
and disease
d. Nongene mapping method of genetic study
• Exome: The nucleotide sequence of the entire
complement of mRNA exons expressed in a Ans. a. Detection of characteristic DNA polymorphism
m

particular cell, tissue, organ or organism. in a family associated with disorders


co

(Ref: Robbins 9/e page 177-179)


56. Study of multiplication of proteins in disease
process is called: (AI 07) Linkage analysis
a. Proteomics • The genetic marker loci very close to the disease allele,
b. Genomics that are transmitted through pedigrees called linkage
c. Glycomics disequilibrium.
d. Nucleomics • Assess the genetic marker loci in family members
Ans. a. Proteomics (Ref: Harper 30/e page 98, 99) having the disease or trait of interest
• With time it becomes possible to define a panel
57. What biologist uses to diagnose and treat diseases of marker loci, all of which co-segregate with the
with disorders with multigenic inheritance? putative diseases
a. Gene card • Hence, Linkage analysis facilitates localization and
m

b. Tag SNPs cloning of the disease allele.


co

c. Flipped card • The genetic markers used are, SNPs and repeat-length
d. Virtual Cell polymorphisms (minisatellite and microsatellite
Ans. b. Tag SNPs (Ref: Robbins 9/e page 179) repeats).
e

e
m

m
m
co

348 |  Self Assessment and Review of Biochemistry


m
co
m
co
m
co
m
co
e

e
m

m
5
m
co
m
co

Section Miscellaneous
m
co

C H A P T E R S

15. Vitamins and Minerals


16. Heme Metabolism and Hemoglobins
m

17. TCA Cycle and Biological Oxidation


co

18. Free Radicals, Xenobiotics and Metabolism


of Alcohol
m
co
co co co co co
m m m m m
m
e

m
e

m
e

m
e
e

e
m

m
m
co

15 Vitamins and Minerals


m
co

Topics Included
• Fat Soluble Vitamins
• Water Soluble Vitamins
• Minerals
m

VITAMINS • Retinoic acid: Normal morphogenesis, growth and


co

cell differentiation
Definition
• Retinol: Reproduction.
Vitamins are organic compounds occurring in small
Vitamin A, in the strictest sense, refers to Retinol
quantities in different natural foods and necessary for
growth and maintenance of good health. Carotenoids
Vitamins are mainly classified into • They are provitamins of Vitamin A present in plants
• Fat soluble vitamins: Vitamins A, D, E and K • More than 600 carotenoids in nature, and
• Water soluble vitamins: B Complex Vitamins and approximately 50 of them can be metabolized to
Vitamin C. vitamin A
Endogenously synthesized Vitamins • β Carotene is the most prevalent carotenoid in the
Vitamins are generally not synthesized by the humans, food supply that has provitamin A activity.
m

but some vitamins can be synthesized endogenously. Nonprovitamin A Carotenoids


co

They are: • Lutein and Zeaxanthin: Protect against macular


• Vitamin D from precursor steroids degeneration
• Vitamin K, Biotin, and pantothenic acid by the • LycopeneQ: Protect against prostate cancer.
intestinal microflora
• Niacin from tryptophan, an essential amino acid. Vitamin A Metabolism
Absorption and transport of Vitamin A
Fat Soluble Vitamins • Beta Carotene from plant sources is absorbed and
Vitamin A cleaved to two molecules of Retinal by Beta Carotene
• Ring structure present in Vitamin A is β ionone ring Dioxygenase. Retinal is reduced to retinol by Retinol
• Provitamin A, β carotene contain 2 β ionone ring Reductase
• Cleaved in the intestine by a dioxygenase. • Retinol ester from animal sources is hydrolyzed in
the intestinal lumen to Retinol and absorbed into the
m

Retinoids intestinal cells


co

All compounds chemically related to retinol are called • Retinol from animal and plant sources is reesterified
retinoids. They are: to retinol esters and transported in ChylomicronsQ
• Retinal: 11 cis retinal for normal vision to Liver
e

e
m

m
m
co

352 |  
Self Assessment and Review of Biochemistry

‒ Uptake takes place in liver cells by means of apo Functions of Vitamin A


E receptors. • Vision
‒ Visual process involve 3 forms of Vitamin A
containing pigments
‒ Rhodopsin
- Most light sensitive pigment present in rods
m

- Formed by covalent association between 11


co

cis retinal and 7-transmembrane rod protein


called opsin.
Three iodopsin each responsive to specific colors in cones
in bright light.
• Regulation of gene expression and differentiation
‒ Retinoic Acids are involved in this function
‒ Biologically important retinoic acids are all Trans-
retinoic acid and 9 -cis retinoic acid
‒ They act like steroid hormones
‒ They bind to nuclear receptors.
Retinoic acid receptors
m

Retinoid receptors regulate transcription by binding to


co

specific DNA site.


• Retinoic Acid Receptors (RARs) binds with high
affinity to all: Transretinoic acid and 9 cis retinoic
acid
• Retinoic X receptor (RXRs) binds only to 9 cis retinoic
acid
• Normal reproduction
‒ Retinol is necessary for this function.
• Maintenance of normal epithelium of skin and
mucosa
• Antioxidant Properties and photo protective
property is attributed to Beta Carotenes
m

• Host resistance to infection.


co

Vitamin A deficiency manifestations


• Most common vitamin deficiency
• Most common cause of preventable blindness
• Eyes
Fig. 15.1: Metabolism of Vitamin A ‒ Loss of sensitivity to green light is the earliest
manifestation
Storage of Vitamin A ‒ All the ocular manifestations are collectively
Stored in the Liver Perisinusoidal Stellate (Ito) cells as called as Xerophthalmia
Retinyl Ester (Retinol Palmitate). ‒ Impairment to adapt in dim light, i.e. night
Transport of Vitamin A from Liver to Target Organs blindness or Nyctalopia is the earliest symptom
m

Carried to target sites in the plasma as trimolecular ‒ Conjunctival Xerosis (Dryness of Conjunctiva)
co

complex bound to Retinol Binding Protein (RBP) and ‒ Bitot’s spots (white patches of keratinized
Transthyretin. epithelium appearing on the sclera)
e

e
m

m
m
co

Vitamins and Minerals   | 353

‒ Blinding corneal ulceration and necrosis • Causes yellow staining of skin but not sclera (Unlike
‒ Keratomalacia (softening of the cornea) Hyperbilirubinemia which stain both skin and sclera).
‒ Corneal scarring that causes blindness. Required Daily Allowance of Vitamin AQ (μg of Retinol)
• Skin and Mucosa (ICMR 2010)
‒ Epithelial metaplasia and keratinization
Children (1–6 yrs) 400 µg/day
‒ Hyperplasia and hyperkeratinization of the Men 600 µg/day
m

epidermis with plugging of ducts of adnexal gland Women 600 µg/day


co

produce Follicular HyperkeratosisQ or Papular Pregnancy 800 µg/day


dermatosis. This is called as Phrynoderma or Lactation 950 µg/day
Toad Skin
‒ Squamous Metaplasia in the mucus secreting Units of Vitamin A
epithelium of upper respiratory tract and urinary • Vitamin A in food is expressed as micrograms of
tract retinol equivalent
‒ Loss of taste sensation. • 6 µg of beta Carotene = 1 µg of preformed retinol
Remember • Pure Vitamin A for pharmaceutical uses is expressed
• Concurrent Zinc deficiency can interfere with mobilization of International Units (IU) 1 IU = 0.3 µg of Retinol
Vitamin A from liver stores. • 1 µg of Retinol = 3.33 IU
• Alcohol interferes with conversion of retinol to retinaldehyde in
the eyes. • In 2001 USA Canadian Dietary Reference value
m

introduced the term Retinol Activity Equivalent


co

Vitamin A as therapeutic agent (RAE) 1 RAE = 1 µg of Retinol or 12 µg of Beta carotene.


• β Carotene used in cutaneous Porphyria Sources of Vitamin A
• All transretinoic acid in acute Promyelocytic • Animal food (mainly as Retinol)
Leukemia [called as differentiation therapy] • Plant food as Carotenes.
• 13 cis retinoic acid [Isotretinoin] in cystic Acne
Animal sources
• 13 cis retinoic acid in childhood neuroblastoma.
• Fish liver oilsQ are the rich sources of Vitamin A
Hypervitaminosis A
• Halibut liver oil is the richest source (900000 µg/100
• Common in arctic explorers who eat polar bear liver. g) followed by cod liver oil
• Organelle damaged in hypervitaminosis is • Other animal sources are liver, egg, butter, cheese,
Lysosomes
whole milk, fish and meat.
• Acute toxicity: Pseudotumor cerebriQ (headache,
m

dizziness, vomiting, stupor, and blurred vision, Plant sources


co

symptoms that may be confused with a brain tumor) • Richest plant source is Carrot
and exfoliative dermatitis. In the liver, hepatomegaly • Others are GLV like Spinach, Amaranth, Green and
and hyperlipidemia yellow fruits like papaya, mango, pumpkin.
• Chronic toxicity: If intake of > 50,000 IU/day for > 3
Treatment of Vitamin A deficiency
months
• Weight loss, anorexia, nausea, vomiting, bony • 200000 IU or 110 mg of Retinol Palmitate orally in
exostosis, bone and joint pain, decreased cognition, two successive days.
hepatomegaly progresses to cirrhosis Prevention of Vitamin A deficiency
• Retinoic acid stimulates osteoclast production and • Single massive dose 200000 IU to children (1–6 years)
activity leading to increased bone resorption and once in 6 months
high risk of fractures, especially hip fractures
• Single massive dose 100000 IU to children (6 mo–
m

• In pregnancy retinoids causes teratogenic effects. 1 year) once in 6 months.


co

Carotenemia Assay of vitamin A


• Persistent excessive consumption of foods rich in • Dark adaptation time
Carotenoids • Serum Vitamin A by Carr and Price reaction.
e

e
m

m
m
co

354 |  
Self Assessment and Review of Biochemistry

Vitamin D 25- Hydroxylases. Most abundant circulatory form of


Group of sterols having a hormone like function Vitamin D. This is because there is little regulation of
• Ergocalciferol (Vit D2): Commercial Vitamin D this liver hydroxylation. The measurement of 25-OH
obtained from the fungus, ergot D is the standard method for determining patients’
Vitamin D status
• Cholecalciferol (Vit D3): Endogenous synthesis from 7
Dehydrocholesterol. • Conversion of 25-OH-D into 1, 25-dihydroxy vitamin
m

D, (1, 25 (OH)2D3) or Calcitriol in the kidneyQ, the


Vitamin D metabolism biologically most activeQ form of vitamin D, through
co

• Sources of Vitamin D the activity of α1-hydroxylase. This is the rate limit-


‒ The major source of vitamin D for humans is ing step. PTH and Hypophosphatemia upregulate
its endogenous synthesis in the skin by photo- 1 α Hydroxylase. Hyperphosphatemia and 1, 25 OH
chemical conversion of a precursor, 7-dehydro- D inhibit the enzyme
cholesterol, to Cholecalciferol or Vitamin D3 via • When Ca2+ level is high, kidney produces the rela-
the energy of solar or artificial UV light in the tively inactive metabolite 24, 25 Dihydroxy Chole-
range of 290 to 315 nm (UVB radiation) in the calciferol (Calcitroic acid) excreted through urine.
stratum corneum of the epidermis of skin
‒ Absorption of vitamin D from foods and supple- Functions of Vitamin D
ments in the gut Regulation of calcium and phosphorus homeostasis
• Binding of vitamin D from both of these sources to Action on intestine
m

plasma α1-globulin (D-binding protein or DBP) • Vitamin D increases Ca2+ absorption


co

and transport into the liver. • By increasing the transcription of TRPV6 (a member
of the transient receptor potential vanilloid family),
which encodes a critical calcium transport channel.
This increases Calcium absorption from duodenum.
Action on kidney
• Vitamin D increases Ca2+ and Phosphorus reabsorption
• Increases calcium influx in distal tubules of the
kidney through the increased expression of TRPV5,
another member of the transient receptor potential
vanilloid family.
Action on bones
m

• 1, 25-dihydroxy vitamin D and parathyroid hormone


co

enhance the expression of RANKL (receptor activator


of NF-κB ligand) on osteoblasts
• RANKL binds to its receptor (RANK) located in
preosteoclasts, inducing the differentiation of these
cells into mature osteoclasts
• They dissolve bone and release calcium and phos-
phorus into the circulation.
Immunomodulatory and antiproliferative effects
• Prevent infection by Mycobacterium tuberculosis
• Within macrophages, synthesis of 1, 25-dihydroxy-
m

vitamin D occurs through the activity of CYP27B


co

located in the mitochondria


Fig. 15.2: Metabolism of Vitamin D
• Pathogen-induced activation of Toll-like receptors in
• Conversion of vitamin D into 25-hydroxy cholecal- macrophages causes a transcription-induced increase
ciferol (25-OH-D) in the liver, through the effect of in vitamin D receptor and CYP27B.
e

e
m

m
m
co

Vitamins and Minerals   | 355

• The resultant production of 1, 25-dihydroxy vitamin • Mutations in the gene encoding renal 1α-hydroxylase
D then stimulates the synthesis of cathelicidin, an • Prevent conversion of 25 D to 1,25 D
antimicrobial peptide from the defensin family, • Even with high PTH, as 1 α Hydroxylase is defective,
which is effective against infection by Mycobacterium 1,25 D is low
tuberculosis.
• Usually presents in first 2 years of life
Antiproliferative role of Vitamin D
• With classic features of rickets.
m

1, 25 (OH)2 D level less than 20 ng/mL is associated with increase


in incidence of
co

Concept of biochemical changes in Vitamin D Dependent Rick-


• Colon cancer ets Type I
• Breast cancer • Inspite of secondary hyperparathyroidism,1,25 D will remain
• Prostate cancer decreased as 1 α hydroxylase gene is mutated.

Mineralization of bones
Vitamin D–dependent rickets type 2 (True vitamin D–
• Vitamin D contributes to mineralization of osteoid
resistant rickets)
matrix and epiphyseal cartilage in both flat and long
bones • An autosomal recessive disorder
• It stimulates osteoblast to synthesize calcium binding • Due to mutations in the gene encoding the vitamin
protein osteocalcin involved in deposition of calcium D receptor causing end-organ resistance to the active
during bone development. metabolite 1,25 D
• Presents in infancy with less severe manifestation
Vitamin D deficiency
m

• 50–70% of children have alopecia


• The normal reference range for circulating 25-(OH)
co

D is 20 to 100 ng/mL • Epidermal cyst is also a common manifestation.


• The concentration circulating 25-(OH) D < 20 ng/mL X-linked hypophosphatemic rickets
is called Vitamin D deficiency. • X-linked dominant disorder
Causes inadequate mineralization of bone osteoid • The most common hypophosphatemic rickets
• Before closure of epiphysis: Rickets in children
• The defective gene is called PHEX (PHosphate-
• After closure of epiphysis: Osteomalacia in adults.
regulating gene with homology to Endopeptidases
Biochemical defect of different types of rickets on the X chromosome)
Nutritional Vitamin D Deficiency • The product of this gene have either a direct or
Most common cause of rickets globally. an indirect role in inactivating a phosphatonin or
phosphatonins (FGF-23)
Concept of biochemical changes that occur in nutritional Vitamin
• Mutation of PHEX gene lead to increased level of
m

D deficiency
FGF-23
co

• Due to Vitamin D deficiency, Serum Calcium level and Phosphorus


level is low • Hypophosphatemia with normal PTH, normal
• This causes Secondary Hyperparathyroidism, so PTH level is high calcium and low or inappropriately normal 1,25 D
• This increases the 1 α hydroxylation in kidney, so 1,25 D level
are the lab findings.
increases
• This will increase the Serum Calcium level, but Phosphorus level Phosphatonins (FGF-23)
remain at low level
• Humoral mediator that decrease renal tubular reabsorption of
• So, Serum Calcium level is variable, Serum Phosphorus is low, S
phosphate, therefore decreases serum phosphorus
PTH increase, 25 D is decreased,1,25 D is low initially but later
increase due to secondary hyperparathyroidism. • This also decreases the activity of 1 α hydroxylase, resulting in
deficiency of 1,25 D
Remember • Fibroblast Growth Factor-23 (FGF-23) is the most well characterized
• Serum calcium need not be always low in Rickets phosphatonin
• 1,25 D level also need not be always low in Rickets • Increased level of phosphatonins causes increased excretion of
m

• Serum Phosphorus remain low. phosphorus in urine


• So serum Phosphorus is decreased.
co

Vitamin D–dependent rickets type 1 (Pseudo-vitamin


D–resistant rickets) Autosomal dominant hypophosphatemic rickets
• An autosomal recessive disorder • An autosomal dominant condition
e

e
m

m
m
co

356 |  
Self Assessment and Review of Biochemistry

• Due to a mutation in the gene encoding FGF-23 which • Unlike the other causes of vitamin D deficiency,
prevents the degradation of FGF-23 by proteases. So patients have hyperphosphatemia as a result of
there is increased levels of phosphatonins decreased renal excretion.
‒ Hypophosphatemia with normal PTH, normal Conditions causing over production of phosphatonins which
calcium and low or inappropriately normal 1,25 causes rachitic findings
D are the lab findings. • Tumor-induced osteomalacia
m

• McCune-Albright Syndrome (entity that has triad of Polyostotic


co

Remember fibrous dysplasia, Hyperpigmented macules, polyendocrinopoathy)


• Biochemical findings of X linked and autosomal dominant • Epidermal nevus Syndrome
Hypophosphatemic rickets is same as phosphatonins is excess • Neurofibromatosis in children.
in both
• Hypophosphatemia is due to increased excretion of phosphates Requirement of Vitamin D
through kidney by phosphatonins
• Children: 10 µg/day (400 IU)
• Low or normal 1,25D is due to decreased activity of 1 α Hydroxylase.
• Adults: 5 µg/day (200 IU)
• Pregnancy, Lactation: 10 µg/day (400 IU).
Autosomal Recessive Hypophosphatemic rickets
• Extremely rare disorder due to mutation in the gene Vitamin D is toxic in excess
encoding dentin matrix protein 1, which results in • Upper limit of Vitamin D intake has been set 4000 IU/
elevated level of FGF-23. day
m

• Some infants are sensitive to intakes of vitamin D as


Hereditary Hypophosphatemic rickets with hypercal-
co

low as 50 µg/day , resulting in an elevated plasma


Q

ciuria (HHRH)
concentration of calcium
• Autosomal recessive disorder due to mutation in the
• This can lead to contraction of blood vessels, high
gene for a sodium phosphate cotransporter in the
bloodpressure, and calcinosis—the calcification of
proximal renal tubules
soft tissues
• Hypophosphatemia, stimulates production of 1,25 D
• Although excess dietary vitamin D is toxic, excessive
• This causes increased intestinal absorption of calcium exposure to sunlight does not lead to vitamin D
• Symptoms of rickets, along with muscle pain, bone poisoning, because there is a limited capacity to form
pain short stature with disproportionate decrease in the precursor, 7-dehydrocholesterol, and prolonged
length of lower extremities, kidney stones. exposure of previtamin D to sunlight leads to formation
Chronic Renal Failure of inactive compounds.
m

• There is decreased activity of 1α-hydroxylase in Beneficial effects of Vitamin D


co

the kidney, leading to diminished production of • Protective against the cancer of Prostate, Colorectal cancer
1,25-D. • Protective against Prediabetes, and metabolic Syndrome.

Laboratory findings in disorders causing rickets


Disorder Serum Calcium S Phosphorus PTH 25 (OH)D 1,25 (OH)D ALP
Vitamin D Deficiency N/Decrease Decrease Increase Decrease Decrease, N, Increase
Increase
Vitamin D Dependent Rickets Type I N/Decrease Decrease Increase N Decrease Increase
Vitamin D Dependent Rickets TypeII N/Decrease Decrease Increase N Increased Increase
Chronic renal Failure N/Decrease Increase Increase N Decrease Increase
X Linked Hypophosphatemic Rickets N Decrease Normal N Relatively Increase
m

Decrease
co

Autosomal Dominant Hypophosphatemic N Decrease Normal N Relatively Increase


Rickets Decreased
e

e
m

m
m
co

Vitamins and Minerals   | 357

Sources of Vitamin D • Bronchopulmonary dysplasia


• Sunlight • Intraventricular hemorrhage of prematurity
• Foods: Only animal sources Liver, Egg yolk, butter • Treat intermittent claudication
and liver oils. Out of the food sources Fish liver oils • Slow the aging process.
are the richest source
Toxicity of Vitamin E
• The richest source of Vitamin D is also Halibut Liver
• Reduce platelet aggregation and interfere with
m

oil.
Vitamin K.
co

Assay of Vitamin D Required daily allowance


• The release into the circulation of osteocalcin • Males 10 mg/day
provides an index of vitamin D status
• Females 8 mg/day
• 25(OH) Vitamin D level is measured in the serum
• Pregnancy 10 mg/day
indicate Vitamin D status.
• Lactation 12 mg/day.
Vitamin E Sources of Vitamin E
• Vitamin E is a collective name for all stereoisomers of Vegetable oils like Wheat germ oil, sunflower oil, Cotton
tocopherols and tocotrienols seed oil, etc.
• The most powerful naturally occurring antioxidantQ.
Vitamin K
Ring Structure present in Vitamin E
• Naphthoquinone derivative with long isoprenoid
m

• Chromane (Tocol) ring with isoprenoid side chain side chain


co

• Vitamin E is carried to liver in Chylomicron. • Letter K is the abbreviation of German word,


Biochemical functions of Vitamin E Koagulation Vitamin.
• Biologically most potent form of Vitamin E is α Three forms of Vitamin K
TocopherolQ • Vitamin K1: Phylloquinone from dietary sources
• Chain-breaking antioxidantQ and is an efficient • Vitamin K2: Menaquinone Synthesized by Bacterial
pyroxyl radical scavenger that protects low-density Flora
lipoproteins (LDLs) and polyunsaturated fats in
• Vitamin K3: Menadione (and Menadiol diacetate):
membranes from oxidation
Synthetic, Water Soluble.
• Lipid soluble antioxidant.
Functions of Vitamin K
Relationship with Selenium
Vitamin K is required for the post-translational carboxy-
• Selenium decrease the requirement of Vitamin E . Q

lation of glutamic acid (Gamma Carboxylation), which is


m

Vitamin E deficiency necessary for calcium binding to γ carboxylated proteins.


co

• Axonal degeneration and of the large myelinated • Prothrombin (factor II)


axons and result in posterior column and spinocere- • Factors VII, IX, and X
bellar symptoms • Protein C, protein S
• Hemolytic anemia: The erythrocyte membranes are • Proteins found in bone (osteocalcin)
abnormally fragile as a result of poor lipid peroxida- • Matrix Gla protein
tion, leading to hemolytic anemia
• Nephrocalcin in kidney
• Peripheral neuropathy initially characterised by
• Product of growth arrest specific gene Gas6.
Areflexia with progression to ataxic gait, decreased
position and vibration sense Drugs causing Vitamin K deficiency
• Spinocerebellar ataxia Warfarin and Dicoumoral inhibit γ carboxylation by com-
• Skeletal myopathy petitively inhibiting the enzyme that convert vitamin K
m

• Pigmented retinopathy to its active hydroquinone form


co

• Ophthalmoplegia. • Antiobesity drug orlistat.


Vitamin E in high doses may protect against Vitamin K Deficiency
• Oxygen-induced retrolental fibroplasia • Elevated prothrombin time, bleeding time
e

e
m

m
m
co

358 |  
Self Assessment and Review of Biochemistry

• Newborns, especially premature infants are parti- Peripheral nervous system


cularly susceptible to Vitamin K deficiency because • Typically a symmetric motor and sensory neuropathy
of low fat stores, low breast milk levels of vitamin K, with pain, paraesthesia and loss of reflexes. The legs
sterility of the infantile intestinal tract, liver imma- are affected more than the arms.
turity, and poor placental transport.
Hypervitaminosis K Central nervous system
Wernicke’s Encephalopathy–in alcoholics with chronic
m

• Hemolysis
Thiamine deficiency
co

• Hyperbilirubinemia
• Kernicterus and brain damage. • Horizontal Nystagmus
• Ophthalmoplegia
Water Soluble Vitamins • Truncal ataxia
• B Complex Vitamins
• Confusion
• Vitamin C
• Wernicke- Korsakoff Syndrome
Thiamin (Vitamin B1) • Along with features of Wernicke’s Encephalopathy
• Thiamin is also called Aneurine • Amnesia
Sources • Confabulatory psychosis.
• Aleurone layer of cereals. Hence whole wheat flour
and unpolished hand pound rice has better nutritive Acute pernicious (fulminating) beriberi (shoshin beriberi),
value. Yeast is also a good source of thiamine. in which heart failure and metabolic abnormalities
m

predominate.
Active form of Thiamin
co

Thiamine Pyrophosphate (TPP) also called Thiamine Biochemical assessment of thiamin deficiency
diphosphate (TDP). • Erythrocyte Transketolase activity is reduced
Thiamine and nerve conduction • Urinary Thiamine excretion.
Thiamin triphosphate has a role in nerve conduction; it phosphory- Thiamin toxicity
lates, and so activates, a chloride channel in the nerve membrane.
• There is no known toxicity of thiamine
Coenzyme Role of Thiamine PyrophosphateQ Recommended Daily Allowance (RDA) of Vitamin B1
Thiamine generally function in the decarboxylation reac- • 1–1.5 mg/day.
tion of alpha keto acids and branched chain amino acids
• Pyruvate DehydrogenaseQ which convert Pyruvate Riboflavin (Vitamin B2)
to Acetyl CoA • Is called Warburg Yellow enzyme Q of cellular
m

• αKetoGlutarate DehydrogenaseQ in Citric Acid respiration


Cycle which convert α KetoGlutarate to Succinyl
co

• Riboflavin is heat stable


CoA
• Enzymes containing riboflavin are called Flavo-
• Branched Chain Ketoacid DehydrogenaseQ which proteins
catalyses oxidative decarboxylation of Branched
Chain Amino acids • Act as respiratory coenzyme and an electron donor.
• Trans KetolaseQ in Pentose Phosphate PathwayQ. Active forms of Riboflavin
This is the biochemical basis of assay of Thiamine • They are FAD (Flavin Adenine Dinucleotide) and FMN (Flavin
status of the body. Mononucleotide)
Coenzyme Role of Riboflavin
Deficiency of Vitamin B1 (Thiamin)
FMN Dependent EnzymesQ
BeriBeriQ • L- Amino Acid Oxidase
Two types • NADH Dehydrogenase (Complex I of ETC)
1. Wet beriberi: Marked peripheral vasodilatation, • Monoamino Oxidase
m

resulting in high output cardiac failure with dysp- FAD Dependent Enzymes
co

noea, tachycardia, cardiomegaly, pulmonary and • Complex II (Succinate Dehydrogenase) of ETC


peripheral edema. • D Amino Acid Oxidase
2. Dry beriberi: Involves both peripheral and central • Acyl CoA Dehydrogenase
nervous system. Contd...
e

e
m

m
m
co

Vitamins and Minerals   | 359

Contd... Contd...
• Alpha Ketoglutarate Dehydrogenase NADPH generating ReactionsQ
• Pyruvate Dehydrogenase • Glucose 6 Phosphate Dehydrogenase in HMP shunt pathway
• Xanthine Oxidase. • 6 PhosphoGluconate Dehydrogenase in HMP shunt pathway
• Cytoplasmic Isocitrate Dehydrogenase
Deficiency manifestation of Vitamin B2 (Riboflavin) • Malic Enzyme. (NADP Malate Dehydrogenase).
Magenta tongue (Glossitis), angular stomatitis, Seborrheic
m

Other function of NAD


Dermatitis, Cheilosis, Corneal vascularization, anemia
co

NAD is the source of ADP-ribose for the ADP-ribosylation of


Biochemical Assessment of Nutritional status of
proteins and polyADP-ribosylation of nucleoproteins involved
Riboflavin
in the DNA repair mechanism.
• Measurement of activation of erythrocyte Glutathione
Reductase by FAD added in vitro Deficiency of niacin
• Urinary excretion of Riboflavin. Pellagra
• Photosensitive Dermatitis: Symmetric dermatitis in
Riboflavin toxicity
the sun exposed areas
• Riboflavin toxicity is not reported yet because of
• Skin lesions are dark, dry and scaling
limited absorption capacity of GIT.
• Casal’s NecklaceQ The rash form a ring around the
RDA of Riboflavin
neck
• 1.5 mg/day.
• Dementia
m

Niacin or Nicotinic Acid (Vitamin B3) • Insomnia, irritability, and apathy and progresses
co

• Not strictly a Vitamin to confusion, memory loss, hallucination, and


• Can be synthesized from Tryptophan depressive psychosis
• 60 mg of Tryptophan yield 1 mg of Niacin. • Diarhea can be severe resulting in malabsorption due
Active form of niacin to atrophy of intestinal villi
• Two Coenzyme forms are NAD+(Nicotinamide • Advanced Pellagra can result in death
Adenine Dinucleotide) and NADP+(Nicotinamide • Depressive psychosis.
Adenine Dinucleotide Phosphate). 4 Ds of Pellagra
• Dermatitis (Photosensitive Dermatitis)
Coenzyme Role of Niacin
• Dementia
• Important in numerous oxidation reduction reactions.
• Diarrhea
NAD+ linked Enzymes
• Death.
m

• Lactate Dehydrogenase
• Pyruvate Dehydrogenase
co

Conditions associated with Pellagra like symptoms


• αKetoGlutarate Dehydrogenase • Hartnup Disease (Due to intestinal malabsorption and renal
• Isocitrate Dehydrogenase reabsorption of Tryptophan)
• Malate Dehydrogenase • Carcinoid Syndrome (Over production of serotonin leads to
• βHydroxy Acyl CoA Dehydrogenase diversion of Tryptophan from NAD+ pathway)
• Glycerol 3 Phosphate Dehydrogenase (cytoplasmic) • Vitamin B6 deficiency (Defective Kynureninase that lead to
• Glutamate Dehydrogenase defective synthesis of Niacin)
• Glyceraldehyde 3 phosphate Dehydrogenase. • Pellagra is common in people whose staple diet is maize and jowar.
NADP + utilizing enzymes Maize-Niacin present in unavailable form Niacytin
Mainly for Reductive BiosynthesisQ of steroids and CholesterolQ, Sorghum vulgare (Jowar)-High Leucine content inhibit QPRTase, rate
Free radical ScavengingQ, Formation of deoxyribonucleotides, limiting enzyme in Niacin synthesis.
One carbon metabolism.
• 3 Keto acyl reductase Recommended Daily Allowance of Niacin (RDA)
• Enoyl reductase
20 mg/day
m

• HMG CoA Reductase


Toxicity of niacin
co

• Folatereductase
• Glutathione Reductase • Prostaglandin mediated cutaneous flushing due to
• Ribonucleotide Reductase. binding of vitamin to a G Protein coupled receptor
Contd... • Gastric irritation
e

e
m

m
m
co

360 |  
Self Assessment and Review of Biochemistry

• Hepatic toxicity is the most serious toxic reaction Tryptophan metabolism


with sustained release niacin presents with jaundice, • Coenzyme of Kynureninase involved in the synthesis
elevated liver enzymes (AST and ALT) even fulmi- of niacin from Tryptophan
nant hepatitis • In Pyridoxine deficiency Xanthurenic acid is excreted
• Other toxic reactions include glucose intolerance, because of defective Kyneureninase in Niacin
hyperuricemia, macular edema and cysts. synthesis.
m

Treatment of cutaneous flushing Heme synthesis


co

• Laropiprant, a selective Prostaglandin D2 receptor • ALA Synthase that catalyse condensation of Succinyl
1 antagonist CoA and Glycine.
• Premedication with Aspirin. Glycogenolysis
Therapeutic uses of Niacin (Nicotinic acid) • Glycogen phosphorylase.
• Used as Lipid modifying Drug Deficiency of Vitamin B6 (Pyridoxine)
• Niacin reduces plasma triglyceride and LDL-C levels • Neurological manifestation: Due to deficiency of
and raises the plasma concentration of HDL-C. Catecholamines
• Peripheral neuropathy
Pyridoxine (Vitamin B6)
• Personality changes that include depression and
Family of 3 related Pyridine derivatives
confusion
• Pyridoxine
• Convulsions: Due to decreased synthesis of GABA
m

• Pyridoxal
• Microcytic hypochromic Anemia: Due to decreased
co

• Pyridoxamine
heme synthesis
Remember • Pellagra due defective niacin synthesis.
Some 80% of the body’s total vitamin B6 is pyridoxal phosphate in
muscleQ, mostly associated with glycogen phosphorylase. Other conditions caused by PLP deficiency.
• Oxaluria: Due to defective Alanine: Glyoxylate Amino
Active form of Pyridoxine Transferase. Glyoxylate converted to Oxalic acid
• Pyridoxal Phosphate (PLP) • Homocystinuria: Due to defective Cystathionine Beta
• Mainly used for Amino Acid metabolismQ. Synthase
Coenzyme Role of Pyridoxal Phosphate (PLP)Q • Xanthurenic Aciduria: Due to defective Kynureninase
Transamination • Cardiovascular risks: Because of homocysteinemia.
• Alanine Amino Transferase (ALT) Drugs that interact with carbonyl group and causes PLP
m

• Aspartate Amino Transferase (AST) deficiencies are L-Dopa, Pencillamine, Cycloserine.


co

• Alanine Glyoxalate Amino Transferase. Pyridoxine dependency syndromes that need pharma-
Decarboxylation of amino acids cological dose of PLP
This results in the formation of Biogenic Amines • Classic homocystinuria (due to cystathionine beta
synthase deficiency)
• Glutamate: GABA
• Sideroblastic anemia (due to ALA Synthase deficiency)
• 5-Hydroxy Tryptophan: Serotonin
• Gyrate atrophy of retina and choroid in δ- ornthine
• Histidine: Histamine
amino transferase.
• Cysteine: Taurine
High doses of Pyridoxine given in
• Serine: Ethanolamine
• Carpal Tunnel syndrome
• DOPA: Dopamine.
• Premenstrual syndrome
Transulfuration
• Schizophrenia
m

• Involved in the metabolism of Sulfur containing


• Diabetic neuropathy.
amino acids
co

Pyridoxine and Hormone dependent cancer


• Synthesis of Cysteine from methionine
• Pyridoxine is important in steroid hormone action
• Enzymes are Cystathionine Beta Synthase and
Cystathioninase. Contd...
e

e
m

m
m
co

Vitamins and Minerals   | 361

Contd... Pantothenate kinase associated neurodegeneration


• Pyridoxal phosphate removes the hormone-receptor complex from (PKAN) (formerly Hallervorden-Spatz syndrome)
DNA binding, terminating the action of the hormones • Rare autosomal recessive neurodegenerative disorder
• In vitamin B6 deficiency, there is increased sensitivity to the
actions of low concentrations of estrogens, androgens, cortisol,
• Chorea, dystonia, parkinsonian features, pyramidal
and vitamin D tract features and MR
• Increased sensitivity to steroid hormone action may be important • MRI-decreased T2 signal in the globuspallidus and
m

in the development of hormone-dependent cancer of the breast, substantianigra, ‘eye of the tiger’ sign (hyperintense
uterus, and prostate, and vitamin B6 status may affect the
co

prognosis.
area within the hypointense area)
• Sometimes acanthocytosis
Biochemical Assay of Vitamin B6 • Neuropathologic examination indicates excessive
• Erythrocyte Transaminase activity accumulation of iron-containing pigments in the
• Tryptophan load test-measurement of Xanthurenic globuspallidus and substantianigra
acid following Tryptophan load • Similar disorders are grouped as neurodegeneration
• Measurement of PLP in the blood. with brain iron accumulation (NBIA).
Toxicity of Vitamin B6 Biotin or Vitamin H or Vitamin B7
• Excess Pyridoxine may lead to Sensory Neuropathy. • Also known as anti-egg white injury factor
RDA of Pyridoxine • Endogenously synthesized by intestinal flora
• 1–2 mg/day • Reactive form is the enzyme bound CarboxyBiocytin.
m

• RDA of Pyridoxine depends on Protein intake.


co

Coenzyme role of Biotin


Play a role in gene expression, fatty acid synthesis, gluconeogenesis
Pantothenic Acid (Vitamin B5) and serve as a CO2 carrier for Carboxylases enzymes and gene
• Derived from the Greek word pantos means regulation by histone biotinylation.
everywhere Coenzyme for ATP dependent Carboxylation reaction (Carbon
Dioxide Fixation)
• Endogenously synthesized by bacterial flora in the • Pyruvate Carboxylase (Pyruvate to Oxaloacetate)
intestine • Propionyl CoA Carboxylase (Propionyl CoA to Methyl Malonyl CoA)
• Vitamin that contains Beta Alanine • Acetyl CoA Carboxylase (Acetyl CoA to Malonyl CoA)
• Methyl Crotonyl CoA Carboxylase.
• Vitamin present in Coenzyme A (CoA) and Acyl
Carrier Protein (ACP) in Fatty Acid Synthase Biotin independent Carboxylation reaction
Complex. • Carbamoyl Phosphate Synthetase –I and II
• Addition of CO2 to C6 in Purine ring (AIR Carboxylase)
The important CoA Derivatives are
m

• Malic Enzyme (Pyruvate to Malate).


• Acetyl CoA
co

• Succinyl CoA • Gamma Carboxylation (Vitamin K dependent).


• HMG CoA Biotin Antagonist
• Acyl CoA. Avidin
Pantothenic acid as a part of CoA take part in • Protein present in the raw egg white
• Fatty acid Oxidation • Eating raw egg is harmful because of Avidin present
in raw egg inhibit biotin
• Acetylation
• Affinity of Avidin to Biotin is stronger than most of
• Citric acid cycle
the Antigen antibody reaction.
• Cholesterol synthesis.
This property is used in
Deficiency of Pantothenic Acid • ELISA test
m

• Gopalan’s Burning feet Syndrome or Nutritional • Labelling of DNA.


co

Melalgia or Peripheral nerve damage.


Streptavidin
RDA of Pantothenic acid • Purified from Streptomyces avidinii
10 mg/day. • Bind 4 molecules of Biotin.
e

e
m

m
m
co

362 |  
Self Assessment and Review of Biochemistry

Deficiency of biotin Source of Formimino THF


• Mental changes (Depression, hallucination) • Histidine ---->FIGLU------->Formimino THF
paresthesia, anorexia, and nausea
Utilization of one carbon groups
• A scaling, seborrheic and erythematous rash around
nose, eyes and mouth. • Serine to Glycine
• Homocysteine to Methionine
Biochemical tests to diagnose Biotin deficiency
m

• Synthesis of Purine Nucleotides


• Decreased concentration of Urinary biotin
co

• Synthesis of TMP
• Increased urinary excretion of 3-hydroxyvaleric acid
• Synthesis of Choline.
after leucine challenge
• Decreased activity of biotin dependent enzymes in
lymphocytes.

Folic Acid or Vitamin B9


• Derived from latin word folium, which means leaf
of vegetable
• Folic Acid is abundant in leafy vegetables
• Folic Acid is absorbed from upper part of JejunumQ.
Functions of folic acid
m

• Active form of Folic acid is Tetra Hydro Folic Acid


co

(THFA)
• THFA is the carrier of One Carbon groups.
One carbon metabolism
Fig. 15.3: One carbon metabolism
One carbon units are:
• Methyl (CH3)
Pharmaceutically used THFA derivative
• Methylene (CH2) • 5-Formyl-tetrahydrofolateQ2013 is more stable than folate and is
• Methenyl (CH) therefore used pharmaceutically (known as folinic acid), and
• Formyl (CHO) the synthetic (racemic) compound (leucovorin).
• It is given orally or parenterally to overcome the toxic effects of
• Formimino (CH = NH). methotrexate or other DHF reductase inhibitors.
One carbon groups bind to THF through
Biochemical assessment of folate deficiency
m

• N5 are Formyl, Formimino or methyl


• Serum Folate (Normal level is 2–20 ng/ml)
co

• N10 are Formyl


• Both N5 and N10 are Methylene and Methenyl. • Red Cell Folate
• Histidine Load testQ or FIGLU excretion test
Sources of one carbon groups
• AICAR [Amino Imidazole Carboxamide Ribose 5
• The major point of entry of one carbon unit is
Phosphate] Excretion Test
Methylene THFQ
• Serum Homocysteine
• SerineQ is the most important source of One Carbon
• Peripheral Blood Smear (Macrocytes, tear drop cells,
units
hypersegmented neutrophils, anisopoikilocytosis).
• SerineQHydroxy Methyl TransferaseQ is the enzyme
involved in this pathway.
Important sources of one carbon groups
Source of Methylene THF
m

• Serine to Glycine by Serine Hydroxy Methyl


co

Transferase
• Glycine
• Choline. Fig. 15.4: FIGLU excretion
e

e
m

m
m
co

Vitamins and Minerals   | 363

Deficiency of folic acid Cobalamin binding proteins


• Reduced DNA Synthesis because THF derivatives • Cobalamin binding proteins in the saliva are called
are involved in purine synthesis and thymidylate Haptocorrins or Cobalophilin or R Binders
Synthesis • Intrinsic Factor of Castle from parietal cells of body
• Megaloblastic Anemia and fundus of the stomach
‒ Vitamin B12 deficiency and Folate deficiency can ‒ Vitamin B12 is freed from binding proteins
m

lead to this condition in food through the action of pepsin in the


co

‒ In Vitamin B12 deficiency Megaloblasticanemia stomach and binds to salivary proteins called
is due to folate trap cobalophilins, or R-binders
• Homocysteinemia due decreased conversion of ‒ In the duodenum, bound vitamin B12 is released
Homocysteine to Methionine. This is because Methyl by the action of pancreatic proteases. It then
THFA is the methyl donor for this reaction
associates with intrinsic factor
• Neural tube defects (like Spina bifida) during
‒ Actively absorbed from the ileumQ by binding
pregnancy
to IF receptor
• Atrophic glossitis
‒ IF receptor in the ileum is called CUBULIN.
• Depression.
Transport of Cobalamin to the target tissues
Folic acid and cancer
• Major Cobalamin transport protein in plasma is
• Low folate status results in impaired methylation
Transcobalamin II (TC II)Q
m

of CpG islands in DNA, which is a factor in the


development of colorectal and other cancers • Transcobalamin I [TC I] play a role in the transport
co

• Prophylactic Folic Acid during pregnancy reduce of Cobalamin analogues


chance of Acute Lymphoblastic Lymphoma • At the target tissues by receptor mediated endocytosis
• But, folate supplements increase the rate of involving TC II receptor.
transformation of preneoplastic colorectal polyps Causes of Vitamin B12 deficiency
into cancers
Nutritional
• Folic acid ‘feed’ tumors by increasing thymidine
• Vitamin B12 is found only in foods of animal origin,
pools and ‘better’ quality DNA
there being no plant sources of this vitamin. This
• So Folic Acid should be avoided in established
means that strict vegetarians (vegans) are at risk of
tumors.
developing B12 deficiency.
Vitamin B12 (Cobalamin)
Malabsorption-pernicious anemia
m

• Other name is Extrinsic factor of castle


• Pernicious anemia is a specific form of megaloblas-
co

• Contain 4.35% cobalt by weight


ticanemia caused by autoimmune gastritis and an
• Contain 4 pyrrole rings coordinated with a cobalt attendant failure of intrinsic factor production, which
atom, called Corrin ring.
leads to vitamin B12 deficiency.
Active forms of Vitamin B12
• Methyl Cobalamin and Adenosyl Cobalamin (Ado B12) Gastric causes
Coenzyme Role of Cobalamin • Congenital absence of intrinsic factor or functional
• Methyl Malonyl CoA Mutase abnormality
• L Methyl Malonyl CoA → Succinyl CoA Methionine Synthase or
Homocysteine Methyl Transferase • Total or partial gastrectomy.
• Homocysteine → Methionine
• Leucine Amino Mutase Intestinal causes
• Intestinal stagnant loop syndrome: jejunal
Vitamin B12 metabolism diverticulosis, ileocolic fistula, anatomic blind loop,
m

Absorption of cobalamin intestinal stricture, etc.


co

• 99% of absorption of Cobalamin are active • Ileal resection and Crohn’s disease.
• Active mechanism: Site is IleumQ
Selective malabsorption with proteinuria
• 1% passive occurs equally in Buccal cavity, Duode-
num, Ileum. • Imerslund Syndrome
e

e
m

m
m
co

364 |  
Self Assessment and Review of Biochemistry

• Imerslund-Gräsbeck Syndrome Biochemical assessment of cobalamin deficiency


• Congenital Cobalamin Malabsorption • Serum Cobalamin
• Autosomal Recessive Megaloblastic Anemia • Serum Methyl Malonate (This helps to distinguish
• Tropical sprue between Megaloblasticanemia due to Cobalamin
• Transcobalamin II deficiency. deficiency and Folate deficiency)
• Serum Homocysteine
m

Fish tapeworm
• Schilling Test using Radioactive labelled Cobalt-60
co

• The fish tapeworm (Diphyllobothriumlatum) lives


• Urine Homocystine and MMA
in the small intestine of humans and accumulates
• Bone marrow and Peripheral Blood Smear.
cobalamin from food, rendering the cobalamin
unavailable for absorption. Vitamin C (Ascorbic Acid)
Vitamin B12 deficiency and Folate trap • Other name is antiscorbutic factor
• When acting as a methyl donor, S-adenosyl methionine • Most animals synthesize Vitamin C from Glucose by
forms homocysteine, which may be remethylated by uronic Acid PathwayQ
methyl-tetrahydrofolate catalyzed by methionine • Humans and higher Primates cannot due to absence
synthase, a vitamin B12–dependent enzyme of Gulonolactone OxidaseQ.
• The reduction of methylene-tetrahydrofolate to Biochemical Functions of Ascorbic Acid
methyl-tetrahydrofolate is irreversible. This is
• Acts as a good reducing agent and a scavenger of free
m

the major source of tetrahydrofolate for tissues is


radicals (Antioxidant)
co

methyltetrahydrofolate
• In Collagen Synthesis: Vitamin C is required for the
• Impairment of methionine synthase in vitamin
post-translational modification, Hydroxylation of
B12 deficiency results in the accumulation of
lysine and Proline
methyltetrahydrofolate—the ‘folate trap’
• Hydroxylation of Tryptophan
• There is therefore functional deficiency of folate,
• Tyrosine Metabolism: Oxidation of P hydroxyl
secondary to the deficiency of vitamin B12.
Phenyl Pyruvate to Homogentisic Acid
• Bile Acid Synthesis in 7 alpha Hydroxylase
• Iron Absorption: Favor Iron absorption by conversion
of Ferric ions to Ferrous ions
• Folate Metabolism: Conversion of Folate to its active
m

form
co

• Adrenal steroid synthesis.


Fig. 15.5: Folate trap
Vitamin C Deficiency
Deficiency manifestation of Vitamin B12 Scurvy
• Megaloblasticanemia • Petechiae, ecchymosis, coiled hairs, inflamed and
• Homocysteinemia: Due decreased conversion of bleeding gums, joint effusion, poor wound healing,
Homocysteine to Methionine fatigue
• Methyl Malonic Aciduria: Due to defective Methyl • Perifollicular hemorrhages
Malonyl CoA Mutase which leads to decreased • Perifollicular hyperkeratotic papules, petechiae,
conversion of L Methyl malonyl CoA to Succinyl purpura
CoA • Splinter hemorrhage, bleeding gums, hemarthroses,
• Subacute Combined Degeneration subperiosteal hemorrhage
m

• Cobalamin deficiency may cause a bilateral peripheral • Anemia


co

neuropathy or degeneration (demyelination) of the • Late stage are characterized by edema, oliguria,
posterior and pyramidal tracts of the spinal cord. neuropathy, intracerebral hemorrhage and death.
e

e
m

m
m
co

Vitamins and Minerals   | 365

Barlows Syndrome (Infantile Scurvy) Deficiency of Vitamins


• In infants between 6-12 months, the diet if not Principal clinical findings of vitamin malnutrition
supplemented with Vitamin C then deficiency will Nutrient Clinical finding
result. Thiamin Peripheral nerve damage (beriberi) or central
nervous system lesions (Wernicke-Korsakoff
Vitamin C toxicity syndrome)
• Gastric irritation, flatulence, diarrhea,
m

Riboflavin Magenta tongue, angular stomatitis, cheilosis,


• Oxalate stones are of theoretic concern. seborrheic dermatitis
co

Niacin Pellagra: pigmented rash of sun-exposed areas


Vitamins at a Glance (photosensitive dermatitis), bright red tongue,
Vitamin deficiencies causing dementia diarrhea, apathy, memory loss, disorientation,
• Thiamin depressive psychosis
• Niacin Vitamin B6 Seborrhea, glossitis, convulsions, neuropathy,
• Cobalamin depression, confusion, microcytic anemia
Folate Megaloblasticanemia, atrophic glossitis, depres-
Sulfur Containing Vitamins
sion, ↑homocysteine
• Biotin
Vitamin B12 Pernicious anemia = megaloblasticanemia
• Thiamin with degeneration of the spinal cord, loss
Antioxidant Vitamin of vibratory and position sense, abnormal gait,
• Vitamin E dementiaQ, impotence, loss of bladder and bowel
• Vitamin C control, ↑homocysteine, ↑methylmalonic acid
m

• Beta Carotene Pantothenic Acid Peripheral nerve damage (nutritional melalgia or


‘burning foot syndrome’)
co

Antioxidant vitamins are also Pro-oxidants


Vitamin C Scurvy: petechiae, ecchymosis, coiled hairs,
• Vitamin C inflamed and bleeding gums, joint effusion, poor
• Beta Carotene wound healing, fatigue
• Vitamin E Vitamin A Xerophthalmia, night blindness, Bitot’s spots,
B complex Vitamins with Toxicity follicular hyperkeratosis, impaired embryonic
development, immune dysfunction
• Niacin
Vitamin D Rickets: skeletal deformation, rachitic rosary,
• Pyridoxine
bowed legs; osteomalacia
Redox Vitamins Vitamin E Peripheral neuropathy, spinocerebellar ataxia,
Vitamins that take part in Oxidation reduction reaction skeletal muscle atrophy, retinopathy
• Niacin and Riboflavin Vitamin K Elevated prothrombin time, bleeding
Endogenously Synthesized Vitamins
• Niacin (Vitamin B3)
m

• Biotin MINERALS
co

• Vitamin D
• Pantothenic Acid Classified into
• Vitamin K. • Macrominerals (Major elements)
‒ Daily requirement > 100 mg
Ring Structures of B-complex Vitamins ‒ Calcium, Magnesium, Phosphorus, Sodium,
Vitamin Ring structure
Potassium, Chloride, Sulfur
Vitamin B1 [Thiamine] Pyrimidine + Thiazole
• Micromineral (Trace element)
Vitamin B2 [Riboflavin] Isoalloxazine ‒ Daily requirement < 100 mg
Vitamin B3 [Niacin] Pyridine ‒ Iron, Iodine, Copper, Cobalt, Mangenese,
Vitamin B6 [Pyridoxine] Pyridine
Molybdenum, Selenium, Zinc, and Fluorine
Vitamin B12 [Cobalamin] Corrin [Tetrapyrrole with Co at its center]
• Ultra trace elements
m

Folic Acid Pteridine + PABA ‒ Daily requirement < 1 mg/day.


co

Biotin Imidazole + Thiophene


Pantothenic Acid No ring Structure Contain Pantoic Acid
IRON
and Beta AlanineQ in amide linkage
Body distribution of Iron.
e

e
m

m
m
co

366 |  
Self Assessment and Review of Biochemistry

Iron content, mg Hemosiderin


Adult male Adult female • A partly degraded form of ferritin that contains iron
Hemoglobin 2500 1700
is Hemosiderin
Myoglobin/Enzymes 500 300
• Iron is not easily mobilized from Hemosiderin unlike
ferritin
Transferrin 3 3
• It can be detected in tissues by histological stains (e.g.
m

Iron Stores 600–1000 0–300


Prussian blue), under conditions of iron overload
Total Body Iron content 3603–4003 2003–2303
co

(hemosiderosis)
• Hemosiderin is an Index of Iron OverloadQ.
Iron Containing Proteins
Transport form Transferrin
Heme ContainingQ
• Hemoglobin Transferrin and Transferrin receptors
• Myoglobin • Iron is transported in plasma in the Fe3+ form by the
• Cytochrome c transport protein, transferrin
• Cytochrome oxidase • Ferric iron combines with apo transferrin to form
• Tryptophan pyrrolase
transferrin
• Catalase
• Nitric Oxide Synthase • Synthesized in the Liver
• Transferrin is a β1 globulin
Nonheme –iron containing Proteins • Transferrin is a bilobed glycoprotein with two iron
m

• Aconitase
binding sites
co

• Transferrin
• Ferritin • Transferrin that carries iron exists in two forms—
• Hemosiderin monoferric (one iron atom) or diferric (two iron
Iron-Sulfur Complex atoms)
• Complex I of ETC • The turnover (half-clearance time) of transferrin-
• Complex II of ETC bound iron is very rapid—typically 60–90 min
• Complex III of ETC
• Normal 1/3rd transferrin saturated with Iron
• Xanthine oxidase
• The iron-transferrin complex circulates in the plasma
Proteins that has role in Iron metabolism until it interacts with specific transferrin receptors
Storage form Ferritin and Hemosiderin • On the surface of marrow erythroid cells
Ferritin • Diferric transferrin has the highest affinity for
• The human body can typically store up to 1 g of iron, transferrin receptors
m

the vast majority of which is bound to ferritin • The greatest number of transferrin receptors (300,000
co

• MW 440 kDa to 400,000/cell) is the developing erythroblast


• The Transferrin receptor 1 (TfR1) can be found on
• Ferric iron + Apoferritin = Ferritin
the surface of most cells
• Poly nuclear complex of hydrous ferric oxide
• Transferrin receptor 2 (TfR2), by contrast, is
• Ferritin is composed of 24 identical subunits, which expressed primarily on the surface of hepatocytes
surround as many as 3000 to 4500 ferric atoms and also in the crypt cells of the small intestine
• The subunits may be of the H (heavy) or the L (light) • The affinity of TfR1 for Tf-Fe is much higher than
type that of TfR2
• The H-subunit possesses ferroxidase activity, which • The major role of TfR2 is sensing iron level, rather
is required for iron-loading of ferritin than internalizing iron.
• The function of the L subunit is not clearly known Reciprocal regulation of TfR1 and Ferritin
but is proposed to play a role in ferritin nucleation
m

• The rates of synthesis of TfR1 and ferritin are reciprocally linked


and stability to intracellular iron levels
co

• Seen in Intestinal cells, Liver, Spleen and Bone marrow • When iron is low, TfR1 synthesis increases and that of ferritin
declines
• Plasma ferritin levels thus are considered to be an
indicator of body iron stores. Contd...
e

e
m

m
m
co

Vitamins and Minerals   | 367

Contd... Carbohydrate Deficient Transferrin (CDT)


• The opposite occurs when iron is abundant • Glycosylation of transferrin is impaired in congenital
• Control is exerted through the binding of iron regulatory proteins disorders of glycosylation as well as in chronic
(IRPs) called iron response elements (IREs) located in the 5’
and 3’ untranslated regions of mRNA.
• Alcoholism
Concept
• The presence of carbohydrate-deficient transferring
When iron level is low, tissue demand for iron is high, increased trans-
(CDT), which can be measured by isoelectric
m

ferrin receptors, help to internalize the available iron in the plasma. focussing (IEF)
co

Decreased ferritin will help to mobilize the maximum iron stores to • This is used as a biomarker of chronic alcoholism
meet the demand of iron. and Congenital Disorders of Glycosylation (CDGs)
m
co

Fig. 15.6: Metabolism of iron


m

Iron Metabolism
co

• Site of absorption: Enterocytes in the proximal


duodenum
• Heme iron is absorbed by a heme transporter
• Iron is absorbed in the ferrous formQ
• Inorganic dietary iron in the ferric state (Fe 3+)
is reduced to its ferrous form (Fe 2+) by a brush
border membrane-bound ferrireductase, duodenal
cytochrome b (Dcytb)
• Vitamin C in food also favors reduction of ferric iron
to ferrous iron
• The transfer of iron from the apical surfaces of
m

enterocytes into their interiors is performed by a


co

proton- coupled divalent metal transporter (DMT1)


• This protein is not specific for iron, as it can transport
a wide variety of divalent cations. (Co2+,Zn2+,Pb2+,Cu2+) Fig. 15.7: Absorption of Iron
e

e
m

m
m
co

368 |  
Self Assessment and Review of Biochemistry

• Once inside the enterocytes, iron can either be stored (TfR-1, TfR-2) and transmembrane protein HFE
as ferritin or transferred across the basolateral protein
membrane into the circulation by the iron exporter • TfR-1 binds to iron bound transferring (Tf-Fe) at the
protein, ferroportin or iron-regulated protein 1 site where it binds to HFE protein
(IREG1 or SLC40A1) • When iron is abundant, Tf-Fe are high, hence HFE is
• This protein may interact with the copper-containing displaced from TfR-1
m

protein hephaestin, a protein similar to ceruloplasmin • The displaced HFE binds to TfR-2
co

• Hephaestin is thought to have a ferroxidase activity, • Binding of HFE to TfR-2 triggers intracellular signal
which is important in the release of iron from cells cascade (ERK-MAPK cascade)
• Thus, Fe2+ is converted back to Fe3+, the form in which • Which activate expression of HAMP gene that codes
it is transported in the plasma by transferrin. for Hepcidin.
Concept is increased level of iron→increased expression of
Dietary Regulation of Iron by Mucosal Block at hepcidin→which inturn decreases circulating iron.
the Level of Enterocyte
Hepcidin Bone Morphogenic Proteins (BMPs) and Hemojuvelin
• Hepcidin is the Chief Regulator of Systemic Iron (HJV)
Homeostasis • BMP binds to a cell-surface receptor (BMPR) whose
• It is a 25-amino acid peptide binding affinity is augmented by binding to a co-
m

• Synthesized in the liver as an 84-amino acid precursor receptor, hemojuvelin (HJV)


co

(prohepcidin). • The activation of the BMPR-HJV complex triggers


the phosphorylation of intracellular signaling
Mechanism of Iron regulation by hepcidin proteins called SMADs, which subsequently results
• Hepcidin binds to the cellular iron exporter, in transcriptional activation of hepcidin.
ferroportin, triggering its internalization and
degradation Erythropoietic signals
• The consequent decrease in ferroportin results • Two molecules secreted by erythroblasts, growth
in decreased export of iron into circulation and differentiation factor 15 (GDF15) and twisted
gastrulation 1 (TWSG1)
depressed iron recycling by macrophages
• They inhibit expression of hepcidin in β-thalassemia
• Together, these result in a reduction in circulating
major.
iron levels (hypoferremia) as well as reduced
placental iron transfer during pregnancy Inflammation
m

• When plasma iron levels are high, hepatic synthesis • Hepcidin synthesis is induced by cytokines such
co

of hepcidin increases, thus reducing circulating iron as interleukin–6 (IL-6) that are released as part of an
level inflammatory response
• The opposite occurs when plasma iron levels are low. • Binding of IL-6 to its cell surface receptor stimulates
gene expression by activating the JAK-STAT (Janus
Regulation of expression of hepcidin
Kinase—Signal Transducer and Activator of
The hepcidin level is influenzed by Transcription) Pathway
• Circulatory level of iron • Anemia that is associated with chronic inflammation
• Bone Morphogenic Proteins (BMPs) and Hemojuvelin (anemia of inflammation or AI) is probably due to
• Erythropoietic signals inflammation-mediated upregulation of hepcidin.
• Inflammation
Hypoxia
• Hypoxia
• Hypoxia is suppress hepcidin expression
m

Circulation level of Iron • This effect is mediated by erythropoietin, whose


co

• Liver cells monitor iron levels using an iron sensing synthesis is controlled by hypoxia-inducible
complex comprised of two transmembrane receptors transcription factors 1 and 2 (HIF-1 and HIF-2).
e

e
m

m
m
co

Vitamins and Minerals   | 369


m
co
m
co

Fig. 15.8: Regulation of Expression of Hepcidin

Conservation of Iron Contd...


• Extracorpuscular hemoglobin is bound by Haptoglobin level in hemolytic anemia
haptoglobin • Patients suffering from hemolytic anemias exhibit low levels of
haptoglobin
• Hemopexin is a β1 globin that binds Heme • The half-life of haptoglobin is approximately 5 days
• Albumin will bind some metheme (ferric heme) to • The Hb-Hp complex is removed rapidly by the hepatocytes (half-
form methemalbumin life 90 minutes)
• Which then transfers the metheme to hemopexin • Thus, when haptoglobin is bound to hemoglobin, it is cleared from
the plasma about 80 times faster than normally
• Transferrin bind free Iron (Fe 3+) in plasma. • So the level of haptoglobin falls rapidly in situations where
Haptoglobin hemoglobin is constantly being released from red blood cells, such
m

• Human haptoglobin exists in three polymorphic forms, known as occurs in hemolytic anemias.
co

as Hp 1-1, Hp 2-1, and Hp 2-2 Haptoglobin-related protein and cancer


• Haptoglobin is an acute phase protein, and its plasma level is A plasma protein that has a high degree of homology to haptoglobin,
elevated in a variety of inflammatory states it is elevated in some patients with cancers, although the significance
• Haptoglobin scavenges hemoglobin that has escaped recycling. of this is not understood.
Haptoglobin protects the kidneys from damage by extracorpus-
cular hemoglobin Iron Deficiency Anemia
• During the course of red blood cell turnover, approximately 10% Stages of iron deficiency
of an erythrocytes hemoglobin is released into the circulation.
• The progression to iron deficiency can be divided
• This free, extracorpuscular hemoglobin is sufficiently small at =65
kDa to pass through the glomerulus of the kidney into the tubules, into three stages
where it tends to form damaging precipitates. • The first stage is negative iron balance, in which the
• Haptoglobin (Hp) is a plasma glycoprotein that binds extra- demands for (or losses of) iron exceed the body’s
corpuscular hemoglobin (Hb) to form a tight noncovalent complex ability to absorb iron from the diet. Serum iron is
(Hb-Hp).
normal and hemoglobin synthesis is unaffected
m

• Since the Hb-Hp complex is too large (≥155 kDa) to pass through
• The second stage is iron-deficient erythropoiesis,
co

the glomerulus, this protects the kidney from the formation of


harmful precipitates and reduces the loss of the iron associated transferrin saturation falls to 15–20%, Serum iron
with extracorpuscular hemoglobin. level begin to fall, hemoglobin synthesis becomes
Contd... impaired
e

e
m

m
m
co

370 |  
Self Assessment and Review of Biochemistry

• The third stage is Iron deficiency anemia, where Lab Parameters that increase in Iron Deficiency Anemia
hemoglobin and hematocrit falls. Microcytic • TIBC
Hypochromic anemia sets in. • RBC Protoporphyrin
Iron • s TR[TRP](Transferrin Receptor Protein)
Negative deficient Iron de- • RBC Distribution Width[RDW]
iron erythro- ficiency
m

Normal balance poiesis anemia Diagnosing Microcytic anemia


co

Iron stores Sidero-


Iron defi- Inflamma- Thalas- blastic
Erythron Tests ciency tion semia anemia
iron Peripheral Microcytic Normal/ Microcytic Variable
Marrow iron 1–3 + 0–1 + 0 0 Smear Hypochro- Micro/ Hypochro-
stores mic Hypo mic with
targeting
Serum 50–200 < 20 < 15 < 15
S Iron (μg/ < 30 < 50 Normal to Normal
ferritin (µg/L)
dL) high to high
TIBC (µg/dL) 300–360 > 360 > 380 > 400
TIBC(μg/dL) > 360 < 300 Normal Normal
SI (µg/dL) 50–150 NL < 50 < 30
Transferrin < 10 10–20 30–80 30–80
Saturation 30–50 NL < 20 < 10 satura-
(%) tion(%)
m

Marrow 40–60 NL < 10 < 10 Ferritin(μg/L) < 15 30–200 50–300 50–300


sideroblasts
co

(%) Hb electro- Normal Normal Abnormal Normal


phoresis pattern in
Protopor- 30–50 NL > 100 > 200 pattern beta Thal-
phyrin (µg/ assemia
dL)
RBC NL NL NL Microcytic/ Iron Overload Conditions
morphology hypochronic
TYPE-I Hereditary Hemochromatosis (HFE related)
Laboratory iron studies in normal and different stages • Mutation in HFE gene located on Chr 6p
of evolution of iron deficiency • Tightly linked to the HLA-A locus
• Most Common Hemochromatosis [80–90%]
Negative Iron defi- Iron
Non HFE related Hereditary Hemochromatosis
iron cient eryth- deficiency
Parameter Normal balance ropoiesis anemia • Juvenile hemochromatosis (type 2A) (hemojuvelin mutations)
• Juvenile hemochromatosis (type 2B) (hepcidin mutation)
m

Marrow Iron 1–3+ 0–1 + 0 0


• Mutated transferrin receptor 2 TFR2 (type 3)
stores
co

• Mutated ferroportin 1 gene, SLC11A3 (type 4)


Serum ferritin 50–200 Decreased Decreased Decreased
(μg/dL) < 20 < 15 < 15 Secondary Hemochromatosis
Total iron bind- 300– Slightly Increased > Increased • Anemia characterized by ineffective erythropoiesis (eg,
ing capacity 360 increased 380 > 400 thalassemia major)
(TIBC) (μg/dL) > 360 • Repeated blood transfusions
• Parenteral iron therapy
Serum iron (μg/ 50–150 Normal Decreased Decreased
dL) < 50 < 30 • Dietary iron overload (Bantu siderosis)
Transferrin 30–50 Normal Decreased Decreased Miscellaneous Conditions Associated with Iron Overload
saturation (%) < 20 < 10 • Alcoholic liver disease
RBC protopor- 30–50 Normal Increased Increased • Nonalcoholic steatohepatitis
phyrin (μg/dL) • Hepatitis C infection.
Soluble trans- 4–9 Increased Increased Increased
m

ferrin receptor
(μg/L) Hemochromatosis
co

RBC morphol- Normal Normal Normal Microcytic Inherited disorder of iron metabolism that lead to iron
ogy Hypochro- overload, leading to deposition of iron in the parenchymal
mic cells leading to fibrosis and organ failure.
e

e
m

m
m
co

Vitamins and Minerals   | 371

Hemosiderosis Diagnosis of Wilson’s Disease


• Acquired condition • 99% of cases Kayser-Fleischer ring is present, but
• Presence of stainable iron in tissues absence of KF ring does not excludes the disease
Hemochromatosis at a glance • Serum Ceruloplasmin [18–35 mg/dL] decreased
• The first organ to be affected in Hemochromatosis Liver • But normal in 10% of affected individuals and
• Maximum deposition of Hemosiderin is seen in Liver
decreased in 20% of carriers
m

• Least Hemosiderin deposition is seen in Skin


• 24 hour Urinary Copper > 100 µg/24 hr
co

Classical Triad of Hemochromatosis is


• Cirrhosis with Hepatomegaly • Gold Standard investigationQ is Liver BiopsyQ with
• Skin Pigmentation [Bronzing] quantitative Copper assays (> 200 µg/g dry weight
Due to the epidermis of the skin is thin, and melanin is increased in of Liver)
the cells of the basal layer and dermis
• Diabetes Mellitus Normal
Test Usefulness Wilson disease
• First joint to be affected in hemochromatosis-2nd and 3rd MCP joint Value
• Most common cause of death in treated patients-Hepatocellular Serum Ceru- + 180–350 Low in 90%
Carcinoma loplasmin mg/L
• Role of HFE Mutations in other diseases
(18–35
– Nonalcoholic Steatohepatitis mg/dL)
– Porphyria CutaneaTarda
Kayser- ++ Absent Present in > 99% if
Fleischer neurologic or psychiat-
COPPER
m

ring ric symptoms are pres-


ent. Present 30–50%
co

Cofactor role of Copper in hepatic presentation


• Amine oxidases and presymptomatic
• Ferroxidase (ceruloplasmin) (Iron metabolism, Copper Transport) patients
• Cytochrome-c oxidase (in Complex IV of Electron Transport Chain) Urine Copper +++ 0.3–0.8 > 100 μg in symptom-
• Superoxide dismutase (Free Radical Scavenging enzyme) (24h) μmol (20– atic patients
• Tyrosinase (Melanin Synthesis) 50 μg) 60–100 μg in pres-
• Component of ferroportin (Iron Metabolism) ymptomatic
• Lysyl Oxidase (Cross linking in Collagen)
Liver Copper ++++ 0.3–0.8 > 3.1 μmol (> 200 μg)
μmol/g
Copper Deficiency Anemia is a microcytic hypochromic type
(20–50
of Anemia.
μg/g of
tissue)
Wilson’s Disease
m

Autosomal recessive
Biochemical defect Treatment
co

• ATP7 B mutation, a gene encoding for Copper Disease status First line Second line
transporting ATPase in the cells Hepatitis or Cirrho- Zinc Trientene
• Defective Biliary Copper Excretion from liver cells sis without decom-
• Defective Copper incorporation into Apoceruloplas- pensation
min Hepatic decompensation
• Copper accumulate in cells leading to copper deposits Mild Trientene and Zinc Penicillamine and
in the liver and brain. Zinc

Quick glance: Wilson’s disease Moderate Trientene and Zinc Hepatic Transplan-
• The most common presentation in Wilson’s disease: Acute or tation
Chronic Liver Disease Severe Hepatic Transplanta- Trientene and Zinc
• Neuropsychiatric manifestation in Wilson’s resembles: Parkinson’s tion
m

Disease like Syndrome


Initial neurologic/ Tetrathiomolybdate Zinc
co

• Most Sensitive test in Wilson’s disease is or gold standard


psychiatric and Zinc
investigation is liver biopsy quantitative copper assay
• False positive is liver biopsy quantitative copper assay in Maintenance / Pre- Zinc Trientene
obstructive liver disease symptomatic/ Preg-
• The most specific Screening Test: Urinary Excretion of Copper nant/ Pediatric
e

e
m

m
m
co

372 |  
Self Assessment and Review of Biochemistry

Method to assess severity of Hepatic Decompensation • The diagnosis of zinc deficiency is usually made by
in Wilson’s disease a serum zinc level < 12 mol/L (< 70 g/dL).
Nazer’s Prognostic Index Zn Toxicity
• Serum Bilirubin • Acute zinc toxicity after oral ingestion causes nausea,
• Serum Aspartate Transferase [AST] vomiting, and fever
• Prolongation of Prothrombin Time • Zinc fumes from welding may also be toxic and
m

‒ Score < 7 Medical management cause fever, respiratory distress, excessive salivation,
co

‒ Score > 9 Liver Transplantation sweating, and headache.

Menke’s (Kinky or Steely) Hair Syndrome SELENIUM


• Mutation in ATP7A gene
• Selenium, in the form of selenocysteine, is a
• X linked recessive condition
component of the all the enzymes that contain
• Defective Copper binding P-type ATPase Selenocysteine
• Copper is not mobilized from Intestine • Selenium is being actively studied as a chemo-
MEDNIK Syndrome preventive agent against certain cancers, such as
• A rare multisystem disorder of copper metabolism with features of prostate cancer.
both Wilson’s and Menke’s disease
• Caused by mutation in AP1S1 gene, which encodes an adaptor Keshan disease
protein necessary for intracellular trafficking of ATP7 A and ATP7B. An endemic cardiomyopathy found in children and young women
m

• MEDNIK stands for Mental retardation, Enteropathy, Deafness, residing in regions of China where dietary intake of selenium is low
co

Neuropathy, Icthyosis, Keratodermia (< 20 g/d).


Selenium toxicity (Kashinbeck Disease)
Chronic ingestion of high amounts of selenium leads to selenosis
ZINC (Kashinbeck Disease)
It is characterized by hair and nail brittleness and loss, garlic breath
• Zinc is an integral component of many metalloenzymes
odor (Due to Dimethyl selenide), skin rash, myopathy, irritability, and
in the body other abnormalities of the nervous system.
• It is involved in the synthesis and stabilization of
proteins, DNA, and RNA and plays a structural role CHROMIUM
in ribosomes and membranes
• Zinc is necessary for the binding of steroid hormone • Chromium potentiates the action of insulin in patients
receptors and several other transcription factors to with impaired glucose tolerance, by increasing
DNA insulin receptor–mediated signalling.
m

• Zinc is absolutely required for normal spermatogenesis, Chromium -6


co

fetal growth, and embryonic development. • Chromium in the trivalent state is found in supplements and is
largely nontoxic
Zn Deficiency • Chromium-6 is a product of stainless steel welding and is a known
pulmonary carcinogen as well as a cause of liver, kidney, and
• Mild chronic zinc deficiency can cause stunted growth CNS damage.
in children, decreased taste sensation (hypogeusia),
impaired immune function
FLUORIDE
• S e ve r e c h r o n i c z i n c d e f i c i e n c y c a n c a u s e
hypogonadism, dwarfism, hypopigmented hair. • An essential function for fluoride in humans has
not been described, although it is useful for the
Acrodermatitis enteropathica
maintenance of structure in teeth and bone
• Rare autosomal recessive disorder characterized by
• Adult fluorosis results in mottled and pitted defects
abnormalities in zinc absorption
in tooth enamel as well as brittle bone (skeletal
• Clinical manifestations include diarrhea, alopecia,
m

fluorosis).
muscle wasting, depression, irritability, and a rash
co

involving the extremities, face, and perineum Minerals at a Glance


• The rash is characterized by vesicular and pustular • Zinc containing protein present in the Saliva: GustenQ
crusting with scaling and erythema • Mineral stabilize hormone insulinQ: Zinc
e

e
m

m
m
co

Vitamins and Minerals   | 373

• Mineral that potentiates action of Insulin: ChromiumQ Contd...


• Mineral deficiency that leads to impaired Glucose Mineral RDA
tolerance: ChromiumQ ZincQ 8–10 mg
• Highest concentration of Zn seen in Hippocampus SeleniumQ 50–200 μg
and Prostatic Secretion
• The mineral deficiency leads to impaired Spermato- Other important Minerals: Functions and Deficiency
m

genesis: Zinc manifestation


co

• Garlicky odor in breath is seen in: Selenosis (Due to Mineral Function Deficiency
Dimethyl selenide)
Cobalt Constituent of Vitamin Macrocytic Anemia
• Selenium toxicity lead to Kaschinbeck Disease B12
• Low Selenium level leads to Keshan disease (Endemic Chromium Potentiate the action Impaired Glucose
Cardiomyopathy) of Insulin Tolerance
• Calcium dependent Cysteine Protease are called Fluoride Constituent of Bone and Dental caries
Calpain teeth
• Calpain associated with Type II Diabetes Mellitus: Iodine Thyroid Hormone Thyroid enlargement,
Calpain 10 Synthesis ↓T4, cretinism

• Normal Blood Calcium level-9: 11 mg/dl Molybdenum Cofactor for Xan- Severe neurologic
thine oxidase and abnormalities,
• Total Calcium level in the bodyQ is 1.5 kg. Sulfite oxidase, Xanthinuria
Aldehyde oxidase
m

Recommended Daily Allowances (RDA) of important


Selenium Cofactor for Glutathione Keshan’s Disease
co

Minerals
Peroxidase Deiodinase, (Cardiomyopathy),
Mineral RDA Thioredoxin Reductase heart failure, striated
Antioxidant along with muscle degeneration
CalciumQ Adult-0.5g Children-1g
Vitamin E
Pregnancy and Lactation-1.5g
Zinc Cofactor for Carbonic Growth retardation,
IronQ Males-15–20 mg
Anhydrase Carboxy ↓taste and smell, alope-
Females-20–25 mg Peptidase cia, dermatitis, diarrhea,
Pregnancy-40–50 mg Lactate immune dysfunction,
IodineQ 150–200 μg Dehydrogenase failure to thrive, gonad-
200–250 μg Alcohol al atrophy, congenital
Dehydrogenase malformation Impaired
Phosphorus 500 mg Alkaline Phosphatase wound healing
Magnesium 400 mg Mangenese Cofactor for Arginase, Impaired growth and
m

Mangenese 5–6 mg Carboxylase, Kinase, skeletal development,


Enolase, Glucosyl Tran- reproduction, lipid and
co

Sodium 5–10 g
ferase, carbohydrate metabo-
Potassium 3–4 g PhosphoGlucoMutase lism; upper body rash
Copper 1.5–3 mg Required for RNA Poly-
merase
Contd...

REVIEW QUESTIONS
Fat Soluble Vitamins 2. All are true about vitamin D metabolism, except:
1. In the crystalline lens, level of tocopherol and (AIIMS Nov 2011)
Ascorbate is maintained by: (AIIMS May 2014) a. 1-alpha hydroxylation occurs in kidney
a. Glutathione b. 25-alpha hydroxylation occurs in Liver
m

b. Glyoprotein c. In absence of sun light, the daily requirement is


co

c. Fatty Acid 400–600 IU per day


d. Glucose d. Williams syndrome is associated with mental
Ans. a. Glutathione (Ref: Harper 30/e p546) retardation, precocious puberty and obesity
e

e
m

m
m
co

374 |  
Self Assessment and Review of Biochemistry

Ans. d. Williams Syndrome is associated with mental Vitamin K is required for the post-translational
        retardation, precocious puberty and obesity carboxylation of glutamic acid (Gamma Carboxylation),
which is necessary for calcium binding to γ carboxylated
3. Vitamin K is required for: (AIIMS Nov 2007) proteins.
a. Hydroxylation
9. All the following have antioxidant action except:
b. Chelation (Kerala 2011)
c. Transamination
m

a. Vitamin A
d. Carboxylation
co

b. Vitamin E
Ans. d. Carboxylation (Ref: Harper 30/e p554) c. Selenium
4. Vitamin A intoxication cause injury to: d. Vitamin D
a. Lysosomes (AIIMS Nov 2006) Ans. d. Vitamin D
b. Mitochondria • Adequate Selenium intake maximize the antioxidant
action of Glutathione Peroxidase
c. Endoplasmic reticulum
• So, Selenium is also considered as a compound
d. Microtubules
having antioxidant action
Ans. a. Lysosomes • Nowadays Vitamin D is also considered as powerful
5. Active form of Vitamin D is: (AIIMS Nov 2006) natural membrane antioxidant
a. Cholecalciferol • But as it is an old question, Vitamin D is the best
m

answer.
b. 24,25(OH)2vit-D
co

c. 1,25(OH)2vit-D 10. Which of the following is true about vitamin K?


d. 25-OH vit-D (NBE pattern Q)
Ans. c. 1, 25 (OH)2Vit D a. Vit K dependent factors undergo post-tran-
scriptional modification
This is otherwise called calcitriol
b. Prothrombin is a vitamin K dependent factor
6. Which of these has antioxidant properties? c. Stuart-Prower factor is not vitamin K
a. Tocopherol (PGI Nov 2012) dependent
b. Reduced Glutathione d. Menadione is a natural water insoluble vitamin
c. Citrulline K used in clinical practice
d. Lycopene Ans. b. Prothrombin is a vitamin K dependent factor
Ans. a, b, d. • Vitamin K helps Post-translational modification
m

• Stuart prower factor is Factor X, which is not Vitamin


7. Vitamin K is involved in the post-translational K dependent
co

modification of: • Prothrombin is factor II, so it is Vitamin K dependent


(AIIMS Nov 08, May 01, AIPGME 2011) • Menadione is Synthetic Vitamin K.
a. Glutamate
11. Vitamin E deficiency causes all except:
b. Aspartate a. Ataxia (NBE pattern Q)
c. Lysine b. Areflexia
d. Proline c. Ophthalmoplegia
Ans. a. Glutamate (Ref: Harper 30/e p554) d. Neuropathy
Ans. c. Ophthalmoplegia
8. Which Vitamin is required for carboxylation of
clotting factors? Principal Clinical Findings of Vitamin Malnutrition
a. Vitamin A
m

Nutrient Clinical finding


b. Vitamin D
co

Thiamin Peripheral nerve damage (beriberi) or central


c. Vitamin E nervous system lesions (Wernicke-Korsakoff
syndrome)
d. Vitamin K
Ans. d. Vitamin K (Ref: Harper 30/e p554) Contd...
e

e
m

m
m
co

Vitamins and Minerals   | 375

Nutrient Clinical finding 2. Chain-breaking antioxidants, which interfere with


Riboflavin Magenta tongue, angular stomatitis, cheilosis,
chain propagation. They are Superoxide Dismutase,
seborrheic dermatitis Uric Acid, Vitamin E (Most powerful).
Niacin Pellagra: pigmented rash of sun-exposed areas
(photosensitive dermatitis), bright red tongue,
Water Soluble Vitamins
diarrhea, apathy, memory loss, disorientation, 14. Biotin act as a coenzyme for all except:
depressive psychosis
(AIIMS Nov 2015)
m

Vitamin B6 Seborrhea, glossitis, convulsions, neuropathy, a. Pyruvate to Oxaloacetate


co

depression, confusion, microcytic anemia


b. Acetyl CoA to Malonyl CoA
Folate Megaloblasticanemia, atrophic glossitis, depres-
sion, ↑homocysteine c. Propionyl CoA to Methyl Malonyl CoA
d. Glutamate to Gamma Carboxy Glutamate
Vitamin B12 Pernicious anemia = megaloblasticanemia with
degeneration of the spinal cord, loss of vibratory Ans. d. Glutamate to Gamma Carboxy Glutamate
and position sense, abnormal gait, dementiaQ, (Ref: Harper 30/e p561)
impotence, loss of bladder and bowel control,
↑homocysteine, ↑methylmalonic acid
Biotin transfer CO2 in
• Acetyl CoA Carboxylase
Pantothenic Peripheral nerve damage (nutritional melalgia or
Acid ‘burning foot syndrome’) • Pyruvate Carboxylase
Vitamin C Scurvy: petechiae, ecchymosis, coiled hairs, • Propionyl CoA carboxylase
inflamed and bleeding gums, joint effusion, poor • Methyl Crotonyl CoA carboxylase.
wound healing, fatigue
m

15. Vitamin B12 is not required for: (AIIMS Nov 2015)


Vitamin A Xerophthalmia, night blindness, Bitot’s spots, fol-
co

licular hyperkeratosis, impaired embryonic develop- a. Glycogen Phosphorylase


ment, immune dysfunction b. Methionine Synthase
Vitamin D Rickets: skeletal deformation, rachitic rosary, bowed c. Methyl malonyl CoA Mutase
legs; osteomalacia d. Leucine Amino Mutase
Vitamin E Peripheral neuropathy, spinocerebellar ataxia, Ans. a. Glycogen Phosphorylase (Ref: Harper 30/e p558)
skeletal muscle atrophy, retinopathy
Coenzyme Role of Cobalamin
Vitamin K Elevated prothrombin time, bleeding
          Methyl Malonyl
CoA Mutase
12. Which coenzyme act as reducing agent in anabolic • L Methyl Succinyl CoA
reaction? (NBE pattern Q) Malonyl CoA
a. FADH2 Methionine Synthase or
b. FMNH2 Homocysteine Methyl
m

          Transferase
c. NADPH
co

• Homocysteine Methionine
d. NADH
• Leucine Amino Mutase.
Ans. c. NADPH
NADPH is used in reductive biosynthesis of Fatty acids, 16. A vitamin derived from amino acid is:
Steroids, etc. So it is used for anabolic reactions. (JIPMER Nov 2015)
a. Biotin
13. Most powerful chain breaking antioxidant:
b. Pantothenic acid
(NBE pattern Q)
c. Niacin
a. Glutathione peroxidase
d. Folic acid
b. Alpha tocopherol
Ans. c. Niacin (Ref: Harper 30/e p556)
c. Superoxide dismutase
Niacin is strictly not a vitamin as it can be synthesized
d. Vitamin C
from Tryptophan.
m

Ans. b. Alpha Tocopherol


co

Antioxidants fall into two classes: 17. Vitamin for which RDA is based on protein
1. Preventive antioxidants, which reduce the rate of intake is: (JIPMER Nov 2015)
chain initiation. They are Glutathione Peroxidase, a. Niacin
Catalase b. Riboflavin
e

e
m

m
m
co

376 |  
Self Assessment and Review of Biochemistry

c. Pyridoxine Sources of NADPH are:


d. Thiamine • HMP Shunt Pathway (Major source)
Ans. c. Pyridoxine • Cytoplasmic Isocitrate Dehydrogenase
• The RDA of Pyridoxine is dependent on Protein • Malic Enzyme (NADP Malate Dehydrogenase)
intake
22. Vitamin deficiency causing circum corneal
• The RDA of Thiamine is dependent of Carbohydrate
vascularization is: (AIIMS 2014 May)
m

intake.
a. Biotin
co

18. Megaloblastic anemia seen in: b. Riboflavin


(JIPMER Nov 2015) c. Thiamine
a. Ornithine Transcarbamoylase defect d. Vitamin D
b. MSUD Ans. b. Riboflavin
c. Citrullinemia
Principal clinical findings of vitamin malnutrition
d. Orotic aciduria
Nutrient Clinical finding
Ans. d. Orotic aciduria
Thiamin Peripheral nerve damage (beriberi) or central nervous
system lesions (Wernicke-Korsakoff syndrome)
19. In one carbon metabolism when Serine converted
to Glycine, which carbon atom is added to THFA: Riboflavin Magenta tongue, angular stomatitis, cheilosis, sebor-
rheic dermatitis, circumcorneal Vascularization
(NBE pattern Q)
m

Niacin Pellagra: pigmented rash of sun-exposed areas


a. Alpha Carbon (photosensitive dermatitis), bright red tongue, diar-
co

b. Beta Carbon rhea, apathy, memory loss, disorientation, depressive


c. Delta Carbon psychosis

d. Gamma Carbon Vitamin B6 Seborrhea, glossitis, convulsions, neuropathy, depres-


sion, confusion, microcytic anemia
Ans. b. Beta Carbon
Folate Megaloblasticanemia, atrophic glossitis, depression,
homocysteine
Vitamin B12 Pernicious anemia = megaloblasticanemia with de-
generation of the spinal cord, loss of vibratory and
position sense, abnormal gait, dementiaQ, impotence,
loss of bladder and bowel control, homocysteine,
methylmalonic acid
Pantothenic Peripheral nerve damage (nutritional melalgia or
Acid ‘burning foot syndrome’)
m

Vitamin C Scurvy: petechiae, ecchymosis, coiled hairs, inflamed


co

and bleeding gums, joint effusion, poor wound heal-


20. Vitamin deficiency that cause oro oculogenital
ing, fatigue
syndrome: (APPG 2012)
Vitamin A Xerophthalmia, night blindness, Bitot’s spots, follicular
a. Vitamin B2 hyperkeratosis, impaired embryonic development,
b. Vitamin B12 immune dysfunction
c. Zinc Vitamin D Rickets: skeletal deformation, rachitic rosary, bowed
d. Vitamin B3 legs; osteomalacia

Ans. a. Vitamin B2 Vitamin E Peripheral neuropathy, spinocerebellar ataxia, skeletal


muscle atrophy, retinopathy
21. NADPH is produced by: (PGI May 2014) Vitamin K Elevated prothrombin time, bleeding
a. Pyruvate Dehydrogenase
b. Isocitrate Dehydrogenase 23. False about folic acid: (AIIMS Nov 2013)
c. Succinate Dehydrogenase
m

a. It is present in all the green leafy vegetables


d. Malate Dehydrogenase
co

b. It is proven to decrease the occurrence of neural


e. αKetoGlutarate Dehydrogenase tube defects when taken preconceptionally
Ans. b. Isocitrate Dehydrogenase, c. Wheat flour in India is fortified with folate as
     d. Malate Dehydrogenase in USA
e

e
m

m
m
co

Vitamins and Minerals   | 377

d. Methyl folate trap is because of methionine 28. Which of the following cannot be synthesized in
synthase defect the body? (PGI Nov 2012)
Ans. c. Wheat flour in India is fortified with folate as in a. Vit K
USA b. Vit C
c. Thiamine
24. Which of the vitamin deficiency lead to lactic d. Riboflavin
acidosis?
m

e. Cyanocobalmin
a. Riboflavin
co

Ans. b, c, d, e.
b. Thiamine
c. Niacin 29. Thiamine deficiency causes decreased energy
d. Pantothenic acid production because: (AIPGMEE 2010)
a. It is required for the process of transamination
Ans. b. Thiamine
b. It is a cofactor in oxidative reduction
Thiamine deficiency affect Pyruvate Dehydrogenase, so
c. It is a coenzyme for transketolase in pentose
it causes Lactic acidosis.
phosphate pathway
25. Thiamin requirement increases in excessive d. It is a coenzyme for pyruvate dehydrogenase
intake of: and alpha ketoglutarate dehydrogenase
a. Carbohydrate Ans. d. It is a coenzyme for pyruvate dehydrogenase and
b. Amino acid alpha ketoglutarate dehydrogenase
m

c. Fat (Ref: Harper 30/e p555)


co

d. Lecithin 30. Vitamin B12 acts as coenzyme to which one of the


Ans. a. Carbohydrate following enzymes?
• Thiamin requirement increases in carbohydrate a. Isocitrate dehydrogenase
intake. b. Homocysteine methyl transferase
• Pyridoxine (B6) requirement increases in protein c. Glycogen synthase
intake. d. Glucose-6- Phosphate dehydrogenase
Ans. b. Homocysteine Methyl Transferase
26. Which of the following statement about Thiamine
true? (AIIMS Nov 2008) 31. Biotin is a cofactor of:
a. It is a coenzyme of lactate dehydrogenase a. Carboxylase
b. Its deficiency is associated with scurvy b. Oxidase
m

c. Its coenzyme function is done by thiamine c. Hydrolase


co

monophosphate d. Decarboxylase
d. It is coenzyme for pyruvate dehydrogenase Ans. a. Carboxylase (Ref: Harper 30/e p556)
and a-ketoglutarate dehydrogenase
Coenzyme role of Biotin
Ans. d. Play a role in gene expression, fatty acid synthesis, gluconeogenesis
It is coenzyme for pyruvate dehydrogenase and and serve as a CO2 carrier for Carboxylases enzymes and gene
a-ketoglutarate dehydrogenase regulation by histone biotinylation
Coenzyme for ATP dependent Carboxylation reaction (Carbon
Dioxide Fixation)
27. Vitamin which is excreted in urine is:
• Pyruvate Carboxylase (Pyruvate to Oxaloacetate)
(AIIMS Nov 2006)
• Propionyl CoA Carboxylase (Propionyl CoA to Methyl Malonyl CoA)
a. Vitamin A • Acetyl CoA Carboxylase (Acetyl CoA to Malonyl CoA)
b. Vitamin C • Methyl Crotonyl CoA Carboxylase
m

c. Vitamin D
co

Biotin independent Carboxylation reaction


d. Vitamin K • Carbamoyl Phosphate Synthetase –I & II
Ans. b. Vitamin C • Addition of CO2 to C6 in Purine ring (AIR Carboxylase)
• Malic Enzyme (Pyruvate to Malate)
Water soluble vitamins are excreted in urine.
e

e
m

m
m
co

378 |  
Self Assessment and Review of Biochemistry

32. Post-translation modification of hydroxylysine


and hydroxyproline is by: (Kerala 2009)
a. Vit C
b. Vit K
c. Vit E
d. Vit D
m

Ans. a. Vit C
co

Apart from Vitamin C, Alpha Ketoglutarate is also


acting as a coenzyme in Prolyl and Lysyl Hydroxylase
reaction.

33. Pantothenic acid containing coenzyme is involved


in: (NBE pattern Q)
a. Decarboxylation
b. Dehydrogenation
c. Acetylation
d. Carboxylation
Ans. c. Acetylation (Ref: Harper 30/e p561) Ans. c. Niacin
The given diagram is Casal’s Necklace in Pellagra
m

Pantothenic acid as a part of CoA take part in:


co

• Fatty acid Oxidation 37. Neurological worsening with anemia what is the
• Acetylation treatment to be given: (NBE pattern Q)
• Citric acid cycle a. Folic Acid alone
• Cholesterol synthesis b. Folic acid along with Hydroxycobalamin
As a part of ACP in fatty acid Synthesis c. Iron
d. Pyridoxine
34. Vitamin given in pregnant women to prevent Ans. b.
neural tube defect: (NBE pattern Q)
Folic acid along with Hydroxycobalamin
a. Folic acid
b. Vitamin B12 38. Vitamin deficiency causing dementia:
c. Vitamin C (NBE pattern Q)
d. Vitamin A a. Biotin
m

Ans. a. Folic Acid (Ref: Harper 30/e p558) b. Thiamin


co

c. Pyridoxine
35. Not needed in TCA cycle: (NBE pattern Q)
d. Vitamin B12
a. Pyridoxine
Ans. d. Vitamin B12
b. Thiamine
Vitamin deficiencies associated with dementia are
c. Riboflavin
Vitamin B1, Niacin, Vitamin B12.
d. Niacin
Ans. a. Pyridoxine 39. Pantothenate Kinase associated
Vitamins required for TCA Cycle are Riboflavin, Niacin, neurodegenaration is (NBE pattern Q)
Thiamin, and Pantothenic acid. a. Wilson’s Disease
b. Hallervorden- Spatz syndrome
36. Identify the vitamin deficiency:
c. McLeod Syndrome
(NBE pattern Q)
m

a. Riboflavin d. LeschNyhan Syndrome


co

b. Ascorbic Acid Ans. b. Hallervorden- Spatz syndrome


c. Niacin Pantothenate kinase associated neurodegeneration
d. Biotin (PKAN)(formerly Hallervorden-Spatz syndrome)
e

e
m

m
m
co

Vitamins and Minerals   | 379

• Rare autosomal recessive neurodegenerative disorder • Branched Chain KetoacidDehydrogenaseQ which


• Chorea, dystonia, parkinsonian features, pyramidal catalyses oxidative decarboxylation of Branched
tract features and MR Chain Amino acids.
• MRI-decreased T2 signal in the globuspallidus and • Trans KetolaseQ in Pentose Phosphate PathwayQ.
substantianigra, ‘eye of the tiger’ sign (hyperintense This is the biochemical basis of assay of Thiamine
area within the hypointense area) status of the body.
m

• sometimes acanthocytosis
43. Sebhoreic Dermatitis is produced by deficiency
co

• Neuropathologic examination indicates excessive


of: (NBE pattern Q)
accumulation of iron-containing pigments in the
a. Vitamin A
globuspallidus and substantianigra.
b. Vitamin B1
• Similar disorders are grouped as neurodegeneration
c. Vitamin B2
with brain iron accumulation (NBIA)
d. Vitamin C
40. The form of THFA used in treatment is: Ans. c. Vitamin B2
(NBE pattern Q) Riboflavin Magenta tongue, angular stomatitis, cheilosis,
a. N5 Formyl THFA seborrheic dermatitis
b. N10 Formyl THFA
c. N 5 Formimino THFA 44. Severe thiamine deficiency is associated with:
d. N5 Methyl THFA (NBE pattern Q)
m

Ans. a. N5 Formyl THFA (Ref: Harper 30/e p559) a. Decreased RBC transketolase activity
co

• 5 Formyl THFA is more stable than Folate, therefore b. Increased clotting time
used pharmaceutically c. Decreased RBC transaminase activity
• Known as Folinic acid d. Increased xanthurenic acid excretion
• The synthetic racemic compound of Folinic acid is Ans. a. Decreased RBC Transketolase activity
Leucovorin Thiamin is a cofactor of Transketolase.
41. Excess of avidin causes deficiency of: Minerals
(NBE pattern Q)
a. Biotin 45. Which of the following is wrongly matched:
b. Choline (JIPMER Nov 2015)
c. Vitamin B12 a. Folate-Anemia
m

d. Folate b. Zinc-Immunodeficiency
co

Ans. a. Biotin (Ref: Harper 30/e p560) c. Iodine –Dry Skin


d. Iron-Anemia
42. Thiamin act as a cofactor in: (NBE pattern Q)
Ans. c. Iodine Dry skin
a. Pyruvate to Oxaloacetate
b. Malonate to Oxaloacetate Mineral Function Deficiency

c. Succinate to Fumarate Cobalt Constituent of Vitamin Macrocytic Anemia


B12
d. Pyruvate to Acetyl CoA
Chromium Potentiate the action Impaired Glucose Tol-
Ans. d. Pyruvate to Acetyl CoA (Ref: Harper 30/e p555) of Insulin erance
Thiamine generally function in the decarboxylation Fluoride Constituent of Bone and Dental caries
reaction of alpha keto acids and branched chain amino teeth
acids Iodine Thyroid Hormone Syn- Thyroid enlargement,
• Pyruvate DehydrogenaseQ which convert Pyruvate thesis ↓T4, cretinism
m

to Acetyl CoA. Molybdenum Cofactor for Xanthine Severe neurologic


co

• Alpha KetoGlutarate DehydrogenaseQ in Citric Acid Oxidase and Sulfite Oxi- abnormalities, Xanthin-
dase, Aldehyde oxidase uria
Cycle which convert α KetoGlutarate to Succinyl
CoA. Contd...
e

e
m

m
m
co

380 |  
Self Assessment and Review of Biochemistry

Mineral Function Deficiency Ans. d. Alcohol Dehydrogenase


Selenium Cofactor for Glutathione Keshan’s Disease (Car- Zinc Cofactor for
Peroxidase diomyopathy), heart • Carbonic Anhydrase
• Deiodinase, failure, striated muscle • Carboxy Peptidase
• Thioredoxin Reduc- degeneration • Lactate Dehydrogenase
tase • Alcohol Dehydrogenase
• Antioxidant along with • Alkaline Phosphatase
m

Vitamin E
co

Zinc Cofactor for Growth retardation, 49. Cardiomyopathy is due to deficiency of


• Carbonic Anhy- ↓taste and smell, alope- a. Selenium (PGI Nov 2012)
drase cia, dermatitis, diarrhea,
• Carboxy Peptidase immune dysfunction, b. Phosphorus
• Lactate Dehydroge-
failure to thrive, gonadal c. Boron
atrophy, congenital
nase
malformation Impaired d. Zinc
• Alcohol Dehydro- wound healing e. Iron
genase
• Alkaline Phospha- Ans. a. Selenium
tase Nutritional Causes of Cardiomyopathy are Deficiency
Mangenese Cofactor for Impaired growth and of Thiamin, Selenium, Calcium and magnesium. Excess
• Arginase, skeletal development, of Iron (Hemochromatosis)
• Carboxylase, reproduction, lipid and
carbohydrate metabo- 50. Selenium deficiency causes: (PGI May 2012)
m

• Kinase,
• Enolase,
lism; upper body rash a. Dermatitis
co

• Glucosyl Tranferase, b. Cardiomyopathy


• Phospho GlucoMu- c. Diarrhea
tase Required for
d. Alopecia
RNA Polymerase
e. Gonadal Atrophy
46. Which of the following is a non-essential metal/ Ans. b. Cardiomyopathy
mineral?
51. Copper containing enzymes are: (PGI May 2012)
a. Sodium
a. Superoxide Dismutase
b. Manganese
b. Cytochrome Oxidase
c. Iron
c. Myeloperoxidase
d. Lead
d. Tyrosinase
Ans. d. Lead
m

e. Amino Acid Oxidase


Lead is a toxic mineral.
co

Ans. a, b, d, e.
47. The 40 nm gap in between the tropocollagen 52. Which of the following is considered the active
molecule in collagen which serve as the site of from of calcium:
bone formation is occupied by: (AIIMS Nov 06) a. Ionized Calcium
a. Carbohydrates b. Albumin bound Calcium
b. Ligand moiety c. Phosphate bound Calcium
c. Calcium d. Protein bound Calcium
d. Ferric ion Ans. a. Ionized Calcium
Ans. c. Calcium
53. Copper involves collagen synthesis by:
48. Zinc is a cofactor for: (AIIMS Nov 2009) (Kerala 2010)
a. Pyruvate Dehydrogenase
m

a. Lysyl oxidase
b. Pyruvate decarboxylase
co

b. Lysyl hydroxylase
c. Alphaketoglutarate Dehydrogenase c. Cytochrome oxidase
d. Alcohol Dehydrogenase d. Tyrosinase
e

e
m

m
m
co

Vitamins and Minerals   | 381

Ans. a. Lysyl Oxidase c. Inactivation of Chloride Channel


Cofactor role of Copper d. Difficult to produce ACh molecules
• Amine oxidases Ans. c. Inactivation of Chloride Channel
• Ferroxidase (ceruloplasmin) (Iron metabolism, Copper Transport) (Ref: Harper 30/e p555)
• Cytochrome-c oxidase (in Complex IV of Electron Transport Chain)
• Superoxide dismutase (Free Radical Scavenging enzyme)
Thiamin triphosphate has a role in nerve conduction; it
• Tyrosinase (Melanin Synthesis) phosphorylates, and so activates, a chloride channel in
m

• Component of ferroportin (Iron Metabolism) the nerve membrane


co

• Lysyl Oxidase (Cross linking in Collagen)


56. Selenium is a cofactor in the following enzyme:
(NBE pattern Q)
54. Zinc is present in: (NBE pattern Q) a. Glutathione Peroxidase
a. Carbonic anhydrase
b. Cytochrome Oxidase
b. Xanthine oxidase
c. Cytochrome Reductase
c. Glutathione reductase
d. Xanthine Oxidase
d. Glutathione synthetase
Ans. a. Glutathione Peroxidase (Ref: Harper 30/e p567)
Ans. a. Carbonic anhydrase
Selenium Cofactor for
55. Cause of thiamine induced nerve weakness: • Glutathione Peroxidase
(NBE pattern Q) • Deiodinase,
m

a. Hypocalcemia • Thioredoxin Reductase


co

b. Hypomagnesimia • Antioxidant along with Vitamin E


m
co
m
co
e

e
m

m
m
co

16 Heme Metabolism
and Hemoglobins
m
co

Topics Included
• Structure of Heme • Heme Catabolism
• Heme Synthesis • Hyperbilirubinemias
• Porphyrias • Abnormal Hemoglobins
m
co

STRUCTURE OF HEME Special Features of Porphyrins


• Porphyrinogens are colorless where as porphyrins are colored
Heme is a metalloporphyrin compounds. The conjugated double bond in the pyrrole ring and
Porphyrins linking methenyl bridges is responsible for characteristic absorption
and fluorescence spectra.
Porphyrins are cyclic compounds formed by the linkage • Sharp absorption band near 400 nm called soret band is
of four pyrrole rings through methyne or methenyl distinguishing feature of all porphyrins after its discoverer, the
bridges (= CH-) French physicist Charles Soret.
• When porphyrins dissolved in strong mineral acids or in organic
solvents are illuminated by ultraviolet light, they emit a strong red
fluorescence.
• Used in cancer phototherapy because tumors often take up more
porphyrins than normal tissue. The tumor is then exposed to an
m

argon laser, which excites the porphyrins, producing cytotoxic


effects. This is called Photodynamic therapy.
co

• Porphyrins cause photosensitivity.

Porphyrins
A characteristic property of porphyrins is the formation
of complexes with metal ions bound to the nitrogen atom
of the pyrrole rings.
• Iron porphyrins: Heme of hemoglobin
• Magnesium-containing porphyrin: Chlorophyll, the
photosynthetic pigment of plants.
The porphyrins found in nature are compounds in which
various side chains are substituted for the eight hydrogen
m

atoms numbered in the porphyrin nucleus. The side


co

chains are
M-Methyl A-Acetate
Fig. 16.1: Structure of porphyrin V-Vinyl P-Propionate
e

e
m

m
m
co

Heme Metabolism and Hemoglobins   | 383

• The three important porphyrins are Uroporphyrin, • Porphyrin in heme is Protoporphyrin.


Coproporphyrin and Protoporphyrin. • Metal in heme is iron.
• The distribution of side chain in each of the above • So Heme is Ferroprotoporphyrin.
porphyrin is given in the figure (16.3) below. • The iron in the Heme is in Ferrous state.
• Most water soluble porphyrin is Uroporphyrin.
• Least water soluble is Protoporphyrin.
m

• Coproporphyrin is having intermediate water


co

solubility.
• Heme is metal + Porphyrin.

Fig. 16.4: Structure of heme

Important Heme containing Proteins


Fig. 16.2: Representation of porphyrin
m

Hemoglobin Transport of oxygen in blood


co

Myoglobin Storage of oxygen in muscle


Cytochrome c Involvement in electron transport chain
Cytochrome P450 Hydroxylation of xenobiotics
Catalase Degradation of hydrogen peroxide
Tryptophan pyrrolase Oxidation of tryptophan

BIOSYNTHESIS OF HEME
• Site: Synthesized in almost all tissues in the body
EXCEPT in mature erythrocytes.
• 85% in erythroid Precursor cells in the bone marrow
and majority of remainder in hepatocyte
m

• Organelle: Partly cytoplasmic and partly mitochon-


co

drial
• Starting Materials: Succinyl CoA and Glycine.

Steps of Synthesis of Heme


Can be divided into
• Synthesis of Porphobilinogen (Monopyrrole)
• Synthesis of Uroporphyrinogen (Tetrapyrrole)
• Conversion of Uroporphyrinogen to Protoporphyrin
• Formation of Heme by incorporation iron.

Synthesis of Porphobilinogen (Monopyrrole)


m

ALA Synthase (ALAS): Catalyze condensation reaction


co

between succinyl-CoA and glycine to form-α amino-β-


ketoadipic acid, which is rapidly decarboxylated to form
-δ aminolevulinate (ALA)
Fig. 16.3: Three types of porphyrins Synthesis of ALA occurs in mitochondria.
e

e
m

m
m
co

384 |  
Self Assessment and Review of Biochemistry

Two isoforms of ALAS • ALA dehydratase is a zinc-containing enzyme


• ALAS-I is expressed throughout the body. • This enzyme is sensitive to inhibition by lead, as can
• ALAS -II is expressed in erythrocyte precursor cells. occur in lead poisoning.
• ALAS-1 is the rate limiting step of hepatic heme Synthesis of Uroporphyrinogen (Tetrapyrrole)
synthesis. Heme acts as the negative regulator of
The formation of a cyclic tetrapyrrole—i.e. a porphyrin—
ALAS-1. ALAS-1 is induced by the drugs whose
occurs by condensation of four molecules of PBG.
m

metabolism require Cytochromes.


co

• ALAS-2 is not feedback regulated by heme. ALAS-2 Uroporphyrinogen–I Synthase or HMB Synthase or
is not induced by drugs. PBG Deaminase
• Four molecules of PBG condense in a head-to-tail
ALA Dehydratase manner to form a linear tetrapyrrole, hydroxy-
• Two molecules of ALA are condensed by the enzyme methylbilane (HMB).
ALA dehydratase to form two molecules of water • 4 mols of NH3 is released. Takes place in the cytosol.
and one mol of porphobilinogen (PBG) Thus the • The reaction is catalyzed by uroporphyrinogen
first precursor monopyrrole is formed. I synthase, also named PBG deaminase or HMB
• Takes place in the cytosol. synthase.
m
co

Fig. 16.5: Synthesis of ALA


m
co
m
co

Fig. 16.6: Synthesis of PBG


e

e
m

m
m
co

Heme Metabolism and Hemoglobins   | 385

Uroporphyrinogen III synthase • This also takes place in the cytosol


• HMB is converted to Uroporphyrinogen III by • Coproporphyrinogen III then enters the mitochondria,
Uroporphyrinogen III synthase. where it is converted to protoporphyrinogen III
• Uroporphyrinogen is thus the first porphyrin • The mitochondrial enzyme coproporphyrinogen
precursor formed. oxidase catalyzes the decarboxylation and oxidation
• Under normal conditions, the uroporphyrinogen of two propionic side chains to form protoporphy-
m

formed is almost exclusively the III isomer. rinogen.


co

• But in certain porphyrias, HMB cyclizes spontaneously • This enzyme is able to act only on type-III copropor-
to form uroporphyrinogen I. phyrinogen, which would explain why type I proto-
porphyrins do not generally occur in nature.
Protoporphyrinogen Oxidase
The oxidation of protoporphyrinogen to protoporphyrin
is catalyzed by another mitochondrial enzyme, protopor-
phyrinogen oxidase.

Formation of Heme by incorporation of Iron


• This is the final step in heme synthesis
• It involves the incorporation of ferrous iron into
m

protoporphyrin in a reaction
co

• This step is catalyzed by ferrochelatase (heme


synthase)
• Takes place in the mitochondria

Regulation of Heme Synthesis


• ALA SynthaseQ is the key regulatory enzyme in
hepatic biosynthesis of Heme
• ALA synthase occurs in both hepatic (ALAS1) and
Fig. 16.7: Synthesis of Uroporphyrinogen
erythroid (ALAS2) forms.
The rate-limiting reaction in the synthesis of heme in
liver is ALAS1.
m

It appears that heme, probably acting through an


aporepressor molecule, acts as a negative regulator
co

of the synthesis of ALAS1 by repression-derepression


mechanism.
Thus, the rate of synthesis of ALAS1 increases greatly
Fig. 16.8: Synthesis of Coproporphyrinogen in the absence of heme and is diminished in its presence.

Factors that Affect Heme Synthesis


Conversion of Uroporphyrinogen to Protoporphyrin
• Drugs: that induce Hepatic Cytochromes, e.g.
by Uroporphyrinogen Decarboxylase
BarbituratesQ, Griseofulvin
• Uroporphyrinogen III is converted to coproporphy-
rinogen III by decarboxylation of all of the acetate • LeadQ: Inhibit steps catalyzed by ALAdehydratase
(A) groups, which changes them to methyl (M) sub- and ferrochelatase
stituent. • INH: Decrease availability of PLP.
m

• The reaction is catalyzed by uroporphyrinogen


co

Basis of administration of glucose to relieve acute attacks of


decarboxylase. porphyrin
• This is also capable of converting uroporphyrinogen High cellular concentration of glucose prevents induction of ALA
Synthase.
I to coproporphyrinogen I
e

e
m

m
m
co

386 |  
Self Assessment and Review of Biochemistry
m
co
m
co

Fig. 16.9: Summary of conversion of PBG to heme

PORPHYRIAS Classification of Porphyrias

The porphyrias are a group of disorders due to abnormali-


ties in the pathway of biosynthesis of heme; they can be
genetic or acquired.
m

Most of the porphyrias are inherited autosomal dominant EXCEPT


• ALAD enzyme Deficiency [ADP]
co

• Congenital Erythropoeitic Porphyria [CEP]


• Erythropoetic Protoporphyria [EPP]
• X Linked Protoporphyria [XLP]

Concept of Clinical features of Porphyria


m
co
e

e
m

m
m
co

Heme Metabolism and Hemoglobins   | 387


m
co
m
co

Fig. 16.10: Biochemical defects of various porphyrias


m

Contd...
Porphyrias at a glance
co

• Erythropoietic porphyrias usually present with Cutaneous • Most common Porphyria in children is Erythropoietic Proto-
photosensitivity. porphyria (EPP)
• Hepatic Porphyria with Cutaneous photosensitivity is Porphyria • Porphyria which can be sporadic is Porphyria Cutanea Tarda (PCT)
Cutanea Tarda (PCT). • Porphyria that is frequently seen in countries where Hepatitis C
• Hepatic Porphyria is symptomatic in adults. and HIV is prevalent is PCT
• Erythropoietic Porphyria usually present at birth or early childhood. • Confirmatory diagnostic test for all Porphyrias are Enzyme analysis
• Porphyria that presents as Nonimmune Hydrops Fetalis is and Mutation analysis.
Congenital Erythropoietic Porphyria. • First line investigation of Porphyrias with neurovisceral symptoms
• Most common Porphyria is PCT. is Spot Urine ALA and PBG.
• Most common acute Porphyria is Acute Intermittent Porphyria (AIP) • First line investigation of Porphyrias with Photosensitivity is Plasma
Contd... Porphyrins.

Major clinical features and laboratory features of Porphyrias


m

Principal Symp-
Porphyria Deficient Enzyme Inheritance toms NV or CP+ Results of Laboratory Tests
co

5-ALA dehydratase-deficient ALA-dehydratase AR NV Zn Protoporphyrin in erythrocytes


porphyria (ADP)
Acute intermittent porphyria HMB-synthase or Uroporphy- AD NV Urinary ALA and PBG increased
(AIP) rinogen I synthase
Contd...
e

e
m

m
m
co

388 |  
Self Assessment and Review of Biochemistry

Contd...

Principal Symp-
Porphyria Deficient Enzyme Inheritance toms NV or CP+ Results of Laboratory Tests
Porphyria cutaneatarda (PCT) Uroporphyrinogen decar- AD CP Urinary uroporphyrin I increased
boxylase
Hereditary coproporphyria Coproporphyrinogen oxidase AD NV & CP Urinary ALA, PBG, and coproporphyrin
(HCP) III and fecalcoproporphyrin III increased
m

Variegate porphyria (VP) Protoporphyrinogen oxidase AD NV & CP Urinary ALA, PBG, and coproporphyrin
co

III and fecal protoporphyrin IX increased


Congenital erythropoietic por- Uroporphyrinogen III syn- AR CP Urinary, fecal, and red cell uroporphyrin
phyria (CEP) thase I increased
Erythropoietic Protoporphyria Ferrochelatase AR CP Fecal and red cell protoporphyrin IX
(EPP) increased
X-linked protoporphyria (XLP) ALA-synthase 2 XL CP Protoporphyrin in Erythrocytes and stool.

ALAD Deficient Porphyria [ADP] • So brownish discoloration of teeth


• Aminolevulinic Acid Dehydratase Deficient • Hemolysis due to erythrocyte porphyrins, hence
Porphyria (ADP) splenomegaly
• Otherwise called Doss Porphyria • Uroporphyrin I and Coproporphyrin I accumulate in
• Rare Porphyria. bonemarrow, erythrocyte, plasma, urine and feces.
m

• Coproporphyrin I is the predominant porphyrin in


Differential Diagnosis of ADP
co

feces.
• Lead intoxication resembles ADP as lead inhibit ALA
Dehydratase and there is increased excretion of ALA. • Portwine urine or Pink stain in diapers
• Hereditory Tyrosinemia Type I resembles ADP, as • Erythrodontia-Reddish Flourescence of teeth when
Succinyl Acetone resembles ALA. illuminated with long uv light
• Prenatal diagnosis by Porphyrin in amniotic fluid,
Acute Intermittent Porphyria [AIP] URO Synthase enzyme activity in chorionic villus
Defect in PBG Deaminase/HMB Synthase/ Uroporphy- and cultured amniotic cells
rinogen I Synthase • Beta Carotene Q to protect from sunlight
• Most common ACUTE Porphyria • Gene therapy using Human cDNA UROS retroviral
• Most common symptom-Abdominal Pain vectors.
• Most common Physical Sign –Tachycardia
Porphyria cutanea Tarda
m

• Neurovisceral
• Uroporphyrinogen Decarboxylase defect
co

• Levels of ALA and PBG increased


• Urine Red colored • 80% is sporadic attributed to Uroporphyrinogen
Decarboxylase inhibitors.
• Currently liver directed gene therapy with Adeno
Associated Viral vector (AAV-HMBS) has been • The aggravating factors of PCT are
proven to prevent drug induced AIP. ‒ Hepatitis C, HIV
• Hepatic targeted RNA interference (RNAi) therapy ‒ Excess Alcohol
directed to inhibit markedly elevated ALAS-1 mRNA, ‒ Elevated Iron
reduced the ongoing attack. ‒ Estrogen
- Most common Porphyria
Congenital Erythropoietic Porphyria [CEP]
Gunther’s Disease - Most readily treated Porphyria
- Associated with Hemochromatosis
Uroporphyrinogen III Synthase defect - Blistering Skin Lesions mostly in the back of
m

• Presents shortly after birth or in Utero as Nonimmune hands.


co

Hydrops
- They are susceptible to develop chronic liver
• Presents with severe Cutaneous Photosensitivity disease and are at risk for Hepatocellular
• Porphyrins deposit in teeth and bones. Carcinoma.
e

e
m

m
m
co

Heme Metabolism and Hemoglobins   | 389

Treatment The fate of Hemoglobin


• Repeated Phlebotomy to reduce hepatic iron. • When hemoglobin is destroyed in the body
• Low dose regimen of Chloroquine or Hydroxy • Globin is degraded to its constituent amino acids,
chloroquine. which are reused.
• In patients with End stage renal Disease, administer • Iron of heme enters the iron pool, also for reuse.
Erythropoietin. • The iron-free porphyrin portion of heme is degraded,
m

mainly in the reticuloendothelial cells of the liver,


Erythropoietic Protoporphyria
co

spleen, and bone marrow.


Due to defect in Ferochelatase (FECH Mutation)
Steps of Catabolism of Heme
• Most common porphyria in children and second most
Site : Microsomal fraction of reticuloendothelial cells of
common in adults.
the liver, spleen, and bone marrow.
• NON BLISTERING Photosensitivity
• Characterized by pain, swelling redness within Hemoxygenase
minutes of sunlight exposure, resembling • Oxygen is added to the α-methyne bridge between
angioedema. pyrroles I and II of the porphyrin.
• Vesicular lesions are uncommon. • Splitting of tetrapyrrole ring.
• Green pigment, BiliverdinQ is produced.
Diagnosis • Carbon monoxide is produced by this reaction.
• A substantial increase in erythrocyte protoporphyrin,
m

• This is the only source of endogenous COQ in the


which is predominantly free and not complexed with
co

body.
Zn is the hallmark of EPP.
Biliverdin Reductase
• Erythrocytes exhibits red fluorescence under
fluorescence microscopy at 620 nm. • Reduces the methyne bridge between pyrrole III and
pyrrole IV of Biliverdin to a methylene group.
• FECH mutation analysis.
• Yellow pigment, bilirubin is produced
Treatment • This takes place in the cytosol.
• Oral Beta Carotene may improve tolerance to
sunlight.
• Afamelanotide, an alfa Melanocyte–stimulating
hormone has completed phase III trials for patients
with EPP and XLP.
m

X-Linked Protoporphyria
co

• Due to increased activity of ALA Synthase-2 due to


Fig. 16.11: Formation of bilirubin
gain of function mutation.

X-Linked Sideroblastic Anemia Transport of Bilirubin


• Not a Porphyria • Bilirubin formed in peripheral tissues is transported
• Due to decreased activity of ALA Synthase-2. to the liver by plasma albumin.
• Bilirubin is only sparingly soluble in water, but its
HEME CATABOLISM solubility in plasma is increased by noncovalent
binding to albumin.
Under normal conditions in human adults, some 200 • Each molecule of albumin appears to have one high-
billion erythrocytes are destroyed per day. affinity site and one low-affinity site for bilirubin.
m

A 70-kg human turns over approximately 6 g of • In 100 ml of plasma, approximately 25 mg of bilirubin


hemoglobin daily.
co

can be tightly bound to albumin at its high-affinity site.


1 g of hemoglobin yields 35 mg of bilirubinQ. • Bilirubin in excess of this quantity can be bound only
The daily bilirubin formation in human adults is loosely and thus can easily be detached and diffuse
approximately 250–350 mg. into tissues.
e

e
m

m
m
co

390 |  
Self Assessment and Review of Biochemistry

Metabolism of Bilirubin • The enzyme is mainly located in the endoplasmic


Occurs Primarily in the Liver reticulum Q , uses UDP-glucuronic acid as the
glucuronyl donor, and is referred to as bilirubin-UGT.
It can be divided into three processes.
• Bilirubin monoglucuronide is an intermediate and is
• Uptake of bilirubin by liver parenchymal cells
subsequently converted to the diglucuronide.
• Conjugation of bilirubin with glucuronate in the
• Most of the bilirubin excreted in the bile of mammals
endoplasmic reticulum
m

is in the form of bilirubin diglucuronide.


• Secretion of conjugated bilirubin into the bile.
co

• However, when bilirubin conjugates exist abnormally


Uptake of Bilirubin in the Liver in human plasma (e.g. in obstructive jaundice), they
• In the liver, the bilirubin is removed from albumin are predominantly monoglucuronides.
• Taken up at the sinusoidal surface of the hepatocytes • Bilirubin-UGT activity can be induced by a number
by a carrier-mediated saturable system (facilitated of clinically useful drugs, including phenobarbital.
transport system) Secretion of conjugated bilirubin into the bile
• This has a very large capacity, so that even under • This occurs by an active transport mechanism
pathologic conditions the system does not appear to
• This is the rate-limiting for the entire process of
be rate limiting in the metabolism of bilirubin
hepatic bilirubin metabolism.
• Once bilirubin enters the hepatocytes, it can bind to
• The protein involved is MRP-2 (multidrug-resistance-
certain cytosolic proteins. This is called intracellular
like protein 2), also called multispecific organic anion
m

binding. transporter (MOAT).


co

The proteins help in intracellular binding are • It is located in the plasma membrane of the biliary
• Ligandin (a member of the family of glutathione canalicular membrane.
S-transferases) • Recently it is found that apart from secretion into
• Protein Y biliary canaliculi, a portion of bilirubin diglucuronide is
transported into portal circulation by MRP-3 (Multi drug
Functions of intracellular binding
Resistance associated protein-3).
• They may also help to prevent efflux of bilirubin back
• They are subjected to reuptake into hepatocyte by
into the blood stream.
transporters, Organic anion Transporter 1B1 (OATP1B1),
• They help to keep bilirubin solubilized prior to and 1B3 (OATP1B3).
conjugation • It is a member of the family of ATP-binding cassette
Conjugation of Bilirubin with Glucuronic Acid Occurs transporters.
m

in the Liver • The hepatic transport of conjugated bilirubin into the


co

• Hepatocytes convert bilirubin to a polar form, which bile is inducible by those same drugs that are capable
is readily excreted in the bile, by adding glucuronic of inducing the conjugation of bilirubin.
acid molecules to it. Conjugated bilirubin is reduced to urobilinogen by
• This process is called conjugation and can employ intestinal bacteria
polar molecules other than glucuronic acid. • The conjugated bilirubin reaches the terminal ileum
• The conjugation of bilirubin is catalyzed by a specific and the large intestine
glucuronyltransferase. • The glucuronides are removed by specific bacterial
enzymes (β-glucuronidases)
• This is subsequently reduced by the fecal flora to a
group of colorless tetrapyrrolic compounds called
urobilinogens.
m

Fates of Urobilinogen
co

• 80–90% of the urobilinogen is converted to


stercobilinogen and stercobilin by intestinal flora
Fig. 16.12: Conjugation of bilirubin and excreted through feces.
e

e
m

m
m
co

Heme Metabolism and Hemoglobins   | 391

• 10–20% enterohepatic circulation reaches the liver. Contd...


This is called enterohepatic urobilinogen cycle. Gilbert Syn-
• A small fraction < 3 mg/dL escape hepatic uptake, Features Crigler-Najjar Syndrome drome
filters across renal glomerulus and is excreted Type-I Type-II
through urine. Inheritance (all Recessive Predomi- 7 of 8 domi-
autosomal) nantly reces- nant; 1 re-
sive portedly
m

HYPERBILIRUBINEMIAS recessive
co

Jaundice
• Scleral icterus indicate serum bilirubin > 3 mg/dL Physiological Jaundice
• Carotenoderma can be distinguished from Icterus by • Predominantly unconjugated hyperbilirubinemia.
sparing of sclera.
Causes
Congenital Hyperbilirubinemias • Incompletely developed hepatic system
• Low UDP Glucuronyl Transferase enzyme
Uncojugated hyperbilirubinemias
• Unconjugated hyperbilirubinemia that develop 2nd
• Gilbert’s disease
to 5th day of birth
• Crigler-Najjar syndrome.
• Peak level of Serum Bilirubin is 5–10 mg/dL
Principal Differential characteristics of Crigler-Najjar • Decline to normal adult concentration in 2 weeks.
and Gilbert syndrome
m

Breast Milk Jaundice


co

Gilbert Syn-
Features Crigler-Najjar Syndrome drome • Bilirubin conjugation is inhibited by certain fatty
Type-I Type-II acids that are present in the breast milk and not in
Total serum biliru- 310–755 100–430 Typically 70 serum.
bin, mol/L (mg/dL) [18–45 (usu- (usually 345) mol/L • A correlation between epidermal growth factor
ally >20)] [6–25 (usu- (4 mg/dL)
ally 20)]
content of breast milk and elevated bilirubin level is
noted in these infants.
Routine liver tests Normal Normal Normal

Response to Phe- None Decreases Decreases


Lucey Driscoll Syndrome
nobarbital bilirubin by bilirubin to • Transient familial neonatal hyperbilirubinemia.
>25% normal • UGT1A1 inhibitor is in maternal serum, and not in
Kernicterus Usual Rare No breast milk unlike breast milk jaundice.
m

Hepatic histology Normal Normal Usually Conjugated Hyperbilirubinemias


co

normal; in-
creased lipo- • Dubin Johnson’s syndrome
fuscin pigment • Rotor syndrome
Bile characteristics • Benign Recurrent intrahepatic Cholestatsis (BRIC)
Color Pale or col- Pigmented Normal dark • Progressive Familial intrahepatic Cholestatsis (FIC)
orless color
Bilirubin fractions >90% un- Largest frac- Mainly di- Dubin Johnson’s Syndrome
conjugated tion (mean: conjugates • Autosomal recessive
57%) mono- but mono-
conjugates conjugates • Cause–Mutation of gene encoding MRP2
increased • A cardinal feature is accumulation in the lysosomes of
(mean: 23%) centrilobular hepatocytes of dark, coarsely granular
Bilirubin UDP- Typically Mark- Reduced: typi- pigment. Liver is grossly black in appearance. This
m

glucuronos- absent; edly reduced: cally 10–33%


pigment is thought to be an epinephrine metabolite
co

yltransferase traces in 0–10% of of normal


activity some pa- normal that are not excreted properly
tients • Black liver jaundice
Contd... • Brom Sulphthalein test (BSP test) shows 2 Peak.
e

e
m

m
m
co

392 |  
Self Assessment and Review of Biochemistry

Rotor Syndrome • This protein is defective in Familial Intrahepatic


• Autosomal recessive Cholestatsis type 2 (FIC-type 2).
• Defective bilirubin excretion
• Recent studies indicate that deficiency of plasma
Progressive Familial Intrahepatic cholestasis (FIC)
transporters OATP1B1 and OATP1B3 is the cause. FIC Type 1 (Byler’s disease)
• This result in reduced reuptake of conjugated
• Due to FIC-1 mutation.
m

bilirubin pumped into portal circulation.


• Present in early infancy as cholestasis, initially
co

Benign Recurrent Intrahepatic Cholestasis (BRIC) episodic.


• Rare disorder characterized by recurrent attacks of • Unlike BRIC, Bylers disease progress to malnutrition,
jaundice and pruritus. growth retardation and End stage Liver Disease.
• There are two types BRIC-1 and BRIC-2
• In BRIC-1 a gene named FIC-1 is mutated, that FIC Type II
encodes a protein that play a role in biliary canalicular • Due to mutation of Bile Salt Excretory Protein (BSEP)
excretion of various compounds.
• In BRIC 2 mutation is in Bile salt Excretory Protein FIC Type III
(BSEP). • Due to mutation of MRP-3.

Differentiating features of important Conjugated Hyperbilirubinemia


m

DJS Rotor PFIC1 BRIC1 PFIC2 BRIC2 PFIC3


co

Gene ABCCA SLCO1B1/SLCO1B3 ATP8B1 ATP8B1 ABCB11 ABCB11 ABCB4


Protein MRP2 OATP1B1,OATP1B3 FIC1 FIC1 BSEP BSEP MDR3
Cholestasis No No Yes Episodic Yes Episodic Yes
Serum γ GT Normal Normal Normal Normal Normal Normal ↑↑
Serum Bile acids Normal Normal ↑↑ ↑↑ during ↑↑ ↑↑ during ↑↑
episodes epidodes
NB: Learn BRIC1 and PFIC1 together, then BRIC2 and PFIC2 together.

Acquired Hyperbilirubinemias Tests in urine


• Hemolytic Jaundice • Urine and fecal urobilinogen-By Ehrlich’s test
• Hepatic Jaundice • Urine Bilirubin (Bile pigment): By Modified
• Obstructive Jaundice Fouchet’s test
m

• Urine Bile salt by Hay’s test, Pettenkofer test.


co

Laboratory Tests Done to Differentiate Jaundice


Liver enzyme panel
Serum Bilirubin • Transaminases (AST and ALT) elevated in Hepatitis
By Van den Bergh test • Alkaline Phosphatase elevated in Obstructive
• Chemical method to estimate bilirubin in serum Jaundice
• Bilirubin + Ehrlich’s Diazoreagent (Diazotized • 5’ Nucleotidase in Obstructive Jaundice.
Sulfanilic Acid) Laboratory tests in three different types of jaundice
• Reddish purple azopigment is formed. Serum Urine
• They are analyzed by photometry at 540 nm Condition Bilirubin Urobilinogen Urine Bilirubin
Normal Direct: 0.1–0.4 0–4 mg/24 h Absent
Two types of reaction in this test: mg/dL
Direct and Indirect Indirect: 0.2–0.7
m

• Conjugated bilirubin: Direct positive, so conjugated mg/dL


co

bilirubin is otherwise Direct bilirubin. Hemolytic- Indirect Increased Absent


anemia
• Unconjugated bilirubin: Indirect positive, so unconju-
gated bilirubin is otherwise indirect bilirubin. Contd...
e

e
m

m
m
co

Heme Metabolism and Hemoglobins   | 393

Contd... Types of Hemoglobin


Serum Urine Embryonic HemoglobinsQ
Condition Bilirubin Urobilinogen Urine Bilirubin
• Hb Portland-ζ2γ2
Hepatitis Direct and indi- Decreased if Present if
• Hb Gower I-ζ2ε2
rect micro-obstruc- micro-obstruc-
tion is present tion occurs • Hb Gower II-α2ε2
Obstructive Direct Absent Present
m

jaundice Fetal HemoglobinQ


co

• Hb F-α2γ2
Delta Bilirubin or Biliprotein Characteristics of Fetal HemoglobinQ
• Conjugated bilirubin that is covalently linked to • Slower Electrophoretic Mobility
albumin. • Increased resistance to Alkali Denaturation
• Half-life of delta bilirubin is 12–14 days [Half life of • Decreased Interaction with 2,3 BPG.
unbound Bilirubin is only 4 hrs]
• In case of conjugated hyperbilirubinemia bilirubinuria Adult Hemoglobins
starts late. • Hb A1-α2β2
• Hb A2--α2δ2
HEMOGLOBIN Characteristics of Hemoglobin
• Hemoglobin, a tetrameric protein of erythrocytes. Hemoglobin is an allosteric Protein.
m

• Consists of a pair of α-like chains 141 amino acids • Binding of one molecule of oxygen to one heme
co

long and a pair of β-like chains 146 amino acids long. residue increases the affinity of binding of oxygen to
other heme residue called Heme-heme interaction or
• Each globin chain enfolds a single heme moiety
Positive Cooperativity
• A single heme moiety, consists of a protoporphyrin • Hence Oxygen Dissociation Curve is Sigmoidal.
IX ring complexed with a single iron atom in the
ferrous state (Fe2+). Functions of Hemoglobin
• Each heme moiety can bind a single oxygen molecule. The functions of hemoglobins are to transports O2 to the
• A molecule of hemoglobin can transport up to four tissues and returns CO2 and protons to the lungs.
oxygen molecules.
Functional Histidine Residues in Hemoglobin
Higher Order Structure of Hemoglobin • Proximal Histidine [F8] and Distal Histidine [E7] are
• Secondary structure: Various globin chains are the functional His residues in Hemoglobin.
m

predominantly alpha helix. • Fifth coordination position of iron is occupied by a


co

• Tertiary structure: It is globular tertiary structures with nitrogen of imidazole ring of the proximal histidine,
the exterior surfaces to be rich in polar (hydrophilic) His F8.
amino acids that enhance solubility, and the interior • Distal Histidine, His E7 lies on the side of heme ring
lined with nonpolar groups, forming a hydrophobic opposite, His F8.
pocket into which heme is inserted
Conformational States of Hemoglobin
• Quaternary structure: It is a tetramer that contains
• Two states are T (taut) state and R (relaxed) state
two αβ dimmers.
• T state is the low affinity to Oxygen state, so it is the
• The hemoglobin tetramer is highly soluble but deoxygenated state.
individual globin chains are insoluble.
• T to R state is formed by breaking the salt bridges.
• Unpaired globin precipitates, forming inclusions that
• R state is the high affinity to oxygen state, it is the
damage the cell. oxygenated state.
m

Genetics of alpha and Beta chains • T state favor oxygen delivery to tissues.
co

The human hemoglobins are encoded in two tightly linked • R state favor binding of Oxygen to Hemoglobin.
gene clusters; the α-like globin genes are clustered on • Binding of first Oxygen to deoxy Hb shifts the
chromosome 16 and the β-like genes on chromosome 11. conformation from T to R state
e

e
m

m
m
co

394 |  
Self Assessment and Review of Biochemistry

• Binding of 2,3 BPG stabilises the T structure of Hb, • Because, Histidine that forms salt bridges with 2,3
so shifts ODC to right. BPG is not present in γ chain of HbF.
• Instead of Histidine it is Serine, so 2,3 BPG has less
interaction with Fetal Hb.
• This accounts for high affinity of HbF towards
Oxygen.
m

Glycated Hemoglobins
co

• These are formed nonenzymatically by condensation


of glucose or other reducing sugars, called Glycation
with alpha- and beta-chains of hemoglobin A
• Overall reaction called Maillard Reaction
• 80% glucose is added to N terminal valine of beta
chain to form HbA1c
• A stands for Aldimine [Linkage between Hb and
NH2 group of Valine]
• Best index of long-term control of Blood Glucose
• Since the half-life of an erythrocyte is typically 60
days, the level of glycated hemoglobin (HbA1c)
m

reflects the mean blood glucose concentration over


co

the preceding 6 to 8 weeks


• Normal level of HbA1c is < 5.5%
• 5.5% to 6% is good control.
HbA1c derived Average glucose value
HbA1c level Average Whole blood glucose
5.5% 110 mg/dL
6% 126 mg/dL
7% 154 mg/dL
8% 183 mg/dL
9% 212 mg/dL
m

10% 240 mg/dL


co

Fig. 16.13: Factors affecting the oxygen dissociation curve


Calculation of average blood glucose values from HbA1c
• Average Plasma Blood Glucose (mg/dl) = (HbA1c x 35.6) - 77.3
Interaction of Hemoglobin with 2, 3 BPG Q
• Average Plasma Blood Glucose (mmol/L) = (HbA1c x 1.98) - 4.29
• 2, 3 BPG is an intermediate of Glycolysis • Average whole blood glucose = Plasma Blood Glucose / 1.12
• Synthesized with the help of 2, 3 BPG Shunt or
Rapaport Leubering Cycle. MYOGLOBIN
• The Hemoglobin tetramer binds to one mol of 2,3
BPG in the central cavity formed by four subunits. • Seen in Muscle
• 2,3 BPG forms salt bridges with terminal amino • Is a Monomer
group of both β globin chain via, Valine, Lysine and • MW is 17000 KDa
Histidine. • Oxygen is stored in red muscle myoglobin is released
m

• Binding of 2,3 BPG decreases the affinity of Hb to during severe exercise for use in muscle mitochondria
co

Oxygen, by stabilizing the T state. for aerobic synthesis of ATP


• Shifts the ODC to right. • Rich in alpha helix
• 2, 3 BPG has decreased interaction with HbF. • Bind only one mol of Oxygen
e

e
m

m
m
co

Heme Metabolism and Hemoglobins   | 395

• High affinty for Oxygen Contd...


• Exhibit no Bohr effect Classification of Hemoglobinopathies
• Exhibit no Cooperativity Altered functional or
• No interaction with 2, 3 BPG Name of physical and chemi-
Hemoglobinopathy cal properties
• ODC is Hyperbolic and not sigmoidal.
III (B) Hb Constant Spring In the alpha chain C ter-
minal termination codon
m

HEMOGLOBINOPATHIES is mutated to a coding


co

codon hence an elon-


Classification of Hemoglobinopathies gated alpha chain of 173
aminoacid is formed.
Altered functional or
Name of physical and chemi- III (C) Hb Lepore α (δβ)2 Unequal cross over and
Hemoglobinopathy cal properties recombination even that
fuses proximal end of δ
I Structural hemoglobinopa- Amino acid sequences gene with distal end of
thies is altered β gene.
I A (1) HbS, Hemo- β6 Glu→Val Abnormal Polymeriza- IV Hereditary persistence of fetal Persistence of high lev-
globin Sickling tion hemoglobin els of HbF into adult lif
IA (2) HbC β6 Glu→Lys
I (B) Altered Oxygen affinity Sickle Cell Anemia
I B(1) Hb Yakima β99Asp→His Increased O2 affinity-
Molecular Defect in Sickle cell anemia
m

hence polycythemia
co

IB(2) Hb Kansas β102 Low O2 affinity—cyano-


Asn→Lys sis, pseudoanemia
IC)Hemoglobin that oxidize readily
Hb Philly β35Tyr→Phe Unstable hemoglobins

Hb Genova β28Leu→Pro
Hb Koln β98Val→Met
Hb M Boston M hemoglobins
Prox or Distal Histidine
HbM Iwata α87His→Tyr
mutated to Tyrosine
HbM Saska-
toon
m

HbM Hyde
co

park
Hb M Milwau-
kee
II Thalassemias Defective biosynthesis
of globin chains
Mutations in Sickle cell syndromes are:
II (A) α Thalassemias Defective biosynthesis
• A partially acceptable missense mutation in sequence of codon
of α globin chains
6 of β globin chain.
II (B) β Thalassemias Defective biosynthesis • GAG to GTG, So A to T mutation hence an example of transversion.
of β globin chains • The sixth amino acid in β globin chain is changed from glutamic
acid to valine
III Thalassemic hemoglobin vari- Structurally abnormal
ants Hb associated with co- • A polar amino acid is replaced by a nonpolar amino acid, so an
example of Nonconservative mutation.
m

inherited thalassemic
phenotype
Pathological changes due to mutation are:
co

III (A) HbE β26 Glu→Lys • HbS polymerizes reversibly when deoxygenated to
form a gelatinous network of fibrous polymers that
Contd... stiffen the RBC membrane
e

e
m

m
m
co

396 |  
Self Assessment and Review of Biochemistry

• There is increased viscosity Contd...


• There is dehydration due to potassium leakage and Condition HbA1 % HbH(β4) %
calcium influx Hydrops fetalis All the four loci de- 0 5–10
• These changes also produce the sickle shape, or holly leted
leaf shape NB: 90–95% is Hb Barts (γ4)
• Sickled cells lose the pliability needed to traverse
Mutations in Alpha and β Thalassemia
m

small capillaries
α Thalassemia Unequal crossing-over
co

• They possess altered ‘sticky’ membranes that are


Large deletions
abnormally adherent to the endothelium of small Less commonly nonsense and frameshift mutations
venules
β Thalassemia Deletions
• These abnormalities provoke unpredictable episodes Nonsense and frameshift mutations, Splice sites
of microvascular vasoocclusion and premature RBC and promoter mutations
destruction (hemolytic anemia).
Remember
Thalassemia Syndromes • Hb H is β4 tetramer
• Hb Barts is γ4 tetramer
Alpha Thalassemia Syndromes
As alpha chain is encoded by two tightly linked gene clus- Novel treatments for Hemoglobinopathies
For Sickle Cell Anemia:
ters and not by a single gene as in the case of β globin gene.
• Direct gene correction in situ (Genomic editing) using Zn finger
Deletion in each gene loci is associated with a syndrome.
m

nucleases or CRISPR Cas 9


• De repressing HbF by interfering with Bcl 11a.
co

Condition HbA1 % HbH(β4) %


Silent 1 α gene loci deleted 98–100 0 For Sickle cell and Thalassemia Syndromes
Thalassemia (-α/αα) • Bone marrow transplantation to provide stem cells used in large
number of β Thalassemia and a few cases of sickle cell anemia
Thalassemia 2 α gene loci deleted 85–95 Rare blood • Gene therapy
trait (-α/-α)or(--/αα) cell inclusion
• Re-establishing high level of HbF by stimulating proliferation of
HbH disease 3α gene loci deleted 70–95 5–30 primitive Hb F producing progenitor cells (i.e. F cell progenitors)
(--/-α) by Cytarabine, Hydroxy Urea, Pulsed or intermittent administration
of Butyrates.
Contd...

REVIEW QUESTIONS
m
co

Heme Synthesis and Porphyria 2. In lead poisoning which of the following is seen
in urine: (AIIMS Nov 2015)
1. Heme biosynthesis do not occur in: a. Delta ALA
(AIIMS Nov 2015)
b. Uroporphyrin
a. Osteocyte
c. Coproporphyrin
b. Liver
d. Protoporphyrin
c. RBC
Ans. a. Delta ALA
d. Erythroid cells of Bone marrow
Lead inhibit ALA Dehydratase. So delta ALA increases,
Ans. c. RBC Ref: Harper 30/e p325
hence found in urine.
Site: Synthesized in almost all tissues in the body
EXCEPT in mature erythrocytes. 3. In HbS, Glutamic acid replaced by valine. What
85% in erythroid Precursor cells in the bone marrow and will be its electrophoretic mobility?
m

majority of remainder in hepatocyte (AIIMS Nov 2015)


co

Organelle: Partly cytoplasmic and partly mitochondrial a. Increased


Starting Materials: Succinyl CoA and Glycine. b. Decreased
e

e
m

m
m
co

Heme Metabolism and Hemoglobins   | 397

c. No change c. Uroporphyrinogen Decarboxylase


d. Depends on level of concentration of HbS d. Uroporphyrinogen Synthase
Ans. b. Decreased Ans. a. Protoporphyrinogen Oxidase
In HbS, Glutamic acid is replaced by Valine. Glutamic acid 8. No. of iron in transferrin: (NBE Pattern Q)
is negatively charged and Valine is neutral. Movement a. 1
of Hb in electrophoresis depends on charge. More the b. 2
m

negative charge, faster the mobility towards the anode.


c. 3
So HbS with less negative charge lags behind HbA1.
co

d. 4
4. Which of the following porpyrias does not Ans. b. 2
present with photosensitivity: (AIIMS may 2012) Transferrin
a. Urophorphyrin decarboxylase • Transport form of Iron
b. HMB Synthase • Has two iron binding state.
c. Protophophrinogen oxidase 9. No. of iron in Ferritin: (NBE Pattern Q)
d. Coproproporphyrinogen oxidase a. 4
Ans. b. HMB Synthase b. 40
• Porphyrias that present with Neurovisceral symp- c. 400
toms are Acute Intermittent Porphyria and ALA d. 4000
Dehydratase deficient Porphyria. Ans. d. 4000
m

• Porphyrias that present with Cutaneous Photosen-


Ferritin
co

sitivity are Congenital Erythropoietic Porphyria,


Porphyria Cutanea Tarda, Erythropoietic Porphyria, • Storage Form of Iron
X linked Protoporphyria. • Poly nuclear complex of Hydrous ferric oxide
• Porphyria that presents with neurovisceral Symptoms • Carry about 4500 iron atoms
and Cutaneous Photosensitivity is Hereditary • Seen in Intestinal cells, Liver, Spleen and Bone
Coproporphyria, Variegate Porphyria. marrow
• Sensitive index of body iron stores
5. A boy with staining of teeth and raised Copropor-
phyrin-I levels and increased risk of photosensi- 10. No of pyrrole rings in Porphyrins:
tivity, the enzyme deficient is: (NBE pattern Q) a. 2 (NBE Pattern Q)
a. Uroporphyrinogen Synthase b. 3
b. Uroporphrinogen III Synthase c. 4
d. 5
m

c. Uroporphyrinogen Decarboxylase
Ans. c. 4
co

d. Coprophorphyrinogen Oxidase
Ans. a. Uroporphrinogen III Synthase Hemoglobin
This is a case of congenital Erythropoietic porphyria 11. Identify the structure given below: (NBE pattern Q)
Enzyme defect in CEP is Uroporphyrinogen III Synthase.

6. Acute Intermittent Porphyria is caused by:


(NBE pattern Q)
a. ALA synthase
b. ALA dehydratase
c. Ferrochelatase
d. Uroporphyrinogen I synthase
Ans. d. Uroporphyrinogen I Synthase
m
co

7. Variegate porphyria enzyme defect is:


(NBE pattern Q)
a. Protophorphyrin oxidase a. Porphyrin
b. Coproporphrinogen Oxidase b. Heme
e

e
m

m
m
co

398 |  
Self Assessment and Review of Biochemistry

c. Chlorophyll Jaundice
d. Pyrrole 13. A 10-year-old boy present with increased serum
Ans. a. Porphyrin bilirubin, increased bilirubin in urine and no
12. 2,3 DPG binds to . urobilinogen. Diagnosis is: (NBE pattern Q)
...................... sites in hemoglobin and a. Gilbert Syndrome
causes ............... in its oxygen affinity: b. Hemolytic jaundice
m

(AIIMS May 2014) c. Viral hepatitis


co

a. Four, increases d. Obstructive jaundice


b. Four, decreases Ans. d. Obstructive Jaundice
c. One, increases
d. One, decreases Laboratory tests in three different types of Jaundice

Ans. d. One and decreases Serum Urine


The Hemoglobin tetramer binds to one mol of 2,3 BPG Condition Bilirubin Urobilinogen Urine Bilirubin
in the central cavity formed by four subunits. Normal Direct: 0.1–0.4 0–4 mg/24 h Absent
2,3 BPG forms salt bridges with terminal amino group mg/dL
of both β globin chain via, Valine, Lysine and Histidine Indirect: 0.2–
Considering these two sentences, If central cavity is taken 0.7 mg/dL
into account 2,3 BPG binds to one site.
Hemolyti- Indirect Increased Absent
But actually 2,3 BPG is forming salt bridges with two
m

canemia
beta subunit, so correctly it is binding to two β sites.
co

Hepatitis Direct and in- Decreased if Present if micro-


So two is a better answer than four, for the number of
direct micro-obstruc- obstruction occurs
binding site. As it is not there in the option, better answer
tion is present
is one.
Obstructive Direct Absent Present
Binding of 2,3 BPG decreases the affinity of Oxygen
jaundice
towards Hb
• Hb F has low affinity towards 2, 3 BPG.
m
co
m
co
e

e
m

m
m
co

17 TCA Cycle and Biological


Oxidation
m
co

Topics Included
• TCA Cycle (Citric Acid Cycle/ Krebs Cycle) • High Energy Compounds
• Shuttle Mechanisms • Electron Transport Chain
• Inhibitors of Electron Transport System • Inhibitors of Electron Transport Chain
m
co

TCA CYCLE (CITRIC ACID Steps of TCA Cycle


CYCLE/KREBS CYCLE) Citrate Synthase
• Acetyl CoA + Oxaloacetate ----> Citrate
Definition • First Tricarboxylic Acid formed is Citrate (6C)
• Sequence of reactions in mitochondria that oxidizes • Irreversible StepQ (AIIMS June 2000)
the acetyl moiety of acetyl-CoA and reduces
• Citrate can cross the mitochondrial membrane and
coenzymes, that are reoxidized through the electron
release Acetyl CoA for the synthesis of Fatty Acid by
transport chain, linked to the formation of ATP
ATP Citrate Lyase.
• The citric acid cycle is the final common pathwayQ
for the oxidation of carbohydrate, lipid, and protein Aconitase (Aconitate Hydratase)
• Site: Mitochondria. • Citrate isomerized to Isocitrate
m

• Reversible reaction
co

Overview of TCA Cycle


• The reaction occurs in two steps: dehydration to cis-
aconitate and rehydration to isocitrate
• Inhibited noncompetitively by FluoroacetateQ
• Aconitase is a Lyase.

Isocitrate Dehydrogenase
• Isocitrate undergoes dehydrogenation catalyzed
by isocitrate dehydrogenase to form, initially,
oxalosuccinate
• Oxalosuccinate is decarboxylation to α –ketoglutarate
(5C)
m

• The decarboxylation requires Mg2+ or Mn2+ ions


co

• First Oxidative decarboxylation


• 1 NADH is formed
Fig. 17.1: Overview of citric acid cycle • Reversible reaction.
e

e
m

m
m
co

400 |  
Self Assessment and Review of Biochemistry

αKetoglutarate Dehydrogenase 5 Coenzymes of this enzyme are


• αKetoGlutarate (5C) oxidised and decarboxylated to 1. Lipomide
Succinyl CoA (4C) 2. Thiamine Pyrophosphate
• Second Oxidative Decarboxylatio 3. NAD+
• 1 NADH is formed 4. FAD
• Physiologically Unidirectional step 5. Coenzyme A
m

• Multienzyme Complex similar to Pyruvate • Alpha KetoGlutarate Dehydrogenase noncompeti-


co

Dehydrogenase tively inhibited by ArseniteQ.


m
co
m
co

Fig. 17.2: TCA cycle

Succinate ThiokinaseQ (Succinyl-CoA Synthetase) Succinate Dehydrogenase


• Convert Succinyl CoA to SuccinateQ • Succinate undergo dehydrogenation reaction,
m

• 1 ATP/GTP is generated forming fumarate


co

• GTP is generated in Gluconeogenic tissues like Liver • The enzyme contains FAD and iron–sulfur (Fe:S) protein
and Kidney • The enzyme directly reduces ubiquinone in the
• Substrate Level Phosphorylation. electron transport chain
e

e
m

m
m
co

TCA Cycle and Biological Oxidation   | 401

• Only enzyme in TCA cycle attached to Inner Concept of regulation of TCA cycle
mitochondrial Membrane • High energy states inhibit TCA Cycle and vice versa
• All other enzymes are in the mitochondrial matrix • High ATP/ADP ratio and High NADH/NAD+ ratio are inhibitors of
TCA Cycle.
• FADH2 is formed
• High ADP and High NAD+ are activators of TCA Cycle
• Succinate Dehydrogenase is competitively inhibited • Products of the pathway inhibit the regulatory enzymes
by Malonate.Q
m

Fumarase (Fumaratehydratase) Allosteric activators and inhibitors of individual


co

enzymes
• Catalyzes the addition of water across the double
bond of fumarate, yielding malate • Long Chain Acyl CoA and ATP inhibit Citrate
• Fumarase is a Lyase. Synthase
• Isocitrate Dehydrogenase is inhibited by ATP and
Malate Dehydrogenase NADH
• Final step in TCA Cycle • Succinate Dehydrogenase is inhibited by Oxaloacetate
• Malate is converted to Oxaloacetate • In Muscle, the dehydrogenases of TCA Cycle are
• Oxalo acetate is regenerated activated by Ca2+, which increases during muscle
• 1 NADH generated contraction
• Oxaloacetate regenerated, hence Oxaloacetate has a • Mitochondrial Isocitrate Dehydrogenase is activated
catalytic role like Ornithine in Urea Cycle. by ADP
m

Inhibitors of TCA CycleQ • In a tissue such as brain, which is largely dependent


co

• Aconitase noncompetitively inhibited by Flouroac- on carbohydrate to supply acetyl-CoA, control


etateQ of the citric acid cycle may occur at pyruvate
• Alpha Ketoglutarate Dehydrogenase noncompeti- dehydrogenase.
tively inhibited by ArseniteQ Remember
• Succinate Dehydrogenase is competitively inhibited To answer whether a compound is an activator or inhibitor of an
by Malonate.Q (Inhibitor of complex II of ETC). enzyme, think whether they are substrate or products of that enzyme/
pathway
Energetics of TCA CycleQ
TCA Cycle is truly an Amphibolic PathwayQ
Method of ATP No of ATP A pathway with both catabolic and anabolic role is called amphibolic
Reaction Production Generated pathway.
Isocitrate Dehydrogenase 1 NADH enter ETC 2.5 ATPs Anabolic Role of TCA Cycle
α KetoGlutarate Dehydro- 1 NADH enter ETC 2.5 ATPs • Citrate to Fatty Acid Synthesis
m

genase • Alphaketoglutarate to GABA & Glutamate


co

Succinate Thiokinase Substrate level Phos- 1 ATP • Succinyl CoA to Heme


phorylation • Oxaloacetate to Gluconeogenesis
Succinate Dehydrogenase 1 FADH2 enter ETC 1.5 ATPs Catabolic role of TCA cycle
Malate Dehydrogenase 1 NADH enter ETC 2.5 ATPs • Acetyl CoA is completely oxidized to CO2.
Total number of ATP per turn 10 ATPs
of TCA Cycle Anaplerotic Reactions of TCA CycleQ
The 6 Carbon, 5 Carbon and 4 Carbon intermediates are used for
Three molecules of NADH Q and one of FADH2 are various synthetic or anabolic reactions mentioned above. So these
produced for each molecule of acetyl-CoA catabolized intermediates get depleted.
To replenish these compounds filling reactions takes place. These
in one turn of the cycle.
filling up reactions are called Anaplerotic reactions.
Regulation of TCA Cycle Filling up (Anaplerotic) Reactions are at the level of Oxaloac-
etate (6C)
Regulatory steps are • Hydroxy Proline, Serine, Cysteine, Threonine, Glycine to Pyruvate
m

• Citrate Synthase • Lactate to Pyruvate


• Isocitrate Dehydrogenase
co

• Tryptophan to Alanine to Pyruvate


• α Ketoglutarate Dehydrogenase • Pyruvate to Oxaloacetate by Pyruvate CarboxylaseQ, is a major
filling up reaction
• Pyruvate dehydrogenase is also considered as the
regulatory step of TCA Cycle. Contd...
e

e
m

m
m
co

402 |  
Self Assessment and Review of Biochemistry

Contd... Glycerophosphate Shuttle


Remember This shuttle is present in some tissues (e.g., brain, white
• Acetyl CoA is a positive Allosteric effector of Pyruvate muscle), but absent in heart tissue.
Carboxylase
The number of ATPs from 1 NADH transported to mitochondria
At the level of Alpha Ketoglutarate (5C)
by Glycerophosphate Shuttle
• Glutamine and Glutamate are the major anaplerotic substrates of
Since the mitochondrial Glycerophosphate Dehydrogenase is linked
Alpha Ketoglutarate
m

to the respiratory chain via a flavoprotein (FAD) rather than NAD, only
At the level of Succinyl CoA (4C) 1.5 mol rather than 2.5 mol of ATP are formed per atom of oxygen
co

• Valine, Isoleucine and Methionine consumed.


• Compounds that form Propionyl CoA
At the level of Fumarate (4C)
• Tyrosine and Phenyl Alanine.
m

Fig. 17.4: Glycerophosphate shuttle


co

Malate Shuttle
Malate shuttle system is of more universal utility. Used
to transport NADH from Cytosol to Mitochondria.

Reactions involved in Malate Shuttle


• NADH converted to NAD+, Oxaloacetate to Malate
• Malate enter mitochondria via α Ketoglutarate
Transporter
• Malate converted to Oxaloacetate, NADH is released
• But there is no transporter for Oxaloacetate
m

• Oxaloacetate react with glutamate to form aspartate


and (α-ketoglutarate by Transamination)
co

Fig. 17.3: Anaplerotic reactions of TCA Cycle


• Aspartate and α Ketoglutarate is transported to
Vitamins in TCA Cycle cytosol and Oxaloacetate is reconstituted.
• Pantothenic Acid as a part of CoA
• Riboflavin as FAD
• Thiamin
• Niacin as NAD+

SHUTTLE SYSTEMS
• NADH cannot penetrate the mitochondrial
membrane, but it is produced continuously in the
m

cytosol by 3-phosphoglyceraldehyde dehydrogenase,


co

an enzyme in the glycolysis sequence


• The transfer of reducing equivalents is carried out by
using the various shuttle systems Fig. 17.5: Malate shuttle
e

e
m

m
m
co

TCA Cycle and Biological Oxidation   | 403

Creatine Phosphate Shuttle • CKm (An isoenzyme of creatine kinase (CKm) is


• Facilitates Transport of High-Energy Phosphate (e.g. found in the mitochondrial intermembrane space)
ATP) from Mitochondria transfer high-energy phosphate from ATP to creatine
• Sites are heart and skeletal muscle. form Creatine Phosphate
• The creatine phosphate is transported into the cytosol
Reactions of Creatine Phosphate Shuttle via protein pores in the outer mitochondrial mem-
m

• ATP emerging from the adenine nucleotide trans- brane, Creatine Kinase generate extramitochondrial
porter to intermembrane space ATP.
co
m
co
m
co

Fig. 17.6: Creatine phosphate shuttle

Redox Potential of Common Redox Couples Contd...


Electrons are transferred in the ascending order of redox Redox couple Redox potential
couple. Fumarate/Succinate +0.03
Redox couple Redox potential Cyt b; Fe3+/Fe2+ +0.08
H+/H2 -0.42 Ubiquinone(CoQ) +0.10
NAD+/NADH -0.32 Cyt c1; Fe3+/Fe2+ +0.22
Lipoate -0.29 Cyt a; Fe3+/Fe2+ +0.29
m

Acetoacetate/β Hydroxybutyrate -0.27 Oxygen/water +0.82


co

Pyruvate/Lactate -0.19
Remember this table is important for national board
Oxaloacetate/Malate -0.19
pattern of exams. It is important to learn the order in
Contd... which they are arranged, not the value of redox potential.
e

e
m

m
m
co

404 |  
Self Assessment and Review of Biochemistry

ELECTRON TRANSPORT CHAIN


m
co

Fig. 17.7: Flow of electrons in ETC

Components of Electron Transport Chain ‒ 2H+ pumped to Intermembrane Space


m

Site: In the Inner Mitochondrial Membrane ‒ The final electron acceptor of ETC is oxygen.
co

Components of the Electron Transport Chain Q are


contained in four large protein complexes Mobile Complexes in the
• Complex I NADH Coenzyme Q Oxidoredutase
Electron Transport Chain
‒ Contain FMN and Fe-S (Iron- Sulfur) Complex Coenzyme Q
‒ Pumps 4 H+Q to Intermembrane Space (PGI Nov • Also called Ubiquinone
09 May 10) • Quinone derivative with a polyisoprenoid side chain
• Complex II Succinate Q Reductase • Lipid solubility and small size make it a mobile
‒ Contain FAD and FeS Complex electron carrier.
‒ No H+ pumped to Intermembrane Space
• Complex III Q Cytochrome c Oxidoreductase Cytochrome C
‒ Contain Cyt b and Cyt c1 • Mobile electron carrier between Complex III and
Complex IV
m

‒ Contain Fe-S Complex


• Also play a role in programmed cell death.
co

OXIDATIVE PHOSPHORYLATION
The flow of electrons through the respiratory chain
generates ATP by the process of oxidative phosphorylation.
Oxidation coupled with Phosphorylation.
The theory behind the oxidative Phosphorylation is the
Chemiosmotic theory.

The Chemiosmotic TheoryQ


• Proposed by Peter Mitchell in 1961
• Postulates that the two processes, oxidation and
m

Fig. 17.8: Components of electron transport chain


Phosphorylation are coupled by a proton gradient
co

Q
Pumps 4 H+ to Intermembrane Space (PGI Nov 09 May 10) across the inner mitochondrial membrane
• Complex IV Cytochrome c Oxidase • The proton motive forceQ caused by the electrochemi-
‒ Contain Cyt a and a3 (now known as Heme a a3) cal potential difference (negative on the matrix side)
and Copper A and Copper B centre drives the mechanism of ATP synthesis.
e

e
m

m
m
co

TCA Cycle and Biological Oxidation   | 405

COMPLEX V–ATP SYNTHASE COMPLEX


• Also called as the Fifth Complex of Electron transport
Chain
• The smallest molecular motor present in the human
body
• Location-ATP synthase is embedded in the inner
m

mitochondrial membrane.
co

Divided into two Subcomplexes


• F0 Subcomplex
• F1 Subcomplex.
F0 Subcomplex
• Hydrophobic in nature
• F0 spans the inner mitochondrial membrane
• Forms a proton channel
• Made up of a disk of 10 ‘C’ protein subunits. Fig. 17.9: ATP synthase
m
co
m
co

Fig. 17.10: Inhibitors of ETC

F1 Subcomplex ATP Synthase Complex as a rotor-stator molecular


• Hydrophilic in nature motor
• Projects into the mitochondrial matrix • Because it has two functional unit.
• F1 is attached to F0 Subcomplex A rotating subunit
• Made up 9 Subunits (α3β3γδε) • Consist of F0 Complex and γ Subunit of F1 Complex
• γ subunit in the form of a ‘bent axle.’ A stationary subunit
• γ subunit is surrounded by 3α and 3β subunit
• F1 Complex other than γ subunit.
alternatively
• The flow of protons through F0 causes, rotation of F0
m

Complex along with γ subunit of F1 complex to rotate Binding Change Mechanism


co

• This causes the production of ATP in the F1 • The theory behind the ATP production in the β
complex subunit of F1 Subcomplex
• β subunit of F1 Complex is called Catalytic Subunit. • Proposed by Paul Boyer
e

e
m

m
m
co

406 |  
Self Assessment and Review of Biochemistry

• States that re-entry of protons through F0 Subcomplex • Dinitrocresol


causes rotation of γ subunit which in turn causes • FCCP [Fluoro Carbonyl Cyanide Phenyl hydrazine]
conformational changes in the β subunits of F1 • ? Aspirin in high dose.
Subcomplex.
Physiological Uncoupler
INHIBITORS OF ELECTRON • Thermogenin [Uncoupling Protein 1] in Brown
TRANSPORT CHAIN Adipose Tissue
m

• Thyroxine
co

Divided into • Long Chain Free Fatty Acid


• Inhibitors of Electron transfer
• ? Unconjugated Bilirubin.
• Inhibitors of Oxidative Phosphorylation
• Uncouplers of Oxidative Phosphorylation Ionophores
• Ionophores
• Ionophores permit specificcations to penetrate
membranes
Inhibitors of Electron Transfer
• Dissipate Proton Gradient
Between NADH and CoQ [At Complex I]
• Valinomycin
• An insecticide and a fish Poison Rotenone
• Gramicidin
• Amobarbital which is a barbiturate
• Nigercin.
• Piericidin A.
P:O Ratio
m

Inhibitor of Complex II
• Represents the number of ATP molecules produced
co

• TTFA (Tri enoyl TriFluoroAcetone) a Fe2+ Chelating in terms of reducing equivalents oxidized
agent
• No of inorganic Phosphates utilized for ATP
• Carboxin production for every atom of oxygen consumed
• Malonate, a competitive inhibitor of Succinate • For NADH - 2.5
Dehydrogenase.
• For FADH2 -1.5.
Between Cyt b and Cyt c [At Complex III]
High Energy CompoundsQ
• Antimycin A
• Compounds which yield energy of atleast 7 kcal/m
• British Antilevisite [Dimercaprol]
on hydrolysis
Inhibitor at Cytochrome c Oxidase [Complex IV] • Compounds whose free energy of hydrolysis more
• CO than that of ATP is called High energy phosphates
• Cyanide • Compounds whose free energy of hydrolysis less
m

• H2S than that of ATP is called low energy phosphates.


co

• Sodium Azide. Classification of High energy Phosphates


• Pyrophosphate, e.g. ATP
Inhibitors of Oxidative Phosphorylation
• Acyl Phosphate, e.g. 1,3 Bisphosphoglycerate
Atractyloside • EnolPhosphate, e.g. Phosphoenolpyruvate
By inhibiting the transporter of ADP into and ATP out of • Thioester, e.g. Acetyl CoA, Succinyl CoA
the mitochondrion. • Phosphagen, e.g. Phosphocreatine, Phosphoarginine.
Remember
Oligomycin an Antibiotic
• All high energy compounds given yield energy higher than ATP.
Completely blocksQ oxidation and phosphorylation. • Most of the compound contain Phosphate group (hence also called
By blocking the flow of protons through F0 Complex of High Energy Phosphates) except Acetyl CoA.
ATP Synthase.
m

Free Energy kJ/ Free Energy


Uncoupler of Oxidative Phosphorylation High Energy Compound mol Kcal/mol
co

Mechanism of Action—Disruption of Proton Gradient Phosphoenolpyruvate -61.9 -14.8


across the inner mitochondrial membrane Carbamoyl phosphate -51.4 -12.3
• 2,4 Dinitrophenol Contd...
e

e
m

m
m
co

TCA Cycle and Biological Oxidation   | 407

Contd... Respiratory QuotientQ


Free Energy kJ/ Free Energy Measurement of the ratio of the volume of carbon dioxide
High Energy Compound mol Kcal/mol produced: volume of oxygen consumed in the oxidation
1,3-Bisphosphoglycerate -49.3 -11.8 of metabolic fuels.
(to 3-phosphoglycerate)
RQ (CO2
Creatine phosphate -43.1 -10.3 Energy Produced/
m

ATP→AMP + PPi -32.2 -7.7 Yield O2 Con- Energy


Metabolic fuel (kJ/g) sumed (kJ)/L O2
co

ATP → ADP + Pi -30.5 -7.3


Carbohydrate 16 1.00 20
Glucose-1-phosphate -20.9 -5.0
PPi -19.2 -4.6 Protein 17 0.81 20
Fructose-6-phosphate -15.9 -3.8 Fat 37 0.71 20
Glucose-6-phosphate -13.8 -3.3
Alcohol 29 0.66 20
Glycerol-3-phosphate -9.2 -2.2

REVIEW QUESTIONS
m

TCA Cycle a. Malonate


b. Arsenite
co

1. Which of the following is not an intermediate of


c. Fluoroacetate
TCA Cycle? (AIIMS 2014 May)
a. Acetyl CoA d. Fumarate
b. Citrate Ans. c. Fluoroacetate. (Ref: Harper 30/e p161-167)
c. Succinyl CoA • The poison fluoroacetate is found in some of plants,
d. Alpha Ketoglutarate and their consumption can be fatal to grazing animals
Ans. a. Acetyl CoA. (Ref: Harper 30/e p161-167) • Some fluorinated compounds used as anticancer
agents and industrial chemicals (including pesticides)
Acetyl CoA and Oxaloacetate are the starting materials
are metabolized to fluoroacetate
of TCA Cycle
• It is toxic because fluoroacetyl-CoA condenses with
2. Which of the following is true about Krebs Cycle? oxaloacetate to form fluorocitrate, which inhibits
(JIPMER May 2015) aconitase, causing citrate to accumulate.
m

a. Pyruvate condenses with Oxaloacetate to form


co

Citrate 4. First substrate of Krebs cycle is:


(AIIMS May 2007)
b. Alpha ketoglutarate is a five Carbon compound
a. Pyruvate
c. Oxidative Phosphorylation occurs in the cyto-
b. Glycine
plasm only
d. Krebs cycle can operate in anaerobic condition c. HCI
Ans. b. Alpha ketoglutarate is a five Carbon compound. d. Lipoprotein
Ref: Harper 30/e p161-167 Ans. a. Pyruvate. (Ref: Harper 30/e p161-167)
• Acetyl CoA condenses with Oxaloacetate to form From these options best answer is Pyruvate.
Citrate 5. Hyperammonemia inhibits TCA cycle by
• Oxidative phosphorylation occur in mitochondria depleting: (PGI June 2009)
by ETC a. Oxaloacetate
m

• Krebs cycle cannot operate in anaerobic condition. b. Alpha ketoglutarate


co

3. Which of the following substance binds to CoA c. Citrate


and condenses oxaloacetate to inhibit the TCA d. Succinyl CoA
cycle: (AIIMS Nov 2010) e. Fumarate
e

e
m

m
m
co

408 |  
Self Assessment and Review of Biochemistry

Ans. b. Alpha ketoglutarate. (Ref: Harper 30/e p168) a. Atractyloside


Hyperammonemia, as occurs in advanced liver disease b. Oligomycin
and a number of (rare) genetic diseases of amino acid c. Rotenone
metabolism, leads to loss of consciousness, coma and d. Cyanide
convulsions, and may be fatal. This is because of the
Ans. a. Atractyloside. (Ref: Harper 30/e p132, 133)
withdrawal of α ketoglutarate to form glutamate
• Atractyloside inhibits oxidative phosphorylation by
(catalyzed by glutamate dehydrogenase) and then
m

glutamine (catalyzed by glutamine synthetase), leading inhibiting the transporter of ADP into and ATP out
co

to reduced concentrations of all citric acid cycle of the mitochondrion


intermediates, and hence reduced generation of ATP. • The antibiotic oligomycin completely blocks
oxidation and phosphorylation by blocking the flow
6. What is liberated when Citrate converted to Cis of protons through ATP synthase
Aconitate? (NBE pattern Q) • Barbiturates such as amobarbital, Rotenone and
a. H2O Piericidin A inhibit electron transport via Complex I
b. H2 • Antimycin A and dimercaprol inhibit the respiratory
c. H2O2 chain at Complex III. The classic poisons H2S, carbon
d. CO2 monoxide, and cyanide inhibit Complex IV and can
Ans. a. H2O. (Ref: Harper 30/e p163) therefore totally arrest respiration. Malonate is a
• Citrate isomerized to Isocitrate by Aconitase competitive inhibitor of Complex II.
m

Reversible reaction
10. The electron flow in cytochrome C oxidase can be
co

• The reaction occurs in two steps: dehydration to cis-


blocked by: (AIIMS May 2006)
aconitate and rehydration to isocitrate.
a. Rotenone
7. False about reducing equivalents is: b. Antimycin-A
(NBE pattern Q) c. Cyanide
a. They are NADH and NADPH d. Actinomycin
b. Only produced during primary metabolic Ans. c. Cyanide. (Ref: Harper 30/e p132,133)
pathway Cytochrome Oxidase is inhibited by CO, HCN, H2S and
c. Formed in TCA cycle Na Azide.
d. Formed in mitochondria
Ans. b. Only produced during primary metabolic 11. Cytosolic Cytochrome C mediates:
        pathway. (AIIMS May 2006)
m

a. Apoptosis
8. High energy phosphate is not produced in:
co

b. Electron transport
a. TCA cycle c. Krebs cycle
b. Hexose Mono Phosphate pathway d. Glycolysis
c. Glycolysis Ans. a. Apoptosis. (Ref: Harper 30/e p127-130)
d. Beta Oxidation of Fatty Acid Mitochondrial Cytochrome c is a mobile electron carrier
Ans. b. HMP Pathway. in Electron Transport Complex. This also mediates
Pathways which do not synthesize ATP are Apoptosis.
• HMP Pathway
• Rapaport Leubering Cycle 12. High energy compounds is/are: (PGI May 2012)
• Uronic acid pathway a. ATP
• Alpha oxidation of Fatty acid b. Creatine Phosphate
c. Glucose 1 Phosphate
m

• Omega Oxidation of fatty acid


d. Glycerol 3 Phosphate
co

Electron Transport Chain e. ADP


9. Transport of ADP in and ATP out of mitochondria Ans. a. ATP, b. Creatine Phosphate.
is inhibited by: (Nov 2010) (Ref: Harper 30/e p116)
e

e
m

m
m
co

TCA Cycle and Biological Oxidation   | 409

Free Energy Free Energy Ans. c. Brown adipose tissue.


High Energy Compound kJ/mol Kcal/mol Brown adipose tissue contains thermogenin, which is a
Phosphoenolpyruvate -61.9 -14.8 physiological uncoupler of oxidative phosphorylation.
Carbamoyl phosphate -51.4 -12.3 This process is called Nonshivering Thermogenesis.
1,3-Bisphosphoglycerate (to -49.3 -11.8
3-phosphoglycerate) 16. Electron transport chain involves all except:
(Kerala 2011)
m

Creatine phosphate -43.1 -10.3


a. NADP
co

ATP→AMP + PPi -32.2 -7.7


ATP → ADP + Pi -30.5 -7.3 b. NAD
Glucose-1-phosphate -20.9 -5.0 c. Coenzyme Q
PPi -19.2 -4.6 d. FAD
Fructose-6-phosphate -15.9 -3.8 Ans. a. NADP.
Glucose-6-phosphate -13.8 -3.3
NADP is involved in reductive biosynthesis, not in ETC.
Glycerol-3-phosphate -9.2 -2.2
17. F0-F1 Complex, ATP synthase inhibitor is:
13. In ETC, oxidative phosphorylation (ATP (Kerala 2007)
formation) is regulated by: (PGI MAY 2011) a. Atractyloside
a. NADH CoQ reductase b. Oligomycin
b. Cytochrome C oxidase c. Antimycin
m

c. Glutathione reductase d. Rotenone


co

d. Isocitrate dehydrogenase Ans. b. Oligomycin. (Ref: Harper 29/e p127)


e. CoQ Cytochrome C reductase
• Atractyloside inhibits oxidative phosphorylation by
Ans. a, b, c, e. (Ref: Harper 30/e p130, 133) inhibiting the transporter of ADP into and ATP out
Components of the Electron Transport Chain Q are of the mitochondrion
contained in four large protein complexes:
• The antibiotic oligomycin completely blocks
• Complex I NADH CoQ Oxidoredutase oxidation and phosphorylation by blocking the flow
• Complex II CoQ Succinate Reductase of protons through ATP synthase.
• Complex III CoQ Cytochrome c Oxidoreductase
• Complex IV Cytochrome C Oxidase. 18. Respiratory Quotient 0.7 is seen in:
(NBE pattern Q)
14. Which component transfer four protons:
a. Carbohydrates
a. NADH-Q Oxidoreductase (PGI Nov 2009)
m

b. Fat
b. Cytochrome-C Oxidase
co

c. Protein
c. Cytochrome C–Q oxidoreductase
d. Isocitrate Dehydrogenase d. Alcohol
e. Succinate Q Reductase Ans. b. Fat.
Ans. a. NADH-Q Oxidoreductase, c. Cytochrome C–Q Respiratory Quotient
        oxidoreductase. (Ref: Harper 29/e p130) Measurement of the ratio of the volume of carbon dioxide
• Complex I and III pumps 4 H+ produced: volume of oxygen consumed
• Complex II pumps no protons (Respiratory Quotient, RQ) is an indication of the
• Complex IV pumps 2 H+. mixture of metabolic fuels being oxidized.
15. The specialized mammalian tissue/organ in RQ (CO2
which fuel oxidation serves not to produce ATP Metabolic Energy Yield produced/O2 Energy
fuel (kJ/g) consumed (kJ)/L O2
but to generate heat is: (AI 2006)
m

Carbohydrate 16 1.00 20
a. Adrenal gland
co

Protein 17 0.81 20
b. Skeletal music
c. Brown adipose tissue Fat 37 0.71 20

d. Heart Alcohol 29 0.66 20


e

e
m

m
m
co

410 |  
Self Assessment and Review of Biochemistry

19. Phenobarbitone inhibits which complex of ETC: Dinitrophenol is an uncoupler of Oxidative Phospho-
(NBE pattern Q) rylation. So no ATP synthesis but electron transfer and
a. Complex I oxidation of reducing equivalents takes place.
b. Complex II
21. Mechanism of Cyanide poisoning: (NBE pattern Q)
c. Complex III a. Inhibition of Cytochrome Oxidase
d. Complex IV b. Inhibition of Carbonic Anhydrase
m

Ans. a. Complex I. (Ref: Harper 30/e p133) c. Inhibition of Cytochrome c


co

Inhibitors of ETC at Complex I d. Inhibition of ATP Synthase


• An insecticide and a fish Poison Rotenone Ans. a. Inhibition of Cytochrome Oxidase.
• Amobarbital which is a barbiturate (Ref: Harper 30/e p133)
• Piericidin. Inhibitors of Complex IV are CO, Cyanide, H2S, Sodium
Azide.
20. Dinitrophenol inhibits the electron transport
chain by: (NBE Pattern Q) 22. Final acceptor of electrons in ETC is:
a. Cytochrome b (AIIMS 2014 May)
b. Inhibit ATP synthesis and electron transport a. Cyt c
chain b. Oxygen
c. Inhibits ATP synthesis but not electron c. FADH2
m

transport chain d. CoQ


co

d. Inhibits electron transport chain but not ATP Ans. b. Oxygen. (Ref: Harper 30/e p133)
synthesis • Electrons are transferred in the ascending order of
Ans. c. Inhibit ATP synthesis and but not electron redox potential, the final oxygen electron acceptor
        transport chain. is oxygen.
m
co
m
co
e

e
m

m
m
co

18 Free Radicals, Xenobiotics


and Metabolism of Alcohol
m
co

Topics Included
• Definition and Generation of Free Radicals
• Measurement of Body Free Radical Burden
• Free Radical Scavenging System
m
co

FREE RADICALS This reaction is called Dismutation reaction. This can


occur spontaneously or as enzyme catalyzed reaction.
Definition Remember
Free radical is molecule or molecular fragment that Hydrogen Peroxide is not a free radical as it does not have an
contains one or more unpaired electrons in its outer orbit unpaired electron, but it is a Reactive oxygen species.
and has an independent existence.
• Hydroxy Radical: The three electron reduction
A free radical is designated by a superscript dot (R•). product of Oxygen is Hydroxy radical (OH•). This
Generation of Oxygen Free Radicals (OFR) is the most powerful oxygen free radical.
or Reactive Oxygen Species (ROS) Free Radicals generated by Iron
The electron is transferred from a Ferrous ion to Hydrogen Peroxide.
Incomplete reduction of Oxygen This reaction is called Fenton reaction.
m

In the body oxidative reactions normally ensures that H2O2 + Fe2+ OH–2 + OH● + Fe3+
co

molecular oxygen is completely reduced to water. Another iron catalyzed reaction for generation of hydroxyl radical
Normally, four electrons are transferred to molecular is Haber Weiss Reaction.
oxygen so that it is completely reduced to form a water Fe2+
molecule. O2– + H2O2 O2 + OH● + OH–
       4H +
4e– + O2 2H2O
Incomplete reduction of oxygen generates Oxygen free
radicals or Reactive Oxygen Species. They are Superoxide,
Hydrogen Peroxide and Hydroxy radical.
• Superoxide Radical are produced when a single Fig. 18.1: Reactive oxygen species (ROS)
electron is transferred to oxygen. It is both an anion
Points to remember
and free radical. • Most powerful oxygen free radical is Hydroxyradical (OH●)Q
m

•O + e– •O – (Superoxide)
2 2 • Least powerful Reactive Oxygen species is Hydrogen peroxide
co

• Hydrogen Peroxide: The two electron reduction (H2O2).


product of oxygen is hydrogen peroxide (H2O2) • Precursor of all reactive Oxygen species is Superoxide radical
(●O2–)
2H+ • H2O2 is not a free radical but a ROS
•O + e– H2O2 (Hydrogen peroxide)
2
e

e
m

m
m
co

412 |  
Self Assessment and Review of Biochemistry

Common sources of free radicals in the body Radical damage to unsaturated fatty acids in cell memb-
• Electron leakage in mitochondrial Electron Transport Chain. ranes and plasma lipoproteins leads to the formation of
• Normal Oxidation reduction reactions in the body lipid peroxides, then highly reactive dialdehydesQ that
– Xanthine Oxidase, Aldehyde Oxidase, dihydroorotate can chemically modify proteins and nucleic acid bases.
Dehydrogenase
– Flavin Coenzymes in Peroxisomes generate H2O2.
– L- Amino Acid Oxidase (Coenzyme-FMN)
m

– D-Amino Acid Oxidase (Coenzyme-FAD)


co

• Respiratory Burst

Respiratory Burst Fig. 18.2: Respiratory burst


NADPH Oxidase in the inflammatory cells (neutrophils,
eosnophils, monocytes and macrophages) produces MEASUREMENT OF BODY FREE
superoxide anion by a process of respiratory burst during RADICAL BURDEN
phagocytosis.
The total body radical burden can be estimated by
Steps of Respiratory Burst (See fig 18.2)
measuring the products of lipid peroxidation.
• The enzyme NADPH Oxidase catalyzes the formation
• FOX (Ferrous Oxidation in Xylenol) Assay
of Superoxide radical from oxygen and NADPH
‒ Lipid peroxides can be measured by the ferrous
• By Dismutation Hydrogen Peroxide is generated
oxidation in xylenol orange (FOX) assay. Under
m

• Hydrogen Peroxide generate Hydroxy radical acidic conditions, they oxidize Fe2+ to Fe3+, which
co

spontaneously forms a chromophore with xylenol orange.


• Hydrogen peroxide generate Hypochlorous acid by • Estimation of Dialdehydes
the action of enzyme Myeloperoxidase exclusively ‒ The dialdehydes formed from lipid peroxides can
present in the neutrophil granules be measured by reaction with thiobarbituric acid,
• They mediate killing of bacteria. This is called when they form a red fluorescent adduct. The
respiratory burst. results of this assay are generally reported as total
Damages produced by free radicals thiobarbituric acid reactive substances, TBARS.
• Free radicals are highly reactive molecular species • Measurement of Pentane and Methane in Exhaled
with an unpaired electron air.
• They persist for only a very short time (of the order ‒ Peroxidation of ω-6 polyunsaturated fatty acids
of 10–9 to 10–12 sec) before they collide with another leads to the formation of pentane, and of-ω3
m

molecule and either abstract or donate an electron in polyunsaturated fatty acids to ethane, both of
which can be measured in exhaled air.
co

order to achieve stability


• In so doing, they generate a new radical from the
molecule with which they collided FREE RADICAL SCAVENGING SYSTEM
• The main way in which a free radical can be quenched, Antioxidants fall into two classes:
so terminating this chain reaction, is if two radicals • Preventive antioxidants, which reduce the rate of
react together, when the unpaired electrons can chain initiation
become paired in one or other of the parent molecules ‒ They are Glutathione Peroxidase, Catalase
• They cause damage to nucleic acids, proteins, and • Chain-breaking antioxidants, which interfere with
lipids in cell membranes and plasma lipoproteins chain propagation
• This can cause cancer, atherosclerosis and coronary ‒ They are Superoxide Dismutase, Uric Acid,
artery disease, and autoimmune diseases. Vitamin E.
m

Lipid Peroxidation Preventive Antioxidants


co

Lipids are most susceptible to damaging effects of free Glutathione Peroxidase


radicals. PUFA present in cell membrane and plasma • Eliminates hydrogen peroxide and organic hydrop-
lipoproteins are especially prone to damage. eroxides by reaction with reduced Glutathione (GSH)
e

e
m

m
m
co

Free Radicals, Xenobiotics and Metabolism of Alcohol


  | 413

• Reduced Glutathione is converted to oxidized stable radicals that persist long enough to undergo
glutathione (GSSG) reaction to nonradical products.
• Glutathione Reductase convert oxidized Glutathione Antioxidants can be prooxidants called as antioxidant
back to reduced Glutathione using the reducing paradox
equivalent NADPH
• Ascorbate, can also be a source of superoxide radicals
• Glutathione Peroxidase is a selenium containing by reaction with oxygen, and hydroxyl radicals by
m

enzyme. reaction with Cu2+ ions


co

• β-carotene is indeed a radical-trapping antioxidant


under conditions of low partial pressure of oxygen,
as in most tissues, at high partial pressures of oxygen
(as in the lungs) and especially in high concentrations,
Fig. 18.3: Free radical scavenging by glutathione peroxidase β-carotene is an autocatalytic prooxidant
• Increased mortality among those taking supplements
Catalase of Vitamin E.
• It is a hemoprotein with four heme groups. Causes
decomposition of peroxides to yield water and XENOBIOTICS
oxygen
A xenobiotic (Gk xenos ‘stranger’) is a compound that is
• Highest concentration of Catalase is present in
foreign to the body.
Peroxisomes.
m

                  Catalase Metabolism of xenobiotics in two phases


co

2H2O2 2H2O + O2 • In phase 1 reactions, xenobiotics are generally


Remember converted to more polar, hydroxylated derivatives
H2O2 is not free radical but can generate free radical. • In phase 2 reactions, these derivatives are conjugated
with molecules such as glucuronic acid, sulfate, or
Chain Breaking Antioxidant glutathione. This renders them even more water-
soluble, and they are eventually excreted in the urine
Superoxide Dismutase or bile.
It is the only enzyme that takes a free radical as its
Phase 1 Reactions
substrate, hence a scavenger. Two isoenzymes are there
for Super Oxide Dismutase (SOD). • Hydroxylation, catalyzed mainly by members of a
class of enzymes referred to as monooxygenases or
Cytosolic SOD is Copper dependent and Mitochondrial cytochrome P450s
m

SOD is Manganese dependentQ. • Deamination


co

Superoxide Dismutase • Dehalogenation


•O2– + •O2– + 2H+ H2O2 + 2H+ • Desulfuration
Other Nonenzymic Antioxidants • Epoxidation
• Peroxygenation
Vitamin E (α-Tocopherol)
• Reduction.
• Most potent chain breaking antioxidant
• Being lipophilic it acts on biological membranes Biotransformation to Toxic Compounds
• It terminates lipid peroxidation. In certain cases, phase 1 metabolic reactions convert xeno-
biotics from inactive to biologically active compounds.
Other antioxidants are:
In these instances, the original xenobiotics are referred
• β Carotene and Ubiquinone are lipid soluble radical- to as ‘prodrugs’ or ‘procarcinogens.’
m

trapping antioxidants in membranes and plasma


lipoproteins Example:
co

• Ascorbate, uric acid and a variety of polyphenols • Vinyl Chloride to Vinyl Chloride Epoxide which can
derived from plant foods act as water-soluble bind to DNA and RNA
radical trapping antioxidants, forming relatively • Mercury methylated, making them neurotoxic
e

e
m

m
m
co

414 |  
Self Assessment and Review of Biochemistry

• Methanol to formic acid Contd...


• Benzopyrene converted to its epoxide by epoxidation. • Basically they catalyze reactions involving introduction of one atom
of oxygen into the substrate and one into water
Cytochrome P450 • Their hydroxylated products are more water-soluble than their
The main reaction involved in phase 1 metabolism is generally lipophilic substrates, facilitating excretion
hydroxylation, catalyzed by a family of enzymes known • Liver contains highest amounts, but found in most if not all tissues,
including small intestine, brain, and lung
as monooxygenases or ‘mixed-function oxidases.’
m

• Located in the smooth endoplasmic reticulum or in mitochondria


• There are at least 57 cytochrome P450 genes in the
co

(steroidogenic hormones)
human genome • In some cases, their products are mutagenic or carcinogenic.
• Cytochrome P450 is a heme enzyme • Many have a molecular mass of about 55 kDa
• They exhibit an absorption peak at 450 nm • Many are inducible, resulting in one cause of drug interactions
• Some exhibit genetic polymorphisms, which can result in atypical
• Approximately 50% of the common drugs that drug metabolism
humans ingest are metabolized by isoforms of • Their activities may be altered in diseased tissues (e.g. cirrhosis),
cytochrome P450 affecting drug metabolism.
• They also act on steroid hormones, carcinogens, and
pollutants Phase 2 Reactions
• In addition to their role in metabolism of xenobiotics, Most abundant Phase 2 reaction is Conjugation
cytochromes P450 are important in the metabolism of
a number of physiological compounds—for example,
Glucuronidation
m

the synthesis of steroid hormones and the conversion • The glucuronidation of bilirubin by UDP-glucuronic
co

of vitamin D to its active metabolite, calcitriol acid


• NADPH is required to reduce cytochrome P450 • Glucuronidation is probably the most frequent
conjugation reaction.
• Lipids which are components of CytP450 is
Phosphatidyl Choline. Sulfation
Sites where CytP450 is present • The sulfate donor in these and other biologic
• In mammals, cytochromes P450 are present in sulfation reactions) is adenosine 3’-phosphate-5’-
highest amount in liver cells and enterocytes but are phosphosulfate (PAPS), this compound is called
‘active sulfate.’
probably present in all tissues
• For example, sulfation of steroids, glycosaminoglycans,
• In liver and most other tissues, they are present
glycolipids, and glycoproteins.
mainly in the membranes of the smooth endoplasmic
reticulum (microsomal fraction) Conjugation with Glutathione
m

• In the adrenal gland, they are found in mitochondria • Glutathione (γ-glutamylcysteinylglycine) is a tripep-
co

as well as in the endoplasmic reticulum tide consisting of glutamic acid, cysteine, and glycine
• In the adrenal gland are involved in cholesterol and • Glutathione is commonly abbreviated GSH (because
steroid hormone biosynthesis. of the sulfhydryl group of its cysteine, which is the
• The mitochondrial cytochrome P450 system differs business part of the molecule)
from the microsomal system in that it uses an • The enzymes catalyzing these reactions are called
NADPH-linked flavoprotein, adrenodoxin reductase, glutathione S-transferases and are present in high
and a nonheme iron-sulfur protein, adrenodoxin. amounts in liver cytosol and in lower amounts in
other tissues.
Properties of Human Cytochromes P450
• Involved in phase I of the metabolism of a large number of Acetylation
xenobiotics, including perhaps 50% of the clinically used drugs;
• Involved in the metabolism of many endogenous compounds • Acetyl-CoA (active acetate) is the acetyl donor. These
(e.g., steroids) reactions are catalyzed by acetyltransferases.
m

• All are hemoproteins


co

• Often exhibit broad substrate specificity, thus acting on many


Methylation
compounds; consequently, different P450s may catalyze formation • A few xenobiotics are subject to methylation by
of the same product methyltransferases, employing S-adenosylmethionine
Contd... as the methyl donor.
e

e
m

m
m
co

Free Radicals, Xenobiotics and Metabolism of Alcohol


  | 415

Conjugation with Glycine Metabolic Changes Following Ingestion of


Benzoic acid conjugated with Glycine to form Hippuric Alcohol
Acid (Benzoyl Glycine) High concentration of NADH leads to high NADH/NAD+
ratio. This is the basic cause of all metabolic alteration in
Conjugation with Glutamine alcoholism.
Phenyl Acetic Acid is conjugated with Glutamine to form • Favor conversion of Pyruvate to Lactate ( This leads
Phenyl Acetyl Glutamine.
m

to Lactic acidosis)
• Deficiecy of Pyruvate leads to deficiency of Oxaloac-
co

METABOLISM OF ALCOHOL etate. This leads to decrease in Gluconeogenesis


which causes hypoglycemia.
Topics included
• Decreased availability of NAD+ and decreased
• Metabolism of Alcohol Oxaloacetate lead to decreased TCA Cycle leads to
• Metabolic alterations following ingestion of alcohols accumulation of Acetyl CoA
Metabolism of Alcohol • Accumulation of Acetyl CoA leads to Ketogenesis
and Lipogenesis and Fatty Liver.
• NAD+ Dependent Cytoplasmic Alcohol Dehydro-
• Lactic Acidosis leads to decreased Uric Acid excretion
genase oxidises Ethanol to Acetaldehyde.
and hence gout.
• Acetaldehyde is further oxidized to Acetate by
Protective Mechanism of Microsomal Ethanol Oxidizing System
mitochondrial NAD+ dependent Aldehyde Dehy- (MEOS)
drogenase. Activity of Alcohol Dehydrogenase is
m

• Some metabolism of ethanol takes place via a cytochrome P450-


more than Aldehyde dehydrogenase. dependent microsomal ethanol oxidizing system (MEOS) involving
co

NADPH and O2.


• So, Acetaldehyde accumulates in the liver
• So NADH/NAD+ ratio is not altered hence account for metabolic
• Aldehyde is toxic and causes cell death. tolerance in chronic alcoholics.
• This system is inducible hence increases in activity in chronic
alcoholism.
• In some Asian populations and Native Americans, alcohol
consumption results in increased adverse reactions to
acetaldehyde owing to a genetic defect of mitochondrial aldehyde
dehydrogenase.
Fig. 18.4: Metabolism of alcohol

REVIEW QUESTIONS
m
co

Free Radicals 2. Most powerful chain breaking antioxidant:


1. Free radical with highest activity: a. Glutathione peroxidase
a. O2– b. Alpha tocopherol
b. OH• c. Superoxide dismutase
c. Hypochlorite d. Vitamin C
d. Peroxynitrite Ans. b. Alpha tocopherol
Ans. b. OH•
3. Enzyme which catalyze the reaction H2O2 give
Points to Remember H2O + O2:
• Most powerful oxygen free radical is Hydroxyradical a. Catalase
(OH•)Q
b. Glutathione reductase
m

• Least powerful Reactive Oxygen species is Hydrogen


c. Glutathione peroxidase
co

peroxide (H2O2).
d. Glutathione s transferase
• Precursor of all reactive Oxygen species is Superoxide
radical (O2– ) Ans. a. Catalase
e

e
m

m
m
co

416 |  
Self Assessment and Review of Biochemistry

Catalase b. Decreased lactate/Pyruvate ratio


It is a hemoprotein with four hemegroups. Causes decom- c. Inhibition of Gluconeogenesis
position of peroxides to yield water and oxygen. Highest d. Stimulation of fatty acid oxidation
concentration of Catalase is present in Peroxisomes.
Ans. a. Increased NADH/NAD+ ratio
         Catalase
2H2O2 2H2O + O2 High concentration of NADH leads to high NADH/NAD+
m

ratio. This is the basic cause of all metabolic alteration


Remember
co

H2O2 is not free radical but can generate free radical. in alcoholism.

4. Which of the following is not a free radical?


a. Hydroxyl radical 6. Best explained pathogenesis of fatty liver in
b. Hydrogen Peroxide alcoholic liver disease: (Jipmer 2013)
c. Superoxide a. Increased hydrolysis of fat from adipocytes
d. O2● b. Decreased synthesis of fatty acids
Ans. b. Hydrogen Peroxide.
c. Decreased [NADH]/[NAD+] ratio
It is a reactive oxygen species, not a free radical.
d. Impaired beta oxidation of fatty acids
Alcohol Metabolism Ans. d. Impaired beta oxidation of fatty acids
5. Toxicity of Ethanol is due to: (Jipmer 2012)
m

Impaired beta oxidation leads to increased TAG in the


a. Increased NADH/NAD+ ratio
co

liver, which leads to fatty liver.


m
co
m
co
6
Section

Image-Based Questions
IMAGE-BASED QUESTIONS

1. Identify the amino acid given in the diagram. c. Nonpolar imino acid
d. Aromatic essential amino acid

5. In the reaction, what is product formed?

a. Cysteine b. Cystine
a. Glutamate
c. Methionine d. Homocysteine
b. Glutamine
2. The best description of the given amino acid is:
c. Oxaloacetate
d. Aspartate

6. What is the vitamin that act as the coenzyme of


the given reaction?
a. Simple, Polar, Nonessential
b. Branched chain, Polar, Essential
c. Branched chain, nonpolar, Nonessential
d. Optically active
3. The amino acid given in the picture is hydroxylated
to form which amino acid?

a. Vitamin B1
b. Vitamin B2
c. Vitamin B5
d. Vitamin B6

7. Identify the separatory technique of protein


given in the picture?

a. Serine b. Threonine
c. Tyrosine d. Tryptophan
4. The amino acid given in the diagram can be is
best suited to which of the following description?

a. Size Exclusion Chromatography


a. Answer Ninhydrin test with purple coloured  
   

complex c. Ion Exchange Chromatography


b. Alpha amino acid with polar side chain d. High Pressure Liquid Chromatography
8. All the following description about the given 10. All the following are true about the given
technique is true except: structure except:

a. Help to sequence the polypeptide


b. Gold standard screening method of metabolic
disorders
        
mass to charge ratio
d. Can also be used to study the secondary and
tertiary structures of Proteins.
9. What is the name of the complex that pumps
2 protons to the intermembrane space of
mitochondria in the given Electron Transport
Chain? a. Intrastrand base pairing is seen
a. NADH CoQ Oxidoreductase b. This structure can be translated to a protein
b. Succinate CoQ Oxidoreductase c. 3’ end has CCA base sequence
c. Cytochrome C Oxidase d. Pseudouridine is an unusual amino acid
d. Q Cyt C Oxidoreductase present in this.
11. The given molecular biology technique can detect which of the following?

a. DNA
b. RNA
c. Protein
d. DNA Protein Interaction
12. All the following about the given techniques are
true except:
a. Invented by Dr Karry B Mullis
b. Can amplify DNA and RNA
c. This is an in vivo technique
   

13. At which subunit ATP is formed in the given


complex:

a. Fo Subcomplex
  !"
c. C disc of Fo subcomplex
 # !"
14. Identify the enzyme marked X:

a. Citrate Isomerase
b. Aconitase
c. Citrate Cis-Trans isomerase
d. Isocitrate Synthase

15. Which of the following is the amino acid marked


X, that enter at the level of Fumarate?
a. Alanine
b. Tryptophan
c. Tyrosine
d. Histidine
16. Identify the intermediate in the given pathway
marked C?

a. 1 Phosphoglycerate
b. 2, 3 Phosphoglycerate
c. 1, 3 Diphosphoglycerate
d. 3 Phosphoglycerate

17. All the following about this pathway are true


except:
a. Takes place in RBC
b. This is the major fate of glucose in RBC
c. No net ATP is produced by this pathway
d. Substrate level phosphorylation step is
bypassed
18. The enzyme catalyzing this pathway is absent in:  $   %
 
  
beings
 
 %
$ &
binemia and Hemolytic anemia
22. Identify the enzyme that catalyse this reaction,
a. Liver b. Muscle that is absent in humans and higher primates?
c. GIT d. Kidney
19. What is the coenzyme of this reaction?

a. L-Gulanolactone Oxygenase
b. L-Gulanolactone Oxidase
c. L-Gulanolactone Hydroxylase
a. Vitamin B12 b. Biotin d. L-Gulanolactone Dehydrogenase
c. Thiamine d. Vitamin B6 23. Which is true regarding the disorder caused by
20. What is the indication for doing this test in  
  
 
laboratory?

a. Cataract is the only clinical feature


b. Presents after 1 year of age
c. Benedict’s test and Glucose oxidase test positive
d. They are at increased risk of neonatal sepsis by
E coli.
24. Identify the given diagram

a. Overt Diabetes mellitus


b. Diabetic nephropathy
c. Symptoms of Diabetes mellitus, but Blood
glucose values are inconclusive
d. Follow-up of Diabetes mellitus
21. Which of the following is NOT true regarding
this enzyme, marked E?

a. Rate limiting step of HMP Pathway a. Liposome b. Aquasome


b. Nonoxidative phase of HMP pathway c. Micelle d. Lipid Bilayer
25. In which of the following pathway the given b. Carnitine Palmitoyl Transferase II and Carni-
multienzyme complex takes part? tine Palmitoyl Transferase I
c. Carnitine Acyl Carnitine Translocase I and
Carnitine Palmitoyl Transferase I
d. Carnitine Palmitoyl Transferase II and
Carnitine Acyl Carnitine Translocase II
28. Identify the product X formed by the reaction
 



   '



 ('

 
 )  '

 * '

26. The enzyme catalyzing this reaction belongs to
which class of enzyme?

a. Beta Hydroxy Butyrate


a. Monooxygenase b. Diooxygenase b. Acetyl CoA
c. Oxidase d. Dehydrogenase c. Acetate
  
    
 
    d. Acetone
respectively:
29. All are true about reaction catalysed by enzyme -1
except:

a. Carnitine Palmitoyl Transferase I and Carnitine


Palmitoyl Transferase II
a. Rate limiting enzyme of the pathway 33. The vitamin that acts as a coenzyme of the given
b. This enzyme defect is the most common cause reaction is
of Hyperammonemia Serine + Homocysteine o Cystathionine + H2O
c. Biotin independent Carboxylation a. Vitamin B2 b. Vitamin B3
d. Takes place in the mitochondria. c. Vitamin B5 d. Vitamin B6
30. This compound is excreted in urine in which 34. The rate limiting enzyme in the synthesis of this
 
 
  compound is

a. Spermine Synthase
b. SAM Decarboxylase
 !
c. Ornithine Decarboxylase
d. Ornithine transcarbamoylase
 

 + 
35. The functions of the given compound is:
 ) 

31. The false statement regarding the given


compound is:

a. Biological rhythm b. Vasoconstriction


c. Mood Depression d. Mood elevation
36. This compound is derived from which amino
acid?

a. Excreted in urine in Alkaptonuria


b. Gives Benedict’s test positive
c. Polymerises to benzoquinone acetate
d. Strong oxidizing agent
    

 

 
 a. Tyrosine b. Histidine
the metabolism of which amino acid? c. Tryptophan d. Leucine
37. The compound marked X, that contribute to the
Nitrogen in the given ring structure is

a. Tyrosine b. Histidine
c. Tryptophan d. Phenylalanine
a. Asparagine b. Aspartate a. Have beads on string appearance
c. Glutamate d. Serine b. DNA complexed with histone octamer
38. All the following statement about the given c. DNA is wound in right handed direction
diagrams are true except? d. Has 146 bp in the DNA helix that wound on
the histone octamer
!"  #$
  ## %
 
binds to which regions of DNA?

a. Enhancers
b. Promotors
c. Introns
d. Exons
41. The cracking of the code in the diagram is done
by:

First Second Nucleotide Third


nucleotide U C A G nucleotide

U Phe Ser Tyr Cys U


a. Half of the parent strand is conserved in the Phe Ser Tyr Cys C
daughter DNA Leu Ser Term Term b
A
b. Proved by Meselson and Stahl
Leu Ser Term Trp G
c. The genetic information is passed from parent
C Leu Pro His Arg U
to progeny
Leu Pro His Arg C
d. This takes place in the M phase of Cell cycle
Leu Pro Gln Arg A
39. Which of the following is not true about the
Leu Pro Gln Arg G
complex ?
A IIe Thr Asn Ser U
IIe Thr Asn Ser C
IIea
Thr Lys Arg b
A
Met Thr Lys Argb G
G Val Ala Asp Gly U
Val Ala Asp Gly C
Val Ala Glu Gly A
Val Ala Glu Gly G

a. Frederick Sanger
b. Karry B Mullis
c. Robert Holley
d. Marshall Nirenberg
42. The components of the given initiation complex c. Super secondary structure
of translation include all except: d. Tertiary structure
45. The transport mechanism depicted in the diagram
is:

a. GTP
b. mRNA
c. 40S Subunit
d. 60S Subunit
! &  

  
 
##

the cell?

 7
 !  
c. Ligand gated transport
d. Ion channels

46. What is the true statement about the regulatory


mechanism of gene expression given in the
picture?

 /'    


b. mRNA degradation
c. Protein folding
d. Protein degradation
44. The given picture is an example of which
organizational level of Protein?

 /'     


  89 

a. Primary structure c. Alter the nucleotide sequence of the DNA


b. Secondary Structure d. Alter organization of the DNA
! &  
 
 '
  

  c. X-linked Protoporphyria
diagram? d. Porphyria Cutanea tarda

50. A 5-year-old boy presented with swelling and


redness in the dorsum of hands. A porphyria
is diagnosed in this child. What is the enzyme
 
   
 #     
 #
#
picture?

a. Autosomal Dominant
b. Autosomal Recessive
c. X-linked Dominant
d. X-linked Recessive
!( &  
 
 '
  

 
diagram?

a. ALA Dehydratase
b. PBG Deaminase
a. Autosomal Dominant c. Uroporphyrinogen Decarboxylase
b. Autosomal Recessive d. Ferrochelatase
c. Mitochondrial )* 
    
  
   

 
d. X-linked Recessive picture:
49. Which of the following porphyria causes the
 
#
#



a. Pellagra
b. Scurvy
  :  ' / 
 c. Beriberi
b. Variegate Porphyria d. Burning foot syndrome
)   #
#


   a. Tay-Sachs disease
seen: b. Niemann Pick disease
c. Gaucher’s disease
 7 ;
55. Which of the following regarding the cytogenetic
technique is false:
a. Can diagnose Cri du chat disease
b. Can diagnose Philadelphia chromosome
c. Can be used to detect molecular defects in
cancer
d. Unknown chromosomal anomaly can be
detected
a. Familial Hypercholesterolemia
b. Familial Chylomicronemia
c. Abetalipoproteinemia
d. Broad Beta Disease
)   #
#


  
seen:

56. What is the chromosomal anomaly not detected


by the technique given in the picture?

a. Familial Hypercholesterolemia
b. Familial Chylomicronemia
c. Abetalipoproteinemia
d. Sitosterolemia
54. Which of the following metabolic disorder cannot

+  
#
#

 a. Copy number variations
b. Microdeletion
c. Translocations with no loss of genetic element
  
) & 
 
'

a. Autosomal Dominant a. Scurvy
b. X linked Dominant b. Beriberi
c. Y Linked  8<(

d. Mitochondrial d. Pellagra
58. What is the diagnosis of the given picture? 60. A 12-year-old male presented with multiple
skeletal defects, dislocated lens, and a
characteristic gait. What is the most probable
enzyme defect?

a. Pellagra
b. Beri beri
c. Rickets
d. Scurvy
59. Identify the clinical case kept for Final year MBBS
OSCE examination in Pediatrics department:

a. Cystathionine Beta Synthase


b. Cystathionase
c. Methionine Synthase
d. Methylene THF Reductase
61. A 40-year-old male, presented to orthopedic OP
with back pain. On examination the given clinical



& 
  
 
that lead to the disorder?
62. This two-year-old girl presented with protuberant
abdomen, coarse facial features, umbilical hernia,
corneal clouding, mental retardation and short
stature. What is the most probable diagnosis?

a. Tyrosine Hydroxylase
b. Homogentisate Oxidase
c. Tyrosine Transaminase
d. Tyrosinase

a. Hurler’s disease
b. Hunter’s disease
c. Moroteaux-Lamy disease
d. Scheie’s disease
EXPLANATORY ANSWERS

1. Ans. c. Methionine  /

 
     
   
  
= 7ulphur containing amino acids are Cysteine  
!
and methionine 8. Ans. d.            

  
          


 
$
  !
   
    Mass Spectrometry
      
    
   *    7      
            
!
   
    !     #

    
  
 
     
   
2. Ans. a. Simple, Polar, Nonessential


!
"  
 3   
  
 #     

 

 
 $ 
  9. Ans. c.  
&(
 %       ( - .%'* 8 &(
 0
 &     !    ( --- .8    &( 9 0
3. Ans. c. 
  :
  !
      
$     (--.#  8&(9 0
'   !  
  !
 & 
(   
 
 !   ( -1 . 
  &(0 
  )
 
  $      ;
  
*
( ! 10. Ans. b. 
 
 
    
 
   )           9%'        9%'   <: =>?
( !      !
       )  $        $
  9 =
,  
!   
 !
4. Ans. c. %  
   @A       
 
   ' 
 $
   7 !
 -   -

 
 
 %  
11. Ans. a. %'
 "  
 % 
    # 
3       7!-  
  %'!
 %    
12. Ans. c.  LQYLYR  7
5. Ans. a."   
PCR
 
    
    '   
 Is an in vitro     7  
 
 
.'/0
  
,

3 
 '    
 $
 !
 '  
  

       !


 ' ,   
  
 "   !
 5(     
6. Ans. d. Vitamin B6
 9

 
 $9  9%'
  
  
    ! 13. Ans. d. E  DE (
 $/$.1  340  )   ATP Synthase
reaction
 & 
    ( 1  5   
7. Ans. a. #)5(  
 

 

 #) 5(   
 
 
    -   (D DE
     )!  '$ )  E  DE!
   (         25. Ans. b. %D  
  

$ 3
 D #  (  )
14. Ans. b. Aconitase  ( 4 )  
 '    
 
   -
     *
 J!
 
! 26. Ans. a.  ( 
 
  '   !  $      *
(     (=
 Cis aconitate to Isocitrate  
  )  D 
  ! 27. Ans. a. 
   $    
 
 -  

  $   
 
--
15. Ans. c. 
 
 $=E  
  
 
  
 '    
    D
 
$=; - 
  
 
 


  $  '   
28. Ans. d. Acetone
16. Ans. d. 3 $ 
 
 ,        
 

17. Ans. b.  
   93    '  !
9
/
   29. Ans. b.   )        
     93 *
 
 #
    
      E @  $#=-  )
3 
 , !   
             
 %  '$   
  !   
!
    
 "  93      (   3   -   
 
   93 
 
(   !
  !        *
  
18. Ans. b.   &
  
 
  !
  )" 4$   30. Ans. a. D  
   ! 31. Ans. d. #
 () 
19. Ans. b. Biotin  * 
 
  
  3    )

(   
   3  A   
 $
  '
(  32. Ans. c.
 
 , 
    
  
  
 '  '
( 
  
 $
 
( 
33. Ans. d. Vitamin B6
20. Ans. c. #     3 
Cytathionine Beta Synthase
  
   G
 1  34  )!
  
 
    &
  "   
 
   )     
 !
  

 -            34. Ans. c. &
  
( 
  
 #
  #
    $
  

21. Ans. b. % ( *$   $  !
" 4$ 
   


&
   / !
 9     *$$   &
   
(   
    
 - 


 (   !  
22. Ans. b./="     &( 35. Ans. a. 3  
 
 "      &(    )     D       
3  
 

     !!        
 1
  !
 

 !  # ) $    

  !
 #1     )  ! 36. Ans. a. 
 
23. Ans. d.  
    

          
 
 
  

E. coli!       .5 
 %
=
24. Ans. c.    
  0 
( !
37. Ans. b. '
  43. Ans. b. 9%'
 
    
  $
 
  $3
 %
    9%'

  
  

            

   $!
 9  
   
 9%$ 
 9%'
   $
  !
 $
 
    

  
9%'!
 $  9%'  )
9%'   9%' (     

9%'7  
 !
 '
 9%'
 9%'   
   $
38. Ans. d.             
44. Ans. c. #
 

 

 
 %'   R  
   
   
    # 
 


 %'
   !
 -    #
 

 

 $
   # 
45. Ans. b. D   
      #  !
 D        


  
        
  

 



  
 !
46. Ans. d. ' 

 )   %'
39. Ans. c. %'  
  
 

  
      
 
% 
    !
 %'  (  *   
!
    
    7 !
 
EK 
  
 
        %' 

 
 
 !

    
   (
 !
 %'          
    
 '     
 
  
   
!

  !
 E!<? 
  %'  (       

 -     

  
  !

 E:?=E?K
 #   
 )   %'!
40. Ans. b. Promoters
47. Ans. d. J=  

9%'$ 
 (
The characteristics of X-linked recessive inheri-
 9%' $ 
  (    
 
 

 
  ! tance.
x V  
          9%'   
  !
$ 
 ( 
 
!  D 
 


!
41. Ans. d. 
 %
 
  '    


 
"    


    
 
     
      
   

%
 
 *
" ,
 ! 48. Ans. c.   

42. Ans. 4K##  The Characteristics of Mitochondrial inheritance

 :L#-     (  D 
        

 "$N-D=;N  9%'Q

 ( 
 !
 

 (N:K#9Q:@#$
     

   !
     (   
  
 
  !
 :@ # $
       ( N 9%' Q :L #  & 
    
  !
-     ( 49. Ans. d. $

   

Porphria Cutanea Tarda 52. Ans. a. D  
  
 
  T


  
(       
     (   
  LKU  
 
   T


     --*

  

(   
! Clinical presentation of Familial Hypercholeste-
  
   
$
G rolemia
E! *  *-1  D  

 
*
;! 5('   
 

@! 5  -
  $  

:! 5
  % 
  
   $

     (  
 
 
 
  
 
 $

   '    !
 '  *
   - 

 
 
!
 3  
  #   /         53. Ans. b. D   
 
 !
      
  
  (   
 
    
  

  - 1*

  
   
  
  
 *   


 ! Clinical presentation of Familial Chylomicro-
nemia Syndrome

 
 9 $   
 
 !  $
    


   
    
  
 /
  
7 
*
(
 
7 !  &    (      
 -          
   
     . 
   0
  
5
 
  !  /   
50. Ans. d. D

    5
  (  .     
Erythropoietic Protoporphyria  
   
    
    D
  .D5*   0 (  


  !

     

    
         

 0
     !  *    
 %&%3/-#59-%"$      $
 
*   
D#!
 
 
   
    54. Ans. c. "
A
       (

        
  


     
 !  
 1 
  
  !  #      


    
51. Ans. b. #
  ="
A-D
A
#
 55. Ans. d. T   
       
 $     
      
     
     
D-#*
   !
Uses of FISH
 $
  



      
  
   
 $
  
 

      = 
.  0



  
   
   
 #  
 

     


=


  

        (


      
   
   
  
 Anaemia
 /  

 
 =  D
     

 
 


 

   D
     8 
 
   

  !
Disadvantages of FISH          
 @ 

 D-#*
7

     
       (      
  
  



   ! .   0 
!   

 #

      !    
  .7
   
56. Ans. c. 
             
0!
   $

intellectual disability is common
Array CGH  Skeletal abnormalities
   
   " '    Marfan syndrome        
     
    
   "    !
 ( 

    
    

  
  =
      
 
3  '

 "*               


  !

           
  
  
 ( 
57. Ans. c. X=      
 
 
The characteristics of Y-linked inheritance  Thromboembolic episodes      
 &  
  
      
 &    
   
 
  
  
 (      X
 61. Ans. b. *  &(


     !   '   



58. Ans. c. 9   Clinical Presentation of Alkaptonuria
 9 

  9          %
 /  @

: !
 
!  T
  
           
59. Ans. c. 9          
 !
Deficiency manifestation of Vitamin B2 (Riboflavin)  -    Ochronosis !! '    3
    ."  0   
         - 


   
  
Seborrheic Dermatitis Corneal vascul-         !
arization   Arthritis
60. Ans. a. Cystathionine Beta Synthase  %&5%'/95'9'-&%8
Clinical Features of Classic Homocystinuria 62. Ans. a. Hurler’s disease
 %
  
  &  3= 

  

 #  
     
     
      * 
A!
     
  
    =  &   Y
   
 
  
   !  
 (=/#A!

You might also like